Wiley Problems in MATHEMATICS For JEE, Volume-2
Wiley Problems in MATHEMATICS For JEE, Volume-2
352%/(06
0$7+(0$7,&6)25
,1
-(( ǁŝƚŚ^ƵŵŵĂƌŝnjĞĚŽŶĐĞƉƚƐ
sK>hDʹ//
8*-&:
352%/(06
,1
0$7+(0$7,&6)25
sK>hDʹ//
-(( ǁŝƚŚ^ƵŵŵĂƌŝnjĞĚŽŶĐĞƉƚƐ
ŽƉLJƌŝŐŚƚΞϮϬϭϴďLJtŝůĞLJ/ŶĚŝĂWǀƚ͘>ƚĚ͕͘ϰϰϯϲͬϳ͕ŶƐĂƌŝZŽĂĚ͕ĂƌLJĂŐĂŶũ͕EĞǁĞůŚŝͲϭϭϬϬϬϮ͘
ŽǀĞƌ/ŵĂŐĞ͗ĂƌůŽƐͺďĐŶͬŝ^ƚŽĐŬƉŚŽƚŽ
ůůƌŝŐŚƚƐƌĞƐĞƌǀĞĚ͘EŽƉĂƌƚŽĨƚŚŝƐŬŵĂLJďĞƌĞƉƌŽĚƵĐĞĚ͕ƐƚŽƌĞĚŝŶĂƌĞƚƌŝĞǀĂůƐLJƐƚĞŵ͕ŽƌƚƌĂŶƐŵŝƚƚĞĚŝŶĂŶLJĨŽƌŵ
ŽƌďLJĂŶLJŵĞĂŶƐ͕ĞůĞĐƚƌŽŶŝĐ͕ŵĞĐŚĂŶŝĐĂů͕ƉŚŽƚŽĐŽƉLJŝŶŐ͕ƌĞĐŽƌĚŝŶŐŽƌƐĐĂŶŶŝŶŐǁŝƚŚŽƵƚƚŚĞǁƌŝƚƚĞŶƉĞƌŵŝƐƐŝŽŶŽĨ
ƚŚĞƉƵďůŝƐŚĞƌ͘
>ŝŵŝƚƐŽĨ>ŝĂďŝůŝƚLJ͗tŚŝůĞƚŚĞƉƵďůŝƐŚĞƌĂŶĚƚŚĞĂƵƚŚŽƌŚĂǀĞƵƐĞĚƚŚĞŝƌďĞƐƚĞĨĨŽƌƚƐŝŶƉƌĞƉĂƌŝŶŐƚŚŝƐŬ͕tŝůĞLJ
ĂŶĚ ƚŚĞ ĂƵƚŚŽƌ ŵĂŬĞ ŶŽ ƌĞƉƌĞƐĞŶƚĂƚŝŽŶ Žƌ ǁĂƌƌĂŶƚŝĞƐ ǁŝƚŚ ƌĞƐƉĞĐƚ ƚŽ ƚŚĞ ĂĐĐƵƌĂĐLJ Žƌ ĐŽŵƉůĞƚĞŶĞƐƐ ŽĨ ƚŚĞ
ĐŽŶƚĞŶƚƐ ŽĨ ƚŚŝƐ Ŭ͕ ĂŶĚ ƐƉĞĐŝĨŝĐĂůůLJ ĚŝƐĐůĂŝŵ ĂŶLJ ŝŵƉůŝĞĚ ǁĂƌƌĂŶƚŝĞƐ ŽĨ ŵĞƌĐŚĂŶƚĂďŝůŝƚLJ Žƌ ĨŝƚŶĞƐƐ ĨŽƌ ĂŶLJ
ƉĂƌƚŝĐƵůĂƌƉƵƌƉŽƐĞ͘dŚĞƌĞĂƌĞŶŽǁĂƌƌĂŶƚŝĞƐǁŚŝĐŚĞdžƚĞŶĚďĞLJŽŶĚƚŚĞĚĞƐĐƌŝƉƚŝŽŶƐĐŽŶƚĂŝŶĞĚŝŶƚŚŝƐƉĂƌĂŐƌĂƉŚ͘
EŽǁĂƌƌĂŶƚLJŵĂLJďĞĐƌĞĂƚĞĚŽƌĞdžƚĞŶĚĞĚďLJƐĂůĞƐƌĞƉƌĞƐĞŶƚĂƚŝǀĞƐŽƌǁƌŝƚƚĞŶƐĂůĞƐŵĂƚĞƌŝĂůƐ͘
ŝƐĐůĂŝŵĞƌ͗dŚĞĐŽŶƚĞŶƚƐŽĨƚŚŝƐŬŚĂǀĞďĞĞŶĐŚĞĐŬĞĚĨŽƌĂĐĐƵƌĂĐLJ͘^ŝŶĐĞĚĞǀŝĂƚŝŽŶƐĐĂŶŶŽƚďĞƉƌĞĐůƵĚĞĚ
ĞŶƚŝƌĞůLJ͕tŝůĞLJŽƌŝƚƐĂƵƚŚŽƌĐĂŶŶŽƚŐƵĂƌĂŶƚĞĞĨƵůůĂŐƌĞĞŵĞŶƚ͘ƐƚŚĞŬŝƐŝŶƚĞŶĚĞĚĨŽƌĞĚƵĐĂƚŝŽŶĂůƉƵƌƉŽƐĞ͕
tŝůĞLJŽƌŝƚƐĂƵƚŚŽƌƐŚĂůůŶŽƚďĞƌĞƐƉŽŶƐŝďůĞĨŽƌĂŶLJĞƌƌŽƌƐ͕ŽŵŝƐƐŝŽŶƐŽƌĚĂŵĂŐĞƐĂƌŝƐŝŶŐŽƵƚŽĨƚŚĞƵƐĞŽĨƚŚĞ
ŝŶĨŽƌŵĂƚŝŽŶĐŽŶƚĂŝŶĞĚŝŶƚŚĞŬ͘dŚŝƐƉƵďůŝĐĂƚŝŽŶŝƐĚĞƐŝŐŶĞĚƚŽƉƌŽǀŝĚĞĂĐĐƵƌĂƚĞĂŶĚĂƵƚŚŽƌŝƚĂƚŝǀĞŝŶĨŽƌŵĂƚŝŽŶ
ǁŝƚŚƌĞŐĂƌĚƚŽƚŚĞƐƵďũĞĐƚŵĂƚƚĞƌĐŽǀĞƌĞĚ͘/ƚŝƐƐŽůĚŽŶƚŚĞƵŶĚĞƌƐƚĂŶĚŝŶŐƚŚĂƚƚŚĞWƵďůŝƐŚĞƌŝƐŶŽƚĞŶŐĂŐĞĚŝŶ
ƌĞŶĚĞƌŝŶŐƉƌŽĨĞƐƐŝŽŶĂůƐĞƌǀŝĐĞƐ͘
KƚŚĞƌtŝůĞLJĚŝƚŽƌŝĂůKĨĨŝĐĞƐ͗
:ŽŚŶtŝůĞLJΘ^ŽŶƐ͕/ŶĐ͘ϭϭϭZŝǀĞƌ^ƚƌĞĞƚ͕,ŽďŽŬĞŶ͕E:ϬϳϬϯϬ͕h^
tŝůĞLJͲs,sĞƌůĂŐ'ŵď,͕WĂƉƉĞůůĂĞĞϯ͕ͲϲϵϰϲϵtĞŝŶŚĞŝŵ͕'ĞƌŵĂŶLJ
:ŽŚŶtŝůĞLJΘ^ŽŶƐƵƐƚƌĂůŝĂ>ƚĚ͕ϰϮDĐŽƵŐĂůů^ƚƌĞĞƚ͕DŝůƚŽŶ͕YƵĞĞŶƐůĂŶĚϰϬϲϰ͕ƵƐƚƌĂůŝĂ
:ŽŚŶtŝůĞLJΘ^ŽŶƐ;ƐŝĂͿWƚĞ>ƚĚ͕ϭ&ƵƐŝŽŶƉŽůŝƐtĂůŬηϬϳͲϬϭ^ŽůĂƌŝƐ͕^ŽƵƚŚdŽǁĞƌ^ŝŶŐĂƉŽƌĞϭϯϴϲϮϴ
:ŽŚŶtŝůĞLJΘ^ŽŶƐĂŶĂĚĂ>ƚĚ͕ϮϮtŽƌĐĞƐƚĞƌZŽĂĚ͕ƚŽďŝĐŽŬĞ͕KŶƚĂƌŝŽ͕ĂŶĂĚĂ͕Dϵtϭ>ϭ
&ŝƌƐƚĚŝƚŝŽŶ͗ϮϬϭϴ
/^E͗ϵϳϴͲϴϭͲϮϲϱͲϳϲϯϬͲϮ
/^E͗ϵϳϴͲϴϭͲϮϲϱͲϴϲϵϬͲϱ;ĞďŬͿ
ǁǁǁ͘ǁŝůĞLJŝŶĚŝĂ͘ĐŽŵ
WƌŝŶƚĞĚĂƚ͗
Note to the Student
Wiley Mathematics Problem Book is specifically designed to meet the needs of engineering (JEE) aspirants and give an edge to
their preparation. The book offers complete coverage of the mathematics curriculum (of Class 12 syllabus) for JEE. It is enriched with
unique elements and features that help students recapitulate the concepts, build problem-solving skills and apply them to solve all
question-types asked in the engineering entrance examinations. The book is a valuable resource for both JEE (Main) and JEE (Advanced)
aspirants. The chapter flow of the book is aligned with JEE Main syllabus and its coverage in the classroom. However, topics specific to
JEE (Advanced) and advanced level questions are also covered both as solved examples and practice exercises.
We will now walk you through the target examinations and some key features of the book that enhance the learning experience.
TARGET EXAMINATION
Admission to Undergraduate Engineering Programs at IITs, NITs and other Center and State (participating) funded Technical
Institutions use the Joint Entrance Examination Main (JEE Main) score as eligibility/merit criteria. The JEE (Main) is also an eligibility test
for the Joint Entrance Examination Advanced [JEE (Advanced)], which is mandatory for the candidate if he/she is aspiring for admission
to the undergraduate program offered by the IITs. The JEE (Advanced) scores are used as an eligibility criteria for admission into IITs.
An effective exam strategy for success in these examinations can be based on the detailed analysis of previous years question papers
and planning your preparation accordingly. The Mathematics Question Paper of these examinations is a judicious mix of easy, moderate
and tough questions. The analysis of question distribution over the units of mathematics syllabus for these examinations is given below.
JEE Main
Year
Unit
2010 2011 2012 2013 2014 2015 2016 2017 2018
Algebra 14 13 13 12 12 11 12 13 12
Calculus 8 10 9 8 9 8 7 10 8
Trigonometry 2 1 1 3 2 3 3 2 3
Analytical Geometry 6 6 7 7 7 8 8 5 7
JEE Advanced
Year
Unit
2010 2011 2012 2013 2014 2015 2016 2017 2018
Algebra 16 17 12 14 12 6 12 10 8
Trigonometry 5 1 2 4 3 1 2 1 1
Analytical Geometry 13 8 9 10 7 3 9 7 9
Differential Calculus 2 7 6 2 11 5 7 8 12
Integral Calculus 8 7 10 7 5 4 5 7 4
Vector 3 3 2 3 2 1 1 3 2
MATHEMATICS JEE MAIN PAPERS ANALYSIS (2010-2018)
AIEEE AIEEE AIEEE JEE Main JEE Main JEE Main JEE Main JEE Main JEE Main
Unit Chapter
2010 2011 2012 2013 2014 (Offline) 2015 (Offline) 2016 (Offline) 2017 (Offline) 2018 (Offline)
Algebra Complex Numbers and Quadratic Equations 2 2 2 3 3 2 2 1 1
Permutations and Combinations 1 1 1 1 2 1 2 1
Binomial Theorem 1 1 1 1 1 1
Sequences and Series 2 1 2 1 2 2 2 3 2
Statistics 1 1 1 1 1 1 1 1 1
Mathematical Reasoning 1 1 1 1 1 1 1
Matrices and Determinants 3 2 2 2 2 2 2 3 3
Vector Algebra 2 2 2 1 1 1 1 1 1
Probability 2 2 1 1 1 1 1 2 1
Calculus Sets, Relations and Functions 1 2 1 1 1 2 1 2 2
Limits, Continuity and Differentiability 3 3 2 2 3 3 2 2 1
Application of Derivatives 1 1 3 1 2 2 1
Integrals 1 2 2 3 2 2 1 2 2
Application of Integrals 1 1 1 1 1 1 1 1
Differential Equations 1 1 1 2 1 1
Trigonometry Trignometric Functions 2 1 1 2 2 2 2 2 3
Inverse Trignometric Functions 1 1 1
Analytical Conic Sections 2 2 3 3 3 4 5 3 5
Geometry Three-Dimensional Geometry 4 4 4 4 4 4 3 2 2
MATHEMATICS JEE ADVANCED PAPERS ANALYSIS (2010-2018)
IIT-JEE 2010 IIT-JEE 2011 IIT-JEE 2012 JEE Advanced 2013 JEE Advanced 2014 JEE Advanced 2015 JEE Advanced 2016 JEE Advanced 2017 JEE Advanced 2018
Unit Chapter
P Q R S T U P Q R S T U P Q R S T U P Q R S T U P Q R S T U P Q R S T U P Q R S T U P Q R S T U P Q R S T U
Complex Numbers 1 1 1 2 1 1 1 2 1 1 1 1 2
Quadratic Equations 1 2 1 1 1 1 1 2 1
Permutations and
1 1 1 1
Combinations
Sequence and Series 1 2 1 1 2 1 1 1 1 2 1 1
Binomial Theorem 1 1 1 1 1 1
Algebra
Logarithms 1 1 1
Matrices and
1 3 2 2 3 1 2 1 2 2 1 1 1 1 1 1 1 1
Determinants
Probability 2 1 2 1 2 1 2 2 2 2 1 2 1 2 1 1 2
Properties and
Solution of Triangles
1 1 1 1 1 1
(Heights and
Distances)
Trigonometric
2 1 1 1
Equations
Trigonometric Ratios
Trigonometry
1 1 1 1
and Identities
Inverse Trigonometric
Function (Principal 1 1 1 1 1
Values Only)
Rectangular
Coordinate System
Straight Lines and
2 1 1 1 1 1 1 2 1
Pair of Lines
Conics 1 3 2 2 2 1 1 1 1 2 1 1 2 1 2 1 2 2 2 3 1 1
Circle 2 1 2 1 2 1 1 4 1 2 1
Analytical Geometry
Three-Dimensional
1 2 1 1 1 1 2 2
Geometry
Sets and Relations 2 1 1 1 1 2
Limits 1 1 2 1 2 1 1 3 1 1
Functions 1 1 1 2 2 1 2 1 1 1 1 1
Continuity and
1 2 1 1 1 2 1 1 1 1 4 1 1 2 1
Differentiability
Differential Calculus
Differentiation 2
IIT-JEE 2010 IIT-JEE 2011 IIT-JEE 2012 JEE Advanced 2013 JEE Advanced 2014 JEE Advanced 2015 JEE Advanced 2016 JEE Advanced 2017 JEE Advanced 2018
Unit Chapter
P Q R S T U P Q R S T U P Q R S T U P Q R S T U P Q R S T U P Q R S T U P Q R S T U P Q R S T U P Q R S T U
Application of
1 1 1 1 2 1 2 2 1 1 1 3 1
Derivatives
Indefinite Integration 1
Definite Integration 1 1 2 1 1 2 1 1 1 1 1 1 1 1 1
Area Under the
3 1 1 1 1 1 1 1
Integral Calculus
Curve
Differential Equations 1 1 1 1 1 1 1 1 1
Vector Vectors 2 1 1 1 1 1 1 1 1 1 1 1 1 1 1 2 2
P: Single Correct Choice Type Q: One or More Than One Option Correct Type R: Paragraph Type
Key Point:
When the curve is given in parametric form, that is, when x =
g(t) and y = h(t), the equation of tangent at the point t = t1 is
h′(t1) 2. Important points to remember about concepts
y − h(t1) = [ x − g (t1)]
g′(t1)
highlighted as Key Points.
and the equation of normal is
g¢(t1)
y - h (t1) = - [ x - g (t1)]
h¢(t1)
⎡ p p⎤
(i) sin−1(sin θ) = θ, ∀q ∈ ⎢ − , ⎥
⎣ 2 2⎦
(ii) cos−1(cos θ) = θ, ∀ θ ∈ [0, π]
⎛ p p⎞
(iii) tan−1(tan θ) = θ, ∀q ∈⎜ − , ⎟
⎝ 2 2⎠
C
A (iv) cot−1(cos θ) = θ, ∀ θ ∈ (0, π)
B Table 17.1 Domain and principal ranges of all the six inverse trig-
⎧p ⎫ onometric functions
(v) sec−1(sec θ) = θ, ∀q ∈[0 , p ] − ⎨ ⎬
⎩2⎭ Function Domain Principal Range
−1 ⎡ p p⎤ (values of x) (values of y)
(vi) cosec (cosec θ) = θ, ∀q ∈ − ,
N r ⎢ 2 2 ⎥ − {0}
L M y ⎣ ⎦ é p pù
y = sin−1 x [−1, 1] ê- 2 , 2 ú
ë û
æ p pö
y = tan−1 x (−∞, ∞) ç- , ÷
è 2 2ø
P (x1, y1)
O
Figure 21.4 ψ
ψ
x
O T M N
Figure 21.2
C. Reinforce Concepts
Illustration 21.1 Find the slope of tangent at the point that has
1. Illustrations pose a specific problem using the ordinate −3 on the curve x3 = 3y2.
dy x 2
⇒ =
dx 2 y
Now, to obtain this value, we require abscissa as well. Substituting
y = −3 in the equation of curve, we have
Your Turn 1
1. Find the slopes of the curve y = (x + 2)(x − 3) at the points
where it meets x−axis. Ans. −5, 5 2. Your Turn within each chapter is present to
2. Find the points on the curve y = x3 − 2x2 + x − 2 when the gra-
dient is zero.
reinforce and check the understanding of the
⎛ 1 50 ⎞ students.
Ans. (1, −2) and ⎜ , − ⎟
⎝ 3 27 ⎠
3. Find the equation of tangent and normal to the curve
x3 = y2
normal, subtangent and subnormal.
13 13 2 3
Ans. 3x − 2y − 1 = 0, 2x + 3y − 5 = 0, , , ,
3 2 3 2
Answer Key
Practice Exercise 1
1. (D) 2. (C) 3. (B) 4. (C) 5. (A) 6. (A)
7. (B) 8. (C) 9. (C) 10. (A) 11. (B) 12. (C)
13. (A) 14. (D) 15. (C) 16. (C) 17. (C) 18. (A)
19. (A) 20. (D) 21. (D) 22. (B) 23. (D) 24. (B)
25. (C) 26. (C) 27. (B) 28. (A) 29. (D) 30. (C)
31. (B) 32. (D) 33. (A) 34. Solutions
(A) 35. (A) 36. (D)
Practice Exercise 1 or x = 2 x1
Matrix Match Type Questions 782 20.5 Derivative of Second Order y″ or y2 877
Integer Type Questions 782 20.6 Differentiation of a Function with Respect to Another
Answer Key 782 Function 878
Solved JEE 2017 Questions 794 Previous Years' Solved JEE Main/AIEEE Questions 881
Previous Years' Solved JEE Advanced/IIT-JEE Questions 882
Chapter 19 Limit, Continuity and Practice Exercise 1 882
Differentiability 799
Practice Exercise 2 886
19.1 Limit of a Function 799
Single/Multiple Correct Choice Type Questions 886
19.2 Definition 799
Comprehension Type Questions 886
19.2.1 Informal Definition of Limit 799 Matrix Match Type Questions 887
19.2.2 Formal Definition of Limit 799 Integer Type Questions 887
19.2.3 Right Hand Limit 800
19.2.4 Left Hand Limit 800 Answer Key 888
19.3 Algebra of Limits 800 Solutions 888
19.4 Evaluation of Limits 800 Solved JEE 2017 Questions 895
19.4.1 Simplification 800
19.5 Use of Standard Limits 801
Chapter 21 Applications
19.6 Some More Standard Forms 802
of Derivatives 897
19.7 Use of Expansion 803
21.1 Geometrical Interpretation of Derivative 897
19.8 L’Hospital’s Rule 804
21.2 Tangent and Normal 897
19.9 Sandwich Theorem (Squeeze Play Theorem) 804
21.2.1 Length of Tangent, Normal, Subtangent and
19.10 Continuity 805 Subnormal 898
19.10.2 Geometrical Meaning of Continuity 805 21.3 Angles Between Two Curves 899
19.10.3 Continuity in an Open Interval 806
21.4 dy/dx as Rate Measures 900
19.10.4 Continuity in a Closed Interval 806
19.10.5 Properties of Continuous Functions 807 21.5 Errors and Approximations 900
19.10.6 Intermediate Value Theorem 807 21.6 Monotonicity of Function 901
19.10.7 Types of Discontinuities 807 21.6.1 Increasing Behaviour of Function 901
19.11 Differentiability 808 21.6.2 Decreasing Behaviour of Function 902
19.11.1 Differentiability in an Interval 809 21.6.3 Non-Decreasing Behaviour 902
19.11.2 Properties of Differentiability 809 21.6.4 Non-Increasing Behaviour 902
21.7 Maxima and Minima of Functions of a Single Variable 903
Additional Solved Examples 810
21.7.1 Concept of Local Maximum and Local Minimum 904
Previous Years’ Solved JEE Main/AIEEE Questions 814 21.8 Mean Value Theorems 906
Previous Years’ Solved JEE Advanced/IIT-JEE Questions 819 21.8.1 Rolle’s Theorem 906
21.8.2 Lagrange’s Mean Value Theorem 907
Practice Exercise 1 830
21.9 Geometrical Problems 907
Practice Exercise 2 842
Additional Solved Examples 909
Single/Multiple Correct Choice Type Questions 842
Comprehension Type Questions 843 Previous Years' Solved JEE Main/AIEEE Questions 911
Matrix Match Type Questions 844 Previous Years' Solved JEE Advanced/IIT-JEE Questions 918
Integer Type Questions 844
Practice Exercise 1 927
Answer Key 845
Practice Exercise 2 933
Solutions 845
Single/Multiple Correct Choice Type Questions 933
Solved JEE 2017 Questions 868
Comprehension Type Questions 934
Matrix Match Type Questions 935
Chapter 20 Differentiation 873 Integer Type Questions 936
20.1 Introduction 873 Answer Key 936
20.2 Differentiation from First Principle 873
Solutions 937
20.3 Derivatives of Some of the Frequently Used Functions 874
20.4 Rules to Find Out Derivatives 874 Solved JEE 2017 Questions 964
Contents xJJJ
Chapter 22 Indefinite Integration 969 Chapter 24 Area Under the Curves 1071
22.1 Primitive or Anti-Derivative of a Function 969 24.1 Curve Tracing 1071
22.2 Indefinite Integral and Indefinite Integration 969 24.2 Steps to Draw Curve 1071
22.2.1 Fundamental Properties of Integration 969 24.3 Area of Bounded Region 1072
22.2.2 Fundamental Formulas on Integration 969 24.4 Area Enclosed Between Two Curves 1073
22.3 Methods of Integration 972
Additional Solved Examples 1076
22.3.1 Integration by Substitution 972
22.3.2 Integration by Parts 976 Previous Years’ Solved JEE Main/AIEEE Questions 1079
22.4 Integration by Partial Fractions 979 Previous Years’ Solved JEE Advanced/IIT-JEE Questions 1083
Additional Solved Examples 993 Practice Exercise 1 1089
Previous Years' Solved JEE Main/AIEEE Questions 998 Practice Exercise 2 1092
Previous Years' Solved JEE Advanced/IIT-JEE Questions 1002 Single/Multiple Correct Choice Type Questions 1092
Practice exercise 1 1003 Comprehension Type Questions 1092
Matrix Match Type Questions 1093
Practice Exercise 2 1008
Integer Type Question 1094
Single/Multiple Correct Choice Type Questions 1008
Answer Key 1094
Comprehension Type Questions 1009
Matrix Match Type Questions 1009 Solutions 1094
Answer Key 1010 Solved JEE 2017 Questions 1109
Solutions 1010
Chapter 25 Differential Equations 1111
Solved JEE 2017 Questions 1019
25.1 Introduction 1111
25.2 Basic Definition 1111
Chapter 23 Definite Integration 1021
25.3 Order of a Differential Equation 1111
23.1 Definition 1021
25.4 Degree of a Differential Equation 1111
23.2 Geometrical Meaning of Definite Integration 1021
25.5 Formation of a Differential Equation 1111
23.3 Definite Integration as the Limit of Sum 1022 25.5.1 Steps for Formation of Differential Equations 1112
23.4 Properties of Definite Integration 1022 25.6 Solution of a Differential Equation 1113
23.5 Properties Based on Periodic Function 1028 25.6.1 General Solution 1113
23.6 Properties Based on Inequality 1030 25.6.2 Particular Solution 1113
23.7 Newton–Leibnitz Rule 1030 25.7 Differential Equations of First-Order and First-Degree 1114
23.8 Summation of Series by Integration 1031 25.7.1 Geometrical Interpretation of the Differential
23.8.1 Method to Express the Infinite Series as Definite Equations of First-Order and First-Degree 1114
Integral 1031 25.8 Solution of First-Order and First-Degree Differential
23.9 Reduction Formulae for Definite Integration 1032 Equations 1114
23.10 Wallis Formulae 1033 25.9 Variable Separable Type Differential Equation 1114
25.10 Equation Reducible to Variable Separable Type
Additional Solved Examples 1034
Differential Equation 1114
Previous Years’ Solved JEE Main/AIEEE Questions 1036 25.11 Homogeneous Type Differential Equation 1115
Previous Years’ Solved JEE Advanced/IIT-JEE Questions 1040 25.11.1 Steps for Solving Homogeneous
Differential Equation 1115
Practice Exercise 1 1048
25.12 Non-Homogeneous Type Differential Equation 1116
Practice Exercise 2 1052 25.13 Exact Differential Equation 1119
Single/Multiple Correct Choice Type Questions 1052 25.13.1 Integrating Factor 1119
Comprehension Type Questions 1052 25.13.2 Some Useful Results 1119
Matrix Match Type Questions 1053 25.14 Linear Differential Equation 1120
Integer Type Questions 1054 25.14.1 Linear Differential Equation of First Order 1120
25.14.2 Equation Reducible to Linear Differential
Answer Key 1054
Equation (Bernoulli’s Differential Equation) 1121
Solutions 1055 25.15 Solution of Differential Equation of the First Order but of
Solved JEE 2017 Questions 1067 Higher Degree 1122
xJW Contents
25.16 Applications of Differential Equation 1124 26.14.3 Vector Normal to the Plane of Two Given Vectors 1176
25.16.1 Problem Based on Rate of Change 1124 26.14.4 Area of Parallelogram and Triangle 1176
25.16.2 Problem Based on Geometry: Some Results on 26.14.5 Moment of a Force 1177
Tangents and Normal 1125 26.14.6 Moment of a Couple 1177
Additional Solved Examples 1128 26.15 Scalar Triple Product 1178
26.15.1 Geometrical Interpretation of Scalar Triple
Previous Years’ Solved JEE Main/AIEEE Questions 1139 Product 1178
Previous Years’ Solved JEE Advanced/IIT-JEE Questions 1143 26.15.2 Properties of Scalar Triple Product 1178
26.15.3 Tetrahedron 1178
Practice Exercise 1 1143 26.15.4 Properties of a Tetrahedron 1179
Practice Exercise 2 1147 26.15.5 Volume of a Tetrahedron 1179
26.15.6 Reciprocal System of Vectors 1179
Single/Multiple Correct Choice Type Questions 1147
26.16 Vector Triple Product 1180
Comprehension Type Questions 1147
26.16.1 Properties of Vector Triple Product 1180
Answer Key 1148 26.17 Scalar or Vector Product of Four Vectors 1181
Solutions 1149 26.17.1 Scalar Product 1181
26.17.2 Vector Product 1181
Solved JEE 2017 Questions 1160
26.18 Method to Prove Collinearity 1181
26.19 Vector Equation 1182
Chapter 26 Vector Algebra 1163
26.1 Introduction 1163 Additional Solved Examples 1183
26.1.1 Scalar and Vector Quantities 1163 Previous Years’ Solved JEE Main/AIEEE Questions 1185
26.2 Representation of a Vector 1163
Previous Years’ Solved JEE Advanced/IIT-JEE Questions 1190
26.3 Types of Vectors 1163
26.4 Rectangular Resolution of Vectors (Orthogonal System of Practice Exercise 1 1197
Vectors): Resolution of a Vector in Two Dimensions 1164 Practice Exercise 2 1200
26.5 Resolution of a Vector in Three Dimensions 1164
Single/Multiple Correct Choice Type Questions 1200
26.6 Properties of Vectors 1165
Comprehension Type Questions 1201
26.7 Fundamental Theorems of Vectors 1168 Integer Type Questions 1201
26.7.1 Fundamental Theorems of Vectors in Two
Dimensions 1168 Answer Key 1201
26.7.2 Fundamental Theorems of Vectors in Three Solutions 1202
Dimensions 1168
Solved JEE 2017 Questions 1211
26.8 Linear Combinations of Vectors 1168
26.8.1 Collinear and Non-Collinear Vectors 1168 Chapter 27 Three-Dimensional
26.8.2 Relation Between Two Parallel Vectors 1168
26.8.3 Test of Collinearity of Three Points 1168
Geometry 1213
26.8.4 Test of Coplanarity of Three Vectors 1168 27.1 Rectangular Coordinate System in Space 1213
26.8.5 Test of Coplanarity of Four Points 1168 27.1.1 Coordinates of a Point in Space 1213
26.9 Linearly Dependent and Independent Vectors 1168 27.1.2 Signs of Coordinates of a Point 1213
26.9.1 Linearly Independent Vectors 1168 27.2 Other Methods of Defining the Position of Any Point
26.9.2 Linearly Dependent Vectors 1168 P in Space 1213
26.10 Position Vector of a Dividing Point (Section 27.2.1 Cylindrical Coordinates 1213
Formulae) 1169 27.2.2 Spherical Polar Coordinates 1213
26.11 Bisector of the Angle Between Two Vectors 1170 27.3 Shifting the Origin 1214
26.12 Product of Two Vectors 1171 27.4 Distance Formula 1214
26.13 Scalar or Dot Product of Two Vectors 1171 27.4.1 Distance of a Point from Coordinate Axes 1214
26.13.1 Geometrical Interpretation of Scalar Product 1171 27.5 Section Formula 1214
26.13.2 Properties of Scalar Product 1172 27.5.1 Internal Division 1214
26.13.3 Components of a Vector Along and Perpendicular to 27.5.4 Coordinates of the General Point 1214
Another Vector 1173 27.6 Triangle and Tetrahedron 1215
26.13.4 Work Done by a Force 1173 27.6.1 Coordinates of the Centroid 1215
26.14 Vector or Cross-Product of Two Vectors 1174 27.6.2 Area of a Triangle 1215
26.14.1 Geometrical Interpretation of the Vector 27.6.4 Condition of Collinearity 1215
Product 1175 27.7 Direction Cosines of a Line 1215
26.14.2 Properties of Vector Product 1175 27.7.1 Relation Between the Direction Cosines 1216
Contents xW
Previous Years’ Solved JEE Main/AIEEE Questions 1281 Answer Key 1299
Previous Years’ Solved JEE Advanced/IIT-JEE Questions 1285 Solutions 1299
Practice Exercise 1 1292 Solved JEE 2017 Questions 1310
Practice Exercise 2 1296
Single/Multiple Correct Choice Type Questions 1296
Appendix: Chapterwise Solved
Comprehension Type Questions 1297 JEE 2018 Questions A-1
Matrix Match Type Questions 1297
Integer Type Questions 1298
17 Inverse Trigonometry
17.1 Introduction p p
Similarly, even if cot æç - ö÷ = - 3 but cot −1 ( - 3 ) ≠ -
The inverse of a function f : A → B exists if f is one-one onto, that is, è 6ø 6
a bijection and is given by f (x) = y ⇒ f −1(y) = x. because principal range of cot −1x is (0,π).
Consider the sine function with domain R and range [−1, 1]. So, cot−1 ( - 3 ) = 5p only.
Clearly this function is not a bijection and so it is not invertible. If 6
we restrict the domain of it in such a way that it becomes one–one, Note:
then it would become invertible. If we consider sine as a function 1. See Fig. 17.1. Here, sin−1 x, cosec−1 x, tan−1 x, belong to I and IV
é p pù quadrants.
with domain ê - , ú and co-domain [−1, 1], then it is a bijection p /2
ë 2 2û
and therefore, invertible. The inverse of sine function is defined as
é p pù I
sin-1 x = q Û sinq - x , where ê - , ú and x Î [ -1,1]
ë 2 2û
Hence, sin−1 x is an angle and it denotes the smallest numerical IV
angle, whose sine is x.
−p/2
17.2 Domain and Range of Inverse Figure 17.1
Trigonometric Functions 2. See Fig. 17.2. Here, cos−1 x, sec−1 x, cot−1 x, belong to I and II
p quadrant.
We know that tan = 3
3
p
This is written in inverse trigonometry as = tan−1 3 . II I
3
0
But, tan 4π is also equal to 3
3
Does it mean, 4π = tan−1 3 ?
3
The answer is no, tan−1 3 is taken as the numerically least angle Figure 17.2
whose tangent is 3 . This is done to associate a single value to 3. I quadrant is common to all the inverse functions.
tan−1 3 to safeguard the definition of a function. 4. III quadrant is not used in inverse functions.
So, the equations tan x = y and x = tan−1 y are not identical π
5. IV quadrant is used in the clockwise direction, that is, − ≤ y ≤ 0.
because the former associates many values of x to a single value 2
of y, while the latter associates a single x to a particular value of y. The principal range of inverse trigonometric functions is the most
In the same way, the remaining five inverse trigonometric func- important thing in this lesson. All formula and problems are linked
tions are also defined. To assign a unique angle to a particular in some way or the other to that only.
value of trigonometric ratio, we introduce a term called ‘principal
range’. The principal ranges of all the inverse trigonometric func- 1. See Fig. 17.3. If sin y = x, then y = Y (1, p /2 )
tions have been fixed. For example, principal range of sin−1 x is sin−1 x, under certain condition.
⎡ p p⎤ −1 ≤ siny ≤ 1, but sin y = x.
⎢ − 2 , 2 ⎥ , that is, we have to search for an angle in this interval only. y = sin−1x
⎣ ⎦ Hence,
X
1 p 5p 1 13p 1 −1 ≤ x ≤ 1 O
sin−1 = only, although sin = , sin = , etc. Again,
2 6 6 2 6 2
p
sin y = -1 Þ y = -
æ ö 2 (−1,−p /2)
ç -1 1 1 ÷ p
ç note that sin ¹ ÷ and sin y = 1⇒ y = Figure 17.3
çç 2 sin 1 ÷ 2
÷
è 2ø Keeping in mind numerically smallest angles or real numbers.
700 Mathematics Problem Book for JEE
Hence, So,
p p Domain: x ∈ R
- £y£
2 2 Range: y ∈ (0, p )
These restrictions on the values of x and y provide us with the 5. See Fig. 17.7. If sec y = x, then
p
domain and range for the function, y = sin−1 x. y = sec−1 x
So, p y = p /2
where | x | ≥ 1 and 0 £ y £ p , y ¹
Domain: x ∈ [−1,1] 2 X
Here,
é p pù
Range: y Î ê - , ú Domain: x ∈ R − (−1,1) (1,0)
ë 2 2û y = sec−1x
ìp ü
2. See Fig. 17.4. If cos y = x, then y = cos−1 x, Range: y Î [0 , p ] - í ý
î2 þ Figure 17.7
under certain conditions.
6. See Fig. 17.8. If cosec y = x, then y
−1 ≤ cos y ≤ 1 ⇒ −1 ≤ x ≤ 1 y = cosec−1 x
p p (1,p /2)
cos y = −1 ⇒ y = p where | x | ≥ 1 and - £ y £ , y ¹ 0
cos y = 1 ⇒ y = 0 2 2
x
Y Here,
(–1, p /2) Domain: x ∈ R − (−1,1)
(−1,−p) O y = cosec−1x
é p pù
y = cos–1x Range: y Î ê - , ú - {0}
ë 2 2û Figure 17.8
X
O (1, 0)
We list below (Table 17.1) the domain and principal ranges of all
the six inverse trigonometric functions.
Table 17.1 Domain and principal ranges of all the six inverse trig-
Figure 17.4 onometric functions
Hence, 0 ≤ y ≤ π {as cos x is a decreasing function in [0, π]}.
Function Domain Principal Range
These restrictions on the values of x and y provide us the (values of x) (values of y)
domain and range for the function, y = cos−1 x.
So, é p pù
y = sin−1 x [−1, 1] ê- 2 , 2 ú
Domain: x ∈ [−1,1] ë û
Range: y ∈ [0, p ] y = cos−1 x [−1, 1] [0, π]
3. See Fig. 17.5. If tan y = x, then y
y = tan−1 x, under certain æ p pö
y = p /2
y = tan−1 x (−∞, ∞) ç- , ÷
conditions. è 2 2ø
tan y ∈ R ⇒ x ∈ R,
p p X é p pù
−∞ < tan y < ∞ ⇒ − < y < O y = cosec−1 x (−∞, −1] ∪ [1, ∞) ê - 2 , 2 ú - {0}
2 2 ë û
Thus,
y = −p /2 y = tan–1x
[0,p ] - ìí 2 üý
Domain: x ∈ R; p
y = sec−1 x (−∞, −1] ∪ [1, ∞)
æ p pö Figure 17.5 î þ
Range: y Î ç - , ÷
è 2 2ø y = cot−1 x (−∞, ∞) (0, π)
p æ 1ö
⇒ 0< -q < p ⇒ ç ÷ ∈ [−1, 1] − {0}
2 èxø
⇒ (π/2 − θ ) ∈ (0, π)
æ 1ö
⇒ q = cos -1 ç ÷
Since, èxø
tan−1x = θ æ 1ö
⇒ x = tanθ Hence, sec -1x = cos-1 çç ÷÷÷
è xø
⇒ x = cot (π/2 − θ)
Proof (iii): Let cot−1x = θ
⇒ cot−1x = π/2 − θ where, −∞ < x < ∞ and 0 < θ < π
⇒ θ + cot−1x = π/2 Now, consider two cases,
Hence, tan−1x + cot−1x = π/2.
Case I: x>0
Proof (iii): sec−1x + cosec−1x = π/2, ∀ x ∈ (−∞, −1] ∪ [1, ∞) cot−1 x = θ ⇒ θ ∈ (0, π/2)
Let sec−1x = θ. Then x ∈ (−∞, −1] ∪ [1, ∞). 1 æ 1ö
⇒x = cotθ ⇒ = tanθ ⇒ θ = tan−1 ç ÷
ìp ü x èxø
where, q Î [0 , p ] - í ý
î2 þ Hence, tan−1(1/x) = cot−1x, for all x > 0.
ìp ü Case II: x<0
⇒ q Î [0 , p ] - í ý
î2 þ ⇒ θ ∈ (π/2, π)
p é p pù p ⇒ π/2 < θ < π
⇒ - q Î ê- , ú , - q ¹ 0
2 ë 2 2û 2 ⇒ −π/2 < (θ − π) < 0
Since, ⇒ (θ − π) ∈ (−π/2, 0)
sec−1x = θ Hence, cot−1x = θ
⇒ x = sec θ ⇒ cotθ = x
1
⇒ x = cosec (π/2 − θ) ⇒ = tanθ
x
⇒ cosec−1x = π/2 − θ
1
⇒ θ + cosec−1x = π/2 ⇒ = − tan(π − θ)
x
Hence, sec−1x + cosec−1 x = π/2.
1
⇒ = tan (θ − π)
Property 5 x
æ 1ö æ 1ö
(i) sin-1 ç ÷ = cosec -1x , " x ∈ (−∞, −1] ∪ [1, ∞) ⇒ (θ − π) = tan−1 ç ÷
èxø èxø
⎛ 1⎞ æ 1ö
(ii) cos −1 ⎜ ⎟ = sec −1 x , ∀ x ∈ (−∞, −1] ∪ [1, ∞) ⇒ tan−1 ç ÷ = − π + θ
⎝ x⎠ èxø
−1
⎛ 1 ⎞ ⎧ cot x , ∀ x > 0 æ 1ö
(iii) tan−1 ⎜ ⎟ = ⎨ So, tan−1 ç ÷ = − π + cot−1x, when x < 0.
⎝ x ⎠ ⎩ −p + cot −1 x , ∀ x < 0 èxø
Proof (i): Let cosec−1x = θ. Then x = cosec θ. Hence,
é p pù 1 ⎧ cot −1 x , ∀ x > 0⎫
where, q Î ê - , ú - {0} tan−1 =⎨ ⎬
ë 2 2û x ⎩ −p + cot −1 x , ∀ x < 0⎭
-1 -1
æ x ö -1
æ 1 ö -1 æ 1 ö æ x2 x3 ö
(iii) tan x = sin çç ÷÷ = cos çç ÷÷ = cot ç ÷ Illustration 17.6 If sin-1 çç x - + - ××× ÷÷
è 1+ x
2
ø è 1+ x
2
ø èxø è 2 4 ø
-1 æ 2 ö p
4 6
æ 1+ x 2 ö + cos çç x -
x
+
x
- ××× ÷÷ = for 0 < | x | < 2 , then find the
= sec -1 1+ x 2 = cosec -1 ç ÷ 2 4
ç x ÷ è ø 2
è ø
value of x.
Illustration 17.3 Evaluate the following: Solution: We know that
æ 5p ö é æ 2p öù
(A) sec−1[sec(−30°)] (B) sin-1 ç sin ÷ (C) sin-1 êsin ç ÷ú p
è 3 ø ë è 3 øû sin-1 y + cos -1 y = , | y | £1
2
Solution:
Hence, according to question,
(A) sec−1[sec(−30°)] = sec−1(sec 30°) = 30°
x2 x3 x4 x6
æ 5p ö -1 æ 3ö p x- + - ××× = x 2 - + - ×××
(B) sin-1 ç sin ÷ = sin çç - ÷÷ = - 2 4 2 4
è 3 ø è 2 ø 3
p öù é æ öù p
p x x2 x x2
-1 é æ -1 ⇒ = , (\ 0 < | x | < 2 ) ⇒ =
(C) sin êsin ç p - ÷ ú = sin êsin ç ÷ ú = x x 2
2 + x 2 + x2
ë è 3 øû ë è 3 øû 3 1+ 1+
2 2
Illustration 17.4 If θ = sin−1 x + cos−1 x − tan−1 x, x ≥ 0, then the ⇒ 2x + x3 = 2x2 + x3 ⇒ x = x2
smallest interval in which θ lies is
Hence,
p 3p p
(A) £q £ (B) 0 £ q £ x − x2 = 0 ⇒ x(1 − x) = 0 ⇒ x = 0 and x = 1, but x ≠ 0.
2 4 4
So, x = 1.
p p
(C) - p £ q £ 0 (D) £q £
4 2 2p
4 Illustration 17.7 If sin-1 x + sin-1 y = , then cos−1 x + cos−1 y =
3
Solution: ______.
p
q = sin-1 x + cos -1 x - tan-1 x = - tan-1 x 2π π π
2 (A) (B) (C) (D) π
We know 3 3 6
⎛ p p⎞
tan−1 x = A where x¨R and A ¨ ⎜ − , ⎟ Solution:
⎝ 2 2⎠ 2p
sin-1 x + sin-1 y =
Hence, 3
p p −1 p p p p p 2p
≤ − tan x≤ ⇒ ≤q ≤ ⇒ - cos -1 x + - cos -1 y =
4 2 2 4 2 2 2 3
p
Illustration 17.5 Find the value of x which satisfies the equation ⇒ cos -1 x + cos -1 y =
3
é æ 1 öù
tan(cos-1 x ) = sin êcot -1 ç ÷ ú .
ë è 2 øû
Your Turn 1
æ 1ö 1
Solution: Put cot -1 ç ÷ = q . Then cot q = .
è2ø 2 -1 -1 æ 1 ö p
1. If sin x + cot ç ÷ = , then x is
Hence, è2ø 2
2
sinq = 1
5 (A) 0 (B)
5
Put cos−1 x = φ, then x = cosφ. 2 3
(C) (D) Ans. (B)
Also 5 2
2
tanf = sinq =
5 2. The value of sin(cos−1 x) is
Therefore (A) (1 + x2)3/2 (B) (1 + x2)−3/2
5 (C) (1 − x2)1/2 (D) (1 + x2)−1/2
x = cosf =
3 Ans. (C)
704 Mathematics Problem Book for JEE
As (A+B) lies in 2nd or 4th quadrant. So, As (A+B) lies in 1st or 3rd quadrant. So,
æ pö
æ p ö A + B Î ç -p , - ÷
A + B Îç - , 0 ÷ è 2ø
è 2 ø
⎛ x+y⎞
⎛ x+y⎞ ⇒ tan−1[tan( A + B + p )] = tan−1 ⎜
⇒ tan−1[tan( A + B )] = tan−1 ⎜ ⎝ 1− xy ⎟⎠
⎝ 1− xy ⎟⎠
⎛ x+y⎞
⎛ x+y⎞ A + B + p = tan−1 ⎜
A + B = tan−1 ⎜ ⎝ 1− xy ⎟⎠
⎝ 1− xy ⎟⎠
⎛ x+y⎞
⎛ x+y⎞ tan−1 x + tan−1 y = −p + tan−1 ⎜
tan−1 x + tan−1 y = tan−1 ⎜ ⎝ 1− xy ⎟⎠
⎝ 1− xy ⎟⎠
Proof (ii): Let tan−1 x = A and tan−1 y = B where
Case (c): When x > 0, y < 0
æ p pö
x , y Î R and A, B Î ç - , ÷ . Then
æ pö æ p ö è 2 2ø
x > 0 Þ A Î ç 0, ÷ , y < 0 Þ B Î ç - , 0 ÷
è 2ø è 2 ø tan A - tan B x - y
tan( A - B ) = =
æ p p ö 1+ tan A tan B 1+ xy
Þ A + B Îç - , ÷
è 2 2ø Case (a): When xy > −1, then
−1 −1 ⎛
x+y⎞ x-y
⇒ tan [tan( A + B )] = tan ⎜ tan( A - B ) =
⎝ 1− xy ⎟⎠ 1+ xy
⎛ x+y⎞ æ pö æ pö
A + B = tan−1 ⎜ if x > 0 , A Î ç 0 , ÷ , y > 0 Þ B Î ç 0 , ÷
⎝ 1− xy ⎟⎠ è 2ø è 2ø
⎛ x+y⎞ æ p pö
tan−1 x + tan−1 y = tan−1 ⎜ Þ A - B Îç - , ÷
⎝ 1− xy ⎟⎠ è 2 2ø
= p + tan í- 2 3
ý
î cos q - sin q
4 4
þ Property 7
-1 ì - sinq cosq ü
ìsin-1( x 1- y 2 + y 1- x 2 ),
= p + tan í 2 ý ï
î cos q - sin2 q þ ï if x , y Î [ -1,1] and x 2 + y 2 £ 1
ï
-1 ì 1 ü ï or iff xy < 0 and x 2 + y 2 > 1
= p + tan í- tan 2q ý ïï
î 2 þ
(1) sin-1 x + sin-1 y = íp - sin-1( x 1- y 2 + y 1- x 2 ),
-1 æ 1 ö ï
= p - tan ç tan 2q ÷ ï if x , y Î (0 ,1] and x 2 + y 2 > 1
è2 ø ï -1
ï-p - sin ( x 1- y + y 1- x ),
2 2
π
since 2q < and tan 2q > 0 ï
2 ïî if x , y Î [ -1, 0 ) and x 2 + y 2 > 1
Hence,
ìsin-1( x 1- y 2 - y 1- x 2 ),
æ1 ö ï
tan−1 ç tan 2q ÷ + tan−1 (cot θ ) + tan−1 (cot3 θ ) = π
è2 ø ï if x , y Î [ -1,1] and x 2 + y 2 £ 1
ï
ï or iff xy > 0 and x 2 + y 2 > 1
π π ïï
Case (b): If < q < , then 0 < cot q < 1, 0 < cot3 q < 1
4 2 (2) sin-1 x - sin-1 y = íp - sin-1( x 1- y 2 - y 1- x 2 ),
Therefore, ï
æ 1 ö ï if x Î (0 ,1], y Î [-1, 0 ) and x 2 + y 2 > 1
tan−1 (cot θ ) + tan−1 (cot3 θ ) = tan−1 ç - tan 2q ÷ ï -1
ï-p - sin ( x 1- y - y 1- x ),
2 2
è 2 ø
ï
æ1 ö ïî if x Î [ -1, 0 ), y Î (0 ,1] and x 2 + y 2 > 1
= −tan−1 ç tan2q ÷ {since 2q > p and tan2q < 0}
è2 ø
Chapter 17 | Inverse Trigonometry 707
x y æ x2 ö æ y2 ö
cos −1 x + cos −1 y = 2π − cos −1( xy − 1− x 2 1− y 2 ) Þ × - çç 1- ÷÷ çç 1- ÷÷ = cosq
2 3 è 4 ø è 9 ø
Proof (2): Replace y by −y in Proof (1).
⇒ (xy − 6 cosθ )2 = (4 − x2)(9 − y2)
Illustration 17.14 If cos−1x + cos−1y +cos−1z = p, then prove that ⇒ 9x2 − 12xy cosθ + 4y2 = 36(1 − cos2θ) = 36sin2θ
x2 + y2 + z2 + 2xyz = 1.
Solution: Given,
Your Turn 2
cos−1x + cos−1y + cos−1z = p ìx 1 ü
1. If f ( x ) = cos -1 x + cos -1 í + 3 - 3 x 2 ý , then
⇒ cos−1x + cos−1y = p − cos−1z = cos−1(-z) î 2 2 þ
⇒ cos[cos−1x + cos−1y] = cos[cos−1(−z)] æ2ö p æ2ö 2 p
(A) f ç ÷ = (B) f ç ÷ = 2 cos -1 -
Let cos−1x = A and cos−1y = B. Then è3ø 3 è3ø 3 3
cos(A + B) = cosA cosB − sinA sinB 1 p 1 p
(C) f æç ö÷ = (D) f æç ö÷ = 2 cos -1 -
1
Ans. (A), (D)
⇒ cos(A + B) = xy - 1- x 2 1- y 2 è3ø 3 è3ø 3 3
æxö æyö x 2 2 xy y2
⇒ (A + B) = cos−1 [ xy - 1- x 2 1- y 2 ] 2. If cos -1 ç ÷ + cos -1 ç ÷ = a , then - cos a + =
èaø èbø a2 ab b2
⇒ cos−1 ( xy - 1- x 2 1- y 2 ) = cos−1(−z) (A) sin2 α (B) cos2 α
⇒ xy − 1- x 1- y = −z
2 2
(C) tan2 α (D) cot2 α Ans. (A)
⇒ (xy + z)2 = (1 − x2) (1 − y2) ⇒ x2y2 + z2 + 2xyz = 1 − x2 − y2 + x2y2 3. All possible values of p and q for which
⇒ x2 + y2 + z2 + 2xyz = 1 3p
cos -1 p + cos -1 1- p + cos -1 1- q = holds is
Hence, proved. 4
4 1 4 1 1 1
Illustration 17.15 If a = sin-1 + sin-1 and b = cos -1 + cos -1 , (A) p = -1, q = (B) q > 1, p =
5 3 5 3 2 2
then 1
(C) 0 £ p £ 1, q = (D) None of these Ans. (C)
(A) α < β (B) α = β 2
(C) α > β (D) None of these -1 -1 p
4. The number of solutions of sin x + sin 2 x = is
Solution: 3
é4 1 1 16 ù (A) 0 (B) 1 (C) 2 (D) Infinite Ans. (B)
a = sin-1 ê 1- + 1- ú
êë 5 9 3 25 úû æ 3ö æ 5ö
5. Obtain the value of cos -1 ç - ÷ + sin-1 ç - ÷ in terms of cos−1
è 5ø è 13 ø
é8 2 3 ù æ 8 2 +3ö æ 16 ö
= sin-1 ê + ú = sin-1 çç ÷÷ function. Ans. cos -1 ç - ÷
êë 15 15 ûú è 15 ø è 65 ø
12 4 63
6. sin-1 + cos -1 + tan-1 =
8 2 +3 13 5 16
Since < 1, therefore
(B) π (D) 2π
15
(A) 0 (C) π Ans. (C)
p 2 3
a<
2 7. If a, b, c be positive real numbers and the value of
a(a + b + c ) b(a + b + c )
æp 4 p 1ö p q = tan-1 + tan-1 , then tanθ is
b = ç - sin-1 + - sin-1 ÷ = (p - a ) > bc ca
è 2 5 2 3 ø 2
⇒α<β (A) 0 (B) 1 (C) a + b + c (D) None of these Ans. (A)
Chapter 17 | Inverse Trigonometry 709
Property 9 ⎡ p p⎤
Proof (2): Let sin−1x = A where x ∈[ −1,1] and A ∈ ⎢ − , ⎥ . Then
ì -1 1 ⎣ 2 2⎦
ï-p - sin (2 x 1- x ), if - 1 £ x < -
2
ï 2 x = sin A
-1 ï -1 1 1 sin 3A = 3 sin A − 4 sin3 A = 3x − 4x3
(1) 2 sin x = ísin (2 x 1- x ), if -
2
£x£
ï 2 2 é 3p 3p ù
ï 1 where 3 AÎê - , ú
-1 ë 2 2 û
ïp - sin (2 x 1- x ), if < x £1
2
î 2
ì 1 é 3p p ö
-1 Case (a): If 3 AÎ ê - , - ÷ , then
ï-p - sin (3 x - 4 x ), if - 1 £ x < - 2
3
ë 2 2ø
ï
ï 1 1 é 3p p ö é
(2) 3 sin-1 x = ísin-1(3 x - 4 x 3 ), if - £ x £ A Î ê-
1ö
, - ÷ Þ x Î ê -1, - ÷
ï 2 2
ë 6 6ø ë 2ø
ï -1 1
ïp - sin (3 x - 4 x ), if 2 < x £ 1
3
sin−1 (sin 3A) = sin−1 (3x − 4x3)
î
⇒ sin−1 [sin(−π − 3A)] = sin−1 (3x − 4x3)
é p pù ⇒ −π − 3A = sin−1 (3x − 4x3)
Proof (1): Let sin-1x = A where x Î [ -1,1] and A Î ê - , ú . Then
ë 2 2û ⇒ 3A = −π − sin−1 (3x − 4x3)
x = sin A ⇒ 3sin−1 x = −π − sin−1 (3x − 4x3)
sin 2 A = 2 sin A cos A = 2 x 1- x 2 æ p 3p ù
Case (b): If 3 AÎ ç , ú , then
where 2A ∈ [−π,π] è2 2 û
⎡ p⎞ æp p ù æ1 ù
Case (a): If 2 A ∈ ⎢ −p , − ⎟ , then A Î ç , ú Þ x Î ç ,1ú
⎣ 2⎠ è 6 2 û è2 û
⎡ p p⎞ ⎡ 1 ⎞ sin−1 (sin 3A) = sin−1 (3x − 4x3)
A ∈ ⎢ − , − ⎟ ⇒ x ∈ ⎢ −1, − ⎟
⎣ 2 4⎠ ⎣ 2⎠ ⇒ sin−1 [sin(π − 3A)] = sin−1 (3x − 4x3)
⇒ π − 3A = sin−1 (3x − 4x3)
sin-1 (sin 2 A) = sin-1 (2 x 1- x 2 ) ⇒ 3A = π − sin−1 (3x − 4x3)
⇒ 3sin−1 x = π − sin−1 (3x − 4x3)
⇒ sin−1 [(sin( −p − 2 A)] = sin−1 (2 x 1− x 2 )
é p pù
⇒ −p − 2 A = sin−1 (2 x 1− x 2 ) Case (c): If 3 AÎ ê - , ú , then
ë 2 2û
Þ 2 A = -p - sin-1 (2 x 1- x 2 )
é p pù é 1 1ù
A Î ê- , ú Þ x Î ê- , ú
-1 -1 ë 6 6û ë 2 2û
Þ 2 sin x = -p - sin (2 x 1- x ) 2
æ 2x ö æ 3x - x3 ö
Þ 2 tan-1 x = p + tan-1 ç ÷ Þ -p + 3 A = tan-1 ç 2 ÷
è 1- x 2 ø è 1- 3 x ø
æ p pö æ 3x - x3 ö
Proof (2): Let tan−1 x = A where x ∈ R and A ∈ ç - , ÷ . Then Þ 3 A = p + tan-1 ç
è 2 2ø 2 ÷
x = tan A è 1- 3 x ø
3 tan A − tan3 A 3 x − x 3 æ 3x - x3 ö
tan 3 A = = 3 tan-1 x = p + tan-1 ç 2 ÷
1− 3 tan2 A 1− 3 x 2 è 1- 3 x ø
æ 3p 3p ö Illustration 17.17 Let α, β and γ are three angles given by
3 AÎ ç - , ÷
è 2 2 ø
æ 3p p ö
a = 2 tan-1 ( )
2 - 1 , b = 3 sin-1
1
2
æ 1ö æ 1ö
+ sin-1 ç - ÷ and g = cos -1 ç ÷ .
è 2ø è3ø
Case (a): If 3 AÎ ç - , - ÷ , then
è 2 2ø Then
⎛ 3x − x3 ⎞ Solution:
tan−1 ( tan 3 A) = tan−1 ⎜ ⎟
⎝ 1− 3 x 2 ⎠ a = 2 tan−1 ( )
2 − 1 = 2 tan−1 tan
p
8
⎛ 3x − x3 ⎞ p p 1
⇒ tan−1 ⎡⎣ tan (p + 3 A)⎤⎦ = tan−1 ⎜ ⎟ = 2× = = cos −1
⎝ 1− 3 x 2 ⎠ 8 4 2
æ 3x - x3 ö p p 7p
Þ p + 3 A = tan-1 ç b = 3× - =
2 ÷ 4 6 12
è 1- 3 x ø
Therefore, β > α. Also,
æ 3x - x 3 ö
Þ 3 A = -p + tan-1 ç 2 ÷
1 1
<
è 1- 3 x ø 3 2
æ 3x - x 3 ö 1 1 p
Þ 3 tan-1 x = -p + tan-1 ç 2 ÷
Þ cos -1 > cos -1 =
è 1- 3 x ø 3 2 4
So, γ > α.
é p pù
Case (b): If 3 AÎ ê - , ú , then ⎛ 1⎞
ë 2 2û Again cos −1 ⎜ ⎟ belongs to the first quadrant and β is in the
⎝ 3⎠
é p pù é 1 1 ù second quadrant.
A Î ê- , ú Þ x Î ê- , ú
ë 6 6û ë 3 3û Hence, β > γ .
é 3ù é 1 ù 3 1 14
sin2 A =
2x
= sin êsin-1 ú + cos êcos -1 ú = + =
ë 5û ë 3 û 5 3 15 1+ x 2
æ 2x ö
Property 12 Þ sin-1[sin(p - 2 A)] = sin-1 ç ÷
è 1+ x 2 ø
⎧ −1 ⎛
2x ⎞
⎪ −p − sin ⎜⎝ 1+ x 2 ⎟⎠ , if x < −1 æ 2x ö
Þ p - 2 A = sin-1 ç
⎪ ÷
è 1+ x 2 ø
⎪ ⎛ 2x ⎞
2 tan x = ⎨sin−1 ⎜
−1
, if − 1 ≤ x ≤ 1
⎪ ⎝ 1+ x 2 ⎟⎠ æ 2x ö
Þ 2 A = p - sin-1 ç ÷
⎪ è 1+ x 2 ø
⎛ 2x ⎞
⎪p − sin−1 ⎜ , if x > 1
⎩ ⎝ 1+ x 2 ⎟⎠ æ 2x ö
Þ 2 tan-1 A = p - sin-1 ç ÷
è 1+ x 2 ø
æ p pö
Proof: Let tan-1 x = A where x Î R and A Î ç - , ÷ . Then
è 2 2ø Illustration 17.19 The solution set of the equation
x = tan A sin−1 x = 2tan−1 x is
2 tan A 2x (A) {1, 2} (B) {−1, 2}
sin 2 A = =
1+ tan A 1+ x 2
2
(C) {−1,1,0} (D) {1, 1/2, 0}
where 2A ∈ (−π,π) Solution:
æ pö sin−1 x = 2tan−1 x ⇒ sin−1 x = sin−1
2x
Case (a): If 2 AÎ ç -p , - ÷ , then
è 2ø 1+ x 2
æ p pö 2x
⇒ = x ⇒ x3 − x = 0 ⇒ x(x + 1) (x − 1) = 0 ⇒ x = {−1,1,0}
A Î ç - , - ÷ Þ x Î ( -¥ , -1) 1+ x 2
è 2 4ø
2x
Illustration 17.20 If 2 tan−1 x + sin−1 is independent of x,
2x 1+ x 2
sin2 A =
1+ x 2 then
⎛ 2x ⎞ (A) x ∈ [1, + ∞) (B) x ∈ [−1,1]
⇒ sin−1[sin( −p − 2 A)] = sin−1 ⎜
⎝ 1+ x 2 ⎟⎠ (C) x ∈ (−∞, −1] (D) None of these
æ 2x ö Solution: Let x = tanθ. Then
Þ -p - 2 A = sin-1 ç 2 ÷
è 1+ x ø 2x 2 tanq
sin-1 = sin-1 = sin-1(sin 2q )
æ 2x ö 1+ x 2 1+ tan2 q
Þ 2 A = -p - sin-1 ç ÷
è 1+ x 2 ø Hence, 2 tan-1 x + sin-1
2x
= 2q + sin-1(sin 2q ).
1+ x 2
æ 2x ö
Þ 2 tan-1 A = -p - sin-1 ç ÷ p p
è 1+ x 2 ø If - £ 2q £ , then
2 2
é p pù 2x
Case (b): If 2 AÎ ê - , ú , then 2 tan-1 x + sin-1 = 2θ + 2θ = 4 tan−1x
ë 2 2û 1+ x 2
é p pù which is not independent of x
A Î ê - , ú Þ x Î [ -1,1]
ë 4 4û p p
If − ≤ p − 2q ≤ , then
2x 2 2
sin2 A =
1+ x 2 2x
2 tan−1 x + sin−1
⎛ 2x ⎞ 1+ x 2
⇒ sin−1[sin(2 A)] = sin−1 ⎜
⎝ 1+ x 2 ⎟⎠ = 2θ + sin−1 [sin(π − 2θ )] = 2θ + π − 2θ = π
⎛ 2x ⎞ which is independent of x
⇒ 2 A = sin−1 ⎜
⎝ 1+ x 2 ⎟⎠ é p pù é p 3p ù
Hence, q Ï ê - , ú but q Î ê - , ú and from the principal
⎛ 2x ⎞ ë 4 4 û ë 4 4 û
⇒ 2 tan−1 A = sin−1 ⎜
⎝ 1+ x 2 ⎟⎠ value of tan−1 x.
é p pù
æp ö q Î ê- , ú
Case (c): If 2 AÎ ç , p ÷ , then ë 2 2û
è2 ø
Hence,
æp p ö p p
A Î ç , ÷ Þ x Î (1, ¥ ) q Î éê , ùú
è4 2ø ë4 2û
Chapter 17 | Inverse Trigonometry 713
2x
Þ tan-1 x + sin-1 =p Illustration 17.21 Write in the simplest form:
1+ x 2
æ cos x ö p 3p
p tan-1 ç ÷ where - £ x £
Also at q = , è 1+ sin x ø 2 2
4
2x p p p Solution:
2 tan-1 x + sin-1 = 2 × + sin-1 1 = + = p
1+ x 2
4 2 2 é æp ö ù
ê sin ç - x ÷ ú
ép p ù æ cos x ö è 2 ø ú
Hence, the given function = π = constant if q Î ê , ú , that is, tan-1 ç ÷ = tan ê
-1
ë4 2û è 1+ sin x ø ê1+ cos æ p - x ö ú
êë ç ÷ú
x ∈ [1, + ∞). è2 øû
Property 13 é æp x ö æp x öù
ê 2 sin ç 4 - 2 ÷ cos ç 4 - 2 ÷ ú
⎧ −1 ⎛
1− x 2 ⎞ = tan ê
-1 è ø è øú
⎪ − cos ⎜⎝ 1+ x 2 ⎟⎠ , if x ≤ 0 ê 2æp xö ú
⎪ 2 cos ç - ÷
2 tan−1 x = ⎨ êë è4 2ø úû
⎪ − cos −1 ⎛ 1− x ⎞ , if x ≥ 0
2
⎪⎩ ⎜
⎝ 1+ x 2 ⎟⎠
é æ p x öù p x
= tan-1 ê tan ç - ÷ ú = -
⎛p p ⎞ ë è 4 2 øû 4 2
Proof: Let tan−1 x = A where x ∈ R and A ∈⎜ , ⎟ . Then
⎝ 2 2⎠
x = tan A Illustration 17.22 Find the angle
1− tan2 A 1− x 2
cos 2 A = = æ 3p ö
1+ tan2 A 1+ x 2 (A) tan-1 ç tan ÷ (B) sin−1 sin 5 (where 5 is in radians).
where 2A ∈ (−π,π) è 4 ø
Solution:
Case (a): If 2 A ∈( −p , 0 ), then
⎛ 3p ⎞
⎛ p ⎞ (A) Let tan−1 ⎜ tan ⎟ = q
A ∈⎜ − , 0⎟ ⇒ x ∈( −∞ , 0 ) ⎝ 4 ⎠
⎝ 2 ⎠
⎛ p⎞
1− x 2 tan−1 tan ⎜ p − ⎟ = q
cos2 A = ⎝ 4⎠
1+ x 2
⎛ p⎞
tan−1 ⎜ − tan ⎟ = q
æ 1- x 2 ö ⎝ 4⎠
Þ cos -1 éëcos ( -2 A ) ùû = cos -1 ç ÷
è 1+ x 2 ø p é æ p p öù
⇒ - tan-1 tan = q êAs tan-1 tanq = q , if q Î ç - , ÷ ú
æ 1- x ö 2 4 ë è 2 2 øû
Þ -2 A = cos -1 ç ÷
è 1+ x 2 ø p
⇒ - =q
æ 1- x ö 2 4
Þ -2 A = - cos -1 ç ÷
è 1+ x 2 ø (B) We know
æ 1- x 2 ö é p pù
Þ 2 tan-1 x = - cos -1 ç ÷ sin−1 sin θ = θ, θ ∈ ê - , ú » [ -1.57,1.57] (1)
è 1+ x 2 ø ë 2 2û
Hence, sin−1 sin 5 ≠ 5 as 5 ∉ [−1.57, 1.57].
Case (b): If 2 A ∈[0 , p ], then
Therefore,
⎡ p⎞ sin 5 = sin (p + 5 − p )
A ∈ ⎢0 , ⎟ ⇒ x ∈[0 , ∞ )
⎣ 2⎠ = − sin (5 − π)
1- x 2 Since (5 − π) ∉ [−1.57, 1.57], so we again add and subtract π.
cos2 A =
1+ x 2
⇒ sin 5 = − sin (p + 5 - 2p )
æ 1- x 2 ö
Þ cos -1[cos(2 A)] = cos -1 ç ÷ = + sin (5 − 2π) [Since (5 − 2π) ∈ [− 1.57, 1.57]]
è 1+ x 2 ø
Hence,
æ 1- x 2 ö
Þ 2 A = cos -1 ç ÷ sin−1 sin 5 = sin−1 sin (5 − 2π) = 5 − 2π
è 1+ x 2 ø
Note: To solve this type of problem, the procedure is to add
æ 1- x 2 ö and subtract π till it belongs to the principal value range of
Þ 2 tan x = cos ç
-1 -1
÷
è 1+ x 2 ø respective inverse trigonometric function.
714 Mathematics Problem Book for JEE
Solution: We know,
Your Turn 3
p
cos -1 x = - sin-1 x
1. If 2 tan−1(cos x) = tan−1(cosec2 x), then x = 2
π Given,
(A) (B) π
2 p
π π cos −1x − cos −1 y =
(C) (D) Ans. (D) 3
6 3
p p p
1 1 1 ⇒ - sin-1 x - + sin-1 y =
2. 4 tan−1 − tan−1 + tan−1 = 2 2 3
5 70 99
π π p
(A) (B) sin-1 y - sin-1 x =
2 3 3
2p
π sin-1 y + sin-1 x = (1)
(C) (D) None of these Ans. (C) 3
4
1 1 2sin−1 y = π
3. 4 tan−1 − tan−1 is equal to
5 239 p
π sin-1 y = (2)
2
(A) π (B) 2
⇒y=1
π π
(C) (D) Ans. (D) Put Eq. (2) in Eq. (1).
3 4
a3 æ1 a ö b3 æ1 bö p 2p
4. cosec2 ç tan-1 ÷ + sec2 ç tan-1 ÷ is equal to + sin-1 x =
2 è2 bø 2 è2 aø 2 3
Thus, only at
1
2 { }
, 1 above expression has a solution 2t 2 - p t -
3p 2
8
=0
Solution: See Fig. 17.10. Hence, the correct answer is option (C).
y
p Previous Years' Solved JEE Main/AIEEE
−1
cos x Questions
p /2
æxö æ5ö p
sin−1x 1. If sin-1 ç ÷ + cosec -1 ç ÷ = , then a value of x is
è5ø è4ø 2
x
(1/ 2,0) (1, 0) (A) 1 (B) 3
(C) 4 (D) 5 [AIEEE 2007]
−p /2
Solution: We have
x 4 p x 4 x 3
sin-1 + sin-1 = Þ sin-1 = cos -1 Þ sin-1 = sin-1
Figure 17.10 5 5 2 5 5 5 5
Therefore, x = 3.
-1 -p p
0 £ cos x £ p ; £ sin-1 x £ Hence, the correct answer is option (B).
2 2
æ 5 2ö
⎛ 1 ⎤ 2. The value of cot ç cosec -1 + tan-1 ÷ is
Clearly, sin−1 x > cos −1 x ∀x ∈⎜ ,1 . è 3ø
⎝ 2 ⎥⎦
3
6 3
Hence, the correct answer is option (C). (A) (B)
17 17
æ -1 ö æ 1ö
9. The value of tan-1 + cos -1 ç ÷ + sin-1 ç - ÷ is 4 5
è 2ø è 2ø (C) (D) [AIEEE 2008]
17 17
p 5p
(A) (B)
4 12 æ 5 2ö
Solution: Let us consider that, E = cot ç cosec -1 + tan-1 ÷ .
3p 11p è 3 3ø
(C) (D)
4 12 Therefore,
é æ 3 2 öù
Solution: é -1 æ 3 ö -1 æ 2 ö ù
ê -1 ç 4 + 3 ÷ ú
E = cot ê tan ç ÷ + tan ç ÷ ú = cot ê tan ç ÷ú
-p p -p p ë è4ø è 3 øû ê çç 1- 3 × 2 ÷÷ ú
< tan-1 x < ; £ sin-1 x £ ; 0 £ cos -1 x £ p
2 2 2 2 êë è 4 3 ø úû
p æ 17 ö 6
tan-1(1) = (1) = cot ç tan-1 ÷ =
4 è 6 ø 17
æ 1 ö 2p Hence, the correct answer is option (A).
cos -1 ç - ÷ = (2)
è 2ø 3
3. Statement I: The equation (sin−1x)3 +(cos−1x)3 − aπ 3 = 0 has a
æ 1 ö -p
-1 1
sin ç - ÷ = (3) solution for all a ³ .
è 2ø 6 32
Thus, Eqs. (1) + (2) + (3) gives p
Statement II: For any x ∈ R, sin-1 x + cos -1 x = and
2
⎛ 1⎞ ⎛ 1⎞ p 2p p
tan−1 (1) + cos −1 ⎜ − ⎟ + sin−1 ⎜ − ⎟ = + - æ
2
p ö 9p 2
⎝ 2⎠ ⎝ 2⎠ 4 3 6 0 £ ç sin-1 x - ÷ £ .
è 4ø 16
3p + 8p - 2p 9p 3p
= = = (A) Both statements I and II are true.
12 12 4
(B) Both statements I and II are false.
Alternative Solution: Since (C) Statement I is true and statement II is false.
p (D) Statement I is false and statement II is true.
sin-1 x + cos -1 x =
2 [JEE MAIN 2014 (ONLINE SET-3)]
Chapter 17 | Inverse Trigonometry 717
Solution: é æ -p ö ù -p
Therefore, tan-1 ê tan ç ÷ú = .
(sin−1 x)3 + (cos−1 x)3 − aπ 3 =0 ë è 4 øû 4
Þ (sin-1 x + cos -1 x )[(sin-1 x )2 - sin-1 x cos -1 x + (cos -1 x )2 ] = ap 3 Hence, the correct answer is option (C).
p
Þ [(sin-1 x + cos -1 x )2 - 3 sin-1 x cos -1 x ] = a p 3p 2 æ 2x ö 1
2 5. Let tan-1 y = tan-1 x + tan-1 ç 2 ÷
, where x < . Then a
è 1- x ø 3
2
æp ö æp ö value of y is
Þ ç ÷ - 3 sin-1 x ç - sin-1 x ÷ = 2ap 2
è2ø è2 ø 3x + x 3 3x - x 3
(A) (B)
p2 1- 3 x 2 1+ 3 x 2
- 2ap 2
4 p
Þ = (sin-1 x ) - (sin-1 x )2 3x + x 3 3x - x 3
3 2 (C) (D)
1+ 3 x 2 1- 3 x 2
p 2 - 8ap 2 p
Þ = (sin-1 x ) - (sin-1x)2
12 2 [JEE MAIN 2015 (OFFLINE)]
2 2
p -1 æ p ö æ p ö p - 8ap
2 2 Solution: Since,
Þ (sin-1 x )2 - sin x + ç ÷ = ç ÷ -
2 è4ø è4ø 12
æ x+y ö
æ
2
p ö p 2 p 2 8 ap 2 32ap 2 - p 2
2
tan-1 x + tan-1 y = tan-1 ç ÷ for xy < 1
Þ ç sin-1 x - ÷ = - + = è 1- xy ø
è 4 ø 16 12 12 3 48
Now,
Now æ 2x ö 2x
2
æ 1- x 2 - 1 ö 2
x ×ç 2 ÷
= = -2 ç 2 ÷
= -2 +
è 1- x ø 1- x è 1- x ø 1- x 2
2
p p -p p p p p
- £ sin-1 x £ Þ - £ sin-1 x - £ -
2 2 2 4 4 2 4 Further
1 1 1 2
-3p p p x < Þ 0 £ x 2 < Þ - < - x 2 £ 0 Þ < 1- x 2 £ 1
or £ sin-1 x - £ 3 3 3 3
4 4 4
1 3 2 2
æ p ö 9p 2
2
Þ 1£ < Þ2£ < 3 Þ 0 £ -2 + <1
Þ 0 £ ç sin-1 x - ÷ £ 1- x 2 2 1- x 2 1- x 2
è 4ø 16
æ 2x ö
Therefore, Þ x ×ç 2 ÷
Î (0 ,1)
è 1- x ø
32ap 2 - p 2 9p 2 32a - 1 9 Hence,
0£ £ Þ0£ £
48 16 48 16 æ 2x ö
æ 2x ö çx+ ÷
⇒ 0 ≤ 32a − 1 ≤ 27 ⇒ 1 £ 32a £ 28
-1 -1
tan ( x ) + tan ç -1
= tan ç 1 - x2 ÷
2 ÷
è 1- x ø
2
ç 1- 2 x ÷
ç ÷
1 28 é 1 7ù è 1- x 2 ø
Þ £a£ Þ aÎ ê , ú
32 32 ë 32 8 û
æ 3x - x3 ö
= tan-1 ç 2 ÷
= tan-1( y ) (given)
Therefore, Statement I is false and II is true. è 1- 3 x ø
Hence, the correct answer is option (D).
æ 3x - x 3 ö
æ 43p ö Þ y =ç 2 ÷
4. The principal value of tan−1 ç cot ÷ is è 1- 3 x ø
è 4 ø
Hence, the correct answer is option (D).
3p 3p
(A) - (B)
4 4 æ 2x ö
6. If f ( x ) = 2 tan-1 x + sin-1 ç 2 ÷
, x > 1, then f(5) is equal to
p p è 1+ x ø
(C) - (D)
4 4 p
(A) (B) π
2
[JEE MAIN 2014 (ONLINE SET-4)] æ 65 ö
Solution: (C) 4 tan−1(5) (D) tan-1 ç ÷
è 156 ø
43p æ 44p p ö æ pö æ pö [JEE MAIN 2015 (ONLINE SET-1)]
cot = cot ç - ÷ = cot ç 11p - ÷ = cot ç p - ÷
4 è 4 4ø è 4ø è 4ø
Solution:
3p æp p ö p æ -p ö æ 2x ö
= cot = cot ç + ÷ = - tan = tan ç ÷ f ( x ) = 2 tan-1 x + sin-1 ç , x > 1, f(5) = ?
2 ÷
4 è 2 4 ø 4 è 4 ø è 1+ x ø
718 Mathematics Problem Book for JEE
æ [cos( x / 2) + sin( x / 2)]2 ö (B) If a = 1 and b = 1, then (x, y) (Q) lies on (x2 − 1)(y2 − 1) = 0
-1
f ( x ) = tan ç ÷
ç [cos( x / 2) - sin( x / 2)]2 ÷ (C) If a = 1 and b = 2, then (x, y) (R) lies on y = x
è ø
æ | cos( x / 2) + sin( x / 2)| ö (D) If a = 2 and b = 2, then (x, y) (S) lies on (4x2 − 1)(y2 − 1) = 0
æ pö
f ( x ) = tan-1 ç ÷ , x Î ç 0,, ÷
è | cos( x / 2) - sin( x / 2)| ø è 2ø [IIT-JEE 2007]
é cos( x / 2) + sin( x / 2) ù Solution: If a = 1 and b = 0, then
f ( x ) = tan-1 ê ú
ë cos( x / 2) - sin( x / 2) û p p
sin-1 x + cos -1 y + =
é1+ tan( x / 2) ù 2 2
f ( x ) = tan-1 ê ú
ë 1- tan( x / 2) û Þ sin-1 x = - cos -1 y
é æ p x öù Þ sin-1 x = - sin-1 1- y 2
f ( x ) = tan-1 ê tan ç + ÷ ú
ë è 4 2 øû Þ - x = 1- y 2
p x
f (x) = + Þ x2 + y2 =1
4 2
(A) → (P)
æ p ö p x 3p + p p
fç ÷= + = = If a = 1 and b = 1, then
è 6 ø 4 12 12 3
æp x ö p
The point is ç , ÷ , Therefore, sin-1 x + cos -1 y + cos -1 xy =
2
è6 3ø
cos -1 x - cos -1 y = cos -1 xy
1
f ’(x) =
2 Þ xy + 1- y 2 1- x 2 = xy
⎛p ⎞ 1 Þ ( x 2 - 1)( y 2 - 1) = 0
⇒ f’ ⎜ ⎟ =
⎝ 6⎠ 2 (B) → (Q)
Chapter 17 | Inverse Trigonometry 719
d2 y dy 6x 3y
Þ (1- x 2 ) -x = -9 y - =3 (1)
dx 2 dx cosq sinq
d2 y dy Since,
Þ ( x 2 - 1) +x = 9y
dx 2 dx
ax by
Therefore, - = a2 - b 2
cosq sinq
(P)→(4)
For (Q) in List I: Therefore, slope of this normal is
æ 6 ö æ- 3ö 6 sinq
n−1
= çç - ÷÷ çç ÷÷ = = 2 tanq
∑ (a
k =1
k × ak +1 ) = a1 × a2 + a2 × a3 + + an−1 × an
è cos q ø è sinq ø 3 cosq
n
So, Eq. (2) becomes
2p
= (n - 1)a2 sin (1)
n 6h 3
− =3
(See Fig. 17.11.) Since all a1 are equal 2 1
3 3
n−1
2p
Also ∑ (a
k =1
k ⋅ ak +1 ) = ( a + a + a )cos
2 2 2
n ⎛ 3
6 ×3⎞
Figure 17.13
⇒⎜ ⎟h=6
2p ⎜⎝ 2 ⎟⎠
= (n - 1)a2 cos (2)
n
From Eqs. (1) and (2), Therefore,
2p 2p h=2
sin = cos Hence,
n n
2p 2p p (R) → (2)
⇒ tan = 1⇒ = ⇒n=8
n n 4 For (S) in List I:
Therefore,
(Q) → (3) æ 1 ö -1 æ 1 ö -1 æ 2 ö
tan-1 ç ÷ + tan ç ÷ = tan ç 2 ÷
For (R) in List I: è 2 x + 1ø è 4 x + 1ø èx ø
(See Fig. 17.12.) Equation of normal at ( 6 cosq , 3 sinq ) is ì 1 1 ü
ïï 2 x + 1 + 4 x + 1 ïï
-1 -1 æ 2 ö
y Þ tan í
1 ý = tan ç 2 ÷
ï 1- ï èx ø
P(h,1) ïî (2 x + 1)( 4 x + 1) ïþ
ì 4 x + 1+ 2 x + 1 ü
ïï ïï æ 2 ö
Þ tan í 8 2x + 6 x + 1 ý = tan-1 ç 2 ÷
x -1
2
ï 8 x + 6 x + 1 - 1 ï èx ø
ïî 8 x 2 + 6 x + 1 ïþ
æ 6x + 2 ö -1 2
Þ tan-1 ç 2 ÷ = tan 2
Figure 17.12 è 8x + 6x ø x
Chapter 17 | Inverse Trigonometry 721
18. If tan-1
1+ x 2 - 1- x 2
= b , then x =
28. If å sin
i =1
-1
x i = np , then åx
i =1
i is
1+ x + 1- x
2 2
(A) n (B) 2n
1- tan b n(n +1)
(A) tan β (B) ± (C) (D) None of these
1+ tan b 2
(C) ± sin2 b (D) ± cos2 b 29. The value of sin-1{sin[2 cot + -1 ( 2 - 1)]} is
x -1 x +1 p p 3p
19. If tan-1 + tan-1 = , then x = (A) - (B)
x -2 x +2 4 4 4
1 7p
(A) 3 (B) (C) (D) None of these
3 4
1
(C) ± (D) None of these -1
2 ìï é p 1 æ a öù ép 1 æ a ö ù üï
30. The value of ítan ê + sin-1 ç ÷ ú + tan ê - sin-1 ç ÷ ú ý ,
20. Let f(x) = sec−1 x + tan−1 x. Then f(x) is real for îï ë 4 2 è b øû ë4 2 è b ø û þï
(A) x ∈ [−1, 1] (B) x ∈ R where 0 < a < b is
(C) x ∈ (−∞, 0) (D) (−∞, −1] ∪ [1, ∞) b a
(A) (B)
21. If cos−1 x − sin−1 x = 0, then x is equal to 2a 2a
1
(A) ± (B) 1 b 2 - a2 b 2 - a2
2 (C) (D)
2b 2a
1
(C) (D) 2 31. If 4 cos−1 x + sin−1 x = π, then x equals to
2
1 1
10 10 (A) (B)
22. If å cos
i =1
-1
x i = 0 , then åx i=1
i is 2 2
(A) 0 (B) 10 3
(C) 1 (D)
(C) 5 (D) None of these 2
Chapter 17 | Inverse Trigonometry 723
æ sin1- 1 ö ìï é æ ìï é æ 3p ö ù üï ö ù üï
32. tan-1 ç ÷ equals 42. Value of sin-1 ísin êcos -1 ç cos ísin-1 êsin ç
ç ÷ ú ý ÷ ú ý equals
è cos 1 ø îï êë è ïî ë è 4 ø û ïþ ÷ø úû þï
p
(A) 0 (B) 1- 3p 5p
2 (A) (B)
4 4
p 1 p
(C) -1 (D) - p
2 2 4 (C) (D) None of these
4
2x
33. If x ≥ 1, then 2 tan-1 x + sin-1 is equal to 43. tan−1(tan4) + cot−1(cot4) equals
1+ x 2
(A) 8 (B) π − 8
(A) 4tan−1x (B) π (C) 8 − 2π (D) 0
(C) 0 (D) None of these
34. The inequality log2 x < sin−1(sin5) hold if p
44. If tan-1( x ) + tan-1 y + tan-1 z = and
2
(A) x ∈ (0, 25−2π) (B) x ∈ (25−2π, ∞)
(x − y)2 + (y − z)2 + (z − x)2 = 0, then x 2 + y 2 + z 2 equals
(C) x ∈ (22π − 5, ∞) (D) None of these
(A) 0 (B) 4
35. If a £ tan-1 x + cot -1 x + sin-1 x £ b "x Î (0 ,1), then (C) 1 (D) None of these
p 45. The greatest of tan1, tan−11, sin1, sin−11 is
(A) a = 0 , b = (B) α = 0, β = π
2 (A) tan1 (B) tan−11
p (C) sin1 (D) sin−1 1
(C) a = -1, b = p (D) a = ,b =p
2 ænö p
46. If cot -1 ç ÷ > , n being a natural number, then maximum
9p 1 1 1 èp ø 6
36. If 2 sin-1 x + 3 sin-1 y + 4 sin-1 z = then + + equals
2 2x 3y 4z value of n is
11 13 (A) 1 (B) 5
(A) (B) (C) 9 (D) None of these
12 12
47. Two angles of a triangle are cot −12 and cot −13. Then the third
15 17
(C) (D) angle is
12 12
p 3p
(A) (B)
é 4 4
æ4ö æ 2 öù
37. tan-1 êcos -1 ç ÷ + sin-1 ç ÷ ú is
ë è 5 ø è 3 øû p p
(C) (D)
17 17 6 3
(A) (B)
6 16 2p ö 2p ö
æ -1 æ
6 48. The value of cos -1 ç cos ÷ - sin ç sin ÷ is equal to
(C) (D) None of these è 3 ø è 3 ø
17
(A) 0 (B) 4p
38. If sin-1 x + sin-1 y + sin-1 z = p , then 3
x 1- x 2 + y 1- y 2 + z 1- z 2 equals 4p p
(C) − 3 (D) 3
(A) xyz (B) 1
(C) 0 (D) 2xyz
49. If in Δ ABC, ∠A = sin−1(x), ∠B = sin−1(y) and ∠C = sin−1(z), then
1- sin x + 1+ sin x x 1- y 2 1- z 2 + y 1- x 2 1- z 2 + z 1- x 2 1- y 2 is equal to
39. The value of cot - 1 if 0° < x < 90° is
1- sin x - 1+ sin x
(A) xyz (B) x + y + z
x
(A) x (B) 1 1 1
2 (C) + + (D) None of these
x y z
x x
(C) p - (D) -
2 2 æ x 3 ö -1 æ 2 x - k ö
50. If A = tan-1 çç ÷÷ and B = tan ç ÷ , then the value of
40. The value of tan2(sec−1 2) + cot2(cosec−1 3) is è 2 k - x ø è k 3 ø
(A) 13 (B) 15 A − B (independent of x) is
(C) 12 (D) 11 p p
(A) (B)
41. The principal value of sin−1(sin10) is 2 3
(A) 10 (B) 10 − 3π p p
(C) (D)
(C) 3π − 10 (D) None of these 6 8
724 Mathematics Problem Book for JEE
Practice Exercise 2 1
(C) −π − 2sin−1(x) for -1 £ x £ -
2
Single/Multiple Correct Choice Type Questions 1
(D) 2cos−1(x) for ë xÅ 1
1. If y = cot−12 + cot−18 + cot−118 + … + ∞, then tan y is 2
p p
(A) (B)
2 3 Matrix Match Type Questions
p
(C) (D) None of these 8. Match the following:
4
æ x ö æ y ö Column I Column II
2. If x, y > 0, then the range of sin-1 ç 2 ÷
+ sin-1 ç 2 ÷
is
è 1+ x ø è 1+ y ø p
⎛ 2 cos x − 3 sin x ⎞
(A) If y = cos −1 ⎜ ⎟⎠ , then
(i)
(A) [0, π] (B) (−π, π] ⎝ 13 14
é 2p ù æ 2p ù dy
(C) ê0 , ú (D) ç 0 , ú is
ë 3 û è 3 û dx
Answer Key
Practice Exercise 1
1. (A) 2. (D) 3. (A) 4. (B) 5. (A)
6. (C) 7. (C) 8. (C) 9. (C) 10. (D)
11. (B) 12. (D) 13. (D) 14. (C) 15. (B)
16. (D) 17. (A) 18. (C) 19. (C) 20. (D)
21. (C) 22. (B) 23. (B) 24. (A) 25. (A)
26. (A) 27. (C) 28. (B) 29. (D) 30. (C)
31. (D) 32. (D) 33. (B) 34. (A) 35. (B)
36. (B) 37. (A) 38. (D) 39. (C) 40. (D)
41. (C) 42. (C) 43. (C) 44. (C) 45. (D)
46. (B) 47. (B) 48. (D) 49. (A) 50. (C)
Practice Exercise 2
Solutions
Practice Exercise 1 = tan[tan-1(1) + tan-1( -1)]
ép p ù
= tan ê - ú = tan (0) = 0
-1 -1 æ x+y ö -1 ë4 4û
1. Hint: tan x + tan y = tan ç ÷
è 1- xy ø 2. Since 0 ≤ cos−1(x) ≤ π and given
é æ 2 + 3 öù cos−1(cos x) = π + x
tan ê tan-1(1) + tan-1 ç ÷ú
ë è 1- 6 ø û is true only when x belongs to some negative angle but no
option is such.
é æ 5 öù 11p
= tan ê tan-1(1) + tan-1 ç ÷ ú 3. Since 5p < 16 < ,
ë è -5 ø û 2
726 Mathematics Problem Book for JEE
p p
Þ 0 < 16 - 5p < 8. Hint: cos -1 x = - sin-1 x
2 2
p p
Þ- < 5p - 16 < 0 sin-1 x + sin-1(1- x ) = - sin-1 x
2 2
Now, p
2 sin-1 x + sin-1(1- x ) =
sin−1(sin16) = sin−1[sin(5π − 16)] 2
= 5π − 16 Clearly x = 0 satisfies equation
4. Given p p
0+ =
θ = cot−1 7 + cot−18 + cot−118 2 2
55
= cot -1 + cot -1 18 x=
1
also satisfies the equation
15 2
æ 11 ö p p p
ç ´ 18 - 1 ÷ 2´ + =
= cot ç 3
-1
÷ 6 6 2
çç 11 + 8 ÷÷
è 3 ø æ 1ö
Þ ç x = 0, ÷
è 2ø
= cot−1 (3)
⇒ cot θ = 3 9. Hint: 2 sin-1 x = sin-1(2 x 1- x 2 )
5. We have
é æ 1 öù æ 7 ö sin-1 x = sin-1(2a 1- a2 ) when
cos ê2 tan-1 ç ÷ ú = cos ç tan-1 ÷ −1 ≤ x ≤ 1
ë è 7 øû è 24 ø
æ 24 ö 24 -1 £ 2a 1- a2 £ 1
= cos ç cos -1 ÷ =
è 25 ø 25 0 £ 4 a2 (1- a2 ) £ 1
æ 1ö 1
sin( 4 cot -1 3) = sin ç 4 tan-1 ÷ 0 £ a2 (1- a2 ) £
è 3ø 4
æ 3ö æ 24 ö 1
= sin ç 2 tan-1 ÷ = sin ç tan-1 ÷ a2 (1- a2 ) £
è 4ø è 7 ø 4
æ 24 ö 24 1
= sin ç sin-1 ÷ = Þ a2 £
2
è 25 ø 25
1 1
æ 1ö Þ- £a£+
Hence, cos ç 2 tan-1 ÷ = sin( 4 cot -1 3). 2 2
è 7ø
10. Hint: cos−1[cos(−x)] = π − x
6. See Fig. 17.14.
æ pö ïì 1 é æ p ö æ p ö ù ïü
tan-1 ç 2 sin ÷ cos -1 í êcos ç p - 10 ÷ - sin ç p - 10 ÷ ú ý
è 3ø 2 îï 2 ë è ø è ø û þï
3
æ
-1 3ö -1 é 1 æ p p öù
tan çç 2 ´ ÷÷ = tan ( 3 ) = cos -1 ê ç - cos - sin ÷ ú
è 2 ø ë 2è 10 10 ø û
/3
p é æ 1 p 1 p öù
= 1 = cos -1 ê( -1) ç cos + sin ÷ ú
3 ë è 2 10 2 10 ø û
Figure 17.14
é æ p p p p öù
= cos -1 ê( -1) ç cos cos + sin × sin ÷ ú
ë è 4 10 4 10 ø û
7. Hint: ( 5 )2 + ( 11)2 = 4 2
é æ p p öù
See Fig. 17.15. = cos -1 ê( -1)cos ç - ÷ ú
ë è 4 10 ø û
sin [α + α] = sin 2α
= 2 sin α cos α æ 3p ö 3p 17p
= p - cos -1 ç cos ÷ = p - =
è 20 ø 20 20
5 11 55
= 2⋅ ⋅ = Figure 17.15 Hence, none of the given alternatives are correct.
4 4 8
Chapter 17 | Inverse Trigonometry 727
x+y æ 1+ 2 ö
11. Hint: tan-1 x + tan-1 y = tan-1 = -p + cos -1 çç ÷÷
1- xy
è 1- 2 ø
æ y ö æ 3 ö
tan-1 x + cos -1 ç ÷ = sin-1 ç ÷ 15. Hint: cos (2π − θ) = cos θ
ç 1+ y 2 ÷ è 10 ø
è ø
é æ 17p ö ù
cos -1 êcos ç - ÷ú
Þ tan-1 x + tan-1 y = tan-1 3 ë è 5 øû
x+y
Þ tan-1 = tan-1 3 ⎡ ⎛ 3p ⎞ ⎤ ⎛ 3p ⎞ 3p
1- xy = cos −1 ⎢cos ⎜ 4p − ⎟ ⎥ = cos −1 ⎜ cos ⎟ =
⎣ ⎝ 5 ⎠⎦ ⎝ 5 ⎠ 5
x+y
Þ =3
1- xy p
16. Hint: sin-1 x + cos -1 x =
⇒ x + y = 3 − 3xy 2
⇒ x + y + 3xy = 3 θ = sin−1 x + cos−1x − tan−1x
Hence, only integral solution possible is (3, 0) and (0, 3). p
= - tan-1 x
2
1- tan2 q 2 tanq
12. Hint: cos 2q = , sin 2q = Given,
1+ tan2 q 1+ tan2 q
p
0 ≤ x ≤ 1 Þ 0 £ tan-1 x £
1
-1 1 1 1 4
A = tan and B = tan-1 Þ tan A = and tan B =
7 3 7 3 p p p
Þ ³ - tan-1 x ³
1 2 2 4
1-
1- tan2 A 49 48 24
cos 2 A = = = =
1+ tan2 A 1+ 1 50 25 a+b
17. Hint: tan-1 a + tan-1 b = tan-1
49 1- ab
1 2 1 4
2´ tan-1 x = 2 tan-1 + tan-1
2 tan B 3 6 3
sin 2B = = = 3 = = 7 3
1+ tan2 B 1+ 1 10 10 5
æ 49 - 1 ö -1 4
9 9 tan-1 x = cot -1 ç ÷ + tan
3 4 24 è 14 ø 3
sin 4 B = 2 sin 2B × cos 2B = 2 × × =
5 5 25 æ 48 ö 4
= cot -1 ç ÷ + tan-1
Hence, cos2A = sin4B. è ø
14 3
7 4
13. Let x = cos A , x Î [ -1,1] and A Î [0 , p ] . Then 7 -1 4 -1 24
+
= tan-1
+ tan = tan 3
24 3 7 4
2 cos -1 ( cos A ) = sin-1(2 cos A 1- cos2 A ) 1- ×
24 3
⇒ 2 cos −1 (cos A) = sin−1(2 cos A . sin A) = sin−1(sin2 A) 7 4 39
+
24
-1 3 -1 24 117
Þ 2 A = sin-1(sin 2 A) = tan = tan = tan-1
7 11 44
1-
Now, left hand and right hand will be equal for 18 18
é pù é pù é 1 ù 117
2 A Î [0 , 2p ] Þ 2 A Î ê0 , ú Þ A Î ê0 , ú Þ x Î ê ,1ú Þ tan-1 x = tan-1
ë 2û ë 4û ë 2 û 44
åx i = 10
1
i =1
Þ x2 + x +1 = Þ x 2 + x + 1= 1
23. We have x2 + x +1
é ⇒ x(x + 1) = 0, x = 0, x = −1
æ 4ö æ 16 ö ù 7
cos ç 2 cos -1 ÷ = cos êcos -1 ç 2 ´ - 1÷ ú = Both the values satisfies the equation, so there are two
è 5ø ë è 25 ø û 25
solutions.
24. Hint: tan (2μπ − θ) = −tan θ, cot(−θ) = cos θ
28. Given
ïì é æ p ö ù ïü ìï é æ p ö ù ïü
cos ítan-1 ê tan ç 4p - ÷ ú ý = cos ítan-1 ê - tan ç ÷ ú ý sin−1 x1 + sin−1 x2 + … + sin−1 x2n = nπ
îï ë è 4 ø û þï îï ë è 4 ø û þï
which is possible only if
Chapter 17 | Inverse Trigonometry 729
p é 1ù
sin-1 x1 = sin-1 x 2 = = sin-1 x 2 n =
2 ê 1- tan 2 ú
-1
= - tan ê ú
⇒ x1 = x2 = … = x2n = 1 ê1+ tan 1 ú
ë 2û
Therefore, x1 + x2 + … + x2n = 2n.
é p 1 ù
29. We have ê tan 4 - tan 2 ú
-1 -1 é æ p 1 öù
= - tan ê ú = - tan ê tan ç - ÷ ú
sin-1{sin[2 cot -1( 2 - 1)]} = sin−1{sin[cot−1(−1)]} p
ê1+ tan × tan ú1 ë è 4 2 øû
ë 4 2û
æ 3p ö
= sin-1 ç sin ÷ æp 1ö 1 p
è 4 ø = -ç - ÷ = -
è 4 2ø 2 4
é æ p öù p
= sin-1 êsin ç p - ÷ ú = 2x
ë è 4 øû 4 33. Hint: 2 tan-1 x = p - sin-1 , when x ≥ 1
1+ x 2
-1
é æp ö æp öù 1 æaö With the given condition
30. We have ê tan ç + a ÷ + tan ç - a ÷ ú , where a = sin-1 ç ÷ 2x
ë è4 ø è4 øû 2 èbø 2 tan-1 x = p - sin-1
1+ x 2
-1
æ 1+ tana 1- tana ö Hence,
=ç + ÷
è 1- tana 1+ tana ø 2x 2x 2x
-1 2 tan-1 x + sin-1 = p - sin-1 + sin-1 =p
é 2(1+ tan2 a ) ù 1+ x 2 1+ x 2 1+ x 2
=ê ú
ë 1- tan a û
2
34. Hint: sin−1(sin5) = sin−1[sin(−2π + 5)]
-1
æ 2 ö 2 cos 2a log2 x < sin−1(sin5)
=ç ÷ =
è cos 2a ø 2 ⇒ log2 x < sin−1[sin(−2π + 5)]
⇒ log2 x < −2π + 5 = 5 − 2π
1 a
Now, a = sin-1 ⇒ x < 25−2π
2 b
Also, x ≠ 0 is positive.
a
Þ = sin2a Therefore, required value of x belongs to x ∈(0 , 25−2p ).
b
b 2 - a2 æ p öü
a2 35. Hint: tan-1 x Î ç 0 , ÷ ï
Þ cos2a = 1- 2
= è 4øï
b b
æ p p öï
b 2 - a2 cot -1 x Î ç , ÷ ý "x Î (0 ,1)
Therefore, given expression is . è 2 4 øï
2b
æ pöï
sin-1 x Î ç 0 , ÷ ï
p è 2 ø ïþ
31. Hint: sin-1 x + cos -1 x =
2
Let tan-1 x + cot -1 x + sin-1 x = z .
p
4 cos -1 x + - cos -1 x = p p p
2 At x = 0 , z = but x ≠ 0, thus a ¹ .
2 2
p p At x = 1,
Þ 3 cos -1 x = Þ cos -1 x =
2 6 z=π⇒β=π
p 3 From option, it is clear that z ≥ 0
Þ x = cos =
6 2 α = 0, β = π
2 -p p
æ A Aö 36. Hint: £ sin-1 p £
32. Hint: 1- sin A = ç sin - cos ÷ 2 2
è 2 2ø
æp ö
æ sin1- 1 ö -1 æ 1- sin1 ö -1 æ 1- sin1 ö 2(sin-1 x ) + 3(sin-1 y ) + 4(sin-1 z ) = 9 × ç ÷
tan-1 ç ÷ = tan ç - ÷ = - tan ç ÷ è2ø
è cos1 ø è cos1 ø è cos1 ø
p
⇒ sin−1 x = sin−1 y = sin−1 z = −
éæ 1 1ö ù
2
é 1 1ù 2
ê ç cos - sin ÷ ú ê cos 2 - sin 2 ú
-1 ê è 2 2ø ú -1 ⇒x=y=z=1
= tan = - tan ê ú
êæ 2 1 2 1öú ê cos 1 + sin 1 ú 1 1 1 1 1 1 13
ê ç cos - sin ÷ ú ë 2û Þ + + = + + =
ëè 2 2 øû 2 2 x 3 y 4 y 2 3 4 12
730 Mathematics Problem Book for JEE
a+b p
37. Hint: tan-1 a + tan-1 b = tan-1 Þ- < 3p - 10 < 0
1- ab 2
Given equation must be as follows: Now,
ì æ4ö æ 2 öü sin-1(sin10 ) = sin-1[sin(3p - 10 )]
tan-1 ícot -1 ç ÷ + sin-1 ç ÷ý
î è3ø è 3 øþ = 3π − 10
⎧ ⎛ 3⎞ ⎛ 2⎞ ⎫ 42. We have
= tan ⎨tan−1 ⎜ ⎟ + tan−1 ⎜ ⎟ ⎬
⎩ ⎝ 4 ⎠ ⎝ 3⎠ ⎭ ìï é æ ì é æ 3p ö ù ü ö ù üï
sin-1 ísin êcos -1 ç cos ísin-1 êsin ç ÷ú ý ÷ú ý
⎧ ⎛ 3 2 ⎞⎫ ç ë è 4 ø û þ ÷ø úû þï
îï êë è î
⎪⎪ −1 ⎜ 4 + 3 ⎟ ⎪⎪ ⎧ −1 ⎛ 17 ⎞ ⎫ 17
= tan ⎨tan ⎜ ⎟ ⎬ = tan ⎨tan ⎜⎝ 6 ⎟⎠ ⎬ = 6 ì é p öù ü
3 2 ⎩ ⎭ æ
⎪ ⎜ 1− × ⎟ ⎪ = sin-1 ísin êcos -1 ç cos ÷ ú ý
⎪⎩ ⎝ 4 3 ⎠ ⎪⎭ î ë è 4 øû þ
38. Given æ pö p
= sin-1 ç sin ÷ =
sin-1 x + sin-1 y + sin-1 z = p è 4ø 4
43. Since
Let sin-1 x = a ,sin-1 y = b ,sin-1 z = y . Then 3p
p <4<
α+β+γ=π 2
Now, p
0 < 4 -p <
2
x 1- x 2 + y 1- y 2 + z 1- z 2
Now,
1
= (sin 2a + sin 2 b + sin 2g ) tan-1(tan 4 ) + cot -1(cot 4 )
2
= tan-1[tan(p + 4 - p )] + cot -1[cot(p + 4 - p )]
1
= ( 4 sina sin b sin g )
2 = tan-1[tan( 4 - p )] + cot -1[cot( 4 - p )]
= 2 sin α sin β sin γ =4−π+4−π
= 8 − 2π
= 2 xyz
44. Given
39. We have
p
tan-1 x + tan-1 y + tan-1 z =
-1 1- sin x + 1+ sin x 2
cot
1- sin x - 1+ sin x Let tan-1 x = a , tan-1 y = b , tan-1 z = y . Then
40. We have ⇒ xy + yz = zx = 1
Now,
tan2 ( sex -12) = cot 2 (cosec -1 3)
( x - y )2 + ( y - z )2 + ( z - x )2 = 0
é æ 3 öù é -1 æ 2 2 ö ù
= tan2 ê tan-1 çç ÷÷ ú = cot êcot çç
2
÷÷ ú Þ x 2 + y 2 + z 2 = xy + yz + zx = 1
êë è 1 ø úû êë è 1 ø úû
45. tan1, tan-1 1,sin1,sin-1 1
= ( 3 ) + (2 2 ) = 3 + 8 = 11
2 2
p p
tan 57°, ,sin 57°,
41. Since 4 2
7p 3.14 3 3.14
3p < 10 < 1.539 , , ,
2 4 2 2
Chapter 17 | Inverse Trigonometry 731
p 3p 1+ x 2
Hence, third angle = p - = . 2. ³ 2 ( ∵ x > 0)
4 4 x
48. We have
x 1 -1 æ x ö æ pù
⇒ £ ⇒ sin ç Î 0, ú
2 ÷ ç
æ 2p ö -1 æ 2p ö 1+ x 2 2 è 1+ x ø è 6 û
cos -1 ç cos ÷ - sin ç ÷
è 3 ø è 3 ø
æ y ö æ pù
2p p p Similarly, sin-1 ç Î 0, ú .
2 ÷ ç
= - = è 1+ y ø è 2 û
3 3 3
æ 2p ö
49. Given So, range of the given expression is ç 0 , ÷.
è 3 ø
x = sin A, y = sin B, z = sin C
p
Now, 3. sin−1sinθ > − sin−1sinθ
2
sin A cos B cos C + sin B cos A cos C + sin C cos A cos B
p
⇒ sin−1sinθ >
−sin A sin B sin C = sin (A + B + C) = sin nπ = 0 4
Therefore, given expression = sin A sin B sin C = xyz. Therefore,
50. We have 1 p 3p
sinθ > Þ <q <
2 4 4
tan A - tan B
tan( A - B ) =
1+ tan A tan B é æ 1 öù
4. sin[cot−1(x + 1)] = sin êsin-1 ç ÷ú
x 3 2x - k êë è x + 2 x + 2 ø úû
2
-
2k - x k 3
= 1
x 3 2x - k ⇒ sin[cot−1(x + 1)] =
1+ ´
2k - x k 3 x + 2x + 2
2
3 xk - 4 kx + 2k 2 + 2 x 2 - xk é æ 1 öù 1
= cos(tan-1 x ) = cos êcos -1 ç ÷ú =
2k 2 3 - xk 3 + 2 x 2 3 - kx 3 êë è 1+ x
2
ø úû 1+ x 2
2 x 2 + 2k 2 - 2kx 1 ⇒
1
=
1
= =
3 (2 x 2 + 2k 2 - 2kx ) 3 x + 2 x + 2 1+ x 2
2
p 1
Hence, A - B = . ⇒x=−
6 2
732 Mathematics Problem Book for JEE
æp ö æp ö 12 3 63
⇒ g(x) = 3 tanx, g ç ÷ = 3, g’ ç ÷ = 12 LHS = tan−1 + tan−1 + tan−1
è ø
4 è3ø 5 4 16
12 3
-1
7. Let y = sin (2 x 1- x )
2
Since × > 1 , we have
5 4
Put x = sinθ 12 3
+
12
-1 -1 3 -1 5 4 = p - tan-1 63
é p pù tan + tan = p + tan
θ ∈ ê- , ú 5 4 12 3
1- × 16
ë 2 2û 5 4
⇒ y = sin−1(sin2θ) So,
⎧ 12 3 63
⎪2 sin−1 x , ⎡ 1 1 ⎤ tan−1 + tan−1 + tan−1 = p
x ∈⎢ − , ⎥ 5 4 16
⎪ ⎣ 2 2⎦
⎪
⎪ −1 ⎡ 1 ⎤ 1
(C) 2 tan−1 = tan−1
2/3
= tan−1
3
⇒ y = ⎨p − 2 sin x x ∈⎢ ,1⎥
3 1 4
⎪ ⎣ 2 ⎦ 1−
⎪ 1
9
⎪ −p − 2 sin−1 x −1 ≤ x ≤ −
⎪⎩ 2 and
4 4
sin−1 = tan−1
5 3
æ 2 cos x - 3 sin x ö Therefore,
8. (A) y = cos -1 ç ÷
è 13 ø 4 1 4 3
sin−1 + 2 tan−1 = tan−1 + tan−1
3 5 3 3 4
y = cos−1 cos (α + x) ∀ α = tan-1
2 4 4 p
= tan-1 + cot -1 =
dy 3 3 2
Therefore, =1.
dx (D) cosec−1x = cot −1 x 2 − 1
-1 æp ö p Hence,
(B) cos cos ç ÷ =
è 14 ø 14
41 41 ⎛ 5⎞
n n cosec−1 = cot −1 − 1 = cot −1 ⎜ ⎟
(C) å æç p ö÷ = å æç 1 ö÷ = finite ⇒ a > ⎝ 4⎠
¥ ¥ 1 4 16
n=1 è 2ap ø n=1 è 2a ø
2
Therefore,
æ 1ö æ 1ö 41 5
sin-1 ç ÷ + cos -1 ç ÷ cot −1 9 + cos ec −1= cot −1 9 + cot −1
(D) f(x) = è ø
x è x ø = p sin x 4 4
cosecx 2 1 4
+
−1 1 −1 4 −1 9 5 = tan−1 1 = p
= tan + tan = tan
⎛ 1⎞ ⎛ 1⎞ ⎛ 1⎞ 9 5 1 4
9. (A) tan−1 ⎜ ⎟ + tan−1 ⎜ ⎟ + tan−1 ⎜ ⎟ + + ∞ 1− ⋅ 4
⎝ 3⎠ ⎝ 7⎠ ⎝ 13 ⎠ 9 5
Chapter 17 | Inverse Trigonometry 733
æ 7 - 5( x 2 + 3) ö -1 é 1 5ù 1
10. (A) sec -1 ç ÷ = sec ê 2 - ú (D) y = tan−1 + tan−1 b , (0 < b < 1)
è 2( x 2
+ 2 ) ø ë ( x + 2 ) 2 û 2
Since ⎛ 1/ 2 + b ⎞ ⎛ 1 ⎞
⇒ y = tan−1 ⎜ ⎜⎝∵ b < 1⎟⎠
⎝ 1− b / 2 ⎟⎠
,
1 1 2
£
x +2 2
2
−1 ⎛ 1+ 2b ⎞ p
0 < tan ⎜ ≤
⎝ 2 − b ⎟⎠ 4
1 5
− ≤ −2
x2 + 2 2 ⎛ 1+ 2b ⎞
⇒0< ⎜ ≤1
2p ⎝ 2 − b ⎟⎠
Therefore, the maximum value is sec−1(−2) = .
3 ⇒ 0 < (1 + 2b) ≤ (2 − b), (1 + 2b > 0)
p 1
(B) Minimum value = cosec−12 + sec−11 = , when ⇒ 3b ≤ 1 ⇒ b ≤
6 3
⎡ 2 1⎤ 1
⇒ bmax =
⎢⎣3 x + 4 ⎥⎦ = 1 3
⎡ ⎛ 1⎞ ⎛ 1⎞ ⎛ 1⎞ ⎛ 1 ⎞⎤
p 11. cot ⎢ tan−1 ⎜ ⎟ + tan−1 ⎜ ⎟ + tan−1 ⎜ ⎟ + tan−1 ⎜ ⎟ ⎥
(C) −1 2 −1
sin x − 1 + cos 2 x − 5 = 2
⎣ ⎝ 3 ⎠ ⎝ 7 ⎠ ⎝ 13 ⎠ ⎝ 21⎠ ⎦
2
⎡x+y⎤
⇒ |x2 − 1| = |2x2 − 5| Using, tan−1 x + tan−1 y = tan−1 ⎢ ⎥ , we get
⎣1− xy ⎦
⇒ x2 = 2
⎡ −1 ⎛ 1⎞ ⎛ 1⎞ ⎤ 3
n ⎜ ⎟ + tan−1 ⎜ ⎟ ⎥ =
cot ⎢ tan
⇒x=± 2 (Two solutions) ⎣ ⎝ 2⎠ ⎝ 8⎠ ⎦ 2
734 Mathematics Problem Book for JEE
3. Square matrix: If m = n, i.e. if the number of rows and columns 18.4 Equality of Matrices
of a matrix are equal, say n, then it is called a square matrix of
order n. Two matrices A and B are said to be equal, written as A = B, if
4. Null (or zero) matrix: If all the elements of a matrix are equal to 1. they both are of the same order, i.e. have the same number of
zero, then it is called a null matrix and is denoted by Om×n or O. rows and columns and
5. Diagonal matrix: A square matrix in which all its non-diagonal 2. the elements in the corresponding places of the two matrices
elements are zero is called a diagonal matrix. Thus, in a diago- are the same.
nal matrix aij = 0 if i ≠ j.
The diagonal matrices of orders 2 and 3 are as follows: 18.5 Addition and Subtraction
k 0 0
of Matrices
k1 0 1
, 0 k2 0 Let A = [aij] and B = [bij] be two matrices of the same order m × n.
0 k2
0 0 k3 Then, their sum (or difference) A + B (or A − B) is defined as another
matrix of the same order, say C = [cij] such that any element of C is
The elements aij of a square matrix for which i = j are called the the sum (or difference) of the corresponding elements of A and B.
diagonal elements of a matrix and the diagonal along which all Therefore,
these elements lie is called the principal diagonal or the lead- C = A ± B = [aij ± bij]
ing diagonal or the diagonal of the matrix. 1 2 4
6. Scalar matrix: A square matrix in which all the diagonal Illustration 18.2 Find A + B and A – B where A and
0 5 3
elements are equal and all other elements are equal to zero is 7 3 2
B
called a scalar matrix. 5 1 9
That is, in a scalar matrix aij = k, for i = j and aij = 0 for i ≠ j. Thus,
k 0 0 Solution: Here, both A and B are 2 × 3 matrices. Therefore,
0 k 0 is a scalar matrix. 1 7 2 3 4 2 8 5 6
AB
0 0 k 0 5 5 1 3 9 5 6 12
7. Unit matrix or identity matrix: A square matrix in which all and
its diagonal elements are equal to 1 and all other elements are 1 7 2 3 4 2 6 1 2
equal to zero is called a unit matrix or an identity matrix, denot- AB
0 5 5 1 3 9 5 4 6
ed by U or I.
For example, unit (or identity) matrices of orders 2 and 3 are
1 0 0 18.5.1 Properties of Matrix Addition
1 0
and 0 1 0 , respectively. 1. A+B=B+A
0 1
0 0 1 2. A + (B + C) = (A + B) + C
3. A + O = O + A = A; here O {null matrix} will be additive identity.
8. Negative of a matrix: Let A = [aij]m×n be a matrix. Then, the
4. If A is a given matrix, then the matrix −A is the additive inverse
negative of the matrix A is defined as the matrix [−aij]m×n and
of A for A + (−A) = null matrix O.
is denoted by −A.
5. If A, B and C are three matrices of the same order, then
9. A square matrix in which all elements below leading diagonal A+B=A+C ⇒ B=C (left cancellation law)
or all elements above leading diagonal are zero is called a tri-
and
angular matrix.
(i) Upper triangular matrix: A square matrix A = [aij] is called an B+A=C+A ⇒ B=C (right cancellation law)
upper triangular matrix if aij = 0, for all i > j. Thus, in an upper
triangular matrix all elements below diagonals are zero. 18.6 Multiplication of a Matrix
a b c by a Scalar
For example, A 0 p q is an upper triangular matrix.
Let A = [aij]m × n be a matrix and k a scalar. Then, the matrix obtained
0 0 r
by multiplying each element of matrix A by k is called the scalar
(ii) Lower triangular matrix: A square matrix A = [aij] is called multiple of A and is denoted by kA.
a lower triangular matrix if aij = 0 for all i < j. Thus, in a low-
er triangular matrix, all elements above diagonal are zero. 18.6.1 Properties of Multiplication of a Matrix
1 0 0 by a Scalar
For example, B 2 3 0 is a lower triangular matrix. 1. If k1 and k2 are scalars and A be a matrix, then (k1 + k2)A = k1A
4 5 6 + k2A.
Chapter 18 | Matrices and Determinants 737
Trace of a Matrix
5. Orthogonal matrix: A square matrix A is said to be orthogonal
n if A′A = I = AA′.
tr(A) = aii = a11 + a22 + + ann
i 1 6. Unitary matrix: A square matrix A is said to be unitary if AqA
= I = AAq.
7. Idempotent matrix: A square matrix A such that A2 = A is called
18.9 Types of a Matrix on the an idempotent matrix.
Basis of Operations 8. Nilpotent matrix: A square matrix A will be called a nilpotent
matrix if Ak = O (null matrix) where k is a positive integer. If how-
1. Symmetric matrix: A square matrix A = [aij] is said to be ever k is the least positive integer for which Ak = O then k is the
symmetric if its (i, j)th element is the same as its (j, i)th element, index of the nilpotent matrix A.
i.e. aij = aji for all i, j.
9. Involutory matrix: A square matrix A such that A2 = I is called
2. Skew-symmetric matrix: A square matrix A = [aij] is said to be the involutory matrix.
skew-symmetric if the (i, j)th element of A is the negative of the
(j, i)th element of A, i.e. if aij = −aji for all i, j.
Your Turn 1
Properties of Symmetric and Skew-Symmetric Matrices
1. If A is a symmetric matrix, then A′ = A. 1 1 1 1 3
2. If A is a skew-symmetric matrix, then A′ = –A. 1. If A 2 0 3 and B 0 2 then AB + BA = O.
3. Diagonal elements of a skew-symmetric matrix are zero. 3 1 2 1 4
(True/False) Ans. False
740 Mathematics Problem Book for JEE
6. Use matrix multiplication to divide Rs. 30000 in two parts such 18.11.3 Determinants of the Third Order
that the total annual interest at 9% on the first part and 11% a1 b1 c1
on the second part amounts Rs. 3060.
Ans. First part → 12000 The notation a2 b2 c2 consisting of 32 elements, arranged
Second part → 18000 a3 b3 c3
in three rows and three columns, is called a determinant of third
0 a2 ab ac
b
c order. Its value is
7. If A c a and B ab b2 bc , show that
0 a1b2c3 + a2b3c1 + a3b1c2 − a1b3c2 − a2b1c3 − a3b2c1
b a 0 ac bc c 2
This may be written as
AB = BA = O3×3.
a1(b2c3 − b3c2) − b1(a2c3 − a3c2) + c1(a2b3 − a3b2)
3 2 3
8. Express the matrix A 4 5 3 as the sum of a symmetric b2 c2 a c2 a2 b2
or a1 - b1 2 + c1
2 4 5
b3 c3 a3 c3 a3 b3
and a skew-symmetric matrix. We can therefore write
3 3 5 / 2 0 1 1/ 2
a1 b1 c1
Ans. A 3 5 7 / 2 1 0 1/ 2 b c2 a c2 a b2
a2 b2 c2 = a1 2 - b1 2 + c1 2 (18.3)
5 / 2 7 / 2 5 1/ 2 1/ 2 0 b3 c3 a3 c3 a3 b3
a3 b3 c3
9. Let A and B be symmetric matrices of the same order. Then,
show that Note that each term of a second-order determinant is the product
of two quantities and each term of a third-order determinant is the
(a) AB − BA is a skew-symmetric matrix.
product of three quantities.
(b) AB + BA is a symmetric matrix.
3 4 2 1 2
18.12 Minors
10. If A ,B , show that (AB)T = BTAT.
1 1 1 3 4 The minor of a given element of a determinant is the determinant
of the elements which remain after deleting the row and the
column in which the given element occurs.
DETERMINANTS The minor of a1 in Eq. (18.2) is b2 and b2 may be considered a
determinant of first order. Similarly, the minor of a2 is b1.
18.10 Definition of a Determinant For example, the minor of a1 in Eq. (18.3) is
b2 c2
and the
b3 c3
Every square matrix A can be associated to a number or an a1 c1
expression which is known as the determinant of A and is denoted minor of b2 in Eq. (18.3) is a c .
3 3
by |A| or det A.
Chapter 18 | Matrices and Determinants 741
18.13 Cofactors The necessary and sufficient condition for a square matrix A to
possess the inverse is that |A| ≠ 0.
In Eq. (18.3), the elements a1, b1, c1 are multiplied by 1
If A be an invertible matrix, then the inverse of A is adj A. It
−1 | A|
b2 c2 a2 c2 a2 b2 is usual to denote the inverse of A by A .
, - ,
b3 c3 a3 c3 a3 b3
18.15.1 Theorem (Uniqueness of Inverse)
These expressions are called the cofactors of the elements a1, b1, c1.
Theorem: Inverse of a square matrix if it exists is unique.
Generally, the cofactor of an element is its minor with its sign or
Proof: Let A = [aij ]n´n be a square matrix. Let inverse of A exist.
opposite sign prefixed in accordance with the following rule.
To prove: Inverse of A is unique.
For any determinant if aij is the element at the intersection of the
If possible, let B and C be two inverses of A. Then
i th row and j th column, then the cofactor of aij has positive sign or
negative sign before minor of aij according to i + j is even or odd. The AB = BA = In and AC = CA = In
determinant may be expanded along any chosen row or column. Now
The cofactors of the elements a1, b1, c1, a2, b2, c2, a3, b3, c3 will B = B ln = B( AC ) [since AC = In ]
be denoted by A1, B1, C1, A2, B2, C2, A3, B3, C3, respectively. = (BA) C = InC = C
For example, element b3 in Eq. (18.3) lies at the intersection of
Hence B = C . This implies that the inverse of A is unique.
the third row and the second column. Since 3 + 2 = 5 is an odd
number, we have 18.15.2 Properties of Inverse of a Matrix
a c
B3 = - 1 1 1. (AB)−1 = B−1A−1 2. (A′)−1 = (A−1)′
a2 c2 3. (A−1)q = (Aq )−1
a1 c1 0 1 2
The cofactor B2 of the element b2 is + because element
a3 c3 Illustration 18.7 Find the inverse of the matrix A 1 2 3 .
b2 lies at the intersection of the second row and the second col- 3 1 1
umn, and 2 + 2 = 4 is an even number. Solution: We find the determinant of A,
Let the determinant in Eq. (18.3) be denoted by Δ. When the 0 1 2
cofactors are used, the expansion of the determinant takes the A= 1 2 3
compact form: 3 1 1
Δ = a1A1 + b1B1 + c1C1 = a2A2 + b2B2 + c2C2 = a3A3 + b3B3 + c3C3 Expanding along R1 we get
Δ = a1A1 + a2A2 + a3A3 = b1B1 + b2B2 + b3B3 = c1C1 + c2C2 + c3C3 |A| = 0(2 − 3) − 1(1 − 9) + 2(1 − 6) = 8 − 10 = −2
and Since |A| ≠ 0, therefore A−1 exists.
a2A1 + b2B1 + c2C1 = 0 = a2A3 + b2B3 + c2C3, etc.
Now the cofactors of the elements of the first row of |A| are
2 3 1 3 1 2
18.14 Adjoint of a Square Matrix ,- , , that is, are −1, 8, −5, respectively.
1 1 3 1 3 1
Let A = [aij]n×n be any n × n matrix. The transpose B′ of the matrix The cofactors of the elements of the second row of |A| are
B = [Cij]n×n, where Cij denotes the cofactor of the element aij in the 1 2 0 2 0 1
determinant |A|, is called the adjoint of the matrix A and is denoted - , ,- , that is, are 1, −6, 3, respectively.
1 1 3 1 3 1
by the symbol adj A.
The cofactors of the elements of the third row of |A| are
⎡a b ⎤
Illustration 18.6 If A = ⎢ ⎥ , then find adj A. 1 2 0 2 0 1
⎣g d ⎦ ,- , , that is, are −1, 2, −1, respectively.
2 3 1 3 1 2
Solution: In |A|, the cofactor of a is d and the cofactor of b is −g.
Also the cofactor of g is −b and the cofactor of d is a. Therefore, the Therefore, adj A = the transpose of the matrix B where
matrix B formed of the cofactor of the elements of |A| is 1 8 5
⎡d −g ⎤ B 1 6 3
B=⎢
⎣− b a ⎥⎦ 1 2 1
So,
⎡d −b ⎤ 1 1 1
Now, adj A = the transpose of the matrix B = ⎢ .
⎣ −g a ⎥⎦ adj A 8 6 2
18.15 Inverse of a Matrix 5 3 1
Now
1
Let A be any n-rowed square matrix. Then, a matrix B, if it exists, A-1 = adj A
such that AB = BA = In, is called inverse of A. | A|
742 Mathematics Problem Book for JEE
Solution: Determinant of coefficient matrix is |A| = −2 which is system of equations has one or more solutions, the equations are
non-zero. said to be consistent, otherwise they are said to be inconsistent.
Therefore, x = y = z = 0 is the only solution. If B ≠ 0, the system (18.7) is said to be non-homogenous.
Alternate method (Using Rank): The given system of equations adj A
-1
can be written in the form of the single matrix equation as 1. If |A| ≠ 0 → X = A-1B , where A =
| A|
1 2 3 x 0 The given system has unique solution.
AX 3 4 4 y 0 O 2. If |A| = 0, since AX = B , we have
7 10 12
z
0 (adj A) AX = (adj A)B ⇒ | A | X = (adj A)B
We shall start reducing the coefficient matrix A to triangular form ⇒ (adj A)B = 0 [since | A | = 0]
by applying only E-row transformations on it. Applying R2 → R2 −
3R1, R3 → R3 − 7R1, the given system of equations is equivalent to which is true for infinite values of X.
18.24 Cofactor of Any Element of a Matrix 2. The numbers ai, bi, ci (i =1, 2, 3) are called the elements of
the determinant.
The minor Mij multiplied by (−1)i+j is called cofactor of the element
aij. We shall denote the cofactor of an element by the Cij. With this 3. The determinant obtained by deleting the i th row and j th
notation, cofactor of aij = Cij = (−1)i+jMij. column is called the minor of element at the i th row and
the j th column. The cofactor of this element is (−1)i+j (minor).
Note that
18.25 Determinant of Any Matrix
a1 b1 c1
If matrix A = [aij] is a square matrix of order ‘n’, then Δ = a2 b2 c2 = a1A1 + b1B1 + c1C1
n n
Determinant of A = a1k C1k a2k C2k a3 b3 c3
k 1 k 1 where A1, B1 and C1 are the cofactors of a1, b1 and c1,
n n
= ak1C k1 ak 2C k 2
respectively.
k 1 k 1
where Cik represents cofactor of the element of the i th row and 18.26 Properties of Determinants
the kth column of matrix A.
1. If two rows (or columns) in a determinant are interchanged,
For 3 × 3 order matrix A;
the sign of the determinant changes. For example, by
det A (or |A|) = a11C11 + a12C12 + a13C13 a1 b1
= a21C21 + a22C22 + a23C23 interchanging the two rows of the determinant , we
a2 b2
= a31C31 + a32C32 + a33C33 a b
2 2
= a11C11 + a21C21 + a31C31 get the determinant a b .
1 1
= a12C12 + a22C22 + a32C32
But we have
= a13C13 + a23C23 + a33C33
a2 b2 a1 b1
Thus determinant of a matrix can be obtained by adding the prod- =−
a1 b1 a2 b2
ucts of elements of any row or column by their cofactors.
Note: If elements of a row (or column) are multiplied by the 2. If the numbers in one row are added m times the numbers in
cofactors of any other row (or column), then the sum of these another row, the value of the determinant remains unaltered.
products is zero. For example, For example,
a11c21 + a12c22 + a13c23 = 0 a1 + ma2 b1 + mb2 a b
= 1 1
2 3 4 a2 b2 a2 b2
Illustration 18.11 Evaluate the determinant Δ = 5 - 2 1 . This rule can be extended to more number of rows for higher
1 2 3 order determinants.
Solution: We can do it in two ways.
(a) Expanding along the second row, we have 3. If rows and columns are interchanged, the value of the
determinant remains unaltered. For example,
3 4 2 4 2 3
D = -5 -2 -1 a1 b1 a a
2 3 1 3 1 2 = 1 2
a2 b2 b1 b2
= − 5 (9 − 8) − 2 (6 − 4) − 1 (4 − 3)
= − 5 − 4 − 1 = − 10 Another way of saying this is that it makes no difference if
(b) Expanding along the third column, we have we reflect the numbers of the determinant in the line of the
principal diagonal. This means that any statement that can
5 -2 2 3 2 3
D=4 -1 +3 truly be made about rows in particular results (1) and (2) can
1 2 1 2 5 -2 equally well be made about columns.
= 4 (10 + 2) − 1 (4 − 3) + 3 (− 4 − 15) 4. If all the numbers in any row are zeros, the value of the
= 48 − 1 − 57 = − 10 determinant is zero. For example,
0 0 1 a b+c c
= (a − b) (b − c) 1 1 c =2 b c +a a (C2 → C2 − C3)
a + b + c b + c + a c 2 - ab c a+b b
= (a − b) (b − c) [(b + c + a) − (a + b + c)] = 0 a b c
(Expanding along R1) =2 b c a
Note: If a determinant can be so transformed that two elements c a b
in a row or column are made zero, then the determinant can be
expanded in terms of that row or column. 1 a a2
1 a bc
1 a a2 1 bc b + c Illustration 18.17 Show that 1 b ca = 1 b b2 .
Illustration 18.15 Show that 1 b b2 = 1 ca c + a . 1 c ab 1 c c2
1 c c 2 1 ab a + b
Solution: Let Δ stand for the determinant on the left. Then
Solution: We have
a a2 abc a a2 1
1 bc b + c 1 bc a + b + c - a
1 abc
1 ca c + a = 1 ca a + b + c - b D = b b2 abc = b b2 1
abc abc
1 ab a + b 1 ab a + b + c - c c c2 abc c c2 1
1 bc a + b + c 1 bc a a 1 a2 1 a a2
= 1 ca a + b + c - 1 ca b = - b 1 b2 = 1 b b2
1 ab a + b + c 1 ab c c 1 c 2
1 c c2
1 bc 1 bc 1 a
= (a + b + c) 1 ca 1 + ca 1 b
Additional Properties of Determinants:
1 ab 1 ab 1 c
1. The determinant remains unaltered if its rows are changed
bc 1 a into columns and the columns into rows.
= ca 1 b , since the first determinant vanishes 2. If all the elements of a row (or column) are zero, then the
ab 1 c determinant is zero.
abc a a2 1 a a2 3. If the elements of a row (column) are proportional (or
1 abc identical) to the elements of any other row (column), then
= abc b b 2
= 1 b b 2
abc abc the determinant is zero.
2
abc c c 1 c c2
4. The interchange of any two rows (columns) of the
determinant changes its sign.
Illustration 18.16 Without expanding the determinants, prove
5. If all the elements of a row (column) of a determinant are
a+b b+c c +a a b c multiplied by a non-zero constant, then the determinant
that b+c c +a a+b = 2 b c a . gets multiplied by the same constant.
c +a a+b b+c c a b 6. A determinant remains unaltered under a column (Ci)
operation of the form Ci + a Cj + b Ck (j, k ≠ i) or a row (Ri)
Solution: The determinant on the left is equal to
operation of the form Ri + a Rj + b Rk (j, k ≠ i).
a+b b+c c +a 7. If each element in any row (column) is the sum of r terms,
b+c c +a a+b (C1 → C1 + C2 + C3) then the determinant can be expressed as the sum of r
c +a a+b b+c determinants.
8. If the determinant Δ = f(x) and f(a) = 0, then (x − a) is a
2 (a + b + c ) b + c c + a
factor of the determinant. In other words, if two rows (or
= 2 (a + b + c ) c + a a + b
two columns) become proportional (identical) for x = a,
2 (a + b + c ) a + b b + c then (x − a) is a factor of determinant. In general, if r rows
a+b+c b+c c +a become identical for x = a, then (x − a)r−1 is a factor of the
=2 a+b+c c +a a+b (C1 → C1 − C2) determinant.
a+b+c a+b b+c 9. If in a determinant (of order 3 or more) the elements in all
the rows (columns) are in AP with same or different common
a b+c c +a difference, the value of the determinant is zero.
=2 b c +a a+b (C3 → C3 − C1) 10. The determinant value of an odd-order skew-symmetric
c a+b b+c determinant is always zero.
750 Mathematics Problem Book for JEE
3 ( 4 k - 1) z 4n - 1
r0
f (r ) (n 1) 2
2 n
n 1 0 (since R1 R2 )
n cos (n ) cos n cos (n 1)
2 2 2 2
å Ak = 0.
k =1
18.28 Multiplication of Determinants
Solution: Observe that all the determinants A1, A2 ,… , An have Two determinants of the same order, i.e. each consisting of the
identical second and third columns. Hence, same number of rows and equal number of columns, can be
n multiplied to give a determinant of the same order. Thus, if A is
2k 1 x 2n 1 a 2 × 2 determinant and B is another 2 × 2 determinant, A × B =
k 1 C is also 2 × 2 determinant. The multiplication is performed by a
n n method of working the row of A on the columns of B.
Ak k 12 (3k 1)
k 1
y 3n 1 The method is as follows: If
n a1 a2 a a2
A= ;B= 1
3 ( 4 k 1)
k 1
z 4n 1 b1 b2 b1 b 2
Chapter 18 | Matrices and Determinants 751
distances from the diagonal are equal both in magnitude and sign. Solving the system we get
For example, c1 b1
a h g c1 b2 - c2b1 c2 b2
x = = ;
h b f = abc + 2fgh - af 2 - bg2 - ch2 a1b2 - a2b1 a1 b1
g f c a2 b2
a1 c1
18.30.2 Skew-Symmetric Determinant a c - a2c1 a c
y = 12 = 2 2
If aij = −aji (where aij is the element of i th row and j th column), then a1b2 - a2b1 a1 b1
the determinant is said to be a skew-symmetric determinant, a2 b2
which means that all the diagonal elements are zero and the
Note: The given equations are consistent and independent if and
elements situated at equal distances from the diagonal are equal
a b
in magnitude but opposite in sign. The value of a skew-symmetric only if 1 1 ≠ 0.
a2 b2
determinant of odd order is zero. For example,
0 3 5 Illustration 18.24 Solve the system 4x + y = 13, 3x − 2y = 7 using
A 3 0 4 determinants.
Solution: The solution requires the values of three determinants.
5 4 0
The denominator Δ is formed by writing the coefficients of x and
A 0
y in order
4 1
18.30.3 Circulant Determinants Δ= = − 8 − 3 = − 11
3 -2
In these determinants, the elements of the rows (or columns) are in Δ1, the numerator of x, is formed by replacing the coefficients of x
cyclic arrangement. For example,
by the constant terms
a b c 13 1
D1 = = − 26 − 7 = − 33
b c a = −(a3 + b3 + c 3 − 3abc ) 7 -2
c a b Δ2, the numerator of y, is formed by replacing the coefficients of y
1 by the constant terms
= − (a + b + c ) × {(a − b )2 + (b − c )2 + (c − a)2 }
2 4 13
D2 = = 28 − 39 = − 11
3 7
a b c
Then
Illustration 18.23 Evaluate the determinant D = b c a D1 - 33
x= = = 3
c a b D - 11
and show that it is negative for all positive values of a, b and c. Δ 2 − 11
and y= = =1
Solution: Expanding along the first row, we have Δ − 11
c a b a b c 18.31.2 Solution of System of Three Linear
D=a -b +c Equations in Three Unknowns
a b c b c a
Consider the system of three linear equations in three unknowns:
Δ = a(bc − a2) − b(b2 − ca) + c(ab − c2) = 3abc − a3 − b3 − c3
a1x + b1y + c1z = d1
= − (a3 + b3 + c3 − 3abc) = − (a + b + c) {a2 + b2 + c2 − ab − bc − ca}
a2x + b2y + c2z = d2
(a + b + c )
= - {(a - b )2 + (b - c )2 + (c - a)2 } a3x + b3y + c3z = d3
2
Consider
is negative if a, b and c are positive.
a1 b1 c1 d1 b1 c1
D = a2 b2 c2 , D1 = d2 b2 c2
18.31 Solution of System of Linear a3 b3 c3 d3 b3 c3
Equations a1 d1 c1 a1 b1 d1
D 2 = a2 d2 c2 , D2 = a2 b2 d2
18.31.1 Solution of System of Two Linear a3 d3 c3 a3 b3 d3
Equations in Two Unknowns
1. If Δ ≠ 0, system has unique solution given by
Consider the system of two linear equations in two unknowns:
D1 D D
a1x + b1y = c1 x= ,y= 2,z= 3
a2x + b2y = c2 D D D
754 Mathematics Problem Book for JEE
That is, system is consistent with independent solution. Let Dj be the determinant obtained from D after replacing the j th
2. If Δ = 0 and Δ1 = Δ2 = Δ3 = 0 then system has infinite many solu- b1
tions. That is, system is consistent with dependent solution. .
3. If Δ = 0 and any of Δ1, Δ2, Δ3 are non-zero then the system has
column by . .
no solution. That is, system is inconsistent.
.
18.31.3 Solution of System of Three Equations bn
in Two Unknowns Then, if D ≠ 0, we have
The following system of equations D1 D D
a1x + b1y + c1 = 0 ; a2 x + b2 y + c2 = 0 ; a3 x + b3 y + c3 = 0 x1 = , x2 = 2 , … , xn = n
D D D
is consistent if
When D = 0, we have the following cases:
a1 b1 c1 Case 1: If D = 0 and the other determinants D1 = D2 = … = Dn
a2 b2 c2 = 0 = 0, then system of equation has infinitely many solutions if all
a3 b3 c3 cofactors of D1, D2, …, Dn and D are zero. If any one cofactor of D1,
D2, D3,…, Dn is non-zero then system has no solution.
Illustration 18.25 Find those values of c for which the Example: x + 3 y + 2 z = 1; 2 x + 6 y + 4 z = 5; 3 x + 9 y + 6 z = 9
equations 2 x + 3 y = 3; (c + 2) x + (c + 4 ) y = (c + 6 ) and (c + 2)2 x + Here, D x = D y = Dz = D = 0 yet system has no solution whereas
(c + 4 )2 y = (c + 6 )2 are consistent. Also solve the equations for x + 3 y + 2 z = 1 ; 2 x + 6 y + 4 z = 2; 3 x + 9 y + 6 z = 3
those values of c. has infinitely many solutions.
Solution: The condition for consistency is Case 2: If D = 0 but any one of the D1, D2, …, Dn is not equal to zero
then the system has no solution, hence is inconsistent.
2 3 3
c 2 c 4 c 6 0 Cramer’s Rule
If
(c 2)2 (c 4 )2 (c 6 )2
a1 b1 c1
−1 3 0 Δ = a2 b2 c2 ≠ 0
⇒ −2 c+4 2 = 0 (C1 → C1 − C2) a3 b3 c3
−2(2c + 6 ) (c + 4 )2 2(2c + 10 ) then solution of linear equations a1x + b1y + c1z = d1, a2 x + b2 y
( 1){(c 4 )(2c 10 ) (c 4 )2 } 3{ 2(2c 10 ) 2(2c 6 )} 0 + c2 z = d2 and a3 x + b3 y + c3 z = d3 is given by {where (d1, d2 , d3 )
≠ (0 , 0 , 0 )}
c 2 8c 16 2c 2 18c 40 12c 60 12c 36 0
Δx Δy Δ
⇒ − c2 −10 c = 0 ⇒ c = 0 or c = −10 x= , y= , z= z
Δ Δ Δ
For c = 0, the three equations are
where
2x + 2y = 3; 2x + 4y = 6; 4x + 16y = 36
d1 b1 c1 a1 d1 c1
and the solution is x = −3; y = 3. For c = −10, the equations are
Δ x = d2 b2 c2 , Δ y = a2 d2 c2 ,
2x + 3y = 3
-8 x - 6 y = -4 Þ 4 x + 3 y = 2 d3 b3 c3 a3 d3 c3
64 x - 36 y = 16 Þ 16 x + 9 y = 4 a1 b1 d1
1 4 Δ z = a2 b2 d2
and the corresponding solution is x ; y .
2 3 a3 b3 d3
18.31.4 Cramer’s Rule 1. If any of Δx, Δy, Δz ∈ R and Δ ≠ 0, the system of equation
Consider the system of n linear equations in n unknowns given by will have unique solution and is said to be consistent
a11x1 + a12 x 2 + + a1n x n = b1 independent.
2. If Δx = Δy = Δz = 0 and Δ is also zero, then the system of
a21x1 + a22 x 2 + + a2n x n = b2 equation will have infinitely many solutions and is said to
……………………………… be consistent dependent.
……………………………… 3. If Δx, Δy, Δz are non-zero and Δ is zero, then the system of
an1x1 + an2 x 2 + + ann x n = bn equations will have no solution and is said to be inconsistent.
Let
a11 a12 … a1n Illustration 18.26 Solve the following system using determi-
a21 a22 … a2n nants:
D= x + 4y + 4z = 7
: :
3x + 2y + 2z = 6
an1 an2 … ann
9x + 6y + 2z = 14
Chapter 18 | Matrices and Determinants 755
Solution: The solution requires the values of four determinants: Solution: The condition for the existence of non-trivial solution
The denominator (trivial solution is x = y = z = 0) is
1 4 4 l sin a cos a
Δ = 3 2 2 = 40 1 cos a sin a =0
9 6 2 −1 sin a − cos a
a b a b 1 0 1
= ⇒S= (A − Aq ) = Q
c d c d 0 1 2i
Also, 2
A − Aq = (R + iS) − (R − iS) = 2iS as a, b, c are positive.
758 Mathematics Problem Book for JEE
Hence,
a 1 a2
det(A) = −1
⇒ 3abc − (a3 + b3 + c3) = −1 |B3| = 0 d 0 = a2df
⇒ a3 + b3 + c3 = 4 f g 0
8. If M is a 3 × 3 matrix, where MTM = I and det(M) = 1, then prove If adf ≠ 0, then |B2| = |B3| ≠ 0. Hence, no solution exists.
that det(M − I) = 0.
10. Show that if A and B are symmetric and commute, then
Solution:
(a) A−1B (b) AB−1
(M − I)T = MT − I = MT − MTM = MT (I − M)
⇒ |(M − I)T| = |M − I| = |MT| |I − M| = |I − M| (c) A−1B−1are symmetric.
⇒ |M − I| = 0 Solution:
Alternate method (a) Since A and B commute: AB = BA
det(M − I) = det(M − I) det(MT) = det(MMT − MT) Pre- and post-multiplying both sides by A−1, we get
= det(I − MT) = − det(MT − I) = − det(M − I)T = −det(M − I)
A−1(AB)A−1 = A−1(BA)A−1
⇒ det(M − I) = 0
⇒ (A−1A)(BA−1) = A−1B(AA−1) (by associativity)
a2 ⇒ I(BA−1) = (A−1B)I
a 0 1 a 1 1 f
⇒ BA−1 = A−1B
9. A = 1 c b , B 0 d c , U g , V 0 . If there is a
Now,
1 d b f g h h 0
(A−1B)′ = (BA−1′ = (A−1)′B′ (by reversal law)
vector matrix X, such that AX = U has infinitely many solutions, = A−1B [as B′ = B (symmetric) and (A−1)′ = (A′)−1 = A−1]
then prove that BX = V cannot have a unique solution. If afd ≠ Hence, A−1B is symmetric.
0 then prove that BX = V has no solution. (b) Pre-and post-multiplying by B−1, we get
Solution: AX = U has infinite solutions. This implies |A| = 0 which B−1(AB)B−1 = B−1(BA)B−1
gives ⇒ (B−1A)BB−1 = B−1B(AB−1)
a 0 1 ⇒ B−1A = AB−1
1 c b = 0 ⇒ ab = 1 or c = d Now,
1 d b (AB−1)′= (B−1A)′ = A′B−1)′
= AB−1
[as A = A′ (symmetric) and (B−1)′ = (B′)−1 = B−1]
and −1
Hence, AB is symmetric.
a 0 f
(c) Since A and B are symmetric, we have
|A1| = 1 c g = 0 ⇒ g = h
AB = BA
1 d h
⇒ (BA) −1 = (AB) −1
a f 1 ⇒ A−1B−1 = B−1A−1
|A2| = 1 g b = 0 ⇒ g = h ⇒ (A−1B−1)′ = (B−1A−1)′ = (A−1)′ ⋅ (B−1)′ = A−1B−1
1 h b [as (A−1)′ = A−1 and (B−1)′ = B−1]
|A3| = g c b =0 11. Let a > 0, d > 0. Find the value of the determinant
h d b 1 1 1
⇒ g = h, c = d ⇒ c = d and g = h a a(a + d ) (a + d )(a + 2d )
Now BX = V. 1 1 1
a 1 1 (a + d ) (a + d )(a + 2d ) (a + 2d )(a + 3d )
1 1 1
|B| = 0 d c = 0 (since C2 and C3 are equal)
( a + 2d ) (a + 2d )(a + 3d ) (a + 3d )(a + 4 d )
f g h
Solution:
This means BX = V has no unique solution. 1 1 1
2 a a(a + d ) (a + d )(a + 2d )
a 1 1
1 1 1
|B1| = 0 d c =0 (since c = d, g = h) D=
(a + d ) (a + d )(a + 2d ) (a + 2d )(a + 3d )
0 g h
1 1 1
( a + 2d ) (a + 2d )(a + 3d ) (a + 3d )(a + 4 d )
a a2 1
|B2| = 0 0 c = a2cf = a2df (since c = d ) Take common 1
from R1
f 0 h a(a + d )(a + 2d )
Chapter 18 | Matrices and Determinants 759
q = 2np, n∈ Z.
Multiplying C1, C2, C3 by a, b, c, respectively, and taking a, b, c
13. If x, y, z are not all zero and if ax + by + cz = 0, bx + cy + az = 0, common from R1, R2, R3, respectively, we get
cx + ay + bz = 0, prove that x:y:z = 1:1:1 or 1:w :w 2 or 1:w 2:w,
a2 + (b2 + c 2 )cos f b2 (1− cos f ) c 2 (1− cos f )
where w is the complex cube roots of unity. abc
Δ= a2 (1− cos f ) b2 + (c 2 + a2 )cos f c 2 (1− cos f )
Solution: For non-trivial solution, abc
a2 (1− cos f ) b2 (1− cos f ) c 2 + (a2 + b2 )cos f
a b c
Applying C1 → C1 + C2 + C3 we have
b c a =0
c a b a2 + b 2 + c 2 b2 (1− cos f ) c 2 (1− cos f )
Δ = a2 + b 2 + c 2 b2 + (c 2 + a2 )cos f c 2 (1− cos f )
⇒ (a + b + c) (a2 + b2 + c2 − ab − bc − ca) = 0 (1)
a2 + b 2 + c 2 b2 (1− cos f ) c 2 + (a2 + b2 )cos f
⇒ (a + b + c) (a + w b + w 2c)(a + w 2b + w c) = 0 (2)
760 Mathematics Problem Book for JEE
Statement-1: tr(A) = 0 1 0 0
Statement-2: A 3 1 12. Let A 2 1 0 . If u1 and u2 are column matrices such
(A) Statement-1 is true, Statement-2 is true; Statement-2 is 3 2 1
not a correct explanation for Statement-1. 1 0
(B) Statement-1 is true, Statement-2 is false.
that Au1 0 and Au2 1 , then u1 + u2 is equal to
(C) Statement-1 is false, Statement-2 is true.
0 0
(D) Statement-1 is true, Statement-2 is true; Statement-2 is a
correct explanation for Statement-1. 1 1
[AIEEE 2010] (A) 1 (B) 1
Solution: Let 0 1
⎛ a b⎞ 1
A = ⎜ ⎟ , a, b , c , d ≠ 0 1
⎝ c d⎠
(C) 1 (D) 1
Therefore,
0 1
a b a b a2 bc ab bd
A2 A2
c d c d ac cd bc d 2 [AIEEE 2012]
Solution: We have
a2 bc 1, bc d 2 1 and ab bd ac cd 0 1 0 0
A 2 1 0
Therefore, c ≠ 0 and b ≠ 0 Þ a + d = 0. Trace A = a + d = 0. Thus,
3 2 1
A = ad - bc = -a2 - bc = -1.
Hence, the correct answer is option (B). a d
10. The number of values of k for which the linear equations Let us consider that u1 b ; u2 e . Therefore
4x + ky+ 2z = 0 c f
kx+ 4y + z = 0 1 1
2x + 2y + z = 0 Au1 0 u1 2
possess a non-zero solution is
0 1
(A) 2 (B) 1
(C) zero (D) 3 0 0
[AIEEE 2011] Au2 1 u2 1
Solution: 0 2
4 k 2
Therefore,
k 4 1 0 k 2 6k 8 0 k 4 and 2 1
2 2 1
u1 u2 1
Hence, the correct answer is option (A). 1
11. Let A and B be two symmetric matrices of order 3. Hence, the correct answer is option (D).
Statement-1: A(BA) and (AB)A are symmetric matrices.
Statement-2: AB is symmetric matrix if matrix multiplication 13. Let P and Q be 3 × 3 matrices with P ≠ Q. If P3 = Q3 and
of A and B is commutative. P2Q = Q2P, then determinant of (P2 + Q2) is equal to
(A) Statement-1 is true, Statement-2 is true; Statement-2 is (A) −2 (B) 1 (C) 0 (D) −1
not a correct explanation for Statement-1. [AIEEE 2012]
(B) Statement-1 is true, Statement-2 is false. Solution: We have P 3 = Q 3. Therefore,
(C) Statement-1 is false, Statement-2 is true. P 3 P 2Q Q 3 Q 2 P P 2 ( P Q ) Q 2 ( Q P )
(D) Statement-1 is true, Statement-2 is true; Statement-2 is a
correct explanation for Statement-1. P 2 (P Q ) Q2 (P Q ) O
[AIEEE 2011] ( P 2 Q 2 )( P Q ) O
Solution: We have, AT = A and B T = B. Therefore, | P 2 Q 2 | 0
( A(BA)) = (BA) A = ( A B ) A = ( AB ) A = A(BA)
T T T T T Hence, the correct answer is option (C).
and (( AB ) A) = A ( AB ) = A(B A ) = A(BA) = ( AB ) A
T T T T T 14. If a, b ≠ 0, and f(n) = a n + b n and
Therefore, Statement-1 is correct. 3 1+ f (1) 1+ f (2)
Also ( AB )T = B T AT = BA = AB (since AB is commutative) 1+ f (1) 1+ f (2) 1+ f (3) = K (1- a )2 (1- b )2 (a - b )2 ,
Therefore, Statement-2 is also correct, but it is not a correct expla- 1 + f ( 2 ) 1 + f ( 3) 1 + f ( 4 )
nation of Statement-1.
Hence, the correct answer is option (A). then K is equal to
764 Mathematics Problem Book for JEE
1 Then A−1 is
(A) 1 (B) −1 (C) ab (D) 3 1 2 3 2 1
ab
[JEE MAIN 2014 (OFFLINE)] (A) 3 0 2 (B) 3 2 0
1 0 1 1 1 0
Solution:
1+ 1+ 1 1+ a + b 1+ a 2 + b 2 1 1 1 1 1 1 0 1 3 1 2 3
1+ a + b 1+ a + b
2 2
1+ a + b
3 3
=1 a b 1 a a2 (C) 0 2 3 (D) 0 1 1
1+ a 2 + b 2 1+ a 3 + b 3 1+ a 4 + b 4 1 a2 b2 1 b b2 1 1 1 0 2 3
= {(1− a) (1− b ) (a − b )2} [JEE MAIN 2014 (ONLINE SET 2)]
On comparison with the given equation, we get K = 1. Solution:
Hence, the correct answer is option (A). 1 2 3 0 0 1
A 0 2 3 1 0 0
15. If A is a 3 × 3 non-singular matrix such that AA′ = A′A and
0 1 1 0 1 0
B = A−1A′, then BB′ equals
(A) B−1 (B) (B−1) (C) I + B (D) I Applying C1 ↔ C3 on both matrices we get
[JEE MAIN 2014 (OFFLINE)]
Solution: 3 2 1 1 0 0
BB′ = A−1A′(A−1A′)′ = A−1A′A(A−1)′ A 3 2 0 0 0 1
= A−1AA′(A−1)′ = IA′(A−1)′ = I(A−1A)′ = I⋅I′ = I2 = I 1 1 0 0 1 0
Hence, the correct answer is option (D). Applying C2 ↔ C3 on both matrices we get
16. If a, b, c are non-zero real numbers and if the system of 3 1 2 1 0 0 3 1 2
equations
A 3 0 2 0 1 0 ⇒ A1 3 0 2
(a − 1) x = y + z
(b − 1) y = z + x 1 0 1 0 0 1 1 0 1
(c − 1) z = x + y
Hence, the correct answer is option (A).
has a non-trivial solution, then ab + bc + ca equals
(A) a + b + c (B) abc (C) 1 (D) −1 18. Let for i = 1, 2, 3, pi(x) be a polynomial of degree 2 in x,
[JEE MAIN 2014 (ONLINE SET 1)] pi ′ ( x ) and pi ′′ ( x ) be the first-and second-order derivatives of
Solution: For the non-trivial solution p ( x ) p ( x ) p ( x )
1 1 1
1- a 1 1
pi(x), respectively. Let A( x ) p2 ( x ) p2( x ) p2( x )
1 1- b 1 =0
p3 ( x ) p3( x ) p3( x )
1 1 1- c
Þ (1- a){(1- b )(1- c ) - 1} - 11
( - c - 1) + 11
( - 1+ b ) and B(x) = [A(x)]T A(x). Then, the determinant of B(x)
Þ (1- a){1- c - b + bc - 1} + c + b = 0 (A)is a polynomial of degree 6 in x.
Þ - c - b + bc + ac + ab - abc + c + b = 0 (B)is a polynomial of degree 3 in x.
⇒ ab + bc + ca = abc (C)is a polynomial of degree 2 in x.
Hence, the correct answer is option (B). (D)does not depend on x.
[JEE MAIN 2014 (ONLINE SET 2)]
17. If B is a 3 × 3 matrix such that B2 = 0, then det[(I + B)50 − 50B] Solution:
is equal to 2
B( x ) = A( x ) A( x ) = A( x )
(A) 1 (B) 2 (C) 3 (D) 50
[JEE MAIN 2014 (ONLINE SET 1)] Now highest power in the determinant of A(x) can be 3, as pi ¢ ( x )
Solution: is of degree 1 and pi ¢¢ ( x ) is constant. Hence, B( x ) must have
det [(I + B) 50 − 50 B]⇒[(I + B) 50 = I + 50 B] maximum degree 6.
[using induction process (I + B)n = I + nB) (assuming B2 = 0)] Hence, the correct answer is option (A).
Therefore,
det[(I + B)50 − 50B]= det[I + 50B − 50B] = 1 a2 b2 c2 a2 b2 c2
Hence, the correct answer is option (A). 19. If (a + l )2 (b + l )2 (c + l )2 = k l a b c , l ≠ 0, then
18. Let A be a 3 × 3 matrix such that (a − l )2 (b − l )2 (c − l )2 1 1 1
1 2 3 0 0 1 k is equal to
A 0 2 3 1 0 0 (A) 4labc (B) −4labc (C) 4l 2 (D) −4l 2
0 1 1 0 1 0 [JEE MAIN 2014 (ONLINE SET 3)]
Chapter 18 | Matrices and Determinants 765
Solution:
a2 b2 c2 r 2r - 1 3r - 2
a2 b2 c2 n −1
n
(a + l )2 (b + l )2 (c + l )2 = 4 al 4 bl 4cl 22. If 2
n -1 a , then the value of ∑ Δr
r =1
(a − l )2 (b − l )2 (c − l )2 l 2 − 2al l 2 − 2bl l 2 − 2c l 1 1
n(n - 1) (n - 1)2 (n - 1)(3n + 4 )
(R2 → R2 − R3 and R3 → R3 − R1) 2 2
(A) depends only on a
a2 b2 c2
(B) depends only on n
= 4l a b c (C) depends both on a and n
l - 2al
2
l - 2bl
2
l - 2c l
2 (D) is independent of both a and n
ì a2 b 2
c 2
a 2
b2 c2 ü [JEE MAIN 2014 (ONLINE SET 4)]
ïï ïï
= 4l í a b c + a b c ý Solution:
ï 2 2 2 -2al -2bl -2c l ïï
ïî l l l þ n −1
a2
∑ Δ r = Δ1 + Δ2 + "+ Δ n −1
b2 c2 r =1
4l 3 a b c 0
1 1 1
1 1 1
(since, two rows are proportional) n
= n -1 a
2 2 2 2
a b c
n æ n - 1ö
4l 3 a b c (since, k l 4 l 3 k 4 l 2 ) (n - 1) (n - 1)2 ç ÷ (3n + 4 )
2 è 2 ø
1 1 1
-8(n - 1) ì n n(n - 1) ü 25. The least value of the product xyz for which the determinant
= í (n - 1) - (n - 1)
2
ý=0
2 î2 2 þ x 1 1
1 y 1 is non−negative is
Hence, the correct answer is option (D).
1 1 z
1 2 2
23. If A 2 1 2 is a matrix satisfying the equation AAT = 9I, (A) -2 2 (B) -16 2
a 2 b (C) −8 (D) −1
where I is a 3 × 3 identity matrix, then the ordered pair (a, b) [JEE MAIN 2015 (ONLINE SET 1)]
is equal to Solution: To find the least value of xyz where
(A) (−1, 1) (B) (2, 1) x 1 1
(C) (−2, −1) (D) (2, −1) 1 y 1 ≥0
[JEE MAIN 2015 (OFFLINE)]
Solution: We have 1 1 z
1 2 2 ⇒ x(yz − 1) − 1 (z − 1) + (1 − y) = 0
A 2 1 2 ⇒ xyz − x − y − z + 2 ≥ 0
⇒ xyz ≥ x + y + z − 2 (1)
a 2 b
For x, y, z,
A AT 9I
x+y+z 3
1 2 2 1 2 a 9 0 0 ³ xyz ⇒ x + y + z ≥ 3 3 xyz (2)
3
2 1 2 2 1 2 0 9 0 From Eqs. (1) and (2)
a 2 b 2 2 b 0 0 9 xyz 3 3 xyz 2
9 0 (a 4 2b ) 4 0 0 ⇒ t3 ≥ 3t − 2 where t = 3 xyz
0 9 (2a 2 2b ) 0 9 0 ⇒ t3 − 3t + 2 ≥ 0 ⇒ (t − 1) (t2 + t − 2) ≥ 0
⇒ (t − 1) (t + 2) (t − 1) ≥ 0
2
(a 4 2b ) (2a 2 2b ) (a 4 b ) 0 0 9
2
⇒ (t − 1)2 (t + 2) ≥ 0 ⇒ t + 2 ≥ 0 ⇒ t ≥ −2
⇒ a + 2b = −4; a2 + b2 + 4 = 9; 2a − 2b = −2; ⇒ 3 xyz ³ -2 ⇒ xyz ≥ −8
⇒ a = −2, b = −1 ⇒ (a, b) ≡ (−2, −1) Hence, the correct answer is option (C).
Hence, the correct answer is option (C).
24. The set of all values of l for which the system of linear 0 1
26. If A , then which one of the following statements is
equations 1 0
2x1 − 2x2 + x3 = lx1
not correct?
2x1 − 3x2 + 2x3 = lx2
(A) A4 − I = A2 + I (B) A3 − I = A(A − I)
−x1 + 2x2 = lx3 (C) A + I = A(A − I)
2 2 (D) A3 + I = A(A3 − I)
has a non-trivial solution [JEE MAIN 2015 (ONLINE SET 1)]
(A) is a singleton. Solution:
(B) contains two elements.
(C) contains more than two elements. 0 1 0 1 0 1 1 0
A A2 ⇒ A2 I
(D) is an empty set. 1 0 ⇒
1 0
1 0
0 1
[JEE MAIN 2015 (OFFLINE)] Statement (A): A4 − I = (−I)2 − I = I − I = 0 and A2 + I = 0. So, option
Solution: The system of linear equation (A) is correct.
2x1 − 2x2 + x3 = lx1 Statement (B): A3 − I = A⋅A2 − I = A(−I) − I = − A − I and A(A − I) =
A2 − AI = −I − A. So, option (B) is correct.
2x1 − 3x2 + 2x3 = lx2
Statement (C): A2 + I = 0 and A(A2 − I) = A(−I − I) = −2AI = −2A. So,
−x1 + 2x2 = lx3
option (C) is incorrect.
has a non-trivial solution if
Statement (D): A3 + I = A⋅A2 + I = A(−I) + I= −A + I and A(A3 − I) =
2−l −2 1 A(−A − I) = −A2 − A= −(−I) − A = I − A. So, option (D)
2 −3 − l 2 = 0 is correct.
−1 2 −l Hence, the correct answer is option (C).
⇒ (2 − l ) [(3l + l2) − 4] + 2(−2l + 2) + 1(4 − 3 − l ) = 0 27. If A is a 3 × 3 matrix such that |5 adj A| = 5, then |A| is equal to
1 1
⇒ l + l − 5l + 3 = 0 ⇒ l = 1, 1, 3
3 2
(A) ± (B) ± 5 (C) ± 1 (D) ±
5 25
Therefore, l ∈{1, 3}.
Hence, the correct answer is option (B). [JEE MAIN 2015 (ONLINE SET 2)]
Chapter 18 | Matrices and Determinants 767
Statement-1: If k ≠ 3, then Δ = 0 and Δ x , Δ y , Δ z ≠ 0. Solution: The six matrices A for which A = 0 are
Hence, the system of equation has no solution.
Therefore Statement-1 is true. 0 0 1
Statement-2 is true and is a correct explanation of Statement-1. 0 0 1 inconsistent
Hence, the correct answer is option (A). 1 1 1
Paragraph for Questions 2 to 4: Let be the set of all 3 × 3 0 1 0
symmetric matrices all of whose entries are either 0 or 1. Five of
1 1 1 incconsistent
these entries are 1 and 4 of them are 0.
0 1 0
[IIT-JEE 2009]
2. The number of matrices in is 1 1 1
(A) 12 (B) 6 (C) 9 (D) 3 1 0 0 infinite solutions
Solution: If two zeros are the entries in the diagonal, then 1 0 0
3
C2 × 3C1. 1 1 0
If all the entries in the principle diagonal is 1, then 3C1. 1 1 0 inconsistent
So total matrices = 12. 0 0 1
Hence, the correct answer is option (A).
1 0 1
3. The number of matrices A in for which the system of linear
x 1 0 1 0 inconsistent
1 0 1
equations A y 0 has a unique solution is
1 0 0
z 0
0 1 1 inffinitesolutions
(A) less than 4 (B) at least 4 but less than 7
0 1 1
(C) at least 7 but less than 10 (D) at least 10
Hence, the correct answer is option (B).
Solution: Let
0 a b 5. The number of 3 × 3 matrices A whose entries are either 0
x 1
A = a 0 c
b c 1 or 1 and for which the system A y 0 has exactly two
z 0
For unique solution, A ≠ 0 . distinct solutions is
Either b = 0 or c = 0 Þ A ¹ 0 ⇒ 2 matrices (A) 0 (B) 29 − 1 (C) 168 (D) 2
[IIT-JEE 2010]
0 a b
Solution: Let
a 1 c either a = 0 or c = 0 Þ A ¹ 0 Þ 2 matrices a1 b1 c1
b c 0 A a2 b2 c2
1 a b a3 b3 c3
a 0 c either a = 0 or b = 0 Þ A ¹ 0 Þ 2 matrices
where ai, bi, ci for i = 1, 2, 3 have values 0 or 1. Then the given sys-
b c 0 tem is equivalent to
1 a b a1x + b1y + c1z = 0
a2x + b2y + c2z = 0
a 1 c a3x + b3y + c3z = 0
b c 1
which represents three distinct planes.
If a = b = 0 Þ A = 0 However, three planes cannot intersect at two distinct points.
If a = c = 0 Þ A = 0 Therefore, the number of such 3 × 3 matrices will be zero.
Hence, the correct answer is option (A).
If b = c = 0 Þ A = 0
Paragraph for Questions 6 to 8: Let p be an odd prime number
So, there will be only 6 matrices. and Tp be the following set of 2 × 2 matrices:
Hence, the correct answer is option (B). a b
Tp A : a, b , c {0 ,1,… , p 1}
4. The number of matrices A in for which the system of linear c d
x 1 [IIT-JEE 2010]
equations A y 0 is inconsistent is 6. The number of A in Tp such that A is either symmetric or
skew-symmetric or both, and det(A) divisible by p is
z 0
(A) (p − 1)2 (B) 2(p − 1)
(A) 0 (B) more than 2 (C) 2 (D) 1 (C) (p − 1)2 + 1 (D) 2p − 1
770 Mathematics Problem Book for JEE
Solution: As tr(A) is not divisible by p ⇒ a ≠ 0. 11. Let w ¹ 1 be a cube root of unity and S be the set of all non-
det(A) is divisible by p ⇒ a2 − bc is divisible by p. é 1 a bù
The number of ways of selection of a, b and c is ê ú
singular matrices of the form ê w 1 c ú , where each of a,
( p - 1)[( p - 1) ´ 1] = ( p - 1)2 ê 2 ú
ëw w 1û
Hence, the correct answer is option (C). b, and c is either w or w 2. Then, the number of distinct matrices
8. The number of A in Tp such that det (A) is not divisible by p is in the set S is
(A) 2p2 (B) p3 − 5p (C) p3 − 3p (D) p3 − p2 (A) 2 (B) 6 (C) 4 (D) 8
Solution: The total number of A = p ´ p ´ p = p . 3 [IIT-JEE 2011]
The number of A such that det(A) is divisible by p equals Solution: For being non-singular
( p 1)2 number of A in which a = 0 é 1 a bù
ê ú
= ( p - 1) + p + p - 1
2 êw 1 cú ¹ 0
ê 2 ú
= p2 ëw w 1û
Þ acw 2 - (a + c )w + 1 ¹ 0 Þ (aw - 1)(cw - 1) ¹ 0
The required number is p3 − p2 .
Hence, the correct answer is option (D). Þ a ¹ w 2 and c ¹ w 2 Þ a = w and c = w and b = w or w 2
9. Let k be a positive real number and let Hence, the number of possible triplets of (a, b, c) is 2, that is,
(w, w 2, w ) and (w, w, w ).
2k 1 2 k 2 k
Hence, the correct answer is option (A).
A 2 k 1 2k 12. Let M be a 3 × 3 matrix satisfying
2 k 2k 1 é0 ù é -1ù é1ù é1ù é1ù é 0 ù
0 M êê 1úú = êê 2 úú , M êê -1úú = êê 1 úú and M êê1úú = êê 0 úú
2k 1 k
êë0 úû êë 3 úû êë 0 úû êë -1úû êë1úû êë12 úû
and B 1 2k 0 2 k .
k 2 k 0 Then, the sum of the diagonal entries of M is _____.
[IIT-JEE 2011]
If det(adj A) + det(adj B) = 106, then [k] is equal to ⎯⎯ . Solution: Let
[Note: adj M denotes the adjoint of a square matrix M and [k] a b c
denotes the largest integer less than or equal to k].
M d e f
[IIT-JEE 2010]
g h i
Chapter 18 | Matrices and Determinants 771
0 1 1 4 4
M 1 2 b 1, e 2, h 3
15. If the adjoint of a 3 × 3 matrix P is 2 1 7 , then the pos-
0 3 1 1 3
1 1 sible value(s) of the determinant of P is (are)
(A) −2 (B) −1 (C) 1 (D) 2
M 1 1 a 0 , d 3, g 2
[IIT-JEE 2012]
0 1
Solution:
1 0 2 n -1
adj P = P as ( adj ( P ) = P )
M 1 0 g h i 12 i 7
adjP = 1(3 − 7) − 4(6 − 7) + 4(2 − 1) = 4
1 12
Hence, P 2 or 2
Therefore, the sum of diagonal elements = 9.
Hence, the correct answers are options (A) and (D).
Hence, the correct answer is (9).
16. For 3 × 3 matrices M and N, which of the following statement(s)
13. Let P = [aij] be a 3 × 3 matrix and let Q = [bij], where is (are) NOT correct?
(A) NTMN is symmetric or skew-symmetric, according to M is
bij = 2i + jaij for 1 ≤ i, j ≤ 3. If the determinant of P is 2, then the
symmetric or skew-symmetric
determinant of the matrix Q is
(B) MN − NM is skew-symmetric for all symmetric matrices M
(A) 210 (B) 211 (C) 212 (D) 213 and N
[IIT-JEE 2012] (C) MN is symmetric for all symmetric matrices M and N
Solution: (D) (adj M)·(adj N) = adj(MN) for all invertible matrices M
and N
22 a11 23 a12 24 a13 [JEE ADVANCED 2013]
Q 23 a21 24 a22 25 a23 Solution: We have
4
2 a31 2 a325
26 a33 (NTMN)T = −NTMT(NT)T = NTMTN
(A) If M is skew symmetric, then (NTMN)T = −NTMN. Therefore,
a11 a12 a13
it is concluded that it is skew-symmetric.
Q 2 2 2 2a21
2 3 4
2a22 2a23 If M is symmetric, then (MTMN)T = NTMN. Therefore, it
22 a31 22 a32 22 a33 is concluded that it is symmetric. Hence, option (A) is
correct.
a11 a12 a13
(B) We have
Q 2 22 a21 a22
9 2
a23
( MN - NM )T = ( MN )T - (NM )T
a31 a32 a33
= N T MT - MT N T
Q 212 P
= -( MT MT - N T MT )
Q 213
= -( MN - NM )
Hence, the correct answer is option (D). Therefore, it is concluded that it is skew-symmetric and
14. If P is a 3 × 3 matrix such that PT = 2P + I, where PT is the trans- hence option (B) is correct.
pose of P and I is the 3 × 3 identity matrix, then there exists a (C) (MN)T = NTMT. Symmetricity and skew-symmetricity depend
x 0 on the nature of M and N; therefore, option (C) is incorrect.
column matrix X y 0 such that (D) adj(MN) = adj(N) adj M; therefore, option (D) is incorrect.
z 0 Hence, the correct answers are options (C) and (D).
0
17. Let w be a complex cube root of unity with w ≠ 1 and P = [pij]
(A) PX 0 (B) PX = X (C) PX = 2X (D) PX = −X
be an n × n matrix with pij = w i+j. Then, P2 ≠ 0, when n = ?
0
[IIT-JEE 2012] (A) 57 (B) 55 (C) 58 (D) 56
[JEE ADVANCED 2013]
Solution: Given
P T = 2P + I Solution: We have P [ Pij ]nn , P 2 ≠ 0. Now
ˆ ˆ
Þ P = 2P T + I = 2(2P + I ) + I Pij = w i + j
ÞP+I =0 ⎡w 2 1 w w 2 1 ... ⎤
Þ PX + X = 0 ⎢ ⎥
⎢ 1 w w2 1 w ... ⎥
PX X P=⎢ ⎥
⎢w w
2
1 w w2 ... ⎥
Hence, the correct answer is option (D). ⎢ ... ... ... ... ... ... ⎥⎦ n×n
⎣
772 Mathematics Problem Book for JEE
⇒ 2(2a 2 )[(2 + 3a ) - (2 + 5a )] = -648a 23. The total number of distinct x ∈R for which
⇒ −8a 3 = −648a x x2 1+ x 2
⇒ a 2 = 81 or a = 0 2x 4 x2 1+ 8 x 3 = 10 is _________.
⇒ a = 0 or ± 9
3x 9 x 2 1+ 27 x 3
Hence, the correct answers are options (B) and (C).
[JEE ADVANCED 2016]
3 1 2
Solution: It is given that
22. Let P 2 0 a , where a ∈R. Suppose Q = [qij] is a
3 5 0 x x2 1+ x 3
matrix such that PQ = kI, where k ∈R, k ≠ 0 and I is the 2 x (2 x )2 1+ (2 x )3 = 10
k k2 3 x (3 x )2 1+ (3 x )3
identity matrix of order 3. If q23 = - and det(Q) = , then
8 2
(A) a = 0, k = 8 (B) 4a − k + 8 = 0 x x2 1 x x2 x3
(C) det(P adj (Q)) = 29 (D) det(Q adj (P)) = 213 ⇒ 2x 4 x2 1 + 2x 4 x2 8 x 3 = 10
[JEE ADVANCED 2016] 3x 9x2 1 3x 9x2 27 x 3
Solution: It is given that
3 1 2 1 1 1 1 1 1
P 2 0 a and Q = [qij] ⇒ x 3 2 22 1 + x 6 2 22 23 = 10
3 5 0 3 32 1 3 32 33
Now,
3 -2 1 1 1 1 1 1
C32 = - = −(3a + 4)
2 a ⇒x 2 2 3 2
1 + 6x 2 2
6 2
23 = 10
Here, |P| = 12a + 20, 3 32 1 3 32 33
PQ = kI ⇒ Q = kIP−1
Now, |P| |Q| = k3. Therefore, 1 1 1
Þ x (1+ 6 x ) 1 2 4 = 10
3 3
k2
(12a + 20 ) = k3 1 3 9
2
774 Mathematics Problem Book for JEE
[JEE ADVANCED 2016] (A) Unit matrix (B) Null matrix (C) A (D) −A
Chapter 18 | Matrices and Determinants 775
a1 b1 c1 1 3
33. Let w = - + i . Then, the value of the determinant
23. If a2 b2 c2 ≠ 0, then the number of solutions of the 2 2
a3 b3 c3 1 1 1
system of equations a1x + b1y + c1z = 0, a2x + b2y + c2z = 0 and
a3x + b3y + c3z = 0 is 1 -1- w 2 w 2 is
1 w2 w4
(A) Infinite number of solutions
(B) Only one unique solution (A) 3w (B) 3w (w− 1) (C) 3w 2 (D) 3w (1 − w)
(C) More than one solution
(D) None of these
24. Let A and B be two square matrices of the same dimension sin 2 x e x sin x + x cos x sin x + x 2 cos x
and let [A, B] = AB − BA. Then for three 2 × 2 matrices A, B, C, 34. If Δ(x) = cos x + sin x ex + x 1+ x 2 , then
[[A, B], C] + [[B, C], A] + [[C, A], B] is equal to x 2x x
(A) 1 (B) 0 e cos x e e
(C) ABC − CBA (D) None of these
25. If the matrices A, B, (A + B) are non-singular, then (A) Δ′(0) = 0 (B) Δ = 0
2
[ A( A + B )-1B ]-1 is equal to
(C) Δ = 0 (D) All the above
(A) A + B (B) A−1 + B−1 4
(C) (A + B) −1 (D) None of these
f ( x + a ) f ( x + 2a ) f ( x + 3a )
26. If A and B matrices commute then 35. Let g(x) = f (a ) f (2a ) f (3a ) , where a is a
(A) A−1 and B also commute
f ′(a ) f ′(2a ) f ′(3a )
(B) B−1 and A also commute
(C) A−1 and B−1 also commute g( x )
(D) All the above constant. Then lim is equal to
x ®0 x
27. If A, B and C are three matrices conformable for multiplication, (A) 0 (B) 1 (C) −1 (D) None of these
then (ACB) −1 is equal to
(A) A−1B−1C−1 (B) B−1C−1A−1 4x - 4 ( x - 2)
2
x 3
N
39. If f(x) = 1 2 cos x 1 , then ò f ( x ) dx equals
∑ Δr
0
numbers, then is equal to 0 1 2 cos x
r =1
(A) N (B) N2 (C) Zero (D) None of these (A) 1/4 (B) 1/3 (C) 1/2 (D) 1
Chapter 18 | Matrices and Determinants 777
2
x cos x ex B B A
p /2
(C) B A B (D) None of these
40. If f(x) = ò
2
sin x x sec x , then the value of f ( x ) dx
tan x 1 2 -p / 2 A B B
50. If A, B, C are angles of a triangle ABC, then the value of the
is equal to A B C
(A) 0 (B) 1 (C) 2 (D) None of these sin sin sin
2 2 2
determinant
x + x +3
2
1 4 sin( A + B + C ) sin
B
cos
A
41. If the expression 2 x + x + 2 x + 1 2 3 is equal to
4 3 2 2
(A + B + C) C
x2 + x 1 1 cos tan( A + B + C ) sin
2 2
ax4 + bx3 + cx2 + dx + e, then the value of e is equal to is less than or equal to
(A) 1/2 (B) 1/4 (C) 1/8 (D) None of these
(A) zero (B) 1 (C) 2 (D) None of these
x 2 5
a b aa + b
51. The sum of two non-integral roots of 3 x 3 = 0 is
42. The determinant D = b c ba + c is equal to
5 4 x
aa + b ba + c 0
zero if (A) 5 (B) −5 (C) −18 (D) None of these
(A) a, b, c are in AP (B) a, b, c are in GP
1 1 1
(C) a, b, c are in HP (D) None of these
52. The value of Δ = (2 x + 2- x )2 (3 x + 3- x )2 (5 x + 5- x )2 is
43. If a, b, g are the roots of the equation x3 + px + q = 0 (where p -x 2 -x 2 -x 2
(2 - 2 )
x
(3 - 3 )
x
(5 - 5 )
x
a b g
≠ 0, q ≠ 0), then the value of the determinant b g a is (A) 0 (B) 30x (C) 30−x (D) None of these
g a b 53. There are three points (a, x), (b, y) and (c, z) such that the
(A) p (B) q (C) p2 − 2q (D) None of these straight lines joining any two of them are not equally
inclined to the coordinate axes where a, b, c, x, y, z ∈ R. If
44. The number of values of k for which the system of equations x +a y +b z +c
(k + 1)x + 8y = 4k, kx + (k + 3)y = 3k − 1 has infinitely many y
y + b z + c x + a = 0 and a + c = −b, then x, − , z are in
solutions is 2
(A) 0 (B) 1 (C) 2 (D) infinite z +c x +a y +b
(A) AP (B) GP (C) HP (D) None of these
a2 (1+ x ) ab ac
n -2
45. The determinant D = ab b2 (1+ x ) bc is
å1 n(n - 1) n2
divisible by ac bc c 2 (1+ x ) k =0
n
(A) 1 + x (B) (1 + x)2 (C) x2 (D) None of these 54. If å1 (n + 1)(n - 1) n( n + 1) = 72, then n is equal to
k =1
46. If the system of equations ax + y + z = 0, x + by + z = 0 and n -1
x + y + cz = 0 (a, b, c ≠ 1) has a non-trivial solution, then the
1 1 1
å1 n2 + 1 n2
k =1
value of + + is
1- a 1- b 1- c
(A) 6 (B) 9 (C) 8 (D) None of these
(A) −1 (B) 0 (C) 1 (D) None of these
47. If the system of equations x + ay = 0, az + y = 0 and ax + z = 0 1+ a2 + a 4 1+ ab + a2b2 1+ ac + a2c 2
has infinite solutions, then the value of a is 55. D = 1+ ab + a2b2 1+ b 2 + b 4 1+ bc + b2c 2 equals
(A) −1 (B) 1 (C) 0 (D) No real values 1+ ac + a c
2 2
1+ bc + b c 2 2
1+ c + c
2 4
ln x x 0 é2 3 ù −1
ln x 69. If A = ê ú , then A is equal to
58. If f(x) = , then 1/ x 1 x is ë 5 -2 û
x 1 1
(A) A′ (B) 2A (C) A (D) A
-1/ x 2 0 2 2 19
(A) x3f ′(x) (B) x 2f ¢¢¢( x ) 70. The inverse of a skew-symmetric matrix is
(C) x 3f ¢¢¢( x ) (D) None of these (A) A symmetric matrix if it exists
59. If the system of equations lx + (b − a)y + (c − a)z = 0, (a − b)x (B) A skew-symmetric matrix if it exists
+ ly + (c − b)z = 0 and (a − c)x + (b − c)y + lz = 0 has a (C) Transpose of the original matrix
non-trivial solution, then the value of λ is (D) May not exist
(A) l = 0 (B) l = 1 (C) l = −1 (D) None of these é1 2 3 ù
60. If A′ is the transpose of a square matrix A, then 71. If A = êê1 3 5 úú , then adj (adj A) is
(A) |A| ≠ |A′| (B) |A| = |A′| êë1 5 12 úû
(C) |A| + |A′| = 0 (D) |A| = |A′| only when A is
é3 3 3 ù é1 2 3 ù
symmetric
(A) ê6 9 15 ú (B) ê1 3 5 ú
é cosq sinq ù ê ú ê ú
61. If I = éê
1 0ù é 0 1ù and B = ê êë9 15 36 úû êë1 5 12 úû
,J=
cosq úû
, then B
ë 0 1úû ê -1 0 ú
ë û
ë - sinq
é3 6 9 ù
equals
(C) ê3 9 15 ú (D) None of these
(A) I cos θ + J sin θ (B) I sin θ + J cos θ ê ú
êë3 15 36 úû
(C) I cos θ − J sin θ (D) − I cos θ + J sinθ
72. Let A be a square matrix of order 3 such that transpose of
62. If In is the identity matrix of order n, then (In) −1 inverse of A is A itself. Then |adj (adj A)| is equal to
(A) does not exist (B) is equal to In (A) 9 (B) 27 (C) 4 (D) 1
(C) equals O (D) nIn éa aù
63. If for a matrix A, A2 + I = O where I is the identity matrix, then 73. If A = êê b b úú , then AAT is
A equals êë g c úû
⎡ 1 0⎤ ⎡ i 0⎤ ⎡ 1 2⎤ ⎡ −1 0 ⎤
(A) ⎢ (A) A non-singular matrix (B) A singular matrix
⎥ (B) ⎢0 i ⎥
(C) ⎢ ⎥ (D) ⎢ ⎥
⎣0 1⎦ ⎣ ⎦ ⎣ −1 1⎦ ⎣ 0 −1⎦ (C) An identity matrix (D) None of these
64. The number of non-zero diagonal matrices of order 4 satisfying 74. If A and B are two non-singular square matrices of the same
A2 = A is order, the adjoint of AB is equal to
(A) 2 (B) 4 (C) 16 (D) 15 (A) (adj A) (adj B) (B) (adj B) (adj A)
65. If A and B are symmetric matrices of order n (A ≠ B), then (C) adj(BA) (D) adj A + adj B
(A) A + B is skew-symmetric (B) A + B is symmetric 75. If Ak = 0 (null matrix) for some positive integral value of k,
(C) A + B is a diagonal matrix (D) A + B is a zero matrix then I + A + A2 + … + Ak−1 is equal to
(A) Null matrix (B) (I + A)k
c -b ù é a2 ab ac ù
é0 ê ú (C) (I − A) −1 (D) None of these
66. If A = êê -c a úú and B = êab b bc ú , then AB is equal
2
0
ê 2ú
76. The matrix X for which ⎡ 1 − 4 ⎤ X = ⎡ −16 −6 ⎤ is
êë b -a 0 úû êë ac bc c úû ⎢3 −2 ⎥ ⎢ 7 2 ⎥⎦
⎣ ⎦ ⎣
to
é 1 2ù
(A) A3 (B) B2 (C) O (D) I ê- ú
(A) é -2 4 ù (B) ê 5 5 ú
-1 2 ê -3 1ú ê- 3 1 ú
é1/ 25 0 ù ìï é 5 0 ù üï ë û
If ê = í ý , then the value of x is êë 10 5 úû
1/ 25úû ï êë -a 5 úû ï
67.
ë x î þ é 6 2ù é -16 -6 ù
(C) ê ú (D) ê
(A) a/125 (B) 2a/25 (C) 2a/125 (D) None of these ê11 2 ú ë 7 2 úû
êë 2 úû
Chapter 18 | Matrices and Determinants 779
éa 0 ù é1 0ù 85. The system of equations 102x + 95y + 88z = 81, 3x + 10y + 17z
77. If A = ê ú and B = ê5 1ú , then the value of a for which = 24, 57x + 50y + 43z = 36 has
ë 1 1 û ë û
(A) Many solutions (B) No solution
A2 = B is
(A) 1 (B) −1 (C) 4 (D) No real values (C) A unique solution (D) None of these.
é1 0 0 ù x2 ( x - 1)2 ( x - 2)2
78. If A = êê0 1 0 úú , then the trace of the matrix A is equal to 86. If ax3 + bx2 + cx + d = ( x - 1)2 ( x - 2)2 ( x - 3)2 , then
êëa b -1úû
( x - 2) 2
( x - 3)2
( x - 4 )2
(A) 1 (B) −1 (C) 0 (D) a + b
79. Let A and B are the non-singular square matrices, then which (A) a = 1, b = 2, c = 3, d = −8
of the following is always correct? (B) a = −1, b = 2, c = 3, d = −8
(A) (AB)θ = Aθ Bθ (B) (AB)′ = B′A′ (C) a = 0, b = 0, c = 0, d = 8
(C) A(adj B) = B (adj A) (D) |adj A| = |A|n−2 (D) a = 0, b = 0, c = 0, d = −8
80. If lr2 + mr2 + nr2 = 1 , where r = 1, 2, 3 and l1l2 + m1m2 + n1n2 = 0 1 sin A sin2 A
2
l1 m1 n1 87. If in a ΔABC, 1 sin B sin2 B = 0, then the triangle is
…etc, then, D = l2 m2 n2 is equal to 1 sin C sin2 C
l3 m3 n3
(A) Equilateral or isosceles
(A) −1 (B) 1 (C) ±1 (D) 3
(B) Equilateral or right angled
81. If a is a root of x4 = 1 with negative principal argument, (C) Right angled or isosceles
then the principal argument of Δ(a) where
(D) None of these
1 1 1 88. If a, b and c are sides of a ΔABC and
Δ(a) = a n a n +1 a n + 3 is
a2 b2 c2
1 1
0 (a + 1)2 (b + 1)2 (c + 1)2 = 0, then
a n +1 a n
(a - 1)2 (b - 1)2 (c - 1)2
5p 3p p p
(A) (B) − (C) (D) −
4 4 4 4
(A) ABC is an equilateral triangle
x a a (B) ABC is a right angled triangle
82. If a, b > 0 and Δ(x) = b x a , then (C) ABC is an Isosceles triangle
b b x (D) None of these
(A) Δ(x) is increasing in ( - ab , ab ) 10
C4 10
C5 11
Cm
(B) Δ(x) is decreasing in ( ab , ∞ ) 11 11 12
89. The value of C6 C7 Cm+2 is zero, when m is
(C) Δ(x) has a local minimum at x = ab 12 12 13
(D) None of these C8 C9 Cm+ 4
Answer Key
Practice Exercise 1
1. (A) 2. (D) 3. (A) 4. (A) 5. (A) 6. (A)
7. (C) 8. (B) 9. (A) 10. (D) 11. (C) 12. (C)
13. (C) 14. (C) 15. (B) 16. (B) 17. (A) 18. (C)
Chapter 18 | Matrices and Determinants 783
19. (A) 20. (B) 21. (A) 22. (D) 23. (B) 24. (B)
25. (D) 26. (C) 27. (B) 28. (A) 29. (C) 30. (D)
31. (C) 32. (C) 33. (B) 34. (D) 35. (A) 36. (D)
37. (B) 38. (D) 39. (B) 40. (A) 41. (A) 42. (B)
43. (D) 44. (B) 45. (C) 46. (C) 47. (A) 48. (A)
49. (A) 50. (C) 51. (B) 52. (A) 53. (A) 54. (C)
55. (B) 56. (C) 57. (C) 58. (A) 59. (A) 60. (B)
61. (A) 62. (B) 63. (B) 64. (D) 65. (B) 66. (C)
67. (C) 68. (C) 69. (D) 70. (B) 71. (A) 72. (D)
73. (B) 74. (B) 75. (C) 76. (C) 77. (D) 78. (A)
79. (B) 80. (B) 81. (B) 82. (C) 83. (C) 84. (B)
85. (A) 86. (D) 87. (A) 88. (C) 89. (C) 90. (D)
91. (A) 92. (A) 93. (D) 94. (B) 95. (C) 96. (B)
97. (D) 98. (A)
Practice Exercise 2
1. (A), (C), (D) 2. (A), (B), (C), (D) 3. (A), (D) 4. (A), (B), (D) 5. (A), (B), (C), (D) 6. (A), (B), (C)
7. (B), (C), (D) 8. (A), (B), (C), (D) 9. (A), (D) 10. (A) 11. (B) 12. (A)
13. (A) 14. (B) 15. (D) 16. (C) 17. (D) 18. (C)
19. (A) 20. (B) 21. (A) 22. (A) → (q), (B) → (s, t), (C) → (r), (D) → (p)
23. (A) → (p, t), (B) → (q), (C) → (r), (D) → (p, t) 24. 1 25. 1 26. 2, 4
27. 7 28. 0 29. 150 30. −1 31. 4
Solutions
Practice Exercise 1 é3ù
é7 1 2 ù ê ú é4 ù é21+ 4 + 10 ù é 8 ù é 35 ù é 8 ù é 43 ù
6. ê ú ê4 ú + 2 ê2 ú = ê 27 + 8 + 5 ú + ê 4 ú = ê 40 ú + ê 4 ú = ê 44 ú
é 1 0 0 ù é 1 0 0 ù é 1 0 0ù ë 9 2 1û ê5ú ë û ë û ë û ë û ë û ë û
1. A = êê0 1 0 úú êê0 1 0 úú = êê0 1 0 úú = I (Unit matrix)
2 ë û
êëa b -1úû êëa b -1úû êë0 0 1úû é10 0 ù é 1 0ù
7. A (adj A) = ê ú = 10 ê0 1ú =10I. Also A(adj A) = |A|I.
ë 0 10 û ë û
2. A adjA = A A 2 = A 3 = (a3 )3 = a9
Hence, |A| = 10.
3. A = 0 as the determinant of any skew-symmetric matrix of 8. Let A be symmetric as well as skew-symmetric matrix. Then
odd order is zero. A¢ = A and A¢ = - A
4. If we consider m = 2 ⇒ A = −A or 2A = 0 or A = 0
(A + B)2 = (A + B) (A + B) = A2 + AB + BA + B2 9. AB = 0 ⇒ AB = 0 ⇒ A B = 0 ⇒ either A = 0 or B = 0
= A2 + 2AB + B2 (if AB = BA)
Similarly é0 0 0 ù
ê ú
(A + B)m = mC Am + mC m−1B + mC Am−2B2 + … + mC m 10. A = ê 1 0 0 ú
0 1A 2 mB
êë0 1 0 úû
holds if AB = BA.
é0 0 0 ù é0 0 0 ù é0 0 0 ù
5. We know that [aij ]¢ = -[aij ] for skew-symmetric matrix. Then
A = êê 1 0 0 úú êê 1 0 0 úú = ê0 0 0 ú
2
ê ú
[aji] + [aij] = 0 êë0 1 0 úû êë0 1 0 úû êë 1 0 0 úû
⇒aji + aij = 0 é0 0 0 ù é0 0 0 ù é0 0 0 ù
For i = j, we get A3 = ê 1 0 0 ú ê0 0 0 ú = ê0 0 0 ú
aii + aii = 0 ⇒ aii = 0 ê úê ú ê ú
êë0 1 0 úû êë 1 0 0 úû êë0 0 0 úû
This means diagonal elements of every skew-symmetric
matrix are zero. Hence, A3 = 0.
784 Mathematics Problem Book for JEE
é0 0 0 ù é 7 -1 -1ù é 7 -3 -3ù
11. A = êê0 0 0 úú C A = êê -3 1 0 úú ⇒ adj A = ê -1 1 0 ú
ê ú
êë0 1 0 úû ëê -3 0 1 úû êë -1 0 1 úû
Since A2 = 0, hence, A is a nilpotent matrix with index 2.
Therefore,
12. A and B are symmetric matrices of same order, i.e. A = A′, B = B′. é 7 -3 -3ù
adj A = êê -1 1 0 úú
1
Then A-1 =
(AB + BA)T = (AB)T + (BA)T A
êë -1 0 1 úû
= BT × AT + AT × BT = BA + AB = AB + BA
éi -i ù é 1 -1ù
So, AB + BA is a symmetric matrix. 21. A = ê and B = ê
ë -i i úû ú
ë -1 1 û
13. A is a square matrix
(AA′)′ = (A′)′⋅A′ = AA′ éi -i ù é i -i ù é -2 2 ù é 1 -1ù
A2 = ê =ê = -2 ê ú = -2B
i úû êë -i ú ú
Hence, AA′ is a symmetric matrix.
ë -i i û ë 2 -2 û ë -1 1 û
14. A is a square matrix and A3 = I ⇒ A3 = I ⇒ A2 × A = I. So A2 is
the inverse of A ⎡ 1 −1⎤ ⎡ 1 −1⎤ ⎡ 2 −2⎤ ⎡ 1 −1⎤
A4 = 4 B 2 = 4 ⎢ ⎥ ⎢ ⎥ = 4⎢ ⎥ = 8⎢ ⎥ = 8B
Hence, A−1 = A2. ⎣ −1 1 ⎦ ⎣ −1 1 ⎦ ⎣ −2 2 ⎦ ⎣ −1 1 ⎦
15. A is a square matrix é 2 -2 ù é 1 -1ù
(A − A′)′ = A′ − (A′)′ = A′ − A A8 = 64 B 2 = 64 ê ú = 128 ê ú = 128B
So, A − A′ is not a symmetric matrix. ë -2 2 û ë -1 1 û
16. Because determinant of every skew-symmetric matrix of odd 22. Because cos(−θ) = cos θ ⇒ A is a symmetric matrix.
Alternative solution:
order is zero, therefore, A = 0 if n is odd.
Given determinant can also be written as the product of two
17. Each diagonal element of skew-symmetric matrix is zero. determinants as follows:
é 4 2ù 1 cos(a - b ) cos(a - g )
18. A = ê ú
ë -1 1û cos( b - a ) 1 cos( b - g )
cos(g - a ) cos(g - b ) 1
é 4 2 ù + é -2 0 ù é 2 2 ù
A − 2I = ê ú ê ú=ê ú cos a sina 0 cos a sina
ë -1 1û ë 0 -2 û ë -1 -1û 0
= cos b sin b 0 ´ cos b sin b 0 (row by row)
é 4 2 ù + é -3 0 ù é 1 2 ù
A − 3I = ê ú ê ú=ê ú cos g sin g 0 cos g sing 0
ë -1 1û ë 0 -3û ë -1 -2 û
=0×0=0
Hence,
1 a c
é 2 2 ù é 1 2 ù é0 0 ù
(A − 2I) (A − 3I) = ê ú=ê ú =ê ú Also A = a 1 b where a = cos (a − b ), b = cos (b − g ),
ë -1 -1û ë -1 -2 û ë0 0 û b c 1
19. O(A) = m × (n + 5) and O(B) = m × (11 − n) c = cos (g − a)
AB exists ⇒ n + 5 = m ⇒ m − n = 5 (1) = 1 − a2 − b2 − c2 + 2abc
BA exists ⇒ 11 − n = m ⇒ m + n = 11 (2) = 1 − [cos2 (a − b ) + cos2 (b − g ) + cos2 (g − a) − 2 cos (a − b)
Solving Eqs. (1) and (2), we have cos (b − g ) cos (g − a)]
m = 8 and n = 3
= 1 − [1+ cos2 (a − b) − sin2 (b − g ) + cos (g − a){cos (g − a)
Therefore, O(A) = 8 × 8 and O(B) = 8 × 8 − 2 cos (a − b ) cos (b − g )}]
Therefore, AB and BA both are square matrices of order
= 1 − [1 + cos (a − g ) cos (a − 2β + g ) + cos (g − a){cos (g − a)
8 × 8.
− cos (a − g ) cos (a − 2b + g )}]
é1 3 3 ù = 1 − [1 + cos (a − g ) cos (a − 2b + g ) − cos (g − a ) cos (a − 2b + g )]
20. A = êê1 4 3 úú =0
êë1 3 4 úû
23. If A ≠ 0 , then homogeneous system of linear equations
1 3 3 AX = 0 has only trivial solution, i.e. X = 0.
A= 1 4 3
24. [[A, B], C] + [[B,C], A] + [[C,A], B]
1 3 4
= [AB − BA, C] + [BC − CB, A] + [CA − AC, B]
= 1(16 − 9) − 3(4 − 3) + 3(3 − 4)
=7−3−3=1 = (AB − BA)C − C(AB − BA) + (BC − CB)A − A(BC − CB) + (CA − AC) B
Chapter 18 | Matrices and Determinants 785
− B(CA − AC) 34. We can write Δ(x) as product of two determinants as follows:
= ABC - BAC - CAB + CBA + BCA - CBA - ABC sin x cos x 1 cos x sin x 0
+ ACB + CAB - ACB - BCA + BAC = 0 Δ (x) = 1 1 1 ´ ex x 0 =0
ex 0 0 1 x2 0
25. [A(A + B)−1B]−1 = B−1 (A + B) A−1 which cannot be simplified
further, in general. which is independent of (x)⇒Δ′ (x) = 0 ∀ x.
26. A and B matrices commute so AB = BA. g( x ) é0 ù
A−1 × B−1 = (BA)−1 = (AB)−1 = B−1 × A−1 35. lim ê 0 form as g(0 ) = 0 ú
x ®0 x ë û
So, A−1 and B−1 also commute. g¢( x )
lim = g¢(0 ) = 0
27. A, B, C are three conformable matrices for multiplication x ®01
(ACB)−1 = B−1×C−1×A−1. Hence, (A) is the correct answer.
⎡1 0 0 ⎤ 36. Note that Δ(x) is a polynomial of degree at most 6 in x.
⎢ ⎥ If Δ (x) = a0 + a1x + a2x2 + … + a6x6, then
28. A = ⎢0 1 0 ⎥
⎢⎣a b −1⎥⎦ Δ′(x) = a1 + 2a2x + … + 6a6x5 ⇒ a1 = Δ′(0).
Now
⎡ 1 0 0 ⎤ ⎡ 1 0 0 ⎤ ⎡ 1 0 0⎤
A = ⎢⎢0 1 0 ⎥⎥ ⎢⎢0 1 0 ⎥⎥ = ⎢⎢0 1 0 ⎥⎥
2 4 ( x - 2)2 x3
⎢⎣a b −1⎥⎦ ⎢⎣a b −1⎥⎦ ⎢⎣0 0 1⎥⎦ Δ′(x) = 8 ( x - 2 2 )2 ( x + 1)3
2
A is a unit matrix of third order. 12 ( x - 2 3 )2 ( x - 1)3
29. Trace of a skew-symmetric matrix is always equal to zero. Now 4x - 4 2( x - 2) x3
A = − A′ ⇒ ∑ aii = 0 + 8x - 4 2 2( x - 2 2 ) ( x + 1)3
p /2
1
39. Direct expansion gives f(x) = cos 3x. Hence, ò cos 3 xdx = - .
3
48. R1 → R1 −R2, R2 → R2 − R3
0
Hence, (B) is the correct answer. sin( A + B ) sin( A - B ) cot A - cot B 0
/2 Δ = sin(B + C ) sin(B - C ) cot B - cot C 0
40. f(−x) = −f(x). It is an odd function. Hence, f ( x ) 0.
sin2 C
/ 2 cot C 1
Hence, (A) is the correct answer. sin( B - A)
sin C sin( A - B ) 0
3 1 4 sin A × sin B
= sin(C - B )
41. Putting x = 0, we get e = 1 2 3 . sin A sin( B - C ) 0
sin B × sin C
0 1 1
sin2 C cot C 1
3 1 1
C3 → C3 − C1 gives 2 2 2 = 0 Expanding along the third column, we have
0 1 1 sin( A - B ) × sin(C - B ) sin(B - C ) × sin(B - A)
Δ= -
sin B sin B
42. By applying R3 → R3 − a R1 − R2, we get
sin( A - B )
( b2 − ac ) ( aa2 + 2ba + c) = 0 [ - sin(B - C ) + sin(B - C )] = 0
sin B
Hence, (B) is the correct answer.
49. We know that
43. Use concept of polynomial roots. Sum of the roots = 0. a c b a c b
By applying R1 → R1 + R2 + R3 and using a + b + g, we get Δ = 0. (a + b + c - 3abc ) = b a c b a c
3 3 3 2
1 a 0 1 1 1
=0
47. 0 1 a = 0 Þ 1+ a(a2 ) = 0 Þ a3 = -1 Þ a = -1 ⇒ 1 1 1
a 0 1 (2 x - 2- x )2 (3 x - 3- x )2 (5 x - 5- x )2
Hence, (A) is the correct answer. Hence, (A) is the correct answer.
Chapter 18 | Matrices and Determinants 787
Hence, (B) is the correct answer. 59. As the given system of equations has a non-trivial solution,
1 1 1 l b-a c -a
cot A /2 cot B /2 cot C /2 =0 a-b l c -b = 0
56.
tan B /2 + tan C /2 tan C /2 + tan A/2 tan A/2 + tan B /2 a-c b-c l
When λ = 0 then determinants become skew-symmetric
Operating C1 − C2 and C2 − C3 determinants of odd order, which is equal to zero. Thus, λ = 0.
0 0 1 60. A = A′. Then |A| = |A′| because the expansion of a determinant
tan B /2 - tan A/2 tan C /2 - tan B /2 row-wise is same as the expansion of a determinant column-
cot C /2 =0
tan A/2 tan B /2 tan B /2 tan C /2 wise.
tan B /2 - tan A /2 tan C /2 - tan B /2 tan A /2 + tan B /2
⎡ 1 0⎤ ⎡ 0 1⎤
61. I = ⎢ ⎥ , J = ⎢ −1 0 ⎥
⇒ (tan B/2 − tan A/2 )(tan C/2 − tan B/2) × ⎣ 0 1⎦ ⎣ ⎦
0 0 1 ⎡ cosq sinq ⎤ ⎡ 1 0 ⎤ ⎡ 0 1⎤
B= ⎢ ⎥ = ⎢ ⎥ cosq + ⎢ −1 0 ⎥ sinq
1 1
cot C /2 =0 ⎣ − sinq cosq ⎦ ⎣ 0 1⎦ ⎣ ⎦
tan A/2 tan B /2 tan B /2 tan C /2 = I cos q + J sin q
1 1 tan A/2 + tan B /2
62. In is an identity matrix.
Expanding along R1 we get
(tan B/2 − tan A/2 )(tan C/2 − tan B/2) (tan C/2 − tan A/2) = 0 ⎡ −1 0 ⎤
63. A2 + I = 0 ⇒ A2 = ⎢ ⎥ of (I is of second order) ⇒
⇒ A = B or B = C or C = A ⎣ 0 −1⎦
⇒ Δ must be an isosceles triangle i 0
0 i .
sin x cos x cos x
57. Let Δ = cos x sin x cos x 64. Each diagonal element is either 0 or 1. So number of matrices
cos x cos x sin x = 24 − 1.
788 Mathematics Problem Book for JEE
⎧⎪ ⎡ 5 0 ⎤ −1⎫⎪
2 a a 0 a b g
1 ⎡5 0⎤ ⎡5 0⎤
⇒ ⎨⎢ ⎥ ⎬ = ⎢a 5⎥ ⎢a 5⎥ = AA = b T
b 0 a b c =0
− a 5
⎩⎪ ⎣ ⎦ ⎪⎭ ⎣ ⎦⎣ ⎦
625
g c 0 0 0 0
1 ⎡ 25 0 ⎤
= ⎢ ⎥ Therefore, AAT is a singular matrix.
625 ⎣10a 25⎦
74. A and B are non-singular, so AB is non-singular. Hence
1 25 0
= AB adj(AB) = |AB| I (1)
2 a 125 1 25
AB(adj B adj A) = A(B adj B) adj A
Now
= A(|B| I) adj A
1 25 0 1 25 0 2a
x = |B| (A(adj A))
x 1 25 2a 125 1 25 125 = |B| |A| I (2)
adj(AB) = (adj B) (adj A)
x +a b c
75. Let B = I + A + A2 + … + Ak−1 so that
68. a x +b c =0
B(I − A) = (I + A + A2 + … + Ak−1) (I − A)
a b x +c
= I − A + A − A2 + … − Ak−1 + Ak−1 + Ak−1 − Ak
1 b c = I − Ak = I − 0 = I ⇒ B = (1 − A)−1
⇒ (x + a +b + c) 1 x + b c 1 4 16 6
76. Let A and B . Then the matrix
1 b x +c 3 2 7 2
1 b c equation is AX = B.
⇒ (x + a +b + c) 0 x 0 =0 1 -4
Since |A| = = −2 + 12 ≠ 0; A is an invertible matrix.
0 0 x 3 -2
Let Cij be the cofactors of elements aij in A = [aij].
⇒ x2 (x + a + b + c) = 0
Then
Hence, x = 0 or x = −(a + b + c). C11 = (−1)1+1 (−2) = −2
2 3 C12 = (−1)1+2 3 = −3
69. A A 4 15 19
5 2 C21 = (−1)2+1 (−4) = 4
C22 = (−1)2+2 1 = 1
⎡ −2 −5⎤ ⎡ −2 −5⎤
CA = ⎢ ⎥ ⇒ adj A = C A′ = ⎢ −3 2 ⎥ 2 4
⎣ −3 2 ⎦ ⎣ ⎦ Therefore, adj A = , so
3 1
1 2 3 1 2 3 1
A1 A 1 1 2 4
19 5 2 19 5 2 19 A-1 =adj A =
| A| 10 3 1
70. If A is a skew-symmetric matrix of odd order, then |A| = 0. Now, AX = B ⇒ A−1(AX) = A−1B ⇒ X = A−1B
So, inverse does not exist. 6 2
1 2 4 16 6
Let A be of even order. Then ⇒ X 11
10 3 1 7 2 2
AA −1 = A −1A = I n ⇒ (AA −1) T = (A −1A) T = I n 2
⇒ (A−1)T AT = AT(A−1)T = In ⇒ (A−1)T (−A) = (−A)(A−1)T = In ⎡a 0 ⎤ ⎡a 0⎤ ⎡ a 2 0⎤
77. A2 = ⎢ ⎢ ⎥ = ⎢ ⎥
So, (A−1)T = −A−1 (inverse of a matrix is unique). ⎣1 1⎥⎦ ⎣ 1 1⎦ ⎣a + 1 1⎦
71. adj(adj A) = |A|n − 2A, where n is the order of matrix. Since n = Clearly, no real value of a.
3 here 78. Trace (A) = sum of diagonal elements = 1
Chapter 18 | Matrices and Determinants 789
79. If A and B are the non-singular matrices, then 86. Apply C1 → C1 − C2; C2 → C2 − C3, we get
( AB )¢ = B¢A¢ is always correct.
(2 x - 1) (2 x - 3) ( x - 2)2
80. Multiply determinant row to row and solve.
Hence, (B) is the correct answer. (2 x - 3) (2 x - 5) ( x - 3)2
81. Clearly, a = − i, where i2 = − 1. So (2 x - 5) (2 x - 7) ( x - 4 )2
R1 → R1 − R2 and R2 → R2 − R3 gives
1 1 1 1 1 1
1
Δ(a) = an × 1 a a 3 = 1 -i i 2 2 ( 2 x − 5)
an
1 i 1 0 2 2 (2 x − 7 )
1 0
a (2 x − 5) (2 x − 7) ( x − 4 )2
3p
= 1(− i) + 1 (i2) + (1 +i2) = − 1 − i, arg is − R1 → R1 − R2 gives
4
x a a 0 0 2
82. Given that Δ(x) = b x a . We have 2 2 (2 x - 7 ) = − 8
b b x (2 x - 5) (2 x - 7) ( x - 4 )2
1 a a x 0 a x a 0 Therefore, the value of determinant is independent of x.
Δ′(x) = 0 x a + b 1 a + b x 0 a = b = c = 0 and d = − 8.
0 b x b 0 x b b 1 87. Since a = 2R sin A, b = 2R sin B, c = 2R sin C
⇒ Δ′(x) = 3(x2 − ab)
1 sin A sin2 A 1 a a2
Now sign scheme for Δ′(x) 1
1 sin B sin2 B = 3
1 b b2 = 0
+ – + 8 R
1 sin C sin2 C 1 c c2
– ab ab ⇒ (a − b)(b − c)(c − a) = 0
⇒ a = b or b = c or c = a
Local max Local min Therefore, at least two of a, b, c are equal. So, the triangle is
Since Δ(x) is increasing in ( -¥ , - ab ) È ( ab , ¥ ) , Δ (x) is isosceles or equilateral.
Hence, (A) is the correct answer.
decreasing in ( - ab , ab )
88. When a = b or b = c or c = a, the determinant reduces to zero.
Δ(x) has a local minimum at x = ab
It is not necessary that a = b = c for determinant to be zero.
Δ(x) has a local maximum at x = − ab Therefore, triangle is isosceles.
83. Clearly, f ′(x) = 0 ⇒ f(x) = constant. But f(2) = 5. Therefore, Hence, (C) is the correct answer.
f(x) = 5. Now 89. C2 → C2 + C1
20 20
å f (r ) = å 5 = 5 × 20 = 100 10
C4 11
C5 11
Cm
r =1 r =1
11 12 12
Hence, (C) is the correct answer. Δ= C6 C7 Cm+2
12 13 13
84. C1 → C1 − sin q C3 and C2 → C2 + sin q C3 C8 C9 Cm+ 4
For m = 5, C2 ≡ C3.
1 0 - sinq
Hence, (C) is the correct answer.
f(q ) = 0 1 cosq
⎪⎧0 ,sin q ≠ 1
2
sinq - cosq 0 90. [sin2 q ] = ⎨
⎩⎪1,sin q = 1
2
Again R3 − sin q R1 + cos q R2, we get
If sin q ≠ 1 ⇒ D = 2 sin q cos q − 2i − 1
2
1 0 − sinq
æp ö Re(D) = 2sin q cos q − 1
f(q ) = 0 1 cosq = 1 ⇒ f ç ÷ = 1
è6ø − 2 ≤ Re(D) ≤ 0
0 0 1 3p p
− ≤ arg D ≤ −
85. Here determinant of coefficient matrix 4 2
102 95 88 If sin2 q = 1, sin q = ±1, cos q = 0
Δ = 3 10 17 arg(D) = arg(1 − 2i) or arg(− 1 − 2i)
57 50 43
ex 2 cos 2 x 2 x sec2 x 2
= 0 (using C1 → C1 + C3 − 2C2 ) 91. Δ′(x) = ln(1+ x ) cos x sin x
Similarly,
Δ1 = Δ2 = Δ3 = 0 cos x 2 ex −1 sin x 2
Hence, system has infinite many solutions.
790 Mathematics Problem Book for JEE
n x y z
0 b2 a2 2.
r 1
Δr n 1
sin q sin
nq
= − 2 a2 c2 0 (R1 → R2 − R1 and R3 → R3 − R1) n 2 2
2n 1
b2 0 c2 n 1 q
sin
2
= 4a2b2c2 = 4 [as a, b, c are the cube roots of unity]
Therefore, Im (Δ) = 0 ⎛ n + 1⎞ nq
sin ⎜ q sin
n ⎝ 2 ⎟⎠ 2
95. Applying R3 → R3 − xR1 − yR2 we get 2 −1n
n +1 sin q / 2
p q px + qy = x y z =0
Δ= q r qx + ry ⎛ n + 1⎞ nq
sin ⎜
⎝ ⎟⎠ q sin
0 0 -( px 2 + 2qxy + ry 2 ) 2n − 1
n 2 2
n +1 sinq / 2
⇒ (q2 − pr)(px2 + 2qxy + ry2) < 0
As q2 − pr < 0 ⇒ Discriminant of quad < 0
Chapter 18 | Matrices and Determinants 791
1 -1 1 Therefore,
⎡ 0 −1⎤
(B) 1 1 -1 = 4 ⎡2 2 ⎤ ⎢ ⎥ = ⎡ −1 −2⎤ = −A
f (A) = (I + A) (I − A)−1 = ⎢ ⎥ 1
1 1 1 ⎣ 1 2 ⎦ ⎢ − 0 ⎥ ⎢ −1 −1⎥
⎣ 2 ⎦ ⎣ ⎦
1 cos a cos b 0 cos a cos b Now, comparing the above equation with f (A) = −lA,
(C) cos a 1 cos g = cos a 0 cos g we have l = 1
cos b cos g 1 cos b cos g 0 26. A′A = I. Therefore
|A′A| = | I | ⇒ |A| = ±1
⇒ sin2 g − cosa (cosa − cosb cosg ) + cosb (cosa cosg − cosb )
= −cosa (−cosb cos g ) + cos b (cos a cos g ) a b c
⇒ b c a = ±1
⇒ sin2g − cos2a + 2 cosa cosb cosg − cos2b
= 2cosa cosb cosg c a b
⇒ 3abc − a3 − b3 − c3 = ±1
⇒ sin2g = cos2a + cos2b ⇒ cos2a + cos2b + cos2 g = 1
⇒ a3 + b3 + c3 = 2 and 4
x2 + x x +1 x -2 27. Here, |A − lI| = 0
(D) 2 x 2 + 3 x - 1 3x 3x - 3 1- l 0
=0
x2 + 2x + 3 2x -1 2x -1 -1 7 - l
⇒ (1 − l) (7 − l) = 0
R2 → R2 − (R1 + R3) gives ⇒ l2 − 8l + 7 = 0
⇒ A2 − 8A + 7I2 = 0
x2 + x x +1
x -2 ⇒ A2 = 8A − 7I2
x +1 x - 2
-4 0 0 =4 ⇒ k = − 7 ⇒ |k| = 7
2 x -1 2x -1
x2 + 2x + 3 2x -1 2x -1 28. Let
Now, |I − A| = 0 − 2 = − 2. So 1 3 1
⇒ lf(1) g(1) = 4 ⇒ l (−4) = 4 ⇒ l = −1
⎡0 2 ⎤
adj (I − A) = ⎢ ⎥
⎣ 1 0⎦ 31. On solving, we get
(2l + 15) f(x + 1) − (l + 10) f(x + 8) − f(x + 1) = 0
⎡ 0 −1⎤
(I − A)−1 = ⎢ 1 ⎥ ⇒ (2l + 14) f(x + 1) = (l + 10) f(x + 8)
⎢− 0⎥ Since, f is periodic with period 7, therefore
⎣ 2 ⎦
f(x + 1) = f(x + 8)
As ⇒ 2l + 14 = l + 10
f (x) = (1 + x) (1 − x)−1 ⇒|l|=4
794 Mathematics Problem Book for JEE
x + ay + z = 1
ax + by + z = 0
H
E
has no solution, then S is (2, b )
(A) an infinite set.
(B) a finite set containing two or more elements.
(C) a singleton.
B(5, 2) C(−2, 2)
(D) an empty set. D
(OFFLINE) Thus, solving the equations of two altitudes, the orthocentre of the
Solution: For Δ = 0 (and at the one of the solutions of Δ1, Δ2, ⎛ 1⎞
triangle is obtained as ⎜ 2, ⎟ .
Δ3 ≠ 0): ⎝ 2⎠
1 1 1 Hence, the correct answer is option (C).
D= 1 a 1 =0 ⎧ 0 cos x − sin x ⎫
a b 1 ⎪ ⎪
4. If S = ⎨ x ∈[0 , 2p ] : sin x 0 cos x = 0⎬ , then
⎪ cos x sin x 0 ⎪
1(a − b) − 1(1 − a) + 1(b − a2) = 0 ⎩ ⎭
2a − b − 1 + b − a2 = 0 ⎛p ⎞
a2 − 2a + 1 = 0 ⇒ a = 1
∑ tan ⎜⎝ 3 + x ⎟⎠ is equal to
x ∈S
Chapter 18 | Matrices and Determinants 795
(A) −2 + 3 (B) 4 + 2 3 Solution: The system of equations can be written in the matrix
form as
(C) −4 − 2 3 (D) −2 − 3 − l ⎤ ⎡ x ⎤ ⎡0 ⎤
⎡2 4
(ONLINE) ⎢4 l 2 ⎥⎥ ⎢⎢ y ⎥⎥ = ⎢⎢0 ⎥⎥
⎢
Solution: Solving the determinant ⎢⎣ l 2 2 ⎥⎦ ⎢⎣ z ⎥⎦ ⎢⎣0 ⎥⎦
0 cos x − sin x The system has infinite solutions; thus, we get
sin x 0 cos x = 0
cos x sin x 0 2 4 −l
we get 4 l 2 =0
0[0 − sin x cos x] − cos x[0 − cos2x] − sin x[sin2x − 0] = 0 l 2 2
⇒ cos3x − sin3x = 0
⇒ 0 = 2(2l − 4) − 4(8 − 2l) − l(8 − l2)
Using a3 − b3 = (a − b)(a2 + b2 + ab). We get ⇒ 4l − 8 − 32 + 8l − 8l + l3 = 0
cos3x − sin3x = (cos x − sin x)(cos2x + sin2x + sin x cos x) = 0 ⇒ l3 + 4l − 40 = 0
Using cos2x + sin2x = 1. Now, We can solve this by graphical method
(cos x − sin x)(1 + sin x cos x) = 0
y = x3 + 4x − 40
⇒ cos x − sin x = 0 ⇒ cos x = sin x
y
⇒ tan x = 1
p
⇒x= (1)
4
x
⎛p ⎞ tan a + tan b
Now, evaluating ∑ tan ⎜ + x ⎟ using, tan(a + b ) = ,
x ∈S
⎝ 3 ⎠ 1−tan a tan b
we get −40
⎛p ⎞ ⎛p p ⎞
∑ tan ⎜ + x ⎟ = tan ⎜ + ⎟
⎝ 3 ⎠ ⎝ 3 4⎠
[from Eq. (1)] For x = 0, y = −40: If we take y = −40, then we have
x =p / 4
−40 = x3 + 4x − 40
⎛ p p ⎞ tan(p / 3) + tan(p / 4 ) 3 + 1 1+ 3
⇒ x3 + 4x = 0
⇒ tan ⎜ + ⎟ = = =
⎝ 3 4 ⎠ 1− tan(p / 3) tan(p / 4 ) 1− 3 × 1 1− 3
⇒ x(x2 + 4) = 0
Multiplying and dividing by 1+ 3 , we get ⇒ x = 0, x2 + 4 = 0
⇒ x = ± 2i
⎛p ⎞ 1+ 3 1+ 3
∑ tan ⎜⎝ 3 + x ⎟⎠ = 1− 3 × 1+ 3 The given equation of line intersects x only at one point; therefore,
the real value of l is only one.
Using a2 − b2 = (a + b)(a − b) Hence, the correct answer is option (B).
5. The number of real values of l, for which the system of linear (D) adj(adj(A)) = A ⋅ A
equations (ONLINE)
A + B = 2B ′ (1) é -1 0 0 ù
Taking transpose on both sides, we get • Option (D): The determinant êê 0 -1 0 úú is not possible:
−1(1) − 0(0) + 0(0) = −1. êë 0 0 -1úû
A′ + B′ = 2B (2)
3A + 2B = I3 (3) Hence, the correct answers are options (B) and (D).
3. How many 3 × 3 matrices M with entries from {0, 1, 2} are ⎡a12 + b12 + c12 a1a2 + b1b2 + c2 a1a3 + b1b3 + c1c3 ⎤
there, for which the sum of the diagonal entries of MTM is 5? ⎢ ⎥
(A) 126 (B) 198 = ⎢a2a1 + b2b1 + c2c1 a22 + b22 + c22 a2a3 + b2b3 + c2c3 ⎥
(C) 162 (D) 135 ⎢ ⎥
⎢⎣a3a1 + b3b1 + c3c1 a3a2 + b3b2 + c3c2 a32 + b32 + c32 ⎥⎦
Solution: Let us consider a 3 × 3 matrix
Þ (a12 + b12 + c12 ) + (a22 + b22 + c22 ) + (a32 + b32 + c32 ) = 5
⎡a1 a2 a3 ⎤
⎢ ⎥ There are two possible cases:
M = ⎢b1 b2 b3 ⎥
(i) 02 + 02 + 02 + 02 + 12 + 12 + 12 + 12 + 12 = 5
⎣⎢c1 c2 c3 ⎦⎥
This has 9C5 combinations possible.
Therefore,
(ii) 12 + 22 + 02 + 02 + 02 +02 +02 +02 +02 = 5
⎡a1 b1 c1 ⎤
⎢ ⎥ This has 9C7 × 2C1 possible combinations.
MT = ⎢a2 b2 c3 ⎥
Therefore,
⎢⎣a3 b3 c3 ⎥⎦
9 9! 9! 2!
C5 + 9C7 × 2C1 = +
T
It is given that sum of diagonal of M M is 5. Therefore, 5 !× 4 ! 7 ! 2 ! 1!1!
9 ´ 8 ´ 7 ´ 6 ´ 5! 9 ´ 8 ´ 7!
⎡a1 b1 c1 ⎤ ⎡a1 a2 a3 ⎤ Þ + ´ 2 Þ 126 + 72 = 198
⎢ ⎥⎢ ⎥ 5 ! ´ 4 ´ 3 ´ 2 ´ 1 7 !´ 2 ´ 1
MT M = ⎢a2 b2 c2 ⎥ ⎢b1 b2 b3 ⎥
Thus, the total number of 3 × 3 matrices is 198.
⎢⎣a3 b3 c3 ⎥⎦ ⎢⎣c1 c2 c3 ⎥⎦
Hence, the correct answer is option (B).
Limit, Continuity and
19 Differentiability
19.1 Limit of a Function 19.2.2 Formal Definition of Limit
The concept of limit is used to discuss the behaviour of a function Let f(x) be defined on an open interval about x0, except possible
close to a certain point. For example, at x0 itself, we say that f(x) approaches the limit L as x approaches
x2 -1 x0 and write lim f ( x ) = L , if for every number ∈ > 0, there exists a
f (x) = x ® x0
x -1 corresponding number δ > 0 such that for all x
Clearly the function is not defined at x = 1, but for values close to
0 < | x − x0 | < δ ⇒ | f(x) −L | < ∈
x = 1 the function can be written as
f(x) = x + 1
Illustration 19.1 Show that lim (5 x - 3) = 2.
As x approaches 1 (written as x → 1), f(x) approaches the value 2 x ®1
[f(x) → 2]. We write this as Solution: See Fig. 19.1. Set x0 = 1, f(x) = 5x − 3, and L = 2 in the
lim f ( x ) = 2 definition of limit. For any given ∈ > 0, we have to find a suitable
x ®1
δ > 0 so that if x ≠ 1 and x is within distance δ of x0 = 1, that is, if
It must be noted that it is not necessary for the function to be
0 < | x − 1| < δ,
undefined at the point where limit is calculated. In the above
example, lim f ( x ) is the same as the value of function at x = 2, then f(x) is within the distance ∈ of L = 2, that is
x ®2 |f(x) − 2| < ∈
that is, 3.
Sometimes, functions approach different values as x-approaches We find δ by working backwards from the ∈ inequality
x0 from left and right. By left we mean x < x0 and right means x > |(5x − 3) − 2| = |5x − 5| < ∈
x0. This is written as x→ x 0- and x→ x 0+ , respectively. For example, 5|x − 1| < ∈
f (x) = [x] (greatest integer function) |x − 1| < (∈/5)
For any integer n, Thus, we can take δ = ∈/5.
lim- f ( x ) = n -1 (19.1)
x ®n
19.2 Definition
19.2.1 Informal Definition of Limit
Let f(x) be defined on an open interval about x0, except possibly at
x0 itself. If f(x) gets arbitrarily close to L for all x sufficiently close to
x0, we say that function approaches the limit L as x approaches x0,
and we write
lim f ( x ) = L
x ® x0
This definition is “informal” because phrases like arbitrarily close Figure 19.1
and sufficiently close are imprecise and their meaning depends on
the context. If 0 < |x − 1| < δ = ∈/5, then
The definition is clear enough and enables us to recognize and |(5x − 3) − 2| = |5x − 5| = 5|x − 1| < 5(∈/5) = ∈
evaluate limits of specific functions. This proves that limx→1(5x − 3) = 2.
800 Mathematics Problem Book for JEE
The value of δ = ∈/5 is not the only value that will make 0 < |x − 1| Left and Right Limits: Let y = f(x) be a given function, and x = a
< δ imply |5x − 5| < ∈. Any smaller positive δ will do as well. The defi- is the point under consideration.
nition does not ask for a “best” positive δ, just one, that will work. Left tendency of f(x) at x = a is called its left limit and right ten-
For limit L to exist as x approaches x0, a function f must be dency is called its right limit.
defined on both sides of x0, and its values f(x) must approach L as x f (a + 0 ) = lim f (a + h) and f (a - 0 ) = lim f (a - h) where ‘h’ is a
approaches x0 from either side. Because of this, ordinary limits are h®0 h®0
sometimes called two-side limits. small positive number.
Thus, for the existence of the limit of f(x) at x = a, f(a− 0) = f(a + 0)
19.2.3 Right Hand Limit
19.3 Algebra of Limits
If lim + f ( x ) = L for every number ∈ > 0, there exists a correspond-
x ® x0
Let lim[f(x) = l1 and lim g(x)
[ = l2. Then
ing number δ > 0 such that for all x. x ®a x ®a
x0 < x < x0 + δ ⇒ |f(x) − L| < ∈ 1. lim[c1f ( x ) ± c2 g( x )] = lim[c1f ( x )] ± lim[c2 g( x )] = c1l1 ± c2l2 ,
x ®a x ®a x ®a
Then we call it right hand limit. For example: where c1 and c2 are given constants.
x
lim = +1 2. lim f ( x ) × g( x ) = lim f ( x ) × lim g( x ) = l1 × l2
x ®0+ |x| x ®a x ®a x ®a
lim f ( x )
f ( x ) x ®a l
19.2.4 Left Hand Limit 3. lim = = 1, l2 ¹ 0
x ® a g( x ) lim g( x ) l2
If lim - f ( x ) = L for every number ∈ > 0, there exists a correspond- x ®a
x ® x0
4. lim f [ g( x )] = f [ lim g( x )] = f (l2 ), if and only if f(x) is continuous
ing number δ > 0 such that for all x. x ®a x ®a
at x = l2.
x0 − δ < x < x0 ⇒ |f(x) − L| < ∈
In particular, lim ln [g(x)] = ln l2 if l2 > 0.
Then we call it left hand limit. For example: x ®a
x All these theorems must be used with utmost care. For example,
lim = -1 we have assumed that l1 and l2 are finite. If these are not finite,
x ®0- | x |
the given theorems will not be applicable.
x
Through graph of , we can easily visualize the things written sin x
| x| For example: lim = 1, and if we try to apply the theorems,
above. x ®0 x
sin x 1
we get lim = lim sin x × lim , which does not exist.
x ®0 x x ®0 x ®0 x
right hand limit Which of course is an absurd result, we are getting this absurd
result because in this case the given limit cannot be written
y=1 1
as the product of two limits as lim does not exist. Similarly,
x ®0 x
é sin x ù , where [.] denotes the greatest integer function ≠
lim
x ®0 êë x û
ú
y = −1 left hand
é sin x ù
ê xlim . Here [x] is not continuous at x = 1.
limit
ë ® 0 x úû
Figure 19.2 1 log[1+ f ( x )]
lim
5. lim [1+ f ( x )] =e
g( x ) x ®a g( x )
Now, from the discussions we have just gone through we can x ®a
easily say that limit of a function will exist iff LHL and RHL both
are finite, unique and equal (Fig. 19.2). 19.4 Evaluation of Limits
For example: lim [ x ] will not exist as LHL = lim- [ x ] = 0;
x ®1 x ®1 Following are indeterminate forms:
RHL = lim+ [ x ] = 1
2. ∞
x ®1
0
1. 3. 0 × ∞ 4. ∞ − ∞
This can be seen graphically, in Fig. 19.3. 0 ∞
y 5. 0° 6. ∞° 7. 1∞
We shall divide the ways of evaluation of limits in five categories:
RHL
1 19.4.1 Simplification
x In this method, we can use:
LHL 1 1. Direct substitution
2. Rationalisation
3. Factorization
4. Use of formulas like binomial expansion, trigonometric
Figure 19.3 formulas, etc.
Chapter 19 | Limit, Continuity and Differentiability 801
1. Direct substitution: We can directly substitute the number at 3. Factorization method: We factorize numerator and denom-
which limit is to be find. For example inator of the rational function, so that common factors in
numerator and denominator cancel out. By doing this, we in
t lim ( x 2 + 3 x - 2) can be find out by this method. turn are eliminating the factors which are making the function
x ®1
in one of the indeterminate form. For example:
lim ( x 2 + 3 x - 2) = 2
x ®1
x -4 ( x 1/ 4 - 2) ( x 1/ 4 + 2)
lim = lim
t lim | x | = 2 x ®16 x 1/ 4 - 2 x ®16 ( x 1/ 4 - 2)
x ®-2
= lim ( x + 2) = 4
But before using this method, we have to see that LHL should 1/ 4
x ®16
remain equal to RHL.
For example: In lim sec -1 x , if we directly substitute, we will 4. Use of formulas: We can use formulas which we have stud-
x ®1 ied in other different topics to make functions simplified.
get lim sec -1 x as 0. But LHL of lim sec -1 x will not exist. So, For example:
x ®1 x ®1
answer should be, limit does not exist. x x
2 sin cos
2. Rationalisation method: Rationalisation is followed when sin x 2 2
lim = lim
we have powers in fractions on expressions in numerator and x ®0 x x ®0 x
sin sin
denominator or in both. After rationalization, the terms are 2 2
factorised, which on cancellation give the result. x
= lim 2 cos = 2
x ®0 2
Illustration 19.2 Find lim ( x - x 2 + x ).
x ®¥ 19.5 Use of Standard Limits
Solution: This is apparently of the form ∞ minus ∞ and can be
∞ These standard forms are used in case f(x) → 0 when x → a.
converted to form by multiplying numerator and the denomi-
∞ sin f ( x )
nator by the conjugate. Therefore, 1. lim =1
x ®a f (x)
( x - x 2 + x )( x + x 2 + x ) 2. lim cos f ( x ) = 1
lim ( x - x 2 + x ) = lim x ®a
x ®¥ x ®¥
x+ x +x
2
tan f ( x )
3. lim =1
x -( x + x )
2 2 æ -x ö x ®a f (x)
= lim = lim ç ÷
x ®¥
x + x 2 + x x ®¥ çè x + x 2 + x ÷ø sin-1[f ( x )]
4. lim =1
x ®a f (x)
æ -1 ö -1 -1
= lim ç ÷= =
x ®¥ ç 1 ÷ 1+ 1 2 tan-1[f ( x )]
ç 1+ 1+ ÷ 5. lim =1
è x ø x ®a f (x)
a
æ x 2 + 8 - 10 - x 2 ö sin
Illustration 19.3 Find lim ç ÷.
Illustration 19.4 Evaluate lim
n
.
x ®1ç 2 ÷
è x +3 - 5- x ø
2 b
n ®¥
tan
3-3 0 n +1
Solution: This is of the form = if we put x = 1.
2-2 0 1 a
Solution: As n ®¥ , ® 0 and also tends to zero.
0 n n
To eliminate the factor, multiply and divide by the conjugate of
0 a
sin
numerator and the conjugate of the denominator. Therefore, a
sin should be written as n so that it looks like lim sinq .
n a q ®0 q
( x 2 + 8 + 10 - x 2 ) n
Limit = lim ( x 2 + 8 - 10 - x 2 )
x ®1
( x 2 + 8 + 10 - x 2 ) a öæ b ö
æ
ç sin n ÷ ç n + 1 ÷ a(n + 1)
( x2 + 3 + 5- x2 ) The given limit = lim ç ÷ç ÷×
´ n ®¥
çç a ÷÷ ç tan b ÷ n × b
( x 2 + 3 + 5 - x 2 )( x 2 + 3 - 5 - x 2 ) è n ø çè n + 1 ÷ø
x2 + 3 + 5- x2 ( x 2 + 8 ) - (10 - x 2 ) æ sin a ö æ b ö
= lim ´ ç n ÷ ç n +1 ÷ a æ 1 ö
x ®1
x 2 + 8 + 10 - x 2 ( x 2 + 3) - ( 5 - x 2 ) = lim ç ÷ç ÷ × ç 1+ ÷
n ®¥
çç a ÷÷ ç tan b ÷ b è n ø
æ x2 + 3 + 5- x2 ö è n ø çè n + 1 ÷ø
= lim ç ÷ ´ 1= 2 + 2 = 2 a a
x ®1ç ÷ 3+3 3 = 1 ´ 1 ´ x1 =
è x + 8 + 10 - x
2 2
ø b b
802 Mathematics Problem Book for JEE
sin-1 x Solution:
-1 3- e tan x - e x e x ´ e(tan x - x ) - e x
3 x - sin x x 3 -1 2
lim = lim = = lim = lim
x ® 0 4 x - tan-1 x x ®0 tan-1 x 4 - 1 3 x ® 0 tan x - x x ®0 tan x - x
4-
x
e x (e tan x - x - 1)
= lim
x ®0 tan x - x
19.6 Some More Standard Forms
= e0 × 1 [as x → 0, tan x − x → 0]
These standard forms are used in case f(x) → 0 when x → a.
=1×1=1
1
6. lim [1+ f ( x )] f ( x ) = e
x ®a Reasons for the non-existence of the limit: lim f(x) will not
x ®a
b f(x)
-1 exist due to any of these three reasons:
7. lim = loge b (b > 0) 1. f(x) is not defined in the neighbourhood of x = a.
x ®a f (x)
2. f(x) does not have a unique tendency.
log[1+ f ( x )]
8. lim =1 3. Left and right tendencies of f(x) are not the same.
x ®a f (x)
Some standard limits of indeterminate forms
x n - an
9. lim = nan -1 1. lim
sin ax
= a ∀a ∈R, where x in radian
x ®a x -a x ®0 x
x ×2 x - x tan ax
Illustration 19.7 Evaluate lim . 2. lim = a ∀a ∈R, where x in radian
x ® 0 1- cos x x ®0 x
0
Solution: This is form, so the given limit becomes sin-1 ax
0 3. lim =a∀a∈R
x ®0 x
2
æ ö
æ 2 -1 ö 1 ç x ÷ tan-1 ax
x (2 -1)
2 x x
4. lim = a ∀a ∈R
lim = lim çç ÷÷ ç ÷ x ®0 x
x ®0 æ 2 x ö
ç 2 sin ÷x
x ®0
è x ø 2 çç sin x ÷÷ x
è 2ø è 2 ø æ aö
5. lim(1+ ax ) = e = lim ç 1+ ÷ ∀a ∈R
1/ x a
2
1æ 1 ö x ®0 x ®¥ è xø
= loge 2 ´ ç ÷ = 2 loge 2 = loge 4
2 è 1/ 2 ø loga (1+ x )
6. lim = loga e (a > 0, a ≠ 1)
x ®0 x
x +4
æ x +6 ö ax -1
Illustration 19.8 Evaluate lim ç ÷ . 7. lim = loge a , a > 0
x ®¥ è x + 1
ø x ®0 x
Solution: The problem depends upon reducing the given expres- log x
x
8. lim = 0 (m > 0)
æ 1ö x ®¥ xm
sion to the form lim ç 1+ ÷ which is equal to e.
x ®¥ è x ø x m - am m m -n
x +4 9. lim = a , where m, n are rational numers
æ 6 ö x ®a x n - an n
ç 1+ x ÷
The given limit = lim ç ÷ (1+ x )m - 1
10. lim =m
x ®¥
çç 1+ 1 ÷÷ x ®0 m
è x ø
Chapter 19 | Limit, Continuity and Differentiability 803
x3 x5 x7
x x2 x3 7. sin x = x - + - +
1. e = 1+ +
x
+ + 3! 5! 7!
1! 2 ! 3 !
x2 x4 x6
x log a x 2 (log a)2 8. cos x = 1- + - +
2. a = 1+
x
+ + (a > 0) 2! 4! 6!
1! 2!
x3 2 5
x2 x3 x4 9. tan x = x + + x +
3. log (1+ x ) = x - + - + (−1 < x < 1) 3 15
2 3 4
æ x 11x 2 ö -1 12 3 12 32 5
4. (1+ x )1/ x = e çç 1- + + ÷÷ 10. sin x = x + x + x +
3! 5!
è 2 24 ø
x3 x5
5. e − x = 1−
x x2 x3 11. tan-1 x = x - + +
+ − + 3 5
1! 2 ! 3 !
p 12 1232 5
6. log (1− x ) = − x −
x2 x3 x4 12. cos -1 x = - x - x3 - x -
− − + 2 3! 5!
2 3 4
log(5 + x ) - log(5 - x ) Illustration 19.12 Find the value of a, b and c such that
Illustration 19.10 Evaluate lim .
x ®0 x
axe x - b log(1+ x ) + cxe - x
Solution: lim =2
x ®0 x 2 sin x
é æ x öù é æ x öù
log ê5 ç 1+ ÷ ú - log ê5 ç 1- ÷ ú
ë è 5 øû ë è 5 øû Solution: Using the expansion, we have
lim
x ®0 x æ x2 ö æ x2 x3 ö æ x x2 x3 ö
lim ax çç 1+ x + + ÷÷ - b çç x - + - ÷÷ + cx çç 1- + - + ÷÷
æ xö æ xö x ®0
è 2! ø è 2 3 ø è 1! 2 ! 3 ! ø =2
log 5 + log ç 1+ ÷ - log 5 - log ç 1- ÷
= lim è 5ø è 5ø 2æ x3 x5 ö
x çç x - + - ÷÷
x ®0 x 3! 5!
è ø
æ xö æ xö
log ç 1+ ÷ log ç 1- ÷ æ b ö æa b cö
è 5ø è 5ø = 1+ 1= 2 x (a - b + c ) + x 2 ç a + - c ÷ + x 3 ç - + ÷ +
= lim - è 2 ø è2 3 2ø
x ®0 æxö æ xö 5 5 5 Þ lim =2
5ç ÷ -5 ç - ÷ x ®0 æ x 3
x 5 ö
è5ø è 5ø x çç x - +
2
- ÷÷
è 3! 5! ø
ae x - b
Illustration 19.11 Evaluate lim = 2. Find a and b. Now, above limit would exist if least power in numerator is greater
x ®0 x than or equal to least power in denominator.
Solution: That is, coefficient of x and x2 must be zero and coefficient of x3
should be 2. That is,
æ x x2 ö
a çç 1+ + + + ¥ ÷÷ - b b a b c
1! 2 ! a - b + c = 0, a + -c =0 , - + =2
lim è ø =2 2 2 3 2
x ®0 x
On solving, we get a = 3, b = 12, c = 9.
ax 2
(a - b ) + xa + + + ¥ tan-1 x - sin-1 x
2! Illustration 19.13 Evaluate lim .
lim =2 x ®0 sin3 x
x ®0 x
Solution:
Since, limit is finite, (a−b) = 0 ⇒b = a æ x3 x5 ö æ 12 x 3 12 32 5 ö
Therefore, çç x - + + ÷÷ - çç x + + x + ÷÷
3 5 3! 5!
lim è ø è ø
ax 2 x ®0
æ 3 5 ö
3
xa + + + ¥ x x
2! çç x - + - ÷÷
lim =2 è 3! 5! ø
x ®0 x
æ æ 1 12 ö 3 æ 1 12 32 ö 5 ö
lim a +
ax
+ + ¥ = 2 ç - çç + ÷÷ x + çç - ÷÷ x + ÷÷
ç 3 3!
x ®0 2! è è ø è 5 5! ø ø
= lim 3
x ®0
æ x2 x4 ö
⇒ a = 2 x 3 çç 1- + - ÷÷
Hence, b = 2. è 3! 5! ø
804 Mathematics Problem Book for JEE
Solution:
æ 1 12 ö æ 1 12 32 ö 2
- çç + ÷÷ + çç - ÷ x + px - qx æ0 ö
3 3 ! ø è 5 5 ! ÷ø -1 ç form ÷
= lim è
lim
3
= x ®0 r x - sx è 0 ø
x ®0 2
æ x 2
x 4 ö
çç 1- + ÷÷ Applying L’Hospital’s rule, we have
è 3! 5! ø
p x log p - q x log q log p - log q log( p / q )
lim = =
x ®0 r x log r - s x log s log r - log s log (r / s )
19.8 L’Hospital’s Rule
L′Hospital’s rule is applicable only in
0
and
∞
indeterminate
19.9 Sandwich Theorem (Squeeze
0 ∞ Play Theorem)
0 ∞
forms. For other forms, first we have to convert them into or Sandwich theorem helps in calculating the limits, when limits can-
0 ∞
form, then we can use it. It states that not be calculated using the above discussed methods.
Sandwich theorem: See Fig. 19.4. If f(x), g(x) and h(x) are any three
f (x) f ¢( x ) f ¢¢( x ) …
lim = lim = lim = functions such that,
x ® a g( x ) x ® a g¢( x ) x ® a g¢¢( x )
f(x) ≤ g(x) ≤ h(x) ∀ x ∈ neighbourhood of x = a
0 ∞ and lim f ( x ) = lim[h( x )] = l (say)
till we are getting or form. x ®a x ®a
0 ∞ Then lim g( x ) = l
x ®a
0
Proof: For form: y-axis
0
f (x) 3π
and if lim g( x ) ¹ 0 , lim exists. However, the converse 2
x
x ®a x ® a g( x ) −π −π 0 π π
2 2
of any of the above is not necessarily true.
2. lim f [ g( x )] = f [ lim g( x )] provided lim g( x ) exists and lies in
x ®a x ®a x ®a
−1
19.10 Continuity −2
Figure 19.7
19.10.1 Continuity of a Function
A function f(x) is said to be continuous at x = a if (ii) lim- f ( x ) and lim+ f ( x ) exist but not equal to f(a).
x ®a x ®a
lim- f ( x ) = f (a) = lim+ f ( x ) ì x2 - 4
x ®a x ®a
ï x ¹2
For example, f(x) = í x - 2 at x = 2. (See Fig. 19.8.)
That is, L.H.L = R.H.L. = Value of the function at ‘a’, that is, ï5
î x =2
lim f ( x ) = f (a).
x ®a
806 Mathematics Problem Book for JEE
2
x t f(x) is continuous from left at x = b, that is,
−π − π1 1 1 1 2
3π 2π π π lim f ( x ) = f (b )
x ®b -
log( x + 2) - x 2n sin x x∈I ∩ D (That is, integer lying in the set of domain)
Illustration 19.18 If f ( x ) = lim , then
n ®¥ x 2n + 1 ⇒ x∈I− {−1}
examine continuity of f(x).
Solution: 19.10.5 Properties of Continuous Functions
If |x| < 1, then Let f (x) and g(x) are continuous functions at x = a. Then
log( x + 2) - x sin x
2n
log( x + 2) 1. c f (x) is continuous at x = a where c is any constant
f ( x ) = lim = = log ( x + 2) 2. f (x) ± g(x) is continuous at x = a
n ®¥ x 2n + 1 1
3. f (x)⋅g(x) is continuous at x = a
If |x| > 1, then
4. f (x)/g(x) is continuous at x = a, provided g(a) ≠ 0
log( x + 2) - x 2n sin x
f ( x ) = lim 19.10.6 Intermediate Value Theorem
n ®¥ x 2n + 1
log( x + 2) If c is any real number between f(a) and f(b), then there exists at
- sin x least one solution of the equation f(x) = c in the open interval (a, b),
= lim x 2n = - sin x
n ®¥ 1 if y = f(x) is continuous in the interval. (See Fig. 19.12.)
1+ 2 n
x
If | x | = 1, then f (b)
log( x + 2) - x 2n sin x log( x + 2) - sin x
f ( x ) = lim =
n ®¥ x 2n + 1 2 c
ì - sin x , x < -1
ï log( x + 2), f (a)
ïï -1< x < 1
Thus, f ( x ) = í - sin x , x >1 a b
ï
ï log( x + 2) - sin x
ïî , x = ±1
2 Figure 19.12
Obviously f(x) is discontinuous at x = ± 1.
19.10.7 Types of Discontinuities
æ p ö
Illustration 19.19 Let f ( x ) = [ x ]sin ç ÷ where [.] denotes Basically there are two types of discontinuity:
è [ x + 1] ø
greatest integer function. Then find domain of f and the points of 1. Removable discontinuity: If lim f ( x ) exists but is not equal
x ®a
discontinuity of f in the domain. to f (a), then f (x) has a removable discontinuity at x = a and it
æ p ö can be removed by redefining f (x) for x = a.
Solution: Let f ( x ) = [ x ]sin ç ÷ . Then domain of f(x) is x∈ R
è [ x + 1] ø
Properties of a continuous function: A function f(x), continu-
excluding the points where [x + 1] = 0 (As denominator cannot
be zero). ous on the interval [a, b], possesses the following properties:
0≤x+1<1 (i) f(x) is bounded on [a, b], that is, there exist m and M such that
⇒ −1 ≤ x < 0 m ≤ f(x) ≤ M for all x ∈ [a, b], m and M are respectively mini-
That is, for all x∈ [−1, 0), denominator is zero. So, domain is mum and maximum values of f(x).
x ∈ R − [−1 , 0) (ii) Let A be any number such that min f(x) ≤ A ≤ max f(x). Then
there exists a point x0∈ [a, b] such that f(x0) = A. It is called
Points of Discontinuity intermediate value theorem.
ì | x - 1| ([ x ] - x ), x ¹1
Illustration 19.22 If f ( x ) = í
î 0 , x =1 Your Turn 3
Test the differentiability at x = 1, where [.] denotes the greatest
1. Which of the following functions, whose graphs are given, will
integer function.
have derivatives at all points:
Solution: Check the differentiability at x = 1 (A) (B)
f (1+ h) − f (1)
R [ f ′(1)] = lim (x > 1)
h→ 0 h
| 1+ h - 1| {[1+ h] - (1+ h)} - 0
= lim
h®0 h (C) (D)
h(1- 1- h) h( -h)
= lim = lim =0
h®0 h h®0 h
f (1- h) - f (1)
L[f ¢(1)] = lim (x < 1) Ans. (A)
h®0 -h
2. Is |x + 1| is differentiable at x = −1? Ans. No
f (1- h) - 0
= lim 1 æ 1ö 1
h®0 -h 3. Derivative of y = sin ç ÷ at x = 0 is . (True/False)
x èxø 2
| 1- h - 1| {[1- h] - (1- h)} - 0 Ans. False
= lim
h®0 -h ìï x 2 + 3 x + b x ³ 1
4. If f ( x ) = í is continuous and differentiable,
h(0 - 1+ h) îï 2ax + 3 x <1
= lim =1
h®0 -h then find out value of a and b? Ans. a = 5/2 and b = 4
Lf ′(1) ≠ Rf ′(1) 5. If f(x) = 3x + x sinx, then f ′(1) is .
Hence, f(x) is not differentiable at x = 1. Ans. 3 + cos1 + sin1
810 Mathematics Problem Book for JEE
f (x) Since
6. If f(x) = x2 and g(x) = x, then derivative of at x = 0 is 1. L.H. limit ≠ R.H. limit
g( x )
(True/False) Therefore, limit does not exist.
Ans. False Hence, the correct answer is option (D).
f (x)
7. If lim = 2 and f (1) = 0, then f ′(1) = . nk sin2 n !
x ®1 x - 1 4. If lim = 0 for
Ans. 2 n ®¥ n +1
8. Find values of ‘x’ for which derivative of f(x) = {x} exists. (A) All k (B) 0 ≤ k < 1
(where {.} denotes fractional part of x) Ans. R − I (C) k = 1 (D) For k > 1
f ¢( x + h) - f ¢( x ) Solution:
9. f ′′(x) = lim . (True/False) Ans. True
h®0 h nk sin2 n ! nk
lim = lim × (sin2 n !)
10. Which of the following functions is/are differentiable n ®¥ n +1 n ®¥ n + 1
(A) y = log |x| (B) y = |x|
lim sin2 n ! does not exist but if
1 n ®¥
(C) y = |x|2 + 2x + 3 (D) Ans. (C) nk nk sin2 n !
1 - | x |2 lim = 0 , then lim =0
n ®¥ n + 1 n ®¥ n +1
Additional Solved Examples But
nk
lim =0 ⇔0≤k<1
æp ö æp ö n ®¥ n + 1
1. If lim n cos ç ÷ sin ç ÷ = k, then k is
n ®¥ è 4n ø è 4n ø Hence, the correct answer is option (B).
π π
(C) π ïì (cos x ) x ¹0
(A) (B) (D) None of these 1/sin x
for
4 3 5. If f ( x ) = í
îï k for x =0
Solution:
p The value of k, so that f is continuous at x = 0 is
sin
n p n 2 n×p =p (A) 0 (B) 1 (C) 1/2 (D) None of these
k = lim sin = lim
n ®¥ 2 2n n®¥ 2 p / 2n 2n 4 Solution: Given, f(0) = k. Now
Hence, the correct answer is option (A). lim f ( x ) = lim (cos x )1/sin x [1∞ form]
x ®0 x ®0
2. If lim(1+ a sin x )cosec x = 3, then a is 1
x ®0 (cos x -1)
= lim e sin x
(A) ln 2 (B) ln 3 (C) ln 4 (D) log 2 x ®0
= lim (e )( -2 sin
2
x / 2 ) (( 2 sin x / 2 ) cos x / 2 )
Solution:
x ®0
x
3 = lim (1+ a sin x )cosec x [1∞ form] - tan
x →0 = lim e 2 = e0 = 1
x ®0
= lim e cosec x . a sin x = e a For f(x) to be continuous at x = 0, k should be equal to 1.
x ®0
Hence, the correct answer is option (B).
Hence,
ea = 3 ⇒a = loge3 = ln 3 6. If f(x) = [ 2sinx ] , where [x] denotes the greatest integer
Hence, the correct answer is option (B). function, then
x sin{ x } (A) f(x) is continuous at x = 0
3. lim , where {x} denotes the fractional part of x, is (B) Maximum value of f(x) is 1 in interval [−2π, 2π]
x ®1 x -1
np p
equal to (C) f(x) is discontinuous at x = + , n ∈I
(A) −1 (B) 0 (C) 1 (D) Does not exist 2 4
(D) f(x) is differentiable at x = nπ, n ∈ I
Solution:
lim { x } = lim ( x - [ x ]) = 1- 0 = 1 Solution:
x ®1- 0 x ®1- 0
f ( x ) = [ 2 sin x ]
lim { x } = lim ( x - [ x ]) = 1- 1 = 0
x ®1+ 0 x ®1+ 0 Hence,
Therefore, p p
f ( x ) = -2, - £x<-
2 4
x sin{ x } x
lim = lim sin { x } = −∞⋅sin (1) = −∞ p
x ®1- 0 x -1 x ®1- 0 x - 1 f(x) = -1, - £ x <0
4
x sin{ x } x sin{ x } { x } x sin{ x } x - 1
lim = lim × = lim p
x ®1+ 0 x -1 x ®1+ 0 {x} x - 1 x ®1+ 0 { x } x - 1 f(x) = 0 , 0 £ x <
4
= 1´ 1´ 1 = 1
Chapter 19 | Limit, Continuity and Differentiability 811
p p From Eqs. (1) and (2), for f(x) to be not differentiable but
f(x) = 1 , £x£ continuous at x = 0, possible values of p are given by 0 < p ≤ 1.
4 2
Hence, the correct answer is option (C).
p 3p
f(x) = 1 , £x£
2 4 e - (1+ x )1/ x
9. lim =
x ®0 tan x
3p
f(x) = 0 , < x £p
4 (A) e (B) e/2 (C) e2 (D) None of these
Clearly f(x) is discontinuous at Solution:
p p 3p e - (1+ x )1/ x
x = - , 0 , , , etc. lim
4 4 4 x ®0 x
(tan x ) ×
General value corresponding to x
p p 3p np p æ x 11 2 ö
x =- , , is + e - e ç 1- + x -÷
4 4 4 2 4 è 2 24 ø
= lim [using expansion]
p x ®0 x
Maximum value of f(x) in [−2π, 2π] is 1 at x = .
2
e x æ 11 ö
f(x) is discontinuous and non-differentiable at x = 0, therefore ç 1- x + ÷ e
2 è 12 ø=
choice (D) is not correct. = lim
x ®0 x 2
Hence, the correct answer is option (B).
(27 - 2 x )1/ 3 - 3 Hence, the correct answer is option (B).
7. The value of f(0), so that the function f(x) =
9 - 3 (243 + 5 x )1/ 5 æ 1 q 1 q 1 q ö
(x ≠ 0) is continuous is 10. lim ç tanq + tan + 2 tan 2 + + n tan n ÷ =
n ®¥ è 2 2 2 2 2 2 ø
(A) 2/3 (B) 6 (C) 2 (D) 4
1 1
Solution: (A) (B) − 2 cot 2θ
(27 - 2 x )1/ 3 - (27)1/ 3 q q
lim f ( x ) = lim
x ®0 x ® 0 3[3 - (243 + 5 x )1/ 5 ] (C) 2 cot 2θ (D) None of these
(27 - 2 x ) - (27)
1/ 3 1/ 3 Solution:
( -2 x ) tan θ = cot θ − 2 cot 2θ
= lim (27 - 2 x ) - 27
x ®0 é (243 + 5 x )1/ 5 - (243))1/ 5 ù Therefore,
-3 ê ×5x ú 1 q 1 q
ë 243 + 5 x - 243 û tan = cot - cot q
2 2 2 2
1
(27)−2 / 3 1 q 1 q 1 q
2 3 2 5 1 tan n = n cot n - n -1 cot n -1
= ⋅ = ⋅ ⋅ ⋅ 81 = 2 2n 2 2 2 2 2
15 1 (243)−4 / 5 15 3 9
5 Hence,
1 q
Hence, the correct answer is option (C). Sn = cot n - 2 cot 2q
2n 2
ì p 1
ï x sin , x ¹0 So, required limit is
8. Let f ( x ) = í x
ïî 0 , x =0 æ ö
ç 1 ÷
Then f(x) is continuous but not differentiable at x = 0 if lim Sn = lim ç n - 2 cot 2q ÷ = 1 - 2 cot 2q
n ®¥ n ®¥ ç 2 tanq / 2n q ÷ q
(A) p < 0 (B) p = 0 (C) 0 < p ≤ 1 (D) p ≥ 1 ç × n ÷
è q /2 n
2 ø
Solution:
f(0) = 0 Hence, the correct answer is option (B).
For f(x) to be continuous at x = 0 | x 3 - 6 x 2 + 11x - 6 |
1 11. Let f ( x ) = . Find the set of points ‘a’, where
lim f ( x ) = 0 Þ lim x sin = 0
p
x 3 - 6 x 2 + 11x - 6
x ®0 x ®0 x
lim f ( x ) does not exists.
This is possible only when p > 0. (1) x ®a
1
h sin - 0
p
Solution: We write,
f ( h) - f ( 0 ) h
f ′(0) = lim = lim ì-1, x <1
h® 0 h h® 0 h ï 1< x < 2
| x - 1| | x - 2 | | x - 3 | ï1,
1 f (x) = × × =í
= lim h p -1 sin x -1 x - 2 x - 3 ï-1, 2< x <3
h®0 h
ïî1, x >3
f ′(0) will exist only when p > 1.
Hence, f(x) will not be differentiable if p ≤ 1. (2) Therefore, the limits exists at all points except at x = 1, 2, 3.
812 Mathematics Problem Book for JEE
é 1 ù px
êæ x2 ö x ú
2 which sin = 1 , that is, x is an odd integer.
= lim ln êçç 1 + ÷÷ ú 2
x ®0 2 ø ú
êè So, x = (2n + 1) where n∈ I.
ë û
1 Check continuity at x = (2n + 1)
æ x2 ö 1
= ln lim çç 1 + ÷ x2 = lne e1/ 2 = LHL = lim f ( x ) = 0 (1)
x ®0
è 2 ÷ø 2 x ®2 n +1
x ¹0
19. If f(x) = ïí 3 - e1/ x
Therefore, ,
⎧0 ; px ï ,
sin <1 î0 x =0
2n ⎪ 2
⎛ p x⎞ ⎪
f ( x ) = lim ⎜ sin ⎟ =⎨ Examine (i) f(x) (ii) x2f(x) for continuity and differentiability
n→∞ ⎝ 2⎠ ⎪ px at x = 0.
⎪⎩ 1; sin =1
2 Solution:
Thus, f(x) is continuous for all x, except for those values of x for (i) For f(x)
814 Mathematics Problem Book for JEE
5e1/ h + 2 æ 1 ö
RHL = lim f ( x ) = lim f (0 + h) = lim ( h - 1)sin ç ÷ - sin 1
x ®0 + h®0 h ® 0 3 - e1/ h f ( h) - f ( 0 ) è h - 1ø
f ¢(0 ) = lim Þ f ¢(0 ) = lim
h®0 h h®0 h
5 + 2e -1/ h 5 + 0
= lim = = -5 This limit also does not exist, which implies that f is also not differ-
h ® 0 3e -1/ h - 1 0 -1
entiable at x = 0.
Value of function = f(0) = 0 Hence, the correct answer is option (A).
RHL ≠ Value of function
3. Let f (x) = x | x | and g(x) = sin x.
Hence, f(x) is not continuous and hence not differentiable
Statement-1: gof is differentiable at x = 0 and its derivative is
also.
continuous at that point.
(ii) Let F(x) = x2 f(x). Then Statement-2: gof is twice differentiable at x = 0.
(A) Statement-1 is true, Statement-2 is true; Statement-2 is a
F ( 0 - h) - F ( 0 ) h2f ( -h) - 0
L[F ¢(0 )] = lim = lim correct explanation for Statement-1
h®0 -h h®0 -h (B) Statement-1 is true, Statement-2 is true; Statement-2 is not
= lim hf ( -h) = 0 a correct explanation for Statement-1
h®0
(C) Statement-1 is true, Statement-2 is false
F ( 0 + h) - F ( 0 ) h2f ( h) - 0 (D) Statement-1 is false, Statement-2 is true [AIEEE 2009]
RF ¢(0 ) = lim = lim = lim hf ( h) = 0
h®0 h h®0 h h®0 Solution: We have,
Hence, LF ′ (0) = RF′(0). f (x) = x | x | and g(x) = sin x
Hence, F(x) is differentiable at x = 0, so it is always continuous ìï- sin x 2 , x <0
at x = 0. gof ( x ) = sin( x | x |) = í
2
îïsin x , x ³0
tan( x - 2) x 2 + kx - 2 x - 2k 1 1
lim × =5 (0 + h)2 sin - g(0 ) h 2 sin
x ®2 ( x - 2) g ’( 0 ) = lim ( 0 + h) = lim h =0
( x - 2)
h® 0 0+h h® 0 h
tan( x - 2) x ( x - 2) + k ( x - 2) h® 0
Þ lim lim =5
x ®2 x - 2 x ®2 ( x - 2) Hence, the correct answer is option (B).
15. Let f : R → R be a function such that |f(x)| ≤ x2, for all x ∈ R. Then é x4 ù x4
at x = 0 is Þ f ( x) = 2 ê - x3 + x2 ú = - 2x3 + 2x2
(A) Continuous but not differentiable êë 4 úû 2
(B) Continuous as well as differentiable ⇒ f(2) = 8 − 16 + 8 = 0
(C) Neither continuous nor differentiable Hence, the correct answer is option (B).
(D) Differentiable but not continuous 2
ö
h®0 h h®0 h lim 2 ç form ÷ = lim ç form
m÷
x ®0 sin x è 0 ø x ®0 sin 2 x è0 ø
Now,
2 2
f (h) £ h2 = h Þ
f ( h)
2
£h 2 x (2 xe x ) + 2e x + cos x 3
= lim = (Using L’Hospital’s rule)
h x ®0 2 cos 2 x 2
f ( h) Hence, the correct answer is option (B).
Þ lim - h £ lim £ lim h
h® 0 h® 0 h h® 0
18. For x ∈ R f(x) = |log 2 − sinx| and g(x) = f [f (x)], then
f ( h)
Therefore, by Squeeze principle lim = 0 , that is, f ¢(0 ) = 0. (A) g is differentiable at x = 0 and g ′(0) = −sin(log 2).
h®0 h
(B) g is not differentiable at x = 0.
Thus, f (x) is differentiable at 0 and hence continuous. (C) g ′(0) = cos(log 2).
Hence, the correct answer is option (B). (D) g ′(0) = −cos(log 2).
[JEE MAIN 2016 (OFFLINE)]
16. Let f (x) be a polynomial of degree four having extreme values
Solution: We have
é f (x)ù f(x) = |log 2 − sinx|
at x = 1 and x = 2. If lim ê1+ 2 ú = 3, then f(2) is equal to
x ®0 ë x û g(x) = f(f(x))
(A) −4 (B) 0 (C) 4 (D) −8 Therefore,
g(x) = |log 2 − sin f(x)|
[JEE MAIN 2015 (OFFLINE)]
f(x) = log 2 − sinx (log 2 − sin x) > 0 in neighbourhood of x = 0
Solution: That is,
f ¢( x ) = a( x - 1)( x - 2)( x - k ) g(x) = |log 2 − sin(log 2 − sin x)|
Þ f ¢( x ) = a[ x 3 - (3 + k ) x 2 + (2 + 3k ) x - 2k ] = log 2 − sin(log 2 − sin x) [g(x) is constant function at
x = 0]
é x 4 (3 + k ) x 3 (2 + 3k ) x 2 ù Therefore,
Þ f (x) = a ê - + - 2kx ú + C
êë 4 3 2 úû g ′(x) = −cos(log 2 − sin x)(−cos x)
= cos x · cos(log 2 − sin x)
f (x) é x 2 (3 + k ) x (2 + 3k ) 2k ù C Substituting x = 0, we get
Þ = aê - + - ú+ 2
x 2
êë 4 3 2 x úû x g ′(0) = cos(log 2)
Hence, the correct answer is option (C).
æ f (x) ö ïì é (2 + 3k ) x 2 - 4 kx + 2C ù ïü 1/ n
lim ç 1+ 2 ÷ = lim í1+ a ê ú ý = 3 (given) æ (n + 1)(n + 2)3n ö
x ®0 è x ø x ®0 îï êë 2x2 úû þï 19. lim ç ÷ is equal to
n ®¥ è n2n ø
ìï [a(2 + 3k ) + 2] x 2 - 4 kax + 2Ca üï 18 27 9
Þ lim í ý=3 (A) 3 log 3 − 2 (B) (C) (D) 2
x ®0 ï
î 2x2 ïþ e4 e2 e
⇒ Ca = 0 (1) [JEE MAIN 2016 (OFFLINE)]
Solution: We have
ì [a(2 + 3k ) + 2](2 x ) - 4 ka ü
ý=3
1/ n
and lim í æ (n + 1)(n + 2)(n + 2n) ö
x ®0 î 4x þ y = lim ç ÷
n ®¥ è n2n ø
⇒ ka = 0
1 é æ n + 1ö æ n+2ö æ n + 2n ö ù
ì [(2 + 3k ) + 2](2) ü Þ ln y = lim êln ç n ÷ + ln ç n ÷ + + ln ç n ÷ ú
and lim í ý = 3 ⇒ a(2 + 3k) + 2 = 6 n ®¥ n ë è ø è ø è øû
x ®0 î 4 þ
⇒ 2a + 3ak = 4 ⇒ a = 2 (As ka = 0) 1é æ 1ö æ 2ö æ 2n ö ù
= lim êln ç 1+ n ÷ + ln ç 1+ n ÷ + + ln ç 1+ n ÷ ú
⇒ C = 0 and k = 0 n ®¥ n ë è ø è ø è øû
818 Mathematics Problem Book for JEE
1 2n æ r ö p +2 p -2 -p - 2
= lim å ln çè1+ n ÷ø (A) (B) (C) (D) −1 − cos−1(2)
n ®¥ n
r =1
2 2 2
2 [JEE MAIN 2016 (ONLINE SET-1)]
2
= ò ln(1+ x )dx = ( x + 1)ln( x + 1) - ( x + 1) 0 Solution: We have
0 f(1+) = f(1) = f(1−) and f ′(1−) = f ′(1+)
= 3(ln 3) − 3 + 1 = 3(ln 3) − 2 = ln 27 − ln e2 1
-1 = - Þ 1- (1+ b )2 = 1 ⇒ (1 + b) = 0 ⇒ b = −1
æ 27 ö 27 1- (1+ b )2
ln y = ln ç 2 ÷ Þ y = 2
èe ø e p
-1 = a + cos -1(0 ) = a +
Hence, the correct answer is option (C). 2
20. If f(x) is a differentiable function in the interval (0, ∞) such that p æp +2ö
a = -1- = -ç ÷
t 2 f ( x ) - x 2f (t ) 2 è 2 ø
f(1) = 1 and lim = 1, for each x > 0, then f(3/2) is
t®x t-x Therefore,
equal to
23 13 25 31 a æp +2ö p
(A) (B) (C) (D) =ç ÷ = 1+
18 6 9 18 b è 2 ø 2
[JEE MAIN 2016 (ONLINE SET-1)] Hence, the correct answer is option (A).
Solution: We have 2x
æ a 4 ö
22. If lim ç 1+ - 2 ÷ = e 3 , then a is equal to
t 2 f ( x ) - x 2 f (t ) x ®¥ è x x ø
lim =1
t®x t-x 3 1 2
(A) 2 (B) (C) (D)
Applying L’Hospital’s rule, we get 2 2 3
[JEE MAIN 2016 (ONLINE SET-1)]
2t f ( x ) - x 2f ¢(t )
lim =1 Solution: We have
t®x 1 2x
æa 4 ö
lim ç - 2 ÷
2 x f ( x ) - x 2f ¢( x ) = 1 e èx x ø
x ®¥
= e3
x f ¢( x ) - 2 x f ( x ) + 1 = 0
2
æ 8ö
Þ lim ç 2a - ÷ = 3
x ®¥ è xø
Therefore,
2 1 3
f ¢( x ) - f ( x ) = - 2 ⇒ 2a = 3 ⇒ a =
x x 2
Integrating factor is Hence, the correct answer is option (B).
2
- ò dx 1 23. Let a, b Î\ , (a ≠ 0). If the function f defined as
e x = e -2 ln x =
x2
ì 2x2
æ 1 ö 1 x -4 +1 æ1 1 ö ï , 0 £ x <1
ç f ( x )× 2 ÷ = - ò 4 + C = - +C =ç ´ 3 ÷+C ï
ï
a
è x ø x -4 + 1 è3 x ø f (x) = í a, 1£ x < 2
ï 2
Therefore, ï 2b - 4b , 2 £ x <¥
ïî x 3
1
f (x) = + Cx 2
3x is continuous in the interval [0, ∞), then an ordered pair (a, b) is
1 2 (A) ( - 2 ,1- 3 ) (B) ( 2 , -1+ 3 )
f (1) = 1Þ 1 = + C Þ C =
3 3 (C) ( 2 ,1- 3 ) (D) ( - 2 ,1+ 3 )
1æ 1 ö [JEE MAIN 2016 (ONLINE SET-2)]
f ( x) = ç + 2x2 ÷
3è x ø Solution: We have
æ 3 ö 1 é 2 æ 9 ö ù 31 ì 2x2
Þ f ç ÷ = ê + ç 2 ´ ÷ú = ï , 0 £ x <1
è 2 ø 3 ë 3 è 4 ø û 18 ï a
ï
Hence, the correct answer is option (D). f (x) = í a, 1£ x < 2
ï 2
21. If the function ï 2b - 4b
, 2 £ x <¥
ïì- x , x <1 ïî x 3
f (x) = í -1
îïa + cos ( x + b ), 1 £ x £ 2 Now,
is differentiable at x = 1, then a/b is equal to f(1−) = f(1) = f(1+)
Chapter 19 | Limit, Continuity and Differentiability 819
1 1 Column I Column II
(A) 2 (B) - (C) −2 (D)
2 2 (A) x|x| (P) continuous in (−1, 1)
[JEE MAIN 2016 (ONLINE SET-2)] (B) (Q) differentiable in (−1, 1)
|x|
Solution: We have
2
æ 1- cos 2 x ö (C) x + [x] (R) strictly increasing in (−1, 1)
( 2 x )4 ç ÷
lim è 4 x2 ø (D) |x − 1| + |x + 1| (S) not differentiable at least at
x ®0 æ tan 2 x ö
2 x ç tan x - ÷ one point in (−1, 1)
è 2 ø
[IIT-JEE 2007]
æ 4 1ö
ç 16 ´ x ´ ÷ Solution:
Þ lim è 4ø
x ®0 æ tan x ö (A) →(P), (Q), (R)
2 x ç tan x - ÷ f(x) = x| x |
è 1- tan2 x ø
2 x 3 (1- tan2 x )
Þ lim = -2
x ® 0 (tan x - tan3 x - tan x )
Statement-1:
f ′(t) = −sin t
f ′(π + t) = sin t
Also, since
f ′(t) · f ′(π + t) = −sin2 t
which is negative, the equation f ′(t) = 0 has at least one solution
in [t, t + π].
Statement-2:
Figure 19.18 Since f(x) = 2 + cos x is a periodic function with period 2π, we get
(C)→(R), (S) f (2π + t) = f (t)
f (x) = x +[x] Hence, the correct answer is option (B).
⎧ x − 1, −1< x < 0 4. Let f(x) be a non-constant twice differentiable function defined
=⎨
⎩ x, 0 ≤ x <1 æ 1ö
on (−∞, ∞) such that f ( x ) = f (1- x ) and f ¢ ç ÷ = 0 . Then
è4ø
(A) f ′′(x) vanishes at least twice on [0, 1]
æ 1ö
(B) f ¢ ç ÷ = 0
è2ø
1/ 2
⎛ 1⎞
−1 1 (C) ∫ f ⎜ x + ⎟ sin x dx = 0
⎝ 2⎠
−1/ 2
1/ 2 1
−2 sin p t sin p t
(D) ò f (t )e dt = ò f (1- t )e dt
0 1/ 2
[IIT-JEE 2008]
Solution: We have
Figure 19.19 f(x) = f(1 − x) (1)
(D)→(P), (Q) 1
Put x = + x . Then
f(x) = | x − 1| + | x + 1| 2
æ1 ö æ1 ö
f ç + x÷=f ç -x÷
è2 ø è2 ø
⎛1 ⎞ æ1 ö
So, f ⎜ + x ⎟ is an even function and sin x × f ç + x ÷ is an odd
⎝2 ⎠ è 2 ø
function. Therefore,
1
2
−1 ⎛ 1⎞
1
∫ f ⎜⎝ x + 2 ⎟⎠ sin x dx = 0
−1
2
Figure 19.20 Differentiate equation (1), we get
Hence, the correct matches are (A)ã(P, Q, R); (B)ã(P, S); f ′(x) = −f ′(1 − x) (2)
(C)ã(R, S); (D)ã(P, Q). 1
Put x = , we get
3. Let f(x) = 2 + cos x for all real x. 2
Statement-1: For each real t, there exists a point c in [t, t + π] æ 1ö æ 1ö
f ¢ ç ÷ = -f ¢ ç ÷
such that f ′(c) = 0. Because è2ø è2ø
Statement-2: f(t) = f(t + 2π) for each real t. æ 1ö
(A) Statement-1 is True, Statement-2 is True; Statement-2 is a Þ f ¢ç ÷ = 0
è2ø
correct explanation for Statement-1
(B) Statement-1 is True, Statement-2 is True, Statement-2 is 1
Now, put x = in Eq. (2)
NOT a correct explanation for Statement-1 4
(C) Statement-1 is True, Statement-2 is False æ 1ö æ3ö
(D) Statement-1 is False, Statement-2 is True f ¢ ç ÷ = -f ¢ ç ÷ = 0
è ø
4 è4ø
[IIT-JEE 2007]
Solution: We have f(x) = 2 + cos x for all values of x. æ 1ö æ 1ö æ3ö
Þ f ¢ç ÷ = f ¢ç ÷ = f ¢ç ÷
f ′(x) = −sin x è4ø è2ø è4ø
Chapter 19 | Limit, Continuity and Differentiability 821
æ 1 1ö Solution:
Using Rolle’s theorem, f ″(x) = 0 has at least one solution in ç , ÷
è4 2ø x2
⎛ 1 3⎞ a - a2 - x 2 -
and also in ⎜ , ⎟ . 4 = lim 1 1
⎝ 2 4⎠ L = lim -
x ®0 x4 x ®0
x (a + a - x )
2 2 2
4 x2
So, f ′′(x) = 0 vanishes at least twice on [0, 1]. Now,
( 4 - a ) - a2 - x 2
1 = lim
x ®0
2
sin p t 4 x 2 ( a + a2 - x 2 )
ò f (t )e dt
0 1
Numerator → 0 if a = 2, then L = .
Put t = 1 − x. Then 64
1
Hence, the correct answers are options (A) and (C).
2
sin p (1- x )
ò f (1- x ) × e × ( -dx ) 7. Let f be a real-valued function defined on the interval (0, ∞) by
1 x
1 f ( x ) = ln x + ò 1+ sin t dt . Then which of the following state-
= ò f (1- x )e sinp x × dx 0
1 ment(s) is (are) true?
2
(A) f ″(x) exists for all x ∈ (0, ∞)
Hence, the correct answers are options (A), (B), (C) and (D). (B) f ′(x) exists for all x ∈ (0, ∞) and f ′ is continuous on (0, ∞), but
not differentiable on (0, ∞)
5. Let f and g be real valued functions defined on interval (−1, 1)
(C) There exists α > 1 such that |f ′(x)| < |f (x)| for all x ∈ (α , ∞)
such that g″(x) is continuous, g(0) ≠ 0, g′(0) = 0, g″(0) ≠ 0, and
f(x) = g(x) sin x. (D) There exists β > 0 such that |f(x)| + |f ′(x)| ≤ β for all x ∈ (0, ∞)
[IIT-JEE 2010]
Statement-1: lim[ g( x )cot x - g(0 )cosec x ] = f ¢¢(0 )
x ®0 Solution:
Statement-2: f ′(0) = g(0) 1
f ¢( x ) = + 1+ sin x
(A) Statement-1 is True, Statement-2 is True; Statement-2 is a x
correct explanation for Statement-1 p
f ′(x) is not differentiable at sin x = -1 or x = 2np - , n ÎN .
(B) Statement-1 is True, Statement-2 is True; Statement-2 is 2
not a correct explanation for Statement-1 In x ∈ (1, ∞) f (x) > 0, f ′(x) > 0
(C) Statement-1 is True, Statement-2 is False Consider,
(D) Statement-1 is False, Statement-2 is True x
1
[IIT-JEE 2008] f (x ) f = ln x + ∫ 1+ sin t dt − − 1+ sin x
Solution: We have 0
x
f(x) = g(x) sin x ⎛x ⎞ 1
= ⎜ ∫ 1+ sin t dt − 1+ sin x ⎟ + ln x −
f ¢( x ) = g( x )cos x + g¢( x ) × sin x ⎝0 ⎠ x
Þ f ¢(0 ) = g(0 ) x
f ¢¢( x ) = 2g¢( x )cos x - g( x ) sin x + sin x × g¢¢( x ) Consider, g( x ) = ò 1+ sin t dt - 1+ sin x
Þ f ¢¢(0 ) = 2g¢(0 ) = 0 0
b-y
0 < x < 1Þ 0 < <1
1+ b 2 1- by
⇒ sin2 q =
2b b-y 1
> 0 Þ y < b or y >
1+ b 2 1- by b
⇒ sin q = 1 as
2
≥1
2b b-y 1
- 1 > 0 Þ -1 < y <
q = ±p / 2 1- by b
ì -1, x £ -p / 2 ⎧ 2 p
ïsin x , -p / 2 < x £ 0 ⎪ x cos x , x≠0
ï ⎪⎪
f ¢( x ) = í
ï 1, 0 < x £1 13. Let f ( x ) = ⎨ x ∈ \. Then f is
ïî 1/ x , x >1 ⎪ 0, x=0
⎪
⎪⎩
Clearly, f (x) is not differentiable at x = 0 as f ′(0−) = 0 and f ′(0+) = 1.
f (x) is differentiable at x = 1 as f ′(1−) = f ′(1+) = 1. (A) Differentiable both at x = 0 and at x = 2
Hence, the correct answers are options (A), (B), (C) and (D). (B) Differentiable at x = 0 but not differentiable at x = 2
Chapter 19 | Limit, Continuity and Differentiability 823
ép p ù ⎝ n⎠ ⎜⎝ 1+ ⎟⎠
n
∑ ⎜⎝ a + n ⎟⎠
(2 - h) sin ê -
2 r =1
ë 2 2 - h úû 1
= lim
∫x
a
h®0 h dx
1
(2 - h)2 é -p h ù = 0
=
= lim × sin ê ú
1
60
h®0 h ë 2(2 - h) û ∫ (a + x ) dx
(2 - h)2 ph p 0
= - lim × sin ´ = -p
h®0 ph 2(2 - h) 2(2 - h) x a+1
1
1
2(2 - h) 0
=
⎡⎛ 1⎤ 60
x2 ⎞
Hence, the correct answer is option (B). (a + 1) ⎢⎜ ax + ⎟ ⎥
⎢⎝ 2⎠ ⎥
14. For every integer n, let an and bn be real numbers. Let function ⎣ 0⎦
f : \ ® \ be given by 2 1
=
ì a + sinp x , for x Î [2n, 2n + 1] (2a + 1)(a + 1) 60
f (x) = í n , for all integers n. If f is
îbn + cos p x , for x Î(2n - 1, 2n) 2a2 + 3a + 1 = 120
continuous, then which of the following hold(s) for all n? 2a2 + 3a - 119 = 0
824 Mathematics Problem Book for JEE
Therefore, Therefore,
-3 ± 9 + 8(119 ) -3 ± 961 -3 ± 31 h(c) = 0
a= = =
4 4 4 ⇒ f(c) − g(c) = 0
17 ⇒ f(c) = g(c)
Thus, a = 7, - .
2 Hence, the correct answers are options (A) and (D).
Hence, the correct answers are options (B) and (D).
17. Let f : [a, b] → [1, ∞) be a continuous function and let g : \ ® \
16. For every pair of continuous function f, g : [0 ,1] ® \ such that be defined as
max {f(x) : x∈[0,1]} = max {g(x) : x∈ [0,1]}, the correct state-
ment(s) is (are): ì
ï0 if x < a,
(A) [f(c)]2 + 3f(c) = [g(c)]2 + 3g(c) for some c ∈ [0, 1] ï x
(B) [f(c)]2 + f(c) = [g(c)]2 + 3g(c) for some c ∈ [0, 1] g( x ) = í ò f (t )dt if a £ x £ b , then
a
ï b
(C) [f(c)]2 + 3f(c) = [g(c)]2 + g(c) for some c ∈ [0, 1] ï f (t )dt
î òa
if x > b
(D) [f(c)]2 = [g(c)]2 for some c∈ [0, 1]
[JEE ADVANCED 2014] (A) g(x) is continuous but not differentiable at a
Solution: Suppose f(x) is maximum at c1 and g(x) is maximum at c2. (B) g(x) is differentiable on \
When f(x) is maximum g(x) may or may not be maximum (Fig. 19.21). (C) g(x) is continuous but not differentiable at b
(D) g(x) is continuous and differentiable at either a or b but
not both
f (x) [JEE ADVANCED 2014]
Solution: Checking continuity of g (Fig. 19.24).
g(x)
c1 c2 f
Figure 19.21
Therefore, in the function h(x) = f(x) − g(x), we get
h(c1) = f(c1) − g(c1) ≥ 0 and h(c2) = f(c2) − g(c2) ≤ 0
Hence, h(x) = 0 for some c ∈ [0, 1]. (See Figs. 19.22 and 19.23.) 1
f (x) b
a
g(x)
Figure 19.24
lim g(a − h) = 0
h→ 0
a+h a
lim g(a + h) = ∫ lim f (t )dt = ∫ f (t )dt
h→ 0 a h→ 0 a
a
g(a) = ò f (t )dt = 0
c1 a
c2
Hence, g is continuous at a.
Figure 19.22 Similarly, g is continuous at b. As
b
g (b - h) = g(b ) = g (b + h) = ò f (t )dt
h®0 h®0 a
f (x) Now
g(x) ì 0, x <a
ï
g¢( x ) = íf ( x ), a < x < b
ï 0, x >b
î
Since f(x) ≥ 1 in [a, b] given, so as we cross a and b according to
c1 c2
g(x) function, there are sharp edges encountered due to abrupt
change in the slopes from 0 to k and then from k to 0, where k ≥ 1
Figure 19.23 (Fig. 19.25).
Chapter 19 | Limit, Continuity and Differentiability 825
Solution: Since,
x2
F ( x ) = ò f ( t ) dt
0
a b = 2x f ( x2 )
(1)
Now according to question
F ′( x ) = f ′( x ) ⇒ 2 x f ( x 2 ) = f ′( x )
Þ 2xf ( x ) = f ¢( x ) ( As x Î (0 , 2))
Sharp edge
Hence,
Figure 19.25 f ¢( x ) f ¢( x )
= 2x Þ ò dx = 2 ò xdx
g′(a+) = f (a + h) ≥ 1, etc. f (x) f (x)
Hence, the correct answers are options (A) and (C). 2 x2
Þ loge f ( x ) = +c
18. The largest value of the non-negative integer a for which 2
1- x 2
+c 2
1+ x
ì sin( x - 1) ü 0
ï x -1 - a ï 1 2
Þ lim í ý = ⇒ F (2 ) = e 2 − 1 = e 4 − 1
x ®1 sin( x - 1) 4
ï +1ï
î x -1 þ Hence, the correct answer is option (B).
1+ 1
ì1 - a ü 1 192 x 3 æ 1ö
Þí ý = 20. Let f ¢( x ) = for all x Î\ with f ç ÷ = 0. If
î 1+ 1 þ 4 2 + sin4 p x è2ø
2 1
æ 1- a ö 1
Þç
è 2 ø 4
÷ = m£ ò f ( x )dx £ M , then the possible values of m and M are
1/ 2
Hence, 1 1
1- a 1 (A) m = 13, M = 24 (B) m = ,M=
=± 4 2
2 2 (C) m = −11, M = 0 (D) m = 1, M = 12
⇒ 1 − a = ±1 ⇒ a = 0, 2
[JEE ADVANCED 2015]
Therefore, largest value = 2.
Solution:
Hence, the correct answer is (2).
192 x 3
19. Let f : [0, 2] → \ be a function which is continuous on [0, 2] f ¢( x ) = "x Î \ ,
2 + sin4p x
and is differentiable on (0, 2) with f (0) = 1.
x2 As
Let F(x) = ò f ( t ) dt for x∈[0, 2]. If F ′(x) = f ′(x) for all 1
0 ≤ x ≤1
x ∈ (0, 2), then F(2) equals 2
x x x
ìï-3ax 2 - 2, x <1
⇒ ∫ 8 ≤ ∫ f ′( x ) ≤ ∫ 96 (B) f ( x ) = í
îï bx + a , x >1
2
12 12 12
⎛ 1⎞
⇒ 8 x − 4 ≤ f ( x ) − f ⎜ ⎟ ≤ 96 x − 48 Therefore, f(x) is differentiable "x Î \. So,
⎝ 2⎠
1 1 1 f(1−) = f(1+)
⇒ ∫ 8 x − 4 ≤ ∫ f ( x ) ≤ ∫ 96 x − 48 ⇒ −3a − 2 = b + a2
12 12 12 ⇒ a2 + 3a + 2 = −b
1
⇒ (a + 2)(a + 1) = −b (1)
⇒ 1 ≤ ∫ f ( x ) ≤ 12
12 Also,
ì-6ax ; x <1
f ¢( x ) = í
Hence, the correct answer is option (D). î b; x >1
21. Match the Column I to Column II ⇒ f ′(1−) = f ′(1+)
⇒ −6a = b (2)
Column I Column II Hence, from Eqs. (1) and (2),
a2 + 3a + 2 = 6a
(A) In \ 3, if the magnitude of the projection (P) 1 ⇒ a = 1 or a = 2
vector of the vector a i + b j on 3i + j ⇒ (B) → (P), (Q)
⇒ m =2 ⎛1 ⎞
So, F(x) is a decreasing function on ⎜⎝ , 3⎠⎟ . Therefore,
n 2
Hence, the correct answer is (2). F(1) > F(x) > F(3) ∀ 1 < x < 3
23. Let f, g: [−1, 2] → \ be continuous functions which are twice ⇒ 0 > F(x) > −4 ∀ x ∈ (1, 3)
differentiable on the interval (−1, 2). Let the values of f and g at Hence, f ′( x ) = x ) + F
′(
xF ( x ) < 0 ∀ x ∈(1, 3).
the points −1, 0 and 2 be as given in the following table: − ve − ve
f(2) = 0, g(2) = −1 3
(f − 3g)′′ ≠ 0 on (−1, 0) and (0, 2) (C) 9f ¢(3) - f ¢(1) + 32 = 0 (D) ò f ( x )dx = -12
Number of solutions of f ′(x) − 3g ′(x) = 0 in (−1, 0) ∪ (0, 2) = ? 1
3 Now,
= [9F (3) - F (1)] - 2 ò f ( x )dx ⎛ 1⎞ ⎛1 ⎞
lim x f ⎜ ⎟ = lim+ x ⎜ + cx ⎟ = lim+ (1+ (cx 2 )) = 1
1 x →0+ ⎝ x ⎠ x →0 ⎝ x ⎠ x →0
3
Hence, option (B) is incorrect.
Þ -12 = -36 - 2 ò f ( x )dx
1
Now,
æ c ö
3
lim x 2 f ¢( x ) = lim+ x 2 ç 1- 2 ÷ = lim+ ( x 2 - c ) = -c ¹ 0
Þ ò f ( x )dx = -12 x ®0+ x ®0 è x ø x ®0
1 (2) Hence, option (C) is incorrect.
Also, We cannot say anything about f ( x ) £ 2 "x Î (0 , 2) because we do
3 3
3 not know the value of c.
ò x F ¢¢( x )dx = x F ¢( x ) 1 - ò 3 x F ¢( x )dx
3 3 2
Hence, option (D) is incorrect.
1 1
Hence, the correct answer is option (A).
Þ 40 = 27F ¢(3) - F ¢(1) - 3( -12)
Þ 27F ¢(3) - F ¢(1) = 4 (3) x 2 sin( b x )
27. Let a , b ∈\ be such that lim = 1. Then 6(α + β)
x → 0 a x − sin x
Hence, from Eqs. (1) and (3), we get
equals _______.
27F ¢(3) - F ¢(1) - 36 = 4 - 36 = -32 [JEE ADVANCED 2016]
⇒ 9f ′(3) − f ′(1) + 32 = 0 x 2 sin b x
Solution: It is given that a , b ∈\ such that lim = 1.
x ® 0 a x - sin x
Hence, the correct answers are options (C) and (D).
Therefore,
26. Let f : (0 , ¥ ) ® \ be a differentiable function such that x 2 sin b x
lim =1
f (x) x ® 0 a x - sin x
f ¢( x ) = 2 - for all x ∈ (0,∞) and f (1) ≠ 1. Then
x æ sin b x ö
x2 ç ÷b x
æ 1ö æ 1ö bx ø
(A) lim+ f ¢ ç ÷ = 1 (B) lim+ xf ç ÷ = 2 Þ lim è =1
x ®0 èxø x ®0 èxø x ®0 a x - sin x
(C) lim+ x 2f ¢( x ) = 0 (D) f ( x ) £ 2 for all x Î (0,2) x3
x ®0 ⇒ b lim =1
[JEE ADVANCED 2016] x →0 ⎛ x3 x5 ⎞
ax −⎜x − + − ⎟
Solution: It is given that ⎝ 3! 5! ⎠
f (x)
f : ( 0 , ∞ ) → \ , f ′( x ) = 2 − x3
x ⇒ b lim =1
x →0 x3 x5
x (a − 1) + − +
Now, the linear differential equation is 3! 5!
f (x)
f ¢( x ) + =2 For finite limit α = 1,
x
1
The integrating factor is 3! × β = 1 Þ b =
1 6
ò x dx
e = eln x = x
æ 1ö
Therefore, Then, 6(a + b ) = 6 ç 1+ ÷ = 6 + 1 = 7
è 6ø
ò d( xf ( x )) = ò 2 x dx + c Hence, the correct answer is (7).
æ c ö
xf ( x ) = x + c Þ f ( x ) = ç x + ÷ "x Î (0 , ¥ )
2
28. Let a, b Î\ and f : \ → \ be defined by
è x ø
f(x) = a cos(|x3 − x|) + b|x| sin(|x3 + x|). Then f is
Now, (A) Differentiable at x = 0 if a = 0 and b = 1.
f (1) ≠ 1⇒ 1 ≠ 1+ c ⇒ c ≠ 0 (B) Differentiable at x = 1 if a = 1 and b = 0.
and (C) NOT differentiable at x = 0 if a = 1 and b = 0.
c æ c ö (D) NOT differentiable at x = 1 if a = 1 and b = 1.
f ¢( x ) = 1- Þ lim+ f ¢( x ) = lim+ ç 1- 2 ÷ = 1
x2 x ®0 x ®0 è x ø [JEE ADVANCED 2016]
Solution: The given function is
æ 1ö
Þ lim+ f ¢ ç ÷ = lim+ (1- cx 2 ) = 1 f(x) = a cos(|x3 − x|) + b|x| sin(|x3 + x|)
x ®0 è x ø x ®0 which is an even function.
Hence, option (A) is correct. f(x) = a cos(x3 − x) + bx sin(x3 + x)
Chapter 19 | Limit, Continuity and Differentiability 829
[JEE ADVANCED 2016] denotes the greatest integer less than or equal to y for y Î\.
Then
Solution: The given function is
é 1 ù
x /n (A) f is discontinuous exactly at three points in ê - , 2 ú .
é nn ( x + n)( x + n/ 2)( x + n/n) ù ë 2 û
f ( x ) = lim ê ú "x > 0
n ®¥ ê n !( x 2 + n2 )( x 2 + n2 / 4 )( x 2 + n2 /n2 ) ú
ë û é 1 ù
(B) f is discontinuous exactly at four points in ê - , 2 ú .
x /n ë 2 û
⎡ ⎤
⎢ nnnn ⎛⎜ 1+ ⎞⎟ ⎛⎜ 1+ ⎞⎟ ⎛⎜ 1+ ⎞⎟
x 2x nx
2 ⎥
æ 1 ö
⎢ ⎝ n⎠ ⎝ n⎠ ⎝ n⎠ (n !) ⎥ (C) g is NOT differentiable exactly at four points in ç - , 2 ÷ .
⇒ f ( x ) = lim ⎢ × è 2 ø
n→∞ ⎛ x 2 ⎞ ⎛ 4 x 2 ⎞ ⎛ n2 x 2 ⎞ (n2 )n ⎥
⎢ n ! n ! 1+ ⎥
⎢⎣ ⎜ n2 ⎟ ⎜ 1+ n2 ⎟ ⎜ 1+ n2 ⎟ ⎥⎦
æ 1 ö
(D) g is NOT differentiable exactly at five points in ç - , 2 ÷ .
⎝ ⎠⎝ ⎠ ⎝ ⎠
è 2 ø
x n
⎡ ⎤ [JEE ADVANCED 2016]
⎢ ⎛⎜ 1+ ⎞⎟ ⎛⎜ 1+ ⎞⎟ ⎛⎜ 1+ ⎞⎟ ⎥
x 2x nx
⎝ ⎠ ⎝ ⎠ ⎝ ⎠ Solution: It is given that
⎢ n n n ⎥
f ( x ) = lim ⎢ ⎡1 ⎤
n→∞ ⎛ x 2⎞ ⎛
4 x 2⎞ ⎛ n2 x 2 ⎞ ⎥ f : ⎢ , 2⎥ → \
⎢ 1+ 1+ 2 ⎟ ⎜ 1+ 2 ⎟ ⎥ ⎣2 ⎦
⎢⎣ ⎜⎝ n2 ⎟⎠ ⎜⎝ n ⎠ ⎝ n ⎠ ⎥⎦
é 1 ù
x /n g : ê - , 2ú ® \
é æ rx öù ë 2 û
ê n ç 1+ ÷ú
= lim Õê ç n ÷ú Therefore,
n ®¥ ê ÷ú
r =1 ç r2x2
f(x) = [x2 − 3], g(x) = [|x| + |4x − 7|]f(x)
ê ç 1+ 2 ÷ú
ë è n øû
f(x) = [x2] − 3
830 Mathematics Problem Book for JEE
⎧ ⎡ 1 ⎞ sin x
=
⎪ −3 x ∈ ⎢ − 2 , 0⎟⎠ ⎧ 8. lim
x ®¥ x
⎪ ⎣ ⎡ 1 ⎞
⎪ −3, x ∈ ⎢ − ,1⎟
⎪
⎪ ⎣ 2 ⎠ (A) 1 (B) 0 (C) Does not exist (D) None of these
⎪ −3 x ∈ [ 0 , 1)
⎪ ⎪ 2 sin 3 x 2
ì sin[ x ] n
Sn +1 - Sn
ï , when [ x ] ¹ 0 57. If Sn = å ak and lim an = a, then lim is equal to
46. If f ( x ) = í [ x ] where [x] is the greatest integer k =1
n ®¥ n ®¥ n
ï
î 0 , when [ x ] = 0 åk
k =1
function, then xlim f (x) =
®0 (A) 0 (B) a (C) 2a (D) 2a
(A) −1 (B) 1 (C) 0 (D) None of these
4 + 3an
1- (10 )n -a 58. If a1 = 1 and an +1 = , n ³ 1 and if lim an = a, then the
47. If lim = , then the value of α is 3 + 2an n→∞
n ®¥ 1+ (10 )n +1 10 value of a is
(A) 0 (B) −1 (C) 1 (D) 2 (A) 2 (B) - 2 (C) 2 (D) None of these
log[1+ x ] 3
48. The value of lim = æxö æxö æxö æ x ö
x ®0 sin3 x 59. The value of lim cos ç ÷ cos ç ÷ cos ç ÷ cos ç n ÷ is
n ®¥ è ø
2 è ø
4 è ø
8 è2 ø
(A) 0 (B) 1 (C) 3 (D) None of these
sin x x
4q (tanq − sinq ) (A) 1 (B) (C) (D) None of these
49. lim is x sin x
q → 0 (1 − cos 2q )2
1 1 1 1
60. lim + 2
+ 3 + + n equals
(A) 1/ 2 (B) 1/2 (C) 1 (D) 2 n ®¥ 2 2 2 2
xn 1- n2
51. The value of lim where x < −1 is 62. The value of lim will be
n ®¥ å n
n ®¥ x +1
n
(A) 1/2 (B) −1/2 (C) 1 (D) None of these (A) −2 (B) −1 (C) 2 (D) 1
1 1 1 1 1
52. The value of lim + + + + + is 1 - 2 + 3 - 4 + 5 - 6 + - 2n
n ®¥ 1.3 3.5 5.7 7.9 (2n - 1)(2n + 1) 63. If x n = , then lim x n is equal to
n2 + 1 + 4 n2 - 1 n ®¥
equal to
(A) 1/2 (B) 1/3 (C) 1/4 (D) None of these 1 2 2
(A) (B) - (C) (D) 1
53. The value of the constant α and β such that 3 3 3
æ x2 +1 ö ( x + 1)10 + ( x + 2)10 + + ( x + 100 )10
lim çç - a x - b ÷÷ = 0 are respectively 64. lim is equal to
x ®¥ x + 1
è ø x ®¥ x 10 + 1010
(A) (1, 1) (B) (−1, 1) (C) (1, −1) (D) (0, 1) (A) 0 (B) 1 (C) 10 (D) 100
54. Let f : R ® R be a differentiable function having 1+ 2 + 3 + + n
f(x)
65. The value of lim is equal to
ò
3
4t dt n→∞ n2 + 100
æ 1 ö
f (2) = 6 , f ’(2) = ç ÷ . Then lim 6
equals 1
è 48 ø x ®2 x -2 (A) ∞ (B) (C) 2 (D) 0
2
(A) 12 (B) 18 (C) 24 (D) 36
x
ò cos t
2
é 1 2 n ù lim 0
55. lim ê + + + is equal to 66. The value of dt is
n ®¥ ë 1- n2 1- n2 1- n2 úû x ®0 x
ïì2 , for x ¹ 0
1/ x
ì1 ìïsin-1 | x |, when x ¹ 0
ï sin x , x ¹ 0
2
84. If f ( x ) = í x , then 91. If f ( x ) = í , then
îï 0 , when x = 0
ïî 0, x = 0
(A) lim f ( x ) ¹ 0 (B) lim f ( x ) ¹ 0
x ®0 + x ®0 -
(A) lim f ( x ) ¹ 0 (B) lim f ( x ) ¹ 0
x ®0 + x ®0 -
(C) f(x) is continuous at x = 0 (D) None of these
(C) f(x) is continuous at x = 0 (D) None of these
ì sin 2 x
ï , when x ¹ 0
ì x - 1, x < 0 92. If f ( x ) = í 5 x is continuous at x = 0, then the
ï ïî k , when x = 0
ï 1
85. If f ( x ) = í , x = 0 , then
value of k will be
ï 4
ïî x 2 , x > 0 2 2
(A) 1 (B) (C) - (D) None of these
lim f ( x ) = 1 lim f ( x ) = 1 5 5
(A) (B)
x ®0 + x ®0 -
ìï1+ x 2 , when 0 £ x £ 1
(C) f(x) is discontinuous at x = 0 (D) None of these 93. If f ( x ) = í , then
ïî1- x , when x > 1
86. Which of the following statements is true for graph f(x) =
(A) lim+ f ( x ) ¹ 0 (B) lim- f ( x ) ¹ 2
log x ? x ®1 x ®1
(A) Graph shows that function is continuous (C) f(x) is discontinuous at x = 1 (D) None of these
(B) Graph shows that function is discontinuous
ì x2 -1
(C) Graph finds for negative and positive values of x ï , when x ¹ -1
94. If f ( x ) = í x + 1 , then
(D) Graph is symmetric along x-axis ï - 2, when x = -1
î
ì x2 -1 (A) lim f ( x ) = -2 (B) lim f ( x ) = -2
ï , when x ¹ 1 x ®( -1)- x ®( -1)+
87. If function f ( x ) = í x - 1 is continuous at x = 1,
ï (C) f(x) is continuous at x = −1 (D) All the above are correct
î k , when x = 1
then the value of k will be ì5
ï 2 - x , when x < 2
(A) −1 (B) 2 (C) −3 (D) −2 ï
95. If f ( x ) = í1, when x = 2 , then
x
88. At which points the function f ( x ) = , where [·] is greatest ï 3
[x] ï x - , when x > 2
integer function, is discontinuous î 2
(A) f(x) is continuous at x = 2
(A) Only positive integers
(B) f(x) is discontinuous at x = 2
(B) All positive and negative integers and (0, 1)
(C) lim f ( x ) = 1
(C) All rational numbers x ®2
(A) x = 1 only
1 1 x = 1 and x = −1 only
(A) - (B) (C) − 1 (D) 1 (B)
2 2 (C) x = 1, x = −1, x = −3 only
(D) x = 1, x = −1, x = −3 and other values of x
⎧1− cos x
⎪ ,x ≠0
132. If f ( x ) = ⎨ x is continuous at x = 0, then k = ⎧1 − x
⎪⎩ k, x = 0 ⎪ , x ≠ −1
140. If f (x) = ⎨ 1+ x , then the value of f ([2x]) will be
⎪ 1, x = −1
1 1 1 ⎩
(A) 0 (B) (C) (D) - (where [ ] shows the greatest integer function)
2 4 2
133. A function f on R into itself is continuous at a point a in R, iff (A) Continuous at x = −1 (B) Continuous at x = 0
for each ∈ > 0, there exists, δ > 0 such that (C) Discontinuous at x =
1
(D) All of these
(A) | f(x) − f(a) | < ∈ ⇒ | x − a | < δ 2
(B) | f(x) − f(a) | > ∈ ⇒ | x − a | > δ 1- cos 4 x
141. If the function f ( x ) = , where x ≠ 0 and f(x) = k where
(C) | x − a | > δ ⇒ | f(x) − f(a) | > ∈ 8x2
(D) | x − a | < δ ⇒ | f(x) − f(a) | < ∈ x = 0 is a continuous function at x = 0, then the value of k
will be
ì e1/ x - 1
ï , x ¹0 (A) k = 0 (B) k = 1 (C) k = −1 (D) None of these
134. For the function f ( x ) = í e1/ x + 1 , which of the follow-
ï0 , x =0
î
ing is correct ìï e x ; x £ 0
142. If f ( x ) = í , then
ïî| 1- x |; x > 0
(A) lim f ( x ) does not exist
x ®0 (A) f(x) is differentiable at x = 0
(B) f(x) is continuous at x = 0 (B) f(x) is continuous at x = 0
(C) lim f ( x ) = 1 (C) f(x) is differentiable at x = 1
x ®0 (D) f(x) is continuous at x = 1
(D) xlim f ( x ) exists but f(x) is not continuous at x = 0
®0 143. Which of the following statements is true:
135. The function ‘f ’ is defined by f(x) = 2x − 1, if x > 2, f(x) = k if x = 2 (A) A continuous function is an increasing function.
and x2 − 1, if x < 2 is continuous, then the value of k is equal to (B) An increasing function is continuous.
(C) A continuous function is differentiable.
(A) 2 (B) 3 (C) 4 (D) −3 (D) A differentiable function is continuous.
838 Mathematics Problem Book for JEE
ì x + 1, when x < 2 153. Let [x] denotes the greatest integer less than or equal to x. If
144. If f ( x ) = í , then f ′(2) equals f (x) = [x sin π x], then f (x) is
î2 x - 1, when x ³ 2
(A) Continuous at x = 0 (B) Continuous in (−1, 0)
(A) 0 (B) 1 (C) 2 (D) Does not exist
(C) Differentiable in (−1,1) (D) All the above
⎧ e(1/ x ) − e(−1/ x )
⎪x ,x ≠0 ì| x - 3 |; x ³1
145. If f ( x ) = ⎨ e(1/ x ) + e( −1/ x ) then which of the following ï
154. The function defined by f ( x ) = í 1 2 3 13 is
⎪ 0 , x = 0 ïî 4 x - 2 x + 4 ; x < 1
⎩
is true
(A) Continuous at x = 1 (B) Continuous at x = 3
(A) f is continuous and differentiable at every point
(B) f is continuous at every point but is not differentiable (C) Differentiable at x = 1 (D) All the above
(C) f is differentiable at every point ìï e x + ax , x < 0
(D) f is differentiable only at the origin 155. If f ( x ) = í is differentiable at x = 0, then (a, b) is
îïb( x - 1) , x ³ 0
2
146. If f(x) = |x − 3|, then f is
(A) Discontinuous at x = 2 (A) (−3, −1) (B) (−3, 1) (C) (3, 1) (D) (3, −1)
(B) Not differentiable x = 2 156. The function y = | sin x | is continuous for any x but it is not
(C) Differentiable at x = 3 differentiable at
(D) Continuous but not differentiable at x = 3
(A) x = 0 only
147. Let h(x) = min{x, x2}, for every real number of x. Then
(B) x = π only
(A) h is continuous for all x (C) x = k π (k is an integer) only
(B) h is differentiable for all x
(D) x = 0 and x = k π (k is an integer)
(C) h′(x) = 1, for all x > 1
(D) h is not differentiable at two values of x 157. The function y = e− | x | is
148. There exists a function f(x) satisfying f(0) = 1, f ′(0) = −1, (A) Continuous and differentiable at x = 0
f(x) > 0 for all x and (B) Neither continuous nor differentiable at x = 0
(A) f(x) < 0, ∀x (B) −1 < f ′′(x) < 0, ∀x (C) Continuous but not differentiable at x = 0
(D) Not continuous but differentiable at x = 0
(C) −2 < f ′′(x) ≤ −1, ∀x (D) f ′′(x) < −2, ∀x
ì x , if 0 £ x £ 1 ì 1+ x , x £ 2
149. The function f ( x ) = í is 158. A function f (x) = í is
î1, if1 < x £ 2 î5 - x , x > 2
(A) Continuous at all x, 0 ≤ x ≤ 2 and differentiable at all x, (A) Not continuous at x = 2
except x = 1 in the interval [0, 2] (B) Differentiable at x = 2
(B) Continuous and differentiable at all x in [0, 2] (C) Continuous but not differentiable at x = 2
(C) Not continuous at any point in [0, 2] (D) None of these
(D) Not differentiable at any point [0, 2] 159. The left-hand derivative of f (x) = [x] sin(π x) at x = k, k is an
150. The function f (x) = | x | at x = 0 is integer and [x] = greatest integer ≤ x, is
(A) Continuous but non-differentiable (A) (−1)k (k − 1)π (B) (−1)k − 1 (k − 1)π
(B) Discontinuous and differentiable (C) (−1)k k π (D) (−1)k − 1 k π
(C) Discontinuous and non-differentiable
(D) Continuous and differentiable ïì 0 , x <0
160. Let f (x) = í 2 , then for all values of x
îï x , x ³0
ì x2
ï ,x ¹0 (A) f is continuous but not differentiable
151. Consider f ( x ) = í| x |
(B) f is differentiable but not continuous
ï 0, x = 0
î (C) f ′ is continuous but not differentiable
(D) f ′ is continuous and differentiable
(A) f (x) is discontinuous everywhere
(B) f (x) is continuous everywhere 161. Which of the following is not true:
(C) f ′(x) exists in (−1, 1) (A) A polynomial function is always continuous
(D) f ′(x) exists in (−2, 2) (B) A continuous function is always differentiable
152. At the point x = 1, the given function (C) A differentiable function is always continuous
(D) ex is continuous for all x
ìï x 3 - 1; 1 < x < ¥
f (x) = í is 1
162. The function f ( x ) = x 2 sin , x ¹ 0 , f (0 ) = 0 at x = 0
îï x - 1; - ¥ < x £ 1 x
(A) Continuous and differentiable (A) Is continuous but not differentiable
(B) Continuous and not differentiable (B) Is discontinuous
(C) Discontinuous and differentiable (C) Is having continuous derivative
(D) Discontinuous and not differentiable (D) Is continuous and differentiable
Chapter 19 | Limit, Continuity and Differentiability 839
ì x -1 ì 1 "x < 0
ïï 2 x 2 - 7 x + 5 for x ¹ 1 175. Let f (x) = í . Then what is the value
163. If f ( x ) = í , then f ′(1) = î1 + sin x "0 £ x £p /2
ï 1
- for x = 1 of f ′(x) at x = 0?
ïî 3
(A) 1 (B) −1 (C) ∞ (D) Does not exist
(A) −1/9 (B) −2/9 (C) −1/3 (D) 1/3
f (5) - f (1)
x 176. If f (x) = x2 − 2x + 4 and = f ¢(c) . Then value of c will
164. If f (x) = for x ∈ R, then f ′(0) = . 5 -1
1+ | x | be
(A) 0 (B) 1 (C) 2 (D) 3
(A) 0 (B) 1 (C) 2 (D) 3
⎧⎪mx 2 , x ≤1
165. The value of m for which the function f ( x ) = ⎨ is 177. Let f(x + y) = f (x) + f (y) and f (x) = for all x, y ∈ R, where
x2g(x)
⎩⎪ 2 x , x >1 g(x) is a continuous function. Then f ′(x) is equal to
differentiable at x = 1 is
(A) g ′(x) (B) g (0) (C) g (0) + g ′(x) (D) 0
(A) 0 (B) 1 (C) 2 (D) Does not exist
178. The function f(x) = (x2 − 1) |x2 − 3x + 2 | + cos(| x |) is not differ-
ì sin x , for x > 0 entiable at
166. Let f (x) = í and g(x) = ex. Then (gof ) (0) is
î1 - cos x , for x £ 0 (A) −1 (B) 0 (C) 1 (D) 2
(A) 1 (B) −1 (C) 0 (D) None of these 179. The function which is continuous for all real values of x and
differentiable at x = 0 is
1
167. Suppose f (x) is differentiable at x = 1 and lim f (1+ h) = 5, 1
h®0 h (A) | x | (B) log x (C) sin x (D) x 2
then f ′(1) equals
180. Which of the following is not true:
(A) 5 (B) 6 (C) 3 (D) 4
(A) Every differentiable function is continuous.
168. If f is a real-valued differentiable function satisfying (B) If derivative of a function is zero at all points, then the
| f(x) − f(y) | ≤ (x − y)2, x, y ∈ R and f (0) = 0, then f (1) is equal to function is constant.
(A) 2 (B) 1 (C) −1 (D) 0 (C) If a function has maximum or minima at a point, then
169. Let f be differentiable for all x. If f (1) = −2 and f ′(x) ≥ 2 for the function is differentiable at that point and its deriv-
x ∈ [1, 6], then ative is zero.
(D) If a function is constant, then its derivative is zero at all
(A) f (6) < 5 (B) f (6) = 5 (C) f (6) ≥ 8 (D) f (6) < 8
points.
170. f (x) = | | x | − 1| is not differentiable at
ì x + 2, -1 < x < 3
(A) 0 (B) ±1, 0 (C) 1 (D) ±1 ï
181. If f (x) = í 5, x = 3 , then at x = 3, f ′(x) =
171. If f (x) is twice differentiable polynomial function such that ï8 - x , x > 3
î
f (1) = 1, f (2) = −4, f (3) = 9, then
(A) f ′′(x) = 2, ∀ x ∈ R (A) 1 (B) − 1 (C) 0 (D) Does not exist
(B) There exist at least one x ∈ (1, 3) such that f ′′(x) = 2 0 £ x £1
ìx,
(C) There exist at least one x ∈ (2, 3) such that f ′(x) = 5 = f ′′(x) 182. If f (x) = í , then
(D) There exist at least one x ∈ (1, 2) such that f (x) = 3 î2 x - 1, 1 < x
187. The number of points at which the function f(x) = |x − 0.5| (A) –2 (B) 0 (C) 1 (D) –1
+ | x − 1 | + tan x does not have a derivative in the interval
(0, 2), is x cos x - log(1+ x )
200. lim is equal to
(A) 1 (B) 2 (C) 3 (D) 4 x ®0 x2
(A) 1/2 (B) 1 (C) 0 (D) 3
x l - 5l
188. If lim = 500, then positive values of λ is
x ®5 x - 5 x +1 + x -1 - 2
201. lim equals
(A) 3 (B) 4 (C) 5 (D) 6 x ®0 x
(A) 1 (B) –1 (C) 2 (D) 0
ìx, x <0
ï
189. If f (x) = í1, x = 0 , then lim f(x) is equal to a tan x - asin x
ï 2 x ®0 202. lim is equal to (a > 0)
x >0 x ® 0 tan x - sin x
îx ,
(A) 0 (B) 1 (C) 2 (D) Does not exist (A) a (B) ln a (C) ln (1/a) (D) 1/a
190. Let f(x) = [α + β sin x], x ∈ (0, π), α ∈ I, β is a prime number æ 1+ e1/ n + e 2 / n + + e ( n -1) / n ö
and [.] denotes G.I.F. The number of points at which f(x) is not 203. lim ç ÷ is equal to
n ®¥ ç n ÷
differentiable is è ø
(A) β (B) 2β + 1 (C) 2β − 1 (D) β + 1 (A) 0 (B) 1 (C) e −1 (D) e
-1
p - cos x
191. lim éë x + [ x ]ùû , n ∈ I, is equal to ([.] denotes greatest integer 204. lim is given by
x ®n x ®-1+ x +1
function)
1 1
(A) 0 (B) 1 (C) −1 (D) Does not exist (A) (B) (C) 1 (D) 0
p 2p
2m / x
æ x 3x ö 205. Let f : R → R be such that f(1) = 3 and f ′(1) = 6. Then
192. lim ç sin + cos ÷ is
x ®0 è m mø 1/ x
æ f (1+ x ) ö equals
lim ç ÷
(A) 1 (B) e2 (C) e6m (D) log 6m x ® 0 è f (1) ø
(A) 0 (B) a/b (C) a + b (D) ea/b (A) 0 (B) 1 (C) 2 (D) 6
1
⎧ ⎛p x⎞
195. lim (3n + 5n + 7n ) n is equal to ⎪ sin ⎜⎝ ⎟⎠ , x <1
n ®¥ 207. The function f(x) = ⎨ 2
⎪ 2 x − 3 [ x ], x ≥1
(A) e3 (B) e5 (C) 5 (D) 7 ⎩
196. Which of the following functions have finite number of (A) Is continuous at x = 1
points of discontinuity? (B) Is differentiable at x = 1
x (C) Is continuous but not differentiable at x = 1
(A) tan x (B) x[x] (C) (D) sin [nπ x]
x (D) None of these
Chapter 19 | Limit, Continuity and Differentiability 841
(A) k/2 (B) 2k (C) k (D) None of these (A) e (B) e2 (C) e3 (D) e4
x
210. Let f (x) = ò | t | dt, x ≥ − 1. Then ì-1, x <0
ï
-1 218. If f ( x ) = í0 , x = 0 and g(x) = sin x + cos x, then points of
f and f ′ are continuous for x + 1 > 0 ï1, x >0
(A) î
(B) f is continuous but f ′ is not continuous for x + 1 > 0 discontinuity of f {g(x)} in (0, 2π) is
(C) f and f ′ are derivable at x = 0
(D) f is continuous at x = 0 but f ′ is not ì p 3p ü ì 2p 5p ü
(A) í , ý (B) í , ý
î2 4 þ î 3 3 þ
211. If f(x) = (x − x0) φ(x) and φ(x) is continuous at x = 0, then f ′(x0)
is equal to ì 3p 7p ü ì 5p 7p ü
(C) í , ý (D) í , ý
(A) φ ′(x0) (B) φ (x0) (C) x0φ (x0) (D) None of these î4 4 þ î4 3 þ
212. If f(x) = {| x | − | x − 1 |}2, then f ′(x) equals 219. The value of the derivative of |x − 1| + |x − 3| at x = 2 is
(A) 0 for all x (A) −2 (B) 0 (C) 2 (D) None of these
(B) 2{| x | − | x − 1 |}
220. g: R → R, g(x) = cos−1[sin f(x)] has exactly two elements in
ì0 for x < 0 and for x > 1 range set. Then
(C) í
î4 (2 x - 1) for 0 < x < 1 (A) f(x) must be discontinuous function
(B) f(x) may be continuous function
ì0 for x < 0
(D) í (C) It’s not possible to have such a function
î4 (2 x - 1) for x > 0 (D) f(x) is discontinuous at finite points only
213. If α and β be the roots of the equation (ax2 + bx + 1) = 0, then æ cos x - cos a ö
221. lim ç ÷ is equal to
x ® a è cot x - cot a ø
lim (1 + ax2 + bx + c)1/(x−α) is
x ®a
sin3 a cosec3a
(A) a (α − β) (B) ln |a (α − β)| (A) sin3a (B) cosec3a (C) (D)
2 2
(C) ea(α − β) (D) None of these
222. f(x + y) = f(x) ⋅ f(y) ∀x and y. If f(3) = 3 and f ′(0) = 11, then
214. If f(x) is differentiable and strictly increasing function and f ′(3) is given by
f ( x2 ) - f ( x) (A) 33 (B) 28 (C) 44 (D) 68
f ′(0) ≠ 0, then the value of lim is
x ®0 f ( x ) - f (0 )
cosecx
æ 1+ tan x ö
(A) 1 (B) 0 (C) −1 (D) 2 223. lim ç ÷
x ® 0 è 1+ sin x ø
is equal to
ì1, x ³1
ï 1
(A) (B) 1 (C) 2 (D) e
ï1 1 1 e
215. The function f ( x ) = í 2 , < x < , n = 2 , 3, .
ïn n n -1
224. Let f : R → R be a differentiable function and f(1) = 4. Then the
ï0 , x =0
î f(x)
2t
(A) Is discontinuous at infinitely many points value of lim
x ®1 ò x -1
dt is
4
(B) Is continuous every where
1 (A) 8f ′(1) (B) 4f ′(1) (C) 2f ′(1) (D) f ′(1)
(C) Is discontinuous only at x =
n
225. The value of f(0), so that the function
(D) None of these
a2 - ax + x 2 - a2 + ax + x 2
ì| x | f (x) = becomes continuous
ï , x ¹0 a+ x - a- x
216. If f(x) = 1 + x − [x], g(x) = í x . If h(x) = g(f(x)), then
ïî0 , for all x is given by
x =0
h′(1) and h′(−1) are (A) a (B) − a (C) a3/2 (D) −a3/2
842 Mathematics Problem Book for JEE
æ xö
lncos x
ì
226. The lim ç 2 sin2 ÷ is equal to ï a(1- x sin x ) + b cos x + 5
x ®0 è 2ø ï x <0
ï x2
(A) Does not exist (B) 1 (C) 1/2 (D) 2 ï
5. Let f ( x ) = í 3 x = 0 . If f is contin-
ï 1
ïé æ 3 öù x
Practice Exercise 2 ï ê1+ ç cx + dx
÷ú x >0
ï êë çè x 2 ÷ø úû
î
Single/Multiple Correct Choice Type Questions uous at x = 0, then
(A) a = −1 (B) b = −4 (C) c = 0 (D) d = loge5
⎛ 1 32 1⎞
1. Let Sn = lim ⎜ 6 + 6 + + 6 ⎟ and 6. If x cos α + y sina = x cos b + y sin b = 2 lim (an + b n )1/ n (where
n→∞ ⎝ n n n ⎠ n ®¥
æ p ö -15 ì 1
(C) g ç ÷ = ïï x + 2 , x < 0
è4ø 8 11. f (x) = í , identify the correct statement(s) ([ ]
ï2 x + 1 , x ³ 0
(D) g (x) is non-differentiable at infinitely many points ïî 3
denotes greatest integer function)
é 1ù
4. lim x 5 ê 3 ú is (where [.] represents greatest integer function) (A) lim [f(x)] = 0 (B) lim f ( x ) does not exist
x ®0 ëx û x® 0 x ®0
List I List II
Matrix Match Type Questions
(A) Let f : R → R be such that f(a) = 1, f′(a) = 2
29. Match the following: 1/ x
æ f 2 (a + x ) ö (p) 0
List I List II and lim çç ÷÷ = ek. Then k =
x® 0
è f (a) ø
æ n 4 öæ n 5 ö
ç å x ÷ç å x ÷ cos[tan-1(tan x )]
lim+ =
(A) If lim è nx =1 ø è nx =1 ø = , then t can be
4 p (q) 1
(p) 6 (B) x®
p
x-
n ®¥ æ ö æ ö 5
ç å x ÷ç å x ÷
t 9 -t 2 2
è x =1 ø è x =1 ø
sin(cos x + 1)
lim =
(B) If m be the slope of tangent to the curve xy = yx (C) x ®p æxö (r) 4
(q) 2 cos ç ÷
at (e, e), then (3 − m) can be è2ø
æ 1 1ö xe sin x - e x sin-1(sin x )
(C) If f : R - {0} ® R , f ( x )f ( y ) = f ( xy ) + 3 ç + ÷ , = (s) 3
èx yø (D) xlim
(r) 3 ®0 sin2 x - x sin x
æ 1 ö
then 2 f ç - ÷ can be (t) Does not exist
è 2ø
{ }
(D) If lim ( 2 + 1)n !+ K = 1 , then K can be (where {.} (s) 4
n ®¥
Integer Type Questions
is a fractional part function) 33. f : R → R be a twice differentiable function satisfying
f ′′(x) − 5f ′(x) + 6f (x) ≥ 0 ∀ x ≥ 0, if f (0) = 1, f ′(0) = 0, If f (x) satisfies
(t) 7 f (x) ≥ ah(bx) − bh(ax), ∀ x ≥ 0, then find (a + b) h(0).
1
30. If lim ( x 4 + ax 3 + 3 x 2 + bx + 2 − x 4 + 2 x 3 − cx 2 + 3 x − d ) = 4 , (1+ a3 ) + 8e x
x →∞ 34. If a and b are positive numbers and lim 1
= 2,
x ®0
then match the values of a, b, c and d. 1+ ( 2 + b + b 2
)e x
then find the value of a2 + b2.
List I List II 35. Let f (x)be a differentiable function such that f ′(x) + f (x) = 4xe−x
(A) a (p) 5 n
- pp ep
· sin 2x and f (0) = 0. If lim å f (kp ) = p , then find value of p.
(B) b (q) 0 n ®¥
k =1 (e - 1)2
(C) c (r) 1 1
tan x
(D) d (s) 2 36. Let f ( x ) = and lim ([f ( x )] + x 2 ) { f ( x )} = e l . Then find λ.
x x →0
1 (where [.] and {.} denotes greatest integer and fractional part
(t)
2 function respectively)
Chapter 19 | Limit, Continuity and Differentiability 845
Answer Key
Practice Exercise 1
1. (B) 2. (A) 3. (B) 4. (D) 5. (A) 6. (B) 7. (C) 8. (B) 9. (C) 10. (A)
11. (A) 12. (C) 13. (C) 14. (C) 15. (C) 16. (B) 17. (B) 18. (A) 19. (B) 20. (C)
21. (B) 22. (A) 23. (A) 24. (D) 25. (C) 26. (B) 27. (C) 28. (A) 29. (A) 30. (C)
31. (C) 32. (D) 33. (D) 34. (C) 35. (B) 36. (D) 37. (B) 38. (B) 39. (C) 40. (B)
41. (C) 42. (D) 43. (D) 44. (B) 45. (D) 46. (D) 47. (C) 48. (B) 49. (B) 50. (D)
51. (C) 52. (A) 53. (C) 54. (B) 55. (B) 56. (C) 57. (A) 58. (A) 59. (B) 60. (C)
61. (A) 62. (A) 63. (B) 64. (D) 65. (B) 66. (B) 67. (D) 68. (B) 69. (B) 70. (A)
71. (A) 72. (B) 73. (D) 74. (B) 75. (C) 76. (C) 77. (B) 78. (C) 79. (C) 80. (D)
81. (C) 82. (B) 83. (D) 84. (C) 85. (C) 86. (A) 87. (B) 88. (B) 89. (B) 90. (B)
91. (C) 92. (B) 93. (C) 94. (D) 95. (B) 96. (B) 97. (B) 98. (C) 99. (A) 100. (C)
101. (A) 102. (A) 103. (D) 104. (B) 105. (B) 106. (B) 107. (B) 108. (D) 109. (D) 110. (D)
111. (D) 112. (D) 113. (A) 114. (C) 115. (A) 116. (C) 117. (A) 118. (C) 119. (A) 120. (C)
121. (A) 122. (A) 123. (C) 124. (A) 125. (D) 126. (C) 127. (B) 128. (B) 129. (D) 130. (C)
131. (C) 132. (A) 133. (A) 134. (D) 135. (B) 136. (B) 137. (C) 138. (D) 139. (C) 140. (D)
141. (B) 142. (B), (D) 143. (D) 144. (D) 145. (B) 146. (D) 147. (A), (C ), (D) 148. (D) 149. (A) 150. (A)
151. (B) 152. (B) 153. (D) 154. (D) 155. (B) 156. (D) 157. (C) 158. (C) 159. (A) 160. (C)
161. (B) 162. (D) 163. (B) 164. (B) 165. (D) 166. (C) 167. (A) 168. (D) 169. (C) 170. (B)
171. (B) 172. (B) 173. (B) 174. (D) 175. (D) 176. (D) 177. (D) 178. (D) 179. (C) 180. (C)
181. (D) 182. (C) 183. (C) 184. (A) 185. (B) 186. (C) 187. (C) 188. (B) 189. (D) 190. (C)
191. (D) 192. (B) 193. (A) 194. (C) 195. (D) 196. (C) 197. (D) 198. (D) 199. (C) 200. (A)
201. (D) 202. (B) 203. (C) 204. (B) 205. (C) 206. (B) 207. (C) 208. (B) 209. (C) 210. (A)
211. (B) 212. (C) 213. (C) 214. (C) 215. (A) 216. (A) 217. (D) 218. (C) 219. (B) 220. (A)
221. (A) 222. (A) 223. (B) 224. (A) 225. (B) 226. (B)
Practice Exercise 2
1. (A), (C), (D) 2. (A), (C) 3. (C), (D) 4. (B), (C), (D) 5. (A), (B), (C) 6. (A), (B), (C), (D)
7. (C) 8. (B), (C) 9. (A), (D) 10. (A), (B) 11. (A), (B) 12. (B), (D)
13. (A), (C), (D) 14. (A), (B), (C), (D) 15. (A), (B), (D) 16. (A), (D) 17. (B) 18. (C)
19. (D) 20. (B) 21. (C) 22. (D) 23. (C) 24. (A)
25. (C) 26. (B) 27. (C) 28. (C) 29. (A) → (q), (t); (B)→ (q), (s); (C)→ (q), (r); (D)→ (p), (q), (s)
30. (A)→ (s); (B)→ (p), (q), (r), (s), (t); (C)− (p); (D)→ (p), (q), (r), (s), (t) 31. (A)→ (t), (B)→ (r), (C)→ (q), (D)→ (p)
32. (A)→ (r), (B)→ (q), (C)→ (p), (D)→ (q) 33. 5 34. 2 35. 2 36. 3
Solutions
2
x
= -1 and lim+
x
= 1, hence limit does not exist. æ q qö
4. Since lim-
1- sinq ç cos - sin ÷
x ®0 x x ®0 x
11. lim = lim è 2 2ø
=0
q ®p / 2 cosq q ®p / 2 æ q q öæ q qö
x +h - x ( x + h )2 - ( x )2 1 ç cos - sin ÷ç cos + sin ÷
5. lim = lim = è 2 2 øè 2 2ø
h®0 h h ® 0 h( x + h + x ) 2 x
ì 2 tan 2 x ü
Alternate solution: Apply L’Hospital’s rule, ï - 1ï
tan 2 x - x 1
12. lim = lim í 2 x ý=
x +h - x x ® 0 3 x - sin x x ®0 sin x 2
1 1 ï 3- ï
lim = lim = î x þ
h®0 h h®0 2 x + h 2 x
Alternate solution: Apply L’Hospital‘s rule,
2x −1 2 x log 2
6. lim = lim = 2 log 2 = log
g4
x → 0 (1+ x )1/ 2
− 1 x → 0 1 (1+ x )−1/ 2 tan 2 x - x 2 sec2 2 x - 1 2 - 1 1
2 lim = lim = =
x ®0 3 x - sin x x ®0 3 - cos x 3 -1 2
⎧ f (x) f ′( x ) ⎫
⎨As lim = lim ⎬ ⎡ p ⎤
⎩ x → a g( x ) x → a g ′( x ) ⎭ ⎢ x− ⎥
13. lim 2 ⎢ 2 ⎥ = −2
x -1 x →p / 2 ⎢ ⎛p ⎞⎥
⎢⎣ sin ⎜⎝ 2 − x ⎟⎠ ⎥⎦
1
7. lim =-
x ®1 ( x - 1)(2 x - 5) 3
Alternate solution: Apply L’Hospital’s rule, Alternate solution: Apply L’Hospital’s rule.
x −1 1 1 2x − p 2
lim = =− lim = lim = −2
x →1 2x2 − 7x + 5 4 x − 7 3 x →p / 2 cos x x →p / 2 − sin x
æ pö x ⋅ (2 x − 1) 2x −1 x2
sin ç a - ÷ 17. lim = lim ⋅
è 4ø x → 0 1 − cos x x →0 x 1− cos x
= 2 lim = 2 ´1 = 2
a ® p/ 4 æ pö
ça - ÷ x2
è 4ø = log 2 × lim = (log 2) × 2 = 2 log 2 = log 4
x ®0 x
Alternate solution: Apply L’Hospital’s rule, 2 sin2
2
sin a - cos a cos a + sin a 1 1 tan x - sin x sin x - sin x cos x
lim = lim = + = 2 18. xlim = lim
a ®p / 4 a - (p / 4 ) a ®p / 4 1 2 2 ®0 x3 x ®0 x 3 cos x
Chapter 19 | Limit, Continuity and Differentiability 847
æ xö é x úù æ x3 x5 ö æ x2 x3 x4 ö
sin x ç 2 sin2 ÷ ê sin2 çç x - + - ÷÷ çç - x - - - - ÷÷
è 2ø sin x 2 2 × 1ú = 1 3! 5! 2 3 4
= lim = lim ê × × = lim è ø + lim è ø
x ®0 x 3 cos x x ®0 ê x cos x æ x ö2 4 ú 2 x ®0 x2 x ®0 x 2
ê ç ÷ ú
êë è2ø úû æ x3 x5 x2 x3 ö
çç∵ sin x = x - + - and log(1- x ) = - x - - - ÷÷
19. Let y = xx. Then log y = x log x. è 3! 5! 2 3 ø
Hence, æ 1 1 ö x4
-x2
- x3 ç + ÷ -
lim log y = lim x log x = 0 = log 1Þ lim x = 1 x
2 è3 ! 3ø 4 1
y ®0 x ®0 x ®0 = lim =-
x ®0 x2 2
40 5
æ 1ö æ 1ö æ sin x ö
(2 x + 1) ( 4 x - 1)
40 5ç2 + ÷ ç4 - ÷ æ x + sin x ö ç 1+ x ÷
è xø è xø
20. lim = lim 26. lim ç ÷ = xlim ç cos x ÷ = lim 1 = 1
(2 x + 3)45 45 x ®¥ è x - cos x ø ®¥
x ®¥ x ®¥
æ 3ö çç 1- ÷÷ x ®¥
ç2 + ÷ è x ø
è xø
240 × 4 5 ⎡ sin x cos x ⎤
= = 32 ⎢As xlim and lim both are equal to 0⎥
245 ⎣ →∞ x x →∞ x ⎦
1/ x
1 æ 1+ tan x ö
21. Let tan−1 2 x = q . Then x = tanq and as x → 0, θ → 0. So, 27. Given limit = lim ç ÷
2 x ® 0 è 1- tan x ø
sin nx æ tan x ö ⎛ a b⎞
2x
32. lim n × lim ç (a - n)n - ÷=0 38. Since, lim ⎜ 1+ + 2 ⎟ = e2
x ®0 nx x ®0 è x ø x →∞ ⎝ x x ⎠
1 Hence,
Þ n((a - n)n - 1) = 0 Þ (a - n)n = 1Þ a = n + 2( ax + b )
n
é x ù 2
x
3×
3
+ 3×
1 ( x + 1)( x + 2) x +2 1
3 cos x + 3 sin x = lim = lim =
lim = 2 2= 1 x →−1 ( x + 1)( x + 3) x →−1 x + 3 2
x ®p / 6 6 6 3
1
2
ö 2 æ sin x 43. lim log(1+ x ) = lim 2 log(1+ x) x
-2 sin ç ÷ x ®0 x x ®0
cos(sin x ) - 1 è 2 ø 1 -1
36. lim = lim = -2 × =
x ®0 x 2 x ®0 x 2
4 2 ⎧⎪ 1 ⎫⎪
= lim 2 loge e = 2 ⎨As lim(1+ x ) x = loge e = 1⎬
x →0 x →0
1 ⎩⎪ ⎭⎪
1 1
é 3n ù n
37. lim (3 n
+ 4n )n = lim ( 4 n ) n ê n + 1ú x +1 x +1
n ®¥ n ®¥ êë 4 úû æ 3x - 4 ö 3 æ 3x + 2 - 6 ö 3
44. lim ç ÷ = lim ç ÷
1/ n 1/n x ®¥ è 3 x + 2 ø x ®¥ è 3 x + 2 ø
é ù é ù
ê ú ê ú -6
×
x +1
1 1
= lim 4 ê1+ ú = 4 lim ê1+ ú x +1 é 3 x +2 ù 3 x +2 3
n ®¥ ê 4ö ú n ®¥ ê 4ö ú æ 6 ö 3 æ 6 ö -6 ú
= lim êç 1-
n n
æ
ê ç ÷ ú æ
ê ç ÷ ú = lim ç 1- ÷ ÷
x ®¥ è 3x + 2 ø x ®¥ êè 3x + 2 ø ú
êë è 3 ø úû êë è 3 ø úû êë úû
0 -2( x +1)
é 1ù ⎧ −2( x + 1) −2 ⎫
= 4 ê1+ ú = 4 ´(1)0 = 4 ´ 1 = 4 = lim e 3 x +2 = e -2 / 3 ⎨As lim = ⎬
ë ¥û x ®¥ ⎩ x →∞ 3x + 2 3⎭
Chapter 19 | Limit, Continuity and Differentiability 849
é æ 1ö æ 4 öù 1
ê x ç 1+ x ÷ x ç 3 + x ÷ ú +2(27 x log 27 - 9 x log 9 - 3 x log 3) ( - sin x )
45. ( x + 1)(3 x + 4 ) è ø è øú 2 4 + cos x
lim = lim ê
x ®¥ x 2 ( x - 8) x ®¥ ê 3æ 8ö ú cos x
êë x ç1- ÷ úû
è xø Applying limit, we have
æ æ 1öæ 4öö 2[(log 27)2 − (log 9 )2 − (log 3)2 ]
ç 1 ç 1+ ÷ ç 3 + ÷ ÷ = 4 +1
= lim ç è
x øè x ø÷ 1
=0
x ®¥ ç x æ 8ö ÷
ç ç 1 - ÷ ÷ é 9(log 3)2 - 4(log 3)2 - (log 3)2 ù
è è xø ø = 2ê ú 5
êë 1 úû
46. In closed interval of x = 0 at right hand side [x] = 0 and at left
hand side [x] = −1. Also [0] = 0. Therefore, function is defined as = 5 .8(log 3)2
ì sin[ x ]
ï ( -1 £ x < 0) = 8 5 (log 3)2
f ( x) = í [x]
ï0 (0 £ x < 1)
î xn 1
51. lim = lim =1
Hence, n ®¥ æ 1 ö n ®¥ æ 1 ö
x n ç 1+ n ÷ ç 1+ n ÷
left hand limit = lim f ( x ) = lim
sin[ x ] è x ø è x ø
x ®0 - x ®0 - [x]
sin( −1) 1 éæ 1 ö æ 1 1 ö æ 1 1 ö
= = sin1 52. lim
n®¥ 2 ëè
êç 1- 3 ÷ + ç 3 - 5 ÷ + ç 5 - 7 ÷ +
−1 ø è ø è ø
Right hand limit = 0. Hence, limit does not exist. æ 1 1 öù
+ç -
⎡ 1 ⎤ ç ( 2n - 1) ( 2n + 1) ÷÷ ú
è ø úû
(10 )n ⎢ − 1⎥
⎢⎣ (10 )
n
1− (10 ) n ⎥⎦ 1 1é 1 ù 1
47. lim = lim =− = lim 1- =
n ®¥ 2 ê 2n + 1úû 2
+
n→∞ 1+ (10 )n 1 x →∞ ⎛ 1 ⎞ 10 ë
(10 )n +1 ⎜ 1+ n +1 ⎟
⎝ 10 ⎠
⎛ x2 +1 ⎞
Hence, α = 1. 53. lim ⎜ −a x − b ⎟ = 0
x →∞ ⎝ x + 1 ⎠
log(1+ x 3 ) 3 x 2 / (1+ x 3 )
48. lim = lim
x ® 0 sin3 x x ® 0 3 sin2 x cos x x 2 (1 − a ) − x (a + b ) + 1− b
⇒ lim =0
x →∞ x +1
[By using L’Hospital’s rule]
Since the limit of the given expression is zero, therefore
é 1 æ x ö2 1 ù 1 1 degree of the polynomial in numerator must be less than
= lim ê ç ÷ × ú = × (1)2 × = 1 denominator. Hence,
x ® 0 ê 1+ x 3 è sin x ø cos x ú
ë û 1 + 0 1
1 − α = 0 and α + β = 0
⇒ α = 1 and β = −1
4q (tanq - sinq ) 4q sinq (1- cosq )
49. lim = lim f(x)
q ®0 (1- cos 2q )2 q ® 0 4 sin4 q cosq
ò 4t 3dt
4[f ( x )]3 ´ f ’( x )
⎛ q ⎞ 2 sin2 q / 2 54. lim 6
(0 / 0 form) = lim
= lim ⎜ x ®2 x -2 x ®2 1
q → 0 ⎝ sin q ⎟
⎠ sin2 q cos q
= 4(f(2))3 × f ′(2) = 18
2
2 sin q / 2 1 é 1 2 n ù
= lim 55. lim ê + + +
/ 2)cos(q / 2)]2 cosq 1- n2 úû
q → 0 [2 sin(q
n ®¥ ë 1- n2 1- n2
1 1 1
= lim = ån 1 n2 + n 1
q ®0 2 cos2 (q / 2) × cosq 2 lim = lim =-
n ®¥ 1- n2 2 n ®¥ 1- n 2
2
50. Applying L’Hospital’s Rule, we have
12 + 22 + 32 + + n2 Sn2
56. Given limit = lim = lim
27 log 27 − 9 log 9 − 3 log 3
x x x
n ®¥ 1+ n3 n ®¥ 1+ n3
lim
x →0 1
− ( − sin x ) æ 1 öæ 1ö
1+ 2+ ÷
2 4 + cos x 1 n(n + 1)(2n + 1) 1 çè n ÷ø çè nø
= lim = lim
2(27 x log 27 - 9 x log 9 - 3 x log 3) 4 + cos x n ®¥ 6 1+ n3 n ®¥ 6 æ 1 ö
Þ lim ç 3 + 1÷
x ®0 sin x èn ø
1 2 æ 1ö
Þ lim 2[27 x (log 27)2 - 9 x (log 9 )2 - 3 x (log 3)2 ] 4 + cos x = ×1× = ç ÷
x ®0 6 (1) è 3 ø
850 Mathematics Problem Book for JEE
57. We have 1 - 2 + 3 - 4 + 5 - 6 + - 2n
63. lim
sn +1 - sn an +1 n ®¥
n2 + 1 + 4 n2 - 1
lim = lim =0
n ®¥ n n ®¥ n(n + 1) -2 -2
åk 2
f (x) = y = =
1+ 2 3
k =1
æ 1ö
73. Since lim f ( x ) ¹ f ç ÷ é sin x 2 ù
x ®1/ 2 è2ø 84. f (0 ) = lim f ( x ) = lim f ( x ) = lim x ê 2 ú = 0
x ®0 + x ®0 - x ®0 ê x úû
Hence, the correct answer is option (D). ë
74. f(a) = 0 85. Clearly from curve (Fig. 19.26) drawn of the given function f(x)
æ x2 ö ìï (a - h)2 üï is discontinuous at x = 0.
lim f ( x ) = lim çç - a ÷÷ = lim í - aý = 0
x ®a - x ®a - a
è ø h®0 ïî a ïþ
ïì (a + h) ïü
2
and, lim f ( x ) = lim ía - ý=0
x ®a + h®0 ï a þï
î (0,1/4)
Hence, it is continuous at x = a. O
75. f(0) = 0
lim f ( x ) = lim e -1/ h = 0 and lim f ( x ) = lim e1/ h = ¥ (0, −1)
x ®0 - h®0 x ®0 + h®0
Hence, function is discontinuous at x = 0.
ïì x - 4 x + 3 ïü
2
Figure 19.26
76. f ( x ) = í ý, for x ≠ 1
ïî x - 1 ïþ
2
86. It is obvious that the correct answer is option (A).
f(x) = 2, for x = 1 87. lim f ( x ) = lim x + 1 = 2 = k
x ®1 x ®1
x2 - 4 x + 3 ( x - 3)
f (1) = 2, f (1+ ) = lim = lim = -1 88. (i) When 0 ≤ x < 1, f(x) doesn’t exist as [x] = 0 here.
x ®1+ x2 -1 x ®1+ ( x + 1)
112. For any x ≠ 1, 2, we find that f(x) is the quotient of two polyno-
æ 3p ö
100. Here, f ç ÷ = 1 and lim f (x) =1 mials and a polynomial is everywhere continuous. Therefore,
è 4 ø x ®3p / 4 -
f(x) is continuous for all x ≠ 1, 2. Check continuity at x = 1, 2.
2 æ 3p ö p 113. Since lim f ( x ) = lim f ( x ) = f (2) = 1 .
lim f ( x ) = lim 2 sin ç + h ÷ = 2 sin = 1 x ®2 - x ®2 +
x ®3p / 4 + h®0 9è 4 ø 6
Also it is continuous for all values of x, less than 2 and greater
3p
Hence, f(x) is continuous at x = . than 2.
4
114. Given function is continuous at all point in (−∞, 6) and at
p -p æp ö p
101. lim f ( x ) = , lim f ( x ) = and fç ÷= x = 1, x = 3 function is continuous.
x ®p / 2 - 2 x ®p / 2 + 2 è2ø 2
If function f(x) is continuous at x = 1, then
æ 2 sin2 2 x ö p
102. lim f ( x ) = lim çç
x ®0 - x ®0 - 2 ÷ ÷4 = 8 lim f ( x ) = lim f ( x ) Þ 1+ sin = a + b
è (2 x ) ø x ®1- x ®1+ 2
⇒a+b=2 (1)
lim f ( x ) = lim 16 + x + 4 = 8. Hence, a = 8.
x →0 + x →0 + If at x = 3, function is continuous, then
103. lim f ( x ) = a - b , lim f ( x ) = 2 Þ a - b = 2 3p
x ®1- x ®1+
lim- f (3) = lim+ f ( x ) Þ 3 a + b = 6 tan
x ®3 x ®3 12
All the given sets of a, b make f(x) continuous at x = 1. ⇒ 3a + b = 6 (2)
104. lim f ( x ) = 1+ 1 = 2 lim f ( x ) = 0 , f (0 ) = 2 From Eqs. (1) and (2), a = 2, b = 0
x ®0 - x ®0 +
115. If f is continuous at x = 0, then
105. lim f ( x ) = lim ( x + 2) ( x 2 + 4 ) = 32, f (2) = 16
x ®2 x ®2 lim f (x) = lim+ f (x) = f (0 ) Þ f (0 ) = lim- f (x)
x ®0- x ®0 x ®0
106. lim f ( x ) = 1, lim f ( x ) = 6
x ®1- x ®1+ p
cos [0 - h]
( x − 2) ( x + 5) −7 7 k = lim f (0 - h) = lim 2
107. lim f ( x ) = = = =a h®0 h®0 [0 - h]
x→ −5 ( x + 5)( x − 3) −8 8
p p
108. By definition of continuity, we know that cos [ − h] cos {− h − 1}
k = lim 2 = lim 2
lim f ( x ) = f (3) = lim f ( x )
x ®3 + x ®3 - h→ 0 [ − h] h→ 0 {−h − 1}
Þ lim f ( x ) = 4 or lim 3 - h + l = 4 æ pö
x ®3 - h®0 cos ç - ÷
⇒ 3+ l = 4 ⇒ l =1 k = lim è 2ø ⇒k=0
h®0 -1
1 116. Clearly the function is defined only in the interval [1, ∞)
109. If x → 0, then the value of sin passes through [−1, 1]
x hence option (B) cannot even apply. For x > 2, y = 3x − 2
infinitely many ways, therefore limit of the function does not which is a straight line, hence continuous. Further y = 4 at
exist at x = 0. Hence, there is no value of k for which the func- x = 2. Hence, the function is continuous at x = 2 also (but not
tion is continuous at x = 0. at x = 2 only).
110. As we are given f(x) = sin x, if x ≠ nπ, that is, x ≠ 0, π, 2π, … 117. f(x) is continuous at every point of its domain, so
and f(x)= 2 otherwise. Hence, lim+ g {f ( x )} = lim+ g {sin x } lim f ( x ) = lim+ f ( x ) = f (1)
x ®0 x ®0 x →1− x →1
= lim+ (sin2 x + 1) = 1.
x ®0 ⇒5×1−4=4×1+3×b×1
Similarly, lim g {f ( x )} = 1 ⇒ 1 = 4 +3b ⇒ 3b = −3 ⇒ b = −1
x ®0 -
118. For continuity at all x ∈ R, we must have
4-h-4
111. lim- f ( x ) = lim f ( 4 - h) = lim +a æ pö
x ®4 h®0 h®0 4-h-4 f ç - ÷ = lim - ( -2 sin x ) = lim + ( A sin x + B )
è 2 ø x ® ( -p / 2 ) x ® ( -p / 2 )
h
= lim − + a = a − 1 ⇒ 2 = −A + B (1)
h→ 0 h
æp ö
4+h−4 and f ç ÷ = lim - ( A sin x + B ) = lim + (cos x )
lim f ( x ) = lim f ( 4 + h) = lim + b = b +1 è 2 ø x ®( p / 2 ) x ®( p / 2 )
x →4 + h→ 0 h→ 0 4+h−4
⇒0=A+B (2)
Chapter 19 | Limit, Continuity and Differentiability 853
-1
120. For continuity at x = 0, we must have é 1 ù
Þ lim+ ê x 2 + e 2 - x ú = k Þ k = lim f (2 + h)
f (0 ) = lim f ( x ) x ®2 ê úû h®0
x ®0 ë
x cot x -1
é 1ù é 1 ù
= lim ( x + 1) cot x
= lim ê(1+ x ) x ú Þ k = lim ê(2 + h)2 + e 2 -( 2 + h ) ú
x ®0 x ®0 ê úû h®0 ê úû
ë ë
lim
æ x ö
1 ù x ®0 çè tan x ÷ø
Þ k = lim[ 4 + h2 + 4 h + e -1/ h ]-1
é h®0
= lim ê(1+ x ) x ú = e1 = e
x ®0 ê ú 1
ë û Þ k = [ 4 + 0 + 0 + e -¥ ]-1 Þ k =
4
121. It is obvious that the correct answer is option (A).
122. It is obvious that | x | is continuous for all x. Now, 129. By L’Hospital’s rule, lim f ( x ) is 2. Therefore, for f(x) to be con-
x ®0
1 1 1
1- Therefore, f ([2x]), for all values of x where x <
a continu-
e x -1 e1/ x 1- e ¥ 2
lim- 1 = lim+ = =1 1
x ®0 x ®0
1 +
1 1+ e ¥ ous function and for x = and x =1, f(x) be a discontinuous
e +1
x
e1/ x 2
function.
So, lim f ( x ) exists at x = 0, but at x = 0 it is not continuous.
x ®0
⎧1− cos 4 x
135. We have ⎪ , x≠0
141. f ( x ) = ⎨ 8 x 2
f(x) = 2x − 1, if x > 2, ⎪⎩ k, x=0
f(x) = k, If x = 2 and If f(x) is a continuous function at point x = 0, then
x2 − 1, if x < 2, lim f ( x ) = lim [f ( x )] = lim[f ( x )] = lim [f (0 + h)]
x →0 + x →0 − x →0 h→ 0 −
Therefore,
1− cos 4 h sin2 2h
lim f ( x ) = f (2) Þ lim(2 x - 1) = k Þ k = 3 = lim[f (h)] = lim 2
= lim
x ®2 x ®2 h→ 0 h→ 0 8h h→ 0 4 h2
2
⎛ 2 x − sin−1 x ⎞ ⎛0 ⎞ æ sin 2h ö
136. f ( x ) = lim ⎜ = lim ç ÷ = (1) = 1
2
⎟ = f (0),
x → 0 ⎝ 2 x + tan−1 x ⎠
⎜⎝ form⎟⎠
0 h ® 0 è 2h ø
1- cos 4( -h)
Applying L’Hospital’s rule, = lim- f ( x ) = lim[f (0 - h)] = lim[f ( -h)] = lim
x ®0 h®0 h®0 h®0 8( -h)2
⎛ 1 ⎞
⎜2 − ⎟ = lim
1- cos 4 h
=1
⎝ 1− x 2 ⎠ 2 − 1 1
f (0 ) = lim = = h®0 8h2
x →0 ⎛ 1 ⎞ 2 +1 3
⎜⎝ 2 + ⎟ ⇒ f ( 0 ) = 1⇒ k = 1
1+ x 2 ⎠
sin-1 x ì ex ; x £0
2- ï
x 2 -1 1 = - < x £1
Alternate solution: f (0 ) = lim -1
= = 142. f ( x ) í1 x ; 0
x ®0 tan x 2 +1 3 ï x - 1;
2+ î x >1
x
|x| f ( 0 + h) - f ( 0 ) 1- h - 1
137. |x| is continuous at x = 0 and is discontinuous at x = 0. Rf ¢(0 ) = lim = lim = -1
x h®0 h h®0 h
Hence, f ( x ) = | x | +
|x| f ( 0 - h) - f ( 0 ) e -h - 1
is discontinuous at x = 0. Lf ¢(0 ) = lim = lim =1
x h®0 -h h ® 0 -h
1/ 0 + h
2x2 + 7 e - e -1/ 0 + h e1/ h - e -1/ h
= = lim(0 + h) 1/ 0 + h -1/ 0 + h
= lim h 1/ h -1/ h = 0
( x - 1)( x + 1)( x + 3) h®0 e +e h®0 e +e
Hence, points of discontinuity are x = 1, x = −1 and x = −3 only. e -1/ h - e1/ h
and f (0 - 0 ) = lim f (0 - h) = lim - h =0
ì1- | x | ì 1 x <0 h®0 h®0 e -1/ h + e1/ h
ï , x ¹ -1 ï
140. f ( x ) = í 1+ x and f ( x ) = í1- x and f(0) = 0; hence, f(0 + 0) = f(0 − 0) = f(0)
ïî 1, ïî1+ x , x ³0
x = -1 Hence, f is continuous at x = 0.
At remaining points f(x) is obviously continuous.
ì 1, x <0 Thus, it is everywhere continuous. Again,
ï
ï 1, 0 £ x < 1 f ( 0 - h) - f ( 0 )
ì 1, x <0 ï 2 Lf ¢(0 ) = lim
ï ï h®0 -h
f ( [ 2 x ]) = í 1 - [ 2 x ] Þ f ( [ 2 x ]) = í 1
ï 1 + [2 x ] , x >0 ï 0, £ x £1
î 2 e -1/ h - e1/ h
ï h
-1/ h
ï- 1 , 1£ x < 3 = lim e + e1/ h = -1
ïî 3 2 h®0 -h
Chapter 19 | Limit, Continuity and Differentiability 855
⎧ x2
⎪ = x, x>0
⎧ x2 x
⎪ , x ≠ 0 ⎪⎪
|x − 3| 151. We have f ( x ) = ⎨| x | =⎨ 0, x=0
⎪ 0, x = 0 ⎪ x2
⎩ ⎪
O x=3 = −x, x<0
⎪⎩ − x
Figure 19.27
We have
Now lim- f ( x ) = lim- - x = 0 , lim+ f ( x ) = lim x = 0 and f (0 ) = 0
f ( 3 - h ) - f ( 3) | 3 - h - 3 | -0 h x ®0 x ®0 x ®0 x ®0
Lf ¢(3) = lim = lim = lim = -1
h®0 -h h ® 0 -h h ® 0 -h So, f(x) is continuous at x = 0.
f ( 3 + h ) - f ( 3) | 3 + h - 3 | -0 Also f(x) is continuous for all other values of x.
Rf ¢(3) = lim = lim =1 Hence, f(x) is continuous everywhere. Clearly, Lf ′(0) = −1 and
h®0 h h®0 h
Rf ′(0) = 1. Therefore f(x) is not differentiable at x = 0.
Since L f ′(3) ≠ R f ′(3). Hence f is not differentiable at x = 3.
152. We have
o Trick: Can be seen by graph it is continuous but tangent is not
defined at x = 3. f (1+ h) - f (1) [(1+ h)3 - 1] - 0
Rf ¢(1) = lim = lim =3
147. x ≤ x2 ⇒ x(1 − x) ≤ 0 ⇒ x(x − 1) ≥ 0 h®0 h h®0 h
é x: x £0 f (1- h) - f (1) [(1- h) - 1] - 0
ê 2 Lf ¢(1) = lim = lim =1
Þ x £ 0 or x ³ 1; therefore h( x ) = ê x : 0 < x < 1 h®0 -h h®0 -h
ê x: x ³1
ë Hence,
h(x) is continuous for every x but not differentiable at x = 0 R f ′(1) ≠ L f ′(1) ⇒ f (x) is not differentiable at x = 1
and 1. Also Now,
é 1 x <0 f (1+ 0 ) = lim f (1+ h) = 0
h®0
ênot exists x =0
ê and f (1- 0 ) = lim f (1- h) = 0
¢
h (x) = ê 2x 0 < x <1 h®0
ê
ênot exists x =1 Hence,
ê 1 x >1 f(1 + 0) = f(1 − 0) = f(0) ⇒ f (x) is continuous at x = 1
ë
Hence, at x = 1, f(x) is continuous and not differentiable.
Therefore, h′(x) = 1 for all x > 1.
153. Here, when −1 ≤ x ≤ 1, 0 ≤ x sin π x < 1. So,
148. It is obvious that the correct answer is option (D).
f (x) = [x sin π x] = 0 for −1 ≤ x ≤ 1,
ìx, 0 £ x £1
149. f ( x ) = í That is, f (x) is constant function (equal to zero) in [−1, 1].
î 1, 1 £ x £ 2
Therefore, f (x) is differentiable in (−1, 1).
lim- f ( x ) = lim f (1- h) = lim(1- h) = 1 =
x
x ®1 h®0 h®0 y y=1 154. Since |x − 3| = x − 3, if x ≥ 3;
lim+ f ( x ) = lim f (1+ h) = 1 |x − 3| = −x + 3, if x < 3
x ®1 h®0 O x=1
Hence, the given function can be defined as
Hence, function is continuous in (0, 2).
ì1 2 3 13
Now ï4 x - 2 x + 4 , x <1
lim f ( x ) = lim(0 + h) = 0 = f (0 ) Figure 19.28 ï
x ®0+ h®0 f (x) = í 3 - x, 1£ x < 3
ï x - 3, x ³3
lim f ( x ) = lim(2 - h) = 1 = f (2) ï
x ®2- h®0 î
856 Mathematics Problem Book for JEE
ìïe - x , x ³0 and
157. We have, f ( x ) = í lim f ¢( x ) = lim f ¢(0 + h) = lim 2(0 + h) = 0
x
ïî e , x <0 x ®0+ h®0 h®0
166. (gof )(x) = g[f (x)] = g[1 − cosx] = e1−cos x, for x ≤ 0 ⇒ f (0) = 0
(gof )′(x) = e1−cos x · sin x, for x ≤ 0 And since there are infinitely many points in the neighbour-
(gof )′(0) = 0 hood of x = 0. Such that f(x) remains constant in the neigh-
bourhood of x = 0. Therefore,
f (1+ h) - f (1)
167. f ¢(1) = lim ; as function is differentiable so it f ′(0) = 0
h®0 h
858 Mathematics Problem Book for JEE
173. f(1) = −3; f ′(x) ≥ 9 for all x ∈ (1, 5); hence, f(5) ≥ 36. ì x + 2, -1 < x < 3
ï
174. We know, 181. If f (x) = í5, x = 3 and f(3) = 5
f ( x + h) − f ( x ) ï8 - x , x >3
f ′( x ) = lim î
h→0 h f ( x ) - f ( 3) f ( 3 - h ) - f ( 3)
Given f(x + y) = f(x) f(y), so L.H.D = lim- = lim
x ®3 x -3 h®0 -h
f ( x )f (h) − f ( x )
f ′( x ) = lim (3 - h + 2 ) - 5 -h
h→0 h = lim = lim =1
h®0 -h h ® 0 -h
f ( x )(f (h) − 1)
⇒ f ′( x ) = lim f ( x ) - f ( 3) f ( 3 + h ) - f ( 3)
h→0 h R.H.D = lim+ = lim
f ( x )(1+ sin(3h)g( h) − 1) x ®3 x 3- h ® 0 h
⇒ f ′( x ) = lim
h→0 h 8 - (3 + h) - 5 -h
= lim = lim = -1
⇒ f ′( x ) = lim 3f ( x )g(h) = 3f ( x )g(0 ) h®0 h h®0 h
h→0
L.H.D ≠ R.H.D f (x) is not differentiable.
⎧1 ∀x < 0 ìx, 0 £ x £1
⎪ 182. f ( x ) = í
175. f ( x ) = ⎨ p
⎪⎩1+ sin x , ∀0 ≤ x ≤ 2 î2 x - 1, x > 1
lim f ( x ) = lim f (1- h) = lim (1- h) = 1
x ®1- h®0 h®0
⎧0 , ∀ x < 0 (LHD)
⇒ f ′( x ) = ⎨ lim f ( x ) = lim f (1+ h) = lim 2(1+ h) - 1 = 1
⎩cos x , 0 ≤ x ≤ p / 2, (RHD) x ®1+ h®0 h®0
ì 2
ïï for x <1 y
= x
Þ y ¢ = í 1+ x
2
1
− +
1
ï -2
x =
for x >1 y=2
y
ïî1+ x 2
Hence for | x | = 1, the derivative does not exist.
186. Since the function is defined for x ≥ 0, that is, not defined for
x < 0. Hence, the function neither continuous nor differentia- −1 0 1
ble at x = 0.
But f(0) = 1 é ( a2 - b 2 ) x ( a3 + b 3 ) 2 ù
So, limit does not exist. lim ê(a + b ) - + x + ú = (a + b )
x ®0 ê 2 ! 3 ! úû
ë
190. f(x) = [α + β sin x] x ∈ (0, π)
1/ n
é æ 3 ön æ 5 ön ù
β 195. lim (3n + 5n + 7n )1/ n = lim 7 ê1+ ç ÷ + ç ÷ ú
ë è 7 ø è 7 ø úû
n ®¥ n ®¥ ê
4
1/ n
3 é æ 3 ön æ 5 ön ù
2 = lim 7 ê1+ ç ÷ + ç ÷ ú = 7 ´ e0 = 7
ë è 7 ø è 7 ø úû
n ®¥ ê
1
x
196. f ( x ) =
x
Figure 19.31
ì-1 x < 0
Þ f (x) = í
Total number of points of discontinuous are 2β − 1 (Fig. 19.31). î1 x > 0
191. [x + [x]] = [x] + [x] = 2[x] So, this function is discontinuous at only 1 point.
2m / x
æ x 3x ö 197. lim ( -1)[ x ] = ( -1)2 = 1, lim- ( -1)[x] = ( -1)1 = -1
192. A = lim ç sin + cos ÷ x ®2+ x ®2
x ®0 è m mø
lim( -1) [x]
does not exist.
2m æ x 3x ö æ0 ö x ®2
ln A = lim ln ç sin + cos ÷ ç form ÷
x ®0 x è m mø è0 ø
1+ sin x - cos x + log(1- x ) æ0ö
Applying L’Hospital’s rule 198. lim ç ÷ form
x ®0 x3 è0ø
2m 1æ x 3x ö 1 Applying L’Hospital’s form, we get
lim × cos - 3 sin ÷ = 2 (1- 0)) = 2
x ®0 æ x 3 x ö m çè m mø 1+ 0
ç sin + cos ÷ 1
è m mø cos x + sin x + ( −1) 0
1− x ⎛ ⎞
lim 2 ⎜⎝ ⎟⎠
Therefore, A = e2. x →0 3x 0
x −1 f ′ (1) 6
= e f (1) = e 3 = e 2
x cos x - log(1+ x ) æ 0 ö
200. lim ç ÷
x ®0 x2 è0ø 206. LHL RHL
1 lim g[f ( x )] lim g[f ( x )]
cos x - x sin x - x ®0 - x ®0 +
= lim 1+ x æ 0 ö
ç ÷ Þ lim g[f (0 - h)] = lim g[f (0 + h)]
x ®0 2x è0ø h®0 h®0
x +1 + x −1 − 2 =1 =1
201. lim Therefore, LHL = RHL = 1.
x →0 x
x + 1- ( x - 1) - 2 207. LHL RHL
= lim
x ®0 x p lim 2(1+ h) - 3 [1+ h]
lim sin (1- h) h®0
2−2 h®0 2
= lim =0
x →0 x =1 = |−1| × 1
=1
a tan x - asin x Function is continuous but not differentiable at x = 1.
202. lim
x ® 0 tan x - sin x
(cos x - 1)(cos x - e x ) 2 sin2 ( x / 2)(e x - 1 + 1- cos x )
a sin x
(a tan x - sin x
- 1) 208. lim n
= lim
= lim x ®0 x x ®0 xn
x ®0 tan x - sin x
2 sin2 ( x / 2)(e x - 1+ 2 sin2 ( x / 2)]
= a0⋅log
a lim
x ®0 xn
= log a 2
1 é sin( x / 2) ù (e x - 1+ 2 sin2 ( x / 2)]
= lim ê
1 2 n −1 x ®0 2 ë x / 2 úû x n -2
1+ e n + e n + + e n
It is non-zero, if
203. lim
n→∞ n n−2=1⇒ n= 3
n -1 r Alternative method:
1
=å en ×
r =0 n
(cos x − 1)(cos x − e x )
lim
1 x →0 xn
= ò e x dx
⎛ x2 x 4 ⎞
⎜ − 2! + 4 ! + ⎟ ( − x − x + ))
2
0
⎝ ⎠
= [e x ]10 = lim is non-zero
x →0 xn
=e−1 ⇒n=3
−1
p − cos x log n log(n + 1) log(n + 2) log nk
204. lim+ 209. lim ×
x →−1 x +1 n ®¥ log( n - 1) log n log(n + 1) log nk - 1
210. Here, 1+ { x }
x x Þ h( x ) = {x2} ≥ 0
ét | t |ù x|x| 1 1+ { x }
f ( x ) = ò | t | dt = ê ú = 2 +2
-1 ë 2 û -1
1+ { x }
1 Þ h( x ) = =1
Þ f ¢( x ) = (| x | + | x |) =| x | 1+ { x }
2
Both f and f ′ are continuous for x + 1 > 0. h(x) = 1
ª h′(x) = 0 for both x = 1 and −1.
f ( x ) − f ( x0 )
211. f ′( x 0 ) = lim
x → x0 x − x0 217. 1∞ form
( x − x 0 )f ( x ) − 0 ⎡f ( x ) = ( x − x 0 ) f ( x )⎤
= lim ⎢ ⎥ lim é x + 5 x + 3
2 ù
x → x0 x − x0 ⎣ f ( x0 ) = 0 ⎦ L = e x ®¥ ê 2 - 1ú x
212. f(x) = {|x| − |x − 1|}2 êë x + x + 2 úû
æ 1ö
⎧ x 2 + x 2 − 2 x + 1− 2 x 2 + 2 x , x2 ç 4 + ÷
x≤0 lim è xø
⎪⎪ = e x ®¥
⇒ f ( x ) = ⎨ x 2 + ( x − 1)2 + 2 x 2 − 2 x , 0 ≤ x ≤1 æ 1 2 ö
x 2 ç 1+ + 2 ÷
⎪ 2 è x x ø
⎪⎩ x + x − 2 x + 1− 2 x + 2 x , x ≥1
2 2
= e4
ì1 x £0 218. g(x) = sin x + cos x
ï
f ( x ) = í4 x 2 - 4 x + 1 0 £ x £ 1 æ pö
ï1 = 2 sin ç x + ÷
î x ³1 è 4ø
+ − +
ì0 for x < 0 and for x > 1
So, f ¢( x ) = í 3π 7π
î4(2 x - 1) for 0 < x < 1 2π
4 4
3π 7π
213. lim (1+ ax 2 + bx + c )1/( x -a ) [Since ax2 + bx + c = a(x − α)(x − β)] f [g(x)] is discontinuous at and .
x ®a 4 4
a( x - b )
⎧ 3p
⎪+1, x < 4
lim a( x - b )
= lim [1+ a( x - a )( x - b )] a( x -a )( x - b ) = e x ®a = e a (a - b )
x ®a ⎪
⎪0 , x = 3p
f ( x2 ) - f ( x) æ 0 ö ⎪ 4
214. lim ç form ÷ f [ g( x )] = ⎨
x ®0 f ( x ) - f (0 ) è 0 ø ⎪ −1, 3p < x < 7p
⎪ 4 4
2 xf ′( x 2 ) − f ′( x ) ⎪ 7p
= lim = 0 − 1 = −1 ⎪1, x>
x →0 f ′( x ) ⎩ 4
215. The function f is clearly continuous for | x | > 1 219. f(x) = |x − 1| + |x − 3|
1 1 1 ⎧ −2 x + 4 , x ≤ 1
lim f ( x ) = 1, lim- f ( x ) = = , lim f ( x ) = 1, lim - f ( x ) = ⎪
x ®1+ x ®1 22 4 x ®-1+ x ®( -1) 4 ⇒ f ( x ) = ⎨2, 1< x < 3
⎪2 x − 4 , x ≥3
1 1 ⎩
Also, lim+ f ( x ) = , lim f ( x ) = .
x®
1 n2 x ® 1 - (n + 1)2 ⇒ f ′(x) at x = 2 is zero
n n
220. Obviously function must be discontinuous. Hence, the
1
Thus, f is discontinuous for x = ± , n = 1, 2, 3, … correct answer is (A).
n
cos x - cos a æ 0 ö
216. h (x) = g[f(x)] 221. lim ç ÷
x ®a cot x - cot a è 0 ø
1+ x - [ x ] - sin x
h( x ) = Þ lim = sin3a
1+ x - [ x ] x ® a -cosec 2 x
862 Mathematics Problem Book for JEE
f ( 3 + h ) - f ( 3) 1 1
222. f ¢(3) = lim n
r5 x6 1
h®0 h Sn = lim å 6
= ò x 5 dx = =
n ®¥ 6 6
r =1 n 0 0
f ( 3) × f ( h ) - f ( 3)
= lim
h®0 h
[f (h) - f (0 )]
= lim f (3)
h®0 h
= f (3) × f ¢(0 ) = 3 ´ 11 = 33
cosec x
⎛ 1+ tan x ⎞
223. L = lim ⎜ ⎟
x → 0 ⎝ 1+ sin x ⎠
0 1
lim æ 1+ tan x - 1- sin x 1 ö
L = e x ®0 ç ´ ÷
è 1+ sin x sin x ø
lim tan x - sin x 1
L = e x ®0 ´ Figure 19.33
1+ sin x sin x
+ -
æ 1 ö æ 1ö æ 1ö
ç cos x - 1 ÷
lim Þ Sn = ç ÷ ; Tn = ç ÷
L=e ç
x ®0
÷ è6ø è6ø
è 1+ sin x ø
But since x5 is concave upward (Fig. 19.34) the area included
= e0 = 1 in Sn for any two consecutive values of r is more than area
f(x) excluded in Tn for same values of r. So,
ò 2t dt
æ0ö 1 1 1
224. lim 4 Sn − > − Tn ⇒ Sn + Tn >
x ®1 x - 1 è 0 ø
ç ÷ form 6 6 3
2f ( x ) × f ¢( x )
= lim
x ®1 1
= 2f(1) f ΄(1) included in Sn excluded in Tn
= 8f ΄(1)
a2 - ax + x 2 - a2 + ax + x 2
225. lim
x ®0 a+ x - a- x
On rationalising, we get
Figure 19.34
a2 − ax + x 2 − ( a2 + ax + x 2 ) a+ x + a− x
= lim ×
x →0 a + x − ( a − x) a − ax + x 2 + a2 + ax + x 2
2 x
æ e x + e2 x + e3 x ö
l nç ÷
ç ÷
2. f { g[n, h( x )]} = sinp ([n] + [n] ) 2 3
è ø
-2ax a+ a
= lim ´
x ®0 2x a2 + a2
a 1
=- =- a
a lim f { g[n, h( x )]} = sinp ([n] + [n]2 ) 2
x ®0
lncos x
⎛ x⎞
y = lim ⎜ 2 sin2 ⎟ = lim (1− cos x )lncos x = lim(1− x )ln x æ
226. 1 ö ö
1/ 2
x →0 ⎝ 2⎠ æ æ p 1 p ö
x →0 x →1
Þ lim sin ç p [n] ç 1+ ÷ ÷ = lim sin ç p [n] + - ´ ¼÷ = 1
n ®¥ ç è [n] ø ø ÷ n ®¥ è 2 [n] 8 ø
è
ln(1- x )
Þ ln y = lim ln x ln(1- x ) = lim = 0 (using L ’Hospital rule) æ p 1 ö
x ®1 x ®0 1 lim lim g[n, h( x )] = lim sin ç p [n] + - + ÷
ln x
n ®¥ x ® 0 n ®¥ è 2 8[n] ø
Does not exist as value can be 1 or −1.
Practice exercise 2 æ 1ö æ 1ö
3. lim tan ç ÷ ln ç ÷ = 0 Þ f ( x ) = 1
⎛ 1 32 243 1⎞
n ®¥ ènø ènø
1. Sn = lim ⎜ 6 + 6 + 6 + + 6 ⎟ (see Fig. 19.33)
n→∞ ⎝ n n n n ⎠ dx -3 3
Þò 11/ 3 1/ 3
= (tan x )-8 / 3 - (tan x )-2 / 3 + C
sin x cos x 8 2
æ 1 32 243 ( n - 1)5 ö÷
Tn = lim ç 6 + 6 + 6 + + æp ö 15
n ®¥ ç n n n n6 ÷ø gç ÷ = -
è è ø
4 8
Chapter 19 | Limit, Continuity and Differentiability 863
é 1ù æ 1ö 1
2
4. y = lim x 5 ê 3 ú Also lim f ( x ) = ç ÷ =
x ®0 ëx û x®1 è3ø 9
æ 1 ö é 1ù 1
x 5 ç 3 - 1÷ < x 5 ê 3 ú < x 5 3 8. By standard results, the correct answers are options (B) and (C).
è x ø ë û
x x
9. If x ∈ Q, then n!πx → multiple of π
x2 − x5 < y < x2 ⇒ y → 0
cos(n!πx) → ±1
1/ h
⎪⎧ ⎛ ch + dh ⎞ ⎪⎫
3 1+1=2
5. RHL = lim f ( x ) = lim ⎨1+ ⎜ 2 ⎟⎬ = e to exist the limit
d
If x ∉ Q then cos(n! πx) be any number between −1 and 1.
x →0 h→ 0 ⎪
⎩ ⎝ h ⎠ ⎭⎪
lim [1 + [any no between −1 & 1]2m]
m®¥
c = 0,
LHL = lim f (0 - h) [1] = 1
h®0
sin(tan t ) tan t
a(1- h sin h) + b cos h + 5 a + b cos h + 5 ah sin h 10. lim = lim =1
= lim = lim - t ®0 sin t t ®0 t
h®0 h2 h®0 h2 h2
sin(cos x )
b
Limit is possible if a + b + 5 = 0 and -a - = 3 lim =1
x →p / 2 cos x
2
|x|
On solving, lim = DNE
x →0 x
a = −1, d = loge 3, c = 0, b = − 4
1- cos x 4
6. x cos α + y sin α = x cos b + y sin b lim = 2
1/ n
x ®p / 2 x2 p
ïì æ b ö ïü
n
= 2 × lim a í1+ ç ÷ ý = 2a (1) é 1ù é 1ù
11. (A) lim x + ú = 0 , lim ê2 x + ú = 0
îï è a ø þï x ® 0- ê
n ®¥
ë2û x ® 0+ ë 3û
é b æbö
n ù Hence, lim [f(x)] = 0.
êAs 0 < < 1 Þ ç ÷ ® 0 as n ® ¥ ú x ®0
êë a èaø úû
æ 1ö 1 æ 1ö 1
(B) lim ç x + ÷ = , lim ç 2 x + ÷ =
Equation (1) shows that α and b are the roots of x cos q + y sin q x ® 0- è 2 ø 2 x ® 0+ è 3ø 3
= 2a (2)
Therefore, Since, f(0−) ≠ f(0+).
f ( x + h) - f ( x ) f ( x ) + f ( h) + xh( x + h) - f ( x ) AB PA q PA PA
Þ lim = lim 17. = ⇒ = =
h®0 h h®0 h DC PD sinq PA − DA PA − (1− cosq )
f ( h) ⇒ q · PA − q (1 − cos q ) = PA · sin q
= lim + lim x (x + h) = −1 + x2
h®0 h h®0 q (1- cosq )
Þ PA =
⇒ f ′(x) = −1 + x2 (q - sinq )
x3 q (1- cosq )
⇒ f (x) = −x+c lim =3
3 q ®0+ q - sinq
Hence, f(x) is a polynomial function, f(x) is twice differentiable
OC 1
for all x ∈ R and f ′(3) = 32 − 1 = 8. 18. In ΔOCQ, cos θ = = (Fig. 19.36)
OQ OQ
-1 x
1 æ t2 ö
x x
1
14. f ( x ) = ò | t + 1| dt = - ò ( t + 1) dt + ò (t + 1) dt = 2 + çç 2 + t ÷÷ OQ =
cosq
-2 -2 -1 è ø -1
x2 C
= + x + 1 for x ≥ −1
2
f(x) is a quadratic polynomial.
Therefore, f(x) is continuous as well as differentiable in [−1, 1].
Also f ′(x) is continuous as well as differentiable in [−1, 1]. P O D A Q
ì 1
ï x + 1, 0 £ x < 1
ï
ï2
15. f (x) = í , 1£ x < 2
ïx Figure 19.36
ï 3 5
ï x - 1, 2 £ x < 2 Area of ΔCPQ =
1
⋅ PQ⋅DC =
1
î (PO + OQ)⋅DC
2 2
Clearly, f(x) is discontinuous and bijective function (see
Fig. 19.35). 1
= {PA − OA + OQ}⋅DC
2
1 ìq (1- cosq ) 1 ü
= í - 1+ ý ⋅sin θ
2 î q - sinq cosq þ
3
1 ìï sin2 q (1- q cot q ) üï
= ítanq + ý
2 îï (q - sinq ) þï
2
ΔCPQ 1 ⎧q sinq − 1+ cosq ⎫
19. lim+ = lim ⎨ ⎬
1
q →0 sin2 q q →0+ 2 ⎩ cosq (q − sinq ) ⎭
½ 1 1 ⎧ h sin h − 1+ cos h ⎫
= lim lim ⎨ ⎬
2 h→0 cos h h→ 0 ⎩ h − sin h ⎭
O 1 2 5/2
1 ⎧ h cos h + sin h − sin h ⎫
= ⋅1⋅ lim ⎨ ⎬ L-Hospital rule
Figure 19.35 2 h→ 0 ⎩ 1− cos h ⎭
1 1 ⎧ h cos h ⎫
lim f ( x ) = = lim ⎨ ⎬
x ®1- 2 2 h → 0 ⎩1− cos h ⎭
lim f ( x ) = 2
x ®1+ 1 ⎧ − h sin h + cos h ⎫
= h lim ⎨ ⎬ L-Hospital rule
2 h→ 0 ⎩ sin h ⎭
min é lim f ( x ), lim+ f ( x )ù = 1 ≠ f(1)
ëê x ®1- x ®1 ûú 2 1
⇒ = not defined
max (1, 2) = 2 = f(1) 0
Chapter 19 | Limit, Continuity and Differentiability 865
é 1 ù
⇒ h(x) = tan−1 êln æç ln ö÷ ú for 0 < x < 1
2
ë è x øû
27.
ln f ( x ) -x / 2 1
20. lim+ = lim =-
x ®0 ln g( x ) x ® 0 + x 2
(B) x y = y x ⇒ y In x = x In y 1/ x æ f 2 ( a + x )- f ( a ) ö
é f 2 (a + x ) ù lim ç
x ®0 xf ( a )
÷
Differentiating with respect to x, we get 32. (A) lim ê ú =e è ø
x® 0ê
ë f (a) úû
( x ln y - y ) y 2 (ln x - 1)
y¢ = = 2f ( a + x ) - f ( a + x )
( y ln x - x ) x 2 (ln y - 1) lim
= e x ®0 f (a) = e4
y (ln x - 1)
2
y Hence, k = 4.
y ¢ = lim = lim
x ®e x (ln y - 1)
2 x ® e x y¢ cos[tan-1(tan x )]
y ®e y ®e (B) lim +
p p
⇒ y ′ = ±1 x®
2
x-
2
So, m = ±1 and 3 − m is equal to 2 or 4.
(C) Put x = y = 1, then é æp öù
cos tan-1 ê tan ç + h ÷ ú
[f (1)]2 = f (1) + 6 ⇒ f (1) = 3, −2 ë è2 øû
lim
3 æ 1+ x ö æ 1+ x ö h®0+ h
Put y = 1, we get f ( x ) = ç ÷ or f ( x ) = - ç ÷.
2è x ø è x ø ⎛ p⎞
cos ⎜ h − ⎟
⎝ 2⎠ sin h
(D) If m is even ( 2 + 1) + ( 2 − 1) = 2I , then
m m
lim+ = lim+ =1
h→ 0 h h→ 0 h
lim {( 2 + 1)n !+ k } = lim {2I − ( 2 − 1)n !+ k }
n→∞ n→∞ é æ x öù
sin ê2 cos2 ç ÷ ú
= lim { −( 2 − 1)n !+ k } = 1 sin(cos x + 1) ë è 2 øû æ xö
n→∞ (C) lim = lim × ç 2 cos ÷
x ®p æ ö
x x ® p æ 2 ö
x è 2ø
⇒ n! + k must be even integer ⇒ k is even cos ç ÷ ç 2 cos ÷
è2ø è 2ø
30. After rationalizing, we get = 1´ 0 = 0
( a - 2 ) x 3 + ( 3 + c ) x 2 + ( b - 3) x + ( 2 + d )
lim =4 xe sin x - e x sin-1(sin x ) xe x e sin x - x - 1
x ®¥
x 4 + ax 3 + 3 x 2 + bx + 2 + x 4 + 2 x 3 - cx 2 + 3 x - d (D) lim = lim × =1
x® 0 sin2 x - x sin x x ® 0 sin x sin x - x
Since limit is finite, so degree of numerator must be 2. So,
a−2=0⇒a=2 33. Given inequality can be written as
Now, dividing numerator and denominator by x2 , we get
f ′′(x) − 2f ′(x) ≥ 3[f ′(x) − 2f (x)]
b -3 2+d
(3 + c ) + + 2 Let f ′(x) − 2f (x) = g(x). Then
lim x x =4
x ®¥ a 3 b 2 2 c 3 d g ′(x) − 3g(x) ≥ 0 (Multiply e−3x)
1+ + 2 + 3 + 4 + 1+ - 2 + 3 - 4 −3x −3x
⇒ [g(x)e ] ≥ 0 ⇒ g(x)e is non-decreasing
x x x x x x x x
Now
3+c g(0) = f ′(0) − 2f (0) = −2
Þ =4Þc =5
2 g(x)e−3x ≥ −2, ∀x ≥ 0
Hence, a = 2, c = 5 and b, d can be any real number. f ′(x) − 2f (x) ≥ −2e3x, ∀x ≥ 0 (Multiply e−2x )
Hence, the correct answer is (A) → (s); (B) → (p), (q), (r), (s), (t); −2x
⇒ [f (x)e ] ≥ −2e , ∀x ≥ 0
x
(C) → (p); (D) → (p), (q), (r), (s), (t). ⇒ [f (x)e−2x + 2ex] ≥ 0
x2 ⇒ f (x)e−2x + 2ex ≥ 3
x2
f (t ) - t
ò (f (t ) - t ) dt ⇒ f (x) ≥ 3e2x − 2e3x, ∀x ≥ 0
31. (A) lim
x ®1 ò ( x - 1)2
dt = lim
x ®1
1
( x - 1)2
Comparing ah(bx) − bh(ax) with 3e2x − 2e3x, we get
1 h(x) = ex, a = 3, b = 2
2 x (f ( x 2 ) - x 2 ) f ( x 2 ) - x 2 + 2 x 2f ¢( x 2 ) - 2 x 2 ⇒ (a + b) h(0) = 5
= lim = lim =4
x ®1 2( x - 1) x ®1 1 (1+ a3 ) + 8e1/ x
34. RHL = lim+ =2
⎛1+ n 4 ⎞
n
⎛ n 4 − 1⎞
n x ®0 1+ (2 + b + b2 )e1/ x
(B) lim ⎜ ⎟ = lim ⎜ 1+
n → ∞⎝ 2 ⎠ n → ∞⎝ 2 ⎟⎠ ⇒ 2 + b + b2 = 4 ⇒ b2 + b − 2 = 0 ⇒ b = 1
41/ n −1 1 41/ n −1 1 (1+ a3 ) + 8e1/ x
lim ⋅n lim ln 4 LHL = lim- =2
=e n→∞ 2 =e n→∞ 2 1/ n = e2 =2 x ®0 1+ (2 + b + b2 )e1/ x
2x ⇒ 1 + a3 = 2 ⇒ a = 1
(C) f ( x ) = lim tan-1(nx ) = x , x > 0
n®¥ p
35. Let f(x) = y. Then
Hence,
dy
lim [f ( x ) - 1] = lim + [ x - 1] = -1 + y = 4 xe - x × sin 2 x
x ®0 + x ®0 dx
é I.F = ex
1ù n
é 1ù
(D) lim ê å r ú = lim ê1- n ú = 0
n ® ¥ê ë 2 û ye x = 4 ò x sin 2 x dx
ë r =1 2 úû
n ® ¥
I II
Chapter 19 | Limit, Continuity and Differentiability 867
é æ cos 2 x ö 1 ù e -p 1
ye x = 4 ê x ç - ÷ + ò cos 2 xdx ú S(1- e -p ) = -p
= p
ë è 2 ø 2 û 1- e e -1
æ x cos 2 x sin 2 x ö 1 ep
= ye = 4 ç - + ÷+c
x
S= = p
è 2 4 ø p -p
(e - 1)(1- e ) (e - 1)2
⇒ yex = (sin 2x − 2x cos 2x) + c
f(0) = 0 ⇒ c = 0 Therefore,
y = e−x (sin 2x − 2x cos 2x)
n
-2p ep
Therefore, lim å f (kp ) =
Now, n ®¥
k =1 (ep - 1)2
f (kπ) = e−kπ (sin 2kπ − 2kπ cos 2kπ) = e−kπ (0 − 2kπ) Hence, p = 2.
f (kπ) = −2π(ke−kπ ) 1
¥ 1 tan x x
−1
å f (kp ) = 2p å ke - kp
36. lim ([f ( x )] +
x →0
x 2 ) { f ( x )} = lim (1+ x
x →0
2
) x = lim (1+ x 2 ) tan x − x
x →0
k =1
-p -2p -3p
S = 1× e + 2e + 3e + ¥ lim
x3
lim
3x2
lim
6x
= e x ®0 tan x - x = e x ®0 sec x -1 = e x ®0 2 sec = e3
2 2
x tan x
-2p -3p
S = e + 2e + ¥
_____________________________ Therefore, l = 3.
-p -p -2p -3p
S(1- e )=e +e +e + ¥
868 Mathematics Problem Book for JEE
⎛ 1⎞ ⎛ 6x x ⎞ 3( x − 3) 2x − 4 + 2
2. If for x ∈⎜ 0 , ⎟ , the derivative of tan−1 ⎜ ⎟ is x ⋅ g( x ), ⇒ lim ×
⎝ 4⎠ ⎝ 1− 9 x 3 ⎠ x →3 3x + 3 2( x − 3)
then g(x) equals
3 2x − 4 + 2 3 2×3−4 + 2
3x x 3x ⇒ lim = ×
(A) (B) x →3 2 3x + 3 2 3×3 +3
1− 9 x 3 1− 9 x 3
3 ( 2 + 2)
(C)
3
(D)
9 = ×
2 3+3
1+ 9 x 3 1+ 9 x 3
(OFFLINE) 3 2 2 2 2 2 1
= × = = =
Solution: We have 2 6 4 2 2
⎛ 6x x ⎞ ⎛ 2(3 x x ) ⎞ Therefore,
y = tan−1 ⎜ 3⎟
= tan−1 ⎜ ⎟ 3x − 3 1
⎝ 1− 9 x ⎠ ⎝ 1− (3 x x )2 ⎠ lim =
x →3 2x − 4 − 2 2
= 2 tan−1(3 x x )
Hence, the correct answer is option (D).
Now, differentiating w.r.t. x, we get
1a + 2a + + na 1
dy 2 ⎛ 1 ⎞ 4. If lim = for
= 3⎜( x) + x (1)⎟ n→∞ [( n + 1) a −1
[(na + 1) + (na + 2) + + (na + n)] 60
dx 1+ (3 x x )2 ⎝ 2 x ⎠
some positive real number a, then a is equal to
6 ⎛ x ⎞ 9 x ⎛ 9 ⎞
= ⎜ 2 + x⎟ = = x⎜ = x ⋅ g( x )
⎝ 1+ 9 x 3 ⎟⎠
3 3
17
1+ 9 x ⎝ ⎠ 1+ 9 x (A) (B) 8
2
9 15
Therefore, g( x ) = 3
. (C) 7 (D)
1+ 9 x 2
Hence, the correct answer is option (D). (ONLINE)
Chapter 19 | Limit, Continuity and Differentiability 869
x p /2 Therefore,
(C) e x − ∫ f (t )sin t dt (D) f ( x ) + ∫ f (t )sin t dt
0 0 p /2
d
Solution: We discuss the options as follows: g( x ) = ∫ dt
(f (t )cosec t )dt
x
Option (A): Let g(x) = x9 – f(x).
p /2
⇒ g( x ) = f (t ) cosec t x
g(0) = −f(0) < 0 [as f ∈(0 ,1)]
⎛p ⎞ ⎛p ⎞
19 [as f ∈(0 ,1)] g( x ) = f ⎜ ⎟ cosec ⎜ ⎟ − f ( x ) cosecx
g(1) = – f(1) = 1 – f(1) > 0 ⎝ 2⎠ ⎝ 2⎠
Hence, option (A) is correct. It is given that f(p/2) = 3 and cosec(p/2) = 1. Therefore,
p /2− x
Option (B): Let g( x ) = x − ∫ f (t )cos t dt.
g( x ) = 3 − f ( x ) cosec x =
3 − f (x)
0 sin x
p /2−0
Now,
g(0 ) = 0 − ∫ f (t )cos t dt < 0
⎛ 3 − f ( x )⎞ f (x)
0 lim g( x ) = lim ⎜ ⎟ = 3 − lim
x →0 x → 0 ⎝ sin x ⎠ x → 0 sin x
p / 2 −1
and g(1) = 1− ∫ f (t )cos t dt > 0 Since f(0) = 0 and sin 0 = 0, we get
0
Hence, option (B) is correct. f (x) 0
lim =
x x →0 sin x 0
Option (C): Let g( x ) = e x − ∫ f (t )sin t dt .
d
0 Taking derivative: Using sin x = cos x, we get
Differentiating w.r.t. x, we get dx
(1) f ′( x ) 1
g′( x ) = e x − f ( x )sin x lim = [as f ′(0 ) = 1 given]
Now x →0 cos x 1
g(0) = 1
Therefore, lim g( x ) = 3 − 1 = 2.
x →0
Also, we know that f ( x ) ∈(0 ,1) ⇒ 0 < f(x) < 1 ⇒ 0 < f(x)sinx < 1
Hence, the correct answer is (2).
Therefore, from Eq. (1), we get
⎡ f ′( x ) = −4 sin 4 x − 2 sin 2 x
⎛p ⎞ ⎛ p ⎞⎤ p
g′ ⎜ ⎟ = [sin−1(sinp )]2 cos p − ⎢sin−1 ⎜ sin ⎟ ⎥ cos
⎝ 2⎠ ⎣ ⎝ 2⎠⎦ 2 Now f ′( x ) = 0 gives
−4sin4x – 2sin2x = 0
⎛p ⎞
⇒ g′ ⎜ ⎟ = 0 Using sin2x = 2sinx cosx, we get
⎝ 2⎠
⎛ p⎞ ⎡ ⎛ p⎞ p⎤ −4(2 sin 2 x cos 2 x ) − 2 sin 2 x = 0
and g′ ⎜ − ⎟ = [sin−1( − sinp )]2 cos p − ⎢sin−1 ⎜ − sin ⎟ cos ⎥ = 0
⎝ 2⎠ ⎣ ⎝ 2 ⎠ 2⎦ −8 sin 2 x cos 2 x − 2 sin 2 x = 0
There is no correct option. −2 sin 2 x ( 4 cos 2 x + 1) = 0
p −p
cos(2 x ) cos(2 x ) sin(2 x ) ⇒ 2sin2x = 0 ⇒ sin2x = 0 ⇒ 2x = 0, p, −p ⇒ x = 0 , ,
2 2
6. If f ( x ) = − cos x cos x − sin x , then
and
sin x sin x cos x −1 ⎛ −1⎞
4cos2x + 1 = 0 ⇒ cos2 x = ⇒ 2 x = cos −1 ⎜ ⎟ ⇒ 2x = 1.8 + 2p n
(A) f ′( x ) = 0 at exactly three points in (−p, p ). 4 ⎝ 4⎠
(B) f ′( x ) = 0 at more than three points in (−p, p). where n = …, −2, −1, 0, 1, 2, …, which gives 4 points in the range
(C) f(x) attains its maximum at x = 0. (−p, p ).
(D) f(x) attains its minimum at x = 0. Thus, the total points in (−p, p ) range are 7.
Solution: It is given that Now,
Δq Δq In general
= lim = lim
h→ 0 tan(q + Δq ) − tanq Δq → 0 tan(q + Δq ) − tanq d du dv dw
(u ± v ± w ± ) = ± ± ±
( Δq ) cos(q + Δq ) cosq 1 1 dx dx dx dx
= lim = cos2q = =
Δq → 0 sin Δq sec2 q 1+ x 2 Illustration 20.5 Differentiate 5 sin x − 2 loge x .
Solution:
20.3 Derivatives of Some of the d d d 2
Frequently Used Functions (5 sin x − 2 loge x ) = (5 sin x ) − (2 loge x ) = 5 cos x −
dx dx dx x
⎣ x⎦
Solution: Put y = x 2 and z = sin y . Then
= e − ax cos( x loge x )(loge x + 1) − 2axe − ax sin( x loge x )
2 2
dy dz
= 2 x and = cos y cot x dy
dx dy Illustration 20.11 If y = 1− x 2 + , find .
x dx
Therefore, Solution:
d dz dz dy dy d d ⎛ cot x ⎞
(sin x 2 ) = = ⋅ = (cos y ) (2 x ) = 1− x 2 + ⎜ ⎟
dx dx dy dx dx dx dx ⎝ x ⎠
= (cos x 2 )(2 x ) = 2 x cos x 2 d d
x (cot x ) − cot x ⋅ (x)
d 1− x 2 d (1− x 2 ) dx dx
= ⋅ +
This solution can be rewritten using a more convenient notation in d (1− x 2 ) dx ( x) 2
the following manner: 2 1
x ( − cosec x ) − cot x ⋅
d d (sin x 2 ) d ( x 2 ) 1
( −2 x ) + 2 x
(sin x 2 ) = ⋅ = cos x 2 ⋅ 2 x = 2 x cos x 2 =
dx d( x 2 ) dx 2 1− x 2 x
5. Differentiation of parametrically defined functions: x 2 x cosec2 x + cot x
=− −
• Working rule: 1− x 2 2x3 2
dx
(a) If x and y are functions of parameter q , then find and
dq
dy Your Turn 1
respectively.
dq
(b) Now dy x2 + 1
dy dy dq 1. Find , if y = .
= × dx x
dx dq dx dy x 2 − 1
For example, Ans. =
dx x2
x = sint + cost dy x3 −1
y = cost 2. Find , if y = .
dx x
dx dy
= cos t − sin t ; = − sin t dy 2 x 2 + 1
dt dt Ans. =
dy dx x2
dy − sin t 3. Find of
⇒ = dx
dx cos t − sin t (a) y = (x + 2)(x + 3)
dy 3x + 4
Illustration 20.9 Find , if x = a(q − sinq ); y = a(1− cosq ) . (b) y =
dx 4x + 5
Solution: Consider
x = a(q − sinq ); y = a(1− cosq ) (c) y = ax 2 + 2bx + c
dy dy 1
dx dy Ans. (a) = 2x + 5; (b) =− ;
⇒ = a(1− cosq ); = a(0 + sinq ) = a sinq dx dx ( 4 x + 5)2
dq dq
dy
(c) = (ax 2 + 2bx + c )−1/ 2 (ax + b )
dx
Therefore, dy
4. If x = e −t and y = tan−1 (2t + 1), find
2
q q .
a sinq 2 sin cos dx
dy dy dx
= ÷ = = 2 2 = cot q 2
dx dq dq a(1− cosq ) 2q
dy −et
2 sin 2 Ans. =
2 dt 2t (2t 2 + 2t + 1)
dy 5. Find dy/dx:
Illustration 20.10 If y = e − ax sin( x loge x ), find
2
. ⎛ x ⎞
dx (a) sin cos x (b) sin ⎜ log
Solution: ⎝ x + 1⎟⎠
dy d − ax 2
= {e sin( x loge x )} (
(c) log x + e
x
)
dx dx dy sin x ⋅ cos x cos x
Ans. (a) =− ;
d (e − ax )
2
d sin( x loge x ) dx 2 cos x
= e − ax
2
+ sin( x loge x ) ⋅
dx dx dy 1 ⎛ x ⎞ 1
− ax 2 d sin( x loge x ) d
(b) = cos ⎜ log ⎟ x ( x + 1) ;
=e ⋅ ( x loge x ) dx 2 ⎝ x + 1 ⎠
d ( x loge x ) dx
dy 2 x +e x
d (e − ax ) d =
2
( )
(c)
+ sin( x loge x ) ⋅ ( − ax 2 ) dx 2 x x + e x
d ( − ax 2 ) dx
876 Mathematics Problem Book for JEE
⎛ n m + n ⎞ dy m + n m ⎛ 5⎞
or − = − where f = cos −1 ⎜ ⎟
⎝⎜ y x + y ⎠⎟ dx x + y x ⎝ 13 ⎠
nx + ny − my − ny dy mx + nx − mx − my ⎛ 5 ⎞ ⎛ 5⎞
= = f − q as x ∈⎜ , 1⎟ = cos −1 ⎜ ⎟ − cos −1 x
y( x + y ) dx x( x + y ) ⎝ 13 ⎠ ⎝ 13 ⎠
nx − my dy nx − my
or = Hence,
y ( x + y ) dx x ( x + y )
1 ⎛ 1⎞ 2
f ′( x ) = ⇒ f ′⎜ ⎟ =
Hence,
dy y 1− x 2 ⎝ 2 ⎠ 3
=
dx x
Illustration 20.19 Differentiate cos −1( 4 x 3 − 3 x ), where 0 ≤ x ≤ 1.
x 3 dy x
Illustration 20.17 If y = e sin x + (tan x ) , then find . Solution: Put x = cosq , then
dx
Solution: Let u = e x sin x 3 and v = (tan x ) x. ⎧ ⎡ p⎤
⎪ 3q , q ∈⎢0 , ⎥
Now, −1 3 ⎪ ⎣ 3⎦
cos ( 4 x − 3 x ) = ⎨
u = e x sin x 3 ⎪2p − 3q , q ∈ p , p ⎤
⎛
⎜⎝
Differentiating with respect to x, we get ⎪⎩ 3 2 ⎥⎦
d[sin( x )3 ] d 3 1
du
= ex⋅
d
+ sin x 3 ⋅ (e x ) [cos −1( 4 x 3 − 3 x )] = − if < x ≤ 1
dx dx dx dx 1− x 2 2
3 1
= e x ⋅ cos x 3 ⋅ 3 x 2 + sin x 3 ⋅ e x = if 0 ≤ x <
1− x 2 2
Hence,
du
= 3 x 2e x cos x 3 + e x sin x 3 20.5 Derivative of Second Order y or y2
dx
and v = (tan x ) x d2 y d ⎛ dy ⎞
= ⎜ ⎟ is the derivative of second order and is denoted
Hence, dx 2 dx ⎝ dx ⎠
by y ′′or y 2
loge v = x loge (tan x )
Illustration 20.20 If y = loge (loge x ), find y 2.
Differentiating with respect to x, we get
1 dv 1 Solution: Let y = loge (loge x ). Then
= 1⋅ loge (tan x ) + x ⋅ sec2 x 1 1 1
v dx tan x y1 = ⋅ =
Hence, loge x x x loge x
dv d ⎛ 1 ⎞ d d
= v [loge (tan x ) + x cot x ⋅ sec2 x ] y2 = = ( x loge x )−1 = −1( x loge x )−2 ⋅ ( x loge x )
dx dx ⎜⎝ x loge x ⎟⎠ dx dx
= (tan x ) x [loge (tan x ) + x cot x sec2 x ]
1 ⎛ 1 ⎞ −(1+ loge x )
=− ⋅ ⎜ x ⋅ + loge x ⋅1⎟ =
Now 2
( x loge x ) ⎝ x ⎠ ( x loge x )2
y =u+v
Illustration 20.21 If y = a cos(loge x ) + b sin(loge x ), prove that
Hence, x 2 y ′′ + xy ′ + y = 0 .
dy du dv
= +
dx dx dx Solution: Consider
= 3 x 2e x cos( x 3 ) + e x sin( x 3 ) + (tan x ) x [loge (tan x ) + x cot x sec2 x ] y = a cos(loge x ) + b sin(loge x ) (1)
8. Differentiation by substitution: Sometimes, it is easier to Differentiating with respect to x, we get
differentiate, by making substitutions. Usually these examples
⎡ 1⎤ ⎡ 1⎤
involve inverse trigonometric functions. y ′ = a ⋅ ⎢ − sin(loge x ) ⋅ ⎥ + b ⎢ cos(loge x ) ⋅ ⎥
⎣ x⎦ ⎣ x⎦
⎛ 5 x + 12 1− x 2 ⎞ 5
Illustration 20.18 If f ( x ) = cos −1 ⎜ ⎟, < x < 1, ⇒ xy ′ = − a sin(loge x ) + b cos(loge x )
⎜⎝ 13 ⎟⎠ 13
Again differentiating with respect to x, we get
⎛ 1⎞ ⎧ 1⎫ ⎧ 1⎫
then find f ′ ⎜ ⎟ . xy ′′ + y ′ ⋅1 = − a ⋅ ⎨cos(loge x ) ⋅ ⎬ + b ⋅ ⎨ − sin(loge x ) ⋅ ⎬
⎝ 2⎠ ⎩ x ⎭ ⎩ x⎭
Solution: Put x = cosq , then
⇒ x 2 y ′′ + xy ′ = −[a cos(loge x ) + b sin(loge x )] = − y [from Eq. (1)]
−1 ⎛ 5 cosq + 12 sinq ⎞
f ( x ) = cos ⎜ −1
⎟⎠ > 0 = cos [cos(q − f )] > 0,
⎝ 13 ⇒ x 2 y ′′ + xy ′ + y = 0
878 Mathematics Problem Book for JEE
dy y
2. Find of x + y = sin (xy). ⇒ − 1 = tan log x
dx dy y cos( xy ) − 1 x2
Ans. = y
dx 1− x cos( xy ) ⇒ = 1 + tan log x
3. Differentiate (log x )
tan x
with respect to sin(m cos–1x). x2
⇒ y = x2 + x2 tan log x
⎛ tan x ⎞ Now,
−(log x )tan x ⎜ sec2 x ⋅ loglog x + 1− x 2 dy 1
⎝ x log x ⎟⎠ = 2x + 2x tan log x + x sec2 (log x) ×
Ans. dx x
m cos( m cos −1 x ) = 2x + 2x tan log x + x sec2 (log x)
dy 2 2 2 2y
4. If xy.yx = 1, then find . = (x + x tan log x) + x sec2 (log x) = + 1 + tan2 (log x)
dx x x
dy ( y + x log y ) y
=− Ans.⋅ ⎡ ⎛ y ⎞ ⎤ 2y
2
dx ( x + y log x ) x 2y y2 2y
⇒ + x ⎢1+ ⎜ 2 − 1⎟ ⎥ = + 1+ 4 + 1 − 2
⎝ ⎠
5. Differentiate sin2x with respect to (logx)2. x ⎣⎢ x ⎦⎥ x x x
x sin x cos x
Ans. xy 2 2 yx 2y
log x = 4
− 2
+ + 2x
dy x x x
6. If x + y + xy = 2, then find
2 2 .
dx dy (2 x + y ) 2
Ans. =− y
2y 2y y2
dx (2 y + x ) = + + 2 −
x = + 2x
x3 x x x3
dy Hence, the correct answer is option (A).
7. If y = sin x + sin x + sin x + ∞ , then find .
dx
dy cos x 3. If y = ( x + 1+ x 2 )m , then (1+ x 2 ) y 2 + xy1 − m2 y = ____.
Ans. =
dx 2 y − 1 (A) 0 (B) 1
⎛ 1⎞ dy (C) – 1 (D) 2
8. If 5f(x) + 3f ⎜ ⎟ = x + 2 and y = xf(x), then find at x = 1.
⎝ x⎠ dx Solution: Substituting the value of y in the given equation, we
have
⎛ dy ⎞ 7 m −1 ⎛ 1⋅ 2 x ⎞
Ans. ⎜ ⎟ =
⎝ dx ⎠ at x =1 8 y1 = m ⎡ x + 1+ x 2 ⎤ ⋅ ⎜ 1+ ⎟
⎣⎢ ⎦⎥ ⎝ 2 1+ x 2 ⎠
Additional Solved Examples
( )
m
m x + 1+ x 2
=
1. If f(x) = sinx, g(x) = x2, h(x) = logx and F(x) = (hogof )(x), then 1+ x 2
F’’(x) is
my
(A) 2cosec3x (B) 2cotx2 − 4x2cosec2x2 =
(C) 2x cotx 2 (D) − 2cosec2x 1+ x 2
Solution: ⇒ y12 (1+ x 2 ) = m2 y 2
F ( x ) = (hogof )( x ) = h{ g[f ( x )]} Differentiating with respect to x, we get
2
= h[ g(sin x )] = h(sin x ) = loge (sin x ) 2 2 y1y 2 (1+ x 2 ) + y12 (2 x ) = 2m2 yy1
f (n) ( x ) = n(n − 1) …1 = n ! n2 ( y 2 + 4 ) n2 ( y 2 − 4 )
(A) 2
(B)
⇒ f ′(0 ) = n, f ′′(0 ) = n(n − 1), … , f n (0 ) = n ! x +4 x2
2
y2 − 4 ⎛ ny ⎞
Therefore, given expression is (C) n (D) ⎜ ⎟ − 4
2
x −4 ⎝ x⎠
n n(n − 1) n!
1+ + ++ Solution:
1 2! n!
dy
= n C 0 + n C1 + nC2 + + nC n = 2 n = n sec n −1q ⋅ secq ⋅ tanq − n ⋅ cosn −1q ⋅ ( − sinq )
dq
Hence, the correct answer is option (B).
⎡ sinq ⎤
d2 y dy = n ⎢sec n q + cosn −1q ⋅ sinq ⎥
6. If y = sin (sinx), and + tan x + f ( x ) = 0 , then f(x) is ⎣ cosq ⎦
2 dx
dx n sinq ⎡ n
= sec q + cosn q ⎤⎦
(A) sin2x sin(cosx) (B) sin2x cos(sinx) cosq ⎣
(C) cos2x sin(cosx) (D) cos2x sin(sinx) = n tanq (sec n q + cosn q )
Solution: and
dy dx sinq
= cos(sin x ) ⋅ cos x = secq tanq + sinq = secq + sinq
dx dq cosq
d2 y sinq
⇒ = − cos(sin x ) ⋅ sin x + cos x [ − sin(sin x )cos x ] = (secq + cosq ) = tanq (secq + cosq )
dx 2 cosq
d2 y dy Therefore,
⇒ + tan x = − cos(sin x ) ⋅ sin x − cos2 x ⋅ sin(sin x )
dx 2 dx dy n tanq (sec n q + cosn q )
+ cos(sin x )⋅⋅ cos x ⋅ tan x =
dx tanq (secq + cosq )
= − cos2 x ⋅ sin(sin x ) n(secn q + cosn q )
=
2
⇒ f ( x ) = cos x ⋅ sin(sin x ) secq + cosq
Hence, the correct answer is option (D). Hence,
7. If x = 2 cos t − cos 2t and y = 2 sin t − sin 2t, then the value of 2
⎛ dy ⎞ n2 (secn q + cosn q )2
d2 y p ⎜⎝ ⎟⎠ =
at t = is dx (secq + cosq )2
dx 2 2
(A) 3/2 (B) −5/2 n2 ⎡⎣(secn q − cosn q )2 + 4 ⎤⎦
=
(C) 5/2 (D) −3/2 (secq − cosq )2 + 4
Chapter 20 | Differentiation 881
=
(
n2 y 2 + 4 ) Solution:
x 2 x − 2 x x cot y − 1 = 0 (1)
x2 + 4
Hence, the correct answer is option (A). Now at x = 1,
p
⎛ 2 x − 1⎞ dy 1− 2 cot y − 1 = 0 ⇒ cot y = 0 ⇒ y =
9. If y = f ⎜ 2 ⎟ and f ′(x) = sin x2, then is 2
⎝ x + 1⎠ dx
On differentiating Eq. (1) with respect to x, we get
(A) cos x2.f ′(x) (B) − cos x2.f ′(x)
2 ⎡ dy ⎤
2(1+ x − x 2 ) ⎛ 2 x − 1⎞ 2 x 2 x (1+ log x ) − 2 ⎢ x x ( − cosec2 y ) + cot y x x (1+ log x )⎥ = 0
(C) sin ⎜ 2 ⎟
⎝ x + 1⎠
(D) None of these ⎣ dx ⎦
( x 2 + 1)2
2x −1 ⎛ p⎞
Solution: Let 2 = z . Then At ⎜ 1, ⎟ , we have
x +1 ⎝ 2⎠
y = f (z) ⎛ ⎞
⎜ ⎛ dy ⎞
2(1+ log1) − 2 1( −1) ⎜ ⎟ + 0⎟ = 0
⎜ ⎝ dx ⎠ ⎛ 1, p ⎞ ⎟
Therefore, ⎜⎝ ⎜⎝
2
⎟⎠ ⎟⎠
dy dz
= f ′( z ) ⋅
dx dx ⎛ dy ⎞ ⎛ dy ⎞
⇒ 2 + 2⎜ ⎟ =0⇒⎜ ⎟ = −1
2 dz ⎝ dx ⎠ ⎛ 1, p ⎞ ⎝ dx ⎠ ⎛ 1, p ⎞
= sin z ⋅ [because f ′(z)=sin z2] ⎜⎝ ⎟
2⎠
⎜⎝ ⎟
2⎠
dx
2
⎛ 2 x − 1⎞ d ⎛ 2 x − 1⎞ Hence, the correct answer is option (A).
= sin ⎜ 2 ⎟
⎝ x + 1⎠ dx ⎜⎝ x 2 + 1⎟⎠ d2 x
2 2 2. equals
⎛ 2 x − 1⎞ 2(1+ x − x ) dy 2
= sin ⎜ 2 ⎟ .
⎝ x + 1⎠ ( x + 1)
2 2 −1 −3
⎛ d2 y ⎞ ⎛ dy ⎞ ⎛ d 2 y ⎞ ⎛ dy ⎞ −2
Hence, the correct answer is option (C). (A) − ⎜ 2 ⎟ (B) ⎜ 2 ⎟ ⎜ ⎟
⎜⎝ ⎟⎠
d ⎛ 3 d2 y ⎞ ⎝ dx ⎠ dx ⎝ dx ⎠ ⎝ dx ⎠
10. If y2 = p(x), a polynomial of degree 3, then 2 ⎜y ⎟ is −1
dx ⎝ dx 2 ⎠ ⎛ d 2 y ⎞ ⎛ dy ⎞ −3 ⎛ d2 y ⎞
equal to (C) − ⎜ 2 ⎟ ⎜ ⎟ (D) ⎜ 2 ⎟
⎝ dx ⎠ ⎝ dx ⎠ ⎝ dx ⎠
(A) p’’’(x) + p’(x) (B) p’’’(x) + p’’(x)
[AIEEE 2011]
(C) p(x) p’’’(x) (D) a constant
Solution: Solution:
⎛ ⎞
y 2 = p( x ) (1)
d ⎛ dx ⎞ d ⎜ 1 ⎟ 1 d ⎛ dy ⎞
⇒ 2 y y1 = p ’( x ) = ⎜ ⎟ =− ⎜ ⎟
(2) dy ⎜⎝ dy ⎟⎠ dy ⎜ ⎛ dy ⎞ ⎟ ⎛ dy ⎞ dy ⎝ dx ⎠
2
⇒ 2( y y2 + y1 y1) = p ’’( x ) ⎜⎝ ⎝⎜ dx ⎠⎟ ⎟⎠ ⎜⎝ ⎟⎠
dx
1 −2
⇒ y y 2 = [ p ’’( x ) − 2 y12 ] ⎛ dy ⎞ 1 d ⎛ dy ⎞
2 = −⎜ ⎟ ⎜ ⎟
Multiplying both sides by y2, we have ⎝ dx ⎠ ⎛ dy ⎞ dx ⎝ dx ⎠
⎜⎝ ⎟⎠
1 dx
⇒ y 3 y 2 = [ p ’’( x ) y 2 − 2( y y1)2 ]
2 ⎛ d 2 y ⎞ ⎛ dy ⎞ −3
2 = −⎜ 2 ⎟ ⎜ ⎟
1 ⎡1 ⎤
= p ’’( x ) ⋅ p( x ) − ⎢ p ’( x )⎥ [using Eqs. (1) and (2)] ⎝ dx ⎠ ⎝ dx ⎠
2 ⎣2 ⎦
Now, Hence, the correct answer is option (C).
⎡1 ⎤ ⎡1 ⎤ dy
d 3 1
( y y 2 ) = [ p ’’’( x )p( x ) + p ’( x )p ’’( x )] − 2 × ⎢ p ’( x )⎥ × ⎢ p ’’( x )⎥ 3. If y = sec(tan−1 x ), then at x = 1 is equal to
dx 2 ⎣2 ⎦ ⎣2 ⎦ dx
1
d 3 (A) (B) 1
⇒ 2 ( y y 2 ) = p( x )p ’’’( x ) 2
dx 1
Hence, the correct answer is option (C). (C) 2 (D)
2
[AIEEE 2013]
Previous Years' Solved JEE Main/AIEEE Solution: We have,
Questions
y = sec(tan−1 x )
1. Let y be an implicit function of x defined by x2x − 2xx cot y − 1 = 0. Therefore,
Then y′(1) equals
dy 1 dy 1 1
(A) − 1 (B) 1 = sec(tan−1 x )tan(tan−1 x ) ⋅ 2
⇒ = 2 × 1× =
(C) log 2 (D) − log 2 dx 1+ x dx x =1 2 2
[AIEEE 2009] Hence, the correct answer is option (D).
882 Mathematics Problem Book for JEE
⎛ d2 y ⎞ ⎛ d2 x ⎞ Put x = 0, we get
4. If y = enx, then ⎜ 2 ⎟ ⎜ 2 ⎟ is equal to 1
⎝ dx ⎠ ⎝ dy ⎠ g′(1) = = 2.
f ′( 0 )
(A) nenx (B) ne−nx
Hence, the correct answer is (2).
(C) 1 (D) −ne−nx
⎡ ⎛ sinq ⎞ ⎤ p p
[JEE MAIN 2014 (ONLINE PAPER SET 1)] 3. Let f (q ) = sin ⎢ tan−1 ⎜ , where − < q < . Then the
⎣ ⎝ cos 2q ⎟⎠ ⎥⎦ 4 4
Solution:
d
y = e nx value of [f (q )] is _____. [IIT-JEE 2011]
d (tanq )
dy d2 y Solution:
= e nx (n) ⇒ 2 = n2e nx (1)
dx dx ⎡ ⎛ sinq ⎞ ⎤ ⎛ p p⎞
sin ⎢ tan−1 ⎜ , where q ∈⎜ − , ⎟
Now ⎣ ⎝ cos 2q ⎟⎠ ⎥⎦ ⎝ 4 4⎠
⎡ ⎞⎤
d2 x
dy 2
=
d ⎛ dx ⎞ d ⎛ 1 ⎞ 1 d − nx
= ⎜ ⎟=
dy ⎜⎝ dy ⎟⎠ dy ⎝ ne nx ⎠ n dy
e ( ) ⎢⎣
⎛
sin ⎢ tan−1 ⎜
sinq
⎝ 2 cos q − 1⎠ ⎥⎦
2
⎟⎥
1 dx ⎛ 1 ⎞ 1 = sin[sin−1(tanq )] = tanq
= e − nx ( − n) = − e nx ⎜ nx ⎟ = − e −2nx (2)
n dy ⎝ ne ⎠ n d (tanq )
=1
Now from Eqs. (1) and (2), we get d (tanq )
⎛ d2 y ⎞ ⎛ d2 x ⎞ Hence, the correct answer is (1).
2 nx ⎛ 1 ⎞ −2 nx
⎜ 2 ⎟ ⎜ 2 ⎟ = n e × ⎜⎝ − ⎟⎠ e = − n e−n x
⎝ dx ⎠ ⎝ dy ⎠ n
Hence, the correct answer is option (D).
Practice Exercise 1
dy
Previous Years' Solved JEE Advanced/ 1. If cos( x + y ) = y sin x , then is equal to
dx
IIT-JEE Questions sin( x + y ) + y cos x sin( x + y ) + y cos x
(A) − (B)
d2 x sin x + sin( x + y ) sin x + sin( x + y )
1. equals y cos x − sin( x + y )
dy 2 (C) (D) None of these
−1 −1 −3 sin x − sin( x + y )
⎛ d2 y ⎞ ⎛ d2 y ⎞ ⎛ dy ⎞
(A) ⎜ 2 ⎟ (B) − ⎜ 2 ⎟ ⎜⎝ ⎟⎠ dy
⎝ dx ⎠ ⎝ dx ⎠ dx 2. If sin2 x + 2 cos y + xy = 0 , then is equal to
dx
⎛ d 2 y ⎞ ⎛ dy ⎞ −2 ⎛ d 2 y ⎞ ⎛ dy ⎞ −3 y + 2 sin x y + sin 2 x
(C) ⎜ 2 ⎟ ⎜ ⎟ (D) − ⎜ 2 ⎟ ⎜ ⎟ (A) (B)
⎝ dx ⎠ ⎝ dx ⎠ ⎝ dx ⎠ ⎝ dx ⎠ 2 sin y + x 2 sin y − x
dy dx
[IIT-JEE 2007] y + 2sin x
Solution: Since × = 1, we get (C) (D) None of these
dx dy sin y + x
dy
−1 3. If x 3 + 8 xy + y 3 = 64 , then is equal to
dx 1 ⎛ dy ⎞ dx
= =⎜ ⎟
dy dy/dx ⎝ dx ⎠
3x2 + 8 y 3x2 + 8 y
−1 (A) − (B)
d ⎛ dx ⎞ d ⎛ dy ⎞ dx 8 x + 3y2 8 x + 3y2
⇒ = ⎜ ⎟ ×
dy ⎜⎝ dy ⎟⎠ dy ⎝ dx ⎠ dy 3x + 8 y2
(C) (D) None of these
−2 8 x2 + 3y
d2 x
⎛ dy ⎞ d2 y
⎛ dx ⎞
⇒ 2 = −⎜ ⎟ × 2 ×⎜ ⎟
dy ⎝ dx ⎠ dx ⎝ dy ⎠ dy
4. If sin(x + y) = log(x + y), then is equal to
dx
−3
⎛ dy ⎞ d2 y (A) 2 (B) −2
×⎜ ⎟ =−
dx 2 ⎝ dx ⎠ (C) 1 (D) −1
Hence, the correct answer is option (D). 5. If ln( x + y ) = 2 xy ,then y ’(0 ) =
(A) 1 (B) −1
2. If the function f ( x ) = x 3 + e x/2 and g( x ) = f −1( x ), then the value (C) 2 (D) 0
of g′(1) is _____. [IIT-JEE 2009]
y x−y dy
6. If x = e , then is equal to
Solution: dx
1
f (0 ) = 1, f ′( x ) = 3 x 2 + e x / 2 (A) log x ⋅ [log(ex )]−2 (B) log x ⋅ [log(ex )]2
2
⇒ f ′[ g( x )]g′( x ) = 1 (C) log x ⋅ (log x )2 (D) None of these
Chapter 20 | Differentiation 883
p dy 2x
16. The differential coefficient of tan−1
tan x
7. y = (tan x )(tan x ) , then at x = , the value of is equal to with respect to
4 dx 1 − x2
−1 2 x
(A) 0 (B) 1 sin is
(C) 2 (D) None of these 1+ x 2
(A) 1 (B) − 1
dy
8. If y = (sin x )tan x, then is equal to (C) 0 (D) None of these
dx
(A) (sin x )tan x ⋅ (1+ sec2 x ⋅ logsin x ) ⎛ x ⎞ d2 y
17. If y = x log ⎜ ⎟ , then x 3 2 is equal to
⎝ a + bx ⎠
(B) tan x ⋅ (sin x )tan x −1 ⋅ cos x dx
2
(C) (sin x )tan x ⋅ sec2 x ⋅ logsin x dy ⎛ dy ⎞
(A) x −y (B) ⎜ x − y⎟
(D) tan x ⋅ (sin x )tan x −1 dx ⎝ dx ⎠
2
dy ⎛ dy ⎞
9. If y = 21/logx 4 , then x is equal to (C) y −x (D) ⎜ y − x⎟
dx ⎝ dx ⎠
(A) y (B) y
18. If y = x 3 logloge (1+ x ), then y ′′ (0 ) equals
2 4
(C) y (D) y
(A) 0 (B) − 1
10. The derivative of y = x ln x is (C) 6 log e 2 (D) 6
(A) x ln x ln x (B) x ln x −1 ln x d2 x
ln x −1 ln x −2 19. is equal to
(C) 2 x ln x (D) x dy 2
dy 1 (d 2 y/dx 2 )
−1 (A) (B)
11. If y = sin(2 sin x ), then is equal to 2
dx (dy/dx ) (dy/dx )2
2 − 4 x2 2 + 4 x2 d2 y ( −d 2 y/dx 2 )
(A) (B) (C) (D)
2
1− x 1− x 2 dx 2
(dy/dx )2
2 − 4 x2 2 + 4 x2 20. If fn ( x ), gn ( x ), hn ( x ), n = 1, 2, 3 are polynomials in x such that
(C) (D)
1+ x 2 1+ x 2 f1( x ) f2 ( x ) f3 ( x )
fn (a) = gn (a) = hn (a), n = 1, 2, 3 and F ( x ) = g1( x ) g2 ( x ) g3 ( x ) ,
⎛ 3 cos x + 4 sin x ⎞ dy
12. If y = cos −1 ⎜ ⎟⎠ , then h1( x ) h2 ( x ) h3 ( x )
⎝ 5 dx
then F ′(a) is equal to
(A) 0 (B) 1
1 (A) 0 (B) f1(a)g2 (a)h3 (a)
(C) −1 (D)
2 (C) 1 (D) None of these
d x − x −1
13. cos −1 is equal to
dx x + x −1 x3 sin x cos x
1 −1 21. Let f ( x ) = 6 −1 0 , where p is a constant. Then
(A) (B)
1+ x 2 1+ x 2 p p2 p3
2 −2 d 3
(C) (D) [f ( x )] at x = 0 is
1+ x 2 1+ x 2 dx 3
d 1+ x 2 (A) p (B) p + p2
14. cos −1 is equal to
dx 2 (C) p + p3 (D) Independent of p
−1 1
(A) (B) x3 x2 3x2
2 1− x 4 2 1− x 4 d 3f ( x )
22. f ( x ) = 1 −6 4 , here p is a constant, then is
−x x dx 3
(C) (D) p p 2
p 3
1− x 4 1− x 4
(A) Proportional to x 2 (B) Proportional to x
x −1 1−
15. Differential coefficient of sin with respect to x is (C) Proportional to x 3 (D) A constant
1+ x
1 x y y1 y2
(A) (B) 23. If y = sin px and y n is the nth derivative of y, then y 3 y4 y5
2 x 1− x
is equal to y6 y7 y8
(C) 1 (D) None of these
884 Mathematics Problem Book for JEE
(A) 1 (B) 0 1 1
(C) – 1 (D) None of these (A) − (B)
4 2
d2 y
24. If y 2 = ax 2 + bx + c , then y 3 2 is (C) −
1
(D)
1
dx 2 4
(A) A constant
(B) A function of x only d 1− sin 2 x
(C) A function of y only 33. is equal to
dx 1+ sin 2 x
(D) A function of x and y
⎛p ⎞
d2 y (A) sec2 x (B) − sec2 ⎜ − x ⎟
x
25. If y = a ⋅ b 2 x −1
, then is ⎝4 ⎠
dx 2
⎛p ⎞ ⎛p ⎞
(C) sec2 ⎜ + x ⎟ (D) sec2 ⎜ − x ⎟
(A) y 2 ⋅log ab2 (B) y ⋅log ab2 ⎝4 ⎠ ⎝4 ⎠
(C) y 2 (D) y ⋅(log ab2 )2
34. If f ( x ) = x tan−1 x , then f ’(1) is equal to
d
26. log(log x ) is equal to p 1 p
dx (A) 1+ (B) +
x log x 4 2 4
(A) (B) 1 p
log x x (C) − (D) 2
2 4
(C) ( x log x )−1 (D) None of these
35. If f ( x ) = log x (log x ), then f ’( x )at x = e is
2
d ⎛ 1 ⎞
27. ⎜ x+ ⎟ is equal to 1
dx ⎝ x⎠ (A) e (B)
e
1 1
(A) 1− 2 (B) 1+ (C) 1 (D) None of these
x x2
1 ⎛ p⎞
(C) 1− (D) None of these 36. If f ( x ) = 1+ cos2 ( x 2 ) , then f ’ ⎜ ⎟ is
2x ⎝ 2 ⎠
x2 x3 dy
28. If y = 1+ x + + + ∞ , then is equal to (A) p /6 (B) − (p / 6 )
2! 3! dx
(A) y (B) y −1 (C) 1/ 6 (D) p / 6
(C) y +1 (D) None of these d
37. tan−1(sec x + tan x ) is equal to
d ⎛ −1 cos x ⎞ dx
29. ⎜ tan ⎟ is equal to
dx ⎝ 1+ sin x ⎠ (A) 1 (B) 1/2
1 1 (C) cos x (D) sec x
(A) − (B)
2 2
d x
(C) −1 (D) 1 38. (e logsin 2 x )
dx
d
30. [cos(1− x 2 )2 ] is equal to
dx (A) e x (logsin 2 x + 2 cot 2 x )
(A) −2 x (1− x 2 )sin(1− x 2 )2 (B) e x (logcos 2 x + 2 cot 2 x )
(B) −4 x (1− x 2 )sin(1− x 2 )2 (C) e x (logcos 2 x + cot 2 x )
2 2 2
(C) 4 x (1− x )sin(1− x ) (D) None of these
(D) −2(1− x 2 )sin(1− x 2 )2 ⎛ x + 1⎞ −1 ⎛ x − 1⎞ dy
39. If y = sec −1 ⎜ ⎟ + sin ⎜ ⎟ , then dx is equal to
dy ⎝ x − 1⎠ ⎝ x + 1⎠
31. If y = sin−1( x 1− x + x 1− x 2 ), then is equal to
dx 1
(A) 0 (B)
−2 x 1 −1 1 x +1
(A) + (B) −
1− x 2 2 x − x2 1− x 2
2 x − x2 (C) 1 (D) None of these
d
(C)
1
+
1
(D) None of these 40. sin−1(3 x − 4 x 3 ) is equal to
dx
1− x 2 2 x − x2
3 −3
(A) (B)
⎛ x⎞ 1− x 2 1− x 2
d ⎜ 1+ cos ⎟
−1 2
32. ⎜ tan x⎟
is equal to 1 −1
dx ⎜ 1− cos ⎟ (C) (D)
2
⎝ 2⎠ 1− x 1− x 2
Chapter 20 | Differentiation 885
1+ tan x dy e2 x cos x dy
41. If y = , then is equal to 48. If y = , then is equal to
1− tan x dx x sin x dx
1 1− tan x ⎛p ⎞ e2 x [(2 x − 1)cot x − x cosec2 x ]
(A) ⋅ sec2 ⎜ + x ⎟ (A)
2 1+ tan x ⎝4 ⎠ x2
e [(2 x + 1)cot x − x cosec2 x ]
2x
1− tan x ⎛p ⎞ (B)
(B) ⋅ sec2 ⎜ + x ⎟ x2
1+ tan x ⎝4 ⎠
e2 x [(2 x − 1)cot x + x cosec2 x ]
1 1− tan x ⎛p ⎞ (C)
(C) ⋅ sec ⎜ + x ⎟ x2
2 1+ tan x ⎝4 ⎠ (D) None of these
(D) None of these d − ax 2
49. [e log(sin x )]
d ⎛ sec x + tan x ⎞ dx
42. ⎜ ⎟ is equal to (A) e − ax (cot x + 2ax logsin x )
2
dx ⎝ sec x − tan x ⎠
(B) e − ax (cot x + ax logsin x )
2
2 cos x cos x
(A) (B)
(1− sin x )2 (1− sin x )2 (C) e − ax (cot x − 2ax logsin x )
2
1
13. The abscissa of the point of contact of tangent for which is
C greatest, is a
1
(A) (B) 1
3
1
E
(C) − 1 (D) –
B D 3
x
A Matrix Match Type Questions
F 14. Match the following:
Figure 20.1 Column I Column II
5. The point which has the greatest value of f ′(x) is (p) Does not exist
(A) B (B) C x − cos(sin−1 x )
(A) lim is equal to
(C) D (D) E x→
1 1− tan(sin−1 x )
2
6. The point where f ′ and f ′′ are non-zero and of the same sign
are ⎛ 1⎞ 1
(A) B and D (B) D and E (B) If f(x) = log x 2 (log x ) , then f ′ ⎜ ⎟ is (q) –
⎝ 2⎠ 2
(C) B and E (D) None of these equal to
7. The points where at least two of f, f ′ and f ′′ are zero, (C) For the function f(x) = ln tan (r) 28
(A) C and D (B) A and D
⎛p x⎞
(C) A and F (D) None of these ⎜⎝ + ⎟⎠
4 2
Paragraph for Questions 8–10: In certain problems, the differ-
entiation of {f(x).g(x)} appears. One student commits mistake and dy
if = sec x + p, then p is equal to
df dg dx
differentiates as ⋅ , but he gets correct result if f(x) = x3 and
1 dx dx (s) 1
g(0) = . 1 1− cos 2 x
(D) lim is equal to
3 x → 0 x 1+ cos 2 x
8. The function g(x) is
3 4 (t) 0
(A) (B)
| x − 3 |3 | x − 3 |3
15. Match the following:
9 27
(C) (D) Column I Column II
| x − 3 |3 | x − 3 |3
(A) If y = cos–1 (cos x), then y′ at x =5
9. Derivative of {f(x − 3)⋅g(x)} with respect to x at x = 100 is (p) Does not exist
is equal to
(A) 0 (B) 1
(C) – 1 (D) 2 (B) For the function f(x) = ln |tan x| (q) 2
⎛ p⎞
f ( x ) ⋅ g( x ) f ′ ⎜ − ⎟ is equal to
10. lim will be ⎝ 4⎠
x→0 x [1+ g( x )]
(A) 0 (B) − 1 ⎛ 1+ x ⎞ 1
(C) The derivative of tan–1 ⎜ ⎟ at (r)
(C) 1 (D) 2 ⎝ 1− x ⎠ 2
1 x = − 1 is
Paragraph for Questions 11–13: Let f ( x ) = 2
. Let m be the
1+ x
log x (s) 1
slope, a be the x-intercept and b be the y-intercept of a tangent (D) The derivative of at x = − 1
to y = f(x), then is x
11. Abscissa of the point of contact of the tangent for which m is (t) –1
greatest
1
(A) (B) 1 Integer Type Questions
3
1 4x 2+3x 2
(C) − 1 (D) − 16. If y = tan–1 + tan–1 , where 0 < x < and
3 1+ 5 x 2 3−2x 3
12. The greatest value of b is dy l
= , then find l.
9 3 dx 1+ 25 x 2
(A) (B)
8 8
17. The function y = f(x) defined by the parametric equations
1 5 x = et sin t, y = et cos t satisfies the relation y′′ (x + y)2 = l(xy′
(C) (D)
8 8 − y), then find l.
888 Mathematics Problem Book for JEE
Answer Key
Practice Exercise 1
1. (A) 2. (B) 3. (A) 4. (D) 5. (A) 6. (A)
7. (C) 8. (A) 9. (C) 10. (C) 11. (A) 12. (B)
13. (D) 14. (C) 15. (D) 16. (A) 17. (B) 18. (A)
19. (D) 20. (A) 21. (D) 22. (D) 23. (B) 24. (A)
25. (D) 26. (C) 27. (A) 28. (A) 29. (A) 30. (C)
31. (C) 32. (A) 33. (B) 34. (B) 35. (B) 36. (B)
37. (B) 38. (A) 39. (A) 40. (A) 41. (A) 42. (A)
43. (B) 44. (C) 45. (B) 46. (C) 47. (D) 48. (A)
49. (C) 50. (C) 51. (A) 52. (A) 53. (A) 54. (B)
55. (B) 56. (B) 57. (A) 58. (C) 59. (B) 60. (A)
61. (C) 62. (A) 63. (C) 64. (A) 65. (A)
Practice Exercise 2
1. (A), (C) 2. (A), (B) 3. (B), (C) 4. (A), (D) 5. (A) 6. (C)
7. (B) 8. (C) 9. (A) 10. (A) 11. (D) 12. (A)
13. (A) 14. (A) → (q), (B) → (p), (C) → (t), (D) → (p) 15. (A) → (t), (B) → (q), (C) → (r), (D) → (s) 16. 5
17. 2
Solutions
Practice Exercise 1 At x = 0 , y = 1 [from ln( x + y ) = 2 xy ]
1. cos( x + y ) = ( y sin x )
Hence,
⎛ dy ⎞ dy 1− 2
⇒ − sin( x + y ) ⎜ 1+ ⎟ = y cos x + sin x y ’(0 ) = =1
⎝ dx ⎠ dx −1
dy y cos x + sin( x + y ) 6. x y = e x − y ⇒ y log x = x − y ⇒ y =
x
⇒ =−
dx sin( x + y ) + sin x 1+ log x
dy
2. sin2 x + 2 cos y + xy = 0 ⇒ = log x (1+ log x )−2 = log x [log ex ]−2
dx
dy dy
⇒ 2 sin x cos x − 2 sin y +y+x =0 7. log y = (tan x )tan x logtan x (1)
dx dx
Hence, Taking log again in Eq. (1), we get
dy y + sin 2 x
= log(log y ) = tan x logtan x + log(logtan x )
dx 2 sin y − x
⎛ dy ⎞ dy Differentiating with respect to x, we get
3. x 3 + 8 xy + y 3 = 64 ⇒ 3 x 2 + 8 ⎜ y + x ⎟ + 3 y 2 =0
⎝ dx ⎠ dx 1 1 dy
⋅
Hence, log y y dx
dy 3x2 + 8 y
=− sec2 x 1 1
dx 8 x + 3y2 = sec2 x logtan x + tan x ⋅ + ⋅ ⋅ sec2 x
tan x logtan x tan x
4. It is an implicit function, so Therefore,
1 ⎡ ⎤
cos( x + y ) − dy 1
dy ∂f / ∂x x+y = y log y sec2 x ⋅ ⎢logtan x + 1+ ⎥
=− =− = −1 dx ⎣ tan x logtan x ⎦
dx ∂f / ∂y cos( x + y ) −
1
⎡ 1 ⎤
x+y = y (tan x )tan x logtan x ⋅ sec2 x ⎢(logtan x + 1) + ⎥
5. ln( x + y ) = 2 xy ⎣ tan x logtan x ⎦
Differentiating both sides with respect to x, we get = y (tan x )tan x sec2 x [logtan x (logtan x + 1) + cot x ]
⎛ 1 ⎞ ⎛ dy ⎞ ⎛ dy ⎞ p ⎛p ⎞
Now at x = , y = 1, logtan ⎜ ⎟ = log1 = 0
⎜⎝ x + y ⎟⎠ ⎜⎝ 1+ dx ⎟⎠ = 2 ⎜⎝ x dx + y ⎟⎠ 4 ⎝ 4⎠
dy 1− 2 xy − 2 y 2 Therefore,
⇒ = dy
dx 2 x 2 + 2 xy − 1 = 1⋅1⋅ 2[0 + 1] = 2
dx
Chapter 20 | Differentiation 889
20. We have dy
f1( x ) f2 ( x ) f3 ( x ) 24. y 2 = ax 2 + bx + c ⇒ 2 y = 2ax + b
dx
F ( x ) = g1( x ) g2 ( x ) g3 ( x ) 2 2
⎛ dy ⎞ d2 y d2 y ⎛ dy ⎞
h1( x ) h2 ( x ) h3 ( x ) ⇒ 2 ⎜ ⎟ + 2 y 2 = 2a ⇒ y 2 = a − ⎜ ⎟
Hence, ⎝ dx ⎠ dx dx ⎝ dx ⎠
2
f1′ ( x ) f2′ ( x ) f3′ ( x ) f1( x ) f2 ( x ) f3 ( x ) d2 y ⎛ 2ax + b ⎞ d 2 y 4 ay 2 − (2ax + b )2
⇒y = a−⎜ ⎟ ⇒y 2=
F ’( x ) = g1( x ) g2 ( x ) g3 ( x ) + g1′ ( x ) g2′ ( x ) g3′ ( x ) dx 2 ⎝ 2y ⎠ dx 4 y2
h1( x ) h2 ( x ) h3 ( x ) h1( x ) h2 ( x ) h3 ( x )
d2 y
⇒ 4 y3 = 4 a(ax 2 + bx + c ) − ( 4 a2 x 2 + 4 abx + b2 )
f1( x ) f2 ( x ) f3 ( x ) dx 2
+ g1( x ) g2 ( x ) g3 ( x ) d2 y d2 y 4 ac − b2
⇒ 4 y3 = 4 ac − b2 ⇒ y 3 = = a constant
h1′ ( x ) h2′ ( x ) h3′ ( x ) dx 2 dx 2 4
⇒ F ’(a) = 0 (since fn (a) = gn (a) = hn (a), n = 1, 2, 3) 25. y = a x b2 x −1
Therefore, two rows in each determinant become identical
on putting x = a. dy
= a x b2 x −1 log a + 2a x b2 x −1 log b = a x b2 x −1(log a + 2 log b )
dx
d3 d3 d3
x3 sin x cos x d2 y
dx 3
dx 3
dx 3 6 − cos x sin x = a x b2 x −1(log a + 2 log b )2 = a x b2 x −1(log ab2 )2
21. f ′′′( x ) = 6 −1 0 = 6 −1 0 dx 2
2 = y (log ab2 )2
p p p3 p p2 p3
d 1 1
26. log(log x ) = ⋅ = ( x log x )−1
Hence, dx x log x
6 −1 0 d ⎛ 1 ⎞ d ⎡
2
1 ⎤ 1
27. ⎜ x+ ⎟ = x + + 1⎥ = 1− 2
f ′′′(0 ) = 6 −1 0 = 0 , which is independent of p. dx ⎝ x⎠ dx ⎢⎣ x ⎦ x
p p2 p3 x2 x3
28. y = 1+ x +
+ + ∞ ⇒ y = e x
2! 3!
x3 x2 3x2 Differentiating with respect to x, we get
22. f ( x ) = 1 −6 4
dy
= ex = y
p p2 p3 dx
⇒f ( x ) = x 3 ( −6 p3 − 4 p2 ) − x 2 ( p3 − 4 p ) + 3 x 2 ( p2 + 6 p ) d ⎡ −1 ⎛ cos x ⎞ ⎤
29. tan ⎜
3 3 2 3
⇒ f ( x ) = −6 p x − 4 p x − x p + 4 px + 3 p x + 18 px 2 3 2 2 2 2 dx ⎢⎣ ⎝ 1+ sin x ⎟⎠ ⎥⎦
Hence, ⎡ ⎛ x x ⎞⎤
d ⎢ −1 ⎜ cos2 − sin2
d
f ( x ) = −18 p3 x 2 − 12 p2 x 2 − 2 xp3 + 8 px + 6 p2 x + 36 px 2 2 ⎟⎥
= ⎢ tan ⎜ ⎟⎥
dx dx ⎢ x x x x
⎜⎝ cos + sin + 2 sin cos ⎟⎠ ⎥
2 2
⎢⎣ 2 2 2 2 ⎥⎦
d2
and f ( x ) = −36 p3 x − 24 p2 x − 2 p3 + 8 p + 6 p2 + 36 p
dx 2 ⎧ ⎡ ⎛ x ⎞ ⎤⎫
1− tan ⎜ ⎟ ⎥ ⎪
d ⎪⎪ −1 ⎢ ⎝ 2 ⎠ ⎪ d ⎡ −1 ⎛p x⎞⎤ 1
d 3f ( x ) = ⎨tan ⎢ ⎥⎬ = ⎢ tan tan ⎜ − ⎟ ⎥ = −
and = −36 p3 − 24 p2 = a constant dx ⎪ ⎢1+ tan ⎛ ⎞ ⎥ ⎪ dx ⎣
x ⎝ ⎠
4 2 ⎦ 2
dx 3 ⎢⎣ ⎜⎝ ⎟⎠ ⎥
⎩⎪ 2 ⎦ ⎪⎭
sin px p cos px − p2 sin px d d
30. [cos(1− x 2 )2 ] = − sin(1− x 2 )2 (1− x 2 )2
23. D = − p3 cos px p 4 sin px p5 cos px dx dx
− p 6 sin px − p7 cos px p 8 sin px = 4 x (1− x 2 )sin(1− x 2 )2
sin px p cos px − p2 sin px
31. Putting x = sin A and x = sin B , we have
= p 9 − cos px p sin px p2 cos px
y = sin−1(sin A 1− sin2 B + sin B 1− sin2 A )
− sin px − p cos px p2 sin
n px
sin px p cos px − p2 sin px = sin−1[sin( A + B )] = A + B = sin−1 x + sin−1 x
= − p 9 cos px p sin px p2 cos px = 0 dy 1 1
⇒ = +
sin px p cos px 2
− p sin px dx 1− x 2
2 x − x2
Chapter 20 | Differentiation 891
32. Let d x 1
38. (e logsin 2 x ) = e x logsin 2 x + 2e x cos 2 x
x x dx sin 2 x
1+ cos 2 cos2
−1 2 −1 4
y = tan = tan = e x logsin 2 x + e x 2 cot 2 x = e x (logsin 2 x + 2 cot 2 x )
x x
1− cos 2 sin2 ⎛ x + 1⎞ −1 ⎛ x − 1⎞
2 4 39. y = sec −1 ⎜ ⎟ + sin ⎜ ⎟
x ⎛p x⎞ p x ⎝ x − 1⎠ ⎝ x + 1⎠
y = tan−1 cot = tan−1 tan ⎜ − ⎟ = −
⎝ 2 4⎠ 2 4 ⎛ x − 1⎞ −1 ⎛ x − 1⎞ p
4
= cos −1 ⎜ ⎟ + sin ⎜ ⎟=
Therefore, ⎝ x + 1⎠ ⎝ x + 1⎠ 2
dy 1
=− dy ⎧ −1 −1 p⎫
dx 4 ⇒ =0 ⎨Assin x + cos x = ⎬
dx ⎩ 2⎭
1− sin 2 x cos x − sin x 40. Put x = sinq , we get
33. y = =
1+ sin 2 x cos x + sin x d d 3
1− tan x ⎛p ⎞ dy ⎛p ⎞ sin−1(3 x − 4 x 3 ) = sin−1(sin 3q ) =
= = tan ⎜ − x ⎟ ⇒ = − sec2 ⎜ − x ⎟ dx dx 1− x 2
1+ tan x ⎝4 ⎠ dx ⎝4 ⎠
34. f ( x ) = x tan−1 x ⎛ 1+ tan x ⎞ ⎛p ⎞
41. y= ⎜ or y = tan ⎜ + x ⎟
⎝ 1− tan x ⎟⎠ ⎝4 ⎠
Differentiating with respect to x, we get
dy 1 ⎛p ⎞
1 = sec2 ⎜ + x ⎟
f ’( x ) = x + tan−1 x dx ⎛p ⎞ ⎝4 ⎠
1+ x 2 2 tan ⎜ + x ⎟
⎝4 ⎠
Now put x = 1, then
1 p 1 1 ⎡1− tan x ⎤ ⎛p ⎞
f ’(1) = + tan−1(1) = + = sec2 ⎜ + x ⎟
2 4 2 2 ⎢⎣1+ tan x ⎥⎦ ⎝4 ⎠
log(log x ) d ⎛ sec x + tan x ⎞ d ⎛ 1+ sin x ⎞ 2 cos x
35. f ( x ) = log x (log x ) = 42. ⎜ ⎟= ⎜ ⎟=
log x dx ⎝ sec x − tan x ⎠ dx ⎝ 1− sin x ⎠ (1− sin x )2
1 1 1 d ⎡ 1− cos x ⎤ d ⎡ ⎛ x⎞⎤
− log(log x ) −0
⇒ f ’( x ) = x x ⇒ f ’( e ) = e =
1 43. ⎢log ⎥= ⎢log ⎜ tan ⎟ ⎥ = cosec x
2 dx ⎣ 1+ cos x ⎦ dx ⎣ ⎝ 2⎠ ⎦
(log x ) 1 e
d ⎡ −1 1− cos x ⎤ d ⎡ −1 x⎤ 1
36. f ( x ) = 1+ cos2 ( x 2 ) 44. ⎢ tan ⎥= tan tan ⎥ =
dx ⎣ 1+ cos x ⎦ dx ⎢⎣ 2⎦ 2
1
f ’( x ) = ⋅ (2 cos x 2 ) ⋅ ( − sin x 2 ) ⋅ (2 x )
⎛ sin x ⎞ ⎡ x⎤ x
45. f ( x ) = tan−1 ⎜ = tan−1 ⎢ tan ⎥ =
2 2
2 1+ cos ( x )
⎝ 1+ cos x ⎟⎠ ⎣ 2⎦ 2
− x sin 2 x 2
f ’( x ) = 1
1+ cos2 ( x 2 ) ⇒ f ’( x ) =
2
p
At x = , Hence,
2 ⎛p ⎞ 1
f ’⎜ ⎟ =
⎝ 3⎠ 2
p 2p p
⎛ p ⎞ − 2 ⋅ sin 4 − ⋅1
f ’⎜ = = 2 d ⎡ e ax ⎤ ae ax sin(bx + c ) − be ax cos(bx + c )
⎟ 46. ⎢ ⎥=
⎝ 2 ⎠ p 3 dx ⎢⎣ sin(bx + c ) ⎥⎦ [sin(bx + c )]2
1+ cos2
4 2
e ax [a sin(bx + c ) − b cos(bx + c )]
=
Hence, sin2 (bx + c )
⎛ p⎞ p ⎛ ex ⎞
dy ⎡1+ ( x − 2)log x ⎤
f ’⎜ ⎟ =− 6 47. = −2 x −3e x log x + x −2 ⎜ e x log x + ⎟ = e x ⎢ ⎥
⎝ 2 ⎠ dx ⎝ x⎠ ⎣ x3 ⎦
d d ⎛ 1+ sin x ⎞ Aliter: Taking log we have,
37. tan−1(sec x + tan x ) = tan−1 ⎜
dx dx ⎝ cos x ⎟⎠ log y = x + loglog x − 2 log x
⎛ ⎛ x⎞ ⎛ x⎞ ⎞ 1 dy 1 2
d −1
⎜ sin ⎜⎝ 2 ⎟⎠ + cos ⎜⎝ 2 ⎟⎠ ⎟ d ⎛p x⎞
⇒
y dx
= 1+ −
x log x x
= tan ⎜ ⎟= tan−1 tan ⎜ + ⎟
dx ⎜ cos ⎛ x ⎞ − sin ⎛ x ⎞ ⎟ dx ⎝ 4 2⎠
⎜⎝ ⎟⎠ ⎜⎝ ⎟⎠ ⎟ dy e x log x ⎡ x log x + 1− 2 log x ⎤
⎝⎜ 2 2 ⎠ ⇒ = ,⎢ ⎥
dx x2 ⎣ x log x ⎦
d ⎛p x⎞ 1
= ⎜ + ⎟= e x [( x − 2)log x + 1]
dx ⎝ 4 2 ⎠ 2 =
x3
892 Mathematics Problem Book for JEE
e2 x cos x x ⎛p x⎞ p x
48. y = ⇒ log y = 2 x + logcos x − log x − logsin x = tan−1 cot = tan−1 tan ⎜ − ⎟ = −
x sin x 2 ⎝ 2 2⎠ 2 2
1 dy ⎛ − sin x ⎞ 1 cos x ⇒
dy
=−
1
= 2+⎜ − −
y dx ⎝ cos x ⎟⎠ x sin x dx 2
dy ⎡2 1 1 cot2 x ⎤ 57. y x 2 + 1 = log[ x 2 + 1 − x ]
⇒ = e2 x ⎢ cot x − − 2 cot x − ⎥
dx ⎢⎣ x x x x ⎥⎦ Differentiating both sides with respect to x, we get
e2 x dy 1 1 ⎧⎪ 1 2 x ⎫⎪
= [(2 x − 1)cot x − x cosec2 x ] x2 + 1+ y ⋅ ⋅2x = ⋅⎨ − 1⎬
x2 dx 2 x2 + 1 x 2 + 1 − x ⎩⎪ 2 x 2 + 1 ⎭⎪
d − ax 2
dy −1
log(sin x )} = e − ax ( −2ax ).log(sin x ) + e − ax cot x
2 2
49. {e ⇒ ( x 2 + 1) + xy = x 2 + 1⋅
dx dx x2 + 1
= e − ax [cot x − 2ax log(sin x )]
2
dy
⇒ ( x 2 + 1) + xy + 1 = 0
dx
y = log x ⋅ e(tan x + x
2
)
50.
58. f ( x ) = | x 2 − x | ⇒ f ′( x ) = 2 x − 1 ⇒ f ′(2) = 3
Therefore,
dy 1 ⎡ ⎧− x , x < 0⎤
= e(tan x + x ) ⋅ + log x ⋅ e(tan x + x ) (sec2 x + 2 x ) 59. f ( x ) = 3 | 2 + x |; f ′( x ) = −3, ⎢∵| x |= ⎨
2 2
⎥
dx x ⎣ ⎩ x, x > 0 ⎦
2 ⎡ 1 ⎤
= e(tan x + x ) ⎢ + (sec2 x + 2 x )log x ⎥ ⎛ log x ⎞
⎣ x ⎦ 60. f ( x ) = log5 (log7 x ) ⇒ f ( x ) = log5 ⎜ e ⎟
⎝ loge 7 ⎠
1+ e x 1+ e x ⇒f ( x ) = log5 loge x − log5 loge 7
51. y= x
or y 2 =
1− e 1− e x loge loge x
⇒f ( x ) = − log5 loge 7
dy (1− e x )e x + (1+ e x )e x 2e x loge 5
2y = =
dx (1− e x )2 (1− e x )2 dy dy/dt a sin t sin t
Hence, 61. = = =
dx dx/dt a(1− cos t ) 1− cos t
dy ex ⎡ 1− e x ⎤ ⎡ 1− e x ⎤ ex
2 sin ⎛⎜ ⎞⎟ .cos ⎛⎜ ⎞⎟
= t t
⎢ ⎥⎢ ⎥=
x 2 x x
dx (1− e ) ⎢⎣1+ e ⎥⎦ ⎢⎣1− e ⎥⎦ (1− e x ) 1− e2 x ⎝ 2⎠ ⎝ 2⎠ ⎛t⎞
= = cot ⎜ ⎟
⎛ t ⎞ ⎝ 2⎠
d x 1 2 sin2 ⎜ ⎟
52. [e log(1+ x 2 )] = e x log(1+ x 2 ) + e x 2x ⎝ 2⎠
dx (1+ x 2 )
⎛ t⎞
⎡ 2x ⎤ 62. Given that x = a ⎜ cos t + logtan ⎟ and y = a sint .
= e x ⎢log(1+ x 2 ) + ⎝ 2⎠
⎣ 1+ x 2 ⎥⎦ Differentiating with respect to t, we get
e2 x + e −2 x dy
53. y= = a cos t (1)
e2 x − e −2 x dt
Hence, ⎡
dx ⎛ t ⎞ ⎛ 1⎞ ⎛ t ⎞⎤
and = a ⎢ − sin t + cot ⎜ ⎟ ⎜ ⎟ sec2 ⎜ ⎟ ⎥
dy (e 2x
−e −2 x
)2(e 2x
−e −2 x
) − (e 2x
+e −2 x
)2(e 2x
+e −2 x
) dt ⎣ ⎝ ⎠
2 2⎝ ⎠ ⎝ 2⎠ ⎦
= −2 x 2 2
dx (e 2x
−e ) ⎛ 1 ⎞ cos t
= a ⎜ − sin t + ⎟⎠ = a = a cos t cot t (2)
−8 ⎝ sin t sin t
=
(e2 x − e −2 x )2 From Eqs. (1) and (2), we get
dy
54.
3 1
log y = log 2 + log( x − sin x ) − log x = tan t
2 2 dx
Now, (1− x )2 1
2
a = 0, ±
1 = ⋅ =
3 2(1+ x ) (1− x ) 2 2
1+ x 2
At x = –1, we have
1 9 d ⎛ 1+ x ⎞ 1
At a = ± ,b= . tan−1 ⎜ =
3 8 dx ⎝ 1− x ⎟⎠ 2
1+ 3a 2 1
13. a= d ln| x | x ⋅ x − ln| x | 1− ln| x | ln| x |
2a (D) = =
1 2a dx x x2 x2 x
= ⇒
a 1+ 3a 2 d ln| x |
⇒ =1 (at x = –1)
1 dx x
Hence, its greatest value is .
3 2
+x
14. (A) −1 4x 2 + 3x −1 −1 5 x − x −1 3
16. y = tan + tan = tan + tan
1+ 5 x 2 3 − 2x 1+ 5 x ⋅ x 2
x − cos(sin−1 x ) x − 1− x 2 1− ⋅ x
lim = lim 3
x→
1 1− tan(sin−1 x ) x→
1
1−
x
2
2 2
1− x 2 = tan–1 5x – tan–1 x + tan–1 + tan–1 x
3
1 2
= lim ( − 1− x 2 ) = − = tan 5x + tan
–1 –1
x→
1 2 3
2
Therefore,
1 ⎛ 1⎞ dy 5
(B) x = is not in the domain. Hence, f ′ ⎜ ⎟ does not exist. =
2 ⎝ 2⎠ dx 1+ 25 x 2
⎛p x⎞ Hence, l = 5
(C) y = f(x) = ln tan ⎜ + ⎟
⎝ 4 2⎠
17. x = et sin t and y = et cos t
Therefore,
⎛p x⎞ ⇒ x2 + y2 = e2t ⇒ et = x2 + y2 (1)
sec2 ⎜ + ⎟
dy ⎝ 4 2⎠ 1 1 1
= ⋅ = x ⎛ x⎞
dx ⎛p x⎞ 2 2 ⎛p x⎞ ⎛p x⎞ and tan t =⇒ t = tan–1 ⎜ ⎟ (2)
tan ⎜ + ⎟ sin ⎜ + ⎟ cos ⎜ + ⎟ y ⎝ y⎠
⎝ 4 2⎠ ⎝ 4 2⎠ ⎝ 4 2⎠
From Eqs. (1) and (2), we have
1
= = sec x
⎛p ⎞ ⎛ x⎞
sin ⎜ + x ⎟ tan−1 ⎜⎝ y ⎟⎠
⎝2 ⎠ e = x2 + y2 (3)
Thus, p = 0 Taking log on both sides, we get
1 1− cos 2 x | tan x | ⎛ x⎞ 1
(D) lim = lim = Does not exist tan–1 ⎜ ⎟ = ln (x2 + y2)
x →0 x 1+ cos 2 x x →0 x ⎝ y⎠ 2
y T
21.2 Tangent and Normal
A tangent to a point is a line which touches the curve at that point.
P(x, y) A normal to a point is the line which is perpendicular to the tan-
gent at that point.
If the equation of a curve is y = f(x) and a point A(x1, y1) lies on it,
then the equation of the tangent at point A is
⎛ dy ⎞
y – y1 = ⎜ ⎟ ( x − x1)
⎝ dx ⎠ A
Y
x and the equation of the normal at point A is
O 1
y - y1 = - ( x - x1)
(dy / dx ) A
Figure 21.1 Illustration 21.2 Find the equation of tangent and normal for
Then illustration 21.1.
dy Dy Solution: The equation of the tangent is
= lim
dx Dx ® 0 Dx 3
y - ( -3) = - ( x - 3)
⎛ y + Δy − y ⎞ 2
= lim ⎜
Δx → 0 ⎝ x + Δx − x ⎟
⎠ ⇒2y + 6 = −3x + 9
= lim (slope of the chord PQ ) ⇒3x + 2y − 3 = 0
Dx ® 0
The equation of the normal is
Now, this is equal to the slope of the tangent PT at point P(x, y) 2
which, in turn, is equal to tany. Here, y is the angle that the tan- y - ( -3) = + ( x - 3)
3
gent at point P makes with the positive direction of x-axis.
⇒3y + 9 = 2x − 6
Illustration 21.1 Find the slope of tangent at the point that has ⇒2x − 3y − 15 = 0
the ordinate −3 on the curve x3 = 3y2.
Solution: Differentiating the equation of the given curve w.r.t. x, Key Point:
we get When the curve is given in parametric form, that is, when x =
g(t) and y = h(t), the equation of tangent at the point t = t1 is
æ dy ö
3x2 = 3´ ç 2y ÷ h′(t1)
è dx ø y − h(t1) = [ x − g (t1)]
g′(t1)
dy x 2
⇒ = and the equation of normal is
dx 2 y
g¢(t1)
Now, to obtain this value, we require abscissa as well. Substituting y - h (t1) = - [ x - g (t1)]
h¢(t1)
y = −3 in the equation of curve, we have
898 Mathematics Problem Book for JEE
Illustration 21.3 Find the points on the curve y = x3 − x2 − x + 3 where m = dy / dx . Similarly, we can conclude with the following
where the tangent is parallel to the x−axis. results:
Solution: Given curve is 1. PN is called the ‘length of the normal’, which is expressed as
y= − −x+3
x3 x2 (PM)secy = y1 1 + m
2
(from ΔMNP)
dy
⇒ = 3x2 − 2x −1
dx 2. TM is called the ‘subtangent’ which is expressed as
Since the tangent is parallel to the x−axis, the slope is y1 y
(PM)cot ψ = = 1 (from ΔTMP)
tan 0° = 0 tanψ m
That is, 3. MN is called the ‘subnormal’ which is expressed as
dy (PM)tan y = | y1m | (from ΔMNP)
=0
dx
Hence, Illustration 21.4 Find the length of tangent, normal, subtan-
3x2 − 2x −1 = 0 or (3x + 1)(x − 1) = 0 x
gent and subnormal to the curve y = at the point having
Therefore, 1− x 2
1 abscissa 2 .
x= − or 1
3
For the first point, we have Solution: At x = 2 and y = − 2 , the point is P ( )
2 , − 2 . Now,
3 2
1 ⎛ 1⎞ ⎛ 1⎞ ⎛ 1⎞ 86 dy (1- x ) - x ( -2 x ) 1+ x
2 2
x = − and y = ⎜ − ⎟ − ⎜ − ⎟ − ⎜ − ⎟ + 3 = = =
3 ⎝ 3 ⎠ ⎝ 3 ⎠ ⎝ 3 ⎠ 27 dx (1- x 2 )2 (1- x 2 )2
For the second point, we have Therefore,
x = 1 and y = 13 − 12 − 1 + 3 = 2 dy 1+ 2
Hence, the points on the given curve are = =3 =m (say)
dx P (1- 2)2
⎛ 1 86 ⎞ The equation of tangent is
⎜⎝ − , ⎟ and (1, 2)
3 27 ⎠
y + 2 = 3( x − 2 )
21.2.1 Length of Tangent, Normal, Subtangent It intersects x−axis at point
and Subnormal æ4 2 ö
T çç , 0 ÷÷
Let the tangent and the normal at point P(x1, y1) meet x−axis at
è 3 ø
points T and N, respectively (Fig. 21.2). Here, PT is called the length
of the tangent, which is equal to The length of the tangent is
y æ4 2 ö
2
(PM)cosecy = 1 (from ΔTMP) PT = çç - 2 ÷÷ + (0 + 2 )2 =
2 5
sinψ
è 3 ø 3
y
The perpendicular drawn from point P on x−axis meets at
M( 2, 0). The subtangent is
4 2 2
MT = - 2=
3 3
The equation of normal is
P (x1, y1) 1
y + 2 = - (x - 2)
ψ 3
ψ which intersects x−axis at point N( -2 2, 0). The length of the
x
O T N normal is PN = 20 and of the subnormal is MN = 3 2.
M
Aliter: The length of the tangent is
y 1 + m2 ( - 2 ) 1+ 9 2 5
Figure 21.2 = =
m 3 3
Hence, the length of tangent PT is
The length of the normal is
y1 sec ψ y 1 + m2
= 1
tanψ m y 1+ m2 = − ( 2 )× 10 = 2 5
Chapter 21 | Applications of Derivatives 899
y × m = − 2 ×3 = 3 2 m2 − m1
tan θ =
1 + m1m2
Note (Tangent and Normal): Let y = f(x) be the given curve. The
equation of the tangent at (x1, y1) would be where m1 = f ′(x) at P and m2 = g′(x) at point P.
⎛ dy ⎞
y − y1 = ⎜ ⎟ ( x − x1) Remarks:
⎝ dx ⎠( x1 , y1 )
(i) The curves intersect orthogonally if m1m2 = −1.
(ii) The curves touch each other if m1 = m2.
æ dy ö
or y − f(x1) = ç ÷ ( x - x1)
è dx ø( x1 , y1 ) Illustration 21.5 Find the angle of intersection of the curves
Similarly, the equation of the normal at (x1, y1) would be y = x3 and 6y = 7 − x2.
Since the product of the slopes results to be −1, the curves intersect
at right angles.
P(x1, y1)
θ Illustration 21.6 Prove that the curves xy = 4 and x2 + y2 = 8
C2 touch each other.
Solution: First, we must find the common points. Solving the two
Ψ1 Ψ2 equations simultaneously, we get
x 16
O A B x2 + =8 or x4 − 8x2 + 16 = 0
x2
That is,
(x2− 4) 2 = 0 or x2 = 4 or x = ± 2
Figure 21.3 Correspondingly, y = ±2. Hence, the common points are (2, 2) and
(−2, −2).
Let PT1 be the tangent at point P to curve C1 and let PT1 make an dy dy −y
angle y1 with OX. Let PT2 be the tangent at P to curve C2 and let 1. For curve C1: x + y = 0 and hence =
dx dx x
PT2 make an angle y2 with OX. The angle between two curves is
defined to be the angle between the two tangents at the point dy dy −x
2. For curve C2: 2x + 2y = 0 and hence =
of intersection. Therefore, from ΔABP, q, the angle between the dx dx y
curves is At points (2, 2) and (−2, −2), m1 = −1 and m2 = −1, respectively.
∠APB = T1PT2 = y2 − y1 Hence, the curves touch each other at both points.
900 Mathematics Problem Book for JEE
Note (Angle of Intersection of Two Curves): Let y = f(x) and y = We should find dy / dt when y = 6 in. Now,
g(x) be two given intersecting curves. The angle of intersection of
these two curves is defined as the acute angle between the tan- V=
1 ⎡ 1 2 ⎤
(p r2)(y)⎢' volume of a cone = 3 p r h⎥ (1)
gents that can be drawn to the given curves at the point of inter- 3 ⎣ ⎦
section. Now, from similar ΔONM and ΔOCB, we get
MN ON r y y
21.4 dy/dx as Rate Measures = or = or r =
BC OC 5 10 2
In this section, we discuss about how dy / dx is useful in
determination of rates of change related to physical situations. Substituting r = y /2 in Eq. (1), we get
Note:
1. d x and d y are known as differentials.
N r
L M 2. If y = f ( x ) and d y is an increment in y, corresponding to an
increment d x in x, then we have
dy
y δy = (δ x )
dx
3. The error d x in x is called an absolute error.
δx ⎛δx ⎞
O 4. is called the relative error and ⎜ × 100 ⎟ is called the
x ⎝ x ⎠
Figure 21.4 percentage error.
Chapter 21 | Applications of Derivatives 901
For example, y
y= x3
dy
⇒ = 3x2
dx
Note (Increasing Function): The function f(x) is said to be increas- Figure 21.7
ing function in D1 if for every x1, x2 ∈ D1,
Note (Non-Decreasing Function): The function f(x) is said to be
x1 > x2 ⇒ f(x1) > f(x2) non-decreasing in D1 if for every x1, x2 ∈ D1,
f(x) is increasing function in [a, b] if f '( x ) > 0 ∀ x ∈ (a , b ). x1 > x2 ⇒ f(x1) ≥ f(x2)
The function f(x) is non-decreasing in [a, b] if f ′ ( x ) ≥ 0 ∀ x ∈ (a , b ).
21.6.2 Decreasing Behaviour of Function
21.6.4 Non-Increasing Behaviour
If in an interval I, for any two points (x1, y1) and (x2, y2) (Fig. 21.6),
See Fig. 21.8. In this case,
we have
x2 > x1 ⇔ y2 ≤ y1
x2 > x1 ⇔ y2 < y1
for all points in that interval. This implies that
The function is said to be monotonically decreasing function or
dy
simply decreasing function in I. Also, for a differentiable function, ≤0
here (dy / dx ) < 0 for all points in the interval. dx
dy
where = 0 for a continuous set of points in the interval.
y dx
y
y2
y1
x x
x1 x2 a x1 x2 b
Solution:
f ′(x) = 1 − ex O
Now, f ′(x) > 0 ⇒ 1 − ex > 0 ⇒ ex < 1 ⇒ x < 0 a X
and f ′(x) < 0 for all x > 0. Therefore, f(x) is increasing in the interval
(b)
(−∞, 0) and decreasing in the interval (0, ∞). Y
(a) x = c is a point of local maxima if f ′′(c) < 0 21.7.1.1 Test for Local Maximum/Minimum
(b) x = c is a point of local minima if f ′′(c) > 0
1. Test for local maximum/minimum at x = a if f(x) is differentiable
2. First Derivative Test (For Continuous Functions) at x = a: If f(x) is differentiable at x = a and if it is a critical point
(i) If f ′(A) = 0 (or it does not exist) and f ′(x) changes its sign of the function; if f ′(a) = 0 and f ′( x ) changes its sign while
from plus to minus as x passes through the point a from left passing through the point x = a, then
to right, then f(x) is maximum at x = a. (i) f(x) would have a local maximum at x = a if f ′(a − 0) > 0 and
(ii) If f ′(B) = 0 or does not exist and f ′(x) changes its sign from f ′(a + 0) < 0 . It means that f ′( x ) should change its sign from
minus to plus as x passes through the point b from left to positive to negative.
right, then f(x) is minimum at x = b.
(ii) f(x) would have local minimum at x = a if f ′(a − 0) < 0 and
(iii) If the derivative does not change its sign in moving from left f ′(a + 0) > 0 . It means that f ′( x ) should change its sign from
to right through the point a, then f(x) has neither maximum negative to positive.
nor minimum at x = a.
(iii) If f(x) does not change its sign while passing through x = a,
th
3. n Derivative Test then f(x) would have neither a maximum nor minimum at
(i) It can be applied to x = c only if f ′(c) = 0 and f ′′(c) = 0. x = a.
(ii) By differentiation, find nth derivative of f(x) denoted by Second−order derivative test for maxima and minima: Let f(x)
f n(x), n ∈ N. be a differentiable function on an interval I. Let a ∈ I and f ′′(x) is
(iii) Step−by−step, find the earliest non−zero f n(c), n = 3, 4, 5, continuous at x = a. Then
6, 7, … (i) x = a is a point of local maximum if f ′(a) = 0 and f ′′(a) < 0.
(iv) In this process, (ii) x = a is a point of local minimum if f ′(a) = 0 and f ′′(a) > 0.
(a) if n is odd, then x = c is neither local maximum nor local (iii) If f ′(a) = f ′′(a) = 0 and f ′ ′′(a) ≠ 0 exists, then x = a is neither a
minimum point. point of local maximum nor a point of local minimum.
(b) if n is even, and if 2. Test for local maximum/minimum at x = a if f(x) is not
⎧Positive, ⇒ x = c is local minimum point differentiable at x = a:
f n (c ) = ⎨
⎩Negative, ⇒ x = c is local maximum point Case I: When f(x) is continuous at x = a and also f ′(a − h) and
f ′(a + h) exist which are non−zero, then f(x) has a local
4. Absolute Maximum/Minimum Points maximum or minimum at x = a if f ′(a − h) and f ′(a + h)
(i) To find the absolute maximum/minimum values of f(x) in are of opposite signs.
open interval (a, b), we proceed as follows: (a) If f ′(a − h) > 0 and f ′(a + h) < 0, then x = a will be a
(a) Find all extremum points of f(x) by using critical points. point of local maximum.
Let these extremum points be c1, c2, c3 … (b) If f ′(a − h) < 0 and f ′(a + h) > 0, then x = a will be a
(b) Compare the lengths of ordinates f(c1), f(c2), f(c3) , … point of local minimum.
(c) The greatest value of these ordinates is called absolute Case II: When f(x) is continuous and f ′(a − h) and f ′ (a + h) exist,
maximum value of f(x). but one of them is zero, we should infer the information
(d) The least value of these ordinates is called absolute min- about the existence of local maximum/minimum from
imum value of f(x). the basic definition of local maximum/minimum.
Absolute maximum value/absolute minimum value can oc- Case III: If f(x) is not continuous at x = a and f ′ (a − h) and/or
cur at more than one extremum point. Absolute maximum/ f ′(a + h) are not finite, then compare the values of f(x) at
minimum value is also called global maximum/minimum the neighbouring points of x = a.
value of f(x).
(ii) To find the absolute maximum/minimum value in closed 21.7.1.2 Concept of Global Maximum/Minimum
interval [a, b], include the values of ordinates at the end points,
namely, f(a) and f(b) in the above procedure of comparison of Let y = f(x) be a given function with domain D. Let [a, b] ⊆ D. The
the lengths of the ordinates at the extremum points. global maximum/minimum of f(x) in [a, b] is basically the greatest/
least value of f(x) in [a, b]. The global maximum and minimum in
21.7.1 Concept of Local Maximum and Local [a, b] would always occur at critical points of f(x) within [a, b] or at
Minimum the end points of the interval.
Let y = f(x) be a function defined at x = a and also in the vicinity of Illustration 21.15 Find the local maximum and minimum val-
the point x = a. Then, f(x) is said to have a local maximum at x = a
ues of f(x) = 2x3 − 15x2 + 36x +11.
if the value of the function at x = a is greater than the value of the
function at the neighbouring points of x = a. Similarly, f(x) is said Solution: Let y = 2x3 − 15x2 + 36x + 11. Therefore,
to have a local minimum at x = a if the value of the function at
x = a is less than the value of the function at the neighbouring dy
= 6 x 2 − 30 x + 36 = 6(x2 − 5x + 6)
points of x = a. dx
Chapter 21 | Applications of Derivatives 905
d2 y Therefore,
= 12 x - 30 d2 y
dx 2 = - e (e )1/ e < 0
dx 2
For extremum,
dy / dx = 0 Hence, y is maximum when x = 1/ e and the maximum value of
1/ e
⇒ x2 − 5x + 6 = 0 y= e .
Þ (x −2) (x − 3) = 0
Illustration 21.17 If y = a log | x | + bx2 + x has its extreme values
That is, x = 2 or x = 3. Now,
at x = −1 and x = 2, find a and b.
d2 y
= 12(2) − 30 = − 6 < 0 Solution:
dx 2 x =2 y = a log | x | + bx2 + x
Therefore, y or f(x) is a maximum when x = 2 and the maximum Therefore,
value of f(x) = f(2). Therefore, dy a
= + 2bx + 1
f(2) = 2(23) − 15(22) + 36(2) + 11 = 39 dx x
Y
Illustration 21.19 Find the points of local extremum of the E
function f(x) = (2x − 1)2/5(x + 2).
Solution: C
2
f ′(x) = (2x − (1) + (x + 2)
1)2/5 (2x − 1)−3/5 (2)
5
4( x + 2)
= (2 x - 1)2/5 + A
5(2 x - 1)3/5 B
(2 x − 1) + 4 x + 8
= D
5(2 x − 1)3/5
X
That is, a c d e b
6x + 7 Figure 21.11
f ′(x) =
5(2 x - 1)3/5 Fig. 21.11 shows that the graphical interpretation of Rolle’s
For critical points, f ′(x) = 0 or the function is not defined. Therefore, theorem. The slope of tangent is zero at points C, D and E.
7 1
x =- , Note (Rolle’s Theorem): Let y = f(x) be a given function and satis-
6 2
fies the following conditions:
1. f(x) be continuous in [a, b].
+ – + 2. f(x) be differentiable in (a, b).
1/2
If f(a) = f(b), then f ′(c) = 0 at least once for some c ∈ (a, b).
–7/6
Figure 21.10 Illustration 21.21 Verify Rolle’s theorem for f(x) = x3(x − 1)2 in
the interval 0 ≤ x ≤ 1. Also find the value of c in between a and b
See Fig. 21.10. Near x = -7 / 6, the sign of f ′(x) changes from where f ′(x)= 0
positive to negative. Therefore, the local maxima is at x = −7 / 6.
Near x = 1/2, the sign of f ′(x) changes from negative to positive. Solution: We have
Therefore, the local minima is at x = 1/2. f(x) = x3 (x − 1)2 (1)
Therefore,
Illustration 21.20 Find the local maximum/minimum points of
f(x) = (x − 2)3 (x − 3). f ′(x) = 3x2 (x − 1)2 + x3[2(x − 1)] = x2(x − 1)[3(x − 1) + 2x]
or f ′(x) = x2(x − 1)(5x − 3) (2)
Solution:
f ′(x) = (x − − 11) = 0
2)2(4x Clearly, f ′(x) is finite for all x and hence f(x) is differentiable at all x.
⇒x = 2, 2, 11/4 Therefore,
are critical points. (i) f(x) is continuous at all x and hence also continuous in the
f ′′(x) = 2(x − 2) (4x − 11) + 4(x − 2)2 closed interval [0, 1].
f ′′(11/4) > 0 ⇒ x = 11/4 (ii) f(x) is differentiable in the open interval (0, 1).
is local minimum point. (iii) from Eq. (1), f (0) = 0 and f (1) = 0. Therefore, f(0) = f(1).
f ′′(2) = 0 Hence, all conditions of Rolle’s theorem are satisfied. Now from
Therefore, the test for second derivative fails in this case. Eq. (2),
f ′′′(x) = 2(4x − 11) + 16 (x − 2) f ′(c) = 0 ⇒ c2(c − 1)(5c − 3) = 0
f ′′′(2) = − 6 ≠ 0 3
At the earliest non−zero derivative at x = 2 is of odd order, x = 2 is or c = 0, 1,
5
neither local maximum nor local minimum point. Hence, x = 11/4
However, 0 < c < 1. Therefore, c = 3/5. Thus, there exists at least
is the one and only local minimum point of f(x).
one c, that is, c = 3 / 5 between 0 and 1 such that f ′(c) = 0. Hence,
Rolle’s theorem has been verified.
21.8 Mean Value Theorems
Illustration 21.22 Taking the functions f(x) = (x − 3)logx, prove
21.8.1 Rolle’s Theorem that there is at least one value of x in (1, 3) which satisfies xlogx
= 3 − x.
If f(x) is continuous in the interval [a, b] and differentiable in (a, b)
and further f(a) = f(b), then there is at least one point x = c on the Solution: We have
interval (a, b), where f ′(c) = 0. f(x) = (x − 3)logx (1)
Chapter 21 | Applications of Derivatives 907
at least once for some c ∈ (a, b). Illustration 21.25 Find the cone of maximum volume that can
be inscribed in a sphere of radius R.
Illustration 21.23 Find c of the Lagrange’s mean value theorem
for the function f(x) = 3x2 + 5x + 7 in the interval [1, 3]. Solution: Let ABC be the cone with radius R (Fig. 21.13).
908 Mathematics Problem Book for JEE
A D x C
A B
B
M C Figure 21.14
The volume (V) of the box is
(a − 2x)(b − 2x)x
That is,
Figure 21.13
V = abx − 2(a + b) x2 + 4x3
Here, BM = MC = x and the height AM = y. In ΔOMB,
Therefore,
BM2 + OM2 = OB2 dV
= ab − 4x (a + b) + 12x2
x2 + (y − R)2 = R2 dx
Therefore, d 2V
x2 = 2Ry − y2 ⇒ = 24x − 4 (a + b)
dx 2
The volume (V ) of the cone is
Now, dV /dx = 0, when 12x2− 4(a + b)x + ab = 0 or when
1 p
p x 2 y = y (2 R y − y 2 )
3 3 4 (a + b ) ± 16 (a + b )2 − 48ab
x=
24
p
= (2 Ry 2 − y 3 )
3 (a + b ) ± a2 − ab + b2
Therefore, =
6
dV p p
= (4 Ry − 3 y 2 ) = y (4 R − 3 y ) The plus sign gives a value of x greater than b /2 and hence not
dy 3 3
admissible.
dV 4R When
=0 ⇒ y = (y = 0 is meaningless in this context)
dy 3
(a + b ) − a2 − ab + b2
x=
Now, 6
d 2V p d 2V / dx 2 is negative. Therefore, V is maximum when
= (4 R − 6 y )
dy 2 3
(a + b ) − a2 − ab + b2
4 R d 2V p x=
When y = , is (4R − 8R) which is negative. Therefore, 6
3 dy 2 3
4R
V is maximum when y = . The cone has maximum volume Your Turn 2
3
1. Separate the intervals in which the function f(x) = sin x + cos
4R 2 2R
when height is and radius is . cos x is increasing and decreasing.
3 3
æ pö ⎛p p ⎞
Ans. Increasing in ç 0, ÷ ; decreasing in ⎜ , ⎟
Illustration 21.26 A rectangular sheet of metal has four equal è 4ø ⎝4 2⎠
square portions removed from the four corners and the sides are 2. Prove that x > sinx for all x ∈ (0, ∞).
then turned up so as to form an open rectangular box. Show that 3. State true or false: The function f(x) = sinx + 3 cos x has a
when the volume contained in the box is maximum, the depth will maximum value at x = p / 6.
be (1/ 6)[(a + b ) − a2 − ab + b2 ] where a and b, (a > b) are the Ans. True
sides of the original rectangle. 4. The global maximum value of f(x) = x2 − 4x + 20 in the interval
Solution: Let ABCD be the given rectangular sheet of metal with AB [0, 5] is ______.
= a, BC = b and x be the side of the four squares cut off (Fig. 21.14). Ans. 25
Chapter 21 | Applications of Derivatives 909
5. State true or false: The function f(x) = x2/3 does not have any 2. Find the coordinates of the points on the curve
local extrema. y = ( x 2 − 1) / ( x 2 + 1), x > 0 such that tangent at these point(s)
Ans. False have the greatest slope.
6. Find the minimum value of xx. Solution:
1/ e 2
Ans. (1/ e ) y=1−
x +1
2
7. Verify Rolle’s theorem for f(x) = (x − 1)2 (x − 2) in the interval The slope(s) is(are)
[1, 2]. dy 4x
s= = 2
8. Find c of mean value theorem for f ( x ) = x − 4 , a = 2 and b
2 dx ( x + 1)2
= 3. Therefore,
d 2 y ds 4 ⎡⎣( x + 1) 1− ( x )2( x + 1)2 x ⎤⎦
2 2 2
Ans. 5 = =
9. Prove that among all rectangles with the given perimeter, the dx 2 dx ( x 2 + 1)4
square has maximum area. ⎛ x 2 + 1− 4 x 2 ⎞
= 4 ⎜⎜ 3 ⎟ ⎟
10. Find the range of the function f(x) = x3 − 3x2 + 6x − 2 where x ⎝ ( x + 1) ⎠
2
∈ [−1, 1].
⎛ 1 ⎞⎛ 1⎞
Ans. [−12, 2] −12 ⎜ x + ⎟ ⎜x− ⎟
4(1− 3 x 2 ) ⎝ 3⎠ ⎝ 3⎠
= =
(1+ x 2 )3 (1+ x )
2 3
Solution: See Fig. 21.16. Let AF = lAB; AE = mAC. The area of the
d 2L
parallelogram = AF·AEsinA. =0
dt 2 t = 0
A(p 2, −p)
d 3L
However, ≠ 0 implies that there is neither maxima nor
dt 3 t = 0
d 2L
minima at t = 0. Also, > 0 at t = 1, which implies that L is
F E dt 2
minimum at t = 1. So, the shortest distance is
13
(Value of I at t = 1) − (Radius of circle) = −1
3
6. Find a polynomial f(x) of degree 5 which increases in the
interval (−∞, 2] and [6, ∞) and decreases in the interval [2, 6].
B(q 2, q) D C(r 2, −r ) Given that f(0) = 3 and f ′(4) = 0.
Figure 21.16 Solution: See Fig. 21.17. The wavy curve of the derivative will be
In similar triangles, namely, ABC and FBD, we have f ′(x)= k(x− 2)(x − 4)2(x − 6)
and k > 0.
FB BD FD
= = ⇒1−l=m
AB BC AC
The area is
lm(AB)·(AC)sinA
That is,
l(1 − l)AB·(AC)sinA 4
The area is maximum when l is 1/2 (the vertex of parabola y = l − l2),
2 6
which implies that
1
m=l=
2
That is, F and E are the mid−points of AB and AC, respectively.
1
Areamax. = AB ⋅ AC sin A Figure 21.17
4
p2 − p 1 f(x) = k ∫ ( x 2 − 8 x + 12)( x 2 − 8 x + 16) dx
1 1 2
= (Area of ΔABC ) = q q 1
= k ∫ [ x 4 − 16 x 3 + 64 x 2 + 28( x 2 − 8 x ) + 192]dx
2 2 2
r −r 1
⎛ x5 x3 ⎞
1 = k ⎜⎜ − 4 x 4 + 92 − 112 x 2 + 192 x ⎟⎟ + c
= (p + q)(q + r)(p − r) ⎝ 5 3 ⎠
2
5. Find the shortest distance between the curves 9x2 + 9y2 − 30y Now, since f(0) = 3, we have
+ 16 = 0 and y2 = x3. ⎡ x5 92 3 ⎤
f(x) = k ⎢ − 4 x 4 + x − 112 x 2 + 192 x ⎥ + 3, k > 0
Solution: The equation 9x2 + 9y2 − 30y + 16 = 0 can be rewritten as ⎣5 3 ⎦
2 7. A conical vessel is to be prepared out of a circular sheet of gold
⎛ 5⎞
x2 + ⎜ y − ⎟ = 1 of unit radius. How much sectorial area is to be removed from
⎝ 3⎠
the sheet so that the vessel has maximum volume?
Any point on the curve y2 = x3 can be taken to be (t2, t3). Let l be
Solution: See Fig. 21.18.
the distance between the centre of the given circle and the point
(t2, t3). Then Lateral height of the cone = Radius of the circle = 1
2
⎛ 5⎞
L = l2 = t 4 + ⎜ t3 − ⎟
⎝ 3⎠
Now, we calculate the minimum value of l. The required distance is
l − Radius of given circle O
dL ⎛ 5⎞
Now, = 4t 3 + 2 ⎜ t 3 − ⎟ ·3t2 = 0
dt ⎝ 3⎠ 2θ
For maximum or minimum, t = 0 or 1. Now,
B
d 2L A
= 12t2 + 30t4 − 20t
dt 2
Figure 21.18
Chapter 21 | Applications of Derivatives 911
The lateral area of the cone is the area of the circle with sectorial real values of b such that f(x) has the smallest value at x = 1.
area removed, that is, Solution: At x = 1, f(x) = −1.
p (1)2 Smallest value of f(x) = −1. At all other points of the interval, f(x) > −1.
p r(1) = (2p − 2q )
2p Now, for x ≥ 1, f(x) = 2x −3. So,
That is,
f ′(x) = 2 > 0
p −q
r= (here r is radius of the cone) ⇒f(x) is an increasing function
p
Height h of the cone is ⇒Least value exists at x =1
Now, for x < 1, f ′(x) = −3x2 < 0. Therefore,
12 − r 2 f(x) is decreasing function in the interval 0 ≤ x < 1.
Volume of the cone is Therefore, f(x) is the smallest at x =1 and
3 ⎛ b − b + b − 1⎞
3 2
1 ⎛ p −q ⎞
2 2 f(1 − 0) = lim −(1− h) + ⎜⎜ 2 ⎟⎟ ≥ −1
1 2 ⎛ p −q ⎞
V = p r2h = p ⎜ ⎟ × 1 −⎜ ⎟
h→0
⎝ b + 3b + 2 ⎠
3 3 ⎝ p ⎠ ⎝ p ⎠
(b2 + 1)(b − 1)
⇒ −1 + ≥ −1
Upon maximizing the volume V, we get (b + 1)(b + 2)
p −q 2 (b2 + 1)(b − 1)
= ⇒ ≥0
p 3 (b + 1)(b + 2)
⎛ 2⎞ (b − 1)
⇒ q = p ⎜ 1− ⇒ ≥0 (since b2 +1 is positive)
⎜ ⎟ (b + 1)(b + 2)
⎝ 3 ⎟⎠
The area of the sectorial area removed is ⇒ b ∈ (−2, −1) ∪ [1, ∞)
1 2 ⎛ 2⎞ 10. Find all the possible values of the parameter a so that the
(1) (2q ) = p ⎜ 1− ⎟
2 ⎜ 3 ⎟⎠ function, f(x) = x3 − 3(7 − a)x2 − 3 (9 − a2)x + 2, has a negative
⎝
point of local minimum.
8. Let a + b = 4, where a < 2, and let g(x) be a differentiable
a b Solution: We have
function. If (dg / dx ) > 0 for all x, prove that ∫ g( x )dx + ∫ g( x )dx f(x) = x3 − 3(7 − a)x2 − 3(9 − a2)x + 2
0 0 f ′(x) = 3x2 − 6(7 − a)x − 3(9 − a2)
increases as (b − a) increases.
For the distinct real roots, D > 0
Solution: Let b − a = t. Given that 36(7 − a)2 + (4 × 3 × 3)(9 − a2) > 0
a+b=4
⇒ 49 + a2 − 14a + 9 − a2 > 0
Thus,
29
t t ⇒ 14a < 58 ⇒ a <
a=2− ;b=2+ 7
2 2 For local minima:
Let us consider f ′′(x) = 6x − 6(7 − a) > 0
a b
⇒x−7+a>0
f(t) = ∫ g( x )dx + ∫ g( x )dx
0 0 Now, 7− a < x as x must be negative. So,
2 −( t /2) 2 + ( t /2) 7−a<0⇒a>7
Now, f(t) = ∫ g( x )dx + ∫ g( x )dx Thus, by contradiction, that is, for real roots, a < (29/7) and for
0 0
negative point of local minimum a > 7. No possible value of a.
⎛ t ⎞ ⎛ 1⎞ ⎛ t ⎞ ⎛ 1⎞
f ′(t) = g ⎜ 2 − ⎟ ⎜ − ⎟ + g ⎜ 2 + ⎟ ⎜ ⎟
⎝ 2⎠ ⎝ 2⎠ ⎝ 2⎠ ⎝ 2⎠ Previous Years' Solved JEE Main/AIEEE
1
f ′(t) = [ g(b ) − g(a)] Questions
2
1. A value of C for which the conclusion of Mean Value Theorem
dg
Since > 0 for all values of x, g(x) is increasing since b > a holds for the function f ( x ) = loge x on the interval [1, 3] is
dx
1
g(b) > g(a) (A) 2log3 e (B) loge 3
2
Hence, f ′(t) > 0, that is, f(t) increasing as t increases. Therefore, f(t)
(C) log3 e (D) loge 3
increases as (b − a) increases.
Solution: Using mean value theorem, we get
⎧ 3 b3 − b2 + b − 1 f (3) − f (1) 1 log3 − log1 2
9. Let f(x) = ⎪ − x + 2 , 0 ≤ x < 1 . Find all possible f ′(c ) = ⇒ = ⇒c = = 2log3 e.
⎨ b + 3b + 2 3 −1 c 2 loge 3
⎪ 2 x − 3, 1≤ x ≤ 3
⎩ Hence, the correct answer is option (A).
912 Mathematics Problem Book for JEE
2. The function f(x) = tan−1(sin x + cos x) is an increasing function 4. How many real solutions does the equation x7 + 14x5 + 16x3
in + 30x − 560 = 0 have?
⎛π π ⎞ ⎛ π π⎞
(A) ⎜ , ⎟ (B) ⎜ − , ⎟ (A) 7 (B) 1
⎝4 2⎠ ⎝ 2 4⎠
(C) 3 (D) 5
⎛ π⎞ ⎛ π π⎞
(C) ⎜ 0, ⎟ (D) ⎜ − , ⎟ [AIEEE 2008]
⎝ 2⎠ ⎝ 2 2⎠
[AIEEE 2007] Solution:
Solution: x 7 + 14 x 5 + 16 x 3 + 30 z − 560 = 0
⎛ π⎞
2 cos ⎜ x + ⎟
1 ⎝ 4⎠ Let f ( x ) = x 7 + 14 x 5 + 16 x 3 + 30 x. Then
f ′( x ) = (cos x − sin x ) =
1+ (sin x + cos x )2 1+ (sin x + cos x )2
π π π f ′( x ) = 7 x 6 + 70 x 4 + 48 x 2 + 30 > 0 ∀x
⎛π ⎞
f(x) is increasing if f ′ ( x ) > 0 for cos ⎜ + x ⎟ > 0 ⇒ − < x + <
⎝ 4 ⎠ 2 4 2 Therefore, f(x) is a strictly increasing function for all x.
3π π So, it can have at the most one solution.
⇒− <x<
4 4 Hence, the correct answer is option (B).
⎛ π π⎞
Hence, f(x) is increasing when x ∈⎜ − , ⎟ 5. Given P( x ) = x 4 + ax 3 + bx 2 + cx + d such that x = 0 is the
⎝ 2 4⎠
only real root of P′(x) = 0. If P(−1) < P(1), then in the interval
Hence, the correct answer is option (B). [−1, 1]
3. Suppose the cubic x3 − px + q has three distinct real roots (A) P(−1) is the minimum and P(1) is the maximum of P
where p > 0 and q > 0. Then which one of the following holds (B) P(−1) is not minimum but P(1) is the maximum of P
true?
(C) P(−1) is the minimum and P(1) is not the maximum of P
p p
(A) The cubic has minima at and maxima at − (D) neither P(−1) is the minimum nor P(1) is the maximum of P
3 3
p p [AIEEE 2009]
(B) The cubic has minima at − and maxima at
3 3 Solution: P (x) = x 4 + ax 3 + bx 2 + cx + d ; P ′( x ) = 4 x 3 + 3ax 2 + 2bx + c
p p
(C) The cubic has minima at both and − As x = 0 is a solution for P ′( x ) = 0, we have c = 0. Therefore, P( x ) =
3 3
x 4 + ax 3 + bx 2 + d
p p
(D) The cubic has maxima at both and −
3 3 Further, we have P( −1) < P(1) , which implies that,
[AIEEE 2008]
1− a + b + d < 1+ a + b + d ⇒ a > 0
Solution: Let f ( x ) = x − px + q , from Fig. 21.19, for maxima and
3
p As P′( x ) has only one root, that is, x = 0, 4x2 + 3ax + 2b = 0 has
3 3a2
no real roots. Therefore, (3a)2 − 32b < 0 ⇒ < b. Thus, b > 0.
32
p
− Therefore, we have a > 0 and b > 0. That is,
3
P′( x ) = 4 x 3 + 3ax 2 + 2bx > 0 ∀x ∈ (0, 1)
6. The shortest distance between the line y − x =1 and the curve That is, at x = p,
x = y2 is f’′(x) < 0 ⇒ maxima;
3 2 2 3 at x = 2p,
(A) (B) f’′(x) > 0 ⇒ minima.
8 8
3 2 3 Hence, the correct answer is option (C).
(C) (D) 4 9. A spherical balloon is filled with 4500p cubic meters of helium
5
gas. If a leak in the balloon causes the gas to escape at the
[AIEEE 2009] rate of 72p cubic meters per minute, then the rate (in meters
Solution: Let us consider that line (1) be x − y + 1 = 0 and line (2) per minute) at which the radius of the balloon decreases
be the tangent to the curve x = y 2 . Therefore, 49 minutes after the leakage began is
dy dy 1 9 7 2 9
1= 2 y ⇒ = = Slope of given line (2) (A) (B) (C) (D)
dx dx 2 y 7 9 9 2
In order to find the shortest distance, these two lines should be [AIEEE 2012]
parallel, therefore equating their slopes, 4 3
Solution: Volume of the sphere is given by: v = p r
2 3
1 1 ⎛ 1⎞ 1 ⎛ 1 1⎞
= 1⇒ y = ⇒ x = ⎜ ⎟ = ⇒ ( x , y ) = ⎜ , ⎟ After 49 minutes of leakage, the volume is: 4500p − 49(72p) = 972p.
2y 2 ⎝2⎠ 4 ⎝4 2⎠ Therefore,
This is the point on the curve from which if a perpendicular is 4 3
drawn on to the given line, then the length of that perpendicular π r = 972π ⇒ r 3 = 729 ⇒ r = 9
3
will be the shortest distance between the line and the curve. Therefore, the rate (in meters per minute) at which the radius of
1 1 the balloon decreases 49 min after the leakage began is
− +1
4 2 3 3 2 4 dv 4 dr dr dr 72 2
Therefore, the shortest distance is = = v = πr3 ⇒ = π 3r 2 ⇒ 72π = 4π r 2 ⇒ = =
1+ 1 4 2 8 3 dt 3 dt dt dt 4 ⋅ 9 ⋅ 9 9
Hence, the correct answer is option (A). Hence, the correct answer is option (C).
4 10. The population p(t) at time t of a certain mouse species
7. The equation of the tangent to the curve y = x + 2 , that is,
x dp(t )
parallel to the x−axis is satisfies the differential equation = 0.5 p(t ) − 450. If
dt
(A) y = 1 (B) y = 2
p(0) = 850, then the time at which the population becomes
(C) y = 3 (D) y = 0
zero is 1
[AIEEE 2010] (A) 2 ln 18 (B) ln 9 (C) ln18 (D) ln 18
2
Solution: Since the equation is parallel to x−axis, we have, [AIEEE 2012]
dy 8 Solution: We have,
= 0 ⇒ 1− 3 = 0
dx x d ( p(t )) 1
= p(t ) − 450
Therefore, dt 2
(C) Statement 1 is true, statement 2 is true; statement 2 is not Therefore, one tangent passes through the point (2, 2) and has
a correct explanation for statement 1. slope 2
(D) Statement 1 is true, statement 2 is false. y − 2 = 2( x − 2) ⇒ y = 2 x − 2
[AIEEE 2012] The other tangent passes through the point (−2, −2) and has slope 2
Solution:
1 y + 2 = 2( x + 2) ⇒ y = 2 x + 2
f ′( x ) = + 2bx + a
x Substituting y = 0, we get x−intercepts as, x = 1 and −1.
It is given that f has extreme values and hence differentiable. Hence, the correct answer is option (D).
Therefore, 14. The real number k for which the equation 2x3 + 3x + k = 0 has
1 1 1
f ′( −1) = 0 ⇒ a − 2b = 1; f ′(2) = 0 ⇒ a + 4b = − ⇒ a = ; b = − two distinct real roots in [0, 1]
2 2 4
(A) lies between 2 and 3 (B) lies between −1 and 0
Therefore, f ′′( −1), f ′′(2) are negative and f has local maxima at x (C) does not exist (D) lies between 1 and 2
= −1 and 2. [JEE MAIN 2013]
Hence, the correct answer is option (B).
Solution: When the given equation, 2x3 + 3x + k = 0 has two dis-
12. Consider the function f ( x ) = x − 2 + x − 5 , x ∈ R. tinct real roots in [0, 1], then f ′(x) will change sign, but f ′(x) = 6x2 +
3 > 0, for all values of x ∈ R. Therefore, no value of k exists.
Statement 1: f ′(4) = 0.
Hence, the correct answer is option (C).
Statement 2: f is continuous in [2, 5], differentiable in (2, 5) 1
and f(2) = f(5). 15. If g is the inverse of a function f and f '( x ) = , then g′(x)
is equal to 1 + x5
(A) Statement−1 is false, statement−2 is true.
(B) Statement−1 is true; statement−2 is true; statement 2 is a 1
(A) (D) 1+ { g( x )}5
correct explanation for statement−1. 1+ { g( x )}5
(C) Statement−1 is true; statement−2 is true; statement 2 is (C) 1 + x5 (D) 5x4
not a correct explanation for statement−1. [JEE MAIN 2014 (OFFLINE)]
(D) Statement−1 is true, statement−2 is false.
[AIEEE 2012] Solution: Given g (x) = f −1 (x). Therefore,
f (g(x)) = x
Solution: f ′(g(x)) g′(x) = 1
f (x)= 7 − 2x; x > 2
1 1
= 3; 2≤ x ≤5 Thus, g' ( x ) = =
f ′ ( g( x )) 1
= 2 x − 7; x > 5 1+ { g( x )}5
f(x) is constant function in [2, 5]. f is also continuous in [2, 5] and Thus, g′ (x) = 1+ {g(x)}5.
differentiable in (2, 5) and f(2) = f(5); by Rolle’s theorem f ′(4) = 0. Hence, the correct answer is option (B).
Therefore, both Statement 2 and Statement 1 are true and
16. If f and g are differentiable functions in [0, 1] satisfying f(0) = 2
Statement 2 is correct explanation for Statement−1.
Hence, the correct answer is option (B). = g(1), g(0) = 0 and f(1) = 6, then for some c ∈[0, 1]
(A) f ′(c) = g′(c) (B) f ′(c) = 2g′(c)
13. The intercepts on x−axis made by tangents to the curve,
x (C) 2f ′(c) = g′(c) (D) 2f ′(c) = 3g′(c)
y = ∫ t dt , x ∈R , which are parallel to the line y = 2x, are [JEE MAIN 2014 (OFFLINE)]
0
equal to Solution: Let h (x) = f (x) − 2g(x). Then
(A) ±2 (B) ±3 h (0) = f (x) − 2 g(x) = 2 − 2 × 0 = 2
and h (1) = f (1) − 2 g(1) = 6 − 2 × 2 = 2
(C) ±4 (D) ±1
[JEE MAIN 2013] Now h(x) is a differentiable function in [0, 1] and h(0) = h(1), so by
Rolle’s theorem h′(c) = 0 for some c ∈ (0, 1). Therefore,
Solution: Slope of the tangent to the curve will be 2. So we can
equate the slope as, 0 = f ′ (c) − 2 g′ (c) ⇒ f ′ (c) = 2 g′ (c)
dy
= x = 2 ⇒ x = ±2 Hence, the correct answer is option (B).
dx
For x = 2, we have, 17. If x = −1 and x = 2 are extreme points of f (x) = a log |x| + b x2
2 + x, then
y = ∫ t dt = 2 1 1
0
(A) a = 2, b = − 2 (B) α = 2, β = 2
For x = −2, we have, 1 1
−2 (C) a = −6, b = 2 (D) α = −6, β = −
2
y= ∫ t dt = −2 [JEE MAIN 2014 (OFFLINE)]
0
Chapter 21 | Applications of Derivatives 915
Solution: 3
f (x) = a log(x) + b x2 + x = × (cosθ − sinθ )
eθ
α
f '( x ) = + 2β x + 1 Since tangent is parallel to x−axis, we have
x
dy π
Now f ′(− 1) = − a − 2b + 1 = 0 = 0 ⇒ sinθ = cosθ (since, eθ > 0) ⇒ θ =
dx 4
α
f '(2) = + 4 β x + 1= 0 Hence, the correct answer is option (C).
2
Thus, the equations are 20. The volume of the largest possible right circular cylinder that
α + 2 β − 1 = 0 and α + 8 β + 2 = 0 can be inscribed in a sphere of radius = 3 is
On solving the above equations, we get 4 8
1 (A) 3π (B) 3π
β = − ,α = 2 3 3
2
(C) 4p (D) 2p
Hence, the correct answer is option (A).
[JEE MAIN 2014 (ONLINE SET-2)]
18. If the Rolle’s theorem holds for the function f(x) = 2x3 + ax2
1 Solution: See Fig. 21.20.
+ bx in the interval [−1, 1] for the point c = , then the value Volume of required cylinder = V = p r2 h (1)
2
of 2a + b is
(A) 1 (B) −1 (C) 2 (D) − 2
[JEE MAIN 2014 (ONLINE SET-1)]
⎛ 1⎞
Solution: Since Rolle’s theorem holds, so f ′ ⎜ ⎟ = 0 . Now
⎝2⎠
f ’ (x) = 6x2 + 2ax + b
2 O h
⎛ 1⎞ ⎛ 1⎞ 1
f ′ ⎜ ⎟ = 6 ⎜ ⎟ + 2a × + b
⎝2⎠ ⎝2⎠ 2 3
6 −3
or +a+b =0⇒a+b = (1)
4 2
A
Since, f (− 1) = f (1) B
⇒ 2 (− 1)3 + a (− 1)2 + b (− 1) = 2 + a + b
⇒ −2 + a − b = 2 + a + b Figure 21.20
Therefore, Now, by Pythagoras theorem, we have
2b=−4⇒b=−2 2
( 3) ⎛h⎞
2
1 = r2 + ⎜ ⎟
Therefore, from Eq. (1), we get a = . Thus, ⎝2⎠
2
h2
⎛ 1⎞ ⇒ r2 = 3 − (2)
2a + b = 2 ⎜ ⎟ − 2 = −1 4
⎝2⎠ Therefore,
Hence, the correct answer is option (B).
⎛ h2 ⎞ ⎛ h3 ⎞
19. For the curve y = 3 sinq cosq, x = eq sin q, 0 ≤ q ≤ p, the tangent V = p ⎜⎜ 3 − ⎟⎟ h = p ⎜⎜ 3h − ⎟⎟ (3)
⎝ 4 ⎠ ⎝ 4 ⎠
is parallel to x−axis when q is
3π π ⎛ 1 ⎞ ⎛ 3h2 ⎞
V ′ = p ⎜ 3 − × 3h2 ⎟ = p ⎜⎜ 3 − ⎟
(A)
4
(B)
2 ⎝ 4 ⎠ ⎝ 4 ⎟⎠
π π For Extreme V,
(C) (D)
4 6 3 2
V′ = 0 ⇒ 3 = h ⇒ h2 = 4 ⇒ h = ±2
[JEE MAIN 2014 (ONLINE SET-2)] 4
Now
Solution: 3p 3p
V ′′ = − × 2h = − × 2 × 2 = −3p < 0
3 d 4 4
sin2q
dy dy / dq 2 dq
= = q Therefore, volume is maximum when h = 2. From Eq. (1), required
dx dx / dq e cosq + (sinq )eq volume is
3 ⎛ 1 ⎞
(cos2θ )2 V = π ⎜ 3 × 2 − × 23 ⎟ = π (6 − 2) = 4π
2 3{cos2 θ − sin2 θ } ⎝ 4 ⎠
= θ = θ
e (cosθ + sinθ ) e (cosθ + sinθ ) Hence, the correct answer is option (C).
916 Mathematics Problem Book for JEE
1 Therefore,
21. If f ( x ) = x 2 − x + 5, x > and g(x) is its inverse function, then
2 f(q ) = 1 (1 + sin q cos q ) − cos q (− sinq − cos q ) + 1 (− sin2 q + 1)
g′(7) equals
= 1 + sin 2 q + 2 cos2 q = 1 + sin 2 q + 1 + cos 2 q
1 1 1 1
(A) − 3 (B) 13 (C) (D) − ⎧ ⎛ π ⎞⎫
3 13 = 2 + 2 ⎨sin ⎜ 2θ + ⎟ ⎬
[JEE MAIN 2014 (ONLINE SET-3)] ⎩ ⎝ 4 ⎠⎭
f ( x ) = x 2 − x + 5, x >
1 ⎛ π⎞
−1 ≤ sin ⎜ 2θ + ⎟ ≤ 1
2 ⎝ 4⎠
Now
Hence, the correct answer is option (C).
(f )
(7) ′ =
−1 1
=
1
=
1
=
1
( −1
f ′ f (7) ) f ′(2) 2(2) − 1 3 24. If the volume of a spherical ball is increasing at the rate of
4p cc/sec, then the rate of increase of its radius (in cm/sec),
Since f ′ (x) = 2x − 1 and because if f is differentiable and non−
when the volume is 288p cc, is
zero at x = a, then f −1 is differentiable at x = f(a) = b and we have
1 1
1 (A) 6 (B) 9
(f −1)(b ) =
(
f ′ f −1(b ) ) (C)
1
(D) 24
1
æ 1- 2x ö = ê 4 ç cos2 x - ÷ - ú
⇒ r =ç ÷ ëê è 4 ø 16 úû
è π ø
2
p (1- 2 x )2 17 æ 1ö
Now, A= x + 2 = - 4 ç cos2 x - ÷
p2 4 è 4ø
(1- 2 x )2 Therefore,
= x2 + 17
p ymax = M =
4
Therefore,
dA 1 17 æ 9 ö 17 9 8
= 2 x + 2(1- 2 x )( -2) ymin = m = -ç4´ ÷ = - =
dx p 4 è 16 ø 4 4 4
918 Mathematics Problem Book for JEE
Hence, 2
17 8 9 f ′( x ) = − sin2 x cos2 x (2) = − sin4 x
M−m= − = 2
4 4 4
That is,
Hence, the correct answer is option (A).
4 x ∈ (π ,2π ) ∪ (3π , 4π ) ∪ (5π ,6π )
29. The minimum distance of a point on the curve y = x2 − 4 from
the origin is ⎛ π π ⎞ ⎛ 3π ⎞ ⎛ 5π 3π ⎞
x ∈⎜ , ⎟ ∪ ⎜ ,π ⎟ ∪ ⎜ , ⎟
15 19 15 19 ⎝4 2⎠ ⎝ 4 ⎠ ⎝ 4 2 ⎠
(A) (B) (C) (D) Hence, the correct answer is option (C).
2 2 2 2
[JEE MAIN 2016 (ONLINE SET−1)]
y=x
(0, 0)
x
O
(−2, 0) (2, 0)
x
P (x1, x 2 − 4)
(0, 4)
y = ke x
y
Figure 21.21
Figure 21.22
Hence, the correct answer is option (A).
Hence, the correct answer is option (B).
30. Let f(x) = sin4x + cos4x. Then, f is an increasing function in the
interval 2. The positive value of k for which kex − x = 0 has only one root is
1
⎡ 5p 3p ⎤ ⎡ p 5p ⎤ (A) (B) 1
e
(A) ⎢ 8 , 4 ⎥ (B) ⎢ , ⎥
⎣ ⎦ ⎣2 8 ⎦ (C) e (D) loge2
[IIT−JEE 2007]
⎡p p ⎤ ⎡ p⎤
(C) ⎢ 4 , 2 ⎥ (D) ⎢0, ⎥ Solution: Let f ( x ) = ke x − x .Then
⎣ ⎦ ⎣ 4⎦
[JEE MAIN 2016 (ONLINE SET−2)] f ′( x ) = ke x − 1
Substituting f ′( x ) = 0 ⇒ x = − log k , we get
Solution: We have
f ′′( x ) = ke x
f(x) = sin4x + cos4x = 1 − 2sin2x cos2x
f ′′( − log k ) = 1 > 0
1
f ( x ) = 1− sin2 2 x which implies that f ( x ) has one minima at point
2 x = −logk
Chapter 21 | Applications of Derivatives 919
3. For k > 0, the set of all values of k for which kex − x = 0 has two (A) n = 1, m = 1
distinct roots is (B) n = 1, m = −1
⎛ 1⎞ ⎛1 ⎞ (C) n = 2, m = 2
(A) ⎜ 0, ⎟ (B) ⎜ ,1⎟
⎝ e⎠ ⎝e ⎠ (D) n > 2, m = n
⎛1 ⎞ [IIT−JEE 2008]
(C) ⎜ , ∞ ⎟ (D) (0, 1)
⎝e ⎠ Solution: Let f ( x ) = x − 1 . Then
[IIT−JEE 2007]
Solution: We have f(x) = kex − x. As discussed in the Solution of ⎧ x − 1, x ≥ 1
f (x) = ⎨
Question 2, we can show that f(x) has a minima at x = −logk. There- ⎩1− x , x < 1
fore, if f(x) has two distinct roots, then f(−logk) < 0. That is, ⎧ 1, x ≥ 1
f ′( x ) = ⎨
1 ⎩ −1, x < 1
k<
e
Therefore, p = −1.
Hence,
( x − 1)n
⎛ 1⎞ Now, g( x ) =
k ∈ ⎜ 0, ⎟ logcos m ( x − 1)
⎝ e⎠
Hence, the correct answer is option (A). hn ⎛0 ⎞
lim+ g( x ) = lim ⎜ form ⎟
x →1 h → 0 logcos h ⎝ 0 ⎠
4. The total number of local maxima and local minima of the
⎧⎪(2 + x )3 , −3 < x ≤ −1 nhn −1
function f ( x ) = ⎨ is = lim
h → 0 m ⋅ ( − tan h)
2/3
⎪⎩ x , −1 < x < 2
n ⋅ hn − 2
(A) 0 (B) 1 = lim
h→0 ⎛ − tan h ⎞
m⋅⎜ ⎟
(C) 2 (D) 3 ⎝ h ⎠
[IIT−JEE 2008]
n ⋅ hn − 2
Solution: See Fig. 21.23. We have = lim
h→0 ( − m)
⎧⎪(2 + x )3 , −3 < x ≤ −1
f (x) = ⎨ As lim+ g( x ) = −1. Therefore,
x →1
2/3
⎪⎩ x , −1 < x < 2
n.hn −2
y lim = −1
h→ 0 ( −m)
⇒ n = 2 and m = 2
Hence, the correct answer is option (C).
1
⎛ π π⎞
6. Let the function g : ( −∞ , ∞ ) → ⎜ − , ⎟ be given by
⎝ 2 2⎠
π
X
g(u ) = 2 tan−1(e u ) − . Then, g is
−2 −1 0 1 2 2
(A) even and is strictly increasing in (0, ∞)
(B) odd and is strictly decreasing in (−∞, ∞)
(C) odd and is strictly increasing in (−∞, ∞)
(D) neither even nor odd, but is strictly increasing in (−∞, ∞)
[IIT−JEE 2008]
Figure 21.23
Solution: We have
Clearly x = −1 is point of local maxima and x = 0 is a point of local π
minima. Therefore, g(u ) = 2 tan−1(e 4 ) −
2
Total no. of local maxima and minima = 2 π
−1 −4
g( −u ) = 2 tan (e ) −
Hence, the correct answer is option (C). 2
920 Mathematics Problem Book for JEE
(C) f ′(1)f ′( −1) = (2 − a)2 Hence, the correct answer is option (B).
10. Match the statements/expressions in Column I with the values
(D) f ′(1)f ′( −1) = (2 − a)2
[IIT−JEE 2008] given in Column II.
Column I Column II
Solution: We have
x2 + 2x + 4 (p) 0
x 2 − ax + 1 (A) The minimum value of is
f (x) = 2 ,0 < a < 2 x +2
x + ax + 1
( x 2 + ax + 1)(2 x − a) − ( x 2 − ax + 1)(2 x + a) (B) Let A and B be 3 × 3 matrices of real (q) 1
f ′( x ) =
( x 2 + ax + 1)2 numbers, where A is symmetric, B is skew−
2a( x 2 − 1) symmetric, and (A + B)(A − B) = (A − B)(A
=
( x 2 + ax + 1)2 + B). If ( AB )t = ( −1)k AB , where ( AB )t is
the transpose of the matrix AB, then the
4 ax ( x 2 + ax + 1)2 − 4 ax ( x 2 − 1)(2 x + a)( x 2 + ax + 1)
f ″( x ) = possible values of k are
( x 2 + ax + 1)4
4a −4 a (C) Let a = log3 log3 2. An integer k is satisfying (r) 2
f ′(1) = , f ″( −1) = −a
xy + 2 y = x 2 + 2 x + 4 Column I Column II
x + (2 − y ) x + 2(2 − y ) = 0
2 (A) Interval contained in the domain of defini- ⎛ π π⎞
tion of non−zero solutions of the differen- (p) ⎜ − 2 , 2 ⎟
As x is real, therefore, ⎝ ⎠
tial equation (x − 3)2y′ + y = 0
D≥0 (B) Interval containing the value of the inte- ⎛ π⎞
5 (q) ⎜ 0, ⎟
(2 − y ) − 4 ⋅ 2(2 − y ) ≥ 0
2
gral ∫ ( x − 1)( x − 2)( x − 3)( x − 4)( x − 5)dx ⎝ 2⎠
y + 4 y − 12 ≥ 0
2
1
y ≤ −6 or y ≥ 2
(C) Interval in which at least one of ⎛ π 5π ⎞
Minimum value is 2. the points of local maximum of (r) ⎜ 8 , 4 ⎟
⎝ ⎠
(B) → (q , s) cos2 x + sin x lies
(D) Interval in which tan−1(sin x + cos x) is in- ⎛ π⎞
( A + B )( A − B ) = ( A − B )( A + B )
creasing (s) ⎜ 0, ⎟
⇒ AB = BA ⎝ 8⎠
As A is symmetric and B is skew-symmetric, so (t) ( −π , π )
( AB ) = − AB
t [IIT−JEE 2009]
⇒ k = 1, and k = 3 Solution:
dy
(A) ( x − 3) +y =0
2
(C) → ( r , s)
dx
a = log3 log3 2 dx dy
∫ ( x − 3)2 = −∫ y
⇒3 −a
= 3− log3 (log3 2) = log2 3
1
Now, ⇒ = ln y + c
x −3
1 < 2− k +log2 3 < 2
so domain is R − {3}.
1< 3 ⋅ 2− k < 2 (B) Put x = t + 3
⎛ 3⎞ 2 2
⇒ log2 ⎜ ⎟ < k < log2 3
∫ (t + 2)(t + 1)t (t − 1)(t − 2)dt = ∫ t (t − 1)(t 2 − 4)dt = 0 (being odd
2
⎝ 2⎠
−2 −2
⇒ k = 1 or k < 2 and k < 3
function)
(D) → ( p , r ) 2
5 ⎛ 1⎞
(C) f ( x ) = − ⎜ sin x − ⎟
We have 4 ⎝ 2⎠
sinθ = cos φ Maximum value occurs when sin x =
1
2
⎛π ⎞ (D) f ′(x) > 0 if cos x > sin x.
⇒ cos ⎜ − θ ⎟ = cos φ
⎝2 ⎠
Hence, the correct matches are (A)−(p, q, s); (B)−(p, t, s); (C)−(p,
π
⇒ − θ = 2nπ ± φ q, r, t); (D)−(s).
2 1
12. For function f ( x ) = x cos , x ≥ 1,
π x
⇒ −2nπ = θ ± φ − (A) for atleast one x in interval [1, ∞), f(x + 2) − f(x) < 2
2
1⎛ π⎞ (B) lim f ′( x ) = 1
x →∞
⇒ −2n = ⎜ θ ± φ − ⎟
π⎝ 2⎠ (C) for all x in the interval [1, ∞), f(x + 2) − f(x) > 2
(D) f ′(x) is strictly decreasing in the interval [1, ∞)
1⎛ π⎞ [IIT−JEE 2009]
Therefore, ⎜ θ ± φ − ⎟⎠ is even number.
π⎝ 2
Solution: For f ( x ) = x cos ⎛⎜ ⎞⎟ , x ≥ 1
1
1⎛ π⎞ ⎝x⎠
Hence, ⎜ θ ± φ − ⎟ = 0,2 ⎛ 1⎞ 1 ⎛ 1⎞
π⎝ 2⎠ f ′( x ) = cos ⎜ ⎟ + sin ⎜ ⎟ → 1 for x → ∞
⎝x⎠ x ⎝x⎠
Hence, the correct matches are (A)−(r); (B)−(q, s); (C)−(r, s);
⎛ 1⎞ 1 ⎛ 1⎞ 1 ⎛ 1⎞ 1 ⎛ 1⎞
(D)−(p, r). Also f ′( x ) = ⎜ ⎟ + sin ⎜ ⎟ − 2 sin ⎜ ⎟ − 3 cos ⎜ ⎟
⎝x⎠ x ⎝x⎠ x ⎝x⎠ x ⎝x⎠
11. Match the statements/expressions in Column I with the open 1 ⎛ 1⎞
intervals in Column II. =− cos ⎜ ⎟ < 0 for x ≥ 1
x3 ⎝x⎠
922 Mathematics Problem Book for JEE
Solution: Solution:
f ′(x) = 6(x − 2)(x − 3) f ( x ) = ln{ g( x )}
So, f (x) is increasing in (3, ∞). g( x ) = e f ( x )
Also A = {4 ≤ x ≤ 5}. Therefore, g′( x ) = e f ( x ) ⋅ f ′( x )
fmax = f (5) = 7 g′( x ) = 0 ⇒ f ′( x ) = 0 as e f ( x ) ≠ 0
Hence, the correct answer is (7). ⇒ 2010( x − 2009)( x − 2010)2 ( x − 2011)3 ( x − 2012)4 = 0
14. Let p(x) be a polynomial of degree 4 having extremum at x = 1, So, there is only one point of local maxima.
2 and lim ⎛⎜ 1+
p( x ) ⎞ Hence, the correct answer is (1).
⎟ = 2. Then the value of p(2) is _____.
x →0 ⎝ x2 ⎠
17. Let f : R → R be defined as f ( x ) = x + x 2 − 1 . The total number
[IIT−JEE 2009]
of points at which f attains either a local maximum or a local
Solution: Let P( x ) = ax 4 + bx 3 + cx 2 + dx + e . Then minimum is _____.
P′(1) = P′(2) = 0 [IIT−JEE 2012]
⎛ x 2 + p( x ) ⎞ Solution:
lim ⎜⎜ ⎟⎟ = 2
x →0
⎝ x2 ⎠
x x2 −1
⇒ p(0) = 0 ⇒ e = 0 f ′( x ) = + 2 ⋅ (2 x )
x x −1
⎛ 2 x + p '( x ) ⎞ ⎧2 x − 1, x < −1
lim ⎜
x →0 ⎝
⎟⎠ = 2 ⎪ −(2 x + 1),
2x ⎪ − 1< x < 0
=⎨
⇒ p '(0) = 0 ⇒ d = 0 ⎪1− 2 x , 0 < x <1
⎛ 2 + p "( x ) ⎞ ⎪⎩2 x + 1, x >1
lim ⎜
x →0 ⎝
⎟=2
2 ⎠
−ve +ve −ve +ve −ve +ve
⇒ c =1
On solving, a = 1/4, b = − 1
So, −1 −1/2 0 −1/2 1
x4 So, f ′(x) changes sign at points
p( x ) = − x3 + x2
4 1 1
x = −1, − ,0, ,1
⇒ p(2) = 0 2 2
Hence, the correct answer is (0). So, total number of points of local maximum or minimum is 5.
Hence, the correct answer is (5).
15. Let f, g and h be real−valued functions defined on2 the
interval [0, 1] by f ( x ) = e x + e − x , g( x ) = xe x + e − x and
2 2 2
18. Let p(x) be a real polynomial of least degree which has a local
h( x ) = x 2e x + e − x . If a, b and c denote, respectively, the maximum at x = 1 and a local minimum at x = 3. If p(1) = 6 and
2 2
absolute maximum of f, g and h on [0, 1], then p(3) = 2, then p′(0) is _____.
(A) a = b and c ≠ b (B) a = c and a ≠ b [IIT−JEE 2012]
(C) a ≠ b and c ≠ b (D) a = b = c Solution: Let p′( x ) = k ( x − 1)( x − 3) . Then
[IIT−JEE 2010]
⎛ x3 ⎞
Solution: p( x ) = k ⎜⎜ − 2 x 2 + 3 x ⎟⎟ + c
f ( x ) = e x + e − x ⇒ f ′( x ) = 2 x (e x − e − x ) ≥ 0 ∀ x ∈ [0,1]
2 2 2 2
⎝ 3 ⎠
For l = 3, the lengths of sides (as shown in Fig. 21.24) are obtained 1
as 45, 24. (C) − 4 < f ( x ) < 1 (D) −∞ < f ( x ) < 0
[JEE ADVANCED 2013]
x 15l
Solution: We have
d2 y dy
− 2 + y ≥ ex
dx 2 dx
2
−x d y dy
8l
⇒e − 2e − x + e− x y ≥ 1
dx 2 dx
d2
⇒ 2 ( ye − x ) ≥ 1
dx (1)
Figure 21.24 ye −x
Hence, the correct answers are options (A) and (C).
0 1
23. The function f ( x ) = 2 x + x + 2 – x + 2 – 2 x has a local min-
imum or a local maximum at x equals
−2
(A) −2 (B)
3
2 Figure 21.26
(C) 2 (D) −x
3 From Eq. (1) and Fig. 21.26, ye is concave up. Hence, −∞ < f(x) < 0.
[JEE ADVANCED 2013] Hence, the correct answer is option (D).
Solution: We have 25. If the function e−x f(x) assumes its minimum in the interval
x < −2, f ( x ) = −2 x − 4 1
[0, 1] at x = , which of the following is true?
2 4
−2 ≤ x < − , f ( x) = 2x + 4 1 3 1
3 (A) f ′( x ) < f ( x ), < x < (B) f ′( x ) > f ( x ),0 < x <
4 4 4
2
− ≤ x ≤ 0, f ( x ) = −4 x 1 3
3 (C) f ′( x ) < f ( x ),0 < x < (D) f ′( x ) < f ( x ), < x < 1
4 4
0≤ x <2 f (x) = 4x [JEE ADVANCED 2013]
x ≥ 2, f (x) = 2x + 4 d
Solution: See Fig. 21.27. We know that ( ye − x ) is an increasing
It is clear from Fig. 21.25 that x = −2 and x = 0 are the points of min- dx
2
ima. Therefore, x = − is point of maxima. function. Therefore,
3
1 1
0< x < x>
4 4
d d
( ye − x ) < 0 ( ye − x ) > 0
dx dx
dy − x dy − x
e− x − e y < 0 e− x −e y >0
dx dx
8/3 dy dy
<y >y
dx dx
f ′( x ) < f ( x ) f ′( x ) > f ( x )
−2 2 2
−
3
Figure 21.25
Hence, the correct answers are options (A) and (B).
1/4
Paragraph for Questions 24 and 25: Let f: [0, 1] → R (the set of O 1
all real numbers) be a function. Suppose the function f is twice
differentiable, f(0) = f(1) = 0 and satisfies f″(x) − 2f ′(x) + f(x) ≥ ex,
x ∈ [0, 1].
æ 1ö
x -ç t + t ÷ dt Then ìmax {f ( x ), g( x )} if x £ 0,
26. Let f : (0, ¥ ) ® R be given by f ( x ) = 1 e h( x ) = í
ò è ø
t
.
îmin {f ( x ), g( x )} if x > 0.
x
(A) f(x) is monotonically increasing on [1, ∞)
(B) f(x) is monotonically decreasing on (0, 1) |x | + 1
æ 1ö
(C) f ( x ) + f ç ÷ = 0, for all x ∈ (0, ∞)
èxø (0, 2)
(D) f(2x) is an odd function of x on R Sharp edge Sharp edge
1×1
[JEE ADVANCED 2014]
1
Solution:
x
æ 1ö
-ç t + ÷
x e è tø
f (x) = ò 1 dt (−1, 0) (1, 0)
x t
⎛ 1⎞ ⎛1 ⎞
−⎜ x + ⎟ −⎜ + x⎟
⎝ x⎠ ⎝x ⎠ Figure 21.28
d e d e d ⎛ 1⎞
⇒ f (x) = x− × ⎜ ⎟
dx x dx 1 dx ⎝ x ⎠ Points of Intersection are, (1, 2) and (−1, 2). [See Fig. 21.28.]
x h(x) = x2 + 1, in (− ∞,−1)
⎛ 1⎞
−⎜ x + ⎟
x2 + 1= |x| + 1 at −1
⎝ x⎠ ⎛ 1⎞
e −⎜ x + ⎟
⎝ x⎠ ⎛ 1⎞ |x| + 1, in (−1, 0)
= + xe ×⎜− 2⎟
x ⎝ x ⎠ |x| + 1 = x2 + 1 at 0
⎛ 1⎞ = x2 + 1, in (0, 1)
−⎜ x + ⎟ ⎛ 1⎞
e ⎝ x⎠
1 − ⎝⎜ x + x ⎠⎟ x2 + 1 = |x| + 1 at 1
= + e = |x| + 1, in (1, ∞)
x x
⎛ 1⎞
At sharp edges i.e. at −1, 0 and 1, there is no smooth turn, so no
−⎜ x + ⎟
⎝ x⎠ derivative exits there. Elsewhere, function is continuous and differ-
2e
= >0 entiable. Hence, there are three such points.
x Hence, the correct answer is (3).
Therefore, f(x) is strictly increasing in (0, ∞). (1)
28. A cylindrical container is to be made from certain solid
æ 1ö æ 1ö
-ç t + ÷ -ç t + ÷ material with the following constraints: It has a fixed inner
1
æ 1ö x e è tø e è tø
Now f (x) + f ç ÷ = ò 1 dt +ò x dt volume of V mm3, has a 2 mm thick solid wall and is open at
èxø x t x t the top. The bottom of the container is a solid circular disc of
æ 1ö
-ç t + ÷
thickness 2 mm and is of radius equal to the outer radius of
1 è tø
e the container.
=ò x
1 dt = 0 (2) If the volume of the material used to make the container is
t
x minimum when the inner radius of the container is 10 mm,
æ 1ö
Now f (2 x ) + f ç x ÷ = f (2 x ) + f (2- x ) = 0 then the value of
V
is _____.
è2 ø 250p
Therefore, f(2x) is an odd function. (3) [JEE ADVANCED 2015]
Note: Let 2x = m as log2μ = x. For μ ∈ (0, ∞), x ∈ (−∞, ∞) Solution:
We can say f(2x) = h(x) an odd function. Then h(−x) = −h(x).
Therefore, from Eqs. (1), (2) and (3), we can conclude that the
r
correct options are (A), (C) and (D).
Hence, the correct answers are options (A), (C) and (D).
27. Let f : R → R and g : R → R be respectively given by f(x) = |x|
+ 1 and g(x) = x2 + 1. Define h : R ® R by
ìmax {f ( x ), g( x )} if x £ 0, V h
h( x ) = í 2 mm
îmin {f ( x ), g( x )} if x > 0.
The number of points at which h(x) is not differentiable is _____.
[JEE ADVANCED 2014]
Solution: 2 mm
f :R®R g:R®R
f (x) = |x| +1 g(x) = x2 + 1
h:R®R Figure 21.29
926 Mathematics Problem Book for JEE
(r + 2) h - 2
pN 2
r h + (
r + 2) (2)
2 10 1/2 10
volume of volume of Volume of
outer cylinder inner cylinder Base disc é x4 1 ù é x4 1 ù
without base without base ⇒ 64 ê - ú £ f ( x ) £ 96 ê - ú
ëê 4 64 úû ëê 4 64 úû
= 4p h + 4rp h + 2p r2 + 8p r + 8p
1 1 1
4π v 4 rπ .v ⎛ 3⎞
∫ (16 x − 1)dx ≤ ∫ f ( x )dx ≤ ∫ ⎜⎝ 24 x − ⎟ dx
4 4
Vm = + + 2π r 2 + 8π 2 + 8π ⇒
π r2 π r2 1/2 1/2 1/2
2⎠
æ -2 ö æ -1 ö æ 26 39 ö
Now, Vm = 4V ç 3 ÷ + 4V ç 2 ÷ + 4p r + 8p = 0 Clearly, ç , ÷ Ì (1,12)
dr è ø
r èr ø è 10 10 ø
r =10
Hence, the correct answer is option (D).
8V 4V
Þ - - + 40p + 8p = 0
1000 100 30. Let f : R ® (0, ¥ ) and g: R → R be twice differentiable function
48V such that f ¢¢ and g′′ are continuous functions on R. Suppose
Þ − = −48π
1000 f ( x )g( x )
f ¢(2) = g(2) = 0, f ′′(2) ≠ g′(2) ≠ 0. If lim = 1, then
x ® 2 f ¢ ( x )g ¢ ( x )
V
⇒ =4
250p (A) f has a local minimum at x = 2.
Hence, the correct answer is (4). (B) f has a local maximum at x = 2.
(C) f ¢¢(2) > f (2).
192 x 3 æ 1ö
29. Let f ′( x ) = for all x ∈R with f ç ÷ = 0. If (D) f ( x ) - f ¢¢( x ) = 0 for at least one x ÎR.
2 + sin4 π x è2ø
1 [JEE ADVANCED 2016]
m£ ò f ( x )dx £ M , then the possible values of m and M are Solution: Let f : R ® (0, ¥ ) and g: R → R
1/2
(A) m = 13, M = 24 f(x) > 0 "x ÎR
1 1
(B) m = , M = It is given that f ¢(2) = 0, g(2) = 0, f ¢¢(2) ¹ 0 and g¢(2) ¹ 0.
4 2
It is also given that
(C) m = −11, M = 0
(D) m = 1, M = 12 f ( x )g( x ) æ0 ö
lim =1 ç form ÷
x ®2 f ¢( x )g¢( x ) è0 ø
[JEE ADVANCED 2015]
Solution: Applying L’Hospital rule, we get
f ′( x )g( x ) + g′( x )f ( x )
192 x 3 lim =1
f ¢( x ) = "x ÎR , x →2 f ′′( x )g′( x ) + f ′( x )g′′( x )
2 + sin4 π x
For finite limit, we get
1 1
f ¢(2)g(2) + g¢(2)f (2)
Here ò f ( x ) = ò (ò f ¢( x )dx ) (1)
f ¢¢(2)g¢(2) + f ¢(2)g¢¢(2)
=1
1/2 1/2
3 g¢(2)f (2)
192 x 192 x 3 192 x 3 =1
Here, ≤ ≤ f ¢¢(2)g¢(2)
3 2 + sin p x
4
2
f (2)
192
x x
192 x 3 192
x
= 1 ⇒ f ¢¢(2) = f (2) > 0 and f ¢(2) = 0
Þ ∫ ≤ ∫ dx ≤ ∫ x dx f ′′(2)
3 3
x dx
1/2 2 + sin p x
4
3 1/2 2 1/2
which means that f(x) has local minima at x = 2.
192 æ x 4 1 ö 192 æ x 4 1 ö
x
Þ - ÷÷ £ ò f ¢( x )dx £ - ÷ Hence, the correct answer is option (A).
çç ç
3 è 4 64 ø 1/2 2 çè 4 64 ÷ø
f (2) - f ¢¢(2) = 0
Chapter 21 | Applications of Derivatives 927
Therefore, we can say that f ( x ) - f ¢¢( x ) = 0 has at least one solution 10. The angle between curves y 2 = 4 x and x 2 + y 2 = 5 at (1, 2) is
in x ÎR. p
Hence, option (D) is correct. (A) tan-1(3) (B) tan−1(2) (C) (D) p / 4
2
Hence, the correct answers are options (A) and (D).
11. For the curve by 2 = ( x + a)3 , the square of the subtangent is
proportional to
Practice Exercise 1 (A) (Subnormal)
1/2
(B) Subnormal
3/2
1. The points on the curve y = 12x − x3 at which the gradient is (C) (Subnormal) (D) None of these
zero are 12. The tangent to the curve y = ax + bx at (2, − 8) is parallel to
2
(A) (0, 2), (2, 16) (B) (0, −2), (2, −16) x−axis. Then
(C) (2, −16), (−2, 16) (D) (2, 16), (−2, −16) (A) a = 2, b = -2 (B) a = 2, b = -4
2. The area of the triangle formed by the coordinate axes and a (C) a = 2, b = -8 (D) a = 4, b = -4
tangent to the curve xy = a at the point ( x1, y1) on it is
2
13. The sum of intercepts on the coordinate axes made by the
a2 x1 a2 y1 tangent to the curve x + y = a is
(A) (B) (C) 2a2 (D) 4a2
y1 x1 (A) a (B) 2a (C) 2 a (D) None of these
3. The slope of tangent to the curve x = t + 3t − 8, y = 2t
2 2
14. The coordinates of a point on the curve y = x log x at which
- 2t - 5 at the point (2, −1) is the normal is parallel to the line 2 x - 2 y = 3 are
(A) 22 / 7 (B) 6 / 7 (C) − 6 (D) None of these (A) (0, 0) (B) (e , e )
−2 −2
4. The point of the curve y 2 = 2( x - 3) at which the normal is (C) (e 2 , 2e 2 ) (D) (e , −2e )
parallel to the line y - 2 x + 1 = 0 is 15. If normal to the curve y = f ( x ) is parallel to x−axis, then the
correct statement is
æ 1 ö
(A) (5, 2) (B) ç - , -2 ÷ dy dy
è 2 ø (A) =0 (B) =1
dx dx
æ3 ö
(C) (5, −2) (D) ç ,2 ÷ dx
è2 ø (C) =0 (D) None of these
dy
5. The line x + y = 2 is tangent to the curve x 2 = 3 - 2 y at its 16. The length of normal to the curve x = a (θ + sinθ ),
point y = a(1- cosq ) at the point q = p / 2 is
(A) (1, 1) (B) (−1, 1)
(A) 2a (B) a / 2 (C) 2a (D) a / 2
(C) ( 3 , 0) (D) (3, −3)
17. The normal of the curve x = a(cos q + q sinq ),
6. If x = t and y = 2t, then the equation of the normal at t = 1 is
2 y = a(sinq - q cosq ) at any q is such that
(A) x + y - 3 = 0 (B) x + y - 1 = 0 (A) It makes a constant angle with x−axis
(C) x + y + 1 = 0 (D) x + y + 3 = 0 (B) It passes through the origin
7. The equation of the normal to the curve y = sin(p x/2) at the (C) It is at a constant distance from the origin
point (1, 1) is (D) None of these
(A) y = 1 (B) x = 1 18. The slope of the tangent to the curve x = 3t 2 + 1, y = t 3 − 1 at
-2 x = 1 is
(C) y = x (D) y - 1 = p ( x - 1)
(A) 0 (B) 1/ 2 (C) ∞ (D) −2
8. The equation of tangent to the curve y = 2cos x at x = p /4 is 19. An equation of the tangent to the curve y = x from the point
4
x y (1, 1) is
9. At which point the line + = 1, touches the curve y = be − x / a
a b ⎛4⎞
(A) (0, 0) (B) (0, a) (A) tan−1 ⎜ ⎟ (B) tan−1(1)
⎝3⎠
(C) (0, b) (D) (b, 0) -1 æ 3 ö
(C) 90° (D) tan ç ÷
è4ø
928 Mathematics Problem Book for JEE
46. For which value of x, the function f ( x ) = x 2 − 2 x is decreasing? 57. The function f defined by f ( x ) = ( x + 2)e − x is
(A) x > 1 (B) x > 2 (C) x < 1 (D) x < 2 (A) Decreasing for all x
47. The function f ( x ) = cos x − 2 px is monotonically decreasing (B) Decreasing in ( −∞ , − 1) and increasing in ( −1, ∞ )
for (C) Increasing for all x
1 1 (D) Decreasing in ( −1, ∞ ) and increasing in ( −∞ , − 1)
(A) p < (B) p > (C) p < 2 (D) p > 2
2 2
58. If f ( x ) = x 3 − 10 x 2 + 200 x − 10, then
48. If f ( x ) = kx 3 − 9 x 2 + 9 x + 3 is monotonically increasing in each
interval, then (A) f ( x ) is decreasing in [ −∞ ,10] and increasing in [10, ∞]
(A) k < 3 (B) k ≤ 3 (B) f ( x ) is increasing in [ −∞ ,10] and decreasing in [10, ∞]
(C) k > 3 (D) None of these (C) f ( x ) is increasing throughout real line
49. In which interval is the given function f ( x ) = 2 x − 15 x 3 2 (D) f ( x ) is decreasing throughout real line
+ 36 x + 1 is monotonically decreasing? x x
59. If f ( x ) = and g( x ) = , where 0 < x ≤ 1, then in this
(A) [2, 3] (B) (2, 3) (C) ( −∞ ,2) (D) (3, ∞ ) sin x tan x
50. The function f ( x ) = tan x − x interval
(A) Always increases (A) Both f ( x ) and g( x ) are increasing functions
(B) Always decreases (B) Both f ( x ) and g( x ) are decreasing functions
(C) Never decreases (C) f ( x ) is an increasing function
(D) Sometimes increases and sometimes decreases (D) g( x ) is an increasing function
2x
51. The function f ( x ) = log(1+ x ) − is increasing on 60. Function f ( x ) = 2 x 3 − 9 x 2 + 12 x + 29 is monotonically decreas-
2+ x
(A) (0, ∞ ) (B) ( −∞ , 0) ing, when
(C) ( −∞ , ∞ ) (D) None of these (A) x < 2 (B) x > 2 (C) x >1 (D) 1< x < 2
52. The values of a, for which the function (a + 2) x 3 − 3ax 2 + 9ax − 1 61. 2 x 3 + 18 x 2 − 96 x + 45 = 0 is an increasing function when
decreases monotonically throughout for all real x, are
(A) x ≤ −8, x ≥ 2 (B) x < −2, x ≥ 8
(A) a < −2 (B) a > −2
(C) −3 < a < 0 (D) −∞ < a ≤ −3 (C) x ≤ −2, x ≥ 8 (D) 0 ≤ x ≤ −2
65. The function which is neither decreasing nor increasing in 75. The function f ( x ) = 1− x 3 − x 5 is decreasing for
⎛ p 3p ⎞
⎜ , ⎟ is (A) 1 ≤ x ≤ 5 (B) x ≤ 1
⎝2 2 ⎠
(C) x ≥ 1 (D) All values of x
(D) | x − 1|
2
(A) cosec x (B) tan x (C) x
l sin x + 6 cos x 76. The function x x is increasing, when
66. Function f ( x ) = is monotonically increasing, if
2sin x + 3cos x 1 1
(A) x > (B) x <
(A) l > 1 (B) l < 1 (C) l < 4 (D) l > 4 e e
(C) x < 0 (D) For all real x
2
67. On the interval (1, 3), the function f ( x ) = 3 x + is
x 77. 2 x 3 − 6 x + 5 is an increasing function if
(A) Strictly decreasing
(A) 0 < x < 1 (B) −1 < x < 1
(B) Strictly increasing
(C) x < −1 or x > 1 (D) −1 < x < −1/ 2
(C) Decreasing in (2, 3) only
(D) Neither increasing nor decreasing 78. The length of the longest interval, in which the function
68. If f ( x ) = sin x − cos x , the function decreasing in 0 ≤ x ≤ 2p is 3sin x − 4 sin3 x is increasing, is
(A) [5p /6, 3p /4] (B) [p /4, p /2] p p 3p
(A) (B) (C) (D) p
(C) [3p /2,5p /2] (D) None of these 3 2 2
(B) Minimum at x = 1
(A) 2 / 27 (B) 4 / 27 (C) 5 (D) 0
(C) Neither maximum nor minimum at x = 0
(D) Maximum at x = 0 103. If from a wire of length 36 m, a rectangle of greatest area is
88. The adjacent sides of a rectangle with given perimeter as made, then its two adjacent sides (in metre) are
100 cm and enclosing maximum area are (A) 6, 12 (B) 9, 9 (C) 10, 8 (D) 13, 5
(A) 10 cm and 40 cm (B) 20 cm and 30 cm
104. The minimum value of 2 x + x − 1 is
2
(C) 25 cm and 25 cm (D) 15 cm and 35 cm
9
89. The area of a rectangle will be maximum for the given (A) −1/ 4 (B) 3 / 2 (C) −9 / 8 (D)
perimeter, when rectangle is a 4
(A) Parallelogram (B) Trapezium 105. The minimum value of the function y = 2 x 3 − 21x 2 + 36 x − 20
(C) Square (D) None of these is
90. When 36 factorises into two factors in such a way that the (A) −128 (B) −126
sum of factors is minimum, then the factors are (C) −120 (D) None of these
(A) 2, 18 (B) 9, 4 106. The sum of two non−zero numbers is 4. The minimum value
(C) 3, 12 (D) None of these of the sum of their reciprocals is
(A) 3 / 4 (B) 6 / 5
91. If f ( x ) = 2 x 3 − 3 x 2 − 12 x + 5 and x ∈ [ −2, 4] , then the
maximum value of function is at the following value of x (C) 1 (D) None of these
(A) 2 (B) −1 (C) −2 (D) 4 107. The minimum value of [(5 + x )(2 + x )] / [1+ x ] for non-negative
92. The point for the curve y = xe x real x is
(A) x = −1 is minimum (B) x = 0 is minimum (A) 12 (B) 1 (C) 9 (D) 8
(C) x = −1 is maximum (D) x = 0 is maximum
108. One maximum point of sinp x cosq x is
x
93. The maximum value of (1/ x ) is −1 −1
(A) x = tan ( p / q ) (B) x = tan (q / p )
e e −e e
(A) (e ) (B) (e ) (C) (e ) (D) (1/ e ) −1 −1
(C) x = tan ( p / q ) (D) x = tan (q / p )
94. The number that exceeds its square by the greatest amount
is 109. When 20 is divided into two parts so that the product of the
cube of one quantity and the square of the other quantity is
(A) −1 (B) 0 (C) 1/ 2 (D) 1
maximum. The parts are
95. If for a function f ( x ), f ’(a) = 0 , f "(a) = 0 , f ′′′(a) > 0 , then at (A) 10, 10 (B) 16, 4 (C) 8, 12 D) 12, 8
x = a, f ( x ) is
(A) Minimum (B) Maximum 110. If f ( x ) = ( x − 1) / ( x + 1), for every real number x, then the
2 2
150. From mean value theorem f (b ) − f ( a) = (b − a)f '( x1); a < x1 < b Single/Multiple Correct Choice Type Questions
if f ( x ) = 1/ x, then what is the value of x1? 1. For the curve represented parametically by the equations,
(A) ab (B) (a + b )/2 x = 2 ln cot t + 1 and y = tan t + cot t
(A) tangent at t = p/4 is parallel to x-axis
(C) 2ab /(a + b ) (D) (b − a)/(b + a)
(B) normal at t = p/4 is parallel to y-axis
151. For the function x + (1/ x ), x ∈ [1,3], the value of c for the (C) tangent at t = p/4 is parallel to the line y = x
mean value theorem is
(D) tangent and normal intersect at the point (2, 1)
(A) 1 (B) 3
2. Let g′(x) > 0 and f ′(x) < 0, ∀ x∈ R. Then
(C) 2 (D) None of these (A) g(f(x +1)) > g(f(x − 1))
f (b ) − f ( a ) (B) f(g(x − 1)) > f(g(x + 1))
152. If from mean value theorem, f '( x1) = , then
b−a (C) g(f(x + 1)) < g(f(x − 1))
(A) a < x1 ≤ b (B) a ≤ x1 < b (D) g(g(x + 1)) < g(g(x − 1))
934 Mathematics Problem Book for JEE
3. If f(x) = x3 − x2 + 100x + 1001, then Paragraph for Questions 12−14: If a continuous function f
⎛ 1 ⎞ ⎛ 1 ⎞ defined on the real line R, assumes positive and negative values
(A) f(2000) > f(2001) (B) f ⎜ ⎟ > f ⎜⎝ 2000 ⎟⎠ in R then the equation f(x) = 0 has a root in R. For example, if it is
⎝ 1999 ⎠
(C) f(x + 1) > f (x − 1) (D) f (3x − 5) > f (3x) known that a continuous function f on R is positive at some point
and its minimum value is negative then the equation f(x) = 0 has
4. The abscissa of a point on the curve xy = (a + x)2, the normal a root in R.
at which the cuts off numerically equal intercepts from the
coordinate axes is Consider f(x) = kex − x for all real x where k is a real constant.
a 12. The line y = x meets y = kex for k ≤ 0 at
(A) − (B) 2 a
2
(A) no point (B) one point
2a (C) two points (D) more than two points
(C) (D) − 2 a
2 13. The positive value of k for which kex − x = 0 has only one root is
In x
5. For the function f(x) = , which of the following statements (A) 1 e (B) 1
x
are true?
(C) e (D) loge 2
(A) f(x) has the horizontal tangent at x = e
(B) f(x) cuts the x−axis only at the one point
14. For k > 0, the set of all the values of ‘k’ for which kex − x = 0 has
(C) f(x) is the many−one function two distinct roots is
(D) f(x) has the one vertical tangent
⎛ 1⎞ ⎛1 ⎞ ⎛1 ⎞
6. The equation of the tangent drawn to the curve y = (x + 1)3 (A) ⎜ 0, ⎟ (B) ⎜ , 1⎟ (C) ⎜ , ∞ ⎟ (D) (0, 1)
from the origin is ⎝ e⎠ ⎝e ⎠ ⎝e ⎠
(A) y = 3x (B) y = − 3x
Paragraph for Questions 15−17: Let f and g are the two functions
(C) 4y = 27x (D) y = 0
such that f(x) and g(x) are continuous in [a, b] and differentiable in
7. If the derivative of an odd cubic polynomial vanishes at two (a, b). Then at least one c ∈ (a, b), such that
different values of ‘x’, then
f (b ) − f ( a )
(A) the coefficient of x3 and x in the polynomial must be the f ′(c) =
same in sign b−a
(B) the coefficient of x3 and x in the polynomial must be the (i) If f(a) = f(b), then f ′(c) = 0 (RMVT)
different in sign (ii) If f(a) ≠ f(b) and a ≠ b (LMVT)
(C) the values of ‘x’ where derivative vanishes are closer to the f (b ) − f ( a ) f ′(c )
origin as compared to the respective roots on the either (iii) If g′(x) ≠ 0, then = (Cauchy’s theorem)
g(b ) − g( a) g ′(c )
side of origin.
(D) the values of ‘x’ where derivative vanishes are far from the 15. The set of the values of k, for which the equation x3 − 3x + k
origin as compared to the respective roots on the either = 0 has two distinct roots in (0, 1) is
side of the origin. (A) (1, 4) (B) (0, ∞) (C) (0, 1) (D) f
8. Let f(x) = (x − 1)4 (x − 2)n , n∈N. Then f(x) has
(A) Local minimum at x = 2 if n is even 16. Which of the following is true?
(B) Local minimum at x = 1 if n is odd
(A) |tan−1x − tan−1y| ≤ |x − y| ∀ x, y ∈ R
(C) Local maximum at x = 1 if n is odd
(B) |tan−1x − tan−1y| ≥ |x − y| ∀ x, y ∈ R
(D) Local minimum at x = 1 if n is even
(C) |sin x − sin y| ≥ |x − y| ∀ x, y ∈ R
Comprehension Type Questions (D) None of these
p sinα − sin β
Paragraph for Questions 9−11: Let a(t) is a function of t such 17. If 0 < a < q < b < , then is equal to
da 2 cos α − cos β
that = 2 for all the values of t and a = 0 when t = 0. Further (A) tan q (B) − tan q (C) cot q (D) − cot q
dt
y = m(t) x + c(t) is the tangent to the curve y = x2 − 2ax + a2 + a at the
point whose abscissa is 0. Then Paragraph for Questions 18−20: Let y = a x + bx be curve, (2x
9. If the rate of change of the distance of the vertex of y = x − 2ax
2 − y) + l (2x + y − 4) = 0 be family of lines.
+ a2 + a from the origin with respect to t is k, then k = 18. If the curve has slope −
1
at (9, 0), then a tangent belonging
2 2
(A) 2 (B) 2 2 (C) (D) 4 2
to the family of lines is
10. If the rate of change of c(t) with respect to t, when t = k, is A, (A) x + 2y − 5 = 0 (B) x − 2y + 3 = 0
then (C) 3x − y − 1 = 0 (D) 3x + y − 5 = 0
(A) 16 − 2 2 (B) 8 2 + 2 19. A line of the family cutting positive intercepts on the axes and
(C) 10 2 + 2 (D) 16 2 + 2 forming the triangle with the coordinate axes, then the mini-
mum length of the line segment between the axes is
11. The rate of change of m(t), with respect to t, at t = A is (A) (22/3 − 1)3/2 (B) (22/3 + 1)3/2
(A) −2 (B) 2 (C) − 4 (D) 4 (C) 73/2 (D) 27
Chapter 21 | Applications of Derivatives 935
20. Two perpendicular focal chords of the curve y2 − 4x − 4y + 4 (B) The area of the triangle formed by the normal
= 0 form the diagonals of a quadrilateral. Minimum area of a 1
at the point (1, 0) on the curve x = esiny with (q)
quadrilateral is 2
the axes is
(A) 16 (B) 32
(C) If the angle between the curves x2y = 1 and y = (r) 7
(C) 64 (D) 50 e2(1−x) at the point (1, 1) is q , then tanq is equal
to
Paragraph for Questions 21−23: A function f(x) having the fol-
lowing properties: (D) The length of the sub−tangent at any point on (s) 3
the curve y = bex/3 is equal to
(i) f(x) is the continuous except at x = 3
(ii) f(x) is the differentiable except at x = − 2 and x = 3 (t) 0
List I List II 32. A cone is made from a circular sheet of the radius 3 by
(B) If a + b = 1; a > 0, b > 0, then the minimum value (q) 2 cutting out a sector and keeping the cut edges of the
remaining piece together. The maximum volume attainable
⎛ 1 ⎞⎛ 1 ⎞ is λπ
of ⎜ 1+ ⎟⎜ 1+ ⎟ for the cone is , then find l.
⎝ a ⎠⎝ b ⎠ 3
33. If the possible values of ‘a’ such that the inequality 3 − x2 >
(C) The maximum value attained by (r) 3
y = 10 − |x − 10|, −1 ≤ x ≤ 3, is ⎛ 13 ⎞
|x − a| has at least one negative solution is a ∈ ⎜ − , l ⎟ , then
⎝ 4 ⎠
(D) If P(t2, 2t), t ∈ [0, 2] is an arbitrary point on the (s) 4 find l.
parabola y2 = 4x and Q is foot of the perpen-
⎧⎪ xe ax , x ≤0
dicular from focus S on the tangent at P, then 34. Let f(x) = ⎨ , where a is positive constant and
⎪⎩ x + ax − x , x > 0
2 3
maximum area of the triangle PQS is
(t) 5 ⎛ λ1 a ⎞
the interval in which f ′ (x) is increasing is ⎜ − , ⎟ , then find
⎝ a λ2 ⎠
Integer Type Questions (l1 + l2).
x2 y 2 35. A cubic f(x) vanishes at x = −2 and has relative minimum/
29. Let α be the angle in radians between + = 1 and the 1
36 4 1 14
circle x2 + y2 = 12 at their points of the intersection. If a = tan−1 maximum at x = −1 and x =
3
. If ∫ f ( x ) dx =
3
, the cubic
−1
k f(x) = l1x3 + l2x2 − x + 2, then find (l1 + l2).
, then find the value of k2.
2 3 2 36. If f(x) is a twice differentiable function such that f(a) = 0, f(b) =
⎛ 9 ⎞
30. Find the minimum value of (x1 − x2)2 + ⎜ 2 − x12 − ⎟, 2, f(c) = −1, f(d) = 2, f(e) = 0, where a < b < c < d < e, then the
⎝ x2 ⎠ minimum number of the zeroes of g(x) = (f ′(x))2 + f ′′(x) f(x) in
where x1 ∈ (0 , 2 ) and x2 ∈ R+. the interval [a, e] is _____.
1 37. The chord of the parabola y = −p2x2 + 5px − 4 touches the
31. The values of ‘a’ for which the function f(x) = sinx − a sin2x − 1
3 curve y = at the point x = 2 and is bisected by that
sin 3x + 2ax increases throughout the number line is [ l, ∞), (1− x )
then find l. point. Find the number of the values of ‘p’.
Answer Key
Practice Exercise 1
1. (D) 2. (C) 3. (B) 4. (C) 5. (A) 6. (A)
7. (B) 8. (C) 9. (C) 10. (A) 11. (B) 12. (C)
13. (A) 14. (D) 15. (C) 16. (C) 17. (C) 18. (A)
19. (A) 20. (D) 21. (D) 22. (B) 23. (D) 24. (B)
25. (C) 26. (C) 27. (B) 28. (A) 29. (D) 30. (C)
31. (B) 32. (D) 33. (A) 34. (A) 35. (A) 36. (D)
37. (A) 38. (C) 39. (B) 40. (D) 41. (B) 42. (C)
43. (B) 44. (D) 45. (A) 46. (C) 47. (B) 48. (C)
49. (B) 50. (A) 51. (A) 52. (D) 53. (A) 54. (C)
55. (A) 56. (C) 57. (D) 58. (C) 59. (C), (D) 60. (D)
61. (A) 62. (B) 63. (C) 64. (D) 65. (A) 66. (D)
67. (B) 68. (D) 69. (B) 70. (A) 71. (B) 72. (B)
73. (A) 74. (C) 75. (D) 76. (A) 77. (C) 78. (A)
79. (D) 80. (A) 81. (B) 82. (D) 83. (C) 84. (A)
85. (C) 86. (C) 87. (C) 88. (C) 89. (C) 90. (D)
91. (D) 92. (A) 93. (C) 94. (C) 95. (C) 96. (B)
97. (B) 98. (B) 99. (B) 100. (D) 101. (A) 102. (B)
Chapter 21 | Applications of Derivatives 937
103. (B) 104. (C) 105. (A) 106. (C) 107. (C) 108. (A)
109. (D) 110. (D) 111. (A) 112. (C) 113. (A) 114. (B)
115. (B) 116. (D) 117. (D) 118. (A) 119. (A) 120. (B)
121. (C) 122. (D) 123. (B) 124. (C) 125. (A) 126. (B)
127. (B) 128. (A) 129. (D) 130. (D) 131. (D) 132. (C)
133. (A) 134. (C) 135. (B) 136. (D) 137. (A) 138. (A)
139. (B) 140. (B) 141. (D) 142. (D) 143. (D) 144. (A)
145. (D) 146. (D) 147. (B) 148. (B) 149. (C) 150. (A)
151. (B) 152. (A) 153. (A) 154. (A) 155. (B) 156. (D)
157. (C) 158. (A) 159. (D)
Practice Exercise 2
1. (A), (B) 2. (B), (C) 3. (B), (C) 4. (A) 5. (A), (B), (C) 6. (C), (D)
7. (B), (C) 8. (A), (C), (D) 9. (B) 10. (D) 11. (C) 12. (B)
13. (A) 14. (A) 15. (D) 16. (A) 17. (D) 18. (B)
19. (B) 20. (B) 21. (C) 22. (B) 23. (D) 24.(A)→(q),(B)→(r),
(C)→ (t), (D)→ (s) 25. (A)→(r), (B)→ (q), (C)→(t), (D)→ (s) 26. (A)→ (q), (B)→ (s), (C)→ (p), (D)→ (r)
27. (A) → (p), (B) → (p), (C) → (s), (D) →(s) 28. (A) →(q), (B) →(r), (C) → (r), (D) → (t) 29. 16 30. 8
31. 1 32. 2 33. 3 34. 5 35. 2 36. 6 37. 2
Solutions
Practice Exercise 1 or x = 2 x1
5. The given curve is Now, the equation of tangent of given curve at point ( x1, y1) is
x2 = 3 − 2y (1) −y x y x1
y − y1 = 1 ( x − x1) ⇒ + = + 1
Differentiating w.r.t. x, we get a a y1 a
dy dy x y
2x = 0 − 2 ⇒ = −x Comparing with + = 1, we get y1 = b and
dx dx a b
The slope of the tangent of the curve is − x. From the given x
1+ 1 = 1 ⇒ x1 = 0
line, the slope is −1 and hence x = 1 and from Eq. (1), y = 1. a
Therefore, the coordinate of the point is (1, 1). Hence, the point is (0, b).
6. We have x = t and y = 2t . At t = 1, x = 1 and y = 2, we have
2
10. For the curve
⎛ dy ⎞ dy / dt 2 1 ⎛ dy ⎞ dy 4
⎜⎝ ⎟⎠ = = = ⇒ ⎜ ⎟ =1 y2 = 4 x ⇒ =
dx dx / dt 2t t ⎝ dx ⎠ t =1 dx 2 y
we have
Therefore, the equation of the normal at (1, 2) is ⎛ dy ⎞
⎜⎝ ⎟⎠ =1
⎛ 1 ⎞ dx (1,2)
y − 2 = −⎜ ⎟ ( x − 1)
⎝ dy / dx ⎠ and for the curve
⇒ y − 2 = −1( x − 1) dy − x
x2 + y2 = 5 ⇒ =
⇒ x + y −3 = 0 Therefore, dx y
⎛ dy ⎞ −1
7. We have ⎜ ⎟ =
px ⎝ dx ⎠ (1,2) 2
y = sin
2 The angle between the curves is
dy p p ( −1/2) − 1
⇒ = cos x q = tan−1 = tan−1(3)
dx 2 2 1+ ( −1/2)
⇒ ⎛⎜ ⎞⎟
dy
=0 11. We have
⎝ dx ⎠(1, 1)
by 2 = ( x + a)3
Therefore, the equation of the normal is
⎛ dy ⎞
1 ⇒ 2by ⎜ ⎟ = 3( x + a)2
y − 1 = ( x − 1) ⎝ dx ⎠
0
dy 3
⇒ x =1 ⇒ = ( x + a)2
dx 2by
8. We have Therefore, the subnormal is
y = 2cos x
dy 3
At x = p /4, y = 2/ 2 = 2 and therefore, y = ( x + a)2
dx 2b
dy
= −2(sin x ) Therefore, the subtangent is
dx
y y 2by 2
⎛ dy ⎞ = =
⇒⎜ ⎟ =− 2 dy / dx 3( x + a)2 / 2by 3( x + a)2
⎝ dx ⎠ x = p / 4
⎛p ⎞ 2b[( x + a)3 / b] 2
Therefore, the equation of the tangent at ⎜ , 2 ⎟ is = = ( x + a)
⎝4 ⎠ 3( x + a)2 3
⎛ p ⎞ 4
y − 2 = − 2⎜x − ⎟ or (Subtangent)2 = ( x + a)2
⎝ 4⎠ 9
9. Let the point be ( x1, y1). Therefore, (Subtangent)2 (4 / 9)( x + a)2 8b
Also = =
Subnormal (3 / 2b )( x + a)2 27
y1 = be − x1 / a (1)
⇒(Subtangent)2 = Constant × (Subnormal)
Also, the curve is
Therefore, (Subtangent)2 ∝ (Subnormal)
y = be − x / a
dy −b − x / a 12. Now,
⇒ = e y = ax 2 + bx
dx a
Chapter 21 | Applications of Derivatives 939
Now, the point (2, −8) is on the curve of y = ax 2 + bx. Therefore, Therefore,
−8 = 4 a + 2b (2) ⎛ dy ⎞ ⎡ q⎤ ⎛ p⎞
⎜⎝ ⎟⎠ ⎛ p ⎞ = ⎢ tan ⎥ ⎛ p ⎞ = 1[ y ] ⎛ p ⎞ = a ⎜⎝ 1− cos ⎟⎠ = a
dx ⎜ θ = ⎟ ⎣ 2 ⎦ ⎜θ = ⎟ ⎜⎝ θ = ⎠⎟ 2
⎝ 2⎠ ⎝ 2⎠ 2
From Eqs. (1) and (2), we get a = 2, b = −8 .
Hence, the length of the normal is
13. We have
a 1+ (1)2 = 2a
1 dy 1
x + y = a; + =0
2 x 2 y dx 17. We have
y = a(sinθ − θ cosθ ), x = a(cosθ + θ sinθ )
Therefore,
Therefore,
dy y
=− dy
dx x = a[cosθ − cosθ + θ sinθ ] = aθ sinθ
dθ
Hence, the tangent at (x, y) is dx
= a( − sinθ + sinθ + θ cosθ ) = aθ cosθ
y dθ
Y−y =− (X − x)
x That is,
dy dy / dq aq sinq
= = = tanq
or X y + Y x = xy ( x + y ) = axy dx dx / dq aq cosq
⎛ dy ⎞ 1 ⇒ 2x − y + 4 = 0
⇒⎜ ⎟ =
⎝ dx ⎠ (1,1) 2
Therefore, the angle of intersection is 24. We have
2 − (1/ 2) dy
m1 − m2 3 y = 2x2 − x +1 ⇒ = 4 x −1
tanθ = = = dx
1+ m1m2 1+ 2 × (1/ 2) 4
We know that this equation gives the slope of tangent to
⇒ θ = tan−1(3 / 4) the curve. Since this tangent is parallel to y = 3 x + 9, the
slope of the tangent is 3 and so 4 x − 1 = 3 or x = 1. Therefore
21. We have y = 2 x 2 − x + 1 = 2 − 1+ 1 = 2. Thus, the point ( x , y ) is (1, 2).
y = x3 − 3x2 − 9 x + 5
25. We have
dy
⇒ = 3x2 − 6 x − 9 x 3 − 8 a2 y = 0
dx
We know that this equation gives the slope of the tangent to ⇒ 3 x 2 − 8a2 ⎛ dy ⎞ = 0
the curve. The tangent is parallel to the x-axis. Therefore, ⎜ ⎟
⎝ dx ⎠
dy
=0 ⇒ 3 x 2 = 8a2 ⎛⎜ ⎞⎟
dy
dx
⎝ dx ⎠
Therefore, dy 3 x 2
⇒ =
3x − 6 x − 9 = 0
2
dx 8a2
Therefore, the slope of the normal is
⇒ x = −1, 3
1 1 8 a2
22. Clearly, the point of intersection of curves is (0, 1). Now, the − = − 2 = −
(dy / dx ) 2
slope of tangent of the first curve is
3 x / 8a 3x2
Given that
dy
m1 = = a x log a −8a2 −2
dx =
3x2 3
⎛ dy ⎞
⇒⎜ ⎟ = m1 = log a Therefore,
⎝ dx ⎠ (0,1)
( x , y ) = (2a, a)
Chapter 21 | Applications of Derivatives 941
26. We have ⎛ dy ⎞ −b
x = a(t + sin t ), y = a(1− cos t ) ⎜⎝ ⎟⎠ =
dx (0, b ) a
Therefore,
Therefore, the equation of tangent is
dy dy / dt a(sin t ) t
= = = tan −b
dx dx / dt a(1+ cos t ) 2 y −b = ( x − 0)
a
The length of the normal is
x y
2 ⇒ + =1
⎛ dy ⎞ a b
y 1+ ⎜ ⎟
⎝ dx ⎠
30. We have
= a(1− cos t ) 1+ tan (t /2) = a(1− cos t )sec(t /2)
2 y = x2
= 2a sin2 (t /2)sec(t /2) = 2a sin(t /2)tan(t /2) ⎛ dy ⎞
⇒⎜ ⎟ = m1 = 2 x = 2
⎝ dx ⎠ (1,1)
27. We have
y = e2 x and 6 y = 7 − x 3 ⇒ 6.
dy
= −3 x 2
dx
dy
⇒ = 2e 2 x ⎛ dy ⎞ 1
dx ⇒⎜ ⎟ = m2 = −
⎝ dx ⎠ (1,1) 2
⎛ dy ⎞
⇒⎜ ⎟ =2 Clearly, m1m2 = −1. Therefore, the angle of intersection is
⎝ dx ⎠(0,1)
π /2.
Therefore, the equation of tangent is
y − 1 = 2( x − 0) ⇒ y = 2 x + 1
31. We have
This tangent meets x−axis. Therefore,
y =0 y = 2x2 − x +1
Let the coordinates of P be (h, k). Then
⇒ 0 = 2 x + 1⇒ x = −1/ 2
⎛ dy ⎞
⎛ 1 ⎞ ⎜⎝ ⎟⎠ = 4h − 1
Therefore, the coordinates of the point are ⎜ − ,0 ⎟ . dx ( h , k )
⎝ 2 ⎠
28. We have Clearly, P is parallel to y = 3x + 4. Therefore, slopes are equal
(1+ x ) y = 2 − x
2
(1) 4h − 1 = 3 ⇒ h = 1
It meets x-axis, where y = 0. That is, Therefore, P is (1, 2).
0=2− x ⇒ x =2 32. We have
So, Eq. (1) meets x-axis at the point (2, 0). Also from Eq. (1), xy = c 2 (1)
2− x Subnormal is y (dy / dx ) . Therefore, from Eq. (1), we get
y=
1+ x 2
c2 dy − c 2
y= ⇒ = 2
dy (1+ x )( −1) − (2 − x )(2 x )
2
x dx x
⇒ =
dx (1+ x 2 )2 Thus, the subnormal is
dy x − 4 x − 1
2 y ( −c 2 ) − yc 2 − yc 2 y 2 − y 3
⇒ = 2
= 2 2
= = 2
dx (1+ x 2 )2 x (c / y ) c4 c
The slope of tangent at (2, 0) is Therefore, the subnormal varies as y 3 .
4 − 8 − 1 −5 −1 33. If sin x = cos x , then x = p /4.
= =
(1+ 4)2 25 5
⎛ dy ⎞ 1
Therefore, the equation of the tangent at (2, 0) is If y = sin x , then ⎜ ⎟ = .
⎝ dx ⎠ x = π / 4 2
1 ⎛ dy ⎞ −1
y − 0 = − ( x − 2) ⇒ x + 5 y = 2 If y = cos x , then ⎜ ⎟ = .
5 ⎝ dx ⎠ x = π / 4 2
m − m2
29. The curve is y = be − x / a . tanθ = 1 = 2 2 ⇒ θ = tan−1(2 2 )
1+ m1m2
Since the curve crosses y−axis (i.e. x = 0), y = b. Now,
dy −b − x / a 34. We have
= e
dx a
y2 = 5x −1 (1)
At point (0, b), we have
942 Mathematics Problem Book for JEE
36. The slope of the normal is To be decreasing, we need to have f '( x ) < 0 , that is,
⎣ 3 3 9
and f ( x ) to be decreasing for x < 0.
⎡⎛ 10 ⎞
2
500 ⎤
⇒ 3 ⎢⎜ x − ⎟ + ⎥>0 64. See Fig. 21.31. From the trend of value of sin x and cos x ,
⎝ 3⎠
⎣⎢ 9 ⎦⎥
we know sin x and cos x decrease in (p /2) < x < p. So,
2 statement S is correct.
⎛ 10 ⎞ 500
⇒3 ⎜x − ⎟ + >0 Y
⎝ 3⎠ 3
This is always increasing throughout the real line.
59. We have
sin x − x cos x cos x (tan x − x )
f '( x ) = =
sin2 x sin2 x ψ1 ψ2
X
0 < x ≤ 1⇒ x ∈Q1 ⇒ tan x > x , cos x > 0 O a x1 x2 b
Therefore,
Figure 21.31
f '( x ) > 0 for 0 < x ≤ 1
Statement R is incorrect which is depicted in the graph
Thus, f ( x ) is an increasing function. Now, (Fig. 21.31). Clearly, f ( x ) is differentiable in (a, b). Also,
a < x1 < x 2 < b. However,
tan x − x sec2 x sin x cos x − x sin2 x − 2 x
g '( x ) = = =
tan2 x sin2 x 2sin2 x f ′( x1) = tanθ1 < tanθ2 = f ′( x 2 )
Y 70. We have
f ′( x ) = e x (1− x ) + x (e x (1− x ) )(1− 2 x )
2
Now, by the sign−scheme (Fig. 21.33) for −2 x 2 + x + 1,
X ⎡ 1 ⎤
O /2 3 /2 f ′( x ) ≥ 0, if x ∈⎢ − ,1⎥ because e x (1− x ) is always positive.
⎣ 2 ⎦
⎡ 1 ⎤
So, f ( x ) is increasing on ⎢ − ,1⎥ .
⎣ 2 ⎦
− + −
Figure 21.32
66. The function is monotonically increasing if
f ′( x ) > 0
−1/2 1
(2sin x + 3cos x )( λ cos x − 6 sin x )
⇒
(2sin x + 3cos x )2 Figure 21.33
( λ sin x + 6 cos x )(2cos x − 3sin x )
− >0 71. We have
(2sin x + 3cos x )2
f ( x) = x3 − 6x2 + 9x + 3
⇒ 3λ (sin2 x + cos2 x ) − 12(sin2 x + cos2 x ) > 0
For the function to be decreasing, we need to have
⇒ 3λ − 12 > 0 ⇒ λ > 4
f ′( x ) < 0
67. We have
f ( x ) = 3x +
2 ⇒ 3 x 2 − 12 x + 9 < 0
x ⇒ x2 − 4 x + 3 < 0
2
⇒ f ′( x ) = 3 − ⇒ ( x − 3) ( x − 1) < 0
x2
Clearly, f ′( x ) > 0 on the interval (1, 3); therefore, f ( x ) is Therefore, x ∈(1,3).
strictly increasing.
72. We have
68. We have
f ( x ) = sin x − cos x 1
f (x) = − log(1+ x )
Therefore, x +1
⎡ ⎛ π⎞⎤ ⎛ π⎞ ⇒ f ′( x ) = −
1 1
f ′( x ) = cos x + sin x = 2 ⎢cos ⎜ x − ⎟ ⎥ = 2 cos ⎜ x − ⎟ −
⎣ ⎝ 4⎠⎦ ⎝ 4⎠ ( x + 1) 1+ x
2
That is,
For f ( x ) to be decreasing,
f ′( x ) < 0 ⎡ 1 1 ⎤
f ′( x ) = − ⎢ + 2⎥
π π ⎞ 3π ⎣ x + 1 ( x + 1) ⎦
⎛
<⎜x− ⎟ < (within 0 ≤ x ≤ 2p )
2 ⎝ 4⎠ 2 Now, f ′( x ) is negative when x > 0 or f ′( x ) < 0, ∀x > 0.
3π 7π Therefore, f ( x ) is decreasing function.
⇒ <x≤
4 4
69. We have 73. We have
f (x) =
log x f ( x ) = x + cos x
x
⇒ f ′( x ) = 1− sin x
Therefore,
1 log x 1− log x Now, f ′( x ) > 0 for all values of x. Therefore, f ( x ) is an
f ′( x ) =− 2 = increasing function.
x2 x x2
For f ( x ) to be increasing, 74. Let us consider
f ′( x ) > 0
y = x 1/ x
⇒ 1− log x > 0
⇒ 1 > log x Taking log both sides, we have
⇒e> x ⎛ 1⎞
⇒ log y = ⎜ ⎟ log x
⎝ x⎠
Therefore, f ( x ) is increasing in the interval (0, e ).
946 Mathematics Problem Book for JEE
⇒
dy ⎛ 1− log x ⎞
= x 1/ x ⎜
dx ⎝ x 2 ⎟⎠
f(x) = x
1− log x
Now, x 1/ x
> 0 for all x and > 0 in (1, e) and
x2
1− log x X
< 0 in (e , ∞ ). Therefore, f ( x ) is increasing in (1, e)
x2
and decreasing in (e , ∞ ).
75. We have
f ( x ) = 1− x 3 − x 5
⇒ f ′( x ) = −3 x 2 − 5 x 4
Figure 21.34
That is, f ′( x ) < 0 for all values of x.
81. Both e x and 1+ x are increasing and e ≥ 1+ (1/ 2), because
76. Let us consider y = x x . Then e = 1.65 (approximately) and so option (A) is not correct.
Since
dy ⎛ 1 22 ⎞
= x x (1+ log x ) p p
dx sin < ⎜⎝ ' < ⎟⎠
6 6 2 42
85. Obviously, it has a maximum at x = 1. 89. We know that the perimeter of a rectangle is
Therefore, x = 25 and y = 25. Hence, the adjacent sides are Therefore, {f ′′( x )} x = −1 is positive. Hence, f ( x ) is minimum at
25 and 25 cm. x = −1.
948 Mathematics Problem Book for JEE
93. We have 1
x Therefore, y is minimum at x = and minimum value
⎛ 1⎞ e
f (x) = ⎜ ⎟
⎝ x⎠ 1/ e
⎛ 1⎞
=⎜ ⎟ = e −1/ e .
x ⎛ 1⎞ ⎝ e⎠
Since, ⎛⎜ ⎞⎟
1 x log ⎜ ⎟
=e ⎝x⎠
⎝x⎠ 99. We have
dy ⎛ 1⎞ x+y=8
Thus, = e u where, u = x ⋅ log ⎜ ⎟
du ⎝x⎠ Therefore,
On solving this equation, we have
x y =8− x (1)
⇒ f '( x ) = ⎛⎜ ⎞⎟ ⎛⎜ log − 1⎞⎟
1 1
⎝ x⎠ ⎝ x ⎠ Now, f ( x ) = xy = x (8 − x ) = 8 x − x 2
Therefore,
1 1 1
Now, f '( x ) = 0 ⇒ log = 1 = log e ⇒ = e ⇒ x = f ′( x ) = 8 − 2 x
x x e
1/e
Therefore, the maximum value of function is e . For maximum value of f ( x ), f '( x ) = 0. Therefore, x = 4 and
y = 4 and hence the maximum value of xy is 4 × 4 = 16.
94. Let the number be x. Then
100. We have
y = x − x2 x
f ( x ) = ∫ te − t dt ⇒ f '( x ) = xe − x = 0 ⇒ x = 0
2 2
2 0
dy d y
⇒ = 1− 2 x and = −2(< 0)
dx 2 f ′′( x ) = e − x (1− 2 x 2 ); f ′′ (0) = 1 > 0
2
dx
1
⇒ 1− 2 x = 0 ⇒ x = Therefore, the minimum value is f (0) = 0.
2
95. This is a fundamental property. 101. Let the first number be 3 − x and the second number be x.
96. We have Accordingly, we have to maximize (3 − x ) x 2. Let us consider
1 1 f ( x ) = (3 − x ) x 2 = 3 x 2 − x 3 ⇒ f '( x ) = 6 x − 3 x 2
f (x) = x + ⇒ f '( x ) = 1− 2
x x
Therefore,
⇒ f '( x ) = 0 ⇒ x 2 − 1 = 0 ⇒ x = 1, −1
f '( x ) = 0 ⇒ x = 0, 2
However, it is given that x is positive; hence, at x = 1, we have
1 Also
f ( x ) = 1+ = 2 f ′′ ( x ) = 6 − 6 x
1
97. Let us consider Obviously, f ′′ (2) = −6 < 0. Therefore, the required maximum
y = x ⇒ log y = x (log x ), ( x > 0)
x
value is (3 − 2)2 = 4.
2
On differentiating, we get
102. Given that
dy
= x x (1+ log x ) f ( x ) = x (1− x )2
dx
Therefore, That is,
dy f ( x) = x3 − 2x2 + x
=0
dx
Now, f '( x ) = 3 x 2 − 4 x + 1
1
⇒ log x = −1 ⇒ x = e −1 = Substituting f ′( x ) = 0 , we have
e
Therefore, the stationary point is x = 1/e . 3x 2 − 4 x + 1= 0
98. We have, 3 x 2 − 3 x − x + 1 = 0 ⇒ x = 1, 1/ 3
d2 y 1 Now, f ′′( x ) = 6 x − 4
= x x (1+ log x )2 + ( x x )
dx 2 x
When x = 1/ e Therefore, f ′′ (1) = 2 is positive and f ''(1/ 3) = −2 is negative.
(1/ e ) −1 Hence, the maximum value is x = 1/ 3. The maximum is
d 2 y ⎛ 1⎞
=⎜ ⎟ >0
dx 2 ⎝ e ⎠ ⎛ 1⎞ 4
f⎜ ⎟=
⎝ 3 ⎠ 27
Chapter 21 | Applications of Derivatives 949
18 − 2a = 0 ⇒ a = 9; b = 9 (5 + 1)(2 + 1) 6 × 3
f (1) = = = 9.
(1+ 1) 2
104. We have
108. Let us consider that
f ( x) = 2x2 + x −1
y = sinp x (cosq x )
1
⇒ f '( x ) = 4 x + 1⇒ f '( x ) = 0 ⇒ x = − dy
4 Now, = p sinp −1 x (cos x )(cosq x ) + q cosq −1 x ( − sin x )sinp x
dx
Therefore, f ′′ ( x ) = 4 is positive. Therefore,
dy
= p sinp −1 x (cosq +1 x ) − q cosq −1 x (sinp +1 x )
2 1 −9 dx
[f ( −1/ 4)]min = − − 1=
16 4 8 Substituting dy/dx = 0, we get
105. Given that p p
tan2 x = ⇒ tan x = ±
f ( x ) = 2 x 3 − 21x 2 + 36 x − 20 q q
and f '( x ) = 6 x 2 − 42 x + 36 Therefore, the point of maxima is
On substituting, we get p
x = tan−1
f '( x ) = 0 ⇒ 6 x 2 − 42 x + 36 = 0 ⇒ x 2 − 7 x + 6 = 0 q
Therefore, ⎛ d2z ⎞ 2
−1 ≤ f ( x ) < 1 Now, ⎜ 2⎟ = =2
⎝ dx ⎠ x =1 1
Hence, f ( x ) has minimum value −1 and also there is no
maximum value. which is positive. Hence, x = 1 is the point of minima and
x = 1 and y = 1. Therefore, minimum value is
Aliter: We have x+y =2
( x 2 + 1)2 x − ( x 2 − 1)2 x 4x 114. If x / (1+ x tan x ) is maxima, then its reciprocal
f ′( x ) = = 2
( x + 1)
2 2
( x + 1)2
1+ x tan x
f ′( x ) = 0 ⇒ x = 0 x
is a minima. Let us consider
( x 2 + 1)2 4 − 4 x .2( x 2 + 1)2 x 1+ x tan x
Now, f ′′ ( x ) = y=
1
( x 2 + 1)4 = + tan x
x x
( x 2 + 1)4 − 16 x ( x ) −12 x 2 + 4 Therefore,
= =
( x 2 + 1)3 ( x 2 + 1)3 dy 1
= − 2 + sec2 x
Therefore, f ′′(0) > 0 and there is only one critical point that dx x
has minima. Hence, f ( x ) has the least value at x = 0.
d2 y 2
−1 = + 2sec x sec x tan x
fmin = f (0) = = −1 dx 2 x 3
1
111. We have On substituting dy/dx = 0, we get
f ( x ) = cos x + cos( 2 x ) 1
− + sec2 x = 0
f '( x ) = − sin x − 2 sin( 2 x ) = 0 x2
Hence, x = 0 is the only solution. 1
⇒ sec2 x =
f ''( x ) = − cos x − 2cos( 2 x ) < 0 at x = 0 x2
Hence, the maxima occurs at x = 0. ⇒ x 2 = cos2 x
⇒ x = cos x
112. Given that
2
− 2 x +1)sin2 x
Therefore,
y = e(2 x
d2 y 2
For minima or maxima, = + 2sec2 x tan x = 2sec2 x (sec x + tan x )
dx 2 cos3 x
dy
=0 which is positive. At x = cos x ,
dx
1+ x tan x
Therefore, x
2
− 2 x +1)sin2 x
e(2 x [(4 x − 2)sin2 x + 2(2 x 2 − 2 x + 1)sin x cos x ] = 0 is minimum and hence, x / (1+ x tan x ) is maximum.
⇒ [(4 x − 2)sin x + 2(2 x − 2 x + 1)sin x cos x ] = 0
2 2 115. Let us consider
⇒ 2sin x [(2 x − 1)sin x + (2 x 2 − 2 x + 1)cos x ] = 0 x2 − x +1
y= 2
x + x +1
⇒ sin x = 0
Therefore, y is minimum for sin x = 0 . Thus, the minimum dy ( x 2 + x + 1)(2 x − 1) − ( x 2 − x + 1)(2 x + 1)
⇒ =
value of y is dx ( x 2 + x + 1)2
2
− 2 x +1)(0)
y = e(2 x = e0 = 1
dy 2x2 − 2
113. We have ⇒ = 2 = 0 ⇒ 2 x 2 − 2 = 0 ⇒ x = −1, + 1
dx ( x + x + 1)2
1
xy = 1⇒ y = Therefore,
x
d 2 y 4( − x 3 + 3 x + 1)
and let z = x + y. Then =
dx 2 ( x 2 + x + 1)3
dz 1
= 1− 2 d2 y
dx x At x = −1 , < 0, the function occupies the maximum
dz 1 dx 2
Now, = 0 ⇒ 1− 2 = 0 d2 y
dx x value; therefore, f ( −1) = 3 and at x = 1 , > 0, the
dx 2
Thus, function occupies the minimum value. Therefore,
d2z 2 1
x = −1, + 1 and = f (1) =
dx 2 x 3 3
Chapter 21 | Applications of Derivatives 951
Substituting dy / dx = 0, we get y
1− log x
=0
x2 x
B D C
d y −3 x + 2 x log x
2
⇒ 1− log x = 0 ⇒ x = e and =
dx 2 x4
E
At x = e,
Figure 21.36
d2 y 1
= <0 or x = y (2r - y )
2
(1)
dx 2 − e 3
The volume of the cone is
Therefore, in [2, ∞), the function p2 = q is maximum and the
minimum value does not exist. 1 1 1
V = p x 2 y = p y (2r - y ) y = p (2ry 2 - y 3 )
3 3 3
117. We have
dV 1 dV
2 3 −x 4 −x
f ( x ) = x 4 e − x ⇒ f ′( x ) = 4 x e + x e ( −2 x )
2 2
⇒ = p (4 ry − 3 y 2 ) ⇒ =0
dy 3 dy
For maximum value
1 4
⇒ p (4 ry - 3 y 2 ) = 0 ⇒ y (4 r - 3 y ) = 0 ⇒ y = r ,0
f ′( x ) = 0 ⇒ 4 x 3 e − x − 2 x 5 e − x = 0
2 2
3 3
⇒ x2 = 2 ⇒ x = ± 2 d 2V 1
Now, = p (4 r - 6 y )
Now, dy 2 3
Substituting y = (4 / 3)r, we get
f ′′( x ) = 12 x 2e − x + 4 x 2e − x ( −2 x ) − 10 x 4 e − x − 2 x 5e − x ( −2 x )
2 2 2 2
f ¢( A) = cos2 A - sin2 A = cos2 A 120. See Fig. 21.37. Let the diameter of sphere be AE = 2r. Let the
radius of the cone be x and its height be y. Therefore, AD = y
p p since BD 2 = AD(DE ).
Now, f ¢( A) = 0 Þ cos2 A = 0 ⇒ 2 A = ÞA=
2 4 A
p
and f ¢¢( A) = -2sin2 A = -2sin = -2 (negative)
2
Hence, f ( A) is maximum at p / 4. Therefore, maximum value
is y
p p 1
cos sin =
4 4 2
119. Let the diameter of the sphere be x
B D C
AE = 2r
Let radius of cone (Fig. 21.36) be x and height be y. There-
fore, AD = y since E
BD = AD(DE )
2
Figure 21.37
952 Mathematics Problem Book for JEE
and hence at x = 1, g( x ), that is, the slope will have its Length of the arc is
maximum value. Therefore, s
[ g(1)]max. = -3 ´ 1+ 6 + 9 = 12 s = rq or q =
r
Therefore, the perimeter of the sector is
127. We have
f ( x ) = 2 x 3 - 3 x 2 - 12 x + 4 p = r + s + r = 2r + s (2)
s
and f ¢( x ) = 6 x 2 - 6 x - 12 Substituting q = in Eq. (1), we have
r
Now, f ¢( x ) = 0 ⇒ x 2 - x - 2 = 0 ⇒ x = 2, - 1 æ 1 ö æ sö 1 2A
A = ç r 2 ÷ ç ÷ = rs ⇒ s =
and f ¢¢( x ) = 12 x - 6 è2 ø è rø 2 r
That is f ′′(2), which is positive and f ¢¢( -1) is negative. Now, substituting the value of s in Eq. (2), we get
Therefore, the given function has one maximum and one æ 2Aö
p = 2r + ç ÷ or 2 A = pr - 2r 2
minimum. è r ø
128. We have Differentiating w.r.t. r, we get
1 dA
f (x) = 2 2 = p - 4r
4 x + 2x +1 dr
-(8 x + 2)
⇒ f ¢( x ) = We know that for the maximum value of area is
(4 x 2 + 2 x + 1)2 dA p
= 0 or p - 4 r = 0 or r =
Substituting f ¢( x ) = 0, we get dr 4
131. We have
8 x + 2 = 0 ⇒ x = -1 dy a
4 = + 2bx + 1
dx x
-[(4 x 2 + 2 x + 1)2 8 - (8 x + 2)2(4 x 2 + 2 x + 1)(8 x + 2)]
f ¢¢( x ) = æ dy ö
(4 x 2 + 2 x + 1)4 ⇒ç ÷ = a + 2b + 1 = 0
è dx ø x =1
Now, f ¢¢( -1/ 4) is negative (i.e. point of maxima). Therefore,
⇒ a = -2b - 1
1 4 æ dy ö a
f ( -1/ 4)max. = = and = + 4b + 1 = 0
4 × (1/ 16) − 2 × (1/ 4) + 1 3 çè ÷ø
dx x = 2 2
129. We have −2b − 1
⇒ + 4b + 1 = 0
1 1 2
f ( x ) = x + ⇒ f ′( x ) = 1 −
x x2 1
⇒ -b + 4b + =0
Substituting f ′( x ) = 0, we get x = −1, 1. Since x > 0, no 2
maximum value can be found. -1 -1
⇒ 3b = ⇒ b=
130. See Fig. 21.38. 2 6
1 -2
A and a= - 1=
3 3
r 132. We have
x
f (x) = ò (t 4 - 4)e -4 t dt ⇒ f ¢( x ) = ( x 4 - 4)e -4 x
s -10
O θ
Now, f ¢( x ) = 0 Þ x = ± 2 , ± 2
Therefore, 1 3
a = 0, f (a) = 0 ⇒ b = , f (b ) =
−1 (1/ b ) − (1/ a) 1 2 8
= = − ⇒ x1 = ab
x12 b−a ab f ′( x ) = ( x − 1)( x − 2) + x ( x − 2) + x ( x − 1)
151. We have f ′(c ) = (c − 1)(c − 2) + c (c − 2) + c (c − 1)
1 1
f ′( x ) = 1 − 2 ⇒ f ′( c ) = 1 − 2 = c 2 − 3c + 2 + c 2 − 2c + c 2 − c
x c
That is,
1 (10 / 3) − 2 1 2
1− = ⇒ 1− 2 = ⇒ c 2 = 3 f' (c ) = 3c 2 − 6c + 2
c2 2 c 3
According to mean value theorem, we have
⇒c = 3
f (b ) − f ( a )
152. According to mean value theorem, in an interval [a, b] for f(x), f ′( c ) =
b−a
we have
f (b ) − f ( a ) (3 / 8) − 0 3
= f ′( c ) ⇒ 3c 2 − 6c + 2 = =
b−a (1/ 2) − 0 4
5
where a < c < b. Therefore, a < x1 < b . ⇒ 3c 2 − 6c + =0
4
153. We have 6 ± 36 − 15 6 ± 21 21
c= = = 1±
f ( x ) = e −2 x sin2 x 2×3 6 6
154. From Rolle’s theorem in (1, 26), f (1) = f (26) = 5. In the given 8 − ( −8)
⇒ = 3x2
interval, the function satisfies all conditions of Rolle’s theo- 4
Therefore,
rem. Therefore, in [1, 26], at least, there is a point for which
2
f ′( x ) = 0. x=±
3
2 159. We have
∫1 f ′( x )dx = [f ( x )]1 = f (2) − f (1) = 0
2
155. We have because f ( x )
f (x) = x
satisfies the conditions of Rolle’s theorem. Therefore,
Therefore,
f (2) = f (1)] ).
f (a) = 4 = 2, f (b ) = 9 = 3
156. We have
1
Now, f ′( x ) =
f ( x ) = x 3 − 6 x 2 + ax + b 2 x
⇒ f ′( x ) = 3 x − 12 x + a
2 f (b ) − f ( a ) 3 − 2 1
Also, f ′( c ) = = =
b−a 9−4 5
⎛ 1 ⎞
⇒ f ′( c ) = 0 ⇒ f ′ ⎜ 2 + ⎟ =0 Therefore,
⎝ 3⎠
1 1 25
2 = ⇒ c= = 6.25
⎛ 1 ⎞ ⎛ 1 ⎞ 2 c 5 4
⇒ 3⎜ 2 + ⎟ − 12 ⎜⎝ 2 + ⎟ +a=0
⎝ 3⎠ 3⎠
Practice Exercise 2
⎛ 1 4 ⎞ ⎛ 1⎞
⇒ 3⎜ 4 + + ⎟ − 12 ⎜⎝ 2 + ⎟ +a=0
⎝ 3 3⎠ 3⎠ dx 2 −2
1. = (− cosec2 t) =
dt cot t sin t cos t
That is,
12 + 1+ 4 3 − 24 − 4 3 + a = 0 ⇒ a = 11 dy sin2 t − cos2 t
= sec2 t − cosec2 t =
dt sin2 t cos2 t
157. From mean value theorem, Now,
f (b ) − f ( a ) −2
at ⎛⎜ t = ⎞⎟ =
f ′( c ) =
dx p
=−4
b−a dt ⎝ 4⎠ 1/ 2
Chapter 21 | Applications of Derivatives 957
dy ⎛ p⎞ ⎧ a + 1⎫
at ⎜ t = ⎟ = 2 − 2 = 0 ⇒ (a + 1)2 ⎨3 – ⎬ =0
dt ⎝ 4⎠ ⎩ a ⎭
dy dx y
Here, =0⇒− =∞
dx dy
P(α , β )
2. g(x) is increasing and f(x) is decreasing. So,
g(x + 1) > g(x − 1) and f(x + 1) < f(x − 1)
x
⇒ f {g(x + 1)} < f {g(x − 1)} and g {f(x + 1)} < g {f(x − 1)} −1 O (0, 0)
2x2 = a2 ⇒ x = ±
a f(−x) = − f(x)
2 ⇒ −ax3 − bx2 − cx − d = −ax3 + bx2 − cx + d
In x It gives b = 0 = d
5. f(x) = (1)
x
f(x) = ax3 + cx = x (ax2 + c)
Since domain is R+, we have
Therefore,
1
x × - In x f ′(x) = 3ax2 + c = 0
x 1- In x
f ′(x) = = c
x2 x2 Only when x2 = − is positive.
3a
(A) For horizontal tangent
Therefore, c and a are of different signs.
f ′(x) = 0 ⇒ In x = 1 ⇒ x = e (True)
(B) If Eq. (1) cuts the x−axis, then c
Let − = k.
In x a
=0⇒x=1 (True)
x
(C) f ′(x) is +ve if x ∈ (0, e) and f ′(x) is −ve if x ∈ (e, ∞). Therefore,
f(x) is not monotonic.
0 k k
Hence, f(x) is many−one. (True)
(D) For vertical tangent f ′(x) = ∞, so 3
1- In x x2 Figure 21.40
2 =∞⇒ =0 ⇒x=0
x 1- In x So, non−zero root of f(x) is ± k .
which is not in the domain of f(x). (False)
6. See Fig. 21.39. k
Also ± is closer to origin than ± k .
Note: Curve is not passing through origin. 3
Let (a, b ) be the point of contact. Then
8. f(x) = (x − 1)4 (x − 2)n , n ∈ N (1)
æ dy ö b –0 b
ç ÷ = a – 0 ⇒ 3(a + 1) = a
2 (1) Therefore,
è dx øP
Also (a, b ) lies on the curve. Therefore, f ′(x) = 4 (x − 1)3 (x − 2)n + (x − 1)4 n (x − 2)n − 1
(a + 1)3 = (x − 1)3 (x − 2)n − 1 (4x − 8 + nx − n)
3(a + 1)2 =
a = (x − 1)3 (x − 2)n − 1 [(n + 4) x − (n + 8)]
958 Mathematics Problem Book for JEE
If n is odd, then f ′(x) > 0 if x < 1 and sufficiently close to 1 and 12.
f ′(x) < 0 if x > 1 and sufficiently close to 1. Therefore, x = 1 is
y
point of local maximum.
y=x
Similarly, if n is even, then x = 1 is a point of local minimum.
Further if n is even, then f ′(x) < 0 for x < 2 and sufficiently close
to 2 and f ′(x) > 0 for x > 2 and sufficiently close to 2. x
(0, 0)
Therefore, x = 2 is a point of local minimum. y = ke x
da
9. = 2 ⇒ a = 2t + c Figure 21.41
dt
With the help of graph shown in Fig. 21.41, we can see that
Since c = 0 {Because a = 0, when t = 0}
there is only one solution.
Therefore, 13. Consider y = kex and y = x.
a = 2t Let (a, kea ) be a point on y = kex.
Therefore, the curve y = x − 2ax + a2 + a becomes
2 If it lies on y = x also, then α = kea.
Now,
y = x2 − 4tx + 4t2 + 2t dy
= kex
If x = 0, then dx
Therefore,
y = 4t2 + 2t
dy
= kea = a = 1
Now, dx
x =α
dy
= 2x − 4t y = x is tangent to y = kex at one point.
dx Therefore,
1 = ke
Therefore, That is,
dy
= − 4t k = 1/e
dx at x = 0 14. See Fig. 21.42. Consider y = kex and y = x.
x
Therefore, equation of the tangent is From the above question, ex =
k
y − (4t2 + 2t) = − 4t (x − 0) 1 x
If we decrease the value of k from , then slope of y =
That is, e k
increases.
y = −4t x + 4t2 + 2t x
Therefore, y = ex and y = intersect at two distinct points
Vertex of y = x2 − 4t x + 4t2 + 2t is (2t, 2t). k
(See Fig. 21.42).
Therefore, distance of vertex from the origin = 2 2 t. y
16. Let f(x) = tan−1x. Then for some a ∈ (x, y), we have y
-1 -1
tan y - tan x
f ′(a) = (LMVT) B
y-x θ P (1, 2)
L 1
-1 -1
1 tan x - tan y æ 1 ö
⇒ = ç £ 1÷ 2
θ
1+ a 2 x-y è 1 + α 2
ø x
O M A
⇒ |tan−1x − tan−1y| ≤ |x − y|
Figure 21.43
17. Let f(x) = sinx and g(x) = cos x.
BP = sec q
π
Also, sin x ≠ 0 for x ∈ æç 0 , ö÷ AB = AP + BP = 2 cosec q + sec q
è 2ø
Therefore, minimum value of AB = (22/3 + 1)3/2.
Then, by Cauchy’s theorem, we have
20. Let y − 2 = m (x − 1) be a focal chord. Then
f ( b ) - f (a ) f ¢(q ) y = mx + 2 − m
=
g( b ) - g(a ) g¢(q ) ⇒ (mx + 2 − m) − 4x − 4 (mx + 2 − m) + 4 = 0
2
dy a ( m2 + 2)2
= +b 2
- 4 = 2 m 4 + 4 m2 + 4 - m 4
dx 2 x = 4
m4 m
dy a 1 4
= +b=− (2) |x2 − x1| = m2 + 1
dx (9, 0) 6 2 m2
4
Length of diagonal = ( x 2 - x1)2 + m2 ( x 2 - x1)2 = (m2 + 1)
Using Eqs. (1) and (2), we get m2
b = − 1 and a = 3 Length of diagonal of perpendicular chord is
Therefore,
⎛ 1 ⎞
y=3 x −x 4m2 ⎜ 2 + 1⎟ = 4 (1 + m2)
⎝m ⎠
Point (1, 2) lies on curve as well as it is point of intersection of
family of lines. ⎛ 2 1 ⎞
1. 4
dy 3 Area = (1 + m2) 4 (1 + m2) = 8 ⎜ m + 2 + 2 ⎟
= −1 2 m2 ⎝ m ⎠
dx 2 x
⇒ Minimum area = 16 + 8 × 2 = 32
dy 1
at (1, 2) is
dx 2 21. Graph of y = f(x). See Fig. 21.44.
1 (−2, f(−2))
y−2= (x − 1)
2 Y
y=3
⇒ x − 2y + 3 = 0
dy 4t
y = |x | (A) =
dx 3
4t
Tangent is y − at4 = (x − at3)
3
at 3 at 4
x-intercept = ; y-intercept = −
4 3
If P divides AB in the ratio l:1, we have
Figure 21.45 at 3
l ⋅0 +
Graph of f(x) = |x| has 3 points of intersection, so equation has at3 = 4 ⇒ l = −3
3 solutions (See Fig. 21.45.). Hence, (C) is the correct answer. l +1 4
y
P
(at 3, at 4)
B
x
Figure 21.46 O
⎛ at
4
⎞ ⎛ at 3
⎞
Hence, (B) is the correct answer. ⎜⎜0, − ⎟⎟ A ⎜⎜ 4
,0⎟
⎟
⎝ 3 ⎠ ⎝ ⎠
23. See Fig. 21.47. Figure 21.48
(−2, f(−2)) ⇒ m = 3, n = 4
⇒m+n=7
dx
(B) = esiny cos y; slope of normal = −1
dy
−2
Equation of normal is x + y = 1.
y = −3x 1
Area =
2
1 dy 2
Figure 21.47 (C) y = : = − 3 : slope of tangent = −2
x 2 dx x
Hence, (D) is the correct answer.
dy
24. (A) r = 5 cm, d dr = 0.06 y = e2−2x : = e2−2x⋅(−2) : slope of tangent = −2
dx
A = p r2dA = 2p rd r = 10p × 0.06 = 0.6p
Therefore,
(B) v = x3, d v = 3x2dx
tan q = 0
δv δx
× 100 = 3 × 100 = 3 × 1 = 3
v x
y be x /3
(D) Length of sub−tangent = = =3
dx dx y′ 1
b e x /3
(C) (x − 2) =2 ⇒x=4
dt dt 3
26. (A) Using LMVT, we get
3 2
(D) A = x g(5) − g(0) −1/ 6 − 4 5
4 g′(c) = = =−
5 5 6
Now,
(B) Let f(x) = f(x) − 2g(x), x ∈ [0, 1]. Then
dA 3 dx 3 1 3 3
= × = ⋅15⋅ =
dt 2 dt 2 10 4 f(0) = 2, f(1) = 6 − 2g(1)
Chapter 21 | Applications of Derivatives 961
⇒ g(1) = 2
(C) f(x) = sin 3x −2 −1
π π π
Clearly, longest length = e − ⎛⎜ − ⎞⎟ =
6 ⎝ 6⎠ 3 Figure 21.49
Therefore, l = 3. Therefore,
⎛ 1⎞⎛ 1⎞
f (5) − f (1) −c − 6 ⎜ 1+ ⎟ ⎜ 1+ ⎟ ≥ 1+ 8 = 3
(D) f ′(c) = ⇒ = ⇒ c2 = 15 ⎝ a⎠⎝ b⎠
5 −1 25 − c 2 2 (C) y = 10 − (10 − x) = x
Therefore, the maximum value is y = 3.
27. (A) f ( x ) = ln(sin x )
(D) Equation of tangent at P is ty = x + t2.
cos x It intersects the line x = 0 at Q. Therefore, coordinates of Q
f ′(x) = >0
sin x are (0, t). Therefore,
Therefore, the required number of values of x is 0. 0 t 1
1 1
Area of ΔPQS = 1 0 1 = [− t (1 − t2) + 2t]
(B) f ′(x) = 3x2 − 3 ≤ 0 if −1 ≤ x ≤ 1 2 2
Therefore, a = −1, b = 1 t2 2t 1
1
Therefore, a + b = 0 = (t + t3)
2
Now,
⎧
⎪ x 2 + 2, 1 ≤ x < 2 dA 1 2
⎪ 2 = (3t + 1) > 0 ∀ t ∈ [0, 2]
dt 2
x +2
(C) f(x) = ⎪⎨ , 2≤ x <3 Therefore, area is maximum for t = 2.
⎪ 2
⎪ x2 + 2 Hence,
⎪ , x =3 1
⎩ 3 maximum area = [2 + 8] = 5
2
⎧2 x , 1 < x < 2
⇒ f ′(x) = ⎨ 29. For the points of intersection, we have
⎩x, 2 < x < 3
12 − y 2 y2
Therefore, least value of f(x) in [1, 2) is 3. + =1
36 4
Least value of f(x) in [2, 3) is 3. So,
⇒ y = ± 3 and x = ±3
11
f(3) = Consider the point P (3, 3 ). Equation of the tangent at P to
3
Therefore, the least value of f(x) is 3. the circle is 3x + 3 y = 12.
(D) f(x) = e2x − (a + 1) ex + 2x Therefore, slope of this tangent is − 3 .
f ′(x) = 2e2x − (a + 1) ex + 2 Equation of the tangent at P to the ellipse is
Now, x 3
+ y=1
2e2x − (a + 1) ex + 2 ≥ 0 for all x ∈ R 12 4
⎛ 1⎞
⇒ 2 ⎜ e x + x ⎟ − (a + 1) ≥ 0 for all x ∈ R 1
⎝ e ⎠ Therefore, slope of this tangent is − .
3 3
⇒ 4 − (a + 1) ∈ 0
⇒a∈3 If a is angle between these tangents, then
⇒a=3 2
tan a =
28. (A) See Fig. 21.49. By graph, it is clear that at x = −1 is local 3
max. and x = 0 is local min. Therefore,
(B) a + b = 1 2
a = tan−1
3
⎛ 1⎞⎛ 1⎞ 1 1 1 2
⎜ 1+ ⎟ ⎜ 1+ ⎟ e = 1+ + + = 1+ Therefore,
⎝ a⎠⎝ b⎠ a b ab ab
k=4
a+b 1 Hence,
ab < =
2 2 k2 = 16
962 Mathematics Problem Book for JEE
(1, 1) 33. See Fig. 21.53. If a < amin, then the curve y = | x − a | will not
,
(3
L1 L2
(0, 3)
Figure 21.50
amin amax
d → shortest distance between two curves will be along
the common normal y = x
Therefore, Figure 21.53
d2 = 4 + 4 = 8 Case Ι:
31. See Fig. 21.51. L1 is tangent to y = − x2 + 3 and its equation is y = x − a
f ′(x) = cos x − 2a cos 2x − cos 3x + 2a ≥ 0 ∀ x ∈ R Therefore,
⇒ cos x − cos 3x + 2a (1 − cos 2x) ≥ 0 dy
= −2x = 1
⇒ 2 sin2x + sin x + 4a sin2x ≥ 0 dx
That is,
⇒ 2 sin2x (cos x + a) ≥ 0
1
⇒ a ≥ −cos x ⇒ a ≥ 1 x=−
2
⎛ 1 11 ⎞
Therefore, ⎜ − , ⎟ lies on y = x − a
h 3 ⎝ 2 4⎠
r ⎛ 1 11 ⎞
Thus, point of contact is ⎜ − , ⎟ .
Figure 21.51 ⎝ 2 4⎠
Now,
ïì e + axe , x £0
ax ax
⇒ f ′(x) = í h(x) = 0 and f(x) = 0
ïî1+ 2ax - 3 x , x > 0
2
or f ′(x) = 0
and As f(x) = 0 has 4 minimum solutions and f ′(x) = 0 has minimum
ïì(2a)e + a xe , x £ 0
ax 2 ax 3 solutions, h(x) = 0 has minimum 7 solutions and h′(x) = g(x) =
f ′′ (x) = í 0 has minimum 6 solutions.
ïî 2a - 6 x , x >0
37. Given
For x < 0, y = −p2x2 + 5 px − 4 (1)
f ′′(x) > 0 1
y= (2)
⇒ (2a + a2x) eax > 0 (1− x )
−2 Chord touches curve (2) at x = 2 which gives y = −1.
⇒ a (2 + ax) > 0 ⇒ x > e
a Let (x1, y1) and (x2, y2) are ends of chord.
Therefore, 1
y=
d2 y dy x x3
( x 2 − 1) 2 = −5 x 2 − 1 + 5 y
dx dx 2
x −1 When we substitute the values from the given options, only option
2 (C) satisfies the above equation.
d y dy
⇒ ( x 2 − 1) 2
= 25 y − x Hence, the correct answer is option (C).
dx dx
d2 y dy 7. The function f defined by f(x) = x3 – 3x2 + 5x + 7 is
⇒ ( x 2 − 1) + 1x − 25 y = 0 (A) decreasing in R.
dx 2 dx
(B) increasing in R.
Therefore, l = 1, k = −25; hence, (C) decreasing in (0 , ∞ ) and increasing in ( −∞ , 0 ).
(D) increasing in (0 , ∞ ) and decreasing in ( −∞ , 0 ).
l + k = −24 (ONLINE)
Hence, the correct answer is option (B). Solution: The given function is
f(x) = x2 – 3x2 + 5x + 7
6. A tangent to the curve, y = f(x) at P(x, y) meets x-axis at A and
y-axis at B. If AP : BP = 1 : 3 and f(1) = 1, then the curve also f ′( x ) = 3 x 2 − 6 x + 5
passes through the point
The discriminant of the above quadratic equation is
⎛1 ⎞ ⎛1 ⎞
(A) ⎜ , 4⎟ (B) ⎜ , 24⎟ Δ = 36 – 4(3)(5) = 36 – 60 < 0
⎝2 ⎠ ⎝3 ⎠
⎛ 1⎞ ⎛ 1⎞ Therefore,
(C) ⎜ 2, ⎟
⎝ 8⎠
(D) ⎜ 3, ⎟
⎝ 28 ⎠ f ′( x ) > 0 ∀x ∈R +
Also,
(ONLINE)
f ′( x ) > 0 ∀x ∈R −
Solution: We have
Solution: It is given that (A) (I) (iii) (P) (B) (II) (iv) (Q)
(C) (III) (i) (R) (D) (II) (iii) (P)
f(x) = x + logex – xlogex, x ∈(0 , ∞ )
Solution: Following explanation of Question 1, we have the
d d 1 1 following conclusions:
⇒ f ′( x ) = f ( x ) = ( x + log x − x log x ) = 1+ − log x − x ⋅
dx dx x x • In Column 2, option (i) is false.
1 • In Column 3, options (P) and (R) are false.
f ′( x ) = − log x
x
Thus, from given options, only (III) (i) (R) combination is INCORRECT.
d d ⎛1 ⎞ −1 1
⇒ f ′′( x ) = f (x) = ⎜ − log x ⎟⎠ = 2 − Hence, the correct answer is option (C).
dx dx ⎝ x x x
• lim f ( x ) = lim ( x + log x − x log x ) = −∞ . Hence, option (ii) is 4. If f: R→R is a differentiable function such that f ′( x ) > 2f ( x ) for
x →∞ x →∞ all x ∈R, and f(0) = 1, then
correct. (A) f(x) is increasing in (0 , ∞ ) .
⎛1 ⎞ (B) f(x) is decreasing in (0 , ∞ ) .
• lim f ′( x ) = lim ⎜ − log x ⎟ = −∞. Hence, option (iii) is correct.
x →∞ x →∞ ⎝ x ⎠ (C) f(x) > e2x in (0 , ∞ ) .
⎛ −1 1 ⎞
• lim f ′′( x ) = lim ⎜ 2 − ⎟ = 0. Hence, option (iv) is correct. (D) f ′( x ) < e2 x in f ′( x ) < e2 x .
x →∞ x →∞ ⎝ x x⎠
Also, we have Solution: It is given that f: R→R is differentiable function such that
f ′( x ) > 2f ( x ) for all x ∈R and f(0) = 1.
⎛1 ⎞
lim f ′( x ) = lim+ ⎜ − log x ⎟ = ∞
x →0+ x →0 ⎝ x ⎠ f ′( x ) > 2f ( x ) ⇒ f ’ ( x ) − 2f ( x ) > 0
Thus, lim+ f ′( x ) = ∞ and lim f ′( x ) = −∞ implies f ′( x ) is decreas- Multiplying this with e−2x, we get
x →0 x →∞
d −2 x
ing function. e −2 x f ′( x ) − 2e −2 x f ( x ) > 0 ⇒ [e f ( x )] > 0
So, options (Q) and (S) are correct. Therefore, from the given dx
options, (II) (ii) (Q) is correct.
Therefore, e −2 x f ( x ) is an increasing function.
Hence, the correct answer is option (B).
Let e −2 x f ( x ) = g( x ) .
2. Which of the following options is the only CORRECT
combination? • for x = 0: e −20f (0 ) = 1 = g(0 ) .
(A) (I) (i) (R) (B) (II) (iii) (S) • for x > 0: g(x) > g(0).
(C) (III) (iv) (P) (D) (IV) (i) (S) That is,
1
Solution: Following equations in the solution of Question 1, we e −2 x f ( x ) > f (0 ) ⇒ e −2 x f ( x ) > 1. ⇒ f ( x ) >
have the following conclusions: e −2 x
Thus,
• In Column 2, option (i) is false and (ii) (iii) and (iv) are correct.
• In Column 3, options (Q) and (S) are correct and options (P) f ( x ) > e 2 x [in (0, ∞ ] (1)
and (R) are false.
It is given that f ′( x ) > 2f ( x ) . Now, using (1), we get
Thus, from the given options, only (II) (iii) (S) is correct.
Hence, the correct answer is option (B). f ′( x ) > 2 f ( x ) > 2 e 2 x
3. Which of the following options is the only INCORRECT Thus, f(x) is an increasing function [in (0, ∞].
combination? Hence, the correct answers are options (A) and (C).
22
<
Indefinite Integration
⎛p x⎞ x a2
16. ∫ sec x dx = ln sec x + tan x + c = lntan ⎜⎝ 4 + 2 ⎟⎠ + c 29. ∫ x 2 − a2 dx =
2
x 2 − a2 − ln x + x 2 − a2 + c
2
⎡ d ⎤
⎢⎣since, dx (ln(sec x + tan x )) = sec x ⎥⎦ x a2
30. ∫ x 2 + a2 dx =
2
x 2 + a2 + ln x + x 2 + a2 + c
2
⎛ x⎞
17. ∫ cosec x dx = ln cosec x − cot x + c = lntan ⎜⎝ 2 ⎟⎠ + c Key points:
1. The signum function has an anti-derivative on any interval
⎡ d ⎤ which does not contain the point x = 0, and does not possess
⎢⎣since, dx (ln(cosec x − cot x )) = cosec x ⎥⎦ an anti-derivative on any interval which contains the point.
2. The anti-derivative of every odd function is an even function
dx
18. ∫ = sin−1 x + c = − cos −1 x + c and vice versa.
1− x 2 sin x
Illustration 22.1 Evaluate ∫ dx.
⎡ d 1 ⎤ 1+ sin x
⎢since, (sin−1 x ) = ⎥ Solution:
⎢⎣ dx 1− x 2 ⎥⎦
sin x sin x (1− sin x ) sin x − sin2 x
dx −1
x + c = − cot −1 x + c
∫ 1+ sin x dx = ∫ (1+ sin x ) ⋅ (1− sin x ) dx = ∫ dx
19. ∫ 1+ x 2 = tan cos2 x
⎡d −1 1 d −1 1 ⎤ (
= ∫ sec x ⋅ tan x − tan2 x dx )
⎢⎣ dx (tan x ) = 1+ x 2 , dx (cot x ) = − 1+ x 2 ⎥⎦
= ∫ (1− sec2 x + sec x ⋅ tan x ) dx = x − tan x + sec x + c
dx
20. ∫ = sec −1 x + c = − cosec −1x + c
x x −12 ( x + 1)2
Illustration 22.2 Evaluate ∫ dx.
⎡d 1 d 1 ⎤ x ( x 2 + 1)
−1
⎢ (sec x ) = , (cosec −1x ) = − ⎥ Solution:
⎢⎣ dx x x 2 − 1 dx x x 2 − 1 ⎥⎦
( x + 1)2 ( x 2 + 2 x + 1)
22.2.2.1 Some Standard Results on Integration ∫ x ( x 2 + 1) dx = ∫
x ( x 2 + 1)
dx
dx x x x2 + 1
21. ∫ = sin−1 + c = − cos −1 + c =∫ dx + ∫
2x
dx
2
a −x 2 a a 2
x ( x + 1) x ( x 2 + 1)
⎡ d ⎛ −1 x ⎞ 1 ⎤
1 2
⎢since ⎜⎝ sin ⎟⎠ = 2 ⎥ = ∫ dx + ∫ 2 dx = ln x + 2 tan−1 x + c
⎢⎣ dx a a − x ⎥⎦
2
x ( x + 1)
dx 1 −1 x 1 x
22. ∫ a2 + x 2 = a ⋅ tan + c = − ⋅ cot −1 + c ax 3 + bx 2 + c
a a a Illustration 22.3 Evaluate ∫ dx.
x4
⎡ d ⎛ −1 x ⎞ a ⎤ Solution:
⎢since dx ⎜⎝ tan a ⎟⎠ = 2 ⎥
⎣ a + x2 ⎦ ax 3 + bx 2 + c ⎛a b c ⎞ b c
∫ x4
dx = ∫ ⎜ + 2 + 4 ⎟ dx = a ln x − − 3 + k
⎝x x x ⎠ x 3x
dx 1 x 1 x
23. ∫ = ⋅ sec −1 + c = − ⋅ cosec −1 + c
x⋅ x −a 2 a 2 a a a
1+ x + x + x 2
⎡ d ⎛ ⎤ Illustration 22.4 Evaluate ∫ dx.
−1 x ⎞ a x + 1+ x
⎢since ⎜⎝ sec ⎟⎠ = ⎥
Solution:
⎢⎣ dx a x ⋅ x − a ⎥⎦
2 2
dx 1 x 1 x −a 1+ x + x + x 2 x + 1( x + x + 1)
24. ∫ = − ⋅ cot h−1 + c = ⋅ ln + c, x > a ∫ dx = ∫ dx
2
x −a 2 a a 2a x+a x + x +1 x + x +1
dx 1 −1 x 1 a+ x 3
25. ∫ a2 − x 2 = − a ⋅ tan h a
+ c = ⋅ ln
2a a− x
+ c, x < a 2
= ∫ x + 1dx = ( x + 1) 2 + c
3
dx x
26. ∫ = ln x + x 2 − a2 + c = cos h−1 + c
2 2 a Illustration 22.5 Evaluate ∫ (sin4 x − cos 4 x ) dx .
x −a
dx x Solution:
27. ∫ = ln x + x 2 + a2 + c = sin h−1 + c
x 2 + a2 a
∫ (sin x − cos 4 x ) dx = ∫ (sin2 x − cos2 x )(sin2 x + cos2 x )dx
4
x 2 a2 x 1
28. ∫ a2 − x 2 dx = a − x 2 + sin−1 + c = ∫ (sin2 x − cos2 x ) dx = − ∫ cos 2 x dx = − sin 2 x + c
2 2 a 2
Chapter 22 | Indefinite Integration 971
2 x 52 x 52
= [( x + 1)3 / 2 − ( x )3 / 2 ] + c (A) (tan−1 x + cot −1 x ) + c (B) (tan−1 x − cot −1 x ) + c
3 52 52
p x 52 p p x 52 p
(C) + +c (D) + +c
Your Turn 1 104 2 52 2 Ans. (A)
sin x
1. ∫ sin( x − a ) dx = 9. ∫ 5sin x dx =
(A) 5cos x + c (B) −5cos x + c
(A) x cos a − sina lnsin( x − a ) + c
(C) 5sin x + c (D) −5sin x + c
(B) x cos a + sina lnsin( x − a ) + c Ans. (B)
972 Mathematics Problem Book for JEE
sin(ln x ) (2 x 3 m + 3 x 2 m + 6 x m ) m
I=∫ dx ⇒I = +c
x 6( m + 1)
x 1 1 f ′( x )
I=∫ dx = ∫ 2 dt 3. I = ∫ dx : In this case, we put f ( x ) = t and f ′( x )dx = dt . So,
x 4 + x2 + 1 2 t + t +1 f (x)
1 1
I= ∫ dt f ′( x ) 1
2 ⎛ 1⎞ 2 ⎛ 3 ⎞ 2 I=∫ dx = ∫ dt = ln(f ( x )) + c
f (x) t
⎜⎝ t + ⎟⎠ + ⎜ ⎟
2 ⎝ 2 ⎠
x3
⎛ ⎞ ⎛ ⎞ Illustration 22.17 Evaluate ∫ dx.
⎛ 1⎞ ⎛ 2 1⎞ 1+ x 4
⎜ ⎜t + ⎟ ⎟ ⎜⎜x + ⎟⎟
1 ⎝ 2⎠ ⎟ 1 ⎝ 2⎠ ⎟ Solution:
⇒I = tan−1 ⎜ +c = tan−1 ⎜ +c
3 ⎜ ⎛ 3⎞ ⎟ 3 ⎜ ⎛ 3⎞ ⎟ x3
⎜ ⎜ ⎟ ⎟ ⎜ ⎜ ⎟ ⎟ I=∫ dx
⎝ ⎝ 2 ⎠ ⎠ ⎝ ⎝ 2 ⎠ ⎠ 1+ x 4
Chapter 22 | Indefinite Integration 973
1 1 x = a tanq or
Illustration 22.19 Evaluate ∫ dx . (b) a2 + x 2 , , a2 + x 2
1+ e x a2 + x 2 a cotq
Solution:
1 x = a secq or
1 e− x x 2 − a2 , , x 2 − a2
I=∫ dx = ∫ dx (c)
x − a2
2
1+ e x
1+ e −x a cosecq
(ln x )5 x x −a
Illustration 22.20 Evaluate ∫ x
dx.
(f) x −a
,
x
,
x = a sec2 q or
Solution: 1 a cosec2q
x ( x − a) ,
(ln x )5 x ( x − a)
I=∫ dx
x
(g) a+ x a− x x = a cos2q
dx ,
Put ln x = t . Then = dt a− x a+ x
x
x −a
t6 (ln x )6 , ( x − a )( b − x ), x = b sin2 q +
⇒I = ∫ t 5dt = +c = +c (h) b −x
6 6 a cos2 q
(b > a )
Illustration 22.21 Evaluate ∫ sin10 x ⋅ cosx dx.
dx
Illustration 22.23 Evaluate ∫ .
Solution: (1+ x ) x − x 2
I = ∫ sin10 x ⋅ cosx dx Solution: Let x = sin2 q . Then
( ) dx.
3
Illustration 22.30 Evaluate ∫ x 2 + 22 + x (C)
1
3
1
lnsin 3 x − lnsin 5 x + c
5
Solution: (D) 3 lnsin 3 x − 5 lnsin 5 x + c Ans. (C)
( ) dx
3
∫
2 2
I= x +2 + x
ex
∫
( )
6. dx =
Put x 2 + 22 + x = t . Then 1− e 2 x
( t 2 + 22 ) ⋅ t 3 1 3 2 1 ⎛ t4 t2 ⎞
I= ∫ dt = ∫ (t + 2 t )dt = ⎜ + 22 ⎟ + c (A) sina x + c (B) a x sin a x + c
2 2 2⎝ 4 2⎠
2t
sin(a x )
( ) + 2(
⎛ 4 ⎞ (C) +c (D) lnsina x + c Ans. (C)
2
x +2 + x 2
ln2 a
1⎜
)
2⎟
2 2
= ⎜ x +2 + x ⎟ +c
2⎜ 4 ⎟ sin x
⎝⎜ ⎟⎠ 8. ∫ (a + b cos x )2 dx =
Your Turn 2 (A)
1
(a + b cos x ) + c (B)
1
+c
b b(a + b cos x )
dx
1. ∫ = (C)
1
ln(a + b cos x ) + c (D) None of these Ans. (B)
x 1− (ln x )2 b
2.
f ′( x )dx
∫ (f ( x ))2 = (
(A) ln tan x + tan2 x + 4 + c )
(A) −(f ( x ))−1 + c (B) ln(f ( x )) + c (B)
1
2 (
ln tan x + tan2 x + 4 + c )
f(x)
(C) e +c (D) None of these Ans. (A) ⎛1 1 ⎞
(C) ln ⎜⎝ tan x + tan2 x + 4 ⎟ + c
2 2 2 ⎠
3. For which of the following functions, the substitution x = t is
applicable? (D) None of these Ans. (A)
976 Mathematics Problem Book for JEE
If F and G are two functions of x, then integral of the product of Illustration 22.35 Evaluate ∫ x 2 + a2 dx.
these two functions is given by
⎛ dF ⎞ Solution:
∫ F ⋅ G dx = F ∫ G dx − ∫ ⎜⎝ dx ∫ G dx ⎟⎠ dx 2x2
I = ∫ x 2 + a2 dx = x 2 + a2 ∫ 1dx − ∫ dx
Or we can say that the integral of the product of two functions 2 x 2 + a2
= (First function) × (Integral of second function) − Integral of x 2 + a2 a2
= x x 2 + a2 − ∫ dx + ∫ dx
{(Differentiation of first function) × (Integral of second function)}. x 2 + a2 x 2 + a2
Note: In applying the above rule, care has to be taken in the selec-
tion of the first function (F ) and the second function (G). ⇒ I = x x 2 + a2 − I + a2 In x + x 2 + a2 + c
Normally, we use the following methods:
1. In the product of two functions, one of the function is not ⇒ 2I = x x 2 + a2 + a2 In x + x 2 + a2 + c
directly integrable (that is, ln x , sin−1 x , cos −1 x , tan−1 x , etc.),
x 2 a2
then we take it as the first function and the remaining function ⇒I = x + a2 + In x + x 2 + a2 + c
2 2
is taken as the second function.
2. If there is no other function, then unity is taken as the
second function. For example, in the integration of sin−1 x − cos −1 x
−1 Illustration 22.36 Evaluate ∫ dx.
∫ ln x dx , ∫ sin x dx , 1 is taken as the second function. sin−1 x + cos −1 x
3. If both of the functions are directly integrable, then the first Solution:
function is chosen in such a way that the derivative of the func-
tion thus obtained under integral sign is easily integrable.
sin−1 x − cos −1 x
In the above stated order, the function on the left is always cho- I=∫
sin −1
x + cos −1
x
dx =
2
p∫
(
sin−1 x − cos −1 x dx )
sen as the first function. This rule is known as ILATE (Inverse,
Logarithmic, Algebraic, Trigonometric, Exponential). ⎛ −1 p⎞
⎜⎝ sin x + cos −1 x = ⎟
For example, in the integration of ∫ x sin x dx , x is taken as the 2⎠
first function and sin x is taken as the second function.
For first expression, ∫ sin−1 x dx
Illustration 22.31 Evaluate ∫ sec3 q dq.
Put x = sin2 q . Then
Solution: 1− 2 x = cos 2q ⇒ dx = sin 2q dq
I = ∫ sec3 q dq = secq ∫ sec2 q dq − ∫ tanq (secq tanq ) dq q cos 2q 1
∫ q sin 2q dq = − 2
+ ∫ cos 2q dq
2
= secq ⋅ tanq − ∫ secq (sec q − 1) dq
2
q cos 2q sin 2q
=− + +c
= secq ⋅ tanq − ∫ sec3 q dq + ∫ secq dq 2 4
(2 x − 1) −1 1
⇒ I = secq ⋅ tanq − I + ∫ secq dq = sin x+ x (1− x ) + c
2 2
1 1
⇒ I = [secq . tanq ] + ln| secq + tanq | + c
2 2 For second expression, ∫ cos −1 x dx
Therefore, ⎛ 1 2x ⎞
= ∫ ex ⎜ − ⎟ dx
2
(
I = (2 x − 1)sin−1 x + x (1− x ) − x + a
p
) ⎝ (1+ x 2 ) (1
1+ x 2 ) 2 ⎠
ex
Illustration 22.37 Evaluate ∫ x 3 ln x dx. ⇒I = +c
(1+ x 2 )
−1
Solution: e2 tan x
(1+ x )2
I = ∫ x 3 ln x dx =
x4
⋅ln x − ∫
x4 1
⋅ dx
Illustration 22.43 Evaluate ∫ 1+ x 2
dx.
4 4 x
x4 x4 Solution: Put tan−1 x = t . Then
= ⋅ ln x − +c
4 16 1
dx = dt
x 2dx 1+ x 2
Illustration 22.38 Evaluate ∫ 2
.
( x sin x + cos x ) −1
e2 tan x
(1+ x )2
Solution: I=∫ dx = ∫ e2t (1+ tan t )2 dt = ∫ e2t (sec2 t + 2 tan t ) dt
1+ x 2
x2 x cos x x
I=∫ dx = ∫ ⋅ dx e 2t e 2t
( x sin x + cos x )2 ( x sin x + cos x )2 cos x ⇒ I = ∫ e 2t sec2 tdt + 2 tan t ⋅ − 2∫ sec2 tdt
−1 x 1 cos x + x sin x 2 2
( x sin x + cos x ) cos x ∫ ( x sin x + cos x )
I= ⋅ + ⋅ dx
cos2 x
I = ∫ e2t sec2 tdt + tan t ⋅ e2t − ∫ e2t sec2 tdt + c
−1 x
( x sin x + cos x ) cos x ∫
I= ⋅ + sec2 x dx −1
I = tan t ⋅ e2t + c ⇒ I = x ⋅ e2 tan x
+c
−1 x
I= ⋅ + tan x + c æ x +3 ö x
( x sin x + cos x ) cos x Illustration 22.44 Evaluate ò ç 2÷
e dx .
è ( x + 4) ø
22.3.2.1 Some Important Results Solution:
∫e
x
1. (f ( x ) + f ′( x ))dx = e x f ( x ) + c ⎛ x +3 ⎞ x ⎛ x + 4 − 1⎞ x
I = ∫⎜ e dx = ∫ ⎜ e dx
⎝ ( x + 4 )2 ⎟⎠ ⎝ ( x + 4 )2 ⎟⎠
Illustration 22.39 Evaluate ∫ e (1− cot x + cot x ) dx. x 2
⎛ x+4 ⎞ x ⎛ 1 ⎞ x
= ∫⎜ e dx − ∫ ⎜ e dx
Solution: ⎝ ( x + 4 )2 ⎟⎠ ⎝ ( x + 4 )2 ⎟⎠
∫e (1− cot x + cot2 x )dx = ∫ e x ( − cot x + cosec2 x )dx = − e x cot x + c
x
ex ex
I=∫ dx − ∫ dx
æ 1- sin x ö ( x + 4) ( x + 4 )2
Illustration 22.40 Evaluate ò e x ç ÷ dx .
è 1- cos x ø ⎛ 1 ⎞ x ⎛ 1 ⎞ x ⎛ 1 ⎞ x
I=⎜ e +∫⎜ e dx − ∫ ⎜ e dx + c
Solution: ⎝ ( x + 4 ) ⎟⎠ ⎝ ( x + 4 )2 ⎟⎠ ⎝ ( x + 4 )2 ⎟⎠
x x
2 sin cos ⎛ 1 ⎞ x
⎛ 1− sin x ⎞ 1 2 2 x x
I=⎜ e +c
⎟= − = cosec2 − cot
⎜⎝
1− cos x ⎠ 2 sin2 x 2 x 2 2 ⎝ ( x + 4 ) ⎟⎠
2 sin
2 2
⎛ 1− sin x ⎞ x⎛ 2 x x⎞ x 2. ∫ (f ( x ) + xf ′( x ))dx = xf ( x ) + c
∫e ⎟ dx = ∫ e ⎜⎝ cosec − cot ⎟⎠ dx = − e cot + c
x x
⎜⎝
1− cos x ⎠ 2 2 2
Illustration 22.45 Evaluate ∫ ( x cos x + sin x )dx.
⎛1 1⎞
Illustration 22.41 Evaluate ∫ e x ⎜ − 2 ⎟ dx.
⎝x x ⎠ Solution:
Solution:
I = ∫ ( x cos x + sin x )dx = x sin x + c
⎛1 1⎞ ex
I = ∫ e x ⎜ − 2 ⎟ dx ⇒ I = +c
⎝x x ⎠ x ⎛ x + sin x ⎞
Illustration 22.46 Evaluate ∫ ⎜ dx .
2 ⎝ 1+ cos x ⎟⎠
⎛ 1− x ⎞
Illustration 22.42 Evaluate ∫ e x ⎜ dx. Solution:
⎝ 1+ x 2 ⎟⎠
⎛ x + sin x ⎞ 1 ⎛ x⎞ x
Solution: I = ∫⎜ dx = ∫ ⎜ x sec2 ⎟ dx + ∫ tan dx
⎝ 1+ cos x ⎟⎠ 2 ⎝ 2 ⎠ 2
2
⎛ 1− x ⎞
I = ∫ ex ⎜ dx x
⎝ 1+ x 2 ⎟⎠ 1 x tan 2 x x x
I= − ∫ tan dx + ∫ tan dx + c = x tan + c
(1+ x 2 − 2 x ) 2 1 2 2 2
= ∫ ex dx
(1+ x 2 )2 2
978 Mathematics Problem Book for JEE
= − e ax
cos bx a ⎛ ax sin bx a ax
+ ⎜e
⎞
− ∫ e sin bx dx ⎟
2. ∫ sin(ln x ) dx =
b b⎝ b b ⎠
1
2 (A) x (cos(ln x ) − sin(ln x )) + c
cos bx a ax ⎛ a⎞
∫e 2
ax ax
= −e + 2 e sin bx − ⎜ ⎟ sin bx dx
b b ⎝ b⎠ (B) cos(ln x ) − x + c
2 1
cos bx a ax ⎛ a⎞ x (sin(ln x ) − cos(ln x )) + c
= − e ax + 2 e sin bx − ⎜ ⎟ I (C)
b b ⎝ b⎠ 2
2 ax
(D) − cos(ln x ) + c Ans. (C)
⎛ a⎞ e
⇒ I + ⎜ ⎟ I = 2 ( −b cos bx + a sin bx ) + c
⎝ b⎠ b 3. If ∫ x sin x dx = − x cos x + A, then A =
ax
e
⇒I = ( −b cos bx + a sin bx ) + c (A) sin x + c (B) cos x + c
a2 + b 2 (C) Constant (D) None of these Ans. (A)
Thus,
e ax 4. ∫ x ln x dx =
∫e
ax
sin bx dx = 2 2
( −b cos bx + a sin bx ) + c
a +b
x2 x2 x2 x2
e ax ⎛ b⎞ (A) ln x − +c (B) ln x − +c
= sin ⎜ bx − tan−1 ⎟ + c 2 2 2 4
a2 + b 2 ⎝ a⎠
x2 x2
Similarly, (C) ln x + +c (D) None of these Ans. (B)
2 2
e ax
∫e
ax
cos bx dx = (b sin bx + a cos bx ) + c
a2 + b 2 5. ∫ x cos dx =
e ax⎛ b⎞
= cos ⎜ bx − tan−1 ⎟ + c (A) x sin x + cos x + c (B) x sin x − cos x + c
2
a +b 2 ⎝ a⎠
(C) x cos x + sin x + c (D) x cos x − sin x + c Ans. (A)
Illustration 22.47 Evaluate ∫ e sin x dx. x
∫ x cos
2
6. x dx =
Solution:
ex ex x2 1 1
− x sin 2 x − cos 2 x + c
∫e
x
sin x dx = (1⋅ sin x − 1⋅ cos x ) + c = (sin x − cos x ) + c (A)
12 + 12 2 4 4 8
x2 1 1
Illustration 22.48 If u = ∫ e ax cos bx dx and v = ∫ e ax sin bx dx , (B) + x sin 2 x + cos 2 x + c
4 4 8
2 2 2 2
then find (a + b )(u + v ). x2 1 1
(C) − x sin 2 x + cos 2 x + c
Solution: 4 4 8
sin bx ⎛ sin bx ⎞ sin bx a x2 1 1
u = ∫ e ax cos bx dx = e ax − ∫ ae ax ⎜ dx = e ax − v + x sin 2 x − cos 2 x + c
⎝ b ⎟⎠
(D) Ans. (B)
b b b 4 4 8
Chapter 22 | Indefinite Integration 979
−1 A x + B1 A2 x + B2
7. ∫ tan x dx = U( x )
=
P1
+
P2
+…+
Pk
+ 1 +
V ( x ) ( x − a ) ( x − a )2 ( x − a )k x 2 + ax + b ( x 2 + ax + b )2
1
(A) x tan−1 x + (ln(1+ x 2 )) + c A x + Br
2 +…+ 2 r +…
( x + ax + b )r
1
(B) x tan−1 x − (ln(1+ x 2 )) + c where P1, P2, …, Pk, A1, A2, …, Ar, B1, B2, …, Br are real constants
2
to be determined. Reducing both sides of the above identity to
(C) ( x − 1)tan−1 x + c the integral form and equating the coefficients of equal powers
of x, which gives a system of linear equations in the coefficient.
(D) x tan−1 x − x ln(1+ x 2 ) + c Ans. (B)
Determine these coefficient. (This method is called the method of
−1 comparison of coefficients.) The constants can also be obtained by
8. ∫ x tan x dx =
substituting suitably chosen numerical values of x in both sides of
the identity.
(1+ x 2 ) x
(A) tan−1 x − + c Key point:
2 2
Before proceeding to write a rational function as a sum of partial
( x 2 − 1) x
(B) tan−1 x − + c fractions, we should ascertain that it is either a proper rational frac-
2 2 tion or is rewritten as one.
( x 2 + 1) x
(C) tan−1 x + + c A rational function U(x)/ V(x) is proper if the degree of polyno-
2 2 mial V(x) is greater than the degree of the polynomial U(x). In case
( x 2 + 1) the degree of U(x) is greater than or equal to the degree of V(x), we
(D) tan−1 x − x + c Ans. (A) U( x ) u( x )
2 first write = h( x ) + , where h(x) is a polynomial and u(x) is
V(x) v( x )
⎛ ⎞ a polynomial of degree less than the degree of polynomial V(x).
1
9. ∫ ⎜⎜⎝ ln(ln x ) + (ln x )2 ⎟⎟⎠ dx = cos x dx
Illustration 22.49 Evaluate ∫ .
(1+ sin x )(2 + sin x )
x
(A) x ln(ln x ) + +c
(ln x ) Solution: Put sin x = t . Then
x
(B) x ln(ln x ) − +c cos x dx = dt
(ln x )
dt dt dt
(C) x ln(ln x ) +
ln x
+c ∫ (1+ t )(2 + t ) = ∫ (1+ t ) − ∫ (2 + t ) = ln(1+ t ) − ln(2 + t ) + c
x
ln x (1+ sin x )
(D) x ln(ln x ) − +c Ans. (B) = ln +c
x (2 + sin x )
Hence, ìïæ b ö 4 ac - b2 üï
2
1 dt 5 dt dt Convert ax 2 + bx + c = a íç x + ÷ + ý
I= ò + ò -ò 4 a2 þï
3 t 3 t -3 t -1 îïè 2a ø
3x + 1
=
A
+
B
+
C ∫ ax 2 + bx + c dx .
2
( x − 2) ( x + 2) ( x − 2) ( x − 2)2 ( x + 2)
1
3 x + 1 = A( x − 2)( x + 2) + B( x + 2) + C ( x − 2)2 (1)
Illustration 22.53 Evaluate ò x 2 + x + 1dx.
Putting x = 2 and −2 successively in Eq. (1), we get Solution:
1 1
B=
7
,C=−
5 I=∫ dx = ∫ dx
4 16 x2 + x + 1 ⎛ 1⎞ ⎛ 3 ⎞
2 2
⎜⎝ x + ⎟⎠ + ⎜ ⎟
5 2 ⎝ 2 ⎠
Now, we put x = 0 and get A= . Therefore,
16
2 æ 2 x + 1ö
3x + 1 = tan-1 ç ÷+c
I= ò dx 3 è 3 ø
( x -2)2 ( x + 2)
5 7 5 1
I= ò dx + ò dx -ò dx Illustration 22.54 Evaluate ò dx.
16( x -2) 4( x -2)2 16( x + 2) 2
2x + 3x + 2
5 7 5 Solution:
I= ln x − 2 − − ln x + 2 + c
16 4( x − 2) 16 1 1 1
I=∫
2∫
dx = dx
5 x −2 7 2
2x + 3x + 2 2
3⎞ ⎛ 7 ⎞
2
I = ln − +c ⎛
16 x + 2 4( x − 2) ⎜⎝ x + ⎟⎠ + ⎜ ⎟
4 ⎝ 4 ⎠
2x2 + 3 æ 2ö
∫ ( x 2 − 1)( x 2 + 4 ) dx .
2
Illustration 22.52 Evaluate 1 çæ 3ö æ 3ö æ 7ö ÷
= ln ç ç x + ÷ + ç x + ÷ + çç ÷ +c
2 çè 4ø è 4 ø è 4 ÷ø ÷÷
Solution: è ø
2x2 + 3 A B
2 2
= 2
+ 2 Illustration 22.55 Evaluate ò x 2 - x dx .
( x − 1)( x + 4 ) ( x − 1) ( x + 4 )
Therefore, Solution:
2 2 2
2 x + 3 = A( x + 4 ) + B( x − 1) æ 1ö æ 1ö
2 2
I = ò x 2 - x dx = ò ç x - ÷ - ç ÷ dx
Comparing the coefficient of x2 and constant terms è 2ø è2ø
æ 1ö
⇒ A + B = 2, 4 A − B = 3 çx- ÷
è 2ø 2 1 ææ 1ö ö
I= x - x - ln ç ç x - ÷ + x 2 - x ÷ + c
⇒ A = 1, B = 1 2 8 èè 2ø ø
1 1
I=∫ 2
dx + ∫ 2
dx
( x − 1) ( x + 4) 1
Illustration 22.56 Evaluate ò 2 x 2 + x + 1dx.
1 x -1 1 x
Þ I = ln + tan-1 + c Solution:
2 x +1 2 2
1 1 1
I=∫
2∫⎛
Integrals of the form: dx = dx
2 2
dx dx 2x + x +1 1⎞ ⎛ 7 ⎞
2
∫ ax 2 + bx + c , ∫ , ∫ ax + bx + c dx
2
⎜⎝ x + ⎟ +
4 ⎠ ⎜⎝ 4 ⎟⎠
1.
ax 2 + bx + c
Chapter 22 | Indefinite Integration 981
æ ö Put ( x + 4 ) = t . Then dx = dt .
ç æç x + 1 ö÷ ÷
ç 4ø÷
tan-1 ç è
1 4 t 2 2 4
I= × +c I = ∫ t 2 − 22 dt = t - 2 - ln t + t 2 - 22 + c
2 7 æ 7ö ÷
ç ç ÷ 2 2
ç ç 4 ÷÷ ÷
è è ø ø
( x + 4) 4
I= ( x + 4 )2 − 22 − ln ( x + 4 ) + ( x + 4 )2 − 22 + c
2 æ ( 4 x + 1) ö 2 2
I= tan-1 ç ÷+c
7 è 7 ø ( x + 4) 2
I= x + 8 x + 12 − 2 ln ( x + 4 ) + x 2 + 8 x + 12 + c
2
cos x
Illustration 22.57 Evaluate ò sin2 x + 4 sin x + 5 dx .
Solution:
Illustration 22.61 Evaluate ò 2ax - x 2 dx.
1 3
⇒ I = ln t + t 2 − ( 2 )2 + c Equating the coefficients, A = , B = . We get
4 4
x +1
⇒ I = ln ( x − 2) + x 2 − 4 x + 2 + c I=∫ dx
2
( 2 x + x − 3)
1 4x +1 3 1
Illustration 22.60 Evaluate ò x 2 + 8 x + 12 dx. I= ∫
4 ( 2 x 2 + x − 3)
dx + ∫
4 ( 2 x 2 + x − 3)
dx
Solution:
1 4x +1 3 1
4 ∫ ( 2 x 2 + x − 3)
Let I1 = dx and I2 = ∫ dx
I = ∫ x 2 + 8 x + 12 dx = ∫ ( x + 4 )2 − (2)2 dx 4 (2 x 2 + x − 3)
982 Mathematics Problem Book for JEE
1 1 1 1 1
I1 = ò dt = 2 t + k1 = 2 6 + x - x 2 + k1
4∫ t
I1 = dt = t + k1 = (2 x 2 + x − 3) + k1
2 2 t
3 1 3 1 And
2ò 2ò
I2 = dx = dx dx dx
4 æ 2 x 3ö 4 ææ 1ö æ 5ö ö
2 2 I2 = ò =
çx + - ÷ çç x + ÷ -ç ÷ ÷ 6 + x - x2 6 - ( x2 - x)
è 2 2ø çè 4 ø è 4 ø ÷ø
è
4 1
æ ææ 2 ö I2 = ∫ dx = ∫ dx
1ö æ 5ö ö
2
3æ 1ö ⎛ 1 1⎞
ln ç ç x + ÷ + ç ç x + ÷ - ç ÷ ÷ ÷ + k2
2
= 6 − (x − x) 6 − ⎜ x2 − x + − ⎟
4 2 çç è 4ø çè
è 4 ø è 4 ø ÷ø ÷÷ ⎝ 4 4⎠
è ø
1
Hence, =∫ dx
2
25 ⎛ 1⎞
1 3 ⎛⎛ 1⎞ ⎛ x 3⎞ ⎞ −⎜x − ⎟
I= (2 x 2 + x − 3) + ln ⎜ ⎜ x + ⎟ + ⎜ x 2 + − ⎟ ⎟ + c 4 ⎝ 2⎠
2 4 2 ⎝⎝ 4⎠ ⎝ 2 2⎠ ⎠
æ2æ 1 öö æ 2 x - 1ö
= sin-1 ç ç x - ÷ ÷ + k2 = sin-1 ç ÷ + k2
3x + 2 è 5è 2 øø è 5 ø
Illustration 22.63 Evaluate ∫ ( 4 x 2 + 4 x + 5) dx.
⎛ 2 x − 1⎞
I1 + 4 I2 = 2 6 + x − x 2 + 4 sin−1 ⎜ + k + 4 k2
Solution: Express ⎝ 5 ⎟⎠ 1
3x + 2 = A (differential coefficient of 4x2 + 4x + 5) + m ⎛ 2 x − 1⎞
⇒ I = 2 6 + x − x 2 + 4 sin−1 ⎜ +c
⇒3x + 2 = A (8x + 4) + m = 8Ax + 4A + m ⎝ 5 ⎟⎠
Comparing the coefficients, we get
x +2
8A = 3 and 4A + m = 2⇒ A = 3/8 and m = 2 − 4A = 1/2 Illustration 22.65 Evaluate ò 2
dx.
x - 2x + 4
3 8x + 4 1 1 Solution:
⇒I = ∫ 2
8 ( 4 x + 4 x + 5)
dx + ∫ 2
2 ( 4 x + 4 x + 5)
dx
x +2 x − 1+ 3
I=∫ dx = ∫ dx
3 1
= ln( 4 x 2 + 4 x + 5) + ∫
1
dx
x2 − 2x + 4 (x 2
− 2x + 4 )
8 8 ⎛ 2 5⎞
⎜⎝ x + x + ⎟⎠ x −1 3
4 I=∫ dx + ∫ dx
3 1 ⎛ 1⎞ (x 2
− 2x + 4 ) ( x − 1) + ( 3 )2
2
= ln( 4 x 2 + 4 x + 5) + tan−1 ⎜ x + ⎟ + c
8 8 ⎝ 2⎠
Put x 2 − 2 x + 4 = t 2 . Then
5 - 2x (2 x − 2)dx = 2tdt ⇒ ( x − 1) dx = tdt
Illustration 22.64 Evaluate ò 2
dx.
6+ x - x t 3
Solution: ⇒ I = ∫ dt + ∫ dx
t ( x − 1) + ( 3 )2
2
5 - 2x
I=ò dx
6 + x - x2 I = x 2 − 2 x + 4 + 3 ln x + x 2 − 2 x + 4 + c
d
Let 5 − 2 x = M (6 + x − x 2 ) + N = M(1− 2 x ) + N
dx Illustration 22.66 Evaluate ∫ (2 x + 3) x 2 + 4 x + 3 dx.
Equating the coefficients of x and constant terms on both sides,
we get d 2
Solution: Let (2 x + 3) = M ( x + 4 x + 3) + N . Then
M = 1, M + N = 5 ⇒ N = 4 dx
( 2 x + 3 ) = M( 2 x + 4 ) + N
Therefore,
5 - 2 x = 1- 2 x + 4 Equating the coefficients of x and constant terms on both sides,
we get
Hence, M = 1, N = 3 − 4 M = −1
(1− 2 x ) + 4 (1− 2 x ) 4 Therefore,
I=∫ dx = ∫ dx + ∫ dx = I1 + 4I2
6+ x − x 2
6+ x − x 2
6+ x − x 2 ( 2 x + 3) = ( 2 x + 4 ) − 1
Now, I = ∫ ((2 x + 4 ) − 1) x 2 + 4 x + 3 dx
(1− 2 x )
I1 = ∫ dx
6 + x − x2 I = ∫ (2 x + 4 ) x 2 + 4 x + 3 dx − ∫ x 2 + 4 x + 3 dx = I1 - I2
Chapter 22 | Indefinite Integration 983
Solution: Solution:
( x 3 + 8 )( x − 1) ( x − 1)( x 2 − 2 x + 4 )( x − 1)
I = ∫ (2 x − 5) x 2 − 4 x + 3 dx I=∫ dx = ∫ dx
x2 − 2x + 4 x2 − 2x + 4
d 2
Let (2 x − 5) = M ( x − 4 x + 3) + N . Then
dx x3 x2
I = ∫ ( x + 2)( x − 1) dx = ∫ ( x 2 + x − 2) dx = + − 2x + c
( 2 x − 5 ) = M( 2 x − 4 ) + N 3 2
Equating the coefficients of x and constant terms on both sides,
2x3 - 3x2 + 5x + 6
we get Illustration 22.69 Evaluate ò x2 + 3x + 2
dx.
M = 1, N = 4 M − 5 = −1 Solution:
Therefore, 2x3 - 3x2 + 5x + 6
I=ò dx
x2 + 3x + 2
(2 x − 5) = (2 x − 4) − 1
⎛ 8x ⎞
I = ∫ ((2 x − 4 ) − 1) x 2 − 4 x + 3 dx I = ∫ ⎜ ( 2 x + 3) − 2 ⎟ dx
⎝ x + 3x + 2⎠
I = ∫ (2 x − 4 ) x 2 − 4 x + 3 dx − ∫ x 2 − 4 x + 3 dx 2x + 3 1
I = ∫ (2 x + 3)dx − 4 ∫ dx + 12∫ dx
I = I1 - I2
2
x + 3x + 2 ( x + 1)( x + 2)
⎛ 1 1 ⎞
( x 2 − 4 x + 3)3 2 I = ( x 2 + 3 x ) − 4 ln x 2 + 3 x + 2 + 12∫ ⎜ − dx
I1 = ∫ (2 x − 4 ) x 2 − 4 x + 3 dx = + c1 ⎝ x + 1 x + 2 ⎟⎠
32
I = ( x 2 + 3 x ) - 4 ln ( x + 1)( x + 2) + 12 ln x + 1 - 12 ln x + 2 + c
I2 = ò x - 4 x + 3 dx = ò x - 4 x + 4 - 4 + 3 dx
2 2
I = ( x 2 + 3 x ) + 8 ln x + 1 - 16 ln x + 2 + c = x 2 - 3 x + 8 ln( x + 1)
= ∫ ( x − 2)2 − 12 dx - 16 ln( x + 2) + c
( x − 2) 2 1
= x − 4 x + 3 − ln ( x − 2) + x 2 − 4 x + 3 + c2
2 2
ax 2 + bx + c ax 2 + bx + c
Now, 4. ∫ px 2 + qx + r dx , ∫ dx
px 2 + qx + r
I = I1 - I2
In this case, substitute
( x 2 − 4 x + 3)3/2 ( x − 2) 2
⇒I = − x − 4x + 3 + d
3/ 2 2 ax 2 + bx + c = M( px 2 + qx + r ) + N ( px 2+ qx + r ) + R
dx
1
ln ( x − 2) + x 2 − 4 x + 3 + c Find M, N and R by comparing the coefficient of x2, x and constant
2
terms on both sides of the identity.
f (x)
3. ∫ ax 2 + bx + c dx where f ( x ) is a polynomial of degree greater x2 + x +1
Illustration 22.70 Evaluate ò x 2 + 3 x + 2 dx.
than 2.
984 Mathematics Problem Book for JEE
Solution:
x2 1 x 2 + a2 x 2 − a2
x2 + x +1
5. ∫ x4 +1 dx , ∫
x4 +1
dx , ∫
x 4 + kx 2 + 1
dx , ∫
x 4 + kx 2 + 1
dx ,(k ∈R )
I=ò dx
x2 + 3x + 2 Rule for this form:
d
x + x + 1 = M( x + 3 x + 2 ) + N ( x 2 + 3 x + 2 ) + R
2 2
(a) To evaluate these types of integrals divide the numerator
dx
and denominator by x2.
x 2 + x + 1= Mx 2 + (3M + 2N ) x + (2M + 3N + R )
1 1 a2 a2
Comparing coefficients of variables and constant terms (b) Put x + = t or x - = t and x + = t or x - = t as
x x x x
M = 1, (3M + 2N ) = 1,(2M + 3N + R ) = 1 required.
M = 1, N = −1, R = 2 dx
Similar form is: ∫ tan x dx , ∫ cot x dx , ∫ ,
d 2 sin x + cos 4 x
4
( x 2 + 3 x + 2) −
( x + 3 x + 2) + 2
I=∫ dx dx dx ( ± sin x ± cos x ) dx
x2 + 3x + 2 ∫ sin6 x + cos6 x , ∫ a + b sin x
2x + 3 2
= ∫ 1dx − ∫ 2 dx + ∫ dx
x + 3x + 2 ( x + 1)( x + 2) Illustration 22.72 Evaluate
5
∫ x 4 + 1dx.
x +1
I = x - ln x 2 + 3 x + 2 + ln +c Solution:
x +2
5 5 ⎛ ( x 2 + 1) − ( x 2 − 1) ⎞
I=∫
2 ⎝∫
x2 - x +1 dx = ⎜ dx ⎟
Illustration 22.71 Evaluate ò dx. x4 +1 x4 +1 ⎠
x2 + 4 x + 3
Solution: ⎛ ⎛ 1⎞ ⎛ 1⎞ ⎞
1+ ⎜⎝ 1− 2 ⎟⎠ ⎟
2
x - x +1 5 ⎜ ⎜⎝ x 2 ⎟⎠ x
I=ò dx = ⎜∫ dx − ∫ dx ⎟
2 ⎜ x2 + 1 2
+
1 ⎟
x2 + 4 x + 3 ⎜⎝ x
x2 x 2 ⎟⎠
d 2
x 2 − x + 1 = M( x 2 + 4 x + 3 ) + N ( x + 4 x + 3) + R ⎛ ⎞
dx ⎛ 1⎞ ⎛ 1⎞
⎜ ⎜ 1+ 2 ⎟ ⎜ 1− 2 ⎟ ⎟ 5
x 2 − x + 1 = Mx 2 + ( 4 M + 2N ) x + (3M + 4 N + R ) 5 ⎝ x ⎠ ⎝ x ⎠
= ⎜∫ dx − ∫ dx ⎟ = (I1 − I2 )
2⎜ ⎛ 1⎞
2
⎛ 1⎞
2 ⎟ 2
Comparing coefficients of variables and constant terms ⎜ ⎜x − ⎟ +2 x + ⎟ −2 ⎟
⎜
⎝
⎝ ⎝ x⎠ x⎠ ⎠
M = 1, ( 4 M + 2N ) = −1, (3M + 4 N + R ) = 1
5 ⎛ 1⎞
M = 1, N = − , R = 8 ⎜⎝ 1+ 2 ⎟⎠
2 I1 = ∫ x dx
2
5 d 2 ⎛ 1⎞
( x 2 + 4 x + 3) − ( x + 4 x + 3) + 8 ⎜⎝ x − ⎟ + 2
2 dx x⎠
I=∫ dx
( x 2 + 4 x + 3)
For I1, we write
5 (2 x + 4 ) 1 1 æ 1 ö
=∫ ( x + 4 x + 3) dx − ∫
2
dx + 8∫ dx x- = t Þ ç 1+ 2 ÷ dx = dt
2 ( x 2 + 4 x + 3) 2
( x + 4 x + 3) x è x ø
5 1
I = I1 - I2 + 8I3 I1 = ò dt
2 t2 + 2
I1 = ∫ x 2 + 4 x + 3 dx = ∫ ( x + 2)2 − 1 dx ⎛ 1⎞
1 t 1 ⎜⎝ x − ⎟⎠
( x + 2) 2 1 I1 = tan−1 +c = tan−1
x
+ c1
= x + 4 x + 3 − ln ( x + 2) + x 2 + 4 x + 3 + c1
2 2 2 2 2 2
(2 x + 4 )
I2 = ∫ dx = 2 x 2 + 4 x + 3 + c2 For I2, we write
x2 + 4 x + 3
1 æ 1 ö
I3 = ∫
1
dx = ln ( x + 2) + x 2 + 4 x + 3 + c3 x+ = t Þ ç 1- 2 ÷ dx = dt
x è x ø
( x + 2)2 − 1
5 1
I = I1 − I 2 + 8I3 1 1 t− 2 1 x+ − 2
2 I2 = ∫ 2 dt = ln +c = ln x + c2
( x − 8) 2 15 t −2 2 2 t+ 2 2 2 x+ 1+ 2
I= x + 4 x + 3 + ln ( x + 2) + x 2 + 4 x + 3 + c
2 2 x
Chapter 22 | Indefinite Integration 985
1 ⎛ 1⎞
For I2, we write t + = z ⇒ ⎜ 1− 2 ⎟ dx = dz (ax 2 + bx + c )dx
t ⎝ t ⎠ (e) ∫ , here, we write
(dx + e ) (fx 2 + gx + h)
1
t+ − 2
1 1 z− 2 1 t
I2 = ∫ 2 dt = ln +c = ln + c1 ax2 + bx + c = A1 (dx + e) (2fx + g) + B1(dx + e) + C1
z −2 2 2 z+ 2 2 2 t + 1+ 2 where A1, B1 and C1 are constants which can be obtained by
t comparing the coefficient of like terms on both sides.
1
tan x + − 2 x dx
I2 =
1
ln tan x + c2 Illustration 22.75 Evaluate ∫ .
2 2 1 ( x − 3) x + 1
tan x + + 2
tan x Solution: Put x + 1 = t 2 . Then dx = 2tdt , we get
Combining the two integrals, we get
2t (t 2 − 1)
æ æ 1 ö 1 ö I= ∫ (t 2 − 4 )t dt
ç ç tan x - ÷ tan x + - 2 ÷
I =ç
1
tan-1 è tan x ø + 1 ln tan x ÷+k ⎛ 3 ⎞ 3 t −2
ç 2 1 ÷ = 2 ∫ ⎜ 1+ 2 dt = 2t + ln +c
⎝ (t − 4 ) ⎟⎠
2 2 2 tan x + + 2 ÷
ç 2 t +2
è tan x ø
3 x +1− 2
= 2 x + 1 + ln +c
Illustration 22.74 Evaluate ∫ ( tan x + cot x ) dx. 2 x +1+ 2
Solution: dx
Illustration 22.76 Evaluate ∫ .
I = ∫ ( tan x + cot x ) dx ( x − 3) x + 1
986 Mathematics Problem Book for JEE
x +1− 2 ⎛ 1⎞
1
= ln +c I = 2 ( x 2 − x + 1) + 2 ln ⎜ x − ⎟ + ( x 2 − x + 1) −
⎝ 2⎠
2 x +1+ 2
dx 1 2 1
Illustration 22.77 Evaluate ∫ . ln ⎛ 1− x ⎞ + ⎛ 1− x ⎞ + +c
2
( x + 1) ( x − x + 1) 3 ⎜⎝ 2(1+ x ) ⎟⎠ ⎜⎝
2(1+ x )
⎟⎠ 12
Solution:
dx
I=∫
dx Illustration 22.79 Evaluate ∫ .
( x + 1) ( x 2 − x + 1) ( x + 2) x 2 + 1
2
1 1 Solution:
Put x + 1 = . Then dx = − dt , we get dx
t t2 I=∫
1 1 ( x + 2) x 2 + 1
2
I = −∫ dt = − ∫ dt
1 3t 2 − 3t + 1
t2. (1− t )2 − t (1− t ) + t 2 1 1
t2 Put x = . Then dx = − 2 dt , we get
t t
2
1 1 1 æ 1ö æ 1ö 1 dt tdt
3 ò æ 1 ö2 1
I=- dt = - ln ç t - ÷ + ç t - ÷ + +c I = −∫ = −∫
3 è 2ø è 2 ø 12 ⎛ 1 ⎞ 1 (1+ 2t 2 ) t 2 + 1
çt - ÷ + t 2 ⎜ 2 + 2⎟ 2 + 1
è 2 ø 12 ⎝t ⎠ t
1 ⎛ 1− x ⎞ ⎛ 1− x ⎞ 1
2 Put (1+ t 2 ) = z 2 . Then tdt = zdz , we get
I= − ln + ⎜ ⎟ + 12 + c
3 ⎜⎝ 2 (1+ x ) ⎟⎠ ⎝ (
2 1 + x ) ⎠ z dz dz
I = −∫ = −∫
⎡1+ 2( z 2 − 1)⎤ z (2 z 2 − 1)
⎣ ⎦
(2 x 2 + 3 x + 2)dx
Illustration 22.78 Evaluate
∫ z-
1
( x + 1) ( x 2 − x + 1) 1 dz 1 2 +c
Solution: I=- ò =- ln
2 æ 2 1ö 2 2 z+
1
çz - ÷
(2 x 2 + 3 x + 2)dx è 2ø 2
I=∫
( x + 1) ( x 2 − x + 1) 1 2z − 1 1 2(1+ t 2 ) − 1
I=− ln +c = − ln +c
d 2 2 2 2z + 1 2 2 2(1+ t 2 ) + 1
(2 x 2 + 3 x + 2) = a( x + 1) ( x − x + 1) + b( x + 1) + c
dx
a = 1, b = 2, c = 1 1 2(1+ x 2 ) − x
I=− ln +c
( x + 1)(2 x − 1) + 2( x + 1) + 1
2 2 2(1+ x 2 ) + x
I=∫ dx
( x + 1) ( x 2 − x + 1)
(2 x − 1) 1 1
I=∫
2
dx + 2∫
2
dx + ∫ dx Your Turn 4
( x − x + 1) ( x − x + 1) ( x + 1) ( x 2 − x + 1)
I = I1 + 2I2 + I3 dx
1. ∫ ( x − x2 ) =
(2 x − 1)
I1 = ∫ dx = 2 ( x 2 − x + 1) + c1
( x 2 − x + 1) (A) ln x − ln(1− x ) + c (B) ln(1− x 2 ) + c
1 1
I2 = ∫ dx = ∫ (C) − ln x + ln(1− x ) + c (D) ln( x − x 2 ) + c Ans. (A)
2
( x − x + 1) ⎛ 1⎞
2
3
⎜⎝ x − ⎟⎠ + dx
2 4 2. ∫ (1+ x + x 2 + x 3 ) =
⎛ 1⎞
= ln ⎜ x − ⎟ + ( x 2 − x + 1) + c2
⎝ 2⎠
1 1
1 (A) ln 1+ x - ln 1+ x 2 + tan-1 x + c
I3 = ∫ dx (Same as Illustration 22.77) 2 2
( x + 1) ( x 2 − x + 1) 1
(B) ln 1+ x - ln 1+ x 2 + tan-1 x + c
2
Chapter 22 | Indefinite Integration 987
1 1 x 2 dx
(C) ln 1+ x 2 - ln 1+ x + tan-1 x + c
2 2
9. ∫ ( x 2 + 2)( x 2 + 3) =
(D) ln 1+ x 2 + ln 1+ x + tan-1 x + c Ans. (A) (A) - 2 tan-1 x + 3 tan-1 x + c
( x − 1)dx x x
(B) - 2 tan-1 + 3 tan-1 +c
3. ∫ ( x − 2)( x − 3) = 2 3
x x
(A) ln( x − 3) − ln( x − 2) + c (C) 2 tan-1 + 3 tan-1 +c
2 3
(B) 2 ln( x − 3) − ln( x − 2) + c (D) None of these Ans. (B)
(C) ln( x − 3) + ln( x − 2) + c dx
10. ò 2 =
Ans. (B) ( x + 1)( x 2 + 4 )
(D) 2 ln( x − 3) + ln( x − 2) + c
1 1 x
dx (A) tan-1 x - tan-1 + c
4. ∫ cos x (1+ cos x ) = 3 3 2
1 -1 1 -1 x
(B) tan x + tan +c
x 3 3 2
(A) ln(sec x + tan x ) + 2 tan + c
2 1 1 x
(C) tan-1 x - tan-1 + c
x 3 6 2
(B) ln(sec x + tan x ) − 2 tan + c
2 -1 -1 x
x (D) tan x - 2 tan +c Ans. (C)
(C) ln(sec x + tan x ) + tan + c 2
2 7. Substitution for trigonometric functions:
x
(D) ln(sec x + tan x ) − tan + c Ans. (D) dx dx
2 (a) ∫ (a + b cos x ) , ∫ (a + b sin x )
dx x x
5. ∫ (1+ x )(2 + x ) = 1− tan2
2 , sin x =
2 tan
2 and put tan x = t.
Use cos x =
2 x 2 x 2
(A) ln( x + 2) − ln( x + 1) + c (B) ln( x + 2) + ln( x + 1) + c 1+ tan 1+ tan
2 2
(C) ln( x + 1) − ln( x + 2) + c (D) None of these Ans. (C) dx
Illustration 22.80 Evaluate ∫ .
(2 + cos x )
x dx
6. Correct evaluation of ∫ is Solution:
( x − 1)( x − 2) x
dx dx sec2
I=∫ 2 dx
(2 + cos x ) ∫ ⎛ x⎞ ∫
(A) 2 ln( x − 2) − ln( x − 1) + c (B) ln( x − 1) − ln( x − 2) + c = =
x
1− tan2 3 + tan2
( x − 1) ( x − 2) ⎜ 2 ⎟ 2
(C)
( x − 2)
+c (D) 2 ln
( x − 1)
+c Ans. (A) ⎜2 + 2 x
⎟
⎜⎝ 1+ tan ⎟
2⎠
dx x 1 x
= t . Then sec2 dx = dt .
7. ∫ ( x − 1)( x 2 + 1) = Put tan
2 2 2
⎛ x⎞
1 1 1 tan
ln( x − 1) − ln( x 2 + 1) − tan−1 x + c dt 2 −1 ⎛ t ⎞ 2 −1 ⎜ 2 ⎟
(A) I = 2∫ = tan ⎜ +c = tan ⎜ ⎟ +c
2 4 2 (3 + t 2 ) 3 ⎝ 3 ⎟⎠ 3 3
1 1 1 ⎜⎝ ⎟⎠
(B) ln( x − 1) + ln( x 2 + 1) − tan−1 x + c
2 4 2
dx
1 1 1 Illustration 22.81 Evaluate ∫ .
(C) ln( x − 1) − ln( x 2 + 1) − tan−1 x + c (5 + 4 cos x )
2 2 2
Solution:
(D) None of these Ans. (A)
dx
I=∫
( x 2 + x − 1) dx (5 + 4 cos x )
8. ∫ ( x 2 + x − 6)
=
⎛ 2 x⎞ x
(A) x + ln( x + 3) + ln( x − 2) + c ⎜⎝ 1+ tan ⎟⎠ dx ec2
se
2 2
I=∫ =∫ dx
⎛ ⎛ 2 x⎞ ⎛ 2 x⎞⎞ − 2 x
(B) x − ln( x + 3) + ln( x − 2) + c 5
⎜⎝ ⎝⎜ 1 + tan ⎟ + 4 ⎜ 1 − tan ⎟ 9 tan
2⎠ ⎝ 2 ⎠ ⎟⎠ 2
(C) x − ln( x + 3) − ln( x − 2) + c x 1 x
Put tan = t . Then sec2 dx = dt .
(D) None of these Ans. (B) 2 2 2
988 Mathematics Problem Book for JEE
⎛ x⎞ æ 2 xö
dt 2 −1 ⎛ t ⎞ 2 −1 ⎜
tan
⎟ ç 1+ tan ÷ dx
I = 2∫ = tan ⎜ ⎟ + c = tan ⎜ 2 +c I=ò è 2ø
(9 + t 2 ) 3 ⎝ 3⎠ 3 3 ⎟ æ 2 x x 2 xö
⎜⎝ ⎟⎠ ç 1+ tan - 2 tan - 1+ tan ÷
è 2 2 2ø
dx dx
(b) ∫ (a sin x + b cos x + c ) , ∫ (a cos x + b sin x ) sec2 dx
x
2
I=ò
æ 2 x xö
x
1− tan2 2 tan
x ç 2 tan - 2 tan ÷
2 2 x è 2 2ø
Use cos x = , sin x = and put tan = t
2 x 2 x 2
1+ tan 1+ tan Put tan
x 1 x
= t . Then sec2 dx = dt . Therefore,
2 2
2 2 2
b
Or a = r cos a and b = r sina ⇒ r = a2 + b2 and a = tan−1 . dt dt ⎛ 1 1⎞
I=∫ 2 =∫
t (t − 1) ∫ ⎝ t − 1 t ⎠
a = ⎜ − ⎟ dt
(t − t )
dx
Illustration 22.82 Evaluate ∫ .
(sin x + cos x + 2) t −1
I = ln(t − 1) − ln t + c = ln +c
t
Solution:
x
⎛ 2 x⎞ tan − 1
dx ⎜⎝ 1+ tan ⎟⎠ dx ⇒ I = ln 2 +c
2
I=∫
(sin x + cos x + 2) ∫ ⎛ ⎛
= x
x⎞ tan
2 ⎞ ⎛
x 2 x⎞ 2
⎜⎝ 2 ⎜⎝ 1+ tan 2 ⎟⎠ + ⎜⎝ 1− tan 2 ⎟⎠ + 2 tan 2 ⎟⎠
dx dx
x
(c) ò 2
(a + b cos x )
,ò
(a + b sin2 x )
,
sec2
dx
I=ò 2
æ 2x ö dx dx dx
x
ç tan + 2 tan + 3 ÷ ∫ (a sin2 x + b cos2 x + c ) , ∫ (a cos2 x + b sin2 x ) , ∫ (a cos x + b sin x )2
è 2 2 ø
x 1 x Divide the numerator and the denominator by sin2 x or cos2 x and
Put tan = t . Then sec2 dx = dt .
2 2 2 put tan x = t.
dt dt 2 ⎛ t + 1⎞ Illustration 22.85 Evaluate ò
dx
I = 2∫ = 2∫ = tan−1 ⎜ +c .
(t 2 + 2t + 3) ⎡(t + 1)2 + 2⎤ 2 ⎝ 2 ⎟⎠ (1+ 2 cos2 x )
⎣ ⎦
Solution:
x ⎞⎛
tan + 1 sec2 x dx sec2 x dx
2 ⎟
−1 ⎜ I=∫
dx
=∫ =∫
⇒ I = 2 tan ⎜ +c
2 ⎟ (1+ 2 cos2 x ) (sec2 x + 2) (tan2 x + 3)
⎜⎝ ⎟⎠
(Dividing the numerator and the denominator by cos2 x )
dx
Illustration 22.83 Evaluate ∫ . Put tan x = t . Then sec2 x dx = dt .
( 4 sin x + 3 cos x )
dt
Solution: I=∫
dx ( t 2 + 3)
I=∫
( 4 sin x + 3 cos x )
1 æ t ö 1 æ tan x ö
I= tan-1 ç ÷+c = tan-1 ç ÷+c
⎛3 4 ⎞ 4 3 è 3ø 3 è 3 ø
3 cos x + 4 sin x = 5 ⎜ cos x + sin x ⎟ = 5 cos( x − a ), tana =
⎝5 5 ⎠ 3
dx
1 dx 1 1 Illustration 22.86 Evaluate ∫ .
I= ∫ = ∫ sec( x − a )dx = [sec( x − a ) + tan( x − a )] + c (3 cos x + sin x )2
5 cos( x − a ) 5 5
Solution:
dx
Illustration 22.84 Evaluate ∫ . dx
(1− sin x − cos x ) I=∫
(3 cos x + sin x )2
Solution:
(Dividing the numerator and the denominator by cos2 x )
dx dx
I=∫
(1− sin x − cos x ) ∫ ⎛
= dx sec2 x dx
x x⎞ I=∫ =∫
2 tan 1− tan2 2
⎜ 2 2 ⎟ (3 cos x + sin x ) (3 + tan x )2
⎜ 1− 2 x
−
2 x
⎟
⎜⎝ 1+ tan 1+ tan ⎟
2 2⎠ Put tan x = t . Then sec2 x dx = dt .
Chapter 22 | Indefinite Integration 989
dt 1 (a cos x + b sin x )
I=∫ =− +c ò ( p cos x + q sin x ) dx
(3 + t )2 (3 + t )
1 (a cos x + b sin x + c ) = l ( p cos x + q sin x + r )
I=− +c
(3 + tan x ) d
+ m ( p cos x + q sin x + r ) + n
dx
cos x
Illustration 22.87 Evaluate ∫ cos 3 x dx . Compare both side coefficients of sin x, cos x and constant term,
and calculate the value of l, m and n.
Solution:
( 4 cos x + 5 sin x )
cos x cos x 1 Illustration 22.89 Evaluate ∫ dx .
I=∫ dx = ∫ dx = ∫ dx (2 cos x + 3 sin x )
cos 3 x ( 4 cos3 x − 3 cos x ) ( 4 cos2 x − 3) Solution:
( 4 cos x + 5 sin x )
(Dividing the numerator and the denominator by cos2 x ) I=∫ dx
(2 cos x + 3 sin x )
sec2 x sec2 x d
( 4 cos x + 5 sin x ) = a(2 cos x + 3 sin x ) + b
(2 cos x + 3 sin x )
I=∫ 2
dx = ∫ dx
( 4 − 3(1+ tan x )) (1− 3 tan2 x ) dx
= a(2 cos x + 3 sin x ) + b( −2 sin x + 3 cos x )
Put tan x = t . Then sec2 x dx = dt . Comparing the coefficients of cos x and sin x , we get
1 1 1 23 2
I=∫ a= ,b=
3 ∫ ⎛ 1 2⎞
2
dt = dt 13 13
(1− 3 t )
⎜⎝ − t ⎟⎠ 23 2 (3 cos x − 2 sin x )
3 I = ∫ 1dx + ∫ dx
13 13 (2 cos x + 3 sin x )
1 1+ 3t 1 1+ 3 tan x 23 2
I= ln +c = ln +c = x + ln 2 coss x + 3 sin x + c
2 3 1- 3t 2 3 1- 3 tan x 13 13
1 1 t2 tan2 q
Solving these equations, we have M = − , N = . I = ln t + + c = ln tanq + +c
2 2 2 2
1 1 sin3 2 x
sin x = − ( − sin x + cos x ) + (sin x + cos x )
2 2
Illustration 22.94 Evaluate ∫ cos5 2 x dx.
sin x Solution:
I=∫ dx
sin x + cos x sin3 2 x
I=∫
dx
1 ( − sin x + cos x ) 1 (sin x + cos x ) cos5 2 x
=− ∫ dx + ∫ dx
2 (sin x + cos x ) 2 (sin x + cos x ) The given equation may be written as
1 1
= − ln (sin x + cos x ) + x + c sin3 2 x ⋅ sec2 2 x
2 2 I=∫ dx = ∫ tan3 2 x ⋅ sec2 2 x dx
cos3 2 x
∫ sin
m
(e) x ⋅ cosn x dx , ( m, n ∈N )
Put tan 2 x = t . Then 2 sec2 2 x dx = dt .
If one out of m and n is odd, then substitute for term of even power.
If both are odd, then substitute either of the term. 1 3 t4 tan4 2 x
If both are even, then use trigonometric identities only.
I=
2 ò t dt = + c =
8 8
+c
The above substitution enables us to integrate any function of
the form R(sin x ,cos x ). However, in practice, it sometimes leads to sin nx
extremely complex rational function. In some cases, the integral
Illustration 22.95 Evaluate ∫ sin x
dx.
Solution: Solution:
x = t 3 Þ dx = 3t 2dt ⎛1 ⎛ x⎞ 1 ⎛ x⎞⎞
(C) ⎜ tan−1 ⎜ ⎟ − tan−1 ⎜ ⎟ ⎟ + c
⎝b ⎝ b⎠ a ⎝ a⎠⎠
3t 2 dt
I=∫ dt = 3∫ = 3 tan−1 t + c ⎛1 ⎛ x⎞ 1 ⎛ x⎞⎞
(D) − ⎜ tan−1 ⎜ ⎟ − tan−1 ⎜ ⎟ ⎟ + c
2 2
t (1+ t ) (1+ t 2 ) Ans. (A)
⎝b ⎝ b ⎠ a ⎝ a⎠⎠
I = 3 tan−1( x1 3 ) + c 1
3. ∫ 1+ cos2 x dx =
m +1
(c) If is an integer and p∈ fraction, then put (a + bx n ) = t k ,
1 1 ⎛1 ⎞
n (A) tan−1(tan x ) + c (B) tan−1 ⎜ tan x ⎟ + c
where k is the denominator of the fraction p. 2 2 ⎝ 2 ⎠
⎛ m +1 ⎞ 1 ⎛ 1 ⎞
(d) If ⎜ + p⎟ is an integer and p∈ fraction, then put (C) tan−1 ⎜ tan x ⎟ + c (D) None of these Ans. (C)
⎝ n ⎠ 2 ⎝ 2 ⎠
( a + bx n ) = t k . x n, where k is the denominator of the frac- 1
tion P.
4. ∫ 1+ 3 sin2 x dx =
−11
(1+ x 4 )−1/2 dx. 1 1
Illustration 22.98 Evaluate ∫x (A)
3
tan−1(3 tan2 x ) + c (B)
2
tan−1(2 tan x ) + c
1 ⎛ t5 t3 ⎞ 1 ⎛ x⎞ 1 ⎛ x⎞
I = − ⎜ − 2 + t⎟ + c (C) tan−1 ⎜ tan ⎟ + c (D) tan−1 ⎜ tan ⎟ + c
4 ⎝ 2⎠ 7 ⎝ 2⎠
2⎝ 5 3 ⎠
Ans. (A)
( )
⎡ ⎤ 1
∫ x 2 + 4 x + 13 dx
−4 5/ 2
7. is equal to
1 ⎢ 1+ x 2
( ) ( )
−4 1/ 2 ⎥
3/ 2
=− ⎢ − 1+ x −4 + 1+ x ⎥ +c
2 5 3 ⎛ x + 2⎞
⎢ ⎥ (A) ln|x 2 + 4 x + 13| + c
1
tan−1 ⎜ tan
⎣ ⎦ (B)
⎝ ⎟ +c
3 3 ⎠
992 Mathematics Problem Book for JEE
(2 x + 4 ) cot n −1 x
(C) ln 2 x + 4 + c (D) 2 2
+c In = − − In − 2
( x + 4 x + 13) n −1
Ans. (B)
dx 1
8. ∫ cos x − sin x is equal to (e) ∫ sec
n
x dx = [sec n − 2 x ⋅ tan x + (n − 2)∫ sec n − 2 x dx ]
n −1
If In = ∫ sec n x dx , then
1 ⎛ x 3p ⎞
(A) ln tan ⎜ + +c
2 ⎝ 2 8 ⎟⎠ (n − 1)In = sec n − 2 x ⋅ tan x + (n − 2)In − 2
1 x 1
ln cot + c [ − cosec n − 2 x ⋅ cot x + (n − 2)∫ cosec n − 2 x dx ]
(B)
∫ cosec
n
2
(f) x dx =
2 n −1
1 ⎛ x 3p ⎞ If In = ∫ cosec n x dx , then
(C) ln tan ⎜ − ⎟ + c
2 ⎝2 8 ⎠
(n − 1)In = − cosec n − 2 x ⋅ cot x + (n − 2)In − 2
1 ⎛x p⎞
(D) ln tan ⎜ − ⎟ + c Ans. (A)
2 ⎝ 2 8⎠ sinq +1⋅ cos p −1 p − 1
∫ sin sin p − 2 x ⋅ cosq x dx
p + q∫
p
(g) x ⋅ cosq x dx = − +
1 p+q
9. ∫ x 2 + 2 x + 2 dx =
If I p , q = ∫ sin p x ⋅ cosq x dx , then
−1 −1
(A) sin ( x + 1) + c (B) sin h ( x + 1) + c
−1
( p + q ) I( p , q ) = − sinq +1⋅ cos p −1 x + ( p − 1)I( p − 2 , q )
(C) tan h ( x + 1) + c (D) tan−1( x + 1) + c Ans. (D)
dx x ( 2n − 3) dx
3 sin x + 2 cos x (h) ∫ ( x 2 + k )n = k(2n − 2)( x 2 + k )n−1 + k(2n − 2) ∫ ( x 2 + k )n−1
10. ∫ 3 cos x + 2 sin x dx =
dx
12 5 If In = ∫ , then
(A) x − ln 3 cos x + 2 sin x + c ( x + k )n
2
13 13
12 5 x
(B) x + ln 3 cos x + 2 sin x + c k (2n − 2)In = + (2n − 3)In −1
13 13 ( x 2 + k )n −1
13 5 sinn x
(C) x + ln 3 cos x + 2 sin x + c (i) Reduction formulae for I(n, m) = ∫ dx is
12 13 cos m x
(D) None of these Ans. (A)
1 sinn −1 x n − 1 sinn −1 x
I(n , m) = ⋅ − ⋅ ⋅I
9. Reduction formulae: m − 1 cos m −1 x m − 1 cosm −1 x (n − 2 , m − 2)
cos x ⋅ sinn −1 x n − 1
∫ sin sinn − 2 x dx
n ∫
n
(a) x dx = − +
n Illustration 22.99 Evaluate ò tan5 x dx .
If In = ∫ sinn x dx , then Solution: Using
n −1
In = −
cos x ⋅ sin x
+
n −1 tann−1 x
In − 2 In = − In−2
n n n −1
sin x ⋅ cosn −1 x n − 1 Put n = 5, we get
∫ cos cosn − 2 x dx
n ∫
n
(b) x dx = +
n tan4 x
I5 = ò tan5 x dx Þ I5 = - I3
If In = ∫ cos x dx , then
n
4
tann −1 x
(c) ∫ tan x dx =
n
− ∫ tann − 2 x dx Illustration 22.100 Evaluate ∫ sin8 x dx .
n −1
If In = ∫ tann x dx , then Solution: Let cos x + i sin x = y . Then
n −1
tan x 1 1
In = − In − 2 2 cos x = y + and 2 cos nx = y n + n
n −1 y y
cot n −1 x 1 1
(d) ∫ cot
n
x dx = − − ∫ cot n − 2 x dx 2i sin x = y − and 2i sin nx = y n − n
n −1 y y
If In = ∫ cot n x dx , then (Remember as the standard results)
Chapter 22 | Indefinite Integration 993
Thus, 1
Let ln x = t. Then dx = dt. Therefore,
8 x
⎛ 1 ⎞ ⎛ 1 ⎞ ⎛ 1 ⎞
(2i sin x )8 = ⎜ y − ⎟ = ⎜ y 8 + 8 ⎟ − 8 ⎜ y 6 + 6 ⎟ dt dt 1 t +2− 5
⎝ y ⎠ ⎝ y ⎠ ⎝ y ⎠ I= ∫ 2 =∫ = ln +c
(t + 4t − 1) ((t + 2) − 5) 2 5 t + 2 + 5
2
⎛ 1 ⎞ ⎛ 1 ⎞
+28 ⎜ y 4 + 4 ⎟ − 56 ⎜ y 2 + 2 ⎟ + 70
⎝ y ⎠ ⎝ y ⎠ 1 ln x + 2 − 5
⇒I= ln +c
2 5 ln x + 2 + 5
= 2 cos 8 x − 16 cos 6 x + 56 cos 4 x − 112 cos 2 x + 70
So Hence, the correct answer is option (D).
∫x
x
1 2. (1+ ln x ) dx is equal to
sin8 x = (cos 8 x − 8 cos 6 x + 28 cos 4 x − 56 cos 2 x + 35)
27 (A) x x ln x + c (B) x x + c
(C) x ln x + c (D) None of these
and
1 Solution:
∫ sin x dx = ∫
8
7
(cos 8 x − 8 cos 6 x + 28 cos 4 x − 56 cos 2 x + 35) dx I = ∫ x x (1+ ln x ) dx
2
Let xx = t. Then
1 ⎛ sin 8 x sin 6 x sin 4 x sin2 x ⎞
∫ sin x dx =
8
⎜ −8 + 28 − 56 + 35 x ⎟ + c x ln x = ln t
27 ⎝ 8 6 4 2 ⎠
1 ⎛ sin 8 x 4 ⎞ ⎛ 1⎞ dt
= − sin 6 x + 7 sin 4 x − 28 sin2 x + 35 x ⎟ + c ⇒ ⎜ 1⋅ ln x + x ⋅ ⎟ dx = ⇒ dx (1 + ln x)xx = dt
⎜ ⎝ x ⎠ t
27 ⎝ 8 3 ⎠
Therefore,
Illustration 22.101 Evaluate ∫ cosec x dx . 4 I = ∫ dt ⇒ I = t + c = xx + c
1 ln x + 2 − 5 1 ln x + 2 − 5 1 5 + 4t 1 5 + 4(sin x − cos x )
(C) ln +c (D) ln +c = ln +c = ln +c
2 5 ln x − 2 + 5 2 5 ln x + 2 + 5 40 5 − 4t 40 5 − 4(sin x − cos x )
5p Solution:
(A) a = − , b is any arbitrary constant ⎛ x⎞
4 I = ∫ cos ⎜ ln ⎟ dx
⎝ a⎠
5p
(B) a = , b is any arbitrary constant x
4 Let ln = t. Then
a
p x = a⋅et ⇒ dx = aet dt
(C) a = − , b is any arbitrary constant
4 aet
p I = a∫ et cos t dt = (cos t + sin t ) + c
(D) a = , b is any arbitrary constant 2
4
x⎛ ⎛ x⎞ ⎛ x⎞⎞
Solution: = ⎜ cos ⎜ ln ⎟ + sin ⎜ ln ⎟ ⎟ + c
⎛ sin2 x cos 2 x ⎞ 2⎝ ⎝ a ⎠ ⎝ a⎠⎠
∫(sin2x – cos 2x) dx = ∫ 2 ⎜ − ⎟ dx =
⎝ 2 2 ⎠ Hence, the correct answer is option (B).
⎛ p⎞ ln( x + x 2 + 1)
− ∫ 2 cos ⎜ 2 x + ⎟ dx 7. ∫ dx is equal to
⎝ 4⎠ x2 + 1
2 ⎛ p⎞ 1 ⎛ 5p ⎞
=− sin ⎜ 2 x + ⎟ + c = sin ⎜ 2 x + ⎟ +c 1 2
2 ⎝ 4⎠ 2 ⎝ 4 ⎠ (A) ln ( x − x 2 + 1) + c (B) ln2 ( x − x 2 + 1) + c
2
5p
a=− , b is any arbitrary constant. 1
4 (C) ln2 ( x + x 2 + 1) + c (D) ln2 ( x + x 2 + 1) + c
2
Hence, the correct answer is option (A).
5. For what value of a and b, the equation Solution:
dx ⎛x ⎞ ln( x + x 2 + 1)
∫ 1+ sin x = tan ⎜⎝ 2 + a⎟⎠ + b holds good. I= ∫ dx
x2 + 1
5p
(A) a = − , b is any arbitrary constant Let ln (x + x 2 + 1) = t. Then
4
5p 1 ⎛ 2x ⎞ dx
(B) a = , b is any arbitrary constant ⎜ 1+ ⎟ dx = dt ⇒ = dt
4 ( x + x + 1) ⎝ 2 x 2 + 1⎠
2
x2 + 1
p
(C) a = − , b is any arbitrary constant t2
4 ⇒ I = ∫ tdt = +c
p 2
(D) a = , b is any arbitrary constant 1⎡ 2
4 I= ln ( x + x 2 + 1)⎤ + c
Solution: ⎢
2⎣ ⎥
⎦
dx dx 1 ⎛p x⎞ Hence, the correct answer is option (C).
∫ 1+ sin x = ∫ ⎛ p
= ∫ sec2 ⎜ − ⎟ dx
⎞ 2 ⎝ 4 2⎠ x cos x
1+ cos ⎜ − x ⎟
⎝2 ⎠ 8. ∫ ( x sin x + cos x )2 dx is equal to
⎛p x⎞ 1 1
tan ⎜ − ⎟ (A) +k (B) − +k
1 ⎝ 4 2⎠ ⎛p x⎞ ⎛x p⎞ ( x sin x + cos x )2 ( x sin x + cos x )
= + c = − tan ⎜ − ⎟ + c = tan ⎜ − ⎟ + c
2 1 ⎝ 4 2⎠ ⎝ 2 4⎠
− 1 1
2 (C) +k (D) +k
( x sin x + cos x )3 ( x sin x + cos x )4
p
a = − , b any arbitrary constant. Solution:
4 x cos x
Hence, the correct answer is option (C). I= ∫ dx
( x sin x + cos x )2
⎛ x⎞
6. ∫ cos ⎜⎝ ln a ⎟⎠ dx is equal to Let
1
= t. Then
x sin x + cos x
⎛ ⎛ x⎞ ⎛ x⎞⎞
(A) x ⎜ cos ⎜ ln ⎟ − sin ⎜ ln ⎟ ⎟ + c ( x sin x + cos x ) ⋅ 0 − 1( x cos x + sin x − sin x ) dt
⎝ ⎝ a⎠ ⎝ a⎠⎠ =
( x sin x + cos x )2 dx
x⎛ ⎛ x⎞ ⎛ x⎞⎞
(B) cos ⎜ ln ⎟ + sin ⎜ ln ⎟ ⎟ + c
2 ⎜⎝ ⎝ a⎠ ⎝ a⎠⎠ ⇒
− x cos x
=
dt
2
( x sin x + cos x ) dx
x⎛ ⎛ x⎞ ⎛ x⎞⎞
(C) cos ⎜ ln ⎟ − sin ⎜ ln ⎟ ⎟ + c
2 ⎜⎝ ⎝ a⎠ ⎝ a⎠⎠ Therefore,
1
I = – ∫ dt = – +c
⎛ ⎛ x⎞ ⎛ x⎞⎞ x sin x + cos x
(D) x ⎜ cos ⎜ ln ⎟ + sin ⎜ ln ⎟ ⎟ + c
⎝ ⎝ a ⎠ ⎝ a⎠⎠ Hence, the correct answer is option (B).
Chapter 22 | Indefinite Integration 995
Solution:
9. ∫( tan x + cot x ) dx is equal to
1 5 5
(A) 2sin–1 (sin x + cos x) + c (B) sin–1 (sin x – cos x) + c x5 +1 +1
5 + x10 x10 x10
2 I= ò x16
dx = ò
x16
dx = ò
x11
dx
1
(C) 2sin–1 (sin x – cos x) + c (D) sin–1 (sin x + cos x) + c 5
2 Let 1 + = t. Then
x10
Solution: ⎛ −10 ⎞ 1 1
5 ⎜ 11 ⎟ dx = dt ⇒ 11 dx = − dt
I = ∫ ( tan x + cot x ) dx ⎝ x ⎠ x 50
3/ 2
sin x + cos x 1 1 2 1⎛ 5 ⎞
= 2∫
2 sin x ⋅ cos x
dx ⇒I=−
50 ∫ tdt = – × t3/2 = − ⎜ 1+ 10 ⎟
50 3 75 ⎝ x ⎠
+c
I= ∫
1
dx = ∫
dx cot5 x 2 3
2 4 3/ 4 3/ 4 − cot x + c
⇒ I = − cot x −
x ( x + 1) ⎛ 1⎞ 5 3
x 5 ⎜ 1+ 4 ⎟
⎝ x ⎠ Hence, the correct answer is option (A).
( x 2 + 1){ln( x 2 + 1) − 2 ln x }
Let 1 + x – 4 = t. Then 13. ∫ x4
dx is equal to
−4 1 1 ( x 2 + 1) x 2 + 1 ⎡ ⎛ x 2 + 1⎞ ⎤
dx = dt ⇒ dx = − dt (A) ⎢ 2 − 3 ln ⎜ 2 ⎟⎥ + c
x5 x 5 4 x3 ⎢⎣ ⎝ x ⎠ ⎥⎦
1 dt 1 1
= − ∫ t −3 / 4 dt = − × 4t 1/ 4 + c
4 ∫ t 3/ 4
⇒I= ( x 2 + 1) x 2 + 1 ⎡ ⎛ x 2 + 1⎞ ⎤
4 4 (B) ⎢ 2 + 3 ln ⎜ 2 ⎟⎥ + c
1/ 4
9 x3 ⎢⎣ ⎝ x ⎠ ⎥⎦
⎛ 1⎞
⇒ I = ⎜ 1+ 4 ⎟ + c
⎝ x ⎠ ( x 2 + 1) x 2 + 1 ⎡ ⎛ x 2 + 1⎞ ⎤
(C) 3
⎢2 + 3 ln ⎜ 2 ⎟ ⎥ + c
Hence, the correct answer is option (B). x ⎢⎣ ⎝ x ⎠ ⎥⎦
5 + x10 ( x 2 + 1) x 2 + 1 ⎡ ⎛ x 2 + 1⎞ ⎤
11. ∫ x 16
dx is equal to (D)
9x 3
⎢2 − 3 ln ⎜ 2 ⎟ ⎥ + c
⎢⎣ ⎝ x ⎠ ⎥⎦
3/ 2 3/ 2
1⎛ 5 ⎞ 1⎛ 5 ⎞ Solution:
(A) − ⎜ 1+ ⎟ +c (B) − ⎜ 1− ⎟ +c
75 ⎝ x10 ⎠ 75 ⎝ x10 ⎠
⎛ x 2 + 1⎞ ⎧⎪ ⎛ x 2 + 1⎞ ⎫⎪
3/ 2 3/ 2 ⎜ ⎟ ⎨ln ⎜ 2 ⎟ ⎬
1⎛ 5 ⎞ 1⎛ 5 ⎞ ⎜⎝ x 2 ⎟⎠
(C) ⎜ 1− ⎟ +c (D) ⎜ 1+ ⎟ +c ( x 2 + 1){ln( x 2 + 1) − 2ln x } ⎩⎪ ⎝ x ⎠ ⎭⎪
75 ⎝ x10 ⎠ 75 ⎝ x10 ⎠ I= ∫ dx = ∫ dx
x4 x3
996 Mathematics Problem Book for JEE
x2 + 1 x
Let = t. Then Let t = tan . Then
x 2 2
2 1 x
– 3
dx = dt dt = sec2 dx
x 2 2
1 2dt
⇒ dx =
2∫
⇒I=− t ln t dt x
1+ tan2
2
1⎡ 2t 3 / 2 2 1 3 / 2 ⎤
= – ⎢(ln t ) ⋅ − ∫ ⋅ t dt ⎥ 2dt
2 ⎢⎣ 3 3 t ⎥⎦ dx =
1+ t 2
1 ⎡ 1/2
= ⎣ ∫ t dt − t 3/2 (ln t )⎤⎦ 2dt
3 1 + t2 dt
1 3/2 ⇒I= ∫ = 2∫ 2
= t [2 – 3 ln t] + c ⎛ 1− t 2 ⎞ t +9
9 5+ 4⎜ ⎟
⎝ 1+ t 2 ⎠
1 ( x 2 + 1) x 2 + 1 ⎡ ⎛ x 2 + 1⎞ ⎤
= ⎢2 − 3 ln ⎜ 2 ⎟ ⎥ + c
9 3
⎢⎣ ⎝ x ⎠ ⎥⎦
2 ⎛1 x⎞
x = tan–1 ⎜ tan ⎟ + c
3 ⎝3 2⎠
Hence, the correct answer is option (D).
Hence, the correct answer is option (A).
1+ x 2
14. ∫ x4
dx is equal to 16. ∫ ( 2 x + 3)
dx
is equal to
4x + 5
3/ 2 3/ 2
1⎛ 1⎞ 1⎛ 1⎞
(A) ⎜⎝ 1+ 2 ⎟⎠ +c (B) ⎜⎝ 1− 2 ⎟⎠ +c (A) tan–1 4 x − 5 + c (B) tan–1 4 x + 5 + c
3 x 3 x
3/ 2 3/ 2 (C) tan–1 5 x + 4 + c (D) tan–1 5 x − 4 + c
1⎛ 1⎞ 1⎛ 1⎞
(C) − ⎜ 1+ 2 ⎟ +c (D) - ⎜ 1− 2 ⎟ +c
3 ⎝ x ⎠ 3 ⎝ x ⎠ Solution:
dx
Solution: I= ∫
1 ( 2 x + 3) 4x + 5
2 1+
1+ x x 2 dx
I= ∫ dx = ∫ Put 4x + 5 = t. Then
x4 x 3
t −5 dt
Let 1 +
1
= t . Then x=⇒ dx =
2 4 4
x
1 dt 1 dt
2 ⇒I= ∫ = ∫
− dx = dt 4 ⎛ 2t − 10 ⎞ 2 (t + 1) t
x3 ⎜⎝ + 3⎟ t
4 ⎠
1
2∫
⇒I= − t dt Let t = u. Then
1
dt = du.
2 t
1 2 Therefore,
= − . t3/2 + c du
2 3 I= ∫ 2 = tan−1 t + c
3/ 2 u +1
1⎛ 1⎞
= − ⎜ 1+ 2 ⎟ +c ⇒ I = tan–1 4 x + 5 + c.
3 ⎝ x ⎠
Hence, the correct answer is option (B).
Hence, the correct answer is option (C).
∫e
ax
17. cos b xd x is equal to
dx
15. ∫ is equal to
5 + 4 cos x (A) Real part of ∫ e( a + bi ) x dx
2 ⎛1 x⎞ 2 ⎛1 x⎞ (B) Imaginary part of ∫ e( a + bi ) x dx
(A) tan–1 ⎜ tan ⎟ + c (B) − tan–1 ⎜ tan ⎟ + c
3 ⎝3 2⎠ 3 ⎝3 2⎠ (C) Neither real nor imaginary part of ∫ e( a + bi ) x dx
1 ⎛1 x⎞ (D) None of these
(C) tan–1 ⎜ tan ⎟ + c (D) None of these
3 ⎝3 2⎠
Solution:
Solution:
I = ∫ e ax cos b xd x
dx dx
I= ò 5 + 4 cos x ò
=
é 2xù
= real part of ∫ e ax e ibx dx
ê 1 - tan ú
ê
5+ 4 ê 2ú
= real part of ∫ e ax + ibx dx
2 xú
ê 1 + tan ú
ëê 2 úû Hence, the correct answer is option (A).
Chapter 22 | Indefinite Integration 997
2 x 2 + ln x
18. ∫e dx is equal to Let
1
= t. Then –
2
dx = dt. Therefore,
x2 x3
2 x2 2 x2
e e
(A) +c (B) +c 1 dt 1 1 1− x 2
4 2 I= - ò = ln |t – 1| + c = – ln
2 (t -1) 2 2 x2
+c
2 2
e2 x x2 xe 2 x
(C) + +c (D) +c
4 2 4 1 x2 1
= ln +c⇒A=
Solution: 2 1− x 2 2
+ln x
I = ∫ e2 x
2 2
dx ⇒ xe2 x dx Hence, the correct answer is option (B).
Let x2 = t. Then ⎛ 1+ x + x 2 ⎞
tan−1 x
2x dx = dt 22. ∫e ⎜
⎝ 1+ x 2 ⎠
⎟ dx is equal to
2
1 e 2t e2 x −1 −1
⇒ I = ∫ e2t dt = +c= +c (A) xe tan x + c (B) x 2e tan x + c
2 4 4
1 1 −1
Solution: 1 sin2 x
= e (3 − sin2 x ) + c (Option A)
cot x cot x 2
∫ sin x cos x dx = ∫ cot x ⋅cosec2x dx 2 ⎛ 1 ⎞
= e sin x ⎜ 1+ cos2 x ⎟ + c (Option B)
⎝ 2 ⎠
cosec2 x
= ∫ dx = −2 cot x + B = A cot x + B Hence, the correct answers are options (A) and (B).
cot x
⇒ A = −2
x3 − 3x + 7
Hence, the correct answer is option (D). 29. ∫ x2 + 4
dx is equal to
ln( x/ e )
26. The value of ∫ (ln x )2
dx is x2 7
(A) + ln( x 2 + 4 ) + c
x +1 x −1 2 2
(A) +c (B) +c x2 7 x 7
(Inx )2 (In x )2 (B) + tan−1 − ln( x 2 + 4 ) + c
2 2 2 2
x Inx
(C) +c (D) +c
Inx x x2 7
(C) −
x2 7
+ tan−1 + ln x 2 + 4 + c
2 2 2 2
( )
Solution:
x 7 −1 x
ln( x/ e ) ln( x ) − 1 (D) + tan +c
I= ∫
(ln x ) 2
dx = ∫ (ln x )2
dx 2 2 2
Solution:
Put ln x = t. Then
x = et ⇒ dx = etdt x3 − 3x + 7 7( x − 1)
=x– 2
x2 + 4 x +4
⎛ t − 1⎞ ⎛1 1⎞ et x
I = ∫ et ⎜ 2 ⎟ dt = ∫ et ⎜ − 2 ⎟ dt = + c = +c x 3 3x 7 ( x 1)
⎝ t ⎠ ⎝t t ⎠ t ln x x 2 4 dx xdx 7x 2 4 dx
Hence, the correct answer is option (C).
x3 − 3x + 7 x2 ( x − 1)
(10 x 9 + 10 x loge 10 ) ∫ 2
x +4
dx =
2
− 7∫ 2
x +4
dx
27. I = ∫ ( x10 + 10 x )
dx is equal to
2
x 7 x 7
= + tan−1 − ln( x 2 + 4 ) + c
(A) 10 x + x10 + c (B) 10 x − x10 + c 2 2 2 2
(C) 10 x + x10 + c (D) ln (10 x + x10) + c
Hence, the correct answer is option (B).
Solution: If p = x 10 + 10 x , then
e x −1 + x e −1
(10 x + 10 loge 10)dx = dp
9 x 30. ∫ ex + xe
dx is equal to
I dp p c 1 1
(A) ln (x e + e x) + c (B) ln (x + e) + c
I ln( x 10 ) c
10 x e e
1
Hence, the correct answer is option (D). (C) ln (x–e + e–x ) + c (D) None of these
e
28. The value of the integral ò e sin x (cos x + cos3 x ) sin x dx is
2
Solution:
1 sin2 x e x −1 + x e −1
(A) e (3 -sin2 x ) + c I= ∫ dx
2 ex + xe
2 ⎛ 1 ⎞ Let e x + x e = t. Then
(B) e sin x ⎜ 1+ cos2 x ⎟ + c
⎝ 2 ⎠ (e x + e⋅x e – 1) dx = dt ⇒ e(e x – 1 + x e – 1) dx = dt
sin2 x 2 2
(C) e (3 cos x + 2 sin x ) + c Therefore,
2 1 dt 1
e∫ t
sin x 2 2
(D) e (2 cos x + 3 sin x ) + c I= = ln(e x + x e) + c
e
Solution:
Hence, the correct answer is option (A).
I =e sin x (cos x cos3 x ) sin xdx =e sin x (2 sin2 x )cos x sin xdx
2 2
(A)
1 ⎛x p ⎞
log tan ⎜ + ⎟ + c (B)
1 ⎛x p ⎞
log tan ⎜ − ⎟ + c 4. If ò f ( x ) dx = y ( x ), then ∫ x 5f ( x 3 ) dx is equal to
2 ⎝ 2 12 ⎠ 2 ⎝ 2 12 ⎠
1 3
⎛x p ⎞ ⎛x p ⎞ (A) x y ( x 3 ) - 3ò x 3y ( x 3 ) dx + C
(C) log tan ⎜ + ⎟ + c (D) log tan ⎜ − ⎟ + c 3
⎝ 2 12 ⎠ ⎝ 2 12 ⎠
1
[AIEEE 2007] (B) x 3y ( x 3 ) - ò x 2y ( x 3 ) dx + C
3
Solution:
1
dx 1 dx (C) ⎡⎣ x 3y ( x 3 ) − ∫ x 3y ( x 3 ) dx ⎤⎦ + C
∫ cos x + 3 sin x
=
2∫ 1 3
3
cos x + 1⎡ 3
sin x (D) x y ( x 3 ) − ∫ x 2y ( x 3 ) dx ⎤⎦ + C [JEE MAIN 2013]
4 4 3⎣
1 dx
= ∫ Solution: We have
2 ⎛ p⎞
cos ⎜ x − ⎟
⎝ 3⎠ ∫ f ( x ) dx = y ( x )
1 ⎛ p⎞
2∫
= sec ⎜ x − ⎟ dx Let x3 = t and x2 dx = dt/3. Then
⎝ 3⎠
1 1
1 ⎛x p p⎞
= log tan ⎜ − + ⎟ + c x 5f ( x 3 ) dx 3tf (t )dt 3 tf (t )dt 1f (t )dt dt
2 ⎝ 2 6 4⎠
1
1 ⎛x p ⎞ x 3y ( x 3 ) x 2y ( x 3 ) dx C
= log tan ⎜ + ⎟ + c 3
2 ⎝ 2 12 ⎠
Hence, the correct answer is option (B).
Hence, the correct answer is option (A).
1
sin x dx ⎛ 1⎞ x+
2. The value of 2 is 5. The integral ∫ ⎜ 1+ x − ⎟ e x dx is equal to
p ⎝ x⎠
sin x
4 x+
1
x+
1
(A) ( x + 1)e x +c (B) − xe x +c
⎛ p⎞ ⎛ p⎞
(A) x + log cos ⎜ x − ⎟ + c (B) x − log sin ⎜ x − ⎟ + c x+
1
x+
1
⎝ 4⎠ ⎝ 4⎠ (C) ( x − 1)e x +c (D) xe x +c
⎛ p⎞ ⎛ p⎞
(C) x + log sin ⎜ x − ⎟ + c (D) x − log cos ⎜ x − ⎟ + c [JEE MAIN 2014 (OFFLINE)]
⎝ 4⎠ ⎝ 4⎠
[AIEEE 2008] Solution:
Solution: ⎧⎪ x + 1 1 ⎞ x + x ⎫⎪
1 1 1
⎛ 1⎞ x+ x x dx + x ⎛ 1−
x+
p p
sin
∫ ⎜⎝ 1 + x − ⎟
x⎠
e dx = ∫⎨ e ⎜⎝ ⎟
x2 ⎠
e ⎬ d x = xe x +c
x dx ⎩⎪ ⎭⎪
2
sin x dx 4 4
2
p p
Since,xf ( x ) f ( x )dx xf ( x ) c
sin
x sin
x
4 4
p p p Hence, the correct answer is option (D).
p
2cos cot x sin dx dx cot x dx
4 4 4 4 sin8 x − cos8 x
p
6. ∫ (1− 2 sin2 x cos2 x ) dx is equal to
x log sin x c
4
1 1
Hence, the correct answer is option (C). (A) sin 2 x + c (B) − sin 2 x + c
2 2
5 tan x 1
3. If the integral ∫ tan x − 2 dx = x + a ln sin x − 2 cos x + k , then a (C) − sin x + c
2
(D) − sin2 x + c
is equal to
(A) −1 (B) −2 [JEE MAIN 2014 (ONLINE SET-1)]
(C) 1 (D) 2 [AIEEE 2012]
Solution:
Solution:
sin8 x cos8 x (sin4 x cos4 x )(sin4 x cos 4 x )dx
5 tan x 5sinx 1 2 sin2 cos2 x dx
tan x 2 dx sin x 2 cos x dx (1 2 sin2 x cos2 x )
2(cos x 2 sin x ) (sin x 2 cos x ) {(sin2 x + cos2 x )2 -2 sin2 x cos2 x }{sin2 x -cos2 x }dx
dx =ò
sin x 2 cos x (1-2 sin2 x cos2 x))
cos x 2 sin x
2 dx dx 2 log sin x 2 cos x x k ( 1 2 sin2 x cos2 x )( cos 2 x )dx sin2 x
sin x 2 cos x ⇒ ⇒− +c
2
( 1 2 sin x cos x ) 2 2
Therefore, a = 2.
Hence, the correct answer is option (D). Hence, the correct answer is option (B).
1000 Mathematics Problem Book for JEE
⎛ 1− x 2 ⎞ x 5m −1 + 2 x 4 m −1
7. The integral ∫ x cos −1 ⎜ ⎟ dx ( x > 0 ) is equal to 9. If m is a non-zero number and ∫ ( x 2m + x m + 1)3 dx = f ( x ) + c ,
⎝ 1+ x 2 ⎠ then f(x) is
(A) −x + (1 + x2) tan−1 x + c (B) x − (1 + x2) cot−1 x + c
(C) −x + (1 + x2) cot−1 x + c (D) x − (1 + x2) tan−1 x + c x 5m x 4m
(A) (B)
2m( x 2 m + x m + 1)2 2m( x 2 m + x m + 1)2
[JEE MAIN 2014 (ONLINE SET-2)]
2m( x 5m + x 4 m ) ( x 5m − x 4 m )
Solution: (C) 2m m 2
(D)
(x + x + 1) 2m( x 2 m + x m + 1)2
1 x 2
I x cos 1 dx , x 0
2
[JEE MAIN 2014 (ONLINE SET-4)]
1 x
Solution:
Put x = tan q. Then dx = sec2q dq.
x 5m −1 + 2 x 4 m −1 x 5m −1 + 2 x 4 m −1
I=∫ dx ⇒ ∫ dx
−1
I = ∫ tanq cos (cos 2q )sec q dq = 2∫ q tanq sec q dq
2 2
(x 2m m
+ x + 1) 3
{x 2m
(1+ x − m + x −2 m )}3
Again put tan q = t. Then sec2q dq = dt. x 5m−1 + 2 x 4 m−1 x − m−1 + 2 x −2m−1
⇒∫ dx ⇒ ∫ dx
Therefore x 6m (1+ x − m + x −2m )3 (1+ x − m + x −2m )3
1 1 1
= t2 tan−1t − [t − tan −1 t] = t2 tan−1t − t + tan −1 t = (t2 + 1) tan−1 t − = +c = +c
2m t 2
2m(1+ x + x −2 m )2
− m
t = {tan 2 q + 1} tan−1 (tanq ) − tan q = q {tan 2 q + 1} − tan q
1 x 4m
Therefore, I = (1 + x2) tan −1 x − x + c. = 2
+c = 2m
+c
⎛ 1 1 ⎞ 2m( x + x m + 1)2
2 m ⎜ 1+ m + 2 m ⎟
Hence, the correct answer is option (A). ⎝ x x ⎠
sin2 x cos2 x Hence, the correct answer is option (B).
8. The integral ∫ dx equal to
(sin3 x + cos3 x )2 dx
1 1 10. The integral ∫ equals
(A) +c (B) − +c x 2 ( x 4 + 1)3 / 4
(1+ cot3 x ) 3(1+ tan3 x ) 1 1
3 3 (A) ( x 4 + 1) 4 + c (B) −( x 4 + 1) 4 + c
sin x cos x
(C) 3
+c (D) − +c 1 1
(1+ cos x ) 3(1+ sin3 x ) ⎛ x 4 + 1⎞ 4 ⎛ x 4 + 1⎞ 4
[JEE MAIN 2014 (ONLINE SET-3)] (C) − ⎜ 4 ⎟ + c (D) ⎜ 4 ⎟ + c
⎝ x ⎠ ⎝ x ⎠
Solution:
[JEE MAIN 2015 (OFFLINE)]
sin2 x cos2 x sin2 x cos2 x
I=∫ 3 3 2
dx = ∫ dx Solution:
(sin x + cos x ) cos6 x (1+ tan3 x )2 dx dx
I=∫ =∫
sin2 x tan2 x sec2 x x 2 ( x 4 + 1)3 / 4 ⎛ 1⎞
3/ 4
=∫ dx = ∫ dx x 5 ⎜ 1+ 4 ⎟
cos 4 x (1+ tan3 x )2 (1+ tan3 x )2 ⎝ x ⎠
Put x−4 = t. Then
Now put tan x = t. Then sec2 x dx = dt.
−4
dx = dt
Therefore, x5
t 2dt − dt 1 (1 t )1/ 4
I = −∫ ⇒ I=∫ c = − (1 + x−4)1/4 + c
(1+ t )3 2 4(1+ t )3 / 4 4 1/ 4
Hence, the correct answer is option (C).
Again substituting 1 + t3 = m, we get
3t2 dt = dm dx
11. The integral ∫ 3/4
is equal to
1 dm 1 1 1 1 1 ( x + 1) ( x − 2)5/4
2 c
3 m 3 m 3 1 t 3 3(1 tan3 x ) 1/4 1/4
⎛ x + 1⎞ ⎛ x − 2⎞
(A) 4 ⎜ +C (B) 4 ⎜ +C
Hence, the correct answer is option (B). ⎝ x − 2 ⎟⎠ ⎝ x + 1 ⎟⎠
Chapter 22 | Indefinite Integration 1001
1/4 1/4
4 ⎛ x + 1⎞ 4 ⎛ x − 2⎞ ⎛ 2 5⎞
(C) − ⎜ ⎟ +C (D) − ⎜ ⎟ +C ⎜⎝ 3 + 6 ⎟⎠
3 ⎝ x − 2⎠ 3 ⎝ x + 1⎠ ⇒∫ x x dx
3
[JEE MAIN 2015 (ONLINE SET-1)] ⎛ 1 1⎞
⎜⎝ 1+ 2 + 5 ⎟⎠
Solution: x x
dx dx Put 1 + x−2 + x−5 = t. Then
I =∫
( x 1)3 / 4 ( x 2)5 / 4 ( x + 1)3 / 4 ( x − 2)2 ( x − 2)5 / 4 − 2 2 5
3 6 dx dt
3 / 4 x x
dx ( x 1) 1
=∫ dx
( x + 1)3 / 4 ( x − 2)2 ( x − 2)−3 / 4 ( x 2)3 / 4 ( x 2)2 1 1
⇒ − 3 dt 2 + C
−3 / 4 t 2t
1 ⎡ x + 1⎤ ⎛ −3 ⎞
−3 ∫ ⎢⎣ x − 2 ⎥⎦
= ⋅⎜ dx 1 x10
⎝ ( x − 2)2 ⎟⎠ ⇒ 2
+C = +C
1 1 2( x + x 3 + 1) 5
x 1 3 2 1+ 2 + 5
Put t dt dx x x
x 2 ( x 2)2
Hence, the correct answer is option (C).
1 4 1/ 4
I = (t )3 / 4 dt (t ) C
3 3 dx
1/ 4
14. If ∫ cos3 x2 sin 2 x
= (tan x ) A + C (tan x )B + k , where k is a
4 é x + 1ù
Þ- ê ú +C constant of integration, then A + B + C equals
3 êë x -2 ûú
16 27
Hence, the correct answer is option (C). (A) (B)
5 10
log(t + 1+ t 2 )
1 7 21
12. If ∫ dt = ( g(t ))2 + C , where C is a constant,
2 2
(C)
10
(D)
15
( 1+ t )
then g(2) is equal to [JEE MAIN 2016 (ONLINE SET-1)]
(A) 2 log(2 + 5 ) (B) log(2 + 5 ) Solution: We have
1 1 dx
(C) log(2 + 5 ) log(2 + 5 )
(D) I=∫
5 2 cos x 4 tan x cos2 x
3
[JEE MAIN 2015 (ONLINE SET-2)]
1 dx 1 sec 4 x dx
Solution: I= ∫ 4
= ∫
( ) dt
2 cos x tan x 2 tan x
log t + 1+ t 2 Put tan x = t. Then sec2x dx = dt. Therefore,
I=∫
1+ t 2 1 (1+ t 2 )dt 1 −1/ 2 3/ 2
2 ∫ t1/ 2
I= = ∫ (t + t )dt
Since,
d⎛
dt ⎝ ( 2 ⎞
⎜ log t + 1+ t ⎟⎠ =
1
1+ t 2
)
, we get
1⎛ 2
2
⎞
= ⎜ 2t 1/ 2 + t 5 / 2 ⎟ + k
1 2 2⎝ 5 ⎠
I log(t 1 t 2 ) C
2 1
= (tan x )1/ 2 + (tan x )5 / 2 + k
2
⇒ g(t ) = log(t + 1+ t ) ⇒ g(2) = log(2 + 5 ) 5
Comparing this with the given equation, we get A = 1/2, B = 5/2
Hence, the correct answer is option (B). and C = 1/5. Therefore,
2 x12 + 5 x 9 1 5 1 1 16
13. The integral ò ( x 5 + x 3 + 1)3
dx is equal to A+B +C = + + = 3+ =
2 2 5 5 5
− x10 − x5 Hence, the correct answer is option (A).
(A) 5 3 2
+C (B) 5 3 2
+C dx
2( x + x + 1) ( x + x + 1) 15. The integral ∫ is equal to (where C is a
x 10
x5 (1+ x ) x − x 2
(C) +C (D) +C constant of integration)
2( x 5 + x 3 + 1)2 2( x 5 + x 3 + 1)2
[JEE MAIN 2016 (OFFLINE)] 1+ x 1− x
(A) −2 +C (B) − +C
Solution: We have 1− x 1+ x
2 x12 + 5 x 9 2 x12 + 5 x 9 1− x 1+ x
∫ ( x 5 + x 3 + 1)3 dx ⇒ ∫ ⎛ 1 1⎞
3
dx (C) −2
1+ x
+C (D) 2
1− x
+C
x15 ⎜ 1+ 2 + 5 ⎟
⎝ x x ⎠ [JEE MAIN 2016 (ONLINE SET-2)]
1002 Mathematics Problem Book for JEE
Solution: We have 1 1
Put t + = m ⇒ 1− 2 dt = d m
dx dx t t
I=∫ =∫
(1+ x ) x − x 2 (1− x ) dm
(1+ x )2 x ⇒ I − I = 2
(1+ x ) m −1
1− x 1 m −1
Put y = . Then = log
1+ x 2 m +1
1
dy dx 1 e x e x 1
x (1 x )2 log x x c
Therefore,
2 e e 1
1 e 2 x e x 1
dy y ( −1/ 2 )+1 log 2 x x c
I = −∫ =− + C = −2 y + C
y ( −1/ 2) + 1
2 e e 1
Hence, the correct answer is option (C).
1− x
I = −2 +C sec2 x
1+ x 2. The integral ∫ dx equals (for some arbitrary
(sec x + tan x )9 / 2
Hence, the correct answer is option (C).
constant K)
Previous Years' Solved JEE Advanced/ 1 ⎧1 1 2⎫
(A) − 11/ 2 ⎨ − (sec x + tan x ) ⎬ + K
(sec x + tan x ) ⎩11 7 ⎭
IIT-JEE Questions
1 ⎧1 1 2⎫
(B) ⎨ − (sec x + tan x ) ⎬ + K
e x
e −x (sec x + tan x )11/ 2 ⎩11 7 ⎭
1. Let I = ∫ dx , J = ∫ −4 x dx . Then, for an
e 4 x + e2 x + 1 e + e −2 x + 1 1 ⎧1 1 2⎫
(C) − 11/ 2 ⎨ + (sec x + tan x ) ⎬ + K
arbitrary constant C, the value of J − I equals (sec x + tan x ) ⎩ 11 7 ⎭
1 ⎛ e 4 x − e2 x + 1⎞ 1 ⎧1 1 2⎫
(A) log ⎜ 4 x ⎟ +C (D) 11/ 2 ⎨ + (sec x + tan x ) ⎬ + K
2 ⎝ e + e2 x + 1⎠ (sec x + tan x ) ⎩11 7 ⎭
[IIT-JEE 2012]
1 ⎛ e2 x + e x + 1⎞
(B) log ⎜ 2 x ⎟ +C Solution:
2 ⎝ e − e x + 1⎠
sec2 x
1 ⎛ e2 x − e x + 1⎞ I=∫ dx
(C) log ⎜ 2 x ⎟ +C (sec x + tan x )9 / 2
2 ⎝ e + e x + 1⎠
Let sec x + tan x = t . Then
1 ⎛ e 4 x + e2 x + 1⎞
(D) log ⎜ 4 x ⎟ +C sec x tan x 1/ t
2 ⎝ e − e2 x + 1⎠ [IIT-JEE 2008] Now,
Solution: We have (sec x tan x + sec2 x )dx = dt ⇒sec x (sec x + tan x )dx = dt
e x dx dt 1 1
I=∫ 4x sec x dx , t sec x
e + e2 x + 1 t 2 t
e x dx e3 x dx 1
J 4 x 2 x 4 x 2 x t
1 t dt 1 9 / 2 13 / 2
e e 1 e e 1 I 9/2 (t t ) dt
2 t t 2
Now,
9 1 13 1
J −I = ∫
(e2 x − 1)e x dx 1 t 2 t 2 1 t 7 / 2 t 11/ 2
9 13 11
e 4 x + e2 x + 1 2 1 1 2 7
2 2 2
Put ex = t. Then 2
exdx = dt 1 1 1 1 1 1
t 7 / 2 t 11/ 2 7 / 2
(t 2 1)dt 7 11 7t 11 t 11/ 2
J I 4 2
t t 1 1 1 t2
1 11/ 2
1 2 dt t 11 7
t
1 1 1
1 2 11/ 2
(sec x tan x )2 k
t 1 (sec x tan x )
11 7
t
Hence, the correct answer is option (C).
Chapter 22 | Indefinite Integration 1003
∫e
x
2 2 15. ( 4 x 2 + 8 x + 3) dx is equal to
(A) +c (B) +c
sin x cos x
(A) (2 x + 1) e x + k
2
(B) ( x + 1)2 e x + k
2 2
(C) +c (D) +c
tan x (sin x )3 / 2 (C) ( 4 x 2 + 3)e x + k (D) None of these
dx 2x
7. ∫ x ln x ln(ln x ) is equal to 16. The anti-derivative of w.r.t. x is
1− 4 x
(A) ln |(ln (ln x))| + c (B) |ln x| + c (A) log2 e ⋅ sin−1(2 x ) + k (B) sin−1(2 x ) + k
⎛ ⎛ 1⎞ ⎞
(C) ln ⎜ ln ⎜ ⎟ ⎟ + c (D) ln |ln x| + c (C) cos −1(2 x )log2 e + k (D) None of these
⎝ ⎝ x⎠⎠
dx
⎡1+ 1− x 2 sin−1 x ⎤ 17. ∫ is equal to
8. Value of the integral ∫ e ⎢ x ⎥ dx is equal to x x4 −1
⎢ 1− x 2 ⎥
⎣ ⎦ 1
(A) sec −1 x 2 + c (B) sec −1 x 2 + c
ex 2
(A) e x sin−1 x + c (B) +c
1− x 2 (C) tan−1 x 2 + c (D) cosec −1x 2 + c
1004 Mathematics Problem Book for JEE
dx 1
18. ∫ sec2 x + tan2 x is equal to (C)
4
sec 2q tan 2q + ln 4 sec 2q + tan 2q + c
x −a log( x /e )
35. The value of ∫
b−x
dx is equal to 43. The value of ∫ (log x )2
dx is equal to
⎡ sin 2q ⎤ x +1 x −1
(A) (a + b ) ⎢ + q ⎥ + c, where a sin2 q + b cos2 q = x (A) +c (B) +c
⎣ 2 ⎦ (log x )2 (log x )2
⎡ sin 2q ⎤ x log x
(B) (a − b ) ⎢ + q ⎥ + c , where a sin2 q + b cos2 q = x (C) +c (D) +c
⎣ 2 ⎦ log x x
⎡ sin 2q ⎤
(C) (a − b ) ⎢ + q ⎥ + c , where a sin2 q − b cos2 q = x dx
⎣ 2 ⎦ 44. ∫ cos( x − a)cos( x − b) =
sin 2q
(D) (a b ) q c , where a sin2 q − b cos2 q = x sin( x − a)
2 (A) cosec(a − b )log +c
sin( x − b )
bx cos 4 x − a sin 4 x a sin 4 x cos( x − a)
36. If ∫ x2
dx =
x
+ c, then a and b may be (B) cosec(a − b )log
cos( x − b )
+c
(A) a = 2, b = 2 (B) a = 1, b = 4
sin( x − b )
1 (C) cosec(a − b )log +c
(C) a = –1, b = 4 (D) a = , b = 2 sin( x − a)
4
cos( x − b )
(D) cosec(a − b )log +c
dx 1− x 3 − 1 cos( x − a)
37. If ∫ = a log + c , then
x 1− x 3 1− x 3 + 1
dx
(A) a =
1
(B) a =
2 45. ∫ x +a + x +b
=
2 3
1 2 2 ⎡
(C) a = (D) a = − (A) ( x + a)3 / 2 − ( x + b )3 / 2 ⎤⎦ + c
3 3 3(b − a) ⎣
2 ⎡
⎛ 1⎞ 1 1 (B) ( x + a)3 / 2 − ( x + b )3 / 2 ⎤⎦ + c
38. If ∫ x loge (1+ 1/ x )dx = P( x )ln ⎜⎝1+ x ⎟⎠ + 2 x − 2 ln(1+ x ) + c , 3(a − b ) ⎣
2 ⎡
then (C) ( x + a)3 / 2 + ( x + b )3 / 2 ⎤⎦ + c
x2 3(a − b ) ⎣
(A) p( x ) = (B) p(x) = –1 (D) None of these
2
(C) p (x) = 1 (D) None of these
3 cos x + 3 sin x
cos x 46. ∫ 4 sin x + 5 cos x dx =
39. ∫ dx is equal to
x 27 3
cos x (A) x − log( 4 sin x + 5 cos x ) + c
(A) 2cos x + c (B) +c 41 41
x 27 3
(B) x + log( 4 sin x + 5 cos x ) + c
(C) sin x + c (D) 2sin x + c 41 41
1+ tan x 27 3
x − log( 4 sin x − 5 cos x ) + c
40. ∫ x + log sec x dx = (C)
41 41
(D) None of these
(A) log (x + log sec x) + c (B) −log (x + log sec x) + c
1
(C) log (x − log sec x) + c (D) None of these 47. If ∫ (sin 2 x + cos 2 x )dx = sin(2 x − c ) + a , then the value of a
2
( x 2 − 1) x2 + 1 and c is
41. If ∫ dx = k log tan−1 + c,
⎛ x 2 + 1⎞ x (A) c = p /4 and a = k (an arbitrary constant)
( x 4 + 3 x 2 + 1)tan−1 ⎜ ⎟ (B) c = −p /4 and c = p / 2 a = p / 2
⎝ x ⎠
(C) c = p /2 and a is an arbitrary constant
then k is equal to
(D) None of these
(A) 1 (B) 2
(C) 3 (D) 5
x3 − x − 2
42. The value of ∫
ln x
dx is equal to
48. ∫ (1− x 2 )
dx =
2
(1+ ln x ) 2 2
⎛ x + 1⎞ x ⎛ x − 1⎞ x
x x ln x (A) log ⎜ ⎟ − +c (B) log ⎜ ⎟ + +c
(A) +c (B) +c ⎝ x − 1⎠ 2 ⎝ x + 1⎠ 2
1− ln x 1+ ln x
2 2
x ln x ⎛ x + 1⎞ x ⎛ x − 1⎞ x
(C) +c (D) +c (C) log ⎜ ⎟ + +c (D) log ⎜ ⎟ − +c
1+ ln x x + x ln x ⎝ x − 1⎠ 2 ⎝ x + 1⎠ 2
1006 Mathematics Problem Book for JEE
1 ⎡ a 2a a2 1 ⎤ 1
(D) 2⎢
x + − log(a + bx ) − ⎥+c (A) ⎡sin−1 x 2 + 1− x 4 ⎤ + c
b ⎢⎣ b b b a + bx ⎥⎦ 2 ⎢⎣ ⎥⎦
1 ⎡ −1 2
(B) sin x − 1− x 4 ⎤ + c
dx 2 ⎢⎣ ⎥⎦
50. ∫ =
(1+ x ) p + q2 (tan−1 x )2
2 2
(C) sin−1 x 2 + 1− x 4 + c
(A)
1
log ⎡q tan−1 x + p2 + q2 (tan−1 x )2 ⎤ + c (D) sin−1 x 2 + 1− x 2 + c
q ⎣⎢ ⎦⎥
1
57. If ∫ f ( x )sin x cos x dx = log(f ( x )) + c , then f ( x ) =
(B) log ⎡q tan−1 x + p2 + q2 (tan−1 x )2 ⎤ + c 2(b2 − a2 )
⎢⎣ ⎥⎦
1 1
2 2 (A) (B)
(C) ( p + q2 tan−1 x )3 / 2 + c a2 sin2 x + b2 cos2 x a2 sin2 x − b2 cos2 x
3q
(D) None of these 1 1
(C) (D)
a2 cos2 x + b2 sin2 x a2 cos2 x − b2 sin2 x
5
x
51. ∫ dx = dx
1+ x 3 58. ∫ ( x − a ) (b − x )
=
2
(A) (1+ x 3 )3 / 2 + c ⎛ x − a⎞ ⎛ x − a⎞
9 (A) 2 sin−1 ⎜ +c (B) sin−1 ⎜ +c
⎝ b − a ⎟⎠ ⎝ b − a ⎟⎠
2 2
(B) (1+ x 3 )3 / 2 + (1+ x 3 )1/ 2 + c
⎛ x + a⎞
9 3 (C) 2 sin−1 ⎜ +c (D) None of these
⎝ b − a ⎟⎠
2 2
(C) (1+ x 3 )3 / 2 − (1+ x 3 )1/ 2 + c
9 3
59. If ∫ x e x cos x dx = ( x ) + c , then f(x) is equal to
(D) None of these
ex
52. ∫
dx
is equal to
(A) {(1− x ) sin x − x cos x }
2
sin x − cos x + 2
1 ⎛x p⎞ 1 ⎛x p⎞ ex
(A) −
2
tan ⎜ + ⎟ + c
⎝ 2 8⎠
(B)
2
tan ⎜ + ⎟ + c
⎝ 2 8⎠
(B) {(1− x ) sin x + x cos x }
2
1 ⎛x p⎞ 1 ⎛x p⎞ ex
(C)
2
cot ⎜ + ⎟ + c
⎝ 2 8⎠
(D) −
2
cot ⎜ + ⎟ + c
⎝ 2 8⎠ (C) {(1+ x ) sin x − x cos x }
2
(D) None of these
adx
53. ∫ b + ce x = dx
60. If I = ∫ = f(x) + c, then f(x) is equal to
a ⎛ e ⎞ x
a ⎛ b + ce x⎞ e x + 4e − x
(A) log ⎜ x⎟
+c (B) log ⎜ ⎟ +c 1 ⎛ ex ⎞
b ⎝ b + ce ⎠ b ⎝ ex ⎠ (A) 2 tan–1 (2 ex) (B) tan−1 ⎜ ⎟
2 ⎝ 2⎠
b ⎛ ex ⎞ b ⎛ b + ce x ⎞
(C) log ⎜ ⎟ +c (D) log ⎜ ⎟ +c ex 1
a ⎝ b + ce x ⎠ a ⎝ ex ⎠ (C) 2 tan−1 (D) tan−1(2 e 2 x )
2 2
54. ∫ sin x dx = dx
61. ∫ (2 x + 1) 1+ is equal to
(A) 2 ⎡⎣sin x − cos x ⎤⎦ + c (B) 2 ⎡⎣sin x − x cos x ⎤⎦ + c ( (2 x + 1) )
Chapter 22 | Indefinite Integration 1007
2x +1 2x +1 1
(A) tan−1 +c (B) loge +c ( x 4 − x )4
1+ 2 x + 1 1+ 2 x + 1 69. ∫ x5
dx is equal to
∫a
2 2 aa
73. ⋅ aa ⋅ a x dx is equal to
sin 2 x sin 2 x
(C) − − sin x + c (D) + sin x + c aa
x
ax
2 2 (A) +c (B) aa (log a)3 + c
3
(log a)
e x (1+ x )
65. ∫ cos2 ( xe x ) dx =
aa
ax
∫ tan
3
66. 2 x sec 2 xdx =
x log5 e x log5 5e
1 (A) (B)
(A) sec3 2x + 3 sec 2x (B) ⎡⎣sec3 2 x − 3 sec 2 x ⎤⎦ log5 e log5 5e
6
x loge 5e + 1
(C) ⎡⎣sec3 2 x − 3 sec 2 x ⎤⎦ (D) None of these (C) (D) None of these
loge 5e + 1
sec x cosec x
67. ∫ dx
logtan x
(A) log (tan x) + c (B) cot (log x) + c
75. ò x 2 sin xdx =
(C) log (log tan x) + c (D) tan (log x) + c (A) x2 sin x − 2 x cos x + c
(B) x2 sin x + c
68. ∫ cos
3
x elog(sin x )dx is equal to
(C) − x2 cos x + 2 x sin x + 2 cos x + c
sin4 x cos 4 x (D) − x2 sin x − 2x cos x + sin x + c
(A) − +c (B) − +c
4 4
⎪⎧ (1− cos x ) ⎪⎫
(C)
e sin x
+c (D) None of these
76. ∫ ⎨⎪[cos x (1+ cos x )] ⎬⎪ dx =
4 ⎩ ⎭
1008 Mathematics Problem Book for JEE
x 2 dx x dx ⎛p ⎞ ⎛p ⎞
4. Let f ( x ) = ∫ and g( x ) = ∫ , if (C) g′ ⎜ ⎟ is equal to 6
⎝ 3⎠
(D) g′ ⎜ ⎟ is equal to 12
⎝ 3⎠
2 2 4
(1+ x )(1+ 1+ x ) 1+ x
p 11. Let f(x) = [b2 + (a – 1)b + 2]x – ∫ (sin2 x + cos 4 x )dx be an
f(0) = g(0) = 0. Then the value of f (1) − 2g(1) + is
4
increasing function of x ∈ R and b ∈ R. Then a can take value(s)
(A) 0 (B) 1 (A) 0 (B) 1
(C) 2 (D) None of these (C) 2 (D) 4
4 Comprehension Type Questions
5. ∫
x10 + x 8 + 1
x 6 (3 x 10
)
+ 2 x 8 − 2 dx =
Paragraph for Questions 12 and 13: Let n be a positive integer
2( x10 + x 8 + 1)5 4 4( x10 + x 8 + 1)5 4 such that In = ∫ x n a2 − x 2 dx. Then answer the following questions:
(A) +c (B) +c
5x5 5
12. The value of I1 is
6 4 −4 5 4
2( x + x + x ) 2 2 1 2
(C) +c (D) None of these (A) ( a − x 2 )1/ 2 + C (B) ( a − x 2 )3/ 2 + C
5x5 3 3
2 1
6. ∫ ( x 6 + x 4 ) (2 x 4 + 3 x 2 )dx , x > 0 = (C) – ( a2 − x 2 )3/ 2 + C (D) – ( a2 − x 2 )3/ 2 + C
3 3
3
1 1 6 2 − x n −1(a2 − x 2 )3 / 2
(A) (2 x 6 + 3 x 4 )3 / 2 dx + c (B) x ( 2 x + 3) 2 + c 13. If In = + kIn − 2, then the value of k is
6 18 n+2
3
1 1 2 2 n −1 n+2
(C) (2 x 6 + 3 x 4 )3 / 2 dx + c (D) x ( 2 x + 3) 2 + c (A) (B)
12 18 n+2 n −1
⎛ n − 1⎞ 2 ⎛ n + 2⎞ 2
(C) ⎜
⎝ n + 2 ⎟⎠
(D) ⎜
x4 −1 ⎝ n − 1 ⎟⎠
a a
7. ∫ ( x 5 + 1)( x + 1) dx =
4
1 ⎛ ( x 5 + 1) ⎞ 1 ⎛ ( x 5 + 1) ⎞ Matrix Match Type Questions
(A) ln ⎜ ⎟ +c (B) ln ⎜ ⎟ +c
5 ⎝ ( x + 1) ⎠ 5 ⎝ ( x + 1) ⎠ 14. Match the following:
Answer Key
Practice Exercise 1
1. (D) 2. (B) 3. (B) 4. (D) 5. (C) 6. (B)
7. (A) 8. (A) 9. (A) 10. (B) 11. (C) 12. (C)
13. (B) 14. (D) 15. (C) 16. (A) 17. (B) 18. (B)
19. (B) 20. (C) 21. (A) 22. (D) 23. (B) 24. (B)
25. (B) 26. (B) 27. (C) 28. (A) 29. (D) 30. (C)
31. (D) 32. (D) 33. (C) 34. (A) 35. (B) 36. (B)
37. (C) 38. (A) 39. (D) 40. (A) 41. (A) 42. (C)
43. (C) 44. (B) 45. (B) 46. (A) 47. (A) 48. (D)
49. (D) 50. (A) 51. (C) 52. (D) 53. (A) 54. (B)
55. (A) 56. (A) 57. (A) 58. (A) 59. (A) 60. (B)
61. (B) 62. (B) 63. (D) 64. (C) 65. (B) 66. (B)
67. (C) 68. (B) 69. (A) 70. (C) 71. (A) 72. (A)
73. (C) 74. (D) 75. (B) 76. (A) 77. (C) 78. (A)
79. (C) 80. (A) 81. (A) 82. (C) 83. (C) 84. (A)
85. (B) 86. (C)
Practice Exercise 2
1. (B) 2. (A) 3. (C) 4. (A) 5. (A) 6. (B)
7. (C) 8. (A) 9. (A), (B) 10. (B), (D) 11. (A), (B), (C) 12. (D)
13. (C) 14. (A) → (r), (B) → (s), (C) → (s), (D) → (p)
Solutions
⎜⎝ t + ⎟⎠ I et tan t c xe tan x
c
2
tan x sin x
=
x 2
− ln(e x + 1) + ln(2e x + 3) + c
6. ∫ cos x
dx = ∫
(cos x )3 / 2
dx (Let cos x = t )
3 3
= − ∫ t −3 / 2dt
(1− sin2 x )cos x dx
3. I=∫
(sin x + sin2 x ) t −1/ 2
=− +c
1
⎛ 1− t 2 ⎞ −
= ∫⎜ ⎟ dt [Put (sin x = t)] 2
⎝ t + t2 ⎠ 2
= +c
cos x
⎛ 1− t ⎞
= ∫⎜ dt
⎝ t ⎟⎠ dx
7. ∫ x ln x ln(ln x ) (Put In x = t)
= log |sin x| − sin x + c
Chapter 22 | Indefinite Integration 1011
dt 1
∫ t lnt ⇒ k=−
7
1
∫e ( 4 x 2 + 8 x + 3)dx = ∫ e x [f ( x ) + f ’( x )] + 3∫ e x dx
x
Let ln t = z. Then, dt = dz . So, 15.
t
dz = 4 x 2e x + 3e x + k
ò z = log z = log (ln t)
16. Put 2 x = t . Then
= ln |ln(ln x)| + c
2 x ln2 dx = dt
⎡1+ 1− x 2 sin−1 x ⎤
1 dt
∫e ⎢ ⎥ dx
x
8. log2 e sin1 t c
⎢ 1− x 2 ⎥ ln 2 1 t 2
⎣ ⎦
x
2x
e
=∫ dx + ∫ e x ⋅ sin−1 x dx dx log2 e sin1 2 x k
1− x 2 1 4 x
ex ex
=∫ dx + e x sin−1 x − ∫ dx 17. Put x 2 = t . Then
2 2
1− x 1− x
2x dx = dt
x −1
= e sin x + c
1 dt 1
x3
sec 1 t c
2 t t2 1 2
9. ∫ 1+ x 8 dx
1 2 x dx 1
sec 1 x 2 c
Let x 4 = t . Then 4 x 3dx = dt . 2 2 2 2
x x2 1
1 dt 1
= tan−1 t + c
4 ∫ 1+ t 2 4
I=
dx (1− sin2 x ) 2 − (1+ sin2 x )
1
18. ∫ sec2 x + tan2 x = ∫ (1+ sin2 x ) dx = ∫ (1+ sin2 x )
dx
I= tan−1 x 4 + c
4 sec2 x
= 2∫ dx − ∫ 1 dx (Let tan x = t )
∫ xe dx = x ∫ e x dx − ∫ e x dx + c = xe x − e x + c 2 tan2 x + 1
x
10.
2 dt
dx dx = ∫ − x = 2 tan−1( 2 tan x ) − x + c
11. ò =ò = sin-1( x - 1) + c 2 2 ⎛ 1 ⎞2
t +⎜
⎝ 2 ⎟⎠
2 2
1- ( x - 2 x + 1) 1- ( x - 1)
12. I 1 p 1 dp 1 p 1
dp
1
dp x2 tan2 x dx
2 p p2 1 2 p p2 1 2 p p2 1 19. I = ∫ 2
sec2 x + ∫
1+ x 1+ x 2
1
f ( p ) ln p p 2 1 sec 1 p sec2 x tan2 x dx
2 = ∫ sec2 x − ∫ dx + ∫
1+ x 2 1+ x 2
13. From given equation, we have −1
= tan x − tan x + c
(sinq + cosq )dq
∫ 1 1 sin x
20. I = − logcos x ⋅ cos 2 x − ∫
1− (sinq − cosq )2 cos 2 x dx
2 2 cos x
Put sin q − cos q = t. Then
1 1 ⎛ 2 cos2 x − 1⎞
(cos q + sin q ) dq = dt = − cos 2 x log cos x − ∫ ⎜ sin x dx
Therefore, 2 2 ⎝ cos x ⎟⎠
35 x 6dx = dt 1 t2 1
= − cos 2 x logcos x + − log t
Therefore, from given equation, we have 2 2 2
1 k (cos x )2 1
35 ∫
sin t ⋅ dt = cos(5 x 7 ) 1
5 = − cos 2 x ⋅ logcos x + − logcos x + k
2 2 2
− cos(5 x 7 ) k 2 ⎛1 ⎞
⇒ = cos(5 x 7 ) = cos x ⎜ − logcos x ⎟ + k
35 5 ⎝2 ⎠
1012 Mathematics Problem Book for JEE
3 + 2 cos x 1
21. I = ∫ dx I2 sec 2q . tan2 2q dq sec2 2q 1. 2sec2q tan 2q dq
(2 + 3 cos x )2 2
sin x 1
Let
3 cos x + 2
= t . Then I2
2
t 2 1dt put sec 2q t
1 t
(3 cos x + 2)cos x − sin x ( −3 sin x ) 1
dx = dt I2 t 2 1 ln t t 2 1 c2
2 2 2
(3 cos x + 2)2
I2 sec 2q sec2 2q 1 In sec 2q sec2 2q 1 c
3 + 2 cos x
1 1
⇒ 2
dx = dt 4 2
(3 cos x + 2)
1 1
sin x I I1 I2 sec 2q .tan2q lnsec 2q tan2q c
⇒ I = ∫ 1 dt = t + c = +c 4 2
3 cos x + 2
27. Let x 2 + sin 2 x + 2 x = t. Then
1
22. ∫ f ( x )cos x dx = f 2 ( x ) + c ⇒ (2x + 2 cos 2x + 2 )dx = dt
2
dt
By putting f (x) = sin x, we get ⇒ ( x + cos 2 x + 1)dx =
2
1 1 1 dt 1 1
⇒ I = ∫ = log t = log( x 2 + sin 2 x + 2 x ) + k
2∫
I= sin 2 x dx = cos 2 x + c
4 2 t 2 2
1 d
= − (1− 2 sin2 x ) + c 28. As ( x + logsec x ) = 1+ tan x , so
4 dx
1 I = log( x + logsec x ) + c
= sin2 x + k
2 ex ex
1
29. ∫ (2 x + 3) dx − 2∫ (2 x + 3)2 dx
= f 2( x ) + k
2
ex
Hence, f(x) = sin x. = ∫ e x [f ( x ) + f ’( x )]dx = +c
2x + 3
23. I = ∫ g( x ) f ( x ) dx − ∫ g( x ) f ’’( x ) dx (ln x )2 − 2(ln x ) + 1 ⎡ (ln x )2 + 1 ln x ⎤
30. ∫ 2
((ln x ) + 1) 2
dx = ∫ ⎢
⎢⎣
2
+ 2
−2 2
+ 2⎥
⎥⎦
dx
( )
((ln x ) 1) ((ln x ) 1)
= f ( x ) ⋅ ∫ g( x )dx − ∫ f ’( x ) ⋅ ∫ g( x )dx − ∫ g(x) f " ( x )dx
1 ln x
=∫ dx − 2∫ dx
= f ( x )g( x ) − [ g( x ) ⋅ f ’( x ) − ∫ f ’’( x ) ⋅ g( x )dx ] − ∫ g( x ) f "( x )dx (ln x )2 + 1 (ln x )2 + 1
x x 2 ln x ln xdx
= g(x) (f(x)) – f ’(x) + c = 2
+∫ 2 2
dx − 2∫ +c
(ln x ) + 1 x ((ln x ) + 1) (1+ (ln x )2 )2
∫e dx = ∫ sin x dx = −cos x + c
ln(sin x )
24. x
= +c
(ln x )2 + 1
k +7
⎛ x2 − x −1 ⎞ ⎛ x 2 + 1⎞
25. I = ∫ ⎜ + 1⎟ ⎜ 2 ⎟ dx e x ⎡⎣ x ( x 2 + 1) + 1⎤⎦
⎝ x ⎠ ⎝ x ⎠ 31. ∫ ( x 2 + 1)3 / 2
dx
k +7
⎛ x 2 − 1⎞ ⎛ x 2 + 1⎞ ⎡ x 1 ⎤
= ∫⎜ = ∫ ex ⎢ 2 + 2 3/ 2 ⎥
dx
⎟ ⎜ 2 ⎟ dx ⎣ ( x + 1)1/ 2
( x + 1) ⎦
⎝ x ⎠ ⎝ x ⎠
Now, (Since, e x [f ’( x ) + f ( x )] = e x f ( x ) + c )
⎛ 1⎞ ⎛ 1⎞ xe x
⎜⎝ x − ⎟⎠ = t ⇒ ⎜⎝ 1+ 2 ⎟⎠ dx = dt = +c
x x ( x 2 + 1)1/ 2
k +8 2 cos2 x − 1
⎛ 1⎞ 32. ∫ dx
t k +8 ⎜⎝ x − ⎟⎠ cos x
k +7 x
I = ∫ t dt = +c = +c
k+8 k+8 = 2∫ cos x − ∫ sec x dx
26. I sec3 (2q ) dq sec( 2q ) 1 tan2 2q dq = 2sin x − log|(sec x + tan x )| + c
sec 2q dq sec 2q tan2 2q dq 33. cos x = t ⇒ − sin x dx = dt
t −3 / 2 +1 2
1 I = − ∫ t −3 / 2dt = − +c = + c = 2 sec x + c
I1 sec 2q dq lnsec 2q tan2q c1 3 t
2 − +1
2
Chapter 22 | Indefinite Integration 1013
x ⎛ x + 2⎞
2
x⎛ 2 ⎞
2
⎛ sec x tan x ⎞
40. x + log sec x = t ⇒ ⎜ 1+ ⎟ dx = dt
34. ∫ e ⎜⎝ x + 4 ⎟⎠ dx = ∫ e ⎜⎝1− x + 4 ⎟⎠ dx ⎝ sec x ⎠
1 Therefore,
= ln q tan−1 x + p2 + (q tan−1 x )2 + c
q 1 f ’( x )
f ( x )sin x ⋅ cos x = 2 2 f (x)
2 (b − a )
51. Put 1+ x 3 = t 2 . Then 3 x 2dx = 2tdt and x 3 = t 2 − 1.
f ’ (x)
⇒ 2 (b2 − a2 )sin x ⋅ cos x =
So, f 2( x )
x5 x2 × x3 f ’ (x)
⇒ 2 (b2 ∫ sin x ⋅ cos x dx − a2 ∫ sin x ⋅ cos x dx ) = ∫
ò dx = ò dx
f 2( x )
dx
1+ x 3 1+ x 3 1
⇒ ( −b2 cos2 x − a2 sin2 x ) = −
2 (t 2 − 1) ⋅ t 2 2 ⎛ t3 ⎞ f (x)
= ∫ dt = ∫ (t 2 − 1) dt = ⎜ − t ⎟ + c
3 t 3 3⎝ 3 ⎠ 1
⇒ −f (x) =
2 ⎛ (1+ x 3 )3 / 2 ⎞ ( −b2 cos2 x − a2 sin2 x )
= ⎜ − (1+ x 3 )1/ 2 ⎟ + c
3⎝ 3 ⎠ 58. Put x = acos2q + bsin2q , the given integral becomes
2(b − a) sinq cosq dq
52. I = ∫
dx
=∫
dx I=∫ 1
sin x − cos x + 2 ⎛ p p ⎞
2 ⎜ sin x sin − cos x cos + 1⎟
⎝ 4 4 ⎠
{ (a cos2 q + b sin2 q − a) (a cos2 q + b sin2 q − b } 2
1 t 5/ 4 4⎛ 1⎞
5/ 4 1− cos x 1− t
= ⋅ + c = ⎜ 1− 3 ⎟ +c =
3 5/ 4 15 ⎝ x ⎠ cos x (1+ cos x ) t (1+ t )
⎛ x + 1⎞ ⎛1 2 ⎞ ⎛ 1 2 ⎞
=⎜ − = −
log ⎜
⎝ x ⎟⎠ 1 ⎝ t 1+ t ⎟⎠ ⎜⎝ cos x 1+ cos x ⎟⎠
70. I=∫ ⋅ 2 dx
( x + 1) x Therefore,
x ⎛ 1 2 ⎞ ⎛ x⎞
I = ∫⎜ − dx = ∫ ⎜ sec x − sec2 ⎟ dx
x +1 1 ⎝ cos x 1+ cos x ⎟⎠ ⎝ 2⎠
Now put t = = 1+ , then
x x ⎛ x⎞
= log (sec x + tan x) − 2 tan ⎜ ⎟
1 ⎝ 2⎠
dt = − dx
x2 77. Put (1 − x) = t.
Therefore, Differentiating above equation, we have
2
1 1 1 ⎡ ⎛ x + 1⎞ ⎤ − dx = dt
I = − ∫ log t ⋅ dt = − (log t )2 + c = − ⎢log ⎜ ⎟ +c
t 2 2 ⎣ ⎝ x ⎠ ⎥⎦ Now,
As,
=
t 25 t 24
− +c=
(1− x ) 25
−
(1− x )24 + c
1 – sin 2x = (sin x − cos x)2 25 24 25 24
Hence, 78. Put log x = t. Then
∫e
sin x
cos xdx dx = et dt ⇒ x = et
Put sin x = t, then cos x dx = dt. Now,
1 dz z sin4 ( p + q ) x f (( p + q ) x )
dt = dz = ∫ = ∫ cosec z dz = ln tan + k
1− t 2 sin z 2 ∫ ( p + q) x = p + q
⎛ sin−1 e x ⎞
= ln tan ⎜ sin4 ( p + q ) dx
⎟ +k ∫ = f (( p + q ) x )
⎝ 2 ⎠ x
x n −1
x
83. elog = x x
1+ ++
1! (n − 1)!
cos2 x 3. I = n ! ∫ dx − n ! ∫ dx
I = ∫ x cos x dx + ∫ sin x cos x dx = x sin x + cos x − +c x xn
2 1+ ++
1! n!
x + sin x 1 x
84. I=∫ dx = ∫ sec2 ( x + sin x )dx ⎛ ex ⎞
1+ cos x 2 2 ⇒ I = n !ln ⎜ +c
2 n⎟
1 2 x 2 x ⎜ 1+ x + x + + x ⎟
I = ∫ x sec + ∫ tan dx ⎝ 1! 2 ! n! ⎠
2 2 2 2
1 tan x / 2 1 tan x / 2
= x − ∫ + ∫ tan x / 2 dx = x tan x/2 + c x 2 dx x 2 ( 1+ x 2 − 1) dx
2 1/ 2 2 1/ 2 4. f (x) = ∫ =∫
(1+ x 2 )(1+ 1+ x 2 ) (1+ x 2 ) x 2
1 1
85. I =∫ dx = ∫ dx dx dx
f (x) = ∫ −∫ = ln x + 1+ x 2 − tan−1 x + c
3
4 cos x − 3 cos x cos 3 x
1+ x 2 1+ x 2
1
= ∫ sec3x dx = ln|sec 3x + tan 3x| + c f(0) = 0 = c
3
p
f (1) = ln 1+ 2 −
86. I = ∫ sin(ln x ) dx 4
1 2 x dx 1
Let ln x = t. Then g( x) =
2∫
= ln x + 1+ x 4 + c
2
x = et ⇒ dx = et dt 1+ x 4 2
Therefore, g (0) = 0 = c
I = ∫ et ⋅ sin t dt g (1) =
1
ln 1+ 2
2
= sin t⋅et − ∫ cost ⋅ et dt
(3 x10 + 2 x 8 − 2)
I = ∫ x . x 6 + x 4 + x −4
4
= sin t⋅et − cos t ⋅ et − ∫ sin t ⋅ e t dt 5. dx
x6
2I = et (sin t − cos t)
= ∫ x 6 + x 4 + x −4 (3 x 5 + 2 x 3 − 2 x −5 ) dx
4
Therefore,
1
I = et(sin t − cos t) Put x 6 + x 4 + x −4 = t . Then
2
1 x 2 (3 x 5 + 2 x 3 − 2 x −5 ) dx = dt
= e ln [sin (ln x) − cos (ln x)] + c
2
1 4 2 (t )5 4 2 ( x10 + x 8 + 1)5 4
x
= [sin (ln x) − cos (ln x)] + c I=
2 ∫ t dt =
5
+c =
5x5
+c
2
1018 Mathematics Problem Book for JEE
f ( x )g ’( x ) − g( x )f ’( x ) f (x) 1
I=∫ dx = ln +c 14. (A) Put f101(x) = t. Then dx = dt .
f ( x )g( x ) g( x ) xf1 ( x )f2 ( x )f100 ( x )
⎛ e x cos x − sin x ⎞ Therefore, given integral = f101(x) + c.
= ln ⎜ x ⎟ +c
⎝ e sin x + cos x ⎠ x + sin x ⎛x x x⎞ x
(B) f ( x ) = ∫ dx = ∫ ⎜ sec2 + tan ⎟ dx = x tan + c
1+ cos x ⎝2 2 2⎠ 2
9. Putting sinq = t, we get
∫ ( 4t − 4t
3
(
) et = At 3 + B (1− t 2 ) + Ct + D 1− t 2 + E et + F ) (C) f ( x ) = ∫ e sin
−1
x
⎛
⎜ 1−
⎝
x ⎞
⎟ dx
1− x 2 ⎠
It follows immediately that D = 0.
Differentiating both sides w.r.t. t, we get −1
⎛ 1− x 2 x ⎞
= ∫ e sin x
⎜ − ⎟ dx
(4t − 4t3) et = [At3 + (3A – B)t2 + (C − 2B)t + C + B + E ]e t ⎜⎝ 1− x 2
1− x 2 ⎟⎠
And hence, A = – 4, B = – 12, C = – 20. −1
f ( x ) = e sin x
1 − x 2 + c ⇒ f ( 0 ) = 1+ c ⇒ c = 0
1 1
10. F(x) = ∫ dx = ∫ dx ⎛ 1⎞ 3 ep 6
4 − 3 cos2 x + 5 sin2 x 9 − 8 cos2 x f⎜ ⎟=
⎝ 2⎠ 2
sec2 x sec2 x 1
=∫ dx = ∫ dx = tan−1 (3 tan x ) + c ⎛ tan x ⎞
2
9 sec x − 8 1+ 9 tan2 x 3 (D) f ( x ) = ∫ ⎜ ⎟ dx
⇒ g(x) = 3 tanx ⎝ sin x ⋅ cos x ⎠
Therefore, = ∫ (tan x )−1 2 sec2 x dx = 2 tan x + c
⎛p ⎞
g⎜ ⎟ = 3 f (0) = c = 0 ⇒ c = 0
⎝ 4⎠
⎛p ⎞ ⎛p ⎞ 2k
and g′ ⎜ ⎟ =12 f⎜ ⎟ =2= ⇒k =p
⎝ 3⎠ ⎝ 4⎠ p
Chapter 22 | Indefinite Integration 1019
1. Let In = ∫ tann x dx , (n > 1). If I4 + I6 = a tan5x + bx5 + C, where = ∫ tan4 x ⋅ sec2 xdx
C is a constant of integration, then the ordered pair (a, b) is
equal to: Substituting t = tanx, we get
⎛1 ⎞ ⎛1 ⎞ t5
(A) ⎜ , 0⎟ (B) ⎜ , −1⎟ I4 + I6 = ∫ t 4 ⋅ dt = +c
⎝5 ⎠ ⎝5 ⎠
5
⎛ 1 ⎞ ⎛ 1 ⎞
(C) ⎜ − , 0⎟ (D) ⎜ − ,1⎟ That is,
⎝ 5 ⎠ ⎝ 5 ⎠ tan5 x
I 4 + I6 = +c
(OFFLINE) 5
Solution: We have
1
In = ∫ tann x dx On comparison, we get a = , b = 0.
5
I4 + I6 = ∫ (tan4 x + tan6 x )dx Hence, the correct answer is option (A).
23 Definite Integration
0 2
b ⇒ I = ∫ −t dt + ∫ t dt
If f(x) > 0 for all x ∈ [a, b]; then òa f(x) is numerically equal to the −1 0
0 2
area bounded by the curve y = f(x), x-axis and the straight lines x ⎡ −t 2 ⎤ ⎡t2 ⎤ 5
⇒ I=⎢ ⎥ +⎢ ⎥ ⇒I =
= a and x = b. ⎣ 2 ⎦ −1 ⎣ 2 ⎦ 0 2
p /2 dx
x =a f(x) x =b Illustration 23.3 Evaluate ∫ 0 1+ sin x
.
S Solution:
L C
p /2 dx
I=∫
0 sin x /2 + cos x /2 + 2sin x /2cos x /2
2 2
+ +
p /2 dx p /2 sec2x /2
I=∫
(sin x /2 + cos x /2)2 ∫0 (1+ tan x /2)2
= dx
0
A − −
O M
B D Put (1+ tan x / 2) = t . Then
Q 1 2
R sec x /2 dx = dt
2
2
2 dt ⎡ 1⎤ ⎡ 1 1⎤
Figure 23.1 I = 2 ∫ 2 = −2 ⎢ ⎥ = −2 ⎢ − ⎥ = 1
1 t
⎣ ⎦1
t ⎣ 2 1⎦
b
In general, ∫a f(x)dx represents algebraic sum of the figures ∞ ∞
dx x 2dx
Illustration 23.4 Let a = ∫ and b = ∫ 4 .
bounded by the curve (Fig. 23.1) y = f(x), the x-axis and the straight 0
x + 7x +1
2 4
0
x + 7x2 +1
line x = a and x = b. The areas above x-axis are taken with plus sign Then show that a = b.
and the areas below the x-axis are taken with minus sign.
That is, Solution:
∞
dx
b
a =∫
∫
a
f(x)dx = area OLA − area AQM – area MRB + area BSCD
0
x4 + 7x2 +1
1022 Mathematics Problem Book for JEE
4
t t 2
⎛ (b − a)2 ⎞ ⎧ (b − a)2 2(b − a)3 ⎫
= (b − a)a + ⎜ 3
⎟ + 3a ⎨a + ⎬
23.3 Definite Integration as the ⎝ 2 ⎠ ⎩ 2 6 ⎭
nh = b - a
Illustration 23.6 Evaluate ∫a
sin x dx by first principle.
Solution:
Now, we form the sum
b
hf (a) + hf (a + h) + hf (a + 2h) + … + hf (a + rh) + … + hf [a + (n − 1)h] I = ò sin x dx
a
n-1
= h[f (a) + f (a + h) + f (a + 2h) + ¼¼ + f (a + rh) + ¼¼ + f {a + (n - 1)h}] I = lim hå sin(a + rh)
n−1 h®0
= h∑ f (a + rh),
r =0
b
Illustration 23.7 Evaluate ò ln( x + 1)dx .
0
∫x
3
Illustration 23.5 Evaluate dx by first principle. Solution:
a
1
Solution: I = ò ln( x + 1)dx = ( x + 1)ln( x + 1) - ( x + 1)|10
b 0
I = ∫ x 3 dx ⇒ I = 2ln2 − 1
a
n−1
I = lim h∑ (a + rh)3 1 1
h→0
r =0
Illustration 23.8 Evaluate ò0 x +1
dx .
b c b 2
Generally, we break the limit first at the points where f(x) is dis- I = ∫ |1− x 2 | dx = ∫ |1− x 2 | dx + ∫ |1− x 2 | dx + ∫ |1− x 2 | dx
−2 −2 −1 1
continuous and second at the points where definition of f(x) −1 1 2
Or 4 4 4
b c1 c2 b ⇒I = + + =4
ò f ( x )dx = ò f ( x )dx + ò f ( x )dx + ¼ + ò f ( x )dx where 3 3 3
a a c1 c n
Solution: Let
Solution: Let 2p
| cos x |
5p
12
I= ∫ cos x
dx
I= ∫ [tan x ]dx
0
p 3p
0
2 2 2p
5p cos x cos x cos x
Value of tan x at x = is 2 + 3. =∫ dx − ∫ dx + ∫ dx
12 0 cos x p cos x 3p cos x
2 2
Value of tan x at x = 0 is 0. p 3p
2 2 2p
Integers between 0 and 2 + 3 are 1, 2 and 3. Therefore, = ∫ 1dx − ∫ 1dx +
tan x = 1, 0 p
∫ 1dx
3p
tan x = 2,
2 2
p p
and = −p + = 0
2 2
tan x = 3
⇒ x = tan−1 1, p /2
p /2 p /2
Therefore,
tan 1 −1
tan 2−1
tan 3−1
5p I= ∫
− p /2
cos x − cos3 x dx = ∫
− p /2
cos x (1− cos2 x ) dx
12
I= ∫ [tan x ]dx + ∫ [tan x ] dx + ∫ [tan x ]dx + ∫ [tan x ]dx p /2 0 p /2
0 tan−1 1 tan−1 2
5p
tan−1 3 I= ò
- p /2
| sin| cos x dx = − ∫ sin x cos x dx + ∫ sin x cos x dx
− p /2 0
tan−1 1 tan−1 2 tan−1 3 12
= ∫ 0 dx + ∫ 1dx + ∫ 2 dx + ∫ 3 dx Put cos x = z , thensin x dx = − dz
0 tan−1 1 tan−1 2 tan−1 3
1 0 1
æ b a
ö
⎛ 5p
= 0 + (tan−1 2 − tan−1 1) + 2(tan−1 3 − tan−1 2) + 3 ⎜
⎞
− tan−1 3⎟ I = ò z dz - ò z dz = 2 ò z dz çç As ò f ( x ) = - ò f ( x ) ÷÷
⎝ 12 ⎠ 0 1 0 è a b ø
5p p 1
= − − tan−1 3 − tan−1 2 2
I = 2 × z 3/2 =
4
4 4 3 3
0
⎡ ⎛ 3+2 ⎞ ⎤
= p − ⎢ tan−1 ⎜ −1
⎟ + p ⎥ = - tan ( −1)
⎣ ⎝ 1− 6 ⎠ ⎦ b b a a
=
p 4. ∫ f ( x )dx = ∫ f (a + b − x )dx . In particular ∫ f ( x )dx =∫ f (a − x )dx
a a 0 0
4
1024 Mathematics Problem Book for JEE
Proof: Let p /4 ⎛ 2 ⎞
⇒I =∫ ln ⎜ dx
b
0 ⎝ 1+ tan x ⎟⎠
I = ∫ f (a + b − x )dx
p /4 p /4
⇒I =∫ ln2 dx − ∫ ln(1+ tan x )dx
a
a + b − x = z ⇒ dx = − dz 0 0
x = a ⇒ z = b, x = b ⇒ z = a p p
Þ 2I = ln2 Þ I = ln2
a b 4 8
I = − ∫ f ( z )dz ⇒ I = ∫ f ( z )dz p /2
a sin x + b cos x
b
b a
Illustration 23.17 Evaluate ∫
0
sin x + cos x
dx .
I = ∫ f ( x )dx Solution:
a
p /2
a sin x + b cos x
(A) ∫
p /2
0
f (sin2 x )sin x dx = ∫
0
p /2
f (sin2 x )cos x dx I= ò
0
sin x + cos x
dx
p /2
p /2
⎛p ⎞ ⎛p ⎞
(B) ∫0
f (sin x )dx = ∫
0
f (cos x )dx p /2 a sin ⎜ − x ⎟ + b cos ⎜ − x ⎟
⎝2 ⎠ ⎝2 ⎠ p /2
a cos x + b sin x
p /2 p /2 I= ∫ ⎛p ⎞ ⎛p ⎞
dx ⇒ I = ∫
sin x + cos x
dx
(C) ∫0
f (tan x )dx = ∫
0
f (cot x )dx 0 sin ⎜ − x ⎟ + cos ⎜ − x ⎟
⎝2 ⎠ ⎝2 ⎠
0
1 1
(D) ∫ f (ln x ) dx = ∫ f [ln(1− x )] dx
0 0 2I = ∫
p /2
(a + b )
cos x + sin x p
dx ⇒ 2I = (a + b )
0 cos x + sin x 2
p /2 sinn x
Illustration 23.14 Evaluate ∫ 0 sin x + cosn x
n
dx .
I=
p
(a + b )
4
Solution:
p /2 sinn x 1
I=∫ dx Illustration 23.18 Evaluate ∫ ( x − 1)(1− x )99 dx .
0 sin x + cosn x
n
0
p /2 cosn x Solution:
I=∫ dx 1
0 sin x + cosn x
n
I = ò ( x - 1)(1- x )99 dx
p p 0
2I = ⇒I = 1 1
2 4
I = ∫ ( − x )( x )99 dx ⇒ I = − ∫ x 100 dx
Similarly we can solve these examples: 0 0
101 1
tann x
p /2 p /2 cot x
n
p x 1
I=− =−
(A) ∫ 1+ tan x
0 n
dx = ∫
0 1+ cot n x
dx =
4 101 0 101
p /2 1 p /2 1 p p
ò
2
Solution:
p /2 sec n x p /2 cosec n x p
(C) ∫ sec x + cosec x
n n
dx = ∫
cosec n x + sec n x
dx =
4
p
I = ∫ e sin x cos3 x dx
0 0 2
p /2 0
Illustration 23.15 Evaluate ∫ 0
ln(tan x )dx . p
I = ∫ e sin
2
(p − x )
cos3 (p − x ) dx
Solution:
0
p /2
I=∫ ln(tan x )dx p
I = − ∫ e sin x cos3 x dx ⇒ I = − I ⇒ I = 0
2
0
p /2
I=∫ ln(cot x )dx 0
0
2a f (x)
p /2 p /2
ln ( tan x ⋅ cotx ) dx
Illustration 23.20 Evaluate ∫ dx .
2I = ∫
0
(ln(tan x ) + ln(cot x ))dx = ∫
0
0 f ( x ) + f (2a − x )
Solution:
2I = 0 ⇒ I = 0
2a f (x) 2a f (2a - x )
I=ò dx = ò dx
p /4 0 f ( x ) + f (2a - x ) 0 f (2a - x ) + f ( x )
Illustration 23.16 Evaluate ∫ 0
ln(1+ tan x )dx .
2a f ( x ) + f (2a − x ) 2a
Solution: 2I = ∫ dx = ∫ dx = [ x ]20a = 2a
0 f ( x ) + f (2a − x ) 0
p /4
I=∫ ln(1+ tan x )dx ÞI =a
0
Chapter 23 | Definite Integration 1025
p /2 tan x p /2 sin x I = I1 + I2
I=ò dx Þ I = ò dx
1+ tan x cos x + sin x
0 0 2a
I2 = ò f ( x )dx
a
⎛p ⎞
sin ⎜ − x ⎟
p /2 ⎝2 ⎠ p /2 cos x Put
I=∫ dx ⇒ I = ∫ dx
0
⎛p ⎞ ⎛p ⎞
0 cos x + sin x x = 2a − t ⇒ dx = −dt
cos ⎜ − x ⎟ + sin ⎜ − x ⎟
⎝2 ⎠ ⎝2 ⎠ x = a ⇒ t = a , x = 2a ⇒ t = 0
0 a
p /2 p I2 = − ∫ f (2a − t )dt = ∫ f (2a − t )dt
2I = ∫ 1dx ⇒ I = a 0
0 4
a
3p /4 dx I2 = ∫ f (2a − x )dx
Illustration 23.22 Evaluate ò
p /4 1+ cos x
.
0
a
Solution:
I = ∫ (f ( x ) + f (2a − x ))dx
3p /4 dx
I=∫
0
p /4 1+ cos x 2p
x × sin2 n x
⎡ ⎛ p 3p ⎞ ⎤
Illustration 23.25 Evaluate ò sin2 n x + cos2 n x
dx x > 0.
3p /4 1
I=∫ dx ⎢cos ⎜⎝ + − x ⎟ = − cos x ⎥ 0
p /4 1− cos x ⎣ 4 4 ⎠ ⎦ Solution:
2p 2p
(2p - x )sin2 n (2p - x ) (2p - x )sin2 n x
2I = ò
3p /4 2
3p /4
2 I= ò dx = ò sin
dx = ò sin (2p - x ) + cos2 n (2p - x ) x + cos2 n x
2n 2n
dx
p /4 1- cos2 x p /4 sin2 x 0 0
2p
2p × sin x 2n
⇒ 2I = 2 ∫
3p /4
cosec2 x dx
Þ 2I = ò sin
0
2n
x + cos2 n x
dx
p /4
p
⇒ 2I = −2[cot x ]p3p/4/4 = 4 ⇒ I = 2 sin2 n x
Þ 2I = 4p ò dx
0
sin x + cos2 n x
2n
3 x p /2
sin2 n x
Illustration 23.23 Evaluate ∫
2
5− x + x
dx. Þ I = 4p ò
0
sin x + cos2 n x
2n
Solution:
3 x p /2
cos2 n x æ a a
ö
I=∫
2
5− x + x
dx Þ I = 4p ò sin2 n x + cos2 n x
dx çç As ò f ( x )2 ò f (a - x ) ÷÷
0 è 0 0 ø
3 5− x
I=∫ dx p /2
2
5− x + x Þ 2I = 4 ò p dx = 2p 2 Þ I = p 2
3 0
2I = ∫ 1dx ⇒ 2I = [ x ]32 = 1 p
2
Illustration 23.26 Evaluate ò x lnsin x dx.
⇒ I = 1/ 2
0
Solution:
1
æ1 ö
Illustration 23.24 Evaluate ò ln ç - 1÷ dx.
p
èx ø I = ò x lnsin x dx
0
0
Solution:
p p
1
⎛ 1− x ⎞ x
1
1− x
1
I = ∫ (p − x )lnsin(p − x )dx = ∫ (p − x )lnsin x dx
I = ∫ ln ⎜ ⎟⎠ dx = ∫ ln dx = − ∫ ln dx
0
⎝ x 0
1 − x 0
x 0 0
p p /2
p /2 p /2 4
x2
2I = p ò lnsin2 x dx - p ò ln2 dx
0 0
I= ∫x
−4
2
+ 16
dx
p I = 2∫ dx ⇒ I = 2 ∫ dx
p p2 x + 16
2
x 2 + 16
2I = ò lnsin t dt - ln2 0 0
20 2 4
⎛ 16 x⎞ ⎛ p⎞
I = 2 ⎜ x − tan−1 ⎟ = 2 ⎜ 4 − 4 ⋅ ⎟ = 8 − 2p
p /2
p2 ⎝ 4 4⎠ 0 ⎝ 4⎠
2I = p ò lnsin t dt -
0 2
ln2
p
æ 5- x ö
2
dx
p2 p2
2I = I - ln2 Þ I = - ln2
Illustration 23.29 Evaluate ò 1+ 5
0
cos x
+ ò ln ç
-2
÷ dx .
è 5+ x ø
2 2
Solution: Let
p
∫
2 p
Illustration 23.27 Evaluate e cos x cos3 (2n + 1) x dx ,(n ∈I ). dx
0 I1 = ò 1+ 5
0
cos x (1)
Solution:
Now
p
I = ∫ e cos
2
(p − x )
⋅ cos3 (2n + 1)(p − x )dx p
dx dx
p
5cos x dx
p
⎛ 5− x ⎞
2
⎪0, if f ( x ) is odd function f ( − x ) = −f ( x ). I2 = ∫ log ⎜
⎩ ⎟ dx
−2 ⎝ 5+ x ⎠
Let
Proof: Let
⎛ 5− x ⎞
a 0 a g( x ) = log ⎜ ⎟
I = ∫ f ( x )dx = ∫ f ( x )dx + ∫ f ( x )dx ⎝ 5+ x ⎠
−a −a 0 Now
I = I1 + I2 æ 5 - (- x ) ö 5- x
g( - x ) = log ç ÷ = - log = - g( x )
0 è 5 + (- x ) ø 5+ x
I1 = ò f ( x )dx
-a
Therefore, g(x) is an odd function.
Therefore,
Put 2
x = −t ⇒ dx = −dt
x = -a Þ t = a , x = 0 Þ t = 0
∫ g( x )dx = 0 ⇒ I
−2
2 =0
0 a I = I1 + I2 = p/2 + 0 = p/2
I1 = - ò f ( -t )dt = ò f ( -t )dt
⎛ e x − 1⎞
log2
∫
a 0
Illustration 23.30 Evaluate sin ⎜ x ⎟ dx.
⎝ e + 1⎠
a
I1 = ò f ( - x )dx log1/2
0 Solution:
a
⎛ e − x − 1⎞ ex −1
I = ∫ (f ( x ) + f ( − x )) dx f ( − x ) = sin ⎜ − x ⎟ = − sin x = −f ( x )
0
⎝ e + 1⎠ e +1
æ e x - 1ö
log2
4
x2 ò sin ç x ÷ dx = 0
Illustration 23.28 Evaluate ∫ (x
−4
2
+ 16)(1+ e x )
5
dx - log2 è e + 1ø
Solution:
x 3 + | x | +1
1
4
x2 Illustration 23.31 Evaluate òx + 2 | x | +1
dx.
∫ (x
2
I= dx
( )
5 -1
−4
2
+ 16) 1+ e x Solution:
4 ì
ïæ ö üï
5
x2 x 2e x 1
x3 | x | +1
1
I = ò íç 2 + I=ò dx + ò 2
ç ( x + 16)(1+ e x 5 ) ( x 2 + 16)(1+ e x 5 ) ÷÷ ý
dx dx
ïè
-4 î ø þï -1
x + 22
| x | +1 -1
x + 2 | x | +1
Chapter 23 | Definite Integration 1027
1
x +1 1/2 ⎛ 1− x ⎞
∫
1
= 0 + 2∫ dx = ln( x 2 + 2 x + 1) 0 Illustration 23.35 Evaluate cos x ⋅ ln ⎜ ⎟ dx .
0
x2 + 2x +1 −1/2
⎝ 1+ x ⎠
Solution: Let
= ln 4
1/2 ⎛ 1− x ⎞
p /2
I=∫ cos x ⋅ ln ⎜ ⎟ dx
Illustration 23.32 If I1 = ∫
0
ln(sin x )dx and −1/2
⎝ 1+ x ⎠
p /4 ⎛ 1− x ⎞
I1 f ( x ) = cos x ⋅ ln ⎜
I2 = ∫ ln(sin x + cos x ) dx, then find . ⎝ 1+ x ⎟⎠
− p /4 I2
⎛ 1+ x ⎞
f ( − x ) = cos x ⋅ ln ⎜ = −f ( x )
Solution: ⎝ 1− x ⎟⎠
p /4
⇒ f ( x ) + f (− x ) = 0
I2 = ∫
−p /4
ln(cos x + sin x )dx 1/2
Using
⇒I = ∫ (f ( x ) + f ( − x ))dx =0
0
a a
b 1
∫ f ( x )dx = ∫ (f ( x )dx + f (− x )dx )
−a 0
7. ∫ f ( x )dx = (b − a)∫ f ((b − a) x + a)dx
a 0
p /4 p /4 p /4
⇒ I2 = ∫ ln(cos x − sin x )dx = ∫ lncos2 x dx = ∫ lnsin2 x dx Proof: Let
2 2
0 0 0
1
∫ lnsin x dx
b b
⇒ 2I 2 = f (t )
I = (b − a )∫ dt = ∫ f (t )dt
0
a ( b − a) a
I1 1 b
⇒ 2I2 = I1 ⇒ =
I2 2 I = ∫ f ( x )dx
a
2
−5 ⎛ 2⎞
⎛ ⎛ 1+ x ⎞ ⎞
2/3
1/2 9⎜ x− ⎟
Illustration 23.33 Evaluate ∫−1/2 ⎜ [ x ] + ln ⎜ ∫e dx + 3 ∫ e
( x +5)2 ⎝ 3⎠
⎟ ⎟ dx. Illustration 23.36 Evaluate dx .
⎝ ⎝ 1− x ⎠ ⎠ −4 1/3
⎛ 1+ x ⎞ Solution:
Solution: ln ⎜ ⎟ is an odd function of x as f ( − x ) = −f ( x ),
⎝ 1− x ⎠ I = I1 + I2
therefore
−5
I1 = ∫ e( x +5) dx
2
1/2
I=∫ [ x ]dx + 0 −4
−1/2 1
= ( −5 + 4)∫ e (
( −5+ 4 ) x −4 +5 )
2
0 1/2 dx
I=∫ [ x ]dx + ∫ [ x ]dx 0
−1/2 0
1
I1 = − ∫ e( x −1) dx
2
0 1
I = ∫ −1dx + 0 = −[ x ] −1/2 = −
0
(1)
−1/2 0
2
Again, let
p 2
∫
⎛ 2⎞
Illustration 23.34 Evaluate (1− x 2 )sin x cos2 x dx . 2/3 9⎜ x − ⎟
−p I2 = −3 ∫ e ⎝ 3⎠
dx
Solution: Let 1/3
2
p ⎛ ⎛ ⎛ 2−1⎞ ⎞ ⎛ 2−1⎞ ⎞
⎛ 2 − 1 ⎞ ⎜⎜⎝ ⎜⎝ ⎜⎝
1
⎟ ⎟ x −⎜
I = ∫ (1− x 2 )sin x cos2 x dx ⎟⎟
3 ⎠ ⎠ ⎝ 3 ⎠ ⎟⎠
−p I2 = 3 ⎜ ⎟∫e dx
⎝ 3 ⎠0
f ( x ) = (1− x ) ⋅ sin x ⋅ cos x is an odd function as
2 2
1
= ∫ e( x −1) dx
2
p I2 = −I1
I = ∫ (f ( x ) + f ( − x ))dx = 0
0
⇒ I = I1 − I1 = 0
1028 Mathematics Problem Book for JEE
m, n ∈I , a ∈R + , then nT T
a+nT a
11. ∫mT
f ( x )dx = (n − m)∫ f ( x )dx
0
8. ∫ nT
f ( x )dx = ∫ f ( x )dx
0 Proof: Let
nT
Proof: Let
a+nT
I= ∫ f ( x )dx
I=∫ f ( x )dx mT
nT
Put Put
x = z + mT ⇒ dx = dz
x = z + nT ⇒ dx = dz x = nT ⇒ z = (n − m)T , x = mT ⇒ z = 0
x = nT ⇒ z = 0, x = a + nT ⇒ z = a ( n− m ) T ( n− m ) T
f ( z + nT ) = f ( z ) I= ∫ f ( z + mT ) dz = ∫ f ( z )dz
a 0 0
a
I = ∫ f ( z + nT )dz = ∫ f ( z )dz T
0
0
I = (n − m)∫ f ( x )dx
a
I = ∫ f ( x )dx
0
0
b +nT b
nT T 12. ∫ f ( x )dx = ∫ f ( x )dx
9. ∫ 0
f ( x )dx = n∫ f ( x )dx
0
a+nT a
Proof: Let
Proof: Let b +nT
nT T 2T
T T T T
Put
I = ∫ f ( x )dx + ∫ f ( x + T )dx + ∫ f ( x + 2T )dx + … + ∫ f ( x + (n − 1)T )dx x = z + nT ⇒ dx = dz
0 0 0 0
T T T T x = a + nT ⇒ z = a, x = b + nT ⇒ z = b
= ∫ f ( x )dx + ∫ f ( x )dx + ∫ f ( x )dx + … + ∫ f ( x )dx (upto n times)
0 0 0 0 b b
a+nT T a+T
10. ∫ a
f ( x )dx = n∫ f ( x )dx
0
13. If f(x) is a periodic function with period T, then ∫
a
f ( x ) is
I3 + I1 = 0 2
px 2 × 50 ⎛ px⎞
2
0 0 100 200
= (1+ 1) =
10p p p
Illustration 23.37 Evaluate ∫ | sin x | dx.
0
Your Turn 1
Solution: Let
3 x +1
10p 1. The value of ò dx is
I= ò | sin x | dx 2 x 2 ( x - 1)
0 1 16 1
We know that |sin x| is a periodic function with period p (A) 2 log2 − (B) log −
6 9 6
p p /2 4 1 16 1
I = 10 ∫ | sin x | dx ⇒ I = 20 ∫ sin x dx (C) log − (D) log +
3 6 9 6
0 0 Ans. (B)
p /2
I = −20 cos x 0 ⇒ I = 20 e
2. The value of ∫1
log x dx is
Illustration 23.38 If f(x) is a continuous periodic function (A) 0 (B) 1 (C) e −1 (D) e +1
a+T
with period T, then prove that the integral of I = ∫ f ( x )dx is Ans. (B)
a
∫
a +T a
f ( x )dx = ∫ f ( y + T )dy = ∫ f ( y )dy
a
4. ∫0
cos3 4q dq =
T 0 0
2 1 1 1
0 T
⇒ g(a) = ∫ f ( x )dx + ∫ f ( x )dx + ∫ f ( x )dx = ∫ f ( x )dx
a T (A) (B) (C) (D)
a 0 0 0 3 4 3 6
Hence, g(a) is independent of a. Ans. (D)
8 2 - 3x
Illustration 23.39 If f(x) is a function satisfying f(x + a) + f(x) = 0
5. ò3
x (1+ x )
dx is equal to
c +b
(A) 2 log(3/2e 3 ) (B) log(3 / e 3 )
for all x ⇒ R and constant a, such that ∫ f ( x )dx is independent of
b
(C) 4 log(3 / e 3 ) (D) None of these
b, then find the least positive value of c.
Ans. (A)
1
Solution: We have 6. The value of
∫ x e dx is equal to
2 x
0
f(x + a) + f(x) = 0 for all x ∈ R (1)
(A) e - 2 (B) e + 2 (C) e 2 − 2 (D) e 2
⇒ f(x + a + a) + f(x + a) = 0 [Replacing x by x + a] (2)
Ans. (A)
⇒ f(x + 2a) + f(x+ a) = 0 (3)
dx 2 dx
and I2 = ∫1
2
Subtracting Eqs. (1) from (2), we get 7. Let I1 = ∫ . Then
1
1+ x 2 x
f(x + 2a) − f(x) = 0 for all x ∈ R
⇒ f(x + 2a) = f(x) for all x ∈ R (A) I1 > I2 (B) I2 > I1
So, f(x) is periodic with period 2a. (C) I1 = I2 (D) I1 > 2I2 Ans. (B)
c +b
∫ f ( x )dx t dt dt
tan x cot x
It is given that is independent of b.
b
8. The value of ò1/ e 1+ t 2 ò1/e t (1+ t 2 )
+ =
The minimum value of ‘c’ is equal to the period of f(x), that is, 2a. (A) −1 (B) 1 (C) 0 (D) None of these
Ans. (B)
1− cos p x
200
Illustration 23.40 Evaluate ∫ dx . 3p /4
dx
0
2 9. ∫
p /4 1+ cos x
is equal to
Solution:
1 1
200
px (A) 2 (B) -2 (C) (D) −
I= ∫ sin
2
dx 2 2
0 Ans. (A)
1030 Mathematics Problem Book for JEE
e2 dx ( x 2 - 9) + 2 x (5 - x ) ( - x 2 + 10 x - 9)
10. The value of ò
1 x (1+ ln x )2
is f ¢( x ) =
(9 - x 2 )2
=
(9 - x 2 )2
(A) 2/3 (B) 1/3 Þ f ¢( x ) = 0 Þ x = 1, 9
(C) 3/ 2 (D) ln2
5 1 3
Ans. (A) Þ f (0) = , f (1) = , f (2) =
9 2 5
23.6 Properties Based on Inequality The maximum and minimum value of f(x) in x Î[0 ,2] is f (1) and
f (2).
14. If functions A (x), B (x) and C (x) are continuous in x Î[a, b] and 2 2
5− x
2
òe dx £ ò 1dx Þ òe dx £ 2p
a xi a xi
1/2
1 dx p
⇒ ≤
2 ∫
0 1− x 2 n
≤
6
0 0 0
2p
eaxi | e 2api - 1|
15. If m and M are the smallest and greatest values of a function £ 2p Þ £ 2p Þ| e 2api - 1| £ 2p | a |
ai 0
|a |
A (x) on an interval [a, b], then
b
m(b - a) £ ò A( x )dx £ M(b - a) 17. If A2 ( x ) and B 2 ( x ) are integral on [a, b], then
a
(∫ A ( x )dx ) (∫ B ( x )dx )
1/2 1/2
Illustration 23.42 Prove that 2 ≤ ∫ 2 + x 5 dx ≤ 2 3 . b b b
∫ A( x )B( x )dx ≤ 2 2
−1 a a a
Solution: Let
f (x) = 2 + x5 23.7 Newton–Leibnitz Rule
f(x) is increasing function for all defined real value and the
minimum and maximum value of f(x) are respectively 1 and 1. If f(x) is continuous and p(x), q(x) are differentiable functions in
the interval [a, b], then
3 in x Î[ -1, 1] . Therefore,
1 1 1 d q( x ) d d
dx ∫p( x )
f (t )dt = f {q( x )} {q( x )} − f ( p( x )} { p( x )}
∫ 1 dx ≤ ∫ f ( x ) dx ≤ ∫
−1 −1 −1
3 dx dx dx
1 2. If f(x) is continuous and p(x), q(x) are differentiable functions at
⇒ 1(1+ 1)dx ≤ ∫ f ( x ) dx ≤ 3 (1+ 1) a point x ∈(a, b ) and f ( x , t ) is continuous, then
−1
d ⎡ q( x ) q( x ) d
f ( x , t )dt ⎤ = ∫
1
Þ 2 £ ò 2 + x 5 dx £ 2 3 ⎢
dx ⎣ ∫p ( x ) ⎥
⎦ p ( x ) dx
f ( x , t )dt
-1
⎧ d q( x ) ⎫ ⎧ d p( x ) ⎫
+⎨ ⎬ f ( x , q( x )) − ⎨ ⎬ f ( x , p( x ))
Illustration 23.43 Prove that 1 £ ò 5 - x2 dx £ 6 .
2
⎩ dx ⎭ ⎩ dx ⎭
0 9- x 5
Solution: Let x
Illustration 23.45 Let f ( x ) = ò 2 - t 2 dt . Then find the real
5− x 1
f (x) = roots of the equation x 2 - f ¢( x ) = 0.
9 − x2
Chapter 23 | Definite Integration 1031
f ′( x ) = 2 − x 2 ⋅1− 2 − 1 ⋅ 0 = 2 − x 2 b a
x = f ′( x ) ⇒ x = 2 − x
2 2 2 ∫ f ( x )dx is a limiting case of the summation of an infinite series,
a
x 4 + x 2 − 2 = 0 ⇒ ( x 2 + 2)( x 2 − 1) = 0 provided f(x) is continuous on [a, b], that is,
b n−1
x = ±1
x
∫ f ( x )dx = lim h∑ f (a + rh),
a
n→∞
r =0
Illustration 23.46 If f ( x ) = cos x − ∫ ( x − t )f (t )dt, then calculate where
0 b−a
f ′′( x ) + f ( x ). h=
n
The converse is also true, that is, if we have an infinite series of the
Solution:
above form, it can be expressed as a definite integral.
æx ö
f ¢( x ) = - sin x - çç ò f (t )dt + xf ( x ) ÷÷ + xf ( x )
è0 ø 23.8.1 Method to Express the Infinite Series as
x
Definite Integral
f ′( x ) = − sin x − ∫ f (t )dt
1 ⎛r⎞
∑ n f ⎜⎝ n ⎟⎠ .
0
f ′′( x ) = − cos x − f ( x ) 1. Express the given series in the form
f ′′( x ) + f ( x ) = − cos x 1 ⎛r⎞
2. Then the limit is its sum when n → ∞ , that is, Lim ∑ f ⎜ ⎟ .
n→∞ n ⎝n⎠
x
Illustration 23.47 Let f : (0, ∞ ) → R and f ( x ) = ∫ f (t )dt. If 3. Replace
r
by x and
1
by dx and Lim
n→∞
∑ by the sign of ∫ .
0 n n
f ( x 2 ) = x 2 (1+ x ), then find f (4)? 4. The lower and the upper limit of integration are the limiting
r
Solution: By definition of f(x) we have values of for the first and the last term of r, respectively.
n
x2 Some particular cases of the above are
f ( x 2 ) = ∫ f (t )dt = x 2 + x 3
0 n-1 1
n
1 ærö 1 ærö
Differentiate both sides, (A) lim
n®¥
å
r =1 n
f ç ÷ or lim å f ç ÷ = ò f ( x )dx
è nø n®¥
r =0 n
è nø 0
f ( x 2 )⋅ 2x + 0 = 2x + 3x 2
b
Put pn
1 ærö
(B) lim å f ç ÷ = ò f ( x )dx
x = 2 ⇒ 4 f (4) = 16 ⇒ f (4) = 4 n®¥
r =1 n è n ø a
1 1 1
Solution: Illustration 23.49 If Sn = + + "+ , then
a
f (t ) 1 + n 2 + 2n n + n2
g( x ) = ∫ dt lim Sn is equal to.
x
t n→∞
g(a) = 0 Solution:
Differentiate w.r.t. x n n
1 1
f (x) lim Sn = lim ∑ = lim ∑
g′( x ) = − ⇒ f ( x ) = − xg′( x )
n→∞ n→∞
r =1 r + rn n→∞
r =1 ⎡r r⎤
x n⎢ + ⎥
a a ⎣ n n ⎦
∫ f ( x )dx = − ∫ xg′( x )dx Now
0 0 1
a
a
a a = lim =0
ò f ( x )dx = - xg( x ) + ò g( x )dx
n→ ∞
n
0 r
0
a a
0
and b = lim = 1
n→∞ n
I = ln2 n −1
In = In−2
n
1/ n
Illustration 23.51 Evaluate lim ⎛⎜ nn! ⎞⎟ . 6. If In = ∫
p /2
cosn x dx , then
n→∞ n
⎝ ⎠ 0
Solution: Let n −1
In = In−2
1/ n
n
(n !)1/n ⎛ 1⋅ 2 ⋅ 3… n ⎞
A = lim ⇒ A = lim ⎜ ⎟⎠
n→∞ ⎝
p /2
n→∞ n nn 7. If In = ∫ x n sin x dx , then
0
1/ n
⎛ 1 2 3 n⎞ In + n(n − 1)In−2 = n(p / 2)n−1
⇒ ln A = lim ln ⎜ ⋅ ⋅ … ⎟
n→∞ ⎝ n n n n⎠
p /2
1 n ⎡ ⎛ r ⎞⎤ 8. If In = ∫ x n cos x dx , then
⇒ ln A = lim ∑ ⎢ln ⎜ ⎟ ⎥
0
n→∞ n
r =1 ⎣ ⎝ n ⎠ ⎦ In + n(n − 1)In−2 = (p / 2)n
1
⇒ ln A = ∫ ln x dx = [ x ln x − x ]0
1
∞ b
∫ e − a x sin bx dx = −
0
9.
1
0 a2 + b 2
⇒ ln A = −1⇒ A =
e ∞ a
10. ∫
0
e − a x cos bx dx =
a2 + b 2
1 + 2 + 3 + …+ n
p p p p
Illustration 23.52 Evaluate lim ( p > 0).
n→∞ n p+1 ∞ n!
Solution: 11. ∫
0
e − a x x n dx =
a2 + 1
1p + 2 p + 3p + … + n p
p
n
rp n
1 ⎛r⎞
lim = lim ∑ = lim ∑ ⋅ ⎜ ⎟
n→∞ n p +1 n→∞
r =1 n ⋅ n
p n→∞
r =1 n
⎝ n⎠ Illustration 23.53 Determine a positive integer n ≤ 5, such that
1
Now
∫e ( x − 1)n dx = 16 − 6e
x
1
a = lim = 0 0
n→∞ n
and Solution: Let
1
r
b = lim = 1 In = ∫ e x ( x − 1)n dx
n→∞ n
0
1
x p+1 Integrating by parts,
1
1
⇒ ∫ x dx =
p
= 1
0 p +1 0 p +1 In = e x ( x − 1)n 0 − nIn−1 = −( −1)n − nIn−1
Chapter 23 | Definite Integration 1033
∞
4. To find the numerical value of ∫−2
( px 2 + qx + s ) dx , it is necessary
∫
n−1 − x
x e dx, n > 0 is called Gamma function and denoted by Γn. to know the values of constants
0
where Γ(n) is called gamma function which satisfy the following (A) 100 2 (B) 200 2
properties:
(C) 50 2 (D) None of these Ans. (B)
Γ(n + 1) = nΓ(n) = n !
1 ⎛ 1+ x ⎞
That is, 6.
∫ −1
log ⎜
⎝ 1− x ⎟⎠
dx =
Γ (1) = 1 , Γ(1/ 2) = p
(A) 2 (B) 1 (C) 0 (D) p
In place of gamma function, we can also use the following formula
Ans. (C)
sinm x cosn x dx = ( m - 1)( m - 3)"(2 or 1)(n - 1)(n - 3)¼(2 or 1)
p /2
∫ 7. If ∫
4
f ( x ) dx = 4 and ∫
4
(3 − f ( x )) dx = 7, then the value of
( m + n)( m + n - 2)¼(2 or 1)
0
−1 2
−1
It is important to note that we multiply by (p/2); when both m
and n are even.
∫ 2
f ( x ) dx is
p
Additional Solved Examples ⇒I= 4
2p
ò 0 dx + ò dx + ò 0 dx + ò ( -1) dx + ò ( -2) dx + ò ( -1) dx Solution: If the given function be F(x), then clearly
0 p
6
5p
6
p 7p
6
11p
6
F(–x) = –F(x)
= -p and hence
Hence, the correct answer is option (B). I=0
Hence, the correct answer is option (D).
1
æpxö æ 1ö 2a
2. If f(x) = a sin ç + b , f 1 ç ÷ = 2 and ò f ( x )dx = , then
è 2 ÷ø è 2ø 0
p 5. Let f: R → R be a differentiable function and f(1) = 4. Then the
f(x)
2t
x ®1 ò x - 1
p p 2 3 value of lim dt is
(A) a = ,b= (B) a = ,b=
2 2 p p 4
[f ( x )]2 - 4 2 æ0 ö
æ 1ö p 1 4 = lim çè form÷ø
Þ f ¢ç ÷ = a ´ = 2 Þa= ; x ®1 x -1 0
è 2ø 2 2 p
2f ( x ) × f ¢( x )
= lim
1
é -2a ù
1
æ px ö 2a 2a [By applying L’ Hospital rule)
ò0 f ( x )dx = êë p cos çè 2 ÷ø + bx úû = b + p = p (given) x ®1 1
0
= 2f (1) f ′(1) = 8f ′(1)
Therefore,
Hence, the correct answer is option (A).
b=0
1.5
The only choice with a = 4/p.
ò [x
2
6. The value of ] dx is.
Hence, the correct answer is option (D). 0
Solution: For
p /2 dx
3. ò 0 1+ tan3 x
= 0 ≤ x < 1, 0 ≤ x2 < 1
(A) 0 (B) 1 and
[ x2 ] = 0
p p
(C) 2 (D) 4 1 £ x < 2 , 1 £ x 2 < 2 and [ x 2 ] = 1
p 0 0 1 2
p
tan3 x p
I= ò 0
2
1+ tan3 x
dx ⇒ 2I = ò 2
0
1 dx = 2
= 2 − 1+ 2(1⋅ 5 − 2 ) = 2 − 2
Chapter 23 | Definite Integration 1035
2 x
ì 1 1 1 ü g (2) = ∫ f (t ) dt = ∫ f (t ) dt + ∫ f (t )dt
9. The value of lim í + + ¼+ ý is equal to 0 0 1
x ®¥ ( x + 1) ( x + 2) ( x + 5x )þ
î b
Using m (b – a) ≤ ∫ f (t ) dt ≤ M(b − a), we get
(A) ln 2 (B) ln 3 a
Solution: Let 1
sin x cos x
1
e2
3. Let I = ò dx and J = ò dx . Then which one of the
loge x x x
I=ò
0 0
dx following is true?
e -1
x
2 2
æ1 ö (A) I> and J > 2 (B) I < and J < 2
Note x > 0 for x ∈ ç , e 2 ÷ 3 3
èe ø
2 2
and (C) I < and J > 2 (D) I > and J < 2
3 3
1 [AIEEE 2008]
loge x is < 0 for <x<1
e Solution:
and 1
1
sin x
1
x
1
2 2 2
for 1 < x < e2, logex > 0 I=∫ dx < ∫ dx = ∫ xdx = x 3/2 = ⇒ I <
Therefore, x x 3 0 3 3
0 0 0
1 1
e2 cos x 1
−1
1 1
I = −∫
loge x
dx + ∫
loge x
dx = + 2 =
3 J=ò dx < ò dx = 2 x = 2
x x 2 2 0 x 0 x 0
1/ e 1
x p p
dt p (A) log2 (B) log2
2. The solution for x of the equation òt =
t -1 2
2
is 8 2
2
(C) log 2 (D) p log2
(A) 2 (B) p
[AIEEE 2011]
3 Solution: We have
(C) (D) 2 2
2 1
log(1+ x )
[AIEEE 2007] I = 8∫ dx
Solution: 0 1+ x 2
x
Put x = tanq, then
p dt
∫ t t2 −1 2 =
That is,
dx = sec2q dq
2
p
p p p 3p
⇒ [sec −1 t ]x 2 =
⇒ sec −1 x − = ⇒ sec −1 x = log(1+ x ) log(1+ tanq )
1 4
2 4 2 4 I = 8∫ dx = 8 ∫ sec2 q dq
1 + x 2
1+ tan2 q
⎛ p⎞ 0
p 0
⇒ x = sec ⎜ p − ⎟ = − sec ⇒ x = − 2 p
⎝ 4⎠ 4 4
log(1+ tanq )
= 8∫ sec2 q dq
There is no correct option for this question. 0 sec2
Chapter 23 | Definite Integration 1037
p p p
4
⎛ ⎛p ⎞⎞
4
⎛ 1− tanq ⎞ (A) 4 3 − 4 (B) 4 3 − 4 −
= 8 ∫ log ⎜ 1+ tan ⎜ − q ⎟ ⎟ dq = 8 ∫ log ⎜ 1+ q
dq 3
0
⎝ ⎝ 4 ⎠ ⎠ 0
⎝ 1+ tanq ⎟⎠ 2p
p p
(C) p − 4 (D) −4−4 3
4 4
3
= 8 ò log2 dq - 8 ò log(1+ tanq )dq [JEE MAIN 2014 (OFFLINE)]
0 0 Solution: See Fig. 23.2.
p
= 8log2 - I Þ 2I = 2p log2 Þ I = p log2 p x x æ
p
xö
2
4 ò 0
1+ 4 sin2
2
- 4 sin dx = ò ç 1- 2sin ÷
2 è 2 ø
Hence, the correct answer is option (D). 0
p
x p x ⎛ x⎞ p ⎛ x⎞
6. If g( x ) = ò0 cos 4t dt , then g( x + p ) equals = ∫ 1− 2sin dx = ∫ 3 ⎜ 1− 2sin ⎟ dx + ∫p − ⎜ 1− 2sin ⎟ dx
0 2 0 ⎝ 2⎠ ⎝ 2 ⎠
3
p /3
dx
7. Statement-I: The value of the integral ∫ 1+
p /6 tan x
is equal
p
to .
6
b b π
Statement-II: ò f ( x )dx = ò f (a + b - x ) dx . π 5π
a a
3 3
(A) Statement-I is True; Statement-II is true; Statement-II is not
a correct explanation for Statement-I.
(B) Statement-I is True; Statement-II is False.
(C) Statement-I is False; Statement-II is True.
(D) Statement-I is True; Statement-II is True; Statement-II is a Figure 23.2
correct explanation for Statement-I.
[JEE MAIN 2013] Hence, the correct answer is option (B).
Solution: We have 1
ln(1+ 2 x )
p /3
dx
p /3
dx
p /3
dx
9. The integral ò 0
2
1+ 4 x 2
dx , equals
I= ∫ 1+
p /6
= ∫
tan x p /6 ⎛p p ⎞
= ∫
p /6 ⎛p ⎞ p p
1+ tan ⎜ + − x ⎟ 1+ tan ⎜ − x ⎟ (A) ln2 (B) ln2
⎝6 3 ⎠ ⎝2 ⎠ 4 8
p /3
dx p p
ln2 ln2
= ∫ 1+
p /6 cot x
(C)
16
(D)
32
[JEE MAIN 2014 (ONLINE SET-1)]
p /3
dx
p /3
tan x
p /3 1+ tan x ( ) Solution:
I= ò Þò dx Þ 2I = ò dx
p /6 1
1+ p /6 1+ tan x p /6 1+ tan x ( ) 1
ln(1+ 2 x )
tan x ∫0
2
1+ 4 x 2
dx
1 ép p ù p Put 2x = y. Therefore,
ÞI = ê - úÞI =
2ë3 6û 12 2dx = dy
Hence, the correct answer is option (C). Therefore, integration changes to
p
x x 1 1 ln(1+ y )
2 ∫0 1+ y 2
I=
8. The integral ∫
0
1+ 4 sin2
2
− 4 sin dx equals
2
dy
1038 Mathematics Problem Book for JEE
Now, put
y = tan q ⇒ dy = sec2q dq
1 p ln(1+ tanq )
I= ò4 sec2q dq 1
2 0 sec2 q
1 p4 ⎛ ⎛p ⎞⎞ 1 p 1− tanq ⎞
I= ∫ ln ⎜ 1+ tan ⎜ − q ⎟ ⎟ dq = ∫ 4 ln ⎛⎜ 1+ ⎟ dq π/ 2 π
2 0 ⎝ ⎝4 ⎠⎠ 2 0 ⎝ 1+ tanq ⎠
p /4
1 p4 æ 1+ tanq + 1- tanq ö æ 2 ö
2 ò0 è ò ln çè 1+ tanq ÷ø dq
= ln ç ÷ dq =
1+ tanq ø −1
0
1 p4 1 p
=
2 ò0
ln2dq - ò 4 ln(1+ tanq )dq
2 0
ln2 p4
2 ∫0
= dq − I Figure 23.3
Put Therefore,
log x = t ⇒ et = x
d
dt (∫
−p
t
)
(f ( x ) + x ) dx =
d 2 2
dt
(p − t )
Therefore,
d
⇒ ( f (t ) + t ) t = −2t ⇒f (t) = − 3t
d x = et d t dt
1 1
Therefore,
Pn = ∫ t ne t dt = [t ne t ]10 − ∫ nt n−1e t dt = 1 − n Pn−1
0 0
⎛ −p ⎞ ⎛ −p ⎞
f⎜ = −3 × ⎜ =p
= 1 − n [1 − (n − 1) Pn−2] = 1 − n + n (n − 1) Pn−2 ⎝ 3 ⎟⎠ ⎝ 3 ⎟⎠
⎡ 1 et x +a t
e ⎤ Therefore,
= e − a ⎢ ∫ dt + ∫ dt ⎥
⎣1+a t 1 t ⎦
− log u . 1
x x
⎛ 1⎞ log t
⎡ x +a e t 1+a t
e ⎤ f (x) + f ⎜ ⎟ = ∫ dt − ∫ du
= e − a ⎢ ∫ dt − ∫ dt ⎥ = e−a [F(x + a) − F(1 + a)] ⎝ x ⎠ 1 1+ t 1 1+
1 u2
⎣ 1 t 1 t ⎦ u
Hence, the correct answer is option (D). x x x x
log t log u log t log t
=∫ dt + ∫ du = ∫ dt + ∫ dt
4
log x 2
1 + t u(1 + u ) 1 + t 1 (1+ t )
t
∫ log x
1 1 1
14. The integral dx is equal to
2
2
+ log(36 − 12 x + x 2 ) x x
⎛ log t ⎞ ⎛ 1 ⎞ 1 (log x )2
(A) 4 (B) 1 = ∫⎜ ⎟ ⋅ ⎜ 1+ ⎟ dt = ∫ log t dt =
1 ⎝ 1+ t ⎠ ⎝ t⎠ 1t 2
(C) 6 (D) 2
[JEE MAIN 2015 (OFFLINE)] Hence, the correct answer is option (B).
1 1
Solution: 1
16. If 2 ∫ tan xdx = ∫ cot (1− x + x )dx , then ∫ tan−1(1− x + x 2 )dx
−1 −1 2
4 0 0
log x 2 0
I=∫ dx is equal to
log x + log(36 − 12 x + x 2 )
2
2
p
4 (A) + log2 (B) log2
log x 2 2
=∫ dx
2 log x + log(6 − x )
2 2
p
4
(C) − log4 (D) log4
log x 2
=∫ dx (1) [JEE MAIN 2016 (ONLINE SET-1)]
2 log x + log(6 − x )
Solution: We have
log(2 + 4 − x )
4
⇒ I=∫ dx 1 1
x 1 1
log t ⎛ 1⎞ ⇒ I = ∫ tan−1 xdx = ( x )tan−1 x 0 −
1 2x
2 ∫0 1+ x 2
1
15. For x > 0, let f ( x ) = ∫ dt . Then f ( x ) + f ⎜ ⎟ is equal to dx
1 1 + t ⎝x⎠ 0
1 1 1
(A)
1
(log x )2 (B)
1
(log x )2 = x tan−1 x 0 − log(1+ x 2 )
4 2 2 0
p 1 p 1
(C) log x
1
(D) log x 2 = − (log2 − 0) = − log2
4 4 2 4 2
[JEE MAIN 2015 (ONLINE SET-1)] Hence,
1
Solution: x > 0; p p
∫ tan
−1
(1− x + x 2 )dx = − + log2 = log2
x 1/ x 2 2
⎛ 1⎞ log t log t 0
f (x) + f ⎜ ⎟ = ∫ dt + ∫ 1+ t dt
⎝ x ⎠ 1 1+ t 1 Hence, the correct answer is option (B).
1040 Mathematics Problem Book for JEE
17. The value of the integral 2. Match the Column I to Column II.
10
[ x 2 ]dx Column I Column II
∫ [x 2
− 28 x + 196] + [ x 2 ]
4 1
dx 1 ⎛2⎞
where [x] denotes the greatest integer less than or equal to x, is
(A) ∫ 1+ x
−1
2
(P)
2
log ⎜ ⎟
⎝3⎠
1
(A) (B) 6 1
dx ⎛2⎞
3 (B) ∫ 1− x 2
(Q) 2log ⎜ ⎟
⎝3⎠
(C) 7 (D) 3 0
10 (B) ã (S)
2I = ∫ 1 dx = 6 ⇒ I = 3 1
dx
4 I=∫
Hence, the correct answer is option (D). 0 1− x 2
p p
= [sin−1 x ]10 = −0=
Previous Years’ Solved JEE Advanced/ 2 2
(C) ã (P)
IIT-JEE Questions 3
dx
I=∫
sec2 x 2 1 − x2
∫ f (t ) dt
1⎡ 1+ x ⎤
3
2
= ⎢log
1− x ⎥⎦ 2
1. limp equals
x→ p2 2⎣
4 x2 −
16 1⎡ 4 3⎤
= ⎢log − log ⎥
8 2 2⎣ 2 1⎦
(A) f (2) (B) f (2)
p p 1 ⎛2⎞
= log ⎜ ⎟
2 ⎛ 1⎞ 2 ⎝3⎠
(C) f⎜ ⎟ (D) 4f(2)
p ⎝ 2⎠ (D) ã (R)
[IIT-JEE 2007]
2
dx
I=∫
Solution: We have 1 x x −1
2
sec2 x
= [sec −1 x ]12
∫ f (t )dt
⎛0 ⎞ p p
lim 2
⎜ form ⎟ = −0=
x→
p x − (p / 16)
2 2
⎝0 ⎠ 3 3
4
Hence, the correct matches are (A)ã(S); (B)ã(S); (C)ã(P);
Using L’ Hospital’s rule, we get
(D)ã(R).
f (sec2 x ) ⋅ (2sec x ) ⋅ (sec x ) ⋅ (tan x ) f (2) ⋅ 2 ⋅ 2 ⋅ 2 ⋅1 8f (2)
lim = = 3. Let f be a non-negative function defined on the interval [0, 1].
x→
p 2x 2 ⋅ (p / 4) p x x
4
also,
(A) f ⎛⎜ ⎞⎟ < and f ⎛⎜ ⎞⎟ >
1 1 1 1
⎝ 2⎠ 2 ⎝ 3⎠ 3 f ’(x) = f(x), x > 0
⇒ f(x) = k, x > 0
⎛ 1⎞ 1 ⎛ 1⎞ 1
(B) f ⎜ ⎟ > and f ⎜ ⎟ > Since, f(0) = 0 and f(x) is continuous, so
⎝ 2⎠ 2 ⎝ 3⎠ 3
f(x) = 0 ∀ x > 0
⎛ 1⎞ 1 ⎛ 1⎞ 1
(C) f ⎜ ⎟ < and f ⎜ ⎟ < Therefore, f (ln 5) = 0.
⎝ 2⎠ 2 ⎝ 3⎠ 3
Hence, the correct answer is (0).
(D) f ⎛⎜ ⎞⎟ > and f ⎛⎜ ⎞⎟ <
1 1 1 1
t ln(1+ t )
x
⎝ 2⎠ 2 ⎝ 3⎠ 3 1
[IIT-JEE 2009]
6. The value of lim
x →0 x3 ∫
0 t +4
4
dt is
Solution: 1
(A) 0 (B)
x x 12
∫ 1− ( f ′(t ))2 dt = ∫ f ′(t )dt 1 1
0 o (C) (D)
24 64
⇒ 1− ( f ′( x ))2 = f ( x ) [IIT-JEE 2010]
⇒ f ′ ( x ) = ± 1 − ( f ( x )) 2 Solution:
t ln(1+ t )dt ⎛ 0
x
⇒ f ( x ) = sin x or f ( x ) = − sin x (not possible) 1 ⎞
lim
x →0 x3 ∫ t4 + 4
= ⎜ form ⎟
⎝0 ⎠
⇒ f ( x ) = sin x 0
x 4 (1− x )4
10 1
1
Now, ⎛ 4 ⎞
p = ∫ ⎜ x 6 − 4 x 5 + 5x 4 − 4 x 2 + 4 − ⎟ dx
sin(n + 2) x − sin nx 0 ⎝ 1 + x2 ⎠
I n+ 2 − I n = ∫ dx
sin x 1
0
⎡ x7 2x6 4 x3 ⎤
p =⎢ − + x5 − + 4x⎥ −p
2cos(n + 1) x ⋅ sin x ⎣ 7 3 3 ⎦0
=∫ dx = 0
0
sin x 1 2 4 22
p = − + 1− + 4 − p = −p
7 3 3 7
⇒ I1 = p , I2 = ∫ 2cos xdx = 0
0 Hence, the correct answer is option (A).
Hence, the correct answers are options (A), (B) and (C).
ln3
x sin x 2
5. Let f: R → R be a continuous function which satisfies
x
8. The value of ∫ sin x + sin(ln6 − x 2 ) dx is
2
ln2
f ( x ) = ∫ f (t ) dt . Then the value of f(ln 5) is _____.
0
1 3 1 3
[IIT-JEE 2009] (A) ln (B) ln
4 2 2 2
Solution:
x 3 1 3
(C) ln (D) ln
f ( x ) = ∫ f (t )dt ⇒ f (0) = 0 2 6 2
0 [IIT-JEE 2011]
1042 Mathematics Problem Book for JEE
Solution: Put
p2
5/6
p
x 2 = t ⇒ 2 x dx = dt (C) The value of ∫ sec(px )dx is
ln3 7/6
(R)
3
sin(ln6 − t )
ln3 ln3
1 sin t 1
I= ∫ sin t + sin(ln6 − t ) dt ⇒ I = 2 ln2∫ sin(ln6 − t ) + sin t dt
2 ln2 (D) The maximum value of ⎛ 1 ⎞
Arg ⎜
(S) p
⎝ 1− z ⎟⎠
1
ln3
1 3 for z = 1, z ≠ 1 is given by
2I = ∫
2 ln2
1dt ⇒ I = ln
4 2
p
(T)
Hence, the correct answer is option (A). 2
9. Let f : [1, ∞ ) → [2, ∞ ) be a differentiable function such that [IIT-JEE 2011]
x Solution:
f(1) = 2. If 6 ∫ f (t ) dt = 3 xf ( x ) − x 3 for all x ≥ 1, then the value of
1
(A)
f(2) is _____.
a ⋅ b = −1+ 3 = 2
[IIT-JEE 2011]
a = 2, b = 2
Solution:
2 1
x cosq = =
2×2 2
6 ∫ f (t )dt = 3 x f ( x ) − x ⇒ 6f ( x ) = 3f ( x ) + 3 xf ′( x ) − 3 x
3 2
1 p 2p 2p
q= , but is opposite to the side of the maximum
⇒ 3f ( x ) = 3 xf ′( x ) − 3 x ⇒ xf ′( x ) − f ( x ) = x
2 2 3 3 3
length.
dy dy 1
⇒x − y = x2 ⇒ − y=x (1) b
dx dx x
∫ (f ( x ) − 3 x )dx = a − b2
2
1
(B)
∫ − x dx − loge x a
I.F = e =e
−a2 + b 2
b
3 2 2
∫a = − + − =
2 2
1 f ( x )dx ( b a ) a b
Multiplying both sides of Eq. (1) by 2 2
x ⇒ f (x) = x
1 dy 1 d ⎛ 1⎞
− y = 1⇒ ⎜ y . ⎟ = 1 p 2 ⎛ ln (sec p x + tanp x ) 7/6 ⎞
5/6
x dx x 2 dx ⎝ x ⎠ (C) ⎜ ⎟
ln3 ⎜ p ⎟
Integrating ⎝ ⎠
p ⎛ 5p 5p 7p 7p ⎞
y
= x +c = ⎜ ln sec + tan − ln sec + tan ⎟=p
x ln3 ⎝ 6 6 6 6 ⎠
Put x = 1, y = 2 (D) Let
⇒ 2 = 1+ c ⇒ c = 1 ⇒ y = x 2 + x 1 1
u= ⇒ z = 1−
⇒ f ( x ) = x 2 + x ⇒ f (2) = 6 1− z u
1
Note: If we put x = 1 in the given equation we get f(1) = 1/3. z = 1 ⇒ 1− = 1
u
Hence, the correct answer is (6**).
⇒ u −1 = u
**Question is ambiguous as 8/3 can also be the answer.
Therefore, locus of u is perpendicular bisector of line segment
10. Match the statements given in Column I with the values given p
joining 0 and 1, so maximum arg u approaches but will not
in Column II. attain. 2
Column I Column II Hence, the correct matches are (A)ã(Q); (B)ã(P); (C)ã(S);
ˆ (D)ã(T).
(A) If a = j + 3kˆ , b = − ˆj + 3kˆ and c = 2 3kˆ (P) p
form a triangle, then the internal angle of 6
p /2
⎛ 2 p +x⎞
the triangle between a and b is 11. The value of the integral ∫
−p /2
⎜ x + ln
⎝
⎟ cos x dx is
p −x⎠
b
2p
(B) If ∫ (f ( x ) − 3 x )dx = a2 − b 2 , then the value (Q) p2
a
3 (A) 0 (B) −4
2
⎛p ⎞ p2 p2
of f ⎜ ⎟ is +4
⎝ 6⎠ (C) (D)
2 2 [IIT-JEE 2012]
Chapter 23 | Definite Integration 1043
−2 x dy
(B)
∫0
2(e u + e − u )17 du
e < 2 ye −2 x log(1+ 2 )
d
dx (C)
∫0
2(e u − e − u )17 du
( ye −2 x ) < 0 log(1+ 2 )
dx (D)
∫ 0
2(e u − e − u )16 du
p 1-h 1 1 1-h
when x = , = lim+ ò dt = lim+ ò dt
2 h®0 h
t -t 2 h®0
æ 2 h
1 1ö
p p -çt - t + - ÷
cosec + cot = e t è 4 4ø
2 2 1-h 1 1-h 1
Therefore, = lim+ ò dt = lim+ ò dt
h®0 h 2 h®0 h 2 2
1–0=e t
ì 1ü 1 æ 1ö æ 1ö
- ít - ý + ç ÷ - çt - ÷
⇒t = 0 î 2þ 4 è2ø è 2ø
Therefore, 1-h
é æ t - 1/ 2 ö ù
[sin-1(2t - 1)]h
1-h
Also cosec2 x – cot2 x = 1 = lim+ êsin-1 ç ÷ ú = hlim
h®0
ë è 1/ 2 ø ûh ® 0+
1
⇒ cosec x − cot x = = e −t = lim[sin-1
{2 - 2h - 1} - sin-1(2h - 1)]
et h®0+
Therefore,
= lim+ + sin-1(1- 2h) - lim+ + sin-1(2h - 1)
2 cosec x = et + e–t h®0 h®0
2 3x2 3. 4 = ∫ 3 x 2 dx = 8
R. ∫−2 (1+ e x )
dx equals 0
Hence,
⎛ 1 4. 0 (R) → (1)
⎛ 1+ x ⎞ ⎞
⎜ ∫−21 cos2 x log ⎜ ⎟ dx ⎟
⎝ 2 ⎝ 1− x ⎠ ⎠ For (S) in List I: Let
S. equals ⎛ 1+ x ⎞
⎛ 1 ⎛ 1+ x ⎞ ⎞ f ( x ) = cos 2 x ln ⎜
⎜ ∫02 cos2 x log ⎜ ⎟ dx ⎟ ⎝ 1− x ⎟⎠
⎝ ⎝ 1− x ⎠ ⎠
⇒ f ( − x ) = cos 2 x ln ⎛⎜ 1− x ⎞⎟
P Q R S ⎝ 1+ x ⎠
(A) 3 2 4 1 ⎡ ⎛ 1+ x 1 − x ⎞ ⎤
⇒ f ( x ) + f ( − x ) = cos 2 x ⎢ln ⎜ × ⎟⎥ = 0
(B) 2 3 4 1 ⎣ ⎝ 1 − x 1+ x ⎠ ⎦
(C) 3 2 1 4
Therefore, f (x) is an odd function. Thus,
(D) 2 3 1 4 1
⎛ 1+ x ⎞
∫ dx = 0
2
cos 2 x ln ⎜
[JEE ADVANCED 2014] − 21 ⎝ 1− x ⎟⎠
2
æ pö 2
x (1) ⎡ x2 ⎤ 1
= 2 sin ç x 2 + ÷
è 4ø
=0+0+ ∫1 2
dx = ⎢ ⎥ =
4
⎣ ⎦1 4
⇒ 4I – 1 = 0
For maximum value of f (x), we have Hence, the correct answer is (0).
p p p
x2 +
= + 2np ⇒ x 2 = 2np +
1
-1 æ 12 + 9 x 2 ö
19. If α = ò (e 9 x +3tan x ) ç dx , where tan-1 x takes only
è 1+ x 2 ÷ø
4 2 4
0
p 9p
⇒x=± , for n = 0 ; x = ± , for n = 1 ⎛ 3p ⎞
4 4 principal values, then the value of ⎜ loge 1+ a − ⎟ is _____.
Hence, ⎝ 4⎠
(Q) → (3) [JEE ADVANCED 2015]
1046 Mathematics Problem Book for JEE
Solution:
æ 1ö
⇒ f ç ÷ =7
1
9 x +3tan-1 x æ 12 + 9 x 2 ö è 2ø
a = ò (e )ç ÷ dx
è 1+ x ø
2
0 ⎛ 1⎞
Here ⇒ f ⎜ ⎟ = ±7,
⎝2⎠
æ p pö
tan-1 x Î ç - , ÷ but being odd function and continuous f(0) = 0, thus if
è 2 2ø
Put ⎛ 1⎞
f ⎜ ⎟ = −7,
1 ⎝ 2⎠
tan−1 x = t ⇒ dx = dt
1+ x 2 æ1 ö
p /4 then f(x) must have another root in ç ,1÷ . So 7
òe è2 ø
9tant +3 t
Þ a= × (12 + 9 tan t ) × dt
2
0 ⎛ 1⎞
Again, put f ⎜ ⎟ ≠ −7
⎝2⎠
9 tan t + 3t = z
⇒ f ⎛⎜ ⎞⎟ = 7
Þ (9 (sec2t) + 3)dt = dz 1
⇒ (9(tan2t) + 12)dt = dz ⎝2⎠
9+3p /4 ⎛ 3p ⎞ Hence, the correct answer is (7).
⎜ 9+ ⎠⎟
⇒ a= ∫
0
e z dz = e ⎝ 4
−1
21. The option(s) with the value of a and L that satisfy the
⇒ 1+ a = e 9+3p /4 following equation is(are)
⇒ ln |1 + a | = 9 + 3p/4 4p
3p ò e (sin at + cos 4 at )dt
t 6
Þ ln (1+ a ) − =9
4 0
p
=L
Hence, the correct answer is (9).
ò e (sin at + cos at )dt
t 6 4
0
20. Let f : R → R be a continuous odd function, which vanishes
1 x
e −1
4p
e 4p + 1
exactly at one point and f (1) = . Suppose that F ( x ) = ∫ f (t ) dt (A) a = 2, L = (B) a = 2, L =
2 −1
ep − 1 ep + 1
e 4p − 1 e 4p + 1
x
for all x ∈ [ −1,2] and G( x ) = ∫ t f (f (t )) dt for all x ∈ [ −1,2]. If (C) a = 4, L = (D) a = 4, L =
−1 ep − 1 ep + 1
F( x ) 1 ⎛ 1⎞ [JEE ADVANCED 2015]
lim = , then the value of f ⎜ ⎟ is _____.
x →1 G ( x ) 14 ⎝2⎠ Solution: For a = 2,
[JEE ADVANCED 2015] 4p
e t (sin6 2t + cos 4 2t )
Solution: f : R → R is a continuous odd function having a single L= ò p
dt
ò e (sin 2t + cos 2t )dt
0 t 6 4
root.
0
1 x Let
f (1) = , F ( x ) = ò f (t )dt ∀x ∈ [ −1,2] 4p
2 L1 = ∫ e (sin 2t + cos 4 2t )dt
t 6
-1
x 0
G( x ) = ∫ t f (f (t )) dt ∀x ∈[1,2] p 2p 3p 4p
−1 = ∫ f (t )dt + ∫ f (t )dt + ∫ f (t )dt + ∫ f (t )dt = I + I 1 2 + I3 + I 4
F( x ) 1 ⎛ 1 ⎞ 0 p 2p 2p
lim = ,f ⎜ ⎟ = ? In the 2nd integration, put
x →1 G ( x ) 14 ⎝ 2 ⎠
t = p + x ⇒dt = dx
Clearly f(t) and t|f(f(t))| are odd functions for t ∈ ( −1, x ) and
x ∈ [ −1,2]. Therefore, and
t = p ⇒ x = 0, t = 2p ⇒ x = p
F( x ) 1 0 That is,
lim = (limit is of form)
x →1 G( x ) 14 0 p
I2 = ∫ f (p + x )dx
f (x) 1
⇒ lim = 0
x ®1 x | f (f ( x ))| 14 In I3, put
t = 2p + x
f (1) 1
⇒ = ⇒ dt = dx, t = 2p ⇒ x = 0, t = 3p, x = p
1| f (f (1))| 14 p
t = 3p ⇒ x = 0, t = 4p ⇒ x = p 1
p = − ∫ (t 5 − t 3 )dt
⇒ l 4 = ò f (3p + x )dx 0
1
0
⎡t6 t4 ⎤ ⎡ 1 1⎤ 1
Therefore, = −⎢ − ⎥ = ⎢ − ⎥ =
L1 I2 + I2 + I3 + I4 ⎣ 6 4 ⎦ 0 ⎣ 4 6 ⎦ 12
L= =
I1 I1 Hence, the correct answers are options (A) and (B).
I1 + ep I1 + e 2p I1 + e 3p I1 23. The total number of distinct x Î[0, 1] for which
=
I1 x
t2
e 4p - 1 ∫0 1+ t 4 dt = 2 x − 1 is _______.
Þ L = (1+ ep + e 2p + e 2p ) =
ep - 1 [JEE ADVANCED 2016]
Similarly, Solution: See Fig. 23.5. We have x Î[0,1] for which
e −14p x
t2
L= for a = 4
ep − 1 ∫0 1+ t 4 dt = 2 x − 1
Hence, the correct answers are options (A) and (C). Let
x
t2
⎛ p p⎞ F( x ) = ∫ dt
22. Let f(x) = 7 tan8x + 7 tan6x – 3 tan4x – 3 tan2x for all x ∈⎜ − , ⎟ .
0 1+ t
4
⎝ 2 2⎠
Then the correct expression(s) is (are) and
f(x) = 2x – 1
p /4 p /4
1
∫ xf ( x )dx = ∫ f ( x )dx = 0
2
(A) (B) x
12 Now, if F ′( x ) = > 0, it means that F(x) is an increasing function.
0 0 1+ x 4
p /4
1 p /4 F(0) = 0
(C) ∫ xf ( x )dx =
6
(D) ∫ f ( x )dx = 1 1 (t 2 + 1) + (t 2 − 1)
x
2 ∫0
0 0
F( x ) = dt
[JEE ADVANCED 2015] 1+ t 4
Solution: æ æ 1ö æ 1ö ö
xç 1+ ç 1- 2 ÷ ÷
⎛ −p p ⎞ 1 ç çè t 2 ÷ø
+ è ø ÷ dt
t
f(x) = 7 tan8x + 7 tan6x – 3 tan4x – 3 tan2x ∀ x ∈⎜ , = ò
⎝ 2 2 ⎟⎠ 2 0 ç æ 1 ö2 æ 1ö
2
÷
çç ç t - ÷ + 2 ç t + ÷ - 2 ÷÷
= 7 tan6x ⋅ sec2x – 3tan2x ⋅ sec2x èè t ø è tø ø
= (7 tan6x – 3tan2x) ⋅ sec2x æ 1ö æ 1ö
1+ çè 1- 2 ÷ø
1 çè t 2 ÷ø
x dt x
p /4 p /4 1 t
= ò + ò dt
⇒ ∫ f ( x )dx = ∫ (7 tan x − 3tan2 x )sec2 dx
6
2 0 æ 1ö 2 2 0 æ 1ö 2
0 0 çè t - ÷ + 2 çè t + ÷ - 2
1
tø tø
= ∫ (7t 6 − 3t 2 )dt = [t 7 − t 3 ]10 = 0 x
⎧ ⎛ 1 ⎞⎫
0
1 ⎪⎪ −1 ⎛ 1 ⎞ ⎜ t + t − 2 ⎟ ⎪⎪
Also, = ⎨tan ⎜ t − ⎟ + ln ⎜ 1 ⎟⎬
p /4
2 2⎪ ⎝ t⎠ ⎜⎜ t + + 2 ⎟⎟ ⎪
⎩⎪ ⎝ t ⎠ ⎪⎭ 0
I= ∫
0
xf ( x )dx
x
1 ⎧⎪ −1 ⎛ t 2 − 1 ⎞ ⎛ t 2 + 1− 2t ⎞ ⎫⎪
p /4
= ⎨tan ⎜ ⎟ + ln ⎜⎜ 2 ⎟⎟ ⎬
= x × ò (7 tan6 x - 3tan2 x )sec2 x dx
0
2 2 ⎪⎩ ⎝ t ⎠ ⎝ t + 1+ 2t ⎠ ⎪⎭ 0
p /4
p /4 1 p 1 ⎛ x 2 − 1⎞ 1 ⎛ x 2 + 1− 2 x ⎞
F( x ) = + tan−1 ⎜ + ln
=0− ∫ tan x (tan x − 1)dx ⎝ x ⎟⎠ 2 2 ⎜⎝ x 2 + 1+ 2 x ⎟⎠
3 4
2 22 2 2
0
p /4
p 1 1 æ2- 2ö
= − ∫ tan3 x (tan2 x − 1)(sec2 x )dx F (1) = + (0) + ln ç ÷ <1
0
4 2 2 2 2 2 è2+ 2ø
1048 Mathematics Problem Book for JEE
y
(A) 1 (B) 2
1 y = f(x) 1
(C) 0 (D) −
2
x
2. Let f ( x ) = ∫ 2 − t 2 dt . Then the real roots of the equation
1
x 2 − f ′( x ) = 0 are
x
0 1
y = f(x) (A) (0, 1) (B) ±
2
−1 1
(C) ± (D) ±1
2
Figure 23.5 3. Let T > 0, be a fixed real number. Suppose f is a continuous
Now, T
f(x) = 2x – 1 ⇒ f(0) = −1 function such that for all x ∈ R, f(x +T ) = f(x). If I = ∫ f ( x )dx ,
0
and 3+3T
f(1) = 2 – 1 = 1 then value of ∫ f (2 x )dx is
Therefore, the total number of distinct values of x ∈ [0.1] is only 3
one. 3
(A) - I (B) 2I
Hence, the correct answer is (1). 2
p /2 (C) 3I (D) 6I
x 2 cos x
24. The value of ∫ 1+ e x dx is equal to 100
òa
x -[ x ]
− ( p /2) 4. The value of dx is
0
p 2
(B) p + 2
2
(A) −2
4 4 100(a - 1) 100(a + 1)
− ( p /2)
(A) (B)
(C) p − e (D) p + e
2 2 ( p /2)
log a log a
[JEE ADVANCED 2016] (C) 100(a − 1) (D) None of these
x
Solution: The given integral is 5. Value of ∫ [sin t ] dt , (2n + 1)p < x < (2n + 2)p , n ∈N
0
is equal to
p /2
x 2 cos x (A) (n + 1)p p + x (B) np + x
I= ∫ 1+ e x dx
− p /2
(C) np − x (D) (n+1)p − x
p
Using the integral property, we get dx
p /2
6. The value of ∫ 1+ 2 tan x is equal to
⎛ x 2 cos x x 2 cos x ⎞ 0
I= ∫ 0
⎜
⎝ 1+ e
x
+ ⎟ dx
1+ e − x ⎠ (A) 0 (B)
p
4
p /2
(C) p (D) p
I= ∫x
2
cos x dx
0
2
That is, p /4
sec2 xdx
∫x
2
cos x dx = x sin x − 2 ∫ x sin x dx
2 7. ∫
− p /4
1+ e x
equals
{ }
(A) 2 (B) 0
= x 2 sin x − 2 − x cos x + ∫ cos x dx
(C) –1 (D) 1
= x2sinx + 2x cosx – 2sinx a
Therefore, 8. If f(x) = f (a – x), then ∫ x f ( x ) dx equals
p /2 ⎛p 2 ⎞ p2 0
I = x sin x + 2 x cos x − 2sin x 0
2
=⎜ + 0 − 2 ⎟ − (0) = −2
⎝ 4 ⎠ 4 a
a a
2 ∫0
Hence, the correct answer is option (A). (A) x f ( x ) dx (B) a∫ x f ( x ) dx
0
a
Practice Exercise 1 (C) ∫ x f ( x ) dx
0
(D) None of these
1. If f(x) = x – [x]: for every real number of x, when [x] is the integral
e
1
9. If l p = ∫ (ln x )p dx , then l p + pl p−1 is less than
point of x. Then ∫ f ( x )dx is equal to
−1
1
Chapter 23 | Definite Integration 1049
(A) 1 (B) 2 ⎡ 5p 4p ⎤
on the interval ⎢ , is
(C) 3 (D) None of these ⎣ 4 3 ⎥⎦
p 3 3 1
2 (A) 3+ (B) −2 3 + +
2 2
10. If ò ln| tan x + cot x | dx = l ln2, then l equals 2
0 3 1
(C) + (D) None of these
p 2 2
(A) (B) p
2 ∞
(C)
1
(D) None of these
19. ∫0
f ′′( x + a) dx =
e (A) f ′(a) − f ′(b ) (B) f ′(b − c + a) − f ′(a)
p
4
(C) f ′(b + c − a) + f ′(a) (D) None of these
12. ∫ [sin x + cos x ] d( x − [ x ]), [.] greatest integer function, equals x
0 20. The greatest value of the function F ( x ) = ∫ | t | dt on the
1
p p ⎡ 1 1⎤
(A) (B) interval ⎢ − , ⎥ is given by
4 2 ⎣ 2 2⎦
3p 3
(C) (D) None of these 1
4 (A) (B) −
8 2
1 2n r 3 2
13. n®¥ n å 2 2 equals
lim (C) -
8
(D)
r =1 n +r 5
p /2
(A) 5 +1 (B) 5 −1 21. ò- p /2
sin2 x cos2 x (sin x + cos x ) dx =
(C) 2 +1 (D) 2 −1 2 4
(A) (B)
15 15
[x ] 6 8
2x
14. ∫ 2[ ] dx
0
x
equals (where [.] denotes the greatest integer (C)
15
(D)
15
function) ∞ dx
∫ =
( )
22. 0 3
∑ ∫ f (k − 1+ x ) dx
1
27. The value of the integral is 1 d ⎡ −1 ⎛ 2 x ⎞ ⎤
∫
0
36. sin ⎜ 2 ⎟⎥
dx is equal to
dx ⎢⎣
k =1
1
0
⎝ 1+ x ⎠ ⎦
∫ f ( x ) dx
2
(A) 0
(B) ∫
0
f ( x ) dx
(A) 0 (B) p
n
∫
1
(C) 0
f ( x ) dx (D) n∫ f ( x ) dx (C) p /2 (D) p /4
0
a+( p /2)
28. The value of ∫ a
(sin4 x + cos 4 x ) dx is
37. ò
¥ xdx
=
2
0 (1+ x )(1+ x 2 )
(B) a ⎛⎜ ⎞⎟
p
(A) Independent of a (A) 0 (B) p /2
⎝2⎠
(C) p /4 (D) 1
3p 3p a2
(C) (D) a
8 8
38. ∫0
x 4 a2 − x 2 dx =
p ⎛ x⎞ p p 6
29. ∫ 0
sin5 ⎜ ⎟ dx equals
⎝ 2⎠ (A)
32
(B) a
32
16 32 p 6 p 6
(A) (B) (C) a (D) a
15 15 16 8
3
8 5 a
(C)
15
(D)
6
39. ò 0
x (2ax - x 2 ) 2 dx =
⎡ 3p ⎤
(A) a5 ⎡⎢ − 1⎤⎥
3p
ïì 1 2 3 n ü (B) a5 ⎢ + 1⎥
30. lim í + + ++ ý= ⎣ 16 ⎦ ⎣ 16 ⎦
n ® ¥ 1 - n2 - 2
- 2
- n2 þ
îï 1 n 1 n 1
⎡ 3p 1 ⎤
(C) a5 ⎢ − ⎥ (D) None of these
(A) 0 (B) - 1 2 ⎣ 16 5 ⎦
(C) 1 2 (D) ∞ a
40. ∫0
x 2 (a2 − x 2 )3/2 dx =
¥ p a6 2a 5
é2ù (A) (B)
31. ò0 êë e x úû dx is [where [ ] represents the greatest integer func- 32 15
a6
tion] (C) (D) None of these
32
(A) 0 (B) 2/e
d æ e sin x ö 4 3
(C) e2 (D) loge2 ÷ ; x > 0. If ò1 e dx = F (k ) - F (1) , then
sin x 3
41. Let F( x ) = ç
dx è x ø x
32. Evaluate one of the possible value of k is
p /4
sin2 x cos2 x (A) 15 (B) 16
∫
0 (sin3 x + cos3 x ) 2
dx
(C) 63 (D) 64
x
42. If f ( x ) = ∫ t sin t dt , then f ′( x ) =
33. If f ( x ) = x − m + 1 , x ∈ [ m − 1, m) , (m ∈ I), then evaluate 0
x x /2 3p /4
1 dx
2 ∫0 ∫ ∫
2 2 2
44. Prove that e zx e − z dz = e x 4
e − z dz . 57. is equal to
0 p /4
1 + cos x
p /2 1 1
(D) −
45. Evaluate ò
- p /2
cos x - cos3 x dx. (A) 2 (B) –2 (C)
2 2
x 1
p /4
sin x + cos x 58. ∫ f (t )dt = x + ∫ t f (t )dt , then the value of f (1) is
∫
0 x
46. Evaluate dx .
9 + 16 sin2 x 1 1
0
(A) (B) 0 (C) 1 (D) −
p p
2 2
p x
47. Show that ò xf (sin x )dx = 2 ò f (sin x ) dx .
0 0
59. If g(x) = ò cos 4 t dt, then g(x + p ) equals
0
3/2 (A) g(x) + g(p ) (B) g(x) – g(p )
48. Find the value of ∫ | x sinp x | dx.
−1
(C) g(x) g(p ) (D)
g( x )
g(p )
⎧⎛ p ⎞ ⎫ e 37
p sin(p lnx )
x 2 sin2 x sin ⎨⎜ ⎟ cos x ⎬ dx
p
⎩⎝ 2 ⎠ ⎭ . 60. The value of ò dx is _____.
49. Evaluate
∫
0
(2 x − p )
1
x
2
4
d e sin x 2e sin x
50. Show that lim ⎛⎜
1
+
1
+
1 1⎞
+ ⋅⋅⋅ + ⎟ = ln6.
61. Let
dx
F( x ) =
x
, x > 0 if ∫ x dx = F ( x ) − F (1) then
1
n→∞ ⎝ n + 1 n+2 n+3 6n ⎠
one of the possible values of k is _____.
1
⎪⎧ ⎛ 1 ⎞ ⎛ 2 ⎞ ⎛ 3 ⎞ ⎛ n ⎞ ⎫⎪ ⎛ 1⎞
2 2 2 n 1
51. Find lim ⎨ ⎜ 1 + 2 ⎟ ⎜ 1 + 2 ⎟ ⎜ 1 + 2 ⎟ ! ⎜ 1 + 2 ⎟ ⎬ . 62. If for non-zero x, a f(x) + b f ⎜ ⎟ = − 5, where a ≠ b, then
n→∞
⎩⎪ ⎝ n ⎠ ⎝ n ⎠ ⎝ n ⎠ ⎝ n ⎠ ⎪⎭ ⎝x⎠ x
2
52. Evaluate: ∫
x sin x
p
dx =
ò f ( x ) dx
1
= _____.
0
2 − sin2 x
63. Let T > 0 be a fixed real number. Suppose f is a continuous
p2 p2 T
(A) (B)
4 2 function such that for all x ∈ R, f(x + T) = f(x). If I = ∫f (x) dx
3+3T 0
p p
(C)
2
(D)
4
then the value of ∫
3
f (2 x ) dx is
p
1 + cos 2 x 3
53. ∫0
2
dx = (A)
2
I (B) 2 I (C) 3 I (D) 6 I
54. ∫ sin p x dx = p /2
x sin x cos xdx
−1 65. Evaluate ∫
0
cos 4 x + sin4 x
.
2 3
(A) (B) 66. A function f(x) is such that it is integrable has interval over
p p
every interval on the real line
4 5 p /2
(C) (D)
p p
∫ sin 2kx
0
7p 8
x dx p /2
55. ∫ cos2 x − sin2 x
= cot x dx = ∫
(sin2kx )cos x
dx
p 8
0 sin x
p p
(A) log (3 - 2 2 ) (B) log (3 − 2 2 ) for every x and real t show that
2 4
p /2
(C) p log ( 2 − 1) (D) p log ( 2 + 1)
∫ 2cos x[cos x + cos3 x + " + cos(2k − 1) x ] dx
0
56. If for a real number y, [y] is the greatest integer less than or is independent of a.
3p /2
∫ x sin−1 x
1/2
equal to y, then the value of the integral [2sin x ] dx is
p /2
67. Evaluate ∫
0 1− x 2
dx .
(A) – p (B) 0
⎧e cos x .sin x , x ≤ 2 3
(C) –
p
2
(D)
p
2
68. If f ( x ) = ⎨
⎩2, otherwise
, then
∫ −2
f ( x )dx = _______ .
1052 Mathematics Problem Book for JEE
ò f ( x ) × g( x ) × h( x ) dx .
2 3
0
(C) 2g(2) < ò g( x )dx
0
(D) 3g(3) < ò g( x )dx
0
p
⎛p ⎞
72. Evaluate
∫
0
x ⋅ sin 2 x ⋅ sin ⎜ cos x ⎟ dx .
⎝2 ⎠
7. If f ( x ) = ⎡⎣ tan100 x ⎤⎦ (where [⋅] denotes greatest integer func-
tion), then
(A) f (x) is discontinuous at x = tan−1 100 3, tan−1 100 100
Practice Exercise 2
(B) f (x) is discontinuous at x = tan−1 3 100 , tan−1 100 99
Single/Multiple Correct Choice Type Questions tan−1 100 4
3
x 2dx
(C) ∫ f ( x )dx = 3tan−1 100 4 − tan−1 100 3 − tan−1 100 2 − 1
ò = I , then the value of 0
1. If
2 x4 - x2 +1 tan−1 100 4
3
xdx
1/3
dx
(D) ∫ f ( x )dx = 3tan−1 100 4 − 2 tan−1 100 3
∫ −∫
0
2
is equal to
⎛ 1⎞ 1 ¥
2
1+ ⎜ x − ⎟
1/2
x3 x2 + −1 8. If In = ò e - x (sin x )n dx (n ÎN , > 1) , then
⎝ x⎠ x2
0
(B) p ⎧x
⎪∫ (5 + 1− t ) dt ,
(A) 2p
if x > 2
p p f(x) = ⎨ 0
(C) (D) ⎪5 x + 1,
2 4 ⎩ if x ≤ 2
p /4
⎛ 1− tan x ⎞
4. The value of ∫ ln ⎜
⎝ 1+ tan2 x ⎟⎠
dx is Then the function is
0
(A) Continuous at x = 2
p ⎛ 1⎞ 3p (B) Differentiable at x = 2
(A) ln ⎜ ⎟ (B) ln2
4 ⎝ 2⎠ 8 (C) Discontinuous at x = 2
3p (D) Not differentiable at x = 2
(C) p ln2 (D) – ln 2
8 8
Comprehension Type Questions
æ 1 æ 1 + n2 ö 2 æ 4 + n2 ö 3 æ 9 + n2 ö
5. Let lim çç 2 sin ç 2 ÷ + 2 sin ç 2 ÷ + 2 sin ç 2 ÷ + ××× Paragraph for Questions 11–13: Let f: R → R be a continuous
n®¥ n
è è n ø n è n ø n è n ø
and bijective function defined such that f(a ) = 0 (a ≠ 0). The area
2 ö bounded by y = f(x), x = a, x = a – t is equal to the area bounded by
+ × sin(5) ÷ = y , then the value of y is
n ø y = f(x), x = a, x = a + t ∀ t ∈ R then
Chapter 23 | Definite Integration 1053
12. The value of f(2a) is equal to Paragraph for Questions 20–22: Let n be a positive integer such
(A) f(a) (B) –f(a) that In = ∫ x n a2 − x 2 dx , then answer the following questions:
(C) f(0) (D) –f(0) 20. The value of I1 is
b
13. The value of ∫f
−1
(t )dt is equal to 2 2 1 2
(A) (a - x 2 )1/2 (B) (a - x 2 )3/2
−b 3
(A) 0 (B) 2ab 3
(C) ab (D) None of these 2 1
(C) – (a2 − x 2 )3/2 (D) – (a2 - x 2 )3/2
3 3
Paragraph for Questions 14–16: Consider a polynomial f(x),
a
which satisfies the following conditions:
∫x a2 − x 2 dx
4
(i) f ( x ) = {f ′( x )} , ∀x
2
0
21. The value of the expression a
is equal to
∫ x a − x dx
1 2 2 2
19
(ii) ∫ f ( x )dx = 0
0 12
(iii) f ′(0) > 0 a2 3a2
(A) (B)
14. The function f(x) can be 6 2
(A) A linear function (B) A quadratic function 3a2 a2
(C) (D)
(C) A cubic function (D) Any polynomial of even 4 2
degree
− x n−1(a2 − x 2 )3/2
15. The value of f ′(0) is 22. If In = + kIn−2 , then the value of k is
n+2
1 1
(A) 0 (B) (C)
2
(D) 1 n −1 n+2
4 (A) (B)
n+2 n −1
16. The function f(x) is
æ n - 1ö 2 n + 2ö 2
(A) Even (B) Odd (C) ç (D) æç
è n + 2 ÷ø
a
è n - 1 ÷ø
a
(C) Neither even nor odd (D) May be either even or
odd
Matrix Match Type Questions
Paragraph for Questions 17–19: If m > 0, n > 0, the definite
1
23. Match the following:
∫x
m−1
integral I = (1− x )n−1 dx depends upon the values of m and n
0
Column I Column II
and is denoted by b (m, n), called the beta function. That is,
1 1 1 æ 2cos x - 3sin x ö dy (p) 2p
(A) If y = cos -1 ç
∫x (1− x )5 dx = ∫ x 5−1(1− x )6−1 dx = b (5, 6) and ∫x (1− x )−1/2 dx ÷ , then is
4 5/2
0 0 0
è 13 ø dx
1
⎛ 7 1⎞
=∫x 7/2−1
(1− x ) 1/2−1
dx = b ⎜ , ⎟ . Obviously, b (n, m) = b (m, n). (B) The value of (q) 0
⎝ 2 2⎠ 1 1
0 é 2 1ù -1 é 2 1ù
ò sin ê x + 2 ú dx + ò cos ê x - 2 ú dx is equal
-1
p /2
m
0
(A) ∫ 1+ tan x
0
(p)
117
(A) (B) 1
n
1054 Mathematics Problem Book for JEE
1
æ pö ⎡ ⎛ 3⎞⎤
d æ 1ö
(D) The value of ò ç tan-1 ÷ dx is
1
(s) –
p 29. If ò t 2f (t )dt = 1- cos x " x Îç 0, ÷ , then the value of ⎢f ⎜
è 2ø ⎟⎥
⎣⎢ ⎝ 4 ⎠ ⎥⎦
dx è xø 2 cos x
-1
is _______ (where [⋅] denotes the greatest integer function).
Answer Key
Practice Exercise 1
1. (A) 2. (D) 3. (C) 4. (A) 5. (D) 6. (C)
7. (D) 8. (D) 9. (C) 10. (B) 11. (A) 12. (A)
13. (B) 14. (B) 15. (D) 16. (C) 17. (B) 18. (C)
19. (B) 20. (C) 21. (B) 22. (A) 23. (D) 24. (D)
25. (B) 26. (C) 27. (C) 28. (C) 29. (A) 30. (B)
1
31. (D) 32. 33. 2n 34. (A) 35. (C) 36. (C)
6 3
37. (C) 38. (B) 39. (C) 40. (A) 41. (D) 42. (B)
p 1 ⎛ 3 − 1⎞ 4 1 3 1
43. − log ⎜ ⎟ 45. 46. ln3 48. +
4 3 ⎝ 2 ⎠ 3 20 p p2
p −4
16 2
49. 50. ln6 51. 2e 52. (A) 53. (A) 54. (D)
p
55. (B) 56. (C) 57. (A) 58. (A) 59. (A) 60. 2
1 ⎡ 7b ⎤ p2
61. k = 16 62. a ln2 + − 5a ⎥ 63. (C) 64. pa cosec a 65.
a2 − b 2 ⎢⎣ 2 ⎦ 16
6 −p 3 35
67. 68. 2 69. 70. −p ln 2 71. 0 72. 8
12 2 2 p
Chapter 23 | Definite Integration 1055
Practice Exercise 2
1. (D) 2. (A) 3. (C) 4. (D) 5. (C) 6. (C), (D)
7. (A), (C) 8. (A), (C) 9. (B), (D) 10. (A), (D) 11. (C) 12. (D)
13. (B) 14. (B) 15. (D) 16. (C) 17. (A) 18. (C)
19. (B) 20. (D) 21. (D) 22. (C) 23. (A) → (s), (B) → (p), (C) → (r), (D) → (q)
24. (A) → (s), (B) → (p), (C) → (r), (D) → (q) 25. (A) → (q), (B) → (r), (C) → (q), (D) → (s) 26. (A) → (q), (B) → (p), (C) → (q), (D) → (r)
27. 2 28. 0 29. 5
Solutions
= ⎜ ⎟ − ∫ ( −1) dx − ∫ (0) dx x
⎝ 2 ⎠ −1 −1 0 5. ∫ [sin t ]dt
0
(( 2n +1)p < x < (2n+2) p)
= 0 + ( x )-1 - (0 )0 = 1- 0 = 1
0 1 p /2 p 3p / 2 2p
ò 0 × dt + ò 0 × dt + ò
0 p /2 p
-1× dt + ò ( -1) dt +
3p
2. x 2 - f ¢( x ) = 0 2
p 2p 3p 4p x
Þ x2 - 2 - x2 - 0 = 0 = ∫ 0 ⋅ dt − ∫ 1⋅ dt + ∫ 0 ⋅ dt + ∫ −1dt + … + ∫ −1dt
0 p 2p 3p (2 n+1)p
⇒ x2 = 2 − x2 = −p − p − …n times − (n)(x2 n+1)p
Þ x4 + x2 - 2 = 0 p
dx dx
p
6. I=∫ =∫ − tan x
1 + 2 tan x
1 + 2
⇒ x4 + 2x2 − x2 − 2 = 0 0 0
p
p
Þ ( x + 2)( x - 1) = 0
2 2 ⇒ 2I = ∫ dx = p ⇒ I =
0
2
⇒ x = 1, − 2 2
p /4 p /4
sec2 x 1 (1+ e x ) + (1− e x )
⇒x=±1
7. ∫
− p /4
1 + e x
dx = ∫
2 − p /4
(1 + e x
)
⋅ sec2 xdx
p /4 p /4
3+3T 3T T 1 1 1− e x
3. ∫ f (2 x ) dx = ∫ f (2 x ) dx = 3∫ f (2 x ) dx = ∫
2 − p /4
sec2 xdx + ∫
2 − p /4 1+ e x
sec2 xdx
3 0 0
1
Let 2x = t. Then = (tan x )p−p/4/4 + 0
2
2T T
3 ⎛ 1− e − x 1− e x ⎞
2 ∫0
f (t ) dt = 3∫ f (t ) dt = 3I ⎜ since, f ( − x ) = sec2 ( − x ) = − sec2 x = − f ( x ) ⎟
−x
0 ⎝ 1+ e 1+ e x
⎠
1
100 = ⋅2 =1
∫a
x −[ x ] 2
4. dx
0
a
100 x
8. I = ∫ (a − x )f (a − x )dx
a
= ∫a
0
[x]
dx 0
a a
= a ∫ f ( x )dx − ∫ x f ( x )dx
1 2 3 100
ax ax ax ax 0 0
=ò dx + ò1 a dx + ò2 a2 dx + " + ò dx a
0
a0 99
a99 ⇒ 2I = a ∫ f ( x ) dx
0
1 2 3 100
æ ax ö æ ax ö æ ax ö æ ax ö a
=ç ÷ +ç ÷ +ç 2 ÷ + " + ç 99 ÷ a
2 ∫0
⇒I = f ( x )dx
è log a ø0 è a × log a ø1 è a log a ø2 è a log a ø99
1056 Mathematics Problem Book for JEE
e
1 1 1
9. IP = ∫ (ln x ) dx
p = + + + " + [ x ] times
ln2 ln2 ln2
1
e e [x]
IP + pIP −1 = ∫ (ln x )p dx + p ∫ (ln x )p−1dx =
ln2
1 1
e e
e p 15. Let
= ⎡⎣ x ⋅ (ln x )p ⎤⎦1 − ∫ (ln x )p−1 ⋅ xdx + p ∫ (ln x )p−1dx = e
x
1 1 1 xb -1 b
1 x log x
which is less than 3. I (b ) = ò dx Þ I '(b ) = ò dx
0 log x 0 log x
b b
p
2 p /2 p /2
(If I (a ) = ∫ f ( x , a )dx , then I '(a ) = ∫ f ′( x , a )dx , where f ′( x , a )
sin2 x 0 0
10. I = ∫ ln| tan x + cot x | dx = − ∫ ln dx I = −2 ∫ lnsin x dx
2 is derivative of f ( x , a ) w.r.t. a keeping x constant)
0 0 0
p /2 1 1
= − ∫ [ln| sin2 x | − ln2] dx I ′(b ) = ∫ x b dx =
0 b +1
0
db
p /2
p p ⇒ I (b ) = ∫ + c = log(b + 1) + c
l = -2I1 = - I1 + ò
0
ln2 dx = - I1 +
2
loge 2 Þ I1 = - ln2
2
b +1
If b = 0 , then I (b ) = 0. So,
Hence,
c = 0 ⇒ I (b ) = log(b + 1)
⎛ p ⎞
−2I1 = −2 ⎜ − ln2 ⎟
⎝ 2 ⎠
⇒ I = p ln2 1 d ⎛ −1 1 ⎞ p
∫
−1
⎜⎝ tan ⎟⎠ dx = −2[tan ( x )]0 = −
1
16. −1 dx x 2
æ 1- t ö
x
F ( x ) = ò ln ç dt
è 1+ t ÷ø
11.
0 17. We have
æ 1- x ö æ 1+ x ö F ′( x ) = 3sin x + 4 cos x
Þ F ¢( x ) + F ¢( - x ) = ln ç + ln ç =0
è 1+ x ÷ø è 1- x ÷ø
é 5p 4p ù
p /4 p /4
Since in ê , ú , F ¢( x ) < 0, so assume the least value at the
p /4 ë 4 3 û
12. ∫ 1⋅ d ( x − [ x ]) = ∫ d { x } = [{ x }] 0
=
p
4p
0 0 4 point x = . Thus,
3
1 2n r
13. lim
n®¥ n
å æ 4p ö
r =1 n + r2
2
f ç ÷ =ò
4 p /3
(3sin u + 4 cos u )du
è 3 ø 5p /4
1 2n r /n
lim =
n®¥
å
n r =1 1 + (r/n)2 3 1
= −2 3 +
2 2 2
x
=ò dx ∞
0 1+ x 2 18.
∫ 0
e −2 x (sin2 x + cos2 x )dx
Let 1+ x 2 = t 2. Then xdx = tdt ¥
é cos2 x ù ¥ æ - sin2 x ö ¥
= ê - e -2 x ú - ò ( -2e -2 x ) ç ÷ø dx + ò0 e sin2 x dx
-2 x
5
t ë 2 û0 0 è 2
ò t dt = (t ) = 5 -1
5
1
1
1
=
2
[x]
2x
14. ∫2
0
[x]
dx
19. ∫ 0
b −c
f ″( x + a)dx = [f ′( x + a)]b0−c = f ′(b − c + a) − f ′(a)
1 2 3
2x 2x ⎡ 1 1⎤
= ∫ 2 x dx + ∫ dx + ∫ 2 dx + … 20. F ′( x ) =| x |> 0∀x ∈ ⎢ − , ⎥
0 1
2 2
2 ⎣ 2 2⎦
⎡ 1 1⎤
æ 2x ö 1 æ 2x ö ¼
1 2
Hence, the function is increasing on ⎢ − , ⎥ and therefore
=ç + + ⎣ 2 2⎦
è log2 ø÷ 0 2 èç log2 ø÷ 1 ⎡ 1 1⎤
F ( x ) has maxima at the right end point of ⎢ − , ⎥ . So,
1 1 0 1 1 ⎣ 2 2⎦
= (2 - 2 ) + (22 - 21 ) + 2 (23 - 22 ) + ¼
ln2 2ln2 2 log2 ⎛ 1 ⎞ 1/2 3
Max F ( x ) = F ⎜ ⎟ = ∫ | t | dt = −
⎝ ⎠
2 1 8
Chapter 23 | Definite Integration 1057
p /2 Therefore,
21. ò- p /2
sin2 x cos2 x (sin x + cos x )dx
n
∑∫
k 1 2 n
p /2 p /2 f ( x )dx = ∫ f ( x )dx + ∫ f ( x )dx + … + ∫ f ( x )dx
= ∫− p /2 sin x cos xdx + ∫− p /2 sin x cos x dx
3 2 2 3 k −1 0 1 n−1
k =1
n
p /2 2 4 = ∫ f ( x ) dx
= 0 + 2∫ 2
sin x cos xdx = 0 + 2 × =3 0
0
15 15 p
28. Since sin4 x + cos 4 x is a periodic function with period ,
2
22. Putting x = tanq , we get
therefore
¥ dx p /2 sec q dq 2
p /2 cosq a+( p /2)
ò 0
( x + x + 1) 2 3
=∫
0 (tanq + secq )3 ∫0 (1+ sinq )3
= dq ∫a
(sin4 x + cos 4 x ) dx
p /2
⎡ 1 ⎤ 1 1 3
p /2
=∫ (sin4 x + cos 4 x )dx
= ⎢− 2⎥
=− + = 0
⎣ 2(1+ sinq ) ⎦ 0 8 2 8 p /2 3G(5/2)G(1/2) 3p
= 2ò sin4 x dx = =
0 æ 4+0+2ö 8
23. We know that 2G ç ÷
è 2 ø
d
dx (∫ f (t )dt ) = dbdx f (b) − dadx f (a)
b
a
p x p /2
6 1
G .G 16
29. ò sin dx = 2ò sin tdt = 2 × 2 2 =
5 5
a and b are functions of x. 0 2 0 7 15
2G
Therefore, 2
n
r 1 n
1⎛r ⎞
⇒ F ¢( x ) =
d 3
(x )
1 d
- (x2 )
1 = lim ∑ = lim ∑ ⎜ ⎟
dx log x 3
dx log x 2 n→∞ 1 − n 2 n→∞ ⎛ 1 ⎞ n ⎝n⎠
⎜ 2 − 1⎟
r =1 r =1
⎝n ⎠
3x 2 2x
= x ( x - 1)(log x )-1 -1
1
= - 1 1 1
3log x 2log x = lim
n®¥ æ 1 ö0 ò x dx = lim ´
n ® ¥ 2 æ 1
=
ö 2
ç 2 - 1÷ ç 2 - 1÷
24. f ¢( x ) = e - ( x
2
+1)2 2 2 4
+1+2 x 2 ) 2 èn ø èn ø
× 2 x - e -( x )
× 2 x = 2 xe - ( x (1- e 2 x +1 )
∞
⇒ f ¢( x ) > 0, "x Î( -¥ ,0) ⎡2⎤
31. ∫ ⎢⎣ e
0
x ⎥
⎦
dx
25. We have
2 é2ù
x4 x ∈ (0, loge 2 ) , varies from 2 to 1 and hence ê x ú = 1
f ( x ) = ∫ 2 sin t dt e x
ëe û
x
n n m n m
Now again, put 2q = j . Therefore,
33. ∫ f ( x ) dx = ∑ ∫ f ( x ) dx = ∑ ∫ ( x − m + 1) dx
⎡ 3p 1 ⎤
a5 é ò sin4 j dj - ò sin4 j cosj dj ù = a5 ⎢ − ⎥
p /2 p /2
0 m=1 m−1 m=1 m−1
êë 0 0 úû ⎣ 16 5 ⎦
n n
2 m 2 2n
= å ( x - m + 1)3/2 =å = a
m=1 3
m-1
m=1 3 3 40. I = ∫ x 2 (a2 − x 2 )3/2 dx
0
x3 Put
34. F ( x ) = ∫ 2 log t dt
x x = a sinq Þ dx = a cos q dq
Applying Leibnitz’s theorem, p /2
I=∫ a2 sin2 q ⋅ a3 cos3 q ⋅ a cosq dq
d d 0
F '( x ) = log x 3 ⋅ x 3 − log x 2 ⋅ x 2
dx dx 3 5
p /2
G ×G
= 3log x × 3 x 2 - 2log x × 2 x = (9 x 2 - 4 x )log x 2 2
=a ò sin q cos q dq = a
6 2 4 6
0 8
p /2
2×G
p /2 2
35. I = ∫ sin4 x cos6 x dx = 2∫ sin4 x cos6 x ⋅ dx
− p /2 0
1 3 1
a a
× p × × × p p a6
since, ò -a
f ( x ) dx = 2 ò f ( x ) dx ,
0
if f ( - x ) = f ( x ) =a 2 6 2 2
2 × 3 × 2 ×1
=
32
= 0, if f ( - x ) = -f ( x )
2
Applying Gamma function, we get 3 sin x 3
4 4 3x
ò e dx = ò
3
41. e sin x dx
3
x 1 1 x
2 G5 / 2 ×G7/ 2
I= Put
2 × G6
3/2 ×1/ 2 × p × 5/2 × 3/ 2 ×1/ 2 × p 3p 3p x 3 = t ⇒ 3 x 2dx = dt
= = 8 = 64 e
sint
64 d
5 × 4 × 3 × 2 ×1 2 256 F (t ) = ò dt = ò F (t )dt = F (64) - F (1),
1 t 1 dt
1
⎡ ⎛ 2 x ⎞⎤ p On comparing, we get
36. I = ⎢sin−1 ⎜ 2 ⎟⎥
= sin−1(1) − sin−1(0) =
⎣ ⎝ 1+ x ⎠ ⎦ 0 2 k = 64
1 æ1 1ö x
42. Since, f ( x ) = ò t sin t dt . Now, according to Leibnitz’s rule,
¥ xdx ¥
- dx ç x + ÷ø
¥è2 0
2 + 2
37. ò 0 (1+ x )(1+ x 2 ) ò0 (1+ x ) ò0 1+ x 2
= dx
f ′( x ) = x sin x ⋅ (1) − 0 = x sin x
∞
⎡ −1 ⎤ 1 1 1 d
= ⎢ log(1+ x )⎥ + × [log(1+ x 2 )] ∞0 + [tan−1 x ] ∞0 43. Observe that sin x + cos x = (sin x − cos x )
⎣ 2 ⎦0 2 2 2 dx
Therefore, express the denominator as a function of
1 ⎡p ⎤ p sin x − cos x .
= 0 + 0 + ⎢ − 0⎥ =
2⎣2 ⎦ 4 2
1 + cos 2 x ⎛ 1 − cos 2 x ⎞
38. Put cos2 x + sin4 x = +⎜ ⎟
2 ⎝ 2 ⎠
x = a sinq Þ dx = a cos q dq 3 cos2 2 x 4 − sin2 2 x
= + = = 1 − sin2 x cos2 x
Now 4 4 4
p /2 2
a
ì1 - (sin x - cos x )2 ü
∫0
x 4 a2 − x 2 dx = a6 ∫
0
sin4 q cos q cos q dq = 1- í ý
î 2 þ
⎛ 5⎞ ⎛ 3⎞
Γ ⎜ ⎟ .Γ ⎜ ⎟
p /2 ⎝ 2⎠ ⎝ 2⎠ p 6 Let z = sin x − cos x ; dz = (cos x + sin x ) dx
= a6 ∫ sin4 q cos2 q dq = a6 = a
0 2Γ 4 32 When x = 0, z = –1, when x = p /4, z = 0
0 0
(Using gamma function) dz 4 dz
I=ò =ò
39. Put -1 æ 1- z ö 2 2
-1
2 - (1 - z 2 )2
2
1- ç
x = a(1− cos2q ) ⇒ dx = 2a sin2q dq è 2 ÷ø
0
4 dz
Therefore, =ò
a -1
(3 - z 2 ) (1 + z 2 )
ò 0
x (2ax - x 2 )3/2 dx
0
⎧ 1 1 ⎫
p /4 =∫ ⎨ + ⎬ dz
=∫ 2a (1− cos2q )sin 2q dq
5 4
−1 ⎩ 1 + z 2
3 − z2 ⎭
0
Chapter 23 | Definite Integration 1059
é 1 3 + zù
0
⎡ ⎤
= ê tan-1 z + 1 2 ⎢ 1 1 ⎥
1
log ú 1 dy 1
ëê 2 3 3 - z úû -1 = − ∫ 2 = − × ∫⎢ − ⎥ dy
16 0 ⎛ 5⎞ 16 5 0 ⎢ ⎛ 5⎞ ⎛ 5 ⎞⎥
y2 − ⎜ ⎟ ⎢⎣ ⎜⎝ y − ⎟ ⎜ y + ⎟
ìï 1 æ 3 - 1 ö üï ⎝ 4⎠ 4⎠ ⎝ 4 ⎠ ⎥⎦
= 0 - ítan-1 (-1) + log çç ÷÷ ý
ïî 2 3 è 3 + 1 ø ïþ 1 ⎡ ⎛ 5⎞ ⎛ 5 ⎞⎤
1
=− ln ⎜ y − ⎟ − ln ⎜ y + ⎟ ⎥
p 1 ⎛ 3 − 1⎞ p 1 ( 3 − 1)2 40 ⎢⎣ ⎝ 4⎠ ⎝ 4 ⎠⎦0
= − log ⎜ ⎟ = − log
4 2 3 ⎝ 3 + 1⎠ 4 2 3 2
⎡ 5⎤
1
æ 3 - 1ö 1 ⎢y+ 4⎥ 1 1 1
p 1 = ln ⎢ ⎥ = ln 9 = ln32 = ln 3
= - log çç ÷÷ 40 ⎢ y − 5 ⎥ 40 40 20
4 3 è 2 ø ⎣ 4 ⎦0
x + 2u Therefore,
44. Put z = . Therefore, dz = du.
2 p /4
⎛ sin x + cos x ⎞ 1
x
z = 0 ⇒ u = − and z = x ⇒ u =
x ∫ ⎜⎝ 9 + 16 sin2 x ⎟⎠ dx = 20 ln3
0
2 2
Therefore, 47. Let
p p
x /2 æ x +2 u ö æ x +2 u ö
1 çè ÷ x-
2 ø çè ÷
I = ∫ xf (sin x )dx = ∫ (p − x )f {sin(p − x )}dx
LHS = I =
2 ò x
e 2 ø .du
0 0
u =-
2 p p p
2 − ∫x /2 ∫e
− u2
= e 4
du = e 4 du p
p
p
2 x ⇒ 2I = p ∫ f (sin x ) dx or I = ∫ f (sin x ) dx
−
2 0
20
x x
1 x 2
2 x2 2 x /2 48. sin x is positive for 0 ≤ x ≤ p
∫e
2
− z2
∫ ∫ e − u du = e
− u2 2
= ⋅ 2e 4 du = e
x /4
e 4 dz = RHS sin p x is positive for 0 ≤ p x ≤ p, that is, 0 ≤ x ≤ 1
2
For −1 ≤ x < 0, −p ≤ p x < 0, for such x, sin p x < 0.
0 0 0
p /2
⎛ 3⎞ ⎛ 3p ⎞
For 1 < x ≤ ⎜ ⎟ , p ≤ p x ≤ ⎜⎝ ⎟⎠ ⇒ sin p x < 0
45. I = ∫ p
cos x − cos3 x dx ⎝ 2⎠ 2
−
2 Therefore,
p /2
−1 ≤ x < 0, x sinp x > 0
=2 ∫
0
cos x − cos3 x dx (as cos x is an even function)
0 ≤ x ≤ 1, x sinp x > 0
p /2 3
1<x≤ x sinp x< 0
=2 ∫
0
cos x sin2 x dx 2
Therefore,
p /2 3 3
=2 ∫ cos x sin x dx (since sin x is positive in (0, p / 2)) 2 1 2
⎣ 3 ⎦0 2
3
= 2 ∫ x sinp x dx − ∫ x sinp x dx
0 1
p /4
(sin x + cos x ) 3 3
46. I = ∫ 9 + 16 sin2 x
dx ⎡ ⎛ − cos p x ⎞ ⎛ sinp x ⎞ 1 1⎤
⎡ ⎛ − cos p ⎞ ⎤ 2 ⎛ sinp x ⎞ 2
0 = 2⎢x ⎜
⎝ ⎟⎠ + ⎜⎝ 2 ⎟
⎠
⎥ − ⎢x ⎜ ⎟⎥ +⎜ 2 ⎟
⎣⎢ p p ⎥ ⎣ ⎝ p ⎠ ⎦1 ⎝ p ⎠ 1
0⎦
Put y = sin x − cos x 0
dy = (cos x + sin x) dx ⎛ 1⎞ ⎧ ⎛ 1⎞ ⎛ 1 ⎞⎫ 3 1
= 2 ⎜ ⎟ − ⎨− ⎜ ⎟ − ⎜ 2 ⎟ ⎬ = + 2
y2 = sin2 x + cos2 x − 2 sin x cos x = 1 − sin 2x ⎝p ⎠ ⎩ ⎝p ⎠ ⎝p ⎠⎭ p p
Therefore, sin 2x = 1− y2
3p
(Since, sin = −1 and cos p = −1 )
x=p ,y=0 2
4
Therefore, 49. Put
0
dy ⎛p⎞
∫−1 9 + 16(1− y 2 ) dx x = ⎜ ⎟ − y ⇒ dx = − dy
⎝ 2⎠
1060 Mathematics Problem Book for JEE
−p p 1
and when x = p , y = or when x = 0, y = . = log 2 − ⎡⎣ 2 x − 2 tan − 1 x ⎤⎦
2 2 0
2 ⎡ 2p ⎤ p −4
−p ⎛p ⎞ ⎛⎛ p ⎞ ⎞ = log 2 − ⎢ 2 − ⎥ = log 2 +
⎜⎝ − y ⎟⎠ sin2 y sin ⎜ ⎜⎝ ⎟⎠ sin y ⎟ ⎣ 4 ⎦ 2
1 2
2 ⎝ 2 ⎠
I=
2 ∫ (− y )
( − dy )
log 2+
p −4 p −4
p
⇒ A=e 2
= 2e 2
2
⎛p 2 ⎞ ⎧p ⎫ p
x sin x
p
(p − x ) sin (p − x )
I=∫ dx = ∫
p
+ y 2 ⎟ sin2 y sin ⎨ sin y ⎬
⎝⎜ 4
52. dx,
−1 2
⎠ ⎩ 2 ⎭ 2 − sin2 x 2 − sin2 (p − x )
2 −∫p
= dy 0 0
y ⎛ a a
⎞
2
⎜ using ∫ f ( x ) dx = ∫ f (a − x ) dx ⎟
p ⎧p ⎫ ⎝ 0 0 ⎠
2 sin2 y sin ⎨ sin y ⎬
+ ∫ py ⎩ 2 ⎭ dy p
sin x
y ⇒ 2I = p ∫ dx
−p
0
2 − sin2 x
2
Therefore,
= −I1 + I2
p p
I1 = 0 (Since integrand is odd) p − d (cos x ) ⎡ p ⎤
2 ∫0 1+ cos2 x ⎢⎣ 2
I= = − tan−1(cos x )⎥
Therefore, ⎦0
p
⎛ −p ⎞ ⎛ −p ⎞ p ⎛ p ⎞ p
2
⎧⎛ p ⎞ ⎫ =⎜ ⎟⎠ + ⎜⎝ ⎟⎠ =
⎝ 2 ⎟⎠ ⎜⎝
2
I = 2p ∫ 2sin y cos y sin ⎨⎜ ⎟ sin y ⎬ dy 4 2 4 4
0
⎝
⎩ 2 ⎠ ⎭
p p p
1+ cos 2 x
Put 53. ∫ 2
dx = ∫ cos2 x dx = ∫ | cos x | dx
⎛p⎞ p 0 0 0
⎜⎝ ⎟⎠ sin y = t ⇒ cos y dy = dt p /2 p
2 2
p
⎡ p
⎤
= ∫ cos x dx + ∫ − cos x dx
0 p /2
2
2t 2 16 ⎢ 2
⎥
⇒ l = 2p × 2 ∫ ⋅ sin tdt = ⎢{t ( − cos t )}0 − ∫ ( − cos t )dt ⎥
p /2
= [ sin x ]0 − [ sin x ]p /2 = 1− ( −1) = 2
p /2 p
p p p
0 ⎢ 0 ⎥
⎣ ⎦
16 54. See Fig. 23.6.
=
p 3/2 0 1 3/2
50. lim ⎛⎜
1 1 1⎞ ∫ | sin p x | dx = ∫ − sin(p x )dx + ∫ sin(p x ) dx + ∫ − sin(p x ) dx
n→∞ ⎝
+ + + ⎟ −1 −1 0 1
n +1 n + 2 6n ⎠
0 1 3/2
⎛ 1 1 ⎞ 5n
1 ⎡ cos p x ⎤ ⎡ − cos p x ⎤ ⎡ cos p x ⎤
1 =⎢ ⎥ +⎢ ⎥ +⎢ p ⎥
= lim ⎜
n→∞ ⎝ n + 1
+
n+2
+ + ⎟ = lim
n + 5n ⎠ n→∞
∑n+r
r =1
⎣ p ⎦ −1 ⎣ p ⎦0 ⎣ ⎦1
5 1 5
dx
= ∫ 1+ x = ln(1+ x ) 0 = ln 6
5 = {1− ( −1) + 1− ( −1) + 0 − ( −1)} =
0
p p
51. Let x
1 −1 3/2
⎧ ⎛ 22 ⎞ ⎛ 32 ⎞ ⎛ n2 ⎞ ⎫⎪
A = lim ⎪⎨ ⎛⎜ 1 + 2 ⎞⎟
n
1 …
⎜⎝ 1 + n2 ⎟⎠ ⎜⎝ 1 + n2 ⎟⎠ ⎜⎝ 1 + n2 ⎟⎠ ⎬
⎩⎪ ⎝ n ⎠ ⎭⎪
n→∞
px
n
1 ⎛ r2 ⎞ π
⇒ loge A = lim
n→∞
∑ n
log ⎜ 1 + 2 ⎟
−p
0
3p /2
r =1 ⎝ n ⎠ Graph of sin p x
1 n ⎧⎪ ⎛r⎞
2
⎫⎪ 1
= lim
n→∞ n
∑ log ⎨ 1 + ⎜ ⎟
⎝ n⎠
⎬ = ∫ log (1 + x ) dx
2 Figure 23.6
r =1 ⎪⎩ ⎭⎪ 0 7 p /8 7 p /8
x dx p−x
= [ x log (1 + x )] − ∫
x ⋅ 2 x dx
2 1
1
55. I = ∫
p /8
cos x − sin2 x
2
=
p /8
∫ cos 2 (p − x )
dx
1+ x 2
0
0
7p /8
1
2 (1 + x 2 ) − 2 (p − x ) dx 7 p /8
dx
= 1log 2 − ∫ dx = ∫ cos 2 x
=p ∫ cos 2 x
−I
0
1+ x 2 p /8 p /8
Chapter 23 | Definite Integration 1061
= F (t ) 1 = F (16) − F (1)
5p /6 p 7p /6 3p /2 16
I= ∫
p /2
dx + ∫
5p /6
0 dx + ∫
p
( −1) dx + ∫
7p /2
( −2) dx
Hence, possible value of k = 16.
5p p 7p 7p
= − +p − + − 3p ⎛ 1⎞ 1
6 2 6 3 62. af(x) + bf ⎜ ⎟ = − 5 (1)
⎝x⎠ x
5p − 3p + 6p − 7p + 14p − 18p 3p p
= =− =−
6 6 2 ⎛ 1⎞
Replacing x by ⎜ ⎟ ,
3p /4 3p /4 3p /4 ⎝x⎠
dx 1 2 x ⎡ x⎤ 3p p
57. ∫p /4 1+ cos x = 2 p∫/4 sec 2 dx = ⎢
⎣
tan ⎥
2 ⎦ p /4
= tan
8
− tan
8 ⎛ 1⎞
af ⎜ ⎟ + bf(x) = x − 5 (2)
⎝x⎠
=
1
−1+ 2
− −1+ 2 ( ) ⎡ p ⎤
⎢since tan 8 = −1+ 2 ⎥
⎣ ⎦
By Eqs. (1) and (2),
1 ⎧a ⎫
f(x) = ⎨ − bx + 5(b − a)⎬
( )
2
1− −1+ 2 a − b2 ⎩ x
2
⎭
=
−1+ 2 Then
2
−2 + 2 2 2 1 ⎧a ⎫
=
−1+ 2
=2
∫ f ( x )dx = a − b2
2 ∫
1
⎨ − bx + 5(b − a)⎬ dx
⎩x ⎭
1
2
58. Differentiate both side w.r.t. x 1 ⎡ bx 2 ⎤
= 2 ⎢ a ln x − + 5(b − a) x ⎥
f(x) ⋅1 − f(0) ⋅ 0 = 1 + 1⋅ f(1) ⋅ 0 − x⋅f(x) ⋅1 a −b ⎣ 2
2 ⎦1
1 1
⇒ (x + 1) f(x) = 1 ⇒ f ( x ) =
= 2 2 ⎡⎢a ln2 − 2b + 10(b − a) + − 5(b − a)⎤⎥
b
x +1 a −b ⎣
At x = 1, we have 2 ⎦
⎡ 3 ⎤
a ln2 + 5b − 5a − b ⎥ = 2 2 ⎡⎢a ln2 + − 5a ⎤⎥
1 1 1 1 7b
f(1) = = =
1+ 1 2 a2 − b 2 ⎢⎣ 2 ⎦ a −b ⎣ 2 ⎦
x +p x x +p 3+3T 6+6 T 6T T
dt 1
59. g( x + p ) = ∫
0
cos 4 t dt = ∫ cos 4 t dt +
0
∫
x
cos 4 t dt 63. ∫ f (2 x ) dx = ∫ f (t )
2 2 ∫0
= f (t )dt = 3∫ f (t ) = 3l
3 6 0
p p+x p
(p − x )
= g( x ) + ∫ cos t dt + ∫ ∫ 1+ cosa sin (p − x ) dx
4 4
cos t dt 64. I=
x p 0
p x x
By taking t = x + p in ∫
4
cos tdt p
p
p ⇒2I = ∫ 1+ cosa sin x dx
g(x+ p) = g(x) + g (p ) 0
x
p p sec2
e 37 p 1 p 2
p sin(p lnx ) I= ∫ dx = ∫ dx
60. I= ∫ 1
x
dx 2 0 1+ cos a sin x 2 0 1+ tan2 x + 2 tan x cos a
2 2
Let Let
p x 1 x
p ln x = t ⇒ dx = dt tan = t ⇒ sec2 dx = dt
x 2 2 2
1062 Mathematics Problem Book for JEE
∞ ∞ p /6
p 2dt p dt
= [q ( − cosq )]0 +
2 ∫0 1+ t 2 + 2t cos a
dx = × 2 ∫
p /6
I=
2 0
( t + cos a )2 + sin2 a
dx ∫ 1⋅ cosq dq
0
p ⎛ t + cos a ⎞
∞
p ⎛p ⎞ = [-q cosq + sinq ]0
p /6
= ⎜ tan−1 ⎟= −1
⎜ − tan cot a ⎟
sina ⎜ sin a ⎟ sin a ⎝ 2 ⎠
⎝ 0 ⎠
p 3 1
p ⎛p ⎛ p ⎞⎞ =− × +
= −1 6 2 2
⎜ − tan tan ⎜ − a ⎟ ⎟
sina ⎝2 ⎝2 ⎠⎠ Hence,
p ⎛p p ⎞ pa 6 -p 3
I= ⎜ − +a ⎟ = I=
sina ⎝2 2 ⎠ sina 12
Hence, I = pa cosec a.
68. f ( x ) = e cos x . sin x is an odd function and
p ⎛p ⎞
2 ⎜ − x ⎟ cos x sin x dx ⎛ a a
⎞ | x |≤ 2 ⇒ −2 ≤ x ≤ 2
65. I = ∫ ⎝2 ⎠ ⎜⎜ Using ∫ f ( x )dx = ∫ f (a − x )dx ⎟⎟
sin4 x + cos 4 x ⎝ 0 0 ⎠
0 Therefore,
p p 3 2 3
ò ò f ( x )dx + ò f ( x )dx
2 2
p cos x sin x dx p cos x sin x dx I= f ( x )dx =
= ∫ 4 −I ⇒I = ∫ 4
2 0 sin x + cos 4 x 4 0 sin x + cos 4 x -2 -2 2
3
p ⇒ I = 0 + ò 2dx = [2 x ]32 = 2
∞ 2
p 2 tan x sec2 x p tdt
I= ∫ dx = ∫ , where t = tan x
4 0 1+ tan4 x 4 0 1+ (t 2 )2 3 5
69. I = 2 ò f ( x )dx + ò f ( x )dx
p 1 ∞ p 2
= ⋅ ⎡⎣ tan−1 t 2 ⎤⎦ = 0 3
4 2 0 16
æ1 2 3
ö 4 5
a +t 0 t a +t
= 2 çç ò f ( x )dx + ò f ( x )dx + ò f ( x )dx ÷÷ + ò f ( x )dx + ò f ( x )dx
66. ∫ f ( x )dx = ∫ f ( x )dx + ∫ f ( x )dx + ∫ f ( x )dx
a a 0 t
è0 1 2 ø 3 4
a +t
⎛ 02 12 22 ⎞ 32 42
= 2⎜ + + ⎟+ +
I3 = ∫ f ( x )dx
t
⎝ 2 2 2⎠ 2 2
Put x = t + y, then 25 35
=1+4+ =
a a 0
2 2
I3 = ∫ f (t + y )dy = ∫ f ( y )dy = − ∫ f ( x )dx (since, f(t + y) = f(y))
0 0 a p /2
Therefore,
70. I =
ò 0
lnsin x dx
t p /2 æp ö p /2
I = I1 + I2 + I3 = ∫ f ( x )dx (Independent of a). I=ò lnsin ç - x ÷ dx Þ I = ò lncos x dx
0
0
è2 ø 0
p /2 p /2 æ sin 2 x ö
67. Let 2I = ò ln(sin x × cos x )dx Þ 2I = ò ln ç ÷ dx
0 0
è 2 ø
x sin−1 x
1/2
I= ∫
0 1− x 2
dx
2I = ò
p /2
ln(sin 2 x )dx - ò
p /2
ln 2dx Þ 2I = ò
p /2
ln(sin 2 x )dx -
p
ln 2
0 0 0 2
Put x = sin q when x = 0, q = 0
p Put 2x = t, then
1
dx = cos q dq when x = , q = . 1
2 6 dx = dt
2
Thus,
1 p p p /2 p
p /6
sinq sin (sinq ) −1
Þ 2I =
2 ò0
ln(sin t )dt - ln 2 Þ 2I = ò ln(sin t )dt - ln 2
2 0 2
I= ∫ 1− sin2 q
cos q dq
p /2 p p
Þ 2I = ò ln(sin x )dx - ln 2 Þ 2I = I - ln 2
0
p /6
0 2 2
= ∫ q sinq dq
0
⇒I =−
p
ln 2
2
Chapter 23 | Definite Integration 1063
a a 3
x 2dx
71.
∫ ∫
I = f ( x )g( x )h( x )dx = f (a − x )g(a − x )h(a − x ) dx Here, I = ò
2 x4 - x2 +1
0 0 3 3 3
xdx xdx tdt
a
=ò =ò =ò
∫
= − f ( x )g( x )h(a − x )dx 2 2
1 æ 1ö æ 1ö
2
x 2 - 1+ 2
1+ ç x - ÷
2
1+ ç t - ÷
0 x2 è xø è tø
7I = 3I + 4I
a a
2. f ′(x) = f(x)
∫
0
f ( x )g( x ){3h( x ) − 4 h(a − x )} dx = 5 f ( x )g( x ) dx = 0
∫
0
Integrating, we get
p
æp ö and
ò
= (p - x )sin 2 x sin ç cos x ÷ dx
0
è2 ø
(2)
g(x) = x – f(x) = x – ex
Therefore,
Adding Eqs. (1) and (2), we get 1 1 1
I = ò e x ( x - e x )dx = ∫ e x x dx − ∫ e 2 x dx
p
æp ö 0 0 0
ò
2I = p sin 2 x sin ç cos x ÷ dx
0
è2 ø e2 1 3 e2 3 - e2
= e - (e - 1) - + = - =
2 2 2 2 2
p
2 1/2
æp ö
ÞI =p
ò
0
2 sin x cos x sin ç cos x ÷ dx
è 2 ø
3. I = ò
-1/2
cos -1(4 x 3 - 3 x )dx
and
Put é p 2p ù
p p x = cosq, q ∈ ê , ú
cos x = z ⇒ − sin xdx = dz ë3 3 û
2 2 Now,
0 0
2z æ 2ö cos–1(4x3 – 3x) = cos–1(cos 3q ) = 2p – 3q
ò 2 × p çè - p ÷ø sin z dz ò z sin z dz = p
8 8
p =-
p Therefore,
p p
p /3
2 2
I=- ò
2p /3
(2p - 3q )sinq dq
Practice Exercise 2 2p /3 2p /3
p
3 1/3
= 2p ò sinq dq - 3 ò q sinq dq =
2
xdx dx p /3 p /3
1. ò 2
- ò 1
2 æ 1ö 1/2
x3 x2 + -1 p /4 p /4
1+ ç x - ÷
è xø x2 4. I = ò lncos2 xdx - ò ln(1+ tan x )dx
0 0
3 1/3
xdx dx
=ò -ò Now,
2 2
2 æ 1ö 1/2 æ 1ö 1
p /2
1æp 1ö p
1+ ç x - ÷ x 3 1+ ç x - ÷
è xø è xø I1 =
2 ò lncos xdx = 2 çè 2 ln 2 ÷ø = - 4 ln2
0
3 3
xdx t 3dt
=ò +ò and
2 2
2 æ 1ö 2 æ1 ö p
1+ ç x - ÷ t 2 1+ ç - t ÷ I2 =
è xø èt ø ln2
8
3 3
xdx tdt
=ò +ò = I + I = 2I Therefore,
2 2
2 æ 1ö 2 æ 1ö 3p
1+ ç x - ÷ 1+ ç t - ÷ I = I1 – I2 = - ln2
è xø è tø 8
1064 Mathematics Problem Book for JEE
1 2n æ r ö æ r2 ö ì x2
5. y = lim
n®μ n
å ç ÷
r =1 è n ø
× sin ç 2 + 1÷
n
ï1+ 4 x + ,
⇒ f(x) = í
x >2
è ø 2
ï5 x + 1, x £2
2 î
= ò x sin( x 2 + 1)dx = sin2 × sin 3
0 Now,
ì4 + x , x >2
6. g(x) < f(x) ∀ x ∈ (0, ∞) f ′(x) = í
î5, x £2
Also,
Therefore,
d(x(f(x)) = g(x)dx
Therefore, f(2+) = f(2−) = f(2) = 11 is continuous at x = 2
x and
xf(x) = ò g( x )dx ∀ x ∈ (0, ∞)
0
f ′(2+) ≠ f ′(2−) ⇒ not differentiable at x = 2
and
a a +t
x
11. ∫ f (t ) dt = − ∫ f (t ) dt ⇒ f(a – t) = –f(a + t) ∀ t ∈ R
xg(x) < ò g( x )dx ∀ x ∈ (0, ∞) a −t a
0
Hence, graph of f(x) is symmetrical about (a, 0).
7. f ( x ) = [tan 100
x] 12. f(a – t) = –f(a + t) ∀ t ∈ R
Þ f (31 100 - ) ¹ f (31 100 ) = f (31 100 + ) and Put x = a , then
f (1001 100 - ) ¹ f (1001 100 ) = f (1001 100 + ) f (2a ) = - f (0)
Hence, f(x) is discontinuous at these points.
a a +t
Now,
tan-1 41 100 tan-1 11 100
13. ∫ f (t ) dt = − ∫ f (t ) dt
a −t a
ò
0
[tan100 x ] dx = ò
0
[tan100 x ] dx ⇒ f(a – t) = –f(a + t) ∀ t ∈ R
tan-1 21 100 tan-1 31 100 tan-1 41 100 ⇒ f(a – t) = –f(a + t) = x
+ ò [tan100 x ] dx + ò [tan100 x ] dx + ò [tan100 x ] dx ⇒ t = a – f –1(x) = f –1(–x) – a
tan-1 11 100 tan-1 21 100 tan-1 31 100
⇒ f –1(x) + f –1(–x) = 2a
Therefore, b
òf
-1
tan−1 100 4 ⇒ ( x )dx = 2ab
∫
0
f ( x )dx = 3tan−1 100 4 − tan−1 100 3 − tan−1 100 2 − 1 -b
è - 1 ø0 0
¥ 15. Let
= ò ( -(sin x )n + (n - 1)(1- sin2 x )(sin x )n-2 )e - x dx
0 f(x) = ax2 + bx + c ⇒ f ′(0) = b > 0
n(n - 1) Also,
= In-2
n2 + 1 f(x) = {f ′(x)}2 ⇒ ax2 + bx + c = 4a2x2 + 4abx + b2 ∀ x
Hence, Thus, a = 4a2, b = 4ab and c = b2
I10 90
= From which, we get a =
1
, since (b ¹ 0 )
I8 101 4
9. In option (B), we have Again,
¥ ¥ 1
1 2 1 19 a b 19
ò xe
- x2
dx = - e - x
2 0
=
2
ò f ( x )dx = 12
0
⇒ + +c =
3 2 12
0
Therefore, Therefore,
¥ ¥ ¥
æ 1 - x2 ö 1 - x2 p
ò0 x e dx = x çè - 2 e ÷ø 2 ò0
2 - x2
+ e dx = b 2 3
0 4 + b = Þ b = 1,
2 2
10. For x > 2, we have (since, (b > 0) and so c = 1)
x 1 x
x2 Therefore,
∫ (5 + 1− t )dt = ∫ (6 − t )dt + ∫ (4 + t )dt
0 0 1
= 1+ 4 x +
2 f ′(0) = b = 1
Chapter 23 | Definite Integration 1065
16. By putting the value of a, b and c in f(x), we have 22. From equation (1), we have
x 2
a2 (n − 1)
+ x +1 f (x) = k=
4 (n + 2 )
17. Writing sin2 q = x, we get 2 sin q cos q dq = dx, and hence the
2cos x − 3sin x ⎞
given integral is 23. (A) y = cos −1 ⎛⎜ ⎟
1
p /2 ⎝ 13 ⎠
∫ cos
2 m−1
sin2 n−1q (2 sin q cos q ) dq 3
2 0
y = cos−1 cos (a + x) ∀ a = tan−1
1 2 m−1 2 n−1
2
1
=
20∫ (cos2 q ) 2 (sin2 q ) 2 dx Therefore,
dy
1 =1
1 dx
= ∫ (1− x )m−1/2 x n−1/2dx
20 1 1
⎡ 2 1⎤ −1 ⎡ 2 1⎤
(B) I = ∫ sin ⎢ x + 2 ⎥ dx + ∫ cos ⎢ x − 2 ⎥ dx
−1
1 ⎛ 1 1⎞ ⎣ ⎦ ⎣ ⎦
= b ⎜m+ , n+ ⎟ −1 −1
2 ⎝ 2 2⎠
⎛ 1 ⎤⎞
1
⎡ 1⎤ ⎡
x = 2∫ ⎜ sin−1 ⎢ x 2 + ⎥ + cos −1 ⎢ x 2 − ⎥⎟ dx
18. Writing = z, we get 0
⎝ ⎣ 2 ⎦ ⎣ 2 ⎦⎠
n Let
n
⎛ x⎞
n 1
⎡ 1⎤ ⎡ 1⎤
f(x) = sin−1 ⎢ x 2 + ⎥ + cos −1 ⎢ x 2 − ⎥
∫ ⎜⎝1− n ⎟⎠
0
x k −1 dx = ∫ (1− z )n (nz )k −1 ndz
0
⎣ 2⎦ ⎣ 2⎦
1 ⎡ 1⎤ ⎛⎡ 1⎤ ⎞
= nk ∫ (1− z )n z k −1 dz = nk b (k , n + 1) = sin−1 ⎢ x 2 + ⎥ + cos −1 ⎜ ⎢ x 2 + ⎥ − 1⎟
⎣ 2⎦ ⎝⎣ 2⎦ ⎠
0
⎡ 2 1⎤
x z 1 dz which is defined if ⎢ x + ⎥ = 0 or 1
19. Writing = z , we get x = ,1+ x = and dx = ⎣ 2⎦
1+ x 1− z 1− z (1− z )2
⎡ 1⎤
1 m−1 1
if ⎢ x 2 + ⎥ = 0 then f(x) = sin−1 0 + cos−1 (−1) = 0 + p = p
z dz ⎣ 2⎦
∫ (1− z ) (1− z )2 ∫0
LHS = m−1
(1− z )m+n = z m−1(1− z )n−1 dz = b (m, n)
⎡ 2 1⎤
0
if ⎢ x + ⎥ = 1 then f(x) = sin−1 1 + cos−1 0 = p
∞
x 1− n ⎣ 2⎦
= ∫ (1+ x )m+ n dx 1
0 I = 2 ∫ p dx = 2p
0
Common Explanation for Questions 20–22: 3
dx
3
dx p
a
⎡ − x (a − x ) 3 2 2 3/2
⎤
a a
(C) ∫ (1+ x )(1+ e sin x ) =2 ∫ 1+ x 2
=
3
2 3 − 3
∫x ⎥ + a ⋅ ∫ x a − x dx
0
4
a2 − x 2 dx = ⎢ 2 2 2
⎣ 3 ⎦0 60 6p 2p
n − 1 n −2 2 ⎝2 ⎠
⎡ 1 2 2 3/2 ⎤
⎢⎣ − 3 (a − x ) ⎥⎦ + 3 ∫ x (a − x ) a − x dx
n−1
=x 2 2 2
p /2 p /2
= tan x ⎡ 1 ⎤
1 n −1 2 n −1
∫0
1+ tan x
dx = ∫ ⎢⎣1− 1+ tan x ⎥⎦ dx
0
= − x n−1(a2 − x 2 )3/2 + a I n −2 − In
3 3 3 Therefore,
⎛ n − 1⎞ 1 n−1 2 a2 (n − 1) p p
⇒ ⎜ 1+ ⎟⎠ In = − x (a − x ) +
2 3/2
I n −2 2I = ⇒I=
⎝ 3 3 3 2 4
x n−1(a2 − x 2 )3 2 a2 (n − 1) (B) Differentiating [ f ( x 2 (1+ x 5 + 7 x 12 ))](2 x + 7 x 6 + 98 x 13 ) = 1
⇒ In = − + I n −2 (1)
(n + 2) (n + 2) Put x = 1, we get
1 1
20. I1 = ∫ x a − x dx = f(9) =
2∫
2 2
2 x a2 − x 2 dx 117
∞
2(a2 − x 2 )3 2 (a2 − x 2 )3 2
∫e
−2 x
=− +c = − +c (C) (sin2 x + cos2 x ) dx
2.3 3 0
∞
x 3 (a2 − x 2 )3 2 3a2 a2
a
a2 ⎛ p⎞
21. I4 = − +
I
I2 = I2 ⇒ 4 = = 2 ∫ e −2 x sin ⎜ 2 x + ⎟ dx
6 6 2 I2 2 0 ⎝ 4⎠
0
1066 Mathematics Problem Book for JEE
Therefore, π π /2
¥ (B) 2 ∫ xf (sin x )dx = π ⋅ 2 ∫ f (sin x )dx
é æ p öù
ê sin ç 2 x + ÷ ú ¥ 0 0
è 4 øú æ pö p /2
I = ê -e -2 x
+ ò e -2 x cos ç 2 x + ÷ dx æ æp öö
ê 2 ú 0 è 4ø = 2p ò f ç sin ç - x ÷ ÷ dx
è è 2 øø
êë úû 0
0
p /2
=
1
+
1
- I ⇒ 2I =
1
⇒ integral value =
1 = 2p ∫ f (cos x ) dx
2 2 2 2 2 2 2 0
k k
Therefore, f ( k + 1- k - x )(1- ( k + 1- k - x )) dx
k
= ∫ (1− x )f ((1− x ) x )dx
1
I =ò cot -1(1+ x 2 - x ) dx
1−k
0
k k
⇒ 2 ∫ xf ( x (1− x ))dx = ∫ f ( x (1− x ))dx
1 1
0 f −1 (0)
1
Put x = in the last integral p p
p p ⎡ ⎛ 3⎞⎤
⇒ 2∫ xf (sin x ) dx = p ∫ f (sin x ) dx ⇒ ⎢f ⎜ ⎟⎥ = 5
0 0 ⎢⎣ ⎝ 4 ⎠ ⎥⎦
Chapter 23 | Definite Integration 1067
1 ⎛ 3⎞ 1 ⎛ 3⎞ 2
dx k
⇒I = ⎜ ⎟+ ⎜ ⎟
16 ⎝ 2 ⎠ 64 ⎝ 2 ⎠ 4. If ∫ ( x 2 − 2 x + 4 )3/ 2 = k + 5 , then k is equal to
1
⇒I =
3
+
3
=
( 4 × 3) + 3 = 12 + 3 = 15 (A) 1 (B) 3
32 128 128 128 128 (C) 4 (D) 2
(ONLINE)
Hence, the correct answer is option (A).
Solution: We have
⎛ p⎞
3. The integral ∫ 1+ 2 cot x (cosec x + cot x )dx ⎜ 0 < x < ⎟ is 2 2
⎝ 2⎠ dx dx
equal to (where C is a constant of integration) ∫ ( x 2 − 2 x + 4 )3/ 2 ⇒ ∫ ⎡ 2 ⎤
3/ 2
⎛ x⎞ ⎛ x⎞
1 1
⎣( x − 1) + 3⎦
(A) 2 log ⎜ sin ⎟ + C (B) 2 log ⎜ cos ⎟ + C
⎝ 2⎠ ⎝ 2⎠ Substituting x − 1 = 3 tanq , where x = 1, q = 0.
⎛ x⎞ ⎛ x⎞ ⎛ 1⎞ p
(C) 4 log ⎜ cos ⎟ + C (D) 4 log ⎜ sin ⎟ + C When x = 2, q = tan−1 ⎜ = .
⎝ 2⎠ ⎝ 2⎠ ⎝ 3 ⎟⎠ 6
(ONLINE) p /6
3 sec2 q
Solution: The given integral is L= ∫ ⎡3 tan2 q + 3⎤
3/ 2
dq
0
⎣ ⎦
I = ∫ 1+ 2cot x (cosec x + cot x )dx
p /6 p /6 p /6
3 sec2q d q 3 sec2 q dq = 1
= ∫ 1+ 2 cot x + cosec x + 2 cot x dx 2 = ∫ = ∫ 3 3 sec2 q 3 ∫ cosq dq
0 [3 sec2 q ]3 / 2 0 0
p /6
= ∫ 1+ cot2 x + 2 cot xcosec x + cot2 x dx 1
= sinq
3 0
Using 1 + cot2x = cosec2x
1⎡ x ⎤
⇒ ⎢sin − sin 0 ⎥
I = ∫ cosec2 x + 2cot x cosec x + cot2 x dx 3⎣ 6 ⎦
(a + b)2 = a2 + b2 + 2ab 1⎡1 ⎤ 1
⇒ ⎢ − 0⎥ =
3 ⎣2 ⎦ 6
⇒ I = ∫ (cot x + cosec x )2 dx 1 k
⇒ = ⇒ k + 5 = 6k ⇒ 5 = 5k ⇒ k = 1
6 k +5
I = ∫ (cot x + cosec x )dx
Hence, the correct answer is option (A).
1 cos x
Substituting cosec x = and cot x = , we get JEE Advanced 2017
sin x sin x
k +1 k +1
1. If I = ∑ k =1∫
98
⎛ cos x 1 ⎞ cos x + 1 dx , then
I = ∫⎜ + dx = ∫ dx k x ( x + 1)
⎝ sin x sin x ⎟⎠ sin x
(A) I > loge99 (B) I < loge99
⎛ x⎞ ⎛ x⎞ ⎛ x⎞ (C) I <
49
(D) I >
49
Substituting cos x + 1 = 2 cos2 ⎜ ⎟ and sin x = 2 sin ⎜ ⎟ cos ⎜ ⎟ ,
⎝ 2⎠ ⎝ 2⎠ ⎝ 2⎠ 50 50
we get
Solution: It is given that
2 cos2 ( x / 2) 2 cos( x / 2)
I=∫ dx = ∫ dx 98 k +1
sin( x / 2)cos( x / 2) sin( x / 2) k +1
I=∑ ∫ dx
⎛ x⎞ ⎛ cos x ⎞ k =1 k x ( x + 1)
⇒ I = 2∫ cot ⎜ ⎟ dx ⎜⎝ since cot x = ⎟
⎝ 2⎠ sin x ⎠ 98 k +1 98 k +1
1 ⎛1 1 ⎞
= ∑ (k + 1) ∫ dx = ∑ (k + 1) ∫ ⎜⎝ − ⎟ dx
k =1 x ( x + 1) k =1 x x + 1⎠
Using the standard integral ∫ cot x dx = log| sin x | + C , we get k k
98
k +1
= ∑ (k + 1) ⎡ln x − ln( x + 1) k ⎤
⎛ x⎞ ⎣ ⎦
I = 2 log sin ⎜ ⎟ + C k =1
⎝ 2⎠ 98
= ∑ (k + 1)[(ln(k + 1) − ln k ) − (ln(k + 1+ 1) − ln(k + 1)]
Hence, the correct answer is option (A). k =1
Chapter 23 | Definite Integration 1069
24.1 Curve Tracing Now y is real, if 0 ≤ x ≤ a, so its region lies between the lines
x = 0 and x = a.
To find the approximate shape of a curve, the following procedure 7. Examine what happens to y when x → ∞ or x → − ∞.
is adopted in order:
8. Asymptotes:
1. Symmetry:
Asymptote(s) is (are) line(s) whose distance from the curve
(a) Symmetry about x-axis: If all the powers of ‘y’ in the
tends to zero as point on curve moves towards infinity along
equation are even, then the curve is symmetrical about the
branch of the curve.
x-axis. For example,
y2 = 4 a x (a) If lim f ( x ) = ¥ or lim f ( x ) = -¥ , then x = a is asymptote of
x ®a x ®a
(b) Symmetry about y-axis: If all the powers of ‘x’ in the
y = f(x).
equation are even, then the curve is symmetrical about the
y-axis. For example, (b) If lim f ( x ) = k or lim f ( x ) = k , then y = k is asymptote of
x ®¥ x ®-¥
x2 = 4 a y
y = f(x).
(c) Symmetry about both axes: If all the powers of ‘x’ and ‘y’ in
the equation are even, then the curve is symmetrical about f (x)
(c) If lim = m or lim (f ( x ) - mx ) = c , then y = mx + c is an
the axis of ‘x’ as well as ‘y’. For example, x ®¥ x x ®¥
Y
x3
(0, 3)
y2 =
2a - x
3x + 4y = 12
LHS is positive. If x is negative or if x is greater than 2a, RHS
X′ X becomes negative. Hence the curve lies only in the interval 0 to 2a.
(0, 4) When x → 2a, y → ∞. Therefore, the line x = 2a is an asymptote for
the curve. A rough figure is shown (Fig. 24.3).
Y
Y′
Figure 24.1
X
Illustration 24.2 Sketch the graph for y = x2 − x. O 2a
dy d2 y
4. y = x2 - x Þ = 2 x - 1 and 2 = 2
dx dx
Now, X
−1 O +1
dy 1
=0⇒ x =
dx 2
1 d2 y 1
At x = , > 0 , so x = is point of local minima.
2 dx 2 2
Figure 24.4
dy 1
5. > 0 Þ 2 x - 1> 0 Þ x > The curve has a loop between −1 and 0.
dx 2
y = f(x)
(1/2, −1/4)
Figure 24.2
y=d A y1 y2 B
x = f(y)
C
Q y2
y=c
X
O x=a x=b
X
O
Figure 24.8
Figure 24.6 The values of y1 and y2 are obtained by solving the equation of the
The area between x = f(y), y-axis and the lines y = c and y = d is curve as a quadratic in y whose larger root y1 and smaller root y2
given by are functions of x.
a and b are the coordinates of the points of contact of tangents
d
A = ò f ( y )dy drawn parallel to the y-axis.
c Case III: When two curves (Fig. 24.9) intersect at a point and the area
between them is bounded by x-axis, area bounded by the curves
d b a b
Note: Sometimes it is better to use the formula x dy instead of ∫y
a y = f1(x), y2 = f2(x) and x-axis is ò f1( x )dx + ò f2 ( x )dx ,
c
a a
dx in the computation of area to simplify calculations.
Y
a
Y
Case II: Figure 24.8 represents the region bounded by a closed curve A1 A3 A5
ACQBP. The area of the region bounded by a closed curve ACQBP is X
O x=a x=b
b A6
A2 A4
ò ( y1 - y2 ) dx , y1 > y2
a
Figure 24.10
1074 Mathematics Problem Book for JEE
x2 y2
Illustration 24.5 Find the area of the ellipse + - 1 = 0. Illustration 24.7 Find the area of the segment cut off from the
a2 b 2
parabola y2 = 2x by the line y = 4x−1.
Solution: See Fig. 24.11. The ellipse is symmetrical about both
axes and hence the area enclosed is Solution: The line y = 4x − 1 intersects the parabola y2 = 2x at
A and B
4 × (area under the curve in a quadrant)
2x = (4x − 1)2 ⇒ 16x2 − 10x + 1 = 0
Y ⇒ (8x − 1) (2x − 1) = 0
Therefore,
B
P (x, y) æ1 ö æ 1 1ö
A = ç , 1÷ and B = ç , - ÷
è 2 ø è8 2ø
a a
x2 C
= 4 ∫ y dx = 4 ∫ b 1 − dx
0 0
a2
O X
a
4b ⎡ x a2 − x 2 a2 −1 x ⎤
a B
4b
= ∫
a 0
(a2 − x 2 ) dx = ⎢
a ⎢ 2
+ sin
2 a⎥
⎥
⎣ ⎦0
Figure 24.13
4b é a2p ù
= ê ú = p ab sq. units 1 1
æ y + 1 y2 ö
a êë 4 úû Required area = ò ( x 2 - x1) dy = ò çç
4
- ÷÷ dy
2 ø
y = -1 2 - 1/ 2 è
1
Illustration 24.6 Find the area included between the parabolas é y2 y y3 ù
=ê + - ú æ 1 1 1ö æ 1 1 1 ö
y2 = 4ax and x2 = 4ay. = + - - - +
êë 8 4 6 ûú -1/ 2 çè 8 4 6 ÷ø çè 32 8 48 ÷ø
Solution: See Fig. 24.12. The two parabolas intersect at O (0, 0) and (3 + 6 - 4 ) (3 - 12 + 2)
A (4a, 4a). = -
24 96
Y
5 7 27 9
x 2 = 4ay = + = = sq. units
24 96 96 32
y 2 = 4ax
Illustration 24.8 Find the area between the curves y = x2 + x −2
P A and y = 2x, for which |x2 + x − 2| + | 2x | = |x2 + 3x − 2| is satisfied.
Solution: To find the area between the curve y = x2 + x −2 and
Q y = 2x such that
O X |x2 + x − 2| + |2x| = |x2 + 3x − 2|
So, (x2 + x − 2) and 2x have same sign (Fig. 24.14).
Thus,
Required area = ar (PQR) + ar (STN)
Figure 24.12 0 2
= ò [2 x - ( x 2 + x - 2)] dx + ò [2 x - ( x 2 + x - 2)] dx
The area included between the two curves is area OQAP, that is,
-1 1
x = 4a 0 2
4a
æ x2 ö é x2 x3 ù é x2 x3 ù
∫ ( y1 − y 2 ) dx =
ò çç 2 a x- ÷ dx
4 a ÷ø
= ê - + 2x ú + ê - + 2x ú
x=0 0 è êë 2 3 úû -1 êë 2 3 úû1
Chapter 24 | Area Under the Curves 1075
T
S
X′ (0, 0) X
−1 P p /4 5p /6 5p /3 2p
(−2, 0) N(1, 0) Y′
Q R
Figure 24.16
π
for 0 ≤ x < , f(x) = cos x
4
Figure 24.14 π 5π
for ≤ x < , f(x) = sin x
4 6
Illustration 24.9 Find out the area enclosed by circle |z| = 2,
5π 5π
é x xù for ≤x< , f(x) = 1/2
parabola y = x2 + x + 1, the curve y = êsin2 + cos ú and x-axis 6 3
ë 4 4û
5p
(where [.] is the greatest integer function). for ≤ x ≤ 2p , f(x) = cos x
3
Solution: See Fig. 24.15. For x ∈ [−2, 2] Hence, required area is
Y p /4 5p / 6 5p / 3 2p
(0, 2) 1
∫ cos x dx + ∫ sin x dx + ∫ 2
dx + ∫ cos x dx
( − 3, 0 ) 0 p /4 5p / 6 5p / 3
1 5p / 3
= [ sin x ] + [ − cos x ]p / 4 + [ x ] + [sin x ]25pp / 3
p /4 5p / 6
(−1, 0) ( 3, 0 )
0
2 5p / 6
X′ X
(−2, 0) (−1/2, 0) (0, 0) (2, 0) ⎛ 5p ⎞
=⎜ + 2 + 3 ⎟ sq. units
⎝ 12 ⎠
6. Let y be the function that passes through (1, 2) having slope |x|
(2x + 1). The area bounded between the curve and x-axis is 2. The area bounded by y = , x ≠ 0 , and the lines y(x − 1) (x − 3)
= 0 is x
(A) 6 sq. units (B) 5/6 sq. units
(A) 3 (B) 1 (C) 2 (D) None of these
(C) 1/6 sq. units (D) None of these Ans. (C)
Solution: See Fig. 24.18.
7. Area bounded by the curve x2 = 4y and the straight line
Y
x = 4y − 2 is given by |x|
y= , x ≠ 0,
8 9 x
(A) sq. units (B) sq. units
9 8 y ( x − 1)( x − 3) = 0 1
Area = 2 × 1 X' X
(C) 4 sq. units (D) None of these Ans. (B) 2
3 = 2 sq. units x=1 x=3
8. The area of the region bounded by the curve y = x | x |, x-axis
and the ordinates x = 1, x = −1 is given by
1 Y'
(A) Zero (B)
3 Figure 24.18
2
(C) (D) 1 Ans. (C) Hence, the correct answer is option (C).
3
9. If the area bounded by y = ax and x = ay , a > 0, is 1, then a =
2 2 3. The area bounded by the curve | x | = cos−1y and the line |x| = 1
1 and the x-axis is
(A) 1 (B) (A) cos 1 (B) sin 1 (C) 2 cos 1 (D) 2 sin 1
3
1 Solution: See Fig. 24.19.
(C) (D) None of these Ans. (B)
3
| x | = cos−1y and
10. The area bounded by the curves y = x , 2y + 3 = x and x-axis line |x| = 1 and x-axis
in the first quadrant is y = cos|x|
27 |x| = 1, x-axis
(A) 9 (B) (C) 36 (D) 18 Ans. (A)
4 1 −1 1
11. The area enclosed between the curve y = loge(x + e) and the Area = 2∫ cos x dx
coordinate axes is 0
(A) 3 (B) 4 (C) 1 (D) 2 Ans. (C)
= 2 [ sin x ]0 = 2 sin1 sq. units
1
1 2 2 4 3a
=- + + +2- (8p − 9 3 ) a2
= 2 ∫ ( 4 a2 − ( x − 2a)2 − ax ) dx = sq. units
e2 e e2 e 3
0
1 2
= 2 + 2 − sq. units Hence, the correct answer is option (D).
e e
7. The area {(x, y); x2 ≤ y ≤ x } is equal to
Y
1 2 1
(A) (B) (C) (D) None of these
3 3 6
Solution: See Fig. 24.22.
X' X {(x, y); x2 ≤ y ≤ x }
1
Area = ∫ ( x − x 2 )dx
0
Y' 1
é x 3/2 x 3 ù 2 1 1
Figure 24.20 =ê - ú = - = sq. units
êë 3 / 2 3 úû 0 3 3 3
Hence, the correct answer is option (A).
x2
5. The whole area of the curves x = a cos3 t, y = b sin3 t is given by
3 5 1 Y x
(A) πab (B) πab (C) πab (D) None of these
8 8 8
Solution:
a
dx X'
Area = 4 ∫ y ⋅ dt X
0
dt
0
=4 ò -3ab sin4 t × cos2 t × dt
Y'
p /2
3 1 1 p Figure 24.22
= 4 ´ 3ab × × × ×
6 4 2 2
Hence, the correct answer is option (A).
3
= p ab sq. units 8. The area enclosed by the curve y = x5, the x-axis and the
8
ordinates x = −1, x = 1 is
Hence, the correct answer is option (A).
1 1
(A) (B) 1 (C) (D) 0
6. Area common to the curves y2 = ax and x2 + y2 = 4ax is equal to 2 3
a2
(A) (9 3 + 4p ) (B) (9 3 + 4p )a2 Solution: See Fig. 24.23.
3
y = x5, x = ±1
a2
(C) (9 3 - 4p ) (D) None of these 1
x6
1
2 1
3 Area = 2 ò x 5dx = 2 = = sq. units
0
6 6 3
Solution: See Fig. 24.21. 0
y2 = ax, x2 + y2 − 4ax = 0 Y
Y
y = 4 ax − x
2 2
y = x5
x = −1
x2 + ax – 4ax = 0
x2 − 3ax = 0
X' X X' X
x (x − 3a) = 0
x = 0, x = 3a x =1
Y'
Figure 24.21 Y'
3a Figure 24.23
Required area = 2 ∫ ( 4 ax − x 2 − ax ) dx
0
Hence, the correct answer is option (C).
1078 Mathematics Problem Book for JEE
9. The area bounded by the curve y2 = 9x and the lines x = 1, x = 4 11. The area bounded by the axes of reference and normal to
and y = 0 in the first quadrant is y = loge x at the point (1, 0) is
(A) 14 (B) 7 (C) 28 (D) None of these (A) 1 sq. units (B) 2 sq. units
1
Solution: See Fig. 24.24. (C) sq. units (D) None of these
2
4 4
x 3/2 4 Solution: See Fig. 24.26.
Area = ∫ 3 x dx = 2 × 3 = 2 ⎡⎣ x 3 / 2 ⎤⎦
3 1 y = ln x
1 1
dy 1
= 2 ⎡⎣23 − 1⎤⎦ = 14 sq. units =
dx x
Y At x = 1,
y 2 = 9x
slope of normal = −1
y = −1(x − 1)
y+x=1
1 1
X Area = ´ 1´ 1 = sq. units
2 2
x=1 x=4 Y 1
x=
y+ x
log e
1 y=
Figure 24.24
X
Hence, the correct answer is option (A). 1
10. The slope of the tangent to a curve y = f (x) at (x, f (x)) is 2x + 1.
If the curve passes through the point (1, 2), then the area
of the region bounded by the curve, the x-axis and the line
x = 1 is
5 6 1
(A) (B) (C) (D) 6
6 5 6 Figure 24.26
Solution: See Fig. 24.25. Hence, the correct answer is option (C).
f ′(x) = 2x + 1 12. The area bounded by the line |x| + |y| = 1 is
⇒ f (x) = x2 + x + c (A) 4 (B) 2 (C) 1 (D) None of these
The curve passes through (1, 2), so
Solution: See Fig. 24.27.
2=1+1+c |x| + |y| = 1
⇒c=0
f (x) = x2 + x æ1 ö
Area = 4 ´ ç ´ 1´ 1÷ = 2 sq. units
1
é 3 2 ù1
1 1 5 è2 ø
ò(x
2
+ x ) dx = ê x + x ú = + = sq. units Y
0 ë3 2 û0 3 2 6
(0, 1)
Y
y = x2+ x
1/2 1/2
X' X
(−1, 0) (1, 0)
1/2 1/2
X
x=1
(0, −1)
Y'
Figure 24.27
Hence, the correct answer is option (B).
Figure 24.25
13. If area bounded by curve f(x) and x-axis, x = 1 to x = b is (b − 1)
Hence, the correct answer is option (A). sin (3b + 4), then f(x) is
Chapter 24 | Area Under the Curves 1079
16 1 17
= + = sq. units
4 4 4
Y (1,0) X
y = x3
x + 2y 2 = 0
(−2, −1)
x + 3y = 1
2
15. The area of the region bounded by y = |x − 1| and y = 1 is Hence, the correct answer is option (D).
1 3. The area of the region bounded by the parabola (y − 2)2 = x − 1,
(A) (B) 1
2 the tangent to the parabola at the point (2, 3) and the x-axis is
(C) 2 (D) None of these
(A) 3 (B) 6 (C) 9 (D) 12
Solution:
1 [AIEEE 2009]
Area = ´ 1´ 2
2 Solution: Equation of tangent at (2, 3) for the parabola, (y − 2)2
= 1 sq. unit = x − 1, is S1 = 0, which implies that x − 2y + 4 = 0.
See Fig. 24.30. The required area is
Hence, the correct answer is option (B).
Area of ΔOCB + Area of OAPD − Area of ΔPCD
Previous Years' Solved JEE Main/AIEEE 1
3
⎡1 ⎤
= ( 4 × 2) + ∫ ( y 2 − 4 y + 5)dy − ⎢ (1× 2)⎥
Questions 2 0
⎣ 2 ⎦
Alternate solution: Y
The area is
y=x
Y
2y = x + 4
D(0, 3)
P(2, 3) 1
y=
x
C(0, 2) X
O 1 e
A
X' X Figure 24.31
B(−4, 0) O
Hence, the correct answer is option (B).
y
6. The area bounded between the parabolas x 2 = and x2 = 9y,
Y' 4
and the straight line y = 2 is
Figure 24.30
10 2
(A) 20 2 (B)
3 3 3
A ∫ (2 y − 4 − y 2 + 4 y − 5)dy = ∫ ( − y 2 + 6 y − 9 )dy 20 2
(C) (D) 10 2
0 0 3
[AIEEE 2012]
3
⎡ ( y − 3) 3 ⎤3
27
= − ∫ (3 − y )2 dy = ⎢ ⎥ = = 9 sq. units Solution: From Figure 24.32, the required area is calculated as
0 ⎣ 3 ⎦0 3 2
⎡2 ⎛ y⎞ ⎤
2
5 y ⎡ y 3/2 ⎤
Hence, the correct answer is option (C). A = 2 ⎢∫ ⎜ 3 y − ⎟ dy ⎥ = 2∫ dy = 5 ⎢ ⎥
⎢⎣ 0 ⎝ 2 ⎠ ⎥⎦ 0
2 ⎣ 3 / 2 ⎦0
4. The area bounded by the curves y = cos x and y = sin x between
3p 10 3 / 2 20 2
the ordinates x = 0 and x = is = [2 − 0 ] = sq. units
2 3 3
(A) 4 2 + 2 (B) 4 2 - 1 Y x 2 = 9y y
x2=
(C) 4 2 + 1 (D) 4 2 - 2 4
[AIEEE 2010]
y=2
Solution: The required area is
p 5p 3p
4 4 2
= ( 2 − 1)+
+ 2 2 +( −1+ 2 ) = 4 2 − 2 sq. units 7. The area (in square units) bounded by the curves y = x ,
2 y - x + 3 = 0 , x-axis, and lying in the first quadrant is
Hence, the correct answer is option (D).
1 (A) 36 (B) 18
5. The area of the region enclosed by the curves y = x, x = e, y = 27
x (C) (D) 9
and the positive x-axis is 4
[JEE MAIN 2013]
3
(A) 1 sq. units (B) sq. units Solution: First solving the equations, we have
2
5 1 2 x = x -3 (1)
(C) sq. units (D) sq. units
2 2
[AIEEE 2011] Squaring on both sides of Eq. (1), we get
Solution: From Figure 24.31, we have,
4 x = x 2 − 6 x + 9 ⇒ x 2 − 10 x + 9 ⇒ x = 9 , x = 1
1 e
1 1 3
Area = ∫ x dx + ∫ dx = + 1 = sq. units Since x = 1 intersects the parabola below the x-axis, this point is
0 1
x 2 2
extraneous.
Chapter 24 | Area Under the Curves 1081
So, for x = 9 we have, y = 3. 9. Let A = {(x, y): y2 ≤ 4x, y − 2x ≥ − 4}. Then the area (in square
Therefore, the required area under the curve (see Fig. 24.33) is units) of the region A is
3 (A) 8 (B) 9 (C) 10 (D) 11
3
⎡
2 ⎤ dy ⇒ y 2 + 3 y − y
3⎤
X 2 ± 4 + 32 2 ± 6
(3, 0) (9, 0) y= = = 4 , -2
2 2
Therefore,
(0, −3/2) x = 4, 1 and P is (1, − 2) and Q is (4, 4)
Y' Y
Figure 24.33
Q
Hence, the correct answer is option (D).
8. The area of the region described by A = {(x, y): x2 + y2 ≤ 1 and
y2 ≤ 1 − x} is
X
p 2 p 2
(A) - (B) +
2 3 2 3
p 4 p 4
(C) + (D) - P
2 3 2 3 y 2 = 4x
y2
[JEE MAIN 2014 (OFFLINE)] y = 2x − 4 x=
4
Solution: See Fig. 24.34. x = (y + 4)/2
Y
Figure 24.35
1
4
ìïæ 4 + y ö y 2 üï
Required area = ò íïçè ÷ - ý dy
2 ø 4 ïþ
-2 î
4
æ y2 ö é y2 y3 ù
4
1
X' X = ò çç 2 + y - ÷÷ dy = ê2 y + - ú
(1, 0) -2 è
2 4 ø êë 4 12 ûú -2
æ 64 ö æ 4 8 ö
= ç 8 + 4 - ÷ - ç 2( -2) + + ÷
è 12 ø è 4 12 ø
⎛ 72 ⎞
= 12 + 4 − 1− ⎜ ⎟ = 15 − 6 = 9 sq. units
⎝ 12 ⎠
Y'
Hence, the correct answer is option (B).
Figure 24.34
10. The area of the region above the x-axis bounded by the curve
y2 = 1 − x ⇒ x = 1 − y2 p p
y = tan x ,0 £ x £ and the tangent to the curve at x = is
1 1 2 4
Required area = (p ´ 12 ) + 2 ò (1- y 2 )dy
2 0
1æ 1ö 1æ 1ö
(A) ç log 2 - ÷ (B) ç log 2 + ÷
3 ⎤1 2è 2ø 2è 2ø
p ⎡ y p ⎡⎛ 1⎞ ⎤ p 4
= + 2 ⎢ y − ⎥ = + 2 ⎢⎜ 1− ⎟ − 0 ⎥ = + sq. units 1 1
2 ⎣ 3 ⎦0 2 ⎣ ⎝ 3 ⎠ ⎦ 2 3 (C) (1- log 2 ) (D) (1+ log 2 )
2 2
Hence, the correct answer is option (C). [JEE MAIN 2014 (ONLINE SET-4)]
1082 Mathematics Problem Book for JEE
Y' 1
1 y2
1
Figure 24.36
= ∫ 4 ( y + 1)dy − ∫ 2 dy
−1/ 2 −1/ 2
p
⎛p ⎞
Required area = ∫ tan x − area under tangent at ⎜ ,1⎟ (1) 1 ⎡ y2 ⎤
1
1 ⎡ y3 ⎤ 9
1
⎝4 ⎠ =
0 ⎢ + y⎥ − ⎢ ⎥ = sq. units
4⎣ 2 ⎦ −1/ 2 2 ⎣ 3 ⎦ −1/ 2 32
d p
Now slope of tangent is tan x at x = = sec2 x p =2
dx 4 at x =
4 Hence, the correct answer is option (C).
æ pö æ pö 12. The area (in square units) of the region bounded by the curves
Therefore, equation of tangent is y - 1 = 2 ç x - ÷ or y = 2 x + ç 1- ÷
è 4ø è 2ø y + 2x2 = 0 and y + 3x2 = 1 is equal to
This tangent cuts x-axis when y = 0 3 3 1 4
(A) (B) (C) (D)
Therefore, 5 4 3 3
p
-1 [JEE MAIN 2015 (ONLINE SET-1)]
p 1
x= 2 = - Solution: See Fig. 24.38.
2 4 2
C1: y + 2x2 = 0;
Thus, required area is
p C2: y + 3x2 = 1
[logsec x ]4
0 − area triangle A B C
Y
1 ⎛ p p 1⎞ 1⎡ 1⎤
= log 2 − 0 − × ⎜ − + ⎟ × 1 = ⎢log 2 − ⎥ sq. units
2 ⎝ 4 4 2⎠ 2⎣ 2⎦
Hence, the correct answer is option (A). 1
−1 1
11. The area (in sq. units) of the region described by {(x, y): ≤ 2x y2 X
and y ≥ 4x − 1} is −1
A B
5 15 9 7 −2
(A) (B) (C) (D)
64 64 32 32
[JEE MAIN 2015 (OFFLINE)]
Solution: See Fig. 24.37.
R = {(x, y): y2 ≤ 2x and y ≥ 4x − 1} Figure 24.38
Y At the point of intersection of C1 and C2
y = 4x − 1 −2x2 = 1 − 3x2 ⇒ x2 = 1 ⇒ x = ±1
Therefore, A (−1, −2) and B (1, −2) are points of intersection as
y 2 = 4x
shown above.
B So, required area is
X′ X 0
O′ 2 ∫ [(1− 3 x 2 ) − ( −2 x 2 )]dx
A
−1
0
0
⎡ x3 ⎤ 4
= 2 ∫ (1− x 2 )dx = 2 ⎢ x − ⎥ = sq. units
−1 ⎣ 3 ⎦ −1 3
Y′
Figure 24.37 Hence, the correct answer is option (D).
Chapter 24 | Area Under the Curves 1083
13. The area (in sq. units) of the region {(x, y): y2 ≥ 2x and x2 + y2 ≤ 4x, y ≥ x2 − 5x + 4
x ≥ 0, y ≥ 0} is y ≥ (x − 1)(x − 4)
p 2 2 4 The area of the region to be measured is
(A) - (B) p - 4 4
2 3 3 1 x3 5x2
× 2 × 2 + ∫ ( x 2 − 5 x + 4 )dx = 2 + − + 4x
8 4 2 2 3 2
(C) p - (D) p - 3 3
3 3
64 5 ´ 16 27 5 ´ 9
[JEE MAIN 2016 (OFFLINE)] =2+ - + 16 - + - 12
3 2 3 2
Solution: We have y2 − 2x ≥ 0 and x2 + y2 − 4x ≤ 0, x ≥ 0, y ≥ 0. 12 + 7 19
(x − 2)2 + y2 ≤ 4 = = sq. units
6 6
Point of intersection of both curves y2 = 2x and (x − 2)2 + y2 = 4 is
Hence, the correct answer is option (A).
(0, 0) and (2, 2) (Fig. 24.39).
Y Previous Years' Solved JEE Advanced/
Q (2, 2)
IIT-JEE Questions
Paragraph for Questions 1–3: Consider the functions defined
P
(0, 0) (2, 0)
X implicitly by the equation y3 − 3y + x = 0 on various intervals in the
real line. If x ∈ (−∞, −2) ∪ (2, ∞), the equation implicitly defines a
unique real valued differentiable function y = f(x).
If x ∈ (−2, 2), the equation implicitly defines a unique real valued
differentiable function y = g(x) satisfying g(0) = 0.
[IIT-JEE 2008]
Figure 24.39 1. If f ( -10 2 ) = 2 2 , then f ¢¢( -10 2 ) =
The required area is
4 2 4 2
2 2 (A) (B) -
7332 7332
ò ( y1 - y2 )dx = ò ( 4 x - x - 2 x )dx
2
0 0 4 2 4 2
(C) (D) -
p (2 )
2 2 733 733
= - 2 ò x dx
4 Solution: We have
0
y3 − 3y + x = 0
⎛ 8⎞
= ⎜ p − ⎟ sq. units Differentiate both sides, we get
⎝ 3⎠ 3y2 y ′ − 3y ′ + 1 = 0 (1)
Hence, the correct answer is option (C). Put y = 2 2 , x = −10 2 . Then
14. The area (in sq. units) of the region described by A = {(x, y) | y −1
≥ x2 − 5x + 4, x + y ≥ 1, y ≤ 0} is y ′( −10 2 ) =
21
19 17 Differentiate equation (1), we get
(A) (B)
6 6
3y2 y ′′ + 6y(y ′)2 − 3y ′′ = 0
7 13
(C) (D) −1
2 6 Put y = 2 2 , x = −10 2 , y ′ = . Then
21
[JEE MAIN 2016 (ONLINE SET-1)]
4 2
Solution: See Fig. 24.40. y ′′( −10 2 ) = −
73 ⋅ 32
Y Hence, the correct answer is option (B).
Area to be 2. The area of the region bounded by the curves y = f(x), the x-axis,
measured and the lines x = a and x = b, where −∞ < a < b < −2, is
b
2 (3, 0) x
(1,0) (4,0)
X (A) ò 3((f ( x ))2 - 1) dx + bf (b) - af (a)
a
x+
y= b
x
(3, −2)
1 (B) - ò dx + bf (b ) - af (a)
a 3(( f ( x )) - 1)
2
Figure 24.40 b
x
We have (C) ò 3((f ( x ))2 - 1) dx - bf (b) + af (a)
A = {(x, y)| y ≥ x2 − 5x + 4, x + y ≥ 1, y ≤ 0} a
1084 Mathematics Problem Book for JEE
b
x ⎛
p x ⎞
(D) - ò dx - bf (b ) + af (a) ⎜ 2 tan
4 ⎟
a 3(( f ( x )) - 1)
2
= ∫⎜ 2 ⎟ dx
0 ⎜ 1 − tan2
x⎟
Solution: ⎜⎝ ⎟
2⎠
b
Required area = ∫ f ( x )dx x
a
Put tan = t . Then
2
b
= [ xf ( x )]ba − ∫ xf ′( x )dx (By parts) 1 2x
a sec dx = dt
b
2 2
xdx 2dt
= bf (b ) − af (a) + ∫ ⇒ dx =
a
3[ f ( x )2 − 1] 1+ t 2
2 −1⎛ ⎞
Hence, the correct answer is option (A). 4t
1
⇒ ∫ ⎜
⎝ (1+ t ) 1− t ⎠
2 2
⎟ dt
0
3. ò g¢( x )dx =
-1 Hence, the correct answer is option (B).
(A) 2g(−1) (B) 0 (C) −2g(1) (D) 2g(1) 5. Area of the region bounded by the curve y = ex and lines x = 0
Solution: and y = e is
e
1
y′ =
3[1− (f ( x ))2 ]
(A) e − 1 (B) ò ln(e + 1- y )dy
1
1 e
Clearly f(x) is an odd function, then g ′(x) is an even function, so
(C) e - ò e x dx (D) In y dy
1 1
0 1
∫ g′( x )dx = 2∫ g′( x )dx [IIT-JEE 2009]
−1 0
1+ sin x Also,
4. The area of the region between the curves y = and e e
cos x
ò ln y dy = ò ln(e + 1- y )dy
1- sin x p 1 1
y= bounded by the lines x = 0 and x = is
cos x 4 Further the required area can be written as
2 -1 2 -1 1
t 4t e × 1− ∫ e x dx
(A) ò (1+ t 2 ) 1- t 2
dt (B) ò (1+ t 2 ) 1- t 2
dt
0
0 0
2 +1 2 +1 y y = ex
4t t
(C) ò (1+ t 2 ) 1- t 2
dt (D) ò (1+ t 2 ) 1- t 2
dt
0 0
y=e
[IIT-JEE 2008]
⎛ p⎞
Solution: Since, both curves lie above x-axis in x ∈⎜ 0 , ⎟ .
⎝ 4⎠
(0, 1)
Therefore, area bounded between the curve is
x
p x=1
4 ⎛ 1+ sin x 1− sin x ⎞
∫ ⎜⎝ cos x
−
cos x ⎟⎠
dx Figure 24.41
0
Hence, the correct answers are options (B), (C) and (D).
⎛ p x x⎞
⎜ 1+ tan 2
4 1− tan ⎟ Paragraph for questions 6–8: Consider the polynomial f(x) = 1 +
= ∫⎜ − 2 2x + 3x2 + 4x3. Let s be the sum of all distinct real roots of f(x) and
x⎟
dx
x
0 ⎜ 1 − tan 1+ tan ⎟ let t = |s|.
⎝ 2 2⎠
[IIT-JEE 2010]
Chapter 24 | Area Under the Curves 1085
⎡ ⎡ 3 ⎤b 3⎤ 1
⎛ 3 1⎞
⇒ ⎢ ( x − 1) ⎥ − ⎢ ( x − 1) ⎥ =
1
Þ S lie in ⎜ − , − ⎟
⎝ 4 2⎠ ⎣ 3 ⎦0 ⎣ 3 ⎦b 4
Hence, the correct answer is option (C). (b − 1)3 1 ⎛ (b − 1)3 ⎞ 1
⇒ + − ⎜0 − =
7. The area bounded by the curve y = f(x) and the lines x = 0, y = 0 3 3 ⎝ 3 ⎟⎠ 4
and x = t, lies in the interval 2(b − 1)3 1 1 1
⇒ = − ⇒ (b − 1)3 = − ⇒ b =
æ3 ö æ 21 11 ö 3 12 8 2
(A) ç , 3 ÷ (B) ç , ÷
è4 ø è 64 16 ø
æ 21 ö
(C) (9, 10) (D) ç 0 , ÷
è 64 ø
Solution: R1
R2
3 1
- <s<- o b 1
4 2
1 3 Figure 24.42
<t <
2 4 Hence, the correct answer is option (B).
1/ 2 3/ 4
10. Let f: [−1, 2] → [0, ∞] be a continuous function such that
ò (4 x + 3 x 2 + 2 x + 1)dx < area < ò (4 x + 3 x 2 + 2 x + 1)dx
3 3
2
0 0
(A)
æ 1ö æ 1 ö
increasing in ç -t, - ÷ and decreasing in ç - , t ÷
R1 = ò xf ( x )dx = ò (2 - 1- x )f (2 - 1- x )dx
è 4ø è 4 ø -1 -1
æ 1ö 2 2
(B) decreasing in ç -t, - ÷ and increasing in æ - 1 , t ö = ò (1- x )f (1- x )dx = ò (1- x )f ( x )dx
è 4ø ç ÷
è 4 ø -1 -1
(C) increasing in (−t, t)
(D) decreasing in (−t, t) 2
Hence, 2R1 = ò f ( x )dx = R2 .
Solution: -1
f(x) = 4x3 + 3x2 + 2x + 1
f ′(x) = 12x2 + 6x + 2 Hence, the correct answer is option (C).
f ′′(x) = 2[12x + 3] = 0 ⇒ x = −1/4 11. Let S be the area of the region enclosed by y = e−x , y = 0, x = 0
2
é 1 1 æ 1 ö æ 1 öù
= -(0 - 1) + (1- 0 ) - ê + - 1- ç 0 - ÷ - ç 1- ÷ú
A (0, 1) P (1 2 , 1) è 2ø è 2 øû
ë 2 2
é 1 1 ù
= 2 - ê 2 - 1+ - 1+ ú = 2 - éë2 2 - 2 ùû
ë 2 2û
= 4 - 2 2 = 2 2 ( 2 - 1)
B (1/ 2, 1/ e)
R Hence, the correct answer is option (B).
C D (1, 1/e ) 13. For a point P in the plane, let d1 (P) and d2 (P) be the distances
of the point P from the lines x − y = 0 and x + y = 0 respectively.
The area of the region R consisting of all points P lying in the
O S X
first quadrant of the plane and satisfying 2 ≤ d1 (P) + d2 (P) ≤ 4,
Figure 24.43
is _____.
1 [JEE ADVANCED 2014]
S> (As area of rectangle OCDS = 1/e)
e
Solution:
Since, Y
e−x ≥ e−x ∀
2
x ∈ [0, 1] 2 2 Y=X
Y = −X
P(T, B)
1 Required
æ 1ö
Þ S > ò e - x dx = ç 1- ÷ d2
Area
0 è eø 2
1æ 1 ö |x-y|
1 d1 =
ç 1+ ÷ < 1- 2
4è eø e
|x+y|
d2 =
Hence, the correct answers are options (A), (B) and (D). 2
12. The area enclosed by the curves y = sin x + cos x and Therefore, according to the question (Fig. 24.45)
é pù
y = cos x - sin x over the interval ê0 , ú is
ë 2û |x-y| |x+y|
2£ + £4
(A) 4( 2 - 1) (B) 2 2 ( 2 - 1) 2 2
2 2 £ x -y + x + y £4 2 Þ 2 £ x £2 2
p
x2 + Column I Column II
6
é 1ù
14. Let F ( x ) = ò 2 cos t dt for all x ∈\ and f : ê0 , ú ® [0 , ¥ )
2
(D) Suppose that F(a ) denotes the area of the region (S) 5
x ë 2û bounded by x = 0, x = 2, y 2 = 4x and y = |a x − 1| +
|a x − 2| + a x, where α ∈ {0, 1} Then the value(s) of
é 1ù
be a continuous function. For aÎ ê0 , ú , if F ′(a) + 2 is the area 8
ë 2û F(a ) + 2 , when a = 0 and a = 1, is (are)
3
of the region bounded by x = 0, y = 0, y = f(x) and x = a, then
f(0) is _____. (T) 6
[JEE ADVANCED 2015]
[JEE ADVANCED 2015]
Solution: We have Solution: See Fig. 24.46.
a
2(a2 − b2) = c2 (1)
F ¢(a) + 2 = ò f ( x )dx
0 sin( x - y )
l= (2)
sin z
Differentiating both sides, we get
cos (nπλ) = 0 (3)
F ¢¢(a) = f (a)
Now, (2m + 1)
Þ nl = (4)
x 2 +p /6 2
∫
2
F( x ) = 2 cos tdt
From Eq. (2),
x
sin x cos y - cos x sin y
æ pö l=
F ¢( x ) = 2 x × 2 cos2 ç x 2 + ÷ - 2 cos2 x sin z
è 6ø
a cos y - b cos x
⎛ p⎞ ⎛ p⎞ Þl = (By Sine formula)
⇒ F ′′( x ) = −16 x 2 cos ⎜ x 2 + ⎟ sin ⎜ x 2 + ⎟ + 4 cos x sin x c
⎝ 6 ⎠ ⎝ 6⎠
æ a2 + c 2 - b 2 ö æ b 2 + c 2 - a2 ö
⎛ p⎞ a çç ÷÷ - b çç ÷÷
+ 4 cos2 ⎜ x 2 + ⎟ 2ac 2bc
⎝ 6⎠ Þl = è ø è ø
⎛ p⎞ ⎛ p⎞ c
⇒ F ′′(a) = −16a2 cos ⎜ a2 + ⎟ sin ⎜ a2 + ⎟ + 4 cos a sin a
⎝ 6⎠ ⎝ 6⎠ 2(a2 - b2 ) 1
Þl = = (5)
⎛ p⎞ 2c 2 2
+ 4 cos ⎜ a2 + ⎟
2
⎝ 6⎠ Therefore, from Eqs. (4) and (5),
⎛p ⎞ ⎛ 3⎞ n 2m + 1
2
⇒ f (0) = 4 cos ⎜ ⎟ = 4 ⎜ ⎟ = 3 = Þ n = (2m + 1)
⎝ 6⎠ ⎝ 4⎠ 2 2
So,
Hence, the correct answer is (3).
(A) → (P), (R), (S)
15. Match the Column I to Column II.
Checking option (B):
Column I Column II x
(A) In ΔXYZ, let a, b and c be the lengths of the (P) 1
sides opposite to the angles X, Y and Z, c
b
sin( X - Y )
respectively. If 2(a2 − b2) = c2 and l = ,
sin Z
Y z
then possible values of n for which cos (nπλ) = 0 a
is (are)
Figure 24.46
(B) In ΔXYZ, let a, b and c be the lengths of the sides (Q) 2
1 + cos 2x − 2cos 2y = 2 sin x sin y
opposite to the angles X, Y and Z, respectively. If
1 + cos 2X − 2 cos 2Y = 2 sin X sin Y, then possible ⇒ 2 cos2 x − 2(2 cos2y − 1) = 2sin x sin y
a ⇒ 2 cos2 x − 4 cos2y + 2 = 2 sin x sin y
value(s) of is (are)
b ⇒ 2 sin2y − 2 sin x sin y + sin x sin y − sin2x = 0
⇒ 2 sin y(sin y − sin x) + sin x(sin y − sin x) = 0
(C) In \ 3, let 3i + j , i + 3 j , and b i + (1- b )j be (R) 3 ⇒ (sin y − sin x) (2 sin y + sin x) = 0
the position vectors of X, Y and Z with respect
⇒ b = a or 2b = −a (impossible)
to the origin O, respectively. If the distance of Z
a
from the bisector of the acute angle of OX with ⇒ =1
b
3
OY is , then possible value(s) of |β| is (are) So,
2
(B)→ (P)
1088 Mathematics Problem Book for JEE
æ 8 ö 1 8
= ç6 - 2 ÷ - (2)(1) = 5 - 2
x è 3 ø 2 3
O
8
Þ F (1) + 2 = 5 Þ (S )
Figure 24.47 3
Therefore,
Vector along the bisector of acute angle between OX and OY is ,
(D) → (T), (S)
3i + j i + 3 j ( 3 + 1) Hence, the correct matches are (A) → (P), (R), (S); (B) → (P); (C) →
+ = (i + j )
2 2 2 (P), (Q); (D) → (S), (T).
Slope of OB = tan(p /4 ) = 1 16. The area of the region {( x , y ) ∈ \ 2 : y ≥ x + 3 , 5 y ≤ x + 9 ≤ 15}
⇒ Equation of OB is y = x
is equal to
Since, 1 4
(A) (B)
b - (1- b ) 3 6 3
ZL = 3/ 2 , Þ =
2 2 3 5
(C) (D)
⇒ |2b − 1| = 3 2 3
⇒ (2b − 1) = ± 3 [JEE ADVANCED 2016]
⇒ b = 2 or b = −1 Solution: It is given that
⇒ |b | = 1 or 2
Therefore, (C)→ (P), (Q). y³ x +3
Checking option (D): See Fig. 24.48.
That is,
Y
y = 3x − 3 ìï ( x + 3), x ³ -3
x +3 = í
y=3−x îï - x - 3 , x < -3
y=x−1 y 2 = 4x
D C It is also given that
y=3
3 B 5y ≤ x + 9 ≤ 15
2
A That is,
1
X x + 9 ≤ 15 ⇒ x ≤ 6
1 2 3
5y ≤ 15 ⇒ y ≤ 3
x=2 5y ≤ x + 9
Figure 24.48 From Fig. 24.49, we have
y = |a x − 1| + |a x − 2| + a x; a ∈ {0, 1} y
Case (I) For a = 0, y = 3 (6, 3)
5y = x + 9 3 S
Case (II) For a = 1, y = |x − 1| + |x − 2| + x
P 9 3
ì 3 - x; x £1 R
ï Q x
Þ y = í x + 1; 1 < x < 2 (6, 0)
(−4, 0)(−3, 0)
ï3 x - 3; x ³2
î
-3 6
(A) 3 − e (B) e − 3
2 2 1 1
= 20 + ( - x - 3)3 / 2 - ( x + 3)3 / 2 (C) (3 - e ) (D) (e - 3)
3 -4 3 -3 2 2
2 2 10. The area between the parabola y2 = 4ax and x2 = 8ay is
= 20 + (0 - 1) - 93 / 2
3 3 8 2 4 2
(A) a (B) a
3 3
2 2
= 20 - - ´ 27 32 2 16 2
3 3 (C) a (D) a
3 3
2 4 11. The area of the region bounded by the curves y = x2 and y = |x | is
= 2 - = sq. units.
3 3 (A) 1/6 (B) 1/3
Hence, the correct answer is option (B). (C) 5/6 (D) 5/3
12. The area bounded by curves y = cos x and y = sin x and
p
ordinates x = 0 and x = is
Practice Exercise 1 4
(A) 2 (B) 2 +1
1. The area of the region bounded by y = |x − 1| and y = 1 is
(C) 2 -1 (D) 2 ( 2 - 1)
(A) 2 (B) 1
1 13. The area in the first quadrant between x2 + y2 = π 2 and
(C) (D) None of these y = sin x is
2
(p 3 - 8 ) π3
2. The area between the curve y2 = 4ax, x-axis and the ordinates (A) (B)
x = 0 and x = a is 4 4
4 2 8 2 (p 3 - 16 ) (p 3 - 8 )
(A) a (B) a (C) (D)
3 3 4 2
2 2 5 2 14. The area bounded by the curves y2 − x = 0 and y − x2 = 0 is
(C) a (D) a
3 3 7 1
(A) (B)
3. The area of the curve xy2 = a2(a − x) bounded by y-axis is 3 3
(A) πa2 (B) 2πa2 5
(C) (D) 1
(C) 3πa2 (D) 4πa2 3
1090 Mathematics Problem Book for JEE
15. The area of region {(x, y) : x2 + y2 ≤ 1 ≤ x + y} is 27. If the area above the x-axis, bounded by the curves y = 2kx and
π2 π2 x = 0 and x = 2 is
3
, then the value of k is
(A) (B)
5 2 ln 2
π2 p 1 1
(C) -
(D) (A) (B) 1
3 4 2 2
16. Area under the curve y = sin 2x + cos 2x between x = 0 and (C) −1 (D) 2
p 28. The area bounded by the x-axis, the curve y = f(x) and the lines
x = is
4 x = 1, x = b is equal to b2 + 1 - 2 for all b > 1, then f(x) is
(A) 2 sq. units (B) 1 sq. units
(C) 3 sq. units (D) 4 sq. units (A) x -1 (B) x +1
17. Area under the curve y = 3 x + 4 between x = 0 and x = 4 is x
(C) x2 +1 (D)
56 64 1+ x 2
(A) sq. units (B) sq. units
9 9 29. The area bounded by the circle x2 + y2 = 4, line x = 3 y and
(C) 8 sq. units (D) None of these x-axis lying in the first quadrant is
a2 π π
18. If area bounded by the curves y2 = 4ax and y = mx is , then (A) (B)
3 2 4
the value of m is π
(A) 2 (B) −2 (C) (D) π
3
1
(C) (D) None of these 30. Area bounded by the curve y = log x, x-axis and the ordinates
2 x = 1, x = 2 is
19. Area bounded by parabola y2 = x and straight line 2y = x is (A) log 4 sq. units (B) (log 4 + 1) sq. units
4 (C) (log 4 − 1) sq. units (D) None of these
(A) (B) 1
3 2
31. Area bounded by the curve y = xe x , x-axis and the ordinates
2 1 x = 0, x = a is
(C) (D)
3 3 2 2
ea + 1 ea - 1
20. Area bounded by lines y = 2 + x, y = 2 − x and x = 2 is (A) sq. units (B) sq. units
2 2
(A) 3 (B) 4 2 2
(C) 8 (D) 16 (C) e a + 1 sq. units (D) e a − 1 sq. units
21. The ratio of the areas bounded by the curves y = cos x and 32. Area bounded by the curve y = sin x between x = 0 and x = 2π is
y = cos 2x between x = 0, x = π/3 and x-axis is (A) 2 sq. units (B) 4 sq. units
(A) 2 :1 (B) 1 : 1 (C) 8 sq. units (D) None of these
(C) 1 : 2 (D) 2 : 1 33. Area bounded by the parabola y = 4x2, y-axis and the lines
22. The area bounded by the x-axis and the curve y = sin x and y = 1, y = 4 is
x = 0, x = π is 7
(A) 3 sq. units (B) sq. units
(A) 1 (B) 2 5
(C) 3 (D) 4 7
23. The area bounded by the parabola y2 = 4ax, its axis and two (C) sq. units (D) None of these
3
ordinates x = 4, x = 9 is
(A) 4a2 (B) 4a2.4 34. Area bounded by the lines y = x, x = −1, x = 2 and x-axis is
152 a 5 3
(C) 4a2 (9 − 4) (D) (A) sq. units (B) sq. units
3 2 2
24. For 0 ≤ x ≤ π, the area bounded by y = x and y = x + sin x is 1
(C) sq. units (D) None of these
(A) 2 (B) 4 2
(C) 2π (D) 4π 35. If the ordinate x = a divides the area bounded by the curve
25. The area of the region bounded by the x-axis and the curves
æ 8 ö
defined by y = tan x, (−π / 3 ≤ x ≤ π / 3) is y = ç 1+ 2 ÷ , x-axis and the ordinates x = 2, x = 4 into two
è x ø
(A) log 2 (B) -log 2
equal parts, then a =
(C) 2log2 (D) 0 (A) 8 (B) 2 2
26. If a curve y = a x + bx passes through the point (1, 2) and the (C) 2 (D) 2
area bounded by the curve, line x = 4 and x-axis is 8 sq. units,
then 36. Area between the curve y = cos x and x-axis when 0 ≤ x is
(A) a = 3, b = −1 (B) a = 3, b = 1 (A) 2 (B) 4
(C) a = −3, b = 1 (D) a = −3, b = −1 (C) 0 (D) 3
Chapter 24 | Area Under the Curves 1091
37. Area bounded by curve y = x3, x-axis and ordinates x = 1 and 48. Let C1 and C2 be the graphs of the function y = x 2 and y = 2x,
x = 4 is 0 ≤ x ≤ 1 respectively. Let C3 be the graph of a function y =
(A) 64 sq. units (B) 27 sq. units f(x); 0 ≤ x ≤ 1, f(0) = 0. For a point P on C1, let the lines through
127 255 P parallel to the axis, meet C2 and C3 at Q and R, respectively
(C) sq. units (D) sq. units
4 4 (see Fig. 24.50). If for every position of P on (C1), the areas
38. Area bounded by curve xy = c, x-axis between x = 1 and x = 4 is of shaded region OPQ and ORP are equal, determine the
(A) c log 3 sq. units (B) 2log c sq. units function f(x).
(C) 2c log 2 sq. units (D) 2c log 5 sq. units
y
39. Area bounded by curve y = k sin x between x = π and x = 2π is
(A) 2k sq. units (B) 0 ⎛ 1 , 1⎞
⎜ ⎟
⎝2 ⎠
k2 (0, 1) (1, 1)
(C) sq. units (D) k sq. units C2 C1
2
Q
40. Area bounded by y = x sin x and x-axis between x = 0 and P
x = 2π is
(A) 0 (B) 2π sq. units
(C) π sq. units (D) 4π sq. units O (1, 0) x
41. The ratio in which the area bounded by the curves y2 = x and
R
1 C3
x2 = y is divided by the line x = is
2
Figure 24.50
4 2 -1 3 2 +3
(A) (B) 49. Let f(x) = maximum {x2, (1 − x)2, 2x (1 − x)} where 0 ≤ x ≤ 1.
9-4 2 9-4 2
Determine the area of the region bounded by the curves
2 -1 2 2 -1 y = f (x), x-axis, x = 0 and x = 1.
(C) (D)
3 -1 3 3 -1 50. Let An be the area bounded by the curve y = (1 + tan x)n and the
42. The area of the curve x + |y| = 1 and the y-axis is p 1
lines x = 0, y = 0, and x = . Prove that for n ³ 2, An + An -2 =
(A) 1 sq. unit (B) 2 sq. units 4 n -1
1 1
(C)
1
sq. units (D) 2 sq. units and deduce < An < .
2 2n + 2 2n - 2
43. The area bounded by the curve y = e| x |, y = e−| x |, x ≥ 0 and x ≤ 5 is 51. In what ratio does the x-axis divide the area of the region
bounded by the parabolas y = 4x − x2 and y = x2 − x.
(A) e5 + e−5 + 2 sq. units (B) e5 + e−5 − 2 sq. units
(C) e5 − e−5 + 2 sq. units (D) e5 − e−5 − 2 sq. units 52. Sketch the region bounded by y = x2 and y =
2
and find
44. Find the area of quadrilateral, combined equation of whose (1+ x 2 )
its area.
sides are (x2 − y2)(x2 − y2 − 8x + 16)
(A) 8 (B) 4 53. Sketch the curves and identify the region bounded by
(C) 2 2 (D) 9 x = 1/ 2 , x = 2, y = loge x and y = 2x. Find area of region.
45. Let f be a real valued function satisfying 54. Compute the area of the region bounded by the curves y = ex
æ ln x ö
æxö f (1+ x ) In x, and y = ç ÷.
f ç ÷ = f ( x ) - f ( y ) and lim =3 è ex ø
x ®0
è ø
y x
55. Find all the maxima and minima of the function f(x) = x(x − 1)2,
Find the area bounded by the curve y = f(x), the y-axis and the (0 ≤ x ≤ 2). Also determine the area bounded by the curve
line y = 3. y = x(x − 1)2, the y-axis and the line y = 2.
46. Let An be the area bounded by y = tann x, x = 0, y = 0 and 56. Find the area of the region bounded by the curve whose
x = π/4. Prove that for n ≥ 2. equation is y = tan x, its tangent drawn at x −p /4 and the
1 1 1 x-axis.
(i) An + An − 2 = (ii) < An <
n -1 2(n + 1) 2(n - 1) 57. Find the area bounded by the curves x 2 + y 2 = 4 , x 2 = - 2 y
47. Let f (x) be a continuous function given by and x = y.
is
3
, then the function g (x) is Practice Exercise 2
4
Single/Multiple Correct Choice Type Questions
(A) g(x) = ± 1- x 2 (B) g(x) = 1- x 2
1. For which of the following values of m, is the area of the region
(C) g(x) = − 1- x 2 (D) g(x) = 1+ x 2 9
bounded by the curve y = x − x2 and the line y = mx equals ?
60. Area bounded by y = g(x), x-axis and the lines x = −2, x = 3, 2
ìïmax i : {f (t ); - 2 £ t £ x} , - 2 £ x < 0 (A) − 4 (B) − 2 (C) 2 (D) 4
where g( x ) = í
ïîmin i : {f (t ); 0 £ t £ x} , 0 £ x < 3 2. Three straight lines are drawn through a point M, lying in the
and f(x) = x2 − | x |, is equal to interior of triangle ABC, parallel to its sides. The areas of the
resulting three triangles are S1, S2 and S3. The area of triangle
113 111 ABC is
(A) sq. units (B) sq. units
24 24 (A) S1 + S2 + S3 (B) ( S1 + S2 + S3 )2
117 121
(C) sq. units (D) sq. units ( S1 + S2 + S3 )3 / 2
24 24 (C) (D) None of these
S1 + S2 + S3
61. Area of the region that consists of all the points satisfying the
conditions |x − y| + |x + y| ≤ 8 and xy ≥ 2 is equal to Comprehension Type Questions
(A) 4(7 − ln 8) sq. units (B) 4(9 − ln 8) sq. units
Paragraph for Questions 3–5: Let f: R → R be a continuous
(C) 2(7 − ln 8) sq. units (D) 2(9 − ln 8) sq. units and bijective function defined such that f(α) = 0 (α ≠ 0). The area
62. Two lines draw through the point P (4, 0) divide the area bounded by y = f(x), x = α, x = α − t is equal to the area bounded by
px y = f(x), x = α , x = α + t ∀ t ∈ R, then
bounded by the curves y = 2 sin and x-axis, between
4 3. Graph of y = f(x) is symmetrical about point
the line x = 2 and x = 4, in to three equal parts. Sum of the (A) (0, 0) (B) (0, α)
slopes of the drawn lines is equal to (C) (α, 0) (D) (α, α)
2 2 2 2 4 2 4. The value of f(2α) is equal to
(A) - (B) - (C) - (D) -
p p p p (A) f(α) (B) −f(α)
(C) f(0) (D) −f(0)
63. The area bounded by the curve y = x (3 − x)2, the x-axis and
b
the ordinates of the maximum and minimum points of the -1
curve is
5. The value of òf (t ) dt is equal to
-b
(A) 2 sq. units (B) 6 sq. units (A) 0 (B) 2αβ
(C) 4 sq. units (D) 8 sq. units (C) αβ (D) None of these
64. What is the area of a plane figure bounded by the points of the Paragraph for Questions 6–8: Let f(x) be a polynomial of degree
lines max (x, y) = 1 and x2 + y2 = 1? 4 satisfying
π π
(A) 1− sq. units (B) 1 − sq. units æx öæ x ö æx öæ x ö
2 3 ç ò A(t ) B(t )dt ÷ ç ò C (t ) D(t ) dt ÷ - ç ò A(t ) C (t ) dt ÷ ç ò B(t ) D(t ) dt ÷
ç ÷ç ÷ ç ÷ç ÷
π è1 øè 1 ø è1 øè 1 ø
(C) 1 − sq. units (D) 1 − π sq. units
4 = f(x) ∀ x ∈ R
65. The area bounded by the curve y = (x − 1) (x − 2) (x − 3) lying where A(x), B(x), C(x) and D(x) are non-constant continuous and
between the ordinates x = 0 and x = 3 is differentiable functions. It is given that the leading coefficient
7 (coefficient of x4) of f(x) is 1.
(A) sq. units (B) 4 sq. units
4 6. The area included between the line y = x − 1 and the curve
11 y = f(x) is
(C) sq. units (D) 3 sq. units
4 2 3
(A) sq. units (B) sq. units
66. The area common to the curves y = and y = x is
x3 5 10
(A) 2 (B) 4 7 7
(C) sq. units (D) sq. units
(C) 8 (D) None of these 10 5
67. The area of the region consisting of points (x, y) satisfying 7. The area of the smaller region intercepted between the curve
|x ± y | ≤ 2 and x2 + y2 ≥ 2 is y = f(x) and x2 + y2 = 1 is
(A) 8 − 2π sq. units (B) 4 − 2π sq. units p 1 π
(A) - sq. units (B) sq. units
(C) 1 − 2π sq. units (D) 2π sq. units 4 5 4
Chapter 24 | Area Under the Curves 1093
20. Match the following: (C) The area bounded by the loop of 4y2 = x2 (4 − x2) is 1
(r)
Column I Column II 2
16
æ 1ö (s)
(A) If y = tan-1 ç ÷ + tan-1(b ), (0 < b < 1) and (p) 3 3
è2ø
π Integer Type Question
0 < y ≤ , then the maximum value of b will be
4
21. Let f(x) be a polynomial of degree 3 if the curve y = f(x) has
2
relative extremities at x = ± and passes through (0, 0)
(B) The number of solutions of sin4 x + cos3 x ≥ 1 in (q) 1 3
(0, 2π) will be 3 and (1, −2) dividing the circle x + y2 = 4 in two parts. Then the
2
Answer Key
Practice Exercise 1
1. (B) 2. (B) 3. (A) 4. (D) 5. (B)
6. (D) 7. (D) 8. (B) 9. (A) 10. (C)
11. (B) 12. (C) 13. (A) 14. (B) 15. (D)
16. (B) 17. (D) 18. (A) 19. (A) 20. (B)
21. (D) 22. (B) 23. (D) 24. (A) 25. (C)
26. (A) 27. (B) 28. (D) 29. (C) 30. (C)
31. (B) 32. (B) 33. (C) 34. (A) 35. (B)
36. (B) 37. (D) 38. (C) 39. (A) 40. (D)
41. (A) 42. (A) 43. (B) 44. (A) 45. 3e sq. units
⎛ 2⎞
47. 761 sq. units 48. x 3 − x2 49.
17
sq. units 51. 121 52. ⎜⎝ p − ⎟⎠ sq. units
3
192 27 4
53. ( 4 - 2 ) + 3 - æ 5 ö ln 2 e2 - 5 10 æ 1ö æ 1ö p+
1
ç ÷
54. sq. units 55. sq. units 56. ç ÷ log 2 - ç ÷ 57.
ln 2 2 è2ø 4e 3 è2ø è4ø 3
58. (B) 59. (A) 60. (A) 61. (A) 62. (A)
63. (C) 64. (C) 65. (C) 66. (D) 67. (A)
Practice Exercise 2
1. (B), (D) 2. (B) 3. (C) 4. (D) 5. (B)
6. (B) 7. (A) 8. (D) 9. (D) 10. (D)
11. (A) 12. (B) 13. (C) 14. (A) 15. (C)
16. (C) 17. (A) 18. (A) → (q), (B) → (s), (C) → (p), (D) → (r) 19. (A) → (s), (B) → (r), (C) → (q), (D) → (p)
20. (A) → (q), (B) → (p), (C) → (s) 21. (6)
Solutions
Practice Exercise 1 é 1 ù é æ 1 öù 1 1
= ê1- ú + ê - ç - 1÷ ú = + = 1 sq. unit
1. y = x − 1, if x > 1 and y = −(x − 1), if x < 1 ë 2 û ë è 2 øû 2 2
1 2
1 2 é x2 ù é x2 ù
Area = ò (1- x )dx + ò ( x - 1)dx = ê x - ú + ê - x ú a 2 3/2 a
0 1
êë 2 úû 0 êë 2 úû1 2. Required area = 2 ò 4 axdx = 4 a ´ [ x ]0
0 3
Chapter 24 | Area Under the Curves 1095
8
=
8 a 8
× a a = a2 sq. units 8 é 4 2 3 / 2 x 2 ù 128 64 32
3 3 ∫0 (2 2 x − x )dx = êê 3 x - 2 úú = 3 - 2 = 3 sq. units
ë û0
3. Since the curve is symmetrical about x-axis, therefore,
9. For points of intersection,
a a- x
Required area A = 2 ò a dx
log x = (log x ) ⇒ log x (log x − 1) = 0
2
0 x
Put x = a sin2 θ. Then ⇒ x = 1, x = e
dx = 2a sinθ . cosθ dθ So,
e
p /2 2
a cos q p /2 cosq Required area A = ò [log x - (log x )2 ] dx
A = 2ò a 2
a sin 2q dq = 2a2 ò 2 sinq cosq dq 1
0 a sin q 0 sinq e e
A = ò log x dx - ò (log x )2 dx
p /2 1 p 1 1
A= 4 a2
0ò cos2 q dq Þ A = 4 a2 . . = p a2 sq. units
2 2 = [ x log x - x ] 1e - [ x (log x )2 - 2 x log x + 2 x ] 1e
4. Given parabolas are x2 = 1 + y, x2 = 1 − y = [e − e − (−1)] − [e(1)2 − 2e + 2e − (2)]
For points of intersection, we have = (1) − (e − 2) = 3 − e
1+ y = 1 − y ⇒ 2 y = 0 ⇒ y = 0 10. For points of intersection,
⇒ x 2 = 1⇒ x = ±1 ⎛ x2 ⎞
2
⎜⎝ 8a ⎟⎠ = 4 ax ⇒ x = 256a x
4 2
So,
1 é x3 ù 8
Required area = 4 ò (1- x 2 ) dx = 4 ê x - ú = sq. units
1
(
⇒ x x 3 − 256a3 = 0 )
0
êë 3 úû 0 3 ⇒ x = 0 , x = a.28/3
3 So,
5. Area of smaller part I = 2 ò 9 - x 2 dx
1 ( a 28 / 3 ) a 28 / 3 x2 32a2
3 Required area, A = ò 4 axdx - ò dx =
1é xù é p æ 1 öù 0 0 8a 3
= 2 × ê x 9 - x 2 + 9 sin-1 ú = ê9 - 8 - 9 sin-1 ç ÷ ú
2ë 3 û1 ë 2 è 3 øû 11. See Fig. 24.51.
é æp æ 1öö ù é æ 1ö ù Y
= ê9 ç - sin-1 ç ÷ ÷ - 8 ú = ê9 cos -1 ç ÷ - 8 ú
ë è2 è 3 øø û ë è3ø û
2 3
é x2 ù é x2 ù 1 1 X' X
= ê2 x - ú + ê - 2 x ú = + = 1 sq. unit
êë 2 úû1 êë 2 úû 2 2 2
Figure 24.51
7. Equations of curves y2 = 4x and
= 4y. The given equations
x2
Required area = 2 × (shaded area in first quadrant)
x2
may be written as y = 2 x and y = . 1 1 1
4 = 2 ò ( x - x 2 ) dx = 2 ´ = sq. units
0 6 3
For points of intersection,
2 12. Given equations of curves y = cos x and y = sin x and ordinates
⎛ x2 ⎞ x = 0 to x = p /4.
⎜⎝ ⎟⎠ = 4 x ⇒ x = 64 x
4
4
We know that area bounded by the curves is
⇒ x ( x 3 − 64 ) = 0
x2 p /4 p /4
⇒ x = 0, x = 4
∫x 1
ydx = ∫
0
cos xdx − ∫
0
sin x dx
We know that area enclosed by the parabolas is
= [sin x ]p0 /4 - [- cos x ]p0 /4
4 4 x2 32 16 16
∫0 2 x dx − ∫
0 4
dx =
3
− =
3 3
sq. units
æ p ö æ p ö æ 1 ö æ 1 ö
= ç sin - sin 0 ÷ + ç cos - cos 0 ÷ = ç -0÷ +ç - 1÷
è 4 ø è 4 ø è 2 ø è 2 ø
8. y2 = 8x and y = x ⇒ x2 = 8x ⇒ x = 0, 8
Therefore, required area is = 2 - 1 sq. units
1096 Mathematics Problem Book for JEE
π (π2) π3 2
13. Area of the circle in first quadrant is , that is, . Also 2 ⎛ y3 2⎞ 4
∫0 ( y − 2 y )dy = ⎜⎝ 3 − y ⎟⎠ = 3 sq. units
2
4 4
area bounded by curve y = sin x and x-axis is 2 sq. units 0
p /4
22. The curve is symmetric about x-axis,
16. Required area = ò (sin 2 x + cos 2 x )dx Therefore, required area is
0
p /2
é cos 2 x sin 2 x ù
p /4 2∫ sin x dx = 2[ − cos x ]p0 / 2 = 2 sq. units
= ê- + 0
ë 2 2 úû 0 9
23. Required area, A = 2 ò 4 axdx
1⎡ p p ⎤ 4
= ⎢ − cos + sin + cos 0 − sin 0 ⎥ = 1 sq. unit
2⎣ 2 2 ⎦ 2 152 a
A = 4 a ´ [ x 3 / 2 ]94 = sq. units
3 3
4
4 (3 x + 4 )3 / 2 24. The curves y = x and y = x + sinx intersect at (0, 0) and (π, π).
17. Area = ò 3 x + 4 dx =
0 3.(3 / 2) 0 Hence, area bounded by the two curves is
p p p
2 112
= ´ 56 =
9 9
sq. units ∫ ( x + sin x )dx − ∫ x dx = ∫ sin x dx
0 0 0
Þ
8 a2 a2
= Þ m3 = 8 Þ m = 2
ò 0 (a x + bx ) dx = 8
3 m3 3
2a 3 / 2 4 b 2 4 2a
19. y2 = x and 2y = x ⇒ y2 = 2y ⇒ y = 0, 2 Þ [ x ]0 + [ x ]0 = 8 , × 8 + 8b = 8
3 2 3
Therefore, required area is ⇒ 2a + 3b = 3 (2)
Chapter 24 | Area Under the Curves 1097
From equations (1) and (2), we get a = 3, b = −1. x 0 π/6 π/2 π 3 π/2 2π
2 kx 3 y −1
27. ∫ 0
2 dx =
log 2
⇒ 22k − 1 = 3k . Now check from options, only 0 0.5 1 0 0
A(
3 ,1
(As area BCD is below x-axis)
p 2p
= ò sin x dx - ò sin x dx = 4 sq. units
O B C 0 p
4 4 y
33. Required area = ò1 x dy = ò1 2
dy
1 2 7
= . | y 3 / 2 |14 = sq. units
2 3 3
( ) ∫
2
1 Y
1= 3 ×1 + 4 − x 2 dx
2 3
3 3 2p x=2
= +p − − (−1, 0)
X
2 2 3 x=1 (2, 0)
p
= sq. units
3
(2, 3) A ⎛ 3⎞
dt C B ⎜ 4, ⎟
We put x 2 = t Þ dx = as x = 0 ⇒ t = 0 and x = a ⇒ t = a2, ⎝ 2⎠
2x
then it reduces to
X
O M D N
a2
1 a t2
1 e -1 (2, 0) (a, 0) (4, 0)
ò
2
e dt = [e t ]a0 = sq. units
2 0 2 2 Figure 24.56
4
32. See Fig. 24.54. 4æ 8 ö é 8ù
Area of AMNB = ò ç 1+ 2 ÷ dx = ê x - ú = 4
2 è x ø ë x û2
We have y = sin x
1098 Mathematics Problem Book for JEE
aæ 8 ö A1 4 2 − 1
Area of ACDM = ò2 ç 1+ 2 ÷ dx = 2 Therefore, =
è x ø A2 9 − 4 2
é pù y2 = x
36. y = cos x, When x Î ê0 , ú , cos x ³ 0
ë 2û (1,1)
é p 3p ù
When x Î ê , ú , cos x £ 0
ë2 2 û . X′ X
(0,0)
é 3p ù x = 1/2
When x Î ê , 2p ú , cos x ³ 0
ë 2 û
Figure 24.58
Thus required area is given by,
42. See Fig. 24.59.
2p p /2 3p / 2 2p
∫0 ydx = ∫
0
cos x dx + ∫
p /2
( − cos x )dx + ∫
3p / 2
cos xdx x + |y| = 1 ⇒ |y| = 1 − x
æ1 ö
= 1 + 2 + 1 = 4 sq. units Area = 2 ´ ç ´ 1´ 1÷ = 1 sq. unit
è 2 ø
4 Y
4 é x4 ù 255
37. Required area = ò x 3dx = ê ú = sq. units
1
êë 4 úû1 4
(0, 1)
4 4 1
38. Required area = ò y dx = c ò dx = 2c log 2 sq. units
1 1 x
X
2p (1, 0)
39. Required area = k ò sin x dx = k [ - cos x ]p2p = -2k
p
Figure 24.57 Y
1é 2 1 ù 1( 4 2 - 1) æ 4 2 - 1 ö
= ê - ú= = çç ÷÷ Y'
3 ë2 2 8 û 3´ 8 è 24 ø
Figure 24.60
1 é 2 3/2 x 3 ù1
44. See Fig. 24.61.
A2 = ò x - x2 = ê x - ú
êë 3 3 úû1/ 2 (x2 − y2)(x2 − y2 − 8x + 16) = 0
1/ 2
(x2 − y2)[(x − 4)2 − y2] = 0
é 2 1 ù é 4 2 - 1ù 1 4 2 − 1
=ê - ú-ê ú = −
( ) y=±x
ë 3 3 û êë 24 úû 3 y = ± (x − 4)
24
y=x−4
8 − 4 2 +1 9 − 4 2 y = −x + 4
= =
24 24 1
Area = ´2´ 4
2
Chapter 24 | Area Under the Curves 1099
An > 1 (2)
2(n + 1)
Figure 24.61
æxö 1 1
45. Given f ç ÷ = f(x) − f(y) (1) From Eqs. (1) and (2), we get < An < .
èyø 2(n + 1) 2(n - 1)
Putting x = y = 1, we get f(1) = 0
47. Since f(x) is continuous (Fig. 24.63).
f ( x + h) − f ( x )
Now, f’(x) = lim So, it must be continuous at x = 1, − 1, that is,
h→ 0 h
⎛ h⎞ lim f ( x ) = lim f ( x ) = f (1)
f ⎜ 1+ ⎟ x →1− x →1+
⎝ x⎠
= lim (from (1))
h→ 0 h Þ lim 2 x = lim x 2 + ax + b = 2
x ®1- x ®1+
⎛ h⎞
f ⎜ 1+ ⎟
⎝ x⎠ ⇒2=1+a+b=2
= lim
h→ 0 ⎛ h ⎞
⎜⎝ ⎟⎠ x ⇒a+b=1 (1)
x
f (1+ x ) Also lim f ( x ) = lim f ( x ) = f ( -1)
3 ⎧ ⎫ x ®1- x ®1+
⇒ f ′(x) = ⎨since lim = 3⎬
x ⎩ x → 0 x ⎭
⇒ f(x) = 3In x + c Þ lim x 2 + ax + b = lim 2 x = -2
x ®1- x ®1+
Putting x = 1
⇒ 1 − a + b = −2
⇒c=0
⇒ f(x) = 3 In x = y (say) ⇒ −a + b = −3 (2)
y
y = tann+2 x
y = tann x x
y = tann−2 x −1
x = −2y 2
(−2, −1)
O π /4 x (−1, −2)
y = x 2 + 2x − 1
-1 -1/ 8 y
é -x ù é -x ù
Required area = ò - + - + ò - 2 x ú dx
2
ê ( x 2 x 1) ú dx ê
-2 ê
ë 2 úû -1 ê ë 2 úû C2 C1
D ⎛ 1 1⎞
(0, 1) B(1, 1)
761 ⎜ , ⎟
= sq. units ⎝ 2 2⎠
192 H y = 2x − 2x 2
⎛ 1 4⎞ F E ⎛ 2 4⎞
48. See Fig. 24.64. ⎜ , ⎟ G⎜ , ⎟
⎝ 3 9⎠ ⎝ 3 9⎠
On the curve C1, that is, y = x2
O x
Let P be (α, α2). Hence, ordinate of point Q on C2 is also α2. ⎛ 1 1⎞ A(1, 0)
y a2 ⎜ , ⎟
⎝ 2 4⎠
Now on C2 (y = 2x) the abscissa of Q is given by x = = .
2 2
æa2 2 ö Figure 24.65
Therefore, Q is çç , a ÷÷ and R on C3 is {α, f(x)}.
è 2 ø
æ 1 1ö
Now area of OPQ is y = x2 and y = (1 − x)2 meet at E ç , ÷ and y = 2x − 2x2 meets
è2 4ø
a2 a2
⎛ y⎞ 2 a2 æ1 4ö æ2 4ö
∫ ( x1 − x2 ) dy = ∫ ⎜⎝ y − ⎟ dy = a 3 −
2⎠ 3 4
(1) y = (1 − x)2 at F ç , ÷ and y = x2 at G
è3 9ø
ç , ÷
è3 9ø
0 0
ì 1
ï(1- x ) 0£ x £
2
y
3
ï
⎛1 ⎞ ï 1 2
⎜ , 1⎟ Therefore, f ( x ) = í2 x (1- x ) £x£
⎝2 ⎠ ï 3 3
(0, 1) (1, 1) ï 2 2
C2 C1 ïx 1³ x >
î 3
Q
P Hence,
1 2
3 3 1
Area = ò (1- x ) dx + ò (2 x - 2 x 2 ) dx + ò x 2dx
2
x
O (1, 0) 0 1 2
3 3
R 1 2
1
C3 é -(1- x )3 ù 3 é 2 2 x 3 ù 3 é x 3 ù
=ê ú + êx - ú +ê ú
Figure 24.64 êë 3 úû 0 êë 3 úû 1 êë 3 úû 2
3 3
a a -8 1 4 16 1 2 1 8
= + + - - + + -
Again, area of DORP = ò ( y1 - y 2 ) dx = ò ( x 2 - f ( x )) dx (2) 81 3 9 81 9 81 3 81
0 0
30 51 17
= 1- = = sq. units
81 81 27
From Eqs. (1) and (2)
a 50. (1 + tanx)n > (tanx)n so, we can replace
2a 3 a 4
4 ò0
- = ( x 2 - f ( x )) dx p p p
3 4 4
n -2 é tann -1 x ù 4
An = ò tan x dx = ò tan
n
x (sec x - 1) dx = ê
2
ú - An -2
Differentiating both sides 0 0 êë n - 1 úû 0
2α2 − = − f(α)
α3 α2 Therefore,
f(α) =α3 − α2 ⇒ f(x) = x3 − x2 An + An -2 =
1
(1)
n -1
49. See Fig. 24.65. y = x2
is parabola (C1) with vertex (0, 0) and
passing through B (1, 1), y = (1 − x)2 is parabola (C2) with vertex π
Again in the interval 0 to , tan x is positive and < 1
at A (1, 0) and passing through D (0, 1). 4
2 Therefore,
æ 1ö 1
y = 2x(1 − x) = 2x − 2x2 = -2 ç x - ÷ + is parabola with tann x ≤ tann−2 x
è 2ø 2
⇒ An ≤ An−2
⎛ 1 1⎞ æ 1 ö
vertex at H ⎜ , ⎟ . ⇒ An + An < An -2 + An ç =
⎝ 2 2⎠ ÷ (1)
è n - 1ø
Chapter 24 | Area Under the Curves 1101
y y = 2x
D
D(1, 0) M(4, 0)
O x
A y = log x
B(1/2, 1/4)
Figure 24.66 C
æ 5 15 ö O
x
The two parabolas intersect at O(0, 0) and A ç , ÷. (1, 0)
è2 4 ø x=2
B
1
1 2 1 x=
x3 x 1
Area OBD = ò ( x - x ) dx = - =
2 2
0
3 2 6
0 Figure 24.68
5 5
2 2
2
Area ODAC = ∫ ( y1 − y 2 ) dx = ∫ [( 4 x − x 2 ) − ( x 2 − x )] dx Area ABCD = ò ( y1 - y 2 ) dx , where y1 = 2x > In x = y2 for all
0 up down 1
0
2
5 5 é1 ù
2
⎡ 5 x 2 2 x 3 ⎤ 2 125 250 125 x Îê , 2ú .
= ∫ (5 x − 2 x ) dx = ⎢
2
− ⎥ = − = ë2 û
0 ⎣ 2 3 ⎦0 8 24 24 2
é 2x ù
2
ú - [ x ln x - x ] 1
2
Þ ò (2 x - ln x ) dx = ê
125 1 121 êë ln 2 úû 1
Lined area = - = 1
2
2
24 6 24 2
1102 Mathematics Problem Book for JEE
4 2 1 1 1 1 æ 1ö 4
= - - (2 ln 2 - 2) + ln - At x = , f ¢¢ ç ÷ = -2 < 0 max. value =
ln 2 ln 2 2 2 2 3 è3ø 27
(4 − 2 ) 3 ⎛ 5 ⎞ y
= + − ⎜ ⎟ ln 2 sq. units
ln 2 2 ⎝ 2⎠ D C
y=2
54. See Fig. 24.69. Given curves are
y = ex In x (1)
ln x
y= (2)
ex
-1 e 2 - 3 1 e 2 - 5
=
1
4e
(
3 - e2 -
2e
=
4e
) - =
2e 4e
sq. units
dy
dx
p = sec2 x p =2
4 4
1 1
⎛ ln x ⎞ ⎡1 1 2 ⎤
∫ ⎜⎝ ex − ex ln x ⎟⎠ dx = ⎢⎣ 2e (ln x ) − 4 ex (2 ln x − 1)⎥⎦ 1
2
1
e
e
1 ⎛π 1⎞⎟
P ⎜ 4,
⎝ ⎠
1 1 e -5 2
= - (3 - e 2 ) = sq. units
2e 4 e 4e
M
55. See Fig. 24.70. π π x
O Q
f(x) = x(x − 1)2 4 2
{ }
Therefore, 3 3 2
p 1 ⇒± = x 2 + ( g( x ))2
QM = - OQ = 4 4
4 2
⇒ x + (g(x))2 = 1
2 (since x2 + (g(x))2 ≠ −1)
Hence,
⇒ (g (x))2 = 1 − x2 ⇒ g (x) = ± 1- x 2
⎛ 1⎞ 1 1 ⎡⎛ 1⎞ ⎛ 1⎞ ⎤
Area = ⎜ ⎟ log 2 − × × 1 = ⎢⎜ ⎟ log 2 − ⎜ ⎟ ⎥ sq. units
⎝ 2⎠ 2 2 ⎝
⎣ 2 ⎠ ⎝ 4⎠ ⎦ 60. See Fig. 24.74. Clearly
⎧
57. See Fig. 24.72. ⎪2, − 2≤ x < 0
Points of intersection of y = x and x 2 = - 2 y are ⎪
⎪ 1
g( x ) = ⎨ x 2 − x , 0≤ x ≤
x = 0, - 2 ⎪ 2
⎪ 1 1
2
æ x2 ö 0
æ -x2 ö ⎪⎩ − 4 , 2
< x ≤3
Area = 2 ò çç - + 4 - x 2 ÷÷ dx - ò çç - x ÷÷ dx
0 è 2 ø - 2è
2 ø 0 1/ 2 3
1
Area= ∫ 2dx + ∫ (x − x
2
) dx + ∫ dx
2 0 4
⎡ − x3 1 4 x⎤ ⎡ x2 x3 ⎤ −2 0 1/ 2
= 2⎢ + x 4 − x 2 + sin−1 ⎥ + ⎢ + ⎥ 1/ 2
⎣3 2 2 2 2 ⎦0 ⎣ 2 3 2 ⎦− 2 æ x2 x3 ö æxö
3
113
= (2 x )0-2 + çç - ÷÷ + ç ÷ = sq. units
é -2 p ù é 2ù é1 p ù 1 1 è 2 3 ø0 è 4 ø1 / 2 24
= 2 ê + 1+ 2 ´ ú - ê1- ú = 2 ê + ú - = p + sq. units
ë3 4 û ë 3û ë3 2 û 3 3 y
y
y = x 2 – |x|
(0, 2)
y=x
1
x
−2 −1 O 3
O x
(−2, 0) (0, 0) (2, 0) Figure 24.74
x2 + y2 = 4
61. See Fig. 24.75. The expression |x − y| + |x + y| ≤ 8, represents the
( − 2, − 2 ) interior region of the square formed by the lines x = ±4, y = ±4
and xy ≥ 2. Represents the region lying inside the hyperbola
(0, −2) x 2 = − 2y xy =2.
Required area is
Figure 24.72
4
58. See Fig. 24.73. ⎛ 2⎞
= 2 ∫ ⎜ 4 − ⎟ dx = 2( 4 x − 2 I n x )14/ 2
1 ⎝ x⎠
Required area = × 2 × 2 = 2 sq. units 1/ 2
2
= 4(7 − 3 ln 2) = 4(7 − ln 8 ) sq. uniits
y y=x
(0, 1) C y=4 B
y = |x| − 1
(−1, 0) (1, 0)
x
O
y = −|x| + 1 x=4
x = −4
(0, −1) D
y = −4 A
y = −x
Figure 24.73
3 Figure 24.75
59. Area of equilateral triangle = (side)2.
4
px
Two vertices are at (0, 0) and [x, g(x)] 62. See Fig. 24.76. Area bounded by y = 2 ×sin and x-axis
4
between the lines x = 2 and x = 4,
Hence, side = ( x - 0 )2 + ( g( x ) - 0 )2 = x 2 + ( g( x ))2
4 4
2 px 4 2 px
Area =
3é 2
x + ( g( x ))2 ù D = 2 ò sin dx = - × cos
4 ê
ë ú
û 2
4 p 4 2
1104 Mathematics Problem Book for JEE
1
4 2 ⎡ x 1 ⎤
= sq. units = ⎢x − 1− x 2 − sin−1 x ⎥
p ⎣ 2 2 ⎦0
Let the drawn lines are L1: y − m1(x − 4) = 0 and L2: y − m2(x − 4) 1⎛p ⎞ p
= 0, meeting the line x = 2 at the points A and B, respectively. =1 − 0 − ⎜ ⎟ = 1− sq.units.
2⎝ 2⎠ 4
Clearly A = (2, −2m1); B = (2, −2m2) (Fig. 24.76).
65. See Fig. 24.79.
Now 3 1 2 3
D 4 2 1 Required area = ò | y | dx = ò | y | dx + ò | y | dx + ò | y | dx
D ACD = Þ = × 2 × -2m1 0 0 1 2
3 3p 2 1 2
æx 4
11 ö æx 11 4 ö
2 2 2D = − çç - 2 x 3 + x 2 - 6 x ÷÷ + çç - 2 x 3 + x 2 - 6 x ÷÷
Þ m1 = - Also D BCD = 4 2
3p 3 è 4 2 ø0 è ø1
3
8 2 1 æ x4 11 ö
Þ = × 2 - 2m2 − çç - 2 x 3 + x 2 - 6 x ÷÷
3p 2 è 4 2 ø2
-4 2
Þ m2 = =
9 1 1 11
3p + + = sq. units
4 4 4 4
2 2
Required sum = -
p
y B D F
πx O
y = 2 sin B
4 A
A
D
x Figure 24.79
O C 1 2 4 L1 = 0
66. See Fig. 24.80.
L2 = 0 1
Figure 24.76 A = ò ( x - x 3 ) dx
0
63. y = x (3 − x)2 1
2 3/2 1 x 4
After solving, we get x = 1 and x = 3 which are points of = x -
3 0 4
maximum and minimum, respectively. 0
Now the shaded region is the required region (Fig. 24.77). 2 1 8-3 5
3
= - = =
3 4 12 12
Therefore, A = ò x (3 - x )2 dx = 4 sq. units
1
Y
C(1,4)
1
X
O O
A(1, 0) B(3, 0)
Figure 24.77
64. By definition, the lines max, (x, y) = 1 means Figure 24.80
x = 1 and y ≤ 1 or y = 1 and x ≤ 1 67. Y
Y
Reqd.
Area
X′ X X′ X
O
x=1
Y′
Y′
Figure 24.78 Figure 24.81
1
Shaded region is the required one. Therefore, required area is
See Fig. 24.78. Required area is ò éêë1- 1- x 2 ù dx.
úû
1
4 ´ ´ 2 ´ 2 - p × 2 = 8 - 2p sq. units
2
0
Chapter 24 | Area Under the Curves 1105
Practice Exercise 2 BC S3 BC S2
PM = , ML = (QM = BN and ML = PC)
1. Solving y = x − and y = mx, we get the points of intersection
x2 S S
(0, 0) and [1−m, m(1−m)]. æ S S S ö
BC = BC ç 1 + 2 + 3 ÷
Case I: m < 1 (Fig. 24.82) ç S S S ÷ø
è
y S = ( S1 + S2 + S3 )2 , where S is the area of ΔABC.
a a +t
O
x 3. ò f (t ) dt = - ò f (t ) dt
a -t a
⇒ f(α − t) = − f(α + t) ∀ t ∈ R
Therefore, y = f(x) is symmetrical about point (α, 0).
4. Putting x = α in the given equation, we have
Figure 24.82 f(0) = − f(2α) ⇒ −f(0) = f(2a)
9
Shaded area = a a +t
2 5. ò f (t ) dt = - ò f (t ) dt ⇒ f(α − t) = − f(α + t) ∀ t ∈ R.
1- m 1- m a -t a
ò ( x - x ) dx - ò
9
Þ 2
mx dx = ⇒ f(α − t) = − f(α + t) = x ⇒ t = α − f −1(x) = f −1(−x) − a
2
0 0 b
-1
Solving this, we get m = −2 ⇒ f −1(x) + f −1(−x) = 2α ⇒ òf ( x ) dx = 2αβ
Case II: m > 1 (Fig. 24.83) -b
6. x = 1 is a root of f(x) and also a root of 1st, 2nd and 3rd derivatives
of f(x). Hence, f(x) has x = 1 repeated root 4 times so f(x) = (x − 1)4.
Therefore,
2
3
Required area = ò ( x - 1- ( x - 1) ) dx =
4
sq. units
1
10
Figure 24.83 1
p 1
Shaded area =
9 7. Required area = ò( 1- x 2 - ( x - 1)4 ) dx = - sq. units
4 5
2 0
1 3
0 0 63
8. Required area = 6 + ò ( x - 1) dx + ò ( x - 1) dx =
4 4
9 sq. units
⇒ ∫ ( − mx ) dx − ∫ ( x − x ) dx =
2
5
1− m 1− m
2 0 1
9. Equation of circle is + = 2. x2 y2
Solving this, we get m = 4
Equation of tangent at A is −x + y = 2
2. See Fig. 24.84. Equation of tangent at D is −x − y = 2
Area of ABC BC 2 Form maximum area of the rectangle QEKF, Q is ( 2 , 0 )
=
Area of PQM QM 2 - 2 -2 - 2 -2 6+ 4 2
Area of rectangle = ´ = =3+2 2
Area of ABC BC 2 2 2 2
=
Area of MNP NP 2 10. Area of ΔBQC = 2 −1
2 1
Area of ABC BC 11. Area of ΔOQM = 2 ´ 2 =1
=
Area of MOL ML2 2
f (x) + f (y)
A 12. f ( x + y ) =
O f (x)
P f ( x + h) - f ( x ) (f ( x ) + f (h)) - (f ( x ) + f (0 ))
M f ¢( x ) = lim = lim
Q S1 S2 L h®0 h h®0 hf ( x )
f ¢(0 ) 2
= =
f (x) f (x)
C
B N P
⇒ f(x) f(x) dx = ∫2 dx ⇒ f ( x ) = 4( x + c )
Figure 24.84 Since, f(0) = 2 ⇒ c = 1.
BC S1
Area of DABC = QM = Therefore, f ( x ) = 2 ( x + 1).
S
1106 Mathematics Problem Book for JEE
3 x x
x2 4 ln t
= 2 7x - - ( x + 1)3 / 2 16. g(x) = ò f ( t ) dt = ò dt
2 3 0 0
t
0
1/ x
é 9 4 ù 43 43 æ 1ö ln t
= 2 ê21- - (8 - 1)ú = 2 ´ =
ë 2 3 û 6 3
sq. units Þ gç ÷ =
èxø ò t
dt
0
1
Let t = . Then
(0, 7) u
x
ln u
∫ u
du
0
(0, 3)
(4, 0) æ 1ö
⇒ g ç ÷ = g(x)
èxø
Figure 24.85 æ 1ö
Therefore, g(n) − g ç ÷ = 0.
14. See Fig. 24.86. ènø
ln x
= ex ln x ⇒
ln x 1 − ex 2
=0
( )
−1 1 8 x x
ln x = 0 ⇒ x = 1
1
1− ex 2 = 0 ⇒ x =
e
Figure 24.86
Hence, the number of points is 5. Therefore,
1
15. f(xy) =
f (x) f (y)
+ ⎛ ln x ⎞ 2e − 5
y x
Required area = ∫ ⎜⎝
x
− ex ln x ⎟ dx =
⎠ 8
sq. units
1 e
f ( x + h) - f ( x )
f(x) = lim
h®0 h 18. See Fig. 24.87.
æ 1 ö é1 1 ù
f ( x )ç -1 (A) Required area = 2 ê (2 + 1) ´ 1- ò x 2dx ú
h ÷ æ hö
ç 1+ ÷ f ç 1+ ÷ - f (1) êë 2 0 úû
= lim è x ø + è xø
h®0 h xh é 3 1ù 7
= 2 ê - ú = sq. units
-f ( x ) f ¢(1) ë2 3û 3
= lim + 2
h®0 æ hö
x ç 1+ ÷ x
è xø x2 = y
-f ( x ) 1 f (x) 1 (0, 2)
⇒ f(x) = + 2 or f ′(x) + = 2
x x x x
This is a linear differential equation. (1, 1)
Solution is f(x) · x = ln x + c
or (2, 0)
ln x
f(x) = , c = 0 since, (f(1) = 0)
x
Now,
ln x æ¥ö
lim ç ÷ form
x ®¥ x è¥ø Figure 24.87
Chapter 24 | Area Under the Curves 1107
2 ´ 3 ´ 4 24 1
(C) Length of the chord =
2r1r2
= = 20. (A) y = tan-1 + tan-1 b , (0 < b < 1)
5 5 2
r12 + r22
æ 1/ 2 + b ö æ1 ö
(D) See Fig. 24.88. ⇒ y = tan-1 ç ÷ , since, ç b < 1÷
è 1- b / 2 ø è2 ø
Parabola is y2 = 4(x + 1) (1)
Focus is (0, 0) æ 1+ 2b ö p æ 1+ 2b ö
0 < tan-1 ç ÷£ Þ0<ç ÷ £1
x -0 y -0 è 2-b ø 4 è 2-b ø
Equation of AB is = =r
1/ 2 3 /2 ⇒ 0 < (1 + 2b) ≤ (2 − b), (1 + 2b > 0)
Substituting parametric coordinates in Eq. (1), we have
1 1
⇒ 3b ≤ 1 ⇒ 0 ≤ ⇒ bmax =
2 3 3
æ 2 ö ær ö
çç r ÷÷ = 4 ç + 1÷
è 2 ø è2 ø (B) sin4 x + cos3 x ≥ 1 (1)
Since sin2 x + cos2 x = 1 and −1 ≤ sin x, cos x ≤ 1
3r 2
Þ - 2r - 4 = 0 Eq. (1) cannot be >1
2
Therefore, sin4 x + cos3 x = ±1 (2)
A Eq. (2) is possible if either,
sin x = 1 and cos x = 0 or sin x = 0 and cos x = 1
p p
⇒ x = (4n + 1) , x = (2n + 1) or x = nπ, x = 2nπ
60° 2 2
(1, 0) p 3p
In (0, 2π), x = , or x = π
2 2
B Therefore, the number of solutions will be 3.
(C) See Fig. 24.89.
Figure 24.88
4y2 = x2 (4 − x2) (1)
1 2
Length AB = PA - PB = ( PA + PB )2 - 4 PAPB ⇒y= x (4 - x 2 )
2
2
æ8ö 16 16 x
= ç ÷ -4´ = ⇒y= (4 - x 2 )
è3ø 3 3 2
1108 Mathematics Problem Book for JEE
Therefore, ⎛ x3 4 x ⎞
2
2x f (x) = a⎜ − ⎟ + b
area (A) = 4 × ò 2´2 ( 4 - x 2 )dx ⎝ 3 3⎠
0
This passes through (0, 0) and (1, −2). So, b = 0 and
Let (4 − x2) = t. Then − 2x dx = dt.
æ1 4ö
a ç - ÷ = -2 ⇒ a = 2
è3 3ø
O 2x 2
B A Therefore, f ( x ) = ( x − 4 ).
(−2, 0) (2, 0) 3
Hence, f(x) meets the x-axis at (0, 0), (−2, 0) and (2, 0).
Since f(−x) = −f(x), the curve y = f(x) is symmetrical about the
origin.
Figure 24.89
Also as a = 2, f ′(x) > 0.
4
-4
0 4
ét ù 3/2
2 2 2 2 2
⇒A= ò t dt = ò t dt = ê ú = ´ [ 64 - 0] For x < - , x> and f ′(x) < 0 for - <x<
4 4 êë 3 / 2 úû 0 3 3 3 3 3
0
16 Therefore,
⇒A= sq. units
3
0 2
2
21. Since y = f(x) has relative extremities at x = ± these points Area of region = 2p - ò f ( x ) dx + ò f ( x ) dx
3 -2 0
are critical points and hence they must be roots of f ′(x) = 0 0 0 2 2
(Clearly f is differentiable everywhere). Therefore, Þ ò f ( x ) dx = - ò f ( x ) dx = - ò f ( -t )( -1) dt = ò f (t ) dt
-2 -2 0 0
⎛ 2 ⎞⎛ 2 ⎞ ⎛ 2 4⎞
f ′( x ) = a ⎜ x − ⎟⎜x+ ⎟ = a ⎜⎝ x − 3 ⎟⎠ Required area = 2π. So, integral part = 6.
⎝ 3⎠ ⎝ 3⎠
Chapter 24 | Area Under the Curves 1109
∫ ⎜⎝1+ x−
4⎠
⎟ dx + ∫ ⎜ 3 − x − ⎟ dx figure. Therefore,
1⎝
4⎠
0 ⎡1 2 ⎤
1 2 A = 2 ⎢ ∫ y 2dx + ∫ y1dx ⎥
⎡ x 3/ 2 x 3 ⎤ ⎡ x2 x3 ⎤ ⎢⎣ 0 1 ⎥⎦
= ⎢x + − ⎥ + ⎢3 x − − ⎥
⎢⎣ 3/ 2 12 ⎥⎦ ⎢⎣
0
2 12 ⎥⎦
1 From Eq. (1), we get
19 11 5 y1 = 4 − x 2
= + =
12 12 2 From Eq. (2), we get
Hence, the correct answer is option (C). y2 = 3 x
Therefore,
2. The area (in sq. units) of the smaller portion enclosed between ⎡1 2 ⎤
the curves, x2 + y2 = 4 and y2 = 3x, is A = 2 ⎢ ∫ 3 xdx + ∫ 4 − x 2 dx ⎥
⎢⎣ 0 1 ⎥⎦
1 p 1 2p
(A) + (B) + ⎡1 2 ⎤
2 3 3 2 3 3 = 2 ⎢ ∫ 3 x1/ 2dx + ∫ 22 − x 2 dx ⎥
1 2p 1 4p ⎢⎣ 0 1 ⎥⎦
(C) + (D) +
3 3 3 3 x n −1
(ONLINE) Using standard integral, ∫ x ndx = , we have
n +1
Solution: The given equation x2 + y2 = 4 is equation of circle of
x 2 a2 x
radius 2 centred at origin and equation y2 = 3x is the equation of ∫ a2 − x 2 dx = a − x 2 + tan−1
parabola. 2 2 a − x2
2
1110 Mathematics Problem Book for JEE
Therefore, a 1
∫ ( x − x )dx = ∫ ( x − x )dx
3 3
⎡⎛ 1
⎛x
2⎤
x 3/ 2 ⎞ 4 x ⎞ ⎥ 0 a
A = 2 ⎢⎜ 3 ⎟ + ⎜ 4 − x 2
+ tan −1
⎟ ⎥
⎢⎝ 3/ 2 ⎠ ⎝2 4 a 1
⎢⎣
2 4 − x2 ⎠ 1 ⎥ x2 x x2 x 4
0 ⎦ ⇒ − = −
2 4 2 4
⎡ 1 2 2 1 1 ⎤ 0 a
= 2 ⎢ 3. −0+ 4 − 4 + 2 tan−1 − 4 − 1 − 2 tan−1
⎣ 3/ 2 2 4−4 2 4 − 1 ⎥⎦ ⎛a2 ⎞ ⎛a 4 ⎞ ⎛ 1 a2⎞ ⎛ 1 a4⎞
⇒⎜ − 0⎟ − ⎜ − 0⎟ = ⎜ − ⎟ −⎜ − ⎟
⎡2 3 3 ⎛ 1 ⎞⎤ ⎝ 2 ⎠ ⎝ 4 ⎠ ⎝2 2 ⎠ ⎝4 4 ⎠
= 2⎢ + 2 tan−1(∞ ) − − 2 tan−1 ⎜ ⎥
⎣ 3 2 ⎝ 3 ⎟⎠ ⎦
a2 a4 1 a2 1 a4
⇒ − = − − +
2 4 2 2 4 4
p p 1
Now, tan = ∞ and tan = . Therefore, the area of the a2 a4 1 a2 1 a4
2 6 3 ⇒ − − + + − =0
smaller portion enclosed between the two curves is obtained as 2 4 2 2 4 4
follows: a4 1
⇒a 2 − − =0
2 4
⎡2 3 3 ⎛ p⎞ ⎛ p ⎞⎤
A = 2⎢ − + 2 tan−1 ⎜ tan ⎟ − 2 tan−1 ⎜ tan ⎟ ⎥ 4a 2 − 2a 4 − 1
⎣ 3 2 ⎝ 2 ⎠ ⎝ 6⎠⎦ ⇒
2 4
= 0 ⇒ 4a − 2a − 1 = 0
4
⎡ 1 p p⎤
= 2⎢ 3 + 2 −2 ⎥ ⇒ −(2a 4 − 4a 2 + 1) = 0 ⇒ 2a 4 − 4a 2 + 1 = 0
⎣ 6 2 6⎦
Now, let us consider the equation 2a 4 – 4a 2 + 1 = 0.
⎡ 1 2p ⎤ ⎡ 3 4p ⎤ 3 4p
= 2⎢ 3 +2 ⎥ = 2⎢ + ⎥ = + Let a 2 = u. Therefore,
⎣ 6 6 ⎦ ⎣ 6 6 ⎦ 3 3
2u2 – 4u + 1 = 0
⎛ 1 4p ⎞
=⎜ + sq.units
⎝ 3 3 ⎟⎠ ⇒u=
4 ± 16 − 4 × 2
2×2
Hence, the correct answer is option (D). ⎛ ⎞
⎜ since for equation ax 2 + bx + c = 0 , ⎟
JEE Advanced 2017 ⎜ 2 ⎟
⎜ the discriminant = b − 4 ac = D and⎟
1. If the line x = a divides the area of region R = {( x , y ) ∈
2 : ⎜ ⎟
⎜ the roots of x are −b ± D ⎟
x 3 ≤ y ≤ x , 0 ≤ x ≤ 1} into two equal parts, then ⎝ 2a ⎠
1 1
(A) 0 < a ≤ (B) <a <1 4 ± 16 − 8 4 ± 8 4 ± 4 × 2
2 2 ⇒u= = =
4 4 4
(C) 2a 4 − 4a 2 + 1 = 0 (D) a 4 + 4a 2 – 1 = 0
4±2 2 2 1
⇒u= = 1± ⇒ u = 1±
Solution: Let us consider y = x3 and y = x. Then the area between 4 2 2
these two curves in region 0 ≤ x ≤ 1 is
Substituting u = a 2, we get
1
A = ∫ ( x − x 3 )dx 1
a 2 = 1±
0 2
It is given that the line x = a divides the area under the curve into 1 1
From a 2 = 1± , we get < a < 1.
two equal parts. Therefore, 2 2
Hence, the correct answers are options (B) and (C).
y=x
x =a
25 Differential Equations
dy d 2 y
Step 1: Write the given equation involving an independent ⇒ 3y + 4y1 − y2 = 0 ⇒ 3y + 4 - =0
variable x (say), a dependent variable y (say) and the dx dx 2
arbitrary constants.
Illustration 25.4 Find the order and degree of the differential
Step 2: Obtain the number of arbitrary constants in step 1. Let
equation of all the parabolas whose axes are parallel to the x-axis
there be n arbitrary constants.
and having a latus rectum a.
Step 3: Differentiate the relation in step 1, n times with respect to x.
Step 4: Eliminate arbitrary constants with the help of n equations Solution: Equation of required parabolas is
involving differential coefficients obtained in step 3 and (y − β)2 = a (x − α) -
an equation in step 2. The equation thus obtained is the On differentiating both sides w.r.t. x, we get
desired differential equation. dy
2( y - b )
=a
Illustration 25.2 Form the differential equation of the dx
following relation: Again differentiating w.r.t. x, we get
(i) x2 + y2 = 2ax 2
d2 y æ dy ö
(ii) x2 + y2 = 2ax + b 2( y - b ) + 2ç ÷ = 0
dx 2 è dx ø
Solution: 2 3
(i) x2 + y2 = 2ax æ dy ö d y æ dy ö
Þ ç 2( y - b ) ÷ 2 + 2 ç ÷ = 0
On differentiating w.r.t. x, è dx ø dx è dx ø
3
2x + 2y
dy
= 2a d2 y æ dy ö
Þa + 2ç ÷ = 0
dx dx 2 è dx ø
Eliminating a,
Thus, the order of the differential equation is 2 and degree is 1.
æ dy ö
x2 + y2 = x ç 2x + 2y ÷
è dx ø Illustration 25.5 Find the differential equation of family of
dy curves whose tangent form an angle of π/4 with the hyperbola
Þ x 2 - y 2 + 2 xy =0
dx xy = c2?
In this case, the relation contains only one arbitrary constant Solution: The slope of the tangent to the family of curves is
and hence, the differential equation contains only dy/dx. dy
m1 =
(ii) x2 + y2 = 2ax + b dx
On differentiating w.r.t. x, The equation of the hyperbola is
dy c2
2x + 2y = 2a xy = c 2 Þ y =
dx x
On differentiating once again w.r.t. x, Therefore,
dy c2
2 2 =- 2
æ dy ö d y dx x
1+ ç ÷ + y 2 = 0
è dx ø dx c2
Therefore, the slope of the tangent to xy = c2 is m2 = - .
which is the differential equation to the given equation and x2
since there are two arbitrary constants a and b, the differential Now,
equation contains (the second order) the derivative d2y/dx2. dy c 2
+ 2
p m1 - m2 dy æ c 2 ö æ c 2 ö
Illustration 25.3 Find the differential equation of the family of tan = Þ 1 = dx 2 x Þ çç 1+ 2 ÷÷ = çç 1- 2 ÷÷
curves y = Aex + Be3x for different values of A and B. 4 1+ m1m2 c dy dx è x ø è x ø
1- 2
x dx
Chapter 25 | Differential Equations 1113
Hence, the required equation is 8. The differential equation whose solution is y = c1 cos ax + c2 sin
dy x - c
2 2 ax is (where c1, c2 are arbitrary constants)
=
dx x 2 + c 2 d2 y d2 y
(A) + y2 = 0 (B) + a2 y = 0
dx 2 dx 2
Your Turn 1 d2 y d2 y
(C) + ay 2 = 0 (D) - a2 y = 0
1. The degree of the differential equation dx 2 dx 2
2 3 Ans. (B)
dy 1 æ dy ö 1 æ dy ö
y ( x ) = 1+ + ç ÷ + ç ÷ + is
dx 1.2 è dx ø 1.2.3 è dx ø 9. The differential equation of the family of curves y2 = 4a(x + a),
(A) 2 (B) 3 (C) 1 (D) 4 where a is an arbitrary constant, is
Ans. (C)
é æ dy ö2 ù dy é æ dy ö2 ù dy
2 (A) y ê1+ ç ÷ ú = 2 x (B) y ê1- ç ÷ ú = 2 x
d y dy êë è dx ø úû dx êë è dx ø úû dx
2. The degree of the differential equation - - 3 = x is
dx 2 dx
3
(A) 2 (B) 3 (C) 1 (D) 4 d2 y dy æ dy ö dy
(C) +2 =0 (D) ç ÷ + 3 + y = 0
Ans. (A) dx 2 dx è dx ø dx
3. The degree and order of the differential equation of the family Ans. (B)
of all parabolas whose axis is x-axis are, respectively d2 y
10. If y = axn+1 + bx−n, then x 2 2 equals
(A) 2, 1 (B) 1, 2 (C) 3, 2 (D) 2, 3 dx
Ans. (B) (A) n(n − 1)y (B) n(n + 1)y
4. If m and n are the order and degree of the differential equation (C) ny (D) n2y
3
æ d2 y ö Ans. (B)
5 çç 2 ÷÷
æ d2 y ö 3
è dx ø + d y = x 2 - 1 , then
çç 2 ÷÷ + 4
è dx ø æ d y ö dx 3
3
25.6 Solution of a Differential Equation
çç 3 ÷÷
è dx ø If we have a differential equation of order ‘n’ then by solving a
(A) m = 3 and n = 5 (B) m = 3 and n = 1 differential equation we mean to get a family of curves with n
(C) m = 3 and n = 3 (D) m = 3 and n = 2 parameters whose differential equation is the given differential
Ans. (D) equation. The solution or integral of a differential equation is a
5. y = 4 sin3x is a solution of the differential equation _____. relation between the variables, not involving the differential coef-
ficients such that this relation and the derivatives obtained from
dy dy
(A) + 8y = 0 (B) - 8y = 0 it satisfy the given differential equation. The solution of a differen-
dx dx tial equation is also called its primitive.
d2 y d2 y For example, y = ex is a solution of the differential equation
(C) + 9y = 0 (D) - 9y = 0
dx 2 dx 2 dy/dx = y.
Ans. (A)
x 25.6.1 General Solution
6. y = is a solution of the differential equation _____.
x +1
The solution which contains as many as arbitrary constants as
dy dy
(A) y 2 = x2 (B) x 2 = y2 the order of the differential equation is called the general solution
dx dx of the differential equation. For example, y = A cos x + B sin x is
dy dy d2 y
(C) y =x (D) x =y the general solution of the differential equation + y = 0 . But
dx dx dx 2
Ans. (B) y = A cos x is not the general solution as it contains one arbitrary
7. The differential equation for all the straight lines which are at a constant.
unit distance from the origin is
2 2 2 2 25.6.2 Particular Solution
æ dy ö æ dy ö æ dy ö æ dy ö
(A) ç y - x ÷ = 1- ç ÷ (B) ç y + x ÷ = 1+ ç ÷
è dx ø è dx ø è dx ø è dx ø The solution obtained by giving particular values to the arbitrary
2 2 2 2 constants in the general solution of a differential equation is
æ dy ö æ dy ö æ dy ö æ dy ö called a particular solution. For example, y = 3 cos x + 2 sin x is a
(C) ç y - x ÷ = 1+ ç ÷ (D) ç y + x ÷ = 1- ç ÷
è dx ø è dx ø è dx ø è dx ø d2 y
particular solution of the differential equation 2 + y = 0 .
Ans. (C) dx
1114 Mathematics Problem Book for JEE
⇒ (1 − 2v − v2) x2 = C ( y / x )3 + 2( y / x ) dv v 3 + 2v dv
⇒ x2 − 2xy − y2 = C Þ =v + x Þ =v + x
1+ 2( y / x )2
dx 1+ 2v 2 dx
Illustration 25.13 Solve the differential equation dv ìï v 2 + 2 ïü ïì 1- v ïü
2
Þx =ví - 1ý = v í 2ý
ïî1+ 2v îï1+ 2v þï
2
dy x 2 + y 2 dx ïþ
=
dx 2 xy 1+ 2v 2 dx 1+ 2v 2 dx
Þ dv = Þ dv =
Solution: Put y = vx. Then differentiating w.r.t. x v (1- v )
2
x v (1 - v )(1 + v ) x
æA B D ö dx
x2 + v2x2 dv Þç + + ÷ dv =
=v + x è v 1- v 1+ v ø x
2 x × vx dx
where,
dx 2v A(1- v )(1+ v ) + Bv (1+ v ) + Dv (1- v ) = 1+ 2v 2
Þ = dv
x 1- v 2 Putting, v = 0, we get A = 1
Integrating both sides, we set 3
C + ln x = −ln(1 − v2) v = 1, we get B =
2
⇒ ln kx + ln(1 − v2) = 0 3
⇒ kx(1 − v2) = 1 ⇒ k(x2 − y2) = x v = − 1, we get D = -
2
Hence,
Illustration 25.14 Solve the differential equation
æ1 3 1 3 1 ö dx
ç + - ÷ dv =
dy è v 2 1- v 2 1+ v ø x
x = y (log y - log x + 1)
dx On integrating both sides, we get
Solution: The given equation may be expressed as 3 ln(1- v ) 3
ln v + - ln(1+ v ) = ln x + ln c
dy y é æ y ö ù 2 -1 2
= log ç ÷ + 1
dx x êë è x ø úû
(1) 3 3
Þ ln v - ln(1- v ) - ln(1+ v ) = ln cx
2 2
y
Let = v . Then æv ö
2
x Þ v / {(1- v )(1+ v )}3 / 2 = cx Þ ç ÷ = (1- v 2 )3
dy dv è cx ø
y = vx ⇒ =v+x
dx dx 3
æ y ö æ y ö
2 2
x2 y2
Therefore, from Eq. (1), Þ ç 2 ÷ = çç 1- 2 ÷÷ Þ ( x 2 - y 2 )3 = 2
è cx ø è x ø c
dv dv
v+x = v (log v + 1) Þ x = v log v Thus, the required solution is
dx dx
⎛ 1 ⎞
dv dx 1 dx ( x 2 − y 2 )3 = Bx 2 y 2 ⎜⎝ 2 = B⎟⎠
= Þò d (log v ) = ò + c c
v log v x log v x
dy dv dy dv
1- = Þ = 1- ⇒(x + y− 2) = C (x−y)3
dx dx dx dx
dv v + 3 2v + 5 Illustration 25.18 Solve the differential equation
Þ 1- = Þ dv = dx
dx 2v + 5 v +2 dy x + y +1
=
æ 1 ö dx 2 x + 2 y + 1
ç2 + ÷ dv = dx
è v +2ø
Solution: x and y coefficients in x + y + 1 are proportional to the x
On integrating both sides, we get and y coefficients in 2x + 2y + 1.
2v + ln(v +2) = x + C Therefore, in this case, take x + y = Z. Then
Put the value of v. Therefore, dy dZ
1+ =
x − 2y + ln(x − y +2) = C dx dx
dZ Z +1
- 1=
Illustration 25.17 Solve the differential equation dx 2Z + 1
dy x + 2 y - 3 dZ Z + 1+ 2 Z + 1 3 Z + 2
= = =
dx 2 x + y - 3 dx 2Z + 1 2Z + 1
Separating the variables and integrating, we get
Solution: Take x = X + A; y = Y + m (A and m are constants). So,
2Z + 1
dy dY ∫ 3Z + 2 dZ = ∫ dx + A
=
dx dX where A is an arbitrary constant.
Therefore, the equation becomes (in X, Y) 2 1
(3 Z + 2 ) -
3 3 dZ = x + A
dY X + 2Y + A + 2m − 3 X + 2Y
= = ò (3 Z + 2 )
dX 2 X + Y + 2A + m − 3 2 X + y
2 1
If A, m are chosen to satisfy Þ Z - log(3 Z + 2) = x + A
3 9
A + 2m − 3 = 0⎫ A = 1⎫ Reverting to x and y (x + y = Z)
⎬⇒ ⎬
2A + m − 3 = 0⎭ m = 1⎭ 2 1
( x + y ) - x - log(3 x + 3 y + 2) = A
In X, Y the equation is homogeneous and of the first degree. Put 3 9
Y = VX. Then 1 1
dV X + 2VX 1+ 2V ⇒ ( - x + 2 y ) - log(3 x + 3 y + 2) = A
V+X = = 3 9
dX 2 X + VX 2 + V ⇒ -3 x + 6 y = log(3 x + 3 y + 2) + C
dV 1+ 2V - (2 + V )V 1- V 2
X = =
dX 2 +V 2 +V Illustration 25.19 Solve the differential equation
Separating the variables (X, V) and integrating, dy x - 3 y + 2
=
2 +V dX dx 3 x - y + 6
ò 1- V 2 dV = ò X
+A
Solution: Given equation is non-homogeneous.
where A is an arbitrary constant. Let x = X + h, y = Y + k. Then
æ1 1 3 1 ö dX dy dY
=
ò çè 2 × 1 + V + 2 × 1 - V ÷ø dV = ò X
+A dx dX
1118 Mathematics Problem Book for JEE
Therefore, x2
dY ( X + h) - 3(Y + k ) + 2 X - 3Y + (h - 3k + 2) (A) y = e x + sin x + + logcos x + c
= = 2
dX 3( X + h) - (Y + k ) + 6 3 X - Y + (3h - k + 6 )
Let us select h and k so that h − 3k + 2 = 0 and 3h − k + 6 = 0. x2
(B) y = e x + sin x + + logsec x + c
Solving, k = 0 and h = −2 2
Therefore,
x2
X = x − h = x + 2, Y = y − k = y (C) y = e x - sin x + + logcos x + c
Hence, 2
dY X - 3Y x2
= , which is homogeneous. (D) y = e x - sin x + + logsec x + c
dX 3 X - Y 2
Now, let Y = vX. Then Ans. (B)
dY dv
=v + X 4. The solution of the differential equation (sin x + cos x)dy +
dX dX
(cos x − sin x)dx = 0 is
X - 3Y dv
Þ =v + X (A) ex (sin x + cos x) + c = 0 (B) ey (sin x + cos x) = c
3X - Y dX
(C) ey (cos x − sin x) = c (D) ex (sin x − cos x) = c
1- 3(Y / X ) dv
Þ =v+ X Ans. (B)
3 - (Y / X ) dX
5. The solution of the differential equation
1- 3v dv
Þ =v + X dy
3-v dX ( x 2 - yx 2 ) + y 2 + xy 2 = 0 is
dx
dv 1- 3v v 2 - 6v + 1
ÞX = -v = æxö 1 1
dX 3 - v 3-v æyö 1 1
(A) log ç ÷ = + + c (B) log ç ÷ = + + c
(3 - v )dv dX èyø x y èxø x y
Þ =
v 2 - 6v + 1 X 1 1 1 1
(C) log ( xy ) = + +c (D) log ( xy ) + + =c
2v - 6 dX x y x y
Þ 2 dv = -2
v - 6v + 1 X Ans. (A)
On integrating both side, we get
6. The solution of the differential equation x(e2y − 1)dy + (x2 − 1)
In(v2 − 6v + 1) = −2 In X + In c eydx = 0 is
⇒ In(v2 − 6v + 1) + In X 2 = In c
⇒ X 2(v2 − 6v + 1) = c x2 x2
(A) e y + e - y = log x - +c (B) e y - e - y = log x - +c
⇒ Y 2 − 6XY + X 2 = c 2 2
Thus, the required solution is x2
y 2 − 6(x + 2)y + (x + 2)2 = c (C) e y + e - y = log x + +c (D) None of these
2
Ans. (A)
7. The solution of the differential equation (3xy + y2)dx +
Your Turn 2 (x2 + xy)dy = 0 is
(A) x2(2xy + y2) = c2 (B) x2(2xy − y2) = c2
1. The solution of the differential equation x cos ydy = (xex log x
+ ex)dx is (C) x2(y2 − 2xy) = c2 (D) None of these
1 x Ans. (A)
(A) sin y = e +c (B) sin y + ex log x + c = 0
x dy xy
8. The solution of the differential equation = is
(C) sin y = ex log x + c (D) None of these dx x 2 + y 2
Ans. (C) 2
/ y2
(A) ay = e (B) ay = e
2 x x/y
dy
2. The solution of the equation = e x - y + x 2e - y is 2 2 2
(C) y = e + e + c (D) y = e + y + c
x y x 2
dx
x3 Ans. (A)
(A) e y = e x + +c (B) ey = ex + 2x + c
3 9. The general solution of the differential equation (2x − y + 1)
(C) ey = ex + x3 + c (D) y = ex + c dx + (2y − x + 1) dy = 0 is
Ans. (A) (A) x2 + y2 + xy − x + y = c (B) x2 + y2 − xy + x + y = c
3. The solution of the differential equation (C) x2 − y2 + 2xy − x + y = c (D) x2 − y2 − 2xy + x − y = c
dy
= e x + cos x + x + tan x is Ans. (B)
dx
Chapter 25 | Differential Equations 1119
æyö 1 æ 1ö dS
Þ ò d ç ÷ = ò - 2 cos ç ÷ dx such that = R , then put S(y) = t. So,
èxø x èxø dy
Hence, dt dS dS dy Rdy
= = × =
y æ 1ö æ 1ö dx dx dy dx dx
x ò
= cos ç ÷ d ç ÷
èxø èxø dt
Thus, the differential equation reduces to + P( x )t = Q( x ),
y ⎛ 1⎞ dx
⇒ = sin ⎜ ⎟ + c , which is the linear differential equation.
x ⎝ x⎠
2. Differential equation of the form
where c is an arbitrary constant. dy
+ Py = Qy n
æ 1ö dx
Thus, y - x sin ç ÷ = cx .
èxø P and Q are functions of x and this equation is called Bernoulli’s
equation.
Illustration 25.25 Solve the differential equation To solve this, divide the equation by yn, then
dy 1 dy 1
+ 2 y tan x = sin x + P n -1 = Q
dx y n dx y
dy 1
Solution: Comparing with + Py = Q , we get Put = t . Then
dx y n-1
(n - 1) dy dt
P = 2 tan x, Q = sin x - =
y n dx dx
I.F. = e ò
2 tan xdx 2 2
= e 2 ln sec x
= eln sec x
= sec2 x
Differential equation reduces to
Multiplying the given equation by I.F. and integrating, we get dt æ 1 ö Q( x )
+ç ÷ P( x )t =
y sec x = ∫ sin x sec xdx = ∫ sec x tan xdx
2 2 dx è n - 1ø (1- n)
1 1 dz dt 1 - x2
e = x 2 . e- x + e- x + c
2 2
− ⋅ = ⇒
(log z )2 z dx dx y 2
Therefore, 2
Thus, ( x 2 + 1+ ce x ) y 2 = 1.
dt t 1
- + = 2
dx x x Illustration 25.30 Solve the differential equation
dt t 1
Þ - =- 2 (1) dy
=
y
dx x x dx y sin y - x
dx 1
∫− x ln 1 Solution: We have
I.F. = e = e − ln x = e x
=
x dy y
1 =
Multiplying Eq. (1) by and integrating, we get dx y sin y - x
x
dx y sin y - x x
t 1 1 1 1 Þ = = sin y -
x ∫ x3
= − dx = 2 + c ⇒ = +c dy y y
2x x log z 2 x 2
Hence, the required solution is dx 1
Þ + x = sin y
dy y
æ 1 ö æ 1ö
÷ x = ç ÷ + cx
2
ç which is a linear differential equation in x and y.
è ln z ø è2ø
1
Illustration 25.29 Solve the differential equation ∫ y dy
Its integrating factor is I.F. = e = elog y = y .
dy
= x 3 y 3 − xy Thus, the solution is
dx
xy = ò y sin y dy
Solution: Here,
dy ⇒ xy = −y cos y + sin y + c
+ xy = x 3 y 3
dx
Therefore,
25.15 Solution of Differential Equation
1 dy 1 of the First Order but of Higher
+ x . 2 = x3 (1)
y 3 dx y Degree
1
Put = z . Then In such differential equations, we substitute the lower degree
y2 derivative by some other variable.
-2 dy dz
= Illustration 25.31 Solve the differential equation
y 3 dx dx
2
æ dy ö dy
Therefore, Eq. (1) becomes x ç ÷ + (y - x) - y = 0
1 dz è dx ø dx
- + xz = x 3 Solution: Here,
2 dx
æ dy ö
dz xp 2 + ( y - x )p - y = 0 ç where p = ÷
Þ - 2 xz = -2 x 3 è dx ø
dx
⇒ xp2 − xp + yp − y = 0
It is in the linear form. So, the integrating factor e ∫
−2 xdx 2
= e− x . ⇒ xp(p − 1) + y(p − 1) = 0
Multiplying by it, ⇒ (p − 1) (xp + y) = 0
2 dz
e- x - 2 xe - x × z = -2 x 3 × e - x
2 2
Thus,
dx p−1=0
d or
( ze - x ) = -2 x 3e - x
2 2
Þ xp + y = 0
dx
Now,
Therefore,
p−1=0
d ( ze − x ) = −2 x 3e − x dx
2 2
dy
Þ =1 ⇒ y = x + c
Þ ò d ( ze - x ) = ò -2 x 3 . e - x dx
2 2
dx
Also,
Þ ze - x = - ò te -t dt xp + y = 0
2
(putting x2 = t)
dy
é e -t e -t ù x +y =0
= - êt . -ò dt ú = te -t - ò e -t dt dx
êë -1 -1 úû ⇒ xdy + ydx = 0
Chapter 25 | Differential Equations 1123
25.16.1 Problem Based on Rate of Change Illustration 25.35 A and B are two separate reservoirs of water.
The capacity of A is double that of B. Both the reservoirs are filled
Illustration 25.33 A particle moves in a straight line with a completely with water. Water is released simultaneously from
dx both the reservoirs. For each of the reservoirs, the rate of flow
velocity given by = ( x + 1) (x is the distance described). What is out at any instant is proportional to the quantity of water left in
dt
the time taken by a particle to transverse a distance of 99 metres? the reservoir. After 1h, the quantity of water in A is 1.5 times the
quantity of water in B. After how many hours from the time of
Solution: We have
dx release of water, do both A and B have the same quantity of water?
= dt
x +1 Solution: Let at time t hours, the volume of water in A and B be
On integrating, we get u and v, respectively. From the question,
99 dx t
du
ò0 x + 1 ò0
= dt
dt
= -k1u (1)
and
⇒ [ln( x + 1)]0 = t
99
dv
Therefore, time taken by the particle is = - k 2v (2)
dt
t = ln100 = loge (10 )2 = 2 loge 10 At t = 0, u = 2V and v = V (from the question). Solving Eq. (1),
du
Illustration 25.34 The rate of cooling of a substance in = -k1dt
u
moving air is proportional to the difference of temperatures of Integrating both sides, we get
the substance and the air. A substance cools from 36°C to 34°C in log u = −k1t + c1
15 min. Find when the substance will have the temperature 32°C,
u = e - k1t + c1 (3)
and it is known that the constant temperature of air is 30°C.
When t = 0, u = 2V. Then
Solution: Let the temperature of the substance at time t minutes 2V = e c1
is T. Hence, Eq. (3) gives
The rate of cooling of the substance = k(T− 30)°C/min (from the
u = 2Ve - k1t (4)
question).
But the rate of cooling = rate of decrease of temperature Similarly, from Eq. (2), we get
= -
dT v = Ve - k2 t (5)
dt 3
Therefore, from the question, From the question, if v = v0 when t = 1, then u = v 0 .
2
dT Therefore, Eq. (4) gives
- = k (T - 30 )
dt 3
dT v 0 = 2Ve - k1
or - = kdt 2
T - 30 and Eq. (5) gives
Hence,
34 15 v 0 = Ve - k2
dT
ò - T - 30 = ò kdt Dividing these, we get
3 2e - k1
36 0
34
⇒ [ - log(T - 30 )] = k [t ]
15 = = 2e k2 - k1
36 0 2 e - k2
Therefore, Thus,
3
6 1
log = k .15 ⇒ k = log
3 e k2 - k1 = (6)
4 15 2 4
Again, Let after T hours, the volume of water in A and B be equal.
32 t
dT Therefore, Eqs. (4) and (5) gives
ò - T - 30 = ò kdt 2Ve - k1T = Ve - k2T
36 0
But in the first quadrant, x, y are positive. So, xy = 1. This is the singular y +4 6+4
solution of Eq. (2), which is a curve, and it passes through (1, 1). When y = 6, then x = = = 5.
2 2
Thus, the required curves (or lines) are x + y = 2 and xy = 1. So, B = (5, 6).
Illustration 25.39 A curve through (1, 2) has its slope at any
The required area = Area (OABDO) − Area (BCAODB) (1)
2
point (x, y) equal to . Find the area of the region bounded Now,
y -2
by the curve and the line 2x − y − 4 = 0. 6 6
y+4 1 ⎡ y2 ⎤
6
Figure 25.3 è dx ø dx
dx 2v
Solving y2 = 4(x + y) − 8 and 2x − y − 4 = 0, we get ⇒ + dv = 0
x 1+ v 2
⇒ y2 = 2(y + 4) + 4y − 8
On integrating, we get
⇒ y2 − 6y = 0
⇒ y = 0, 6 log x + log(1 + v2) = log c
1128 Mathematics Problem Book for JEE
Therefore, x(1 + v2) = c 8. The slope of a curve at any point is the reciprocal of twice the
ordinate at the point and it passes though the point (4, 3). The
æ y2 ö
Þ x çç 1+ 2 ÷÷ = c equation of the curve is
è x ø (A) x2 = y + 5 (B) y2 = x − 5
⇒ x + y2 = cx
2
(C) y = x + 5
2 (D) x2 = y − 5
Ans. (C)
Solution: Solution:
dy ( y + y + 1)
2
(1 + cos 2 x ) dy = (1 + e y ) sin 2 x dx
(i) =- 2
dx ( x + x + 1) Separating the variables
dy dx dy sin 2 x
⇒ 2 =- 2 = dx
y + y +1 x + x +1 1 + e y 1 + cos 2 x
On integrating, é (1 + e y ) - e y ù
⇒ ò êê ú dy = ò tan x dx
ë 1+ e
dy dx y
úû
ò 2
=-ò 2
+k
1ö æ 3 ö 1ö æ 3 ö
2 2
æ æ ⇒ y - log (1 + e ) + log cos x = A
y
ç y + ÷ + çç ÷ ç x + ÷ + çç ÷
è 2 ø è 2 ÷ø è 2 ø è 2 ÷ø
æ cos x ö
⇒ y + log çç ÷=A
y ÷
æ 1 ö æ 1 ö è 1+ e ø
2 ç y+ ÷ 2 ç x+ ÷
-1
tan ç 2 ÷=- -1
tan ç 2 ÷+k p
Taking y = 0, when x = we get
3 ç 3 ÷ 3 ç 3 ÷ 4
ç ÷ ç ÷
è 2 ø è 2 ø æ 1 ö 3
0 + log ç ÷ = A Þ A = - log 2
which may also be written as è2 2 ø 2
æ 2y + 1 ö -1 æ 2x + 1 ö The solution (called particular solution) is
tan - 1 ç ÷ + tan ç ÷=c (1)
è 3 ø è 3 ø éæ cos x ö ù
2 y + log êçç y ÷
÷ 2 2ú = 0
where c is now arbitrary constant in which is also êëè 1 + e ø ûú
3
absorbed.
Therefore, 2 2 cos x = (1 + e y ) e - y = e - y + 1.
We know,
3. Find the equation of the curve through the origin which
æ a+b ö
tan-1 a + tan-1 b = tan-1 ç ÷ dy
è 1 - ab ø satisfies the differential equation = ( x - y )2 .
dx
So, Eq. (1) reduces to
Solution: Put x − y = z. Then
2xy + x + y + C′ (x + y + 1) = 1
dy dz
dy dx 1- =
(ii) = dx dx
1 + cos 2 y 4 x - x2 The equation in z is
dz
1 dy dx 1- = z2
2 ò cos2 y ò 4 - ( x - 2)2
⇒ = +c dx
dz
⇒ = 1- z 2
The solution is dx
tan y 1 - 1 æ x - 2 ö dz
2
= sin ç
2 è 2 ø
÷+c ⇒ ò 1 - z 2 = ò dx + A
æ x -2ö 1 æ 1+ z ö
⇒ tan y = sin-1 ç ÷+k ⇒ log ç ÷= x + A
è 2 ø 2 è 1- z ø
where ‘k’ is an arbitrary constant.
æ 1+ x - y ö
dy y 2 y So, log ç ÷ = 2 x + 2 A is the solution which may also be
(iii) + = è 1- x + y ø
dx x 2 x
dv æ 1+ x - y ö
On setting y = vx, the equation is v + x + v 2 = v . Separating written as ç ÷ = Ce , where C is an arbitrary constant.
2x
dx è 1- x + y ø
the variables and integrating,
The curve passes through the origin. Put x = 0, y = 0, we get
dx dv
ò x ò v2
+ =A 1
= C Þ C =1
1
1 Therefore, the particular solution is (1 + x − y) = (1 − x + y) e2x.
⇒ log x − = A.
v dy
This simplifies to the form x = cex/y. 4. Solve the differential equation ( x 2 - y 2 ) = 2 xy given that
y = 1, x = 1. dx
dy
2. Solve the differential equation (1 + cos 2 x ) - (1 + e y ) sin2x = 0;
dx Solution: Taking y = vx and rewriting, the equation is
p dv 2 x × vx
given that y = 0, when x = . v+x = 2
4 dx x - v 2 x 2
1130 Mathematics Problem Book for JEE
dV æ 12 + 5V + 2V 2 + 5V ö
⇒ X = - çç ÷÷
dX è 5 + 2V ø
D
Separating the variables and integrating,
2V + 5 dX Figure 25.4
ò 2V 2 + 10V + 12 dV = - ò X
+A
Let y = vx. Then
dv (1+ v 2 ) 1+ v dx
1 x=- Þ 2 dv = -
⇒ log (2V 2 + 10V + 12) + log X = A dx 1+ v v +1 x
2
On integrating both sides, we get
⇒ (2V 2 + 10V + 12) X 2 = e 2 A = C 1 2v dv dx
2 ò v2 +1
dv + ò 2
v +1 ò x
=-
⇒ 2Y 2 + 10 XY + 12 X 2 = C
1
C Þ log| v 2 + 1| + tan-1 | v | = - log x + c
⇒ Y 2 + 5 XY + 6 X 2 = =C’ 2
2
⇒ ( y + 3)2 + 5( x − 2) ( y + 3) + 6( x − 2)2 = C ’ { }
Þ log ( v 2 + 1) x + tan-1 v = c
y
which gives the solution in the form 6x2 + 5xy + y2 − 9x − 4y = C′. Þ log x 2 + y 2 + tan-1 =c
Alternative method: The equation may be rewritten as x
As x = 1 and y = 0, we get
(12 x + 5 y − 9 ) dx + (5 x + 2 y − 4 ) dy = 0
log1 + tan−1 0 = c
(12 x − 9 ) dx + (2 y − 4 ) dy + 5 y dx + 5 x dy = 0 ⇒c=0
(12 x − 9 ) dx + (2 y − 4 ) dy + 5d ( xy ) = 0
2 2
( èxø
)
-1 æ y ö
Therefore, the required curve is log x + y + tan ç ÷ = 0 .
Chapter 25 | Differential Equations 1131
dy Therefore,
7. Solve + y f ′( x ) = f ( x ) . f ′( x ) where f (x) is a given function
dx dy1
of x. + P ( x ) × y1 = Q ( x ) (2)
dx
Solution: Here,
dy dy 2
= {f ( x ) - y } f ¢( x ) (1) and + P ( x ) × y2 = Q ( x ) (3)
dx dx
Put f (x) − y = z. Then From Eqs. (1) and (2),
dy dz æ dy dy1 ö
f ¢( x ) - =
dx dx ç - ÷ + P ( x ) × ( y - y1) = 0
è dx dx ø
Hence, Eq. (1) becomes
Therefore,
dz d
f ¢( x ) - = zf ¢( x ) ( y - y1) + P ( x ) . ( y - y1) = 0 (4)
dx dx
dz dz From Eqs. (2) and (3),
= (1- z ) f ¢( x ) ⇒ = f ¢( x ) dx
dx 1- z d
( y1 - y 2 ) + P ( x ) × ( y1 - y 2 ) = 0 (5)
dz dx
⇒ ò
1- z ò
= f ¢( x ) dx ⇒ −log (1 − z) = f (x) − c
From Eqs. (4) and (5),
f(x) + log(1 − z) = c d
( y - y1)
Thus, f(x) + log {1 + y − f (x)} = c. dx y - y1
=
1 - y2
d y
( y1 - y 2 )
8. Find the equation of the curve passing through (1, 2) whose dx
differential equation is y(x + y3) dx = x(y3 − x)dy. Therefore, on integrating,
Solution: Here, log(y − y1) = log(y1 − y2) + log c
(xy + y4)dx = (xy3 − x2)dy ⇒ log(y − y1) = log [c(y1 − y2)]
⇒ y3(ydx − xdy) + x(ydx + xdy) = 0 ⇒ y = y1 + c (y1 − y2)
dx
d2 y dy
dp (C) =y (D) =y
or = adx dx 2 dx
p3
Solution:
On integrating, we get
dy d2 y
1 = a cosx−b sinx ⇒ = −a sinx − bcosx = −y
- 2 = ax + b dx dx 2
2p
d2 y
-1 1 Hence, the equation is + y = 0.
⇒ = p2 ⇒ p = - dx 2
2(ax + b ) 2(ax + b )
Hence, the correct answer is option (A).
dy -1 15. For any differential function y = f(x), the value of
⇒ =
dx 2(ax + b ) 3
d 2 y æ dy ö d 2 x
Therefore, +ç ÷ is:
dx 2 è dx ø dy 2
1 -2ax - b
y=ò dx Þ y = +c (A) always zero (B) always non-zero
-2ax - b -a
where a, b and c are arbitrary constants. (C) equal to 2y2 (D) equal to x2
11. A curve that passes through (2, 4) and having subnormal of Solution: For a differential equation,
constant length of 8 units can be -1
(A) y2 = 16x − 8 (B) y2 = −16x + 24 æ dy ö æ dx ö
ç ÷ = ç ÷
(C) x = 16y − 60
2 (D) x2 = −16y + 68 è dx ø è dy ø
-2
Solution: Let the curve be y = f(x). d2 y æ dx ö d æ dx ö dy
dy Þ = -1ç ÷ ç ÷
Subnormal at any point = y dx 2 è dy ø dy è dy ø dx
dx -2 3
æ dx ö d 2 x dy d 2 x æ dy ö
dy y 2 = -ç ÷ =- 2 ç ÷
y = ±8 ⇒ ydy = ±8 dx ⇒ = ±8x + c è dy ø
2
dy dx dy è dx ø
dx 2
3
⇒ y2 = 16x + 2c1 ⇒ c1 = −8 d2 y 2
æ dy ö d x
Þ +ç ÷ =0
or y2 = −16x + 2c2 ⇒ c2 = 24 dx 2 è dx ø dy 2
Hence, the correct answers are options (A) and (B). Hence, the correct answer is option (A).
2
12. Equation of a curve that would cut x2 + y2 − 2x − 4y − 15 = 0 d2 y æ dy ö d2 y
16. The degree of differential equation + 3 ç ÷ = x log
orthogonally can be is dx 2 è dx ø dx 2
(A) (y − 2) = l(x − 1) (B) (y − 1) = l(x − 2)
(A) 1 (B) 2
(C) (y + 2) = l(x + 1) (D) (y + 1) = l(x + 2)
(C) 3 (D) none of these
where l∈R.
Solution: Any line passing through the centre of the given circle Solution: Since the equation is not a polynomial in all the
would meet the circle orthogonally. differential coefficient so the degree of equation is not defined.
Hence, the correct answer is option (A). Hence, the correct answer is option (D).
13. Let m and n be the order and the degree of the differential 17. The differential equation of all ellipses centred at origin is
equation whose solution is y = cx +c2 − 3c3/2 +2, where c is a (A) y2 + xy12 − yy1 = 0 (B) xyy2 + xy12 − yy1 = 0
parameter. Then (C) yy2 + xy12 − xy1 = 0 (D) none of these
(A) m = 1, n = 4 (B) m = 1, n = 3
(C) m = 1, n = 2 (D) None Solution: The ellipse centred at origin is given by
2x 2y x y Solution:
+ y1 = 0 ⇒ 2 + 2 y1 = 0 (2) y = (c1 + c4ecs)ex + c2e2x + c3e3x
a2 b 2 a b
On differentiating again, we get ⇒ y = k1ex + k2e2x + k3e3x
Since there are three arbitrary constants.
1 1
+ (y12 + yy2) = 0 (3) Hence, the correct answer is option (C).
a2 b 2
dx
Multiplying Eq. (3) with x and then subtracting from Eq. (2) we get 21. I.F. for y ln y + x - ln y = 0 is
dy
1
( yy1 - xy12 - xyy 2 ) = 0 Þ xyy 2 = xy12 - yy1 = 0 (A) ln x (B) ln y
b2 (C) ln xy (D) none of these
Hence, the correct answer is option (B). Solution:
18. Particular solution of y1 + 3xy = x which passes through (0, 4) is dy
∫ y In y
-
3x 2
I.F. = e = ln y
1
+ 11 e - x
2
(A) 3y = 1 + 11 e 2 (B) y = Hence, the correct answer is option (B).
3
11 - x 2 1 3 2
x 22. Which one of the following is a differential equation of the
(C) y = 1 + e (D) y = + 11 e 2 family of curves y = Ae2x + Be−2x
3 3
Solution: d2 y dy d2 y dy
dy (A) - 2 + 2y = 0 (B) x + 2 - xy + x 2 - 2 = 0
+ (3x)y = x dx 2 dx dx 2 dx
dx 3
d2 y æ dy ö æ dy ö
(D) ç ÷ = 4 y ç x - 2y ÷
3 2
ò 3 x dx x (C) = 4y
I.F. = e = e2 dx 2 è ø
dx è dx ø
Therefore, the solution of given equation is Solution:
3 2 3 2 3 dy
x 1 x2 x y = Ae2x + Be−2x ⇒ = 2(Ae2x − Be−2x)
=ò
ye 2 + c = e2 + c
x .e 2 dx dx
3
If the curve passes through (0, 4), then d2 y
11 = 4 (Ae2x + Be−2x) = 4 y
1 dx 2
4− =c⇒c=
3 3
3 3
Hence, the correct answer is option (C).
1 11 - x 2 - x 2
y = + e 2 ⇒ 3y = 1 + 11 e 2 dy æx+yö æ x - y ö is
3 3 23. Solution of + sin ç ÷ = sin ç ÷
dx è 2 ø è 2 ø
Hence, the correct answer is option (A). y x y x
(A) log tan = c − 2 sin (B) log cot = c − 2 sin
dy 3 x - 4 y - 2 4 2 4 2
19. Solution of equation = is
dx 3 x - 4 y - 3 y x
(C) log tan = c − 2 cos (D) None of these
(A) (x − y)2+ c = log(3x − 4y + 1) 4 2
(B) x − y + c = log(3x − 4y + 1) Solution:
(C) x − y + c = log(3x − 4y − 3) dy x y x y
(D) x − y + c = log(3x − 4y − 1) = -2 cos sin Þ - ò 2 cos dx = ò cosec dy
dx 2 2 2 2
Solution: Let 3x − 4y = z. Then x
⇒ c − 2 sin
dy dz dy 1 æ dz ö 2
3−4 = ⇒ = ç3- ÷
dx dx dx 4 è dx ø y
= log tan
Therefore, the given equation becomes 4
3 1 dz z - 2 Hence, the correct answer is option (A).
- =
4 4 dx z - 3 dy y y
24. Solution of = + tan is
æ z -3 ö æ 4 ö dx x x
Þ -ç ÷ dz = dx Þ - ç 1- ÷ dz = dx
è z +1 ø è z + 1ø æyö y
⇒ −z + 4 log(z + 1) = x + c ⇒ log(3x − 4y + 1) = x − y + c (A) sin ç ÷ = kx (B) cos = kx
èxø x
Hence, the correct answer is option (B). y
(C) tan = kx (D) none of these
20. The order of the differential equation, whose general solution x
is y = c1ex + c2e2x + c3e3x + c4 e x + c5 , where c1, c2, c3, c4 and c5 Solution:
are arbitrary constants, is dy y y
= + tan
(A) 5 (B) 4 (C) 3 (D) None of these dx x x
1134 Mathematics Problem Book for JEE
Put y = vx. Then 3. The differential equation of the family of circles with fixed
dv radius 5 units and centre on the line y = 2 is
v+x = v + tan v
dx (A) (x − 2)y ′2 = 25 − (y − 2)2 (B) (y − 2)y ′2 = 25 − (y − 2)2
dx (C) (y − 2)2 y ′2 = 25 − (y − 2)2 (D) (x − 2)2 y ′2 = 25 − (y − 2)2
⇒ cot vdv =
x [AIEEE 2008]
On integrating, we get
y Solution: The equation of the circle is
ln sin v = ln x + ln k ⇒ sin = kx (x − a)2 + (y − 2)2 = 25 (1)
x
On differentiating w.r.t. x,
Hence, the correct answer is option (A).
dy dy
( x − α ) + ( y − 2) = 0 ⇒ x − α = −( y − 2) (2)
dx dx
Previous Years' Solved JEE Main/AIEEE From Eqs. (1) and (2), on eliminating a,
Questions 2
æ dy ö
( y - 2)2 ç ÷ + ( y - 2)2 = 25 Þ ( y - 2)2 ( y ¢)2 = 25 - ( y - 2)2
1. The differential equation of all circles passing through the è dx ø
origin and having their centres on the x-axis is
Hence, the correct answer is option (C).
dy dy
(A) x 2 = y 2 + xy (B) x 2 = y 2 + 3 xy 4. The differential equation which represents the family of curves
dx dx
y = c1e c2 x where c1 and c2 are arbitrary constants is
dy dy
(C) y 2 = x 2 + 2 xy (D) y 2 = x 2 - 2 xy (A) y′ = y2 (B) y″ = y′y (C) yy″ = y′ (D) yy″ = (y′)2
dx dx
[AIEEE 2007] [AIEEE 2009]
Solution:
Solution: General equation of such circle is
y = c1e c2 x (1)
( x + g ) + y = g Þ x + g + 2 xg + y = g
2 2 2 2 2 2 2
y ¢ = c2c1e c2 x
Þ y ¢ = c2 y (2)
⇒ x2 + y2 + 2gx = 0 (1) y ¢¢ = c2 y ¢
Now differentiating w.r.t. x, we get
y¢
dy dy From (2), we get, c2 = . Therefore,
2x + 2y + 2g = 0 Þ x + y +g=0 (2) y
dx dx
( y ¢)2
Therefore, from Eq. (1) y ¢¢ = Þ yy ¢¢ = ( y ¢)2
y
⎛ dy ⎞
x2 + y2 + 2⎜ − x − y ⎟ x = 0 Hence, the correct answer is option (D).
⎝ dx ⎠
dy 5. Solution of the differential equation
⇒ x 2 + y 2 − 2 x 2 − 2 yx =0
dx p
cos x dy = y (sin x - y )dx , 0 < x < is
dy 2
⇒ y 2 = x 2 + 2 xy
dx (A) y sec x = tan x + c (B) y tan x = sec x + c
Hence, the correct answer is option (C). (C) tanx = (sec x + c)y (D) sec x = (tan x + c)y
dy x + y
2. The solution of the differential equation = satisfying [AIEEE 2010]
dx x
the condition y (1) = 1 is Solution: We have
dy
(A) y = In x + x (B) y = x In x + x2 cos x dy = y (sin x − y ) dx ⇒ = y tan x − y 2 sec x
dx
(C) y = xe(x − 1) (D) y = x In x + x 1 dy 1
[AIEEE 2008] ⇒ − tan x = − sec x
y 2 dx y
Solution: Put y = vx. Then
1
dy dv dv dx Let us consider that = t . Then
=v + x Þv + x = 1+ v Þ dv = y
dx dx dx x
1 dy dt dt dt
Therefore, − 2
= ⇒ − − t tan x = − sec x ⇒ + (tan x ) t = sec x
y dx dx dx dx
y
v = ln x + c Þ = ln x + c
So, I.F. = e ∫
tan x dx
x = sec x .
As y (1) = 1, we have, y = x In x + x. Solution is
Hence, the correct answer is option (D). t (I.F) = ∫(I.F) sec xdx
Chapter 25 | Differential Equations 1135
1
⇒sec x = tan x + c Þ sec x = y (tan x + c ) kT 2 kT 2
I= +c Þc =I -
y 2 2
Hence, the correct answer is option (D). Now, at t = T (for scrap value)
6. Let f: (−1, 1) → R be a differentiable function with f(0) = −1 kT 2
and f ′(0) = 1. Let g(x) = [f(2f(x) + 2)]2. Then g′(0) = V (T ) = c = I −
2
(A) −4 (B) 0 Hence, the correct answer is option (A).
(C) −2 (D) 4 9. At present, a firm is manufacturing 2000 items. It is estimated
[AIEEE 2010] that the rate of change of production P w.r.t. additional
Solution: We have dP
æ d ö number of workers x is given by = 100 − 12 x . If the firm
g¢( x ) = 2(f (2f ( x ) + 2)) ç (f (2f ( x ) + 2)) ÷ dx
è dx ø employs 25 more workers, then the new level of production
= 2f (2f ( x ) + 2)f ¢(2f ( x ) + 2) × (2f ¢( x )) of items is
This implies that (A) 3000 (B) 3500
g¢(0 ) = 2f (2f (0 ) + 2) × f ¢(2f (0 ) + 2) × 2(f ¢(0 )) (C) 4500 (D) 2500
[JEE MAIN 2013]
= 4f (0 )f ¢(0 ) = 4( -1)(1) = -4
Solution: Given that
Hence, the correct answer is option (A).
dP
dy = 100 - 12 x Þ dP = (100 - 12 x )dx
7. If = y + 3 > 0 and y(0) = 2, then y(ln2) is equal to dx
dx
Therefore, the new level of production of items is
(A) 5 (B) 13
P 25
(C) −2 (D) 7
[AIEEE 2011]
∫ dP = ∫ (100 − 12 x )dx
2000 0
Solution: We have 12 × 2
dy dy ⇒ ( P − 2000 ) = 25 × 100 − (25)3 / 2
= y +3Þ = dx Þ ln( y + 3) = x + c 3
dx y +3 ⇒ P = 3500
when x = 0, y = 2. So, Hence, the correct answer is option (B).
In 5 = 0 + c 10. Let the population of rabbits surviving at a time t be governed
Þ c = ln5 dp(t ) 1
by the differential equation = p(t ) - 200. If p(0) = 100,
So, ln( y + 3) = x + ln 5 Þ y + 3 = e x +ln 5 dt 2
then p(t) equals
Þ y + 3 = eln 2 +ln 5 Þ y + 3 = 10 Þ y = 7
(A) 600 − 500 et/2 (B) 400 − 300 e−t/2
Hence, the correct answer is option (D). (C) 400 − 300 et/2 (D) 300 − 200 e−t/2
8. Let I be the purchase value of an equipment and V(t) be the value [JEE MAIN 2014 (OFFLINE)]
after it has been used for t years. The value V(t) depreciates at a Solution:
dV (t ) dp(t ) 1 dp(t )
rate given by differential equation = -k (T - t ), where k > = p(t ) − 200 ⇒ = dt
dt pt 2 1
0 is a constant and T is the total life in years of the equipment. p(t ) − 200
2
Then the scrap value V(T) of the equipment is
On integrating, we get
kT 2 k (T - t )2
(A) I - (B) I - log
p(t )
- 200
2 2 2 p(t ) - 400 1 c1
= t + c1 Þ log = t+ (1)
1
(C) e - kT (D) T 2 - (1/ 2) 2 2 2
k
Using initial conditions
[AIEEE 2011] 100 − 400 c1
Solution: We have, log = ⇒ 2 log150 = c1 (2)
dV 2 2
= -k (T - t ) Þ dV = -k (T - t )dt
dt From Eqs. (1) and (2), we get
On integrating the above equation, we get
p(t ) - 400 1 p(t ) - 400 1
log = t + log150 Þ log = t
-k (T - t )2 k (T - t )2 2 2 300 2
V= +c ÞV = +c
( -2) 2 p(t ) - 400
= e t / 2or p(t ) - 400 = 300e t / 2
As V(0) = I, we have 300
1136 Mathematics Problem Book for JEE
(dy
) 2 2
touching x-axis at the origin is x 2 - y 2 = g( x ) y , then g( x ) ⎛ x⎞ y2 ⎛ y⎞ ⎛ 1 ⎞
f ⎜ ⎟ = − 2 = −⎜ ⎟ = −⎜
⎝ x / y ⎟⎠
dx ⎝ x⎠
equals ⎝ y⎠ x
2
1 ⎛ 1⎞ 1
(A) x (B) 2x2 Therefore, f (t ) = − ⎜ ⎟ and f (2) = − .
2 ⎝t⎠ 4
1 2 Hence, the correct answer is option (D).
(C) 2x (D) x
2 13. The general solution of the differential equation
[JEE MAIN 2014 (ONLINE SET-1)] æ dy ö
sin 2 x ç - tan x ÷ - y = 0 is
Solution: Equation of family is (x − 0)2 + (y − a)2 = a2 è dx ø
(A) y tan x = x + c (B) y cot x = tan x + c
(C) -
4
(D)
2 ( )
Þ 8 t13 - t 3 = 3t ´ 4 t12 - t 2 ( )
5 5
[JEE MAIN 2016 (OFFLINE)] Þ 2 (t 3
1 - t3 ) = 3t (t - t ) 2
1
2
Þ 2 (t + t t ) = 3t (t + t )
Solution: We have 2
1 + t2 1 1 (t ≠ t1)
y(1 + xy)dx = xdy
⇒ ydx − xdy = − xy2dx Þ 2t12 + 2t 2 + 2t1t = 3tt1 + 3t 2
⎛ y dx − x dy ⎞
⇒ ⎜ ⎟⎠ = − x dx Þ 2t12 + t 2 + t1t = 0
⎝ y2
Therefore, Þ t12 + t 2 + t12 - t1t = 0
æxö That is,
ò d çè y ÷ø = -ò x dx (t1 − t)(t1 + t) + t1(t1 − t) = 0 ⇒ 2t1 + t = 0 ⇒ t = −2t1 ⇒ t1 = −
t
2
x x2
Þ =- +c (1)
y 2 Hence, the coordinates of point Q are obtained as follows:
x x 1 2 constant)
=- - 1
y 2 2 1 C -x
(A) Cx 3 (B) e
ex x2
x ( x 2 + 1)
Þ =- C -x
1
C -x
1
y 2 (C) e (D) e
x x3
-2 x 1
Þy= and x = - [JEE MAIN 2016 (ONLINE SET-2)]
x2 +1 2
Solution: On applying Newton−Leibniz rule, we get
Therefore,
æ 1ö -2( -1/ 2) 4
f ç- ÷ = - = x x
è 2ø 1+ (1/ 4 ) 5 x [ y ( x ) - 0] + ò y (t )dt = ò ty (t )dt + ( x + 1)( xy ( x ) - 0 )
1 1
Hence, the correct answer is option (A).
x x
19. If the tangent at a point P, with parameter t, on the curve xy ( x ) + ò y (t )dt = ò ty (t )dt + x 2 y ( x ) + xy ( x )
x = 4t2 + 3, y = 8t3 − 1, t ∈\ meets the curve again at a point 1 1
ò y (t )dt = ò ty (t )dt + x
2
(A) (16t2 + 3, − 64t3 − 1) (B) (4t2 + 3, − 8t3 − 1) y( x )
(C) (t + 3, t − 1)
2 3 (D) (t2 + 3, − t3 − 1) 1 1
dx 8t e = eè xø
= x 3 × e1/ x
( 8t
3
1 ) (
− 1− 8t 3 + 1 = 3t 4t12 + 3 − 4t 2 − 3 ) y(x)x3e1/x = C
Hence, the correct answer is option (D).
Chapter 25 | Differential Equations 1139
dx dy æp ö p
2
æp ö p
2
Þò =ò (A) y ç ÷ = (B) y ¢ ç ÷ =
x x -1 2
y y -1 2
è4ø 8 2 è 4 ø 18
sec -1 x = sec -1 y + c æp ö p
2
æ p ö 4p 2p
2
(C) y ç ÷ = (D) y ¢ ç ÷ = +
æ 2 ö è3ø 9 è3ø 3 3 3
sec -1 2 = sec -1 ç ÷+c
è 3ø
[IIT-JEE 2012]
p p p
c= - = Solution:
3 6 6
dy
p - y tan x = 2 x sec x
Þ sec -1 x = sec -1 y + dx
6
dy
æ pö Þ cos x + ( - sin x ) y = 2 x
y = sec ç sec -1 x - ÷ dx
è 6ø d
Þ ( y cos x ) = 2 x
Now, dx
1 1 p Þ y ( x )cos x = x 2 + c , where c = 0 since y (0 ) = 0
cos -1 = cos -1 +
x y 6
p
1 1 3 When x = . Then
cos -1 = cos -1 + cos-1 4
y x 2 2
⎛p ⎞ p
1 1 3 1 3 y⎜ ⎟ =
= × - 1- 2 1- ⎝ 4⎠ 8 2
y x 2 x 4
1 3 1 1 p
= - 1- 2 × When x = . Then
y 2x x 2 3
2
⎛ p ⎞ 2p
1 3 1 1 y⎜ ⎟ =
= - 1- 2 × ⎝ 3⎠ 9
y 2x x 2
2
p ⎛p ⎞ p p
2
=
3 1
- 1- 2 When x = . Then y′ ⎜ ⎟ = +
4 ⎝ 4⎠ 8 2 2
y x x
Hence, the correct answer is option (C). p æ p ö 2p
2
4p
When x = . Then y¢ç ÷ = + .
4. Let y′(x) + y(x)g′(x) = g(x)g′(x), y(0) = 0, x ∈ \, where f ′(x)
3 è3ø 3 3 3
df ( x ) Hence, the correct answers are options (A) and (D).
denotes and g(x) is a given non-constant differentiable
dx
6. The function y = f (x) is the solution of the differential equation
function on \ with g(0) = g(2) = 0. Then, the value of y(2) is
dy xy x4 + 2x
_____. + 2 = in (−1, 1) satisfying f (0) = 0. Then,
[IIT-JEE 2011] dx x - 1 1- x 2
Solution: 3
y′(x) + y(x)g′(x) = g(x)g′(x) 2
Therefore, ex 1
where, P = ,Q =
c=0 1+ e x 1+ e x
ex
x5 ∫ 1+ e x dx
I.F. = e ∫ = eln(1+ e ) = (1+ e x )
pdx x
p p
sin2 q cosq dq æ 1- cos 2q ö y = e−x y = x + e−x
ò0
3
cosq
=ò3ç
0 è 2
÷ dq
ø 3
p p
é1 1 é sin 2q
ù3 ù3
=ê qú - ê ú
ë û0
2 2 ë 2 û0 x’ x
−3 −2 −1 0
1 p 1 é 2p ù
= ´ - êsin ú
2 3 4ë 3 û
p 1 3 p 3
= - = -
6 4 2 6 8
Hence, from Eq. (3) y’
æp 3ö p 3 Figure 25.5
2 çç - ÷÷ = -
è 6 8 ø 3 4 Therefore, y(x) has a critical point in (−1, 0). (See Fig. 25.5)
Hence, the correct answer is option (B). Hence, the correct answers are options (A) and (C).
1142 Mathematics Problem Book for JEE
8. Consider the family of all circles whose centres lie on the Now,
straight line y = x. If this family of circles is represented by dx
the differential equation Py ′′ + Qy ′ + 1 = 0, where P, Q are ( x + 2)2 + y ( x + 2) - y 2 =0
dy
æ dy d2 y ö
functions of x, y and y ′ çç here y ¢ = , y ¢¢ = 2 ÷÷ , then which dt
è dx dx ø Þ t 2 + yt - y 2 =0
dy
of the following statements is (are) true?
dt
(A) P = y + x (B) P = y − x Þ y2 - yt - t 2 = 0
dy
(C) P + Q = 1 − x + y + y ′ + (y ′)2 (D) P − Q = x + y − y ′ − (y ′)2
1 dt 1 1
[IIT-JEE 2015] Þ - =
t 2 dy yt y 2
Solution: Let the family of circles be
(x − h)2 + (y − h)2 = r2 (1) 1
Let = z . Therefore,
⇒ x2 + y2 − 2xh − 2yh + 2h2 − r2 = 0 t
On differentiating this w.r.t. x, we get dt æ 1 ö dz
2x + 2yy ′ – 2h – 2hy ′ = 0 ç- ÷ =
dy è t 2 ø dy
⇒ x + yy ′ – h – hy ′ = 0 (2)
On differentiating again w.r.t. x, we get Now,
1 + yy ′′ + (y ′)2 – hy ′′ = 0 -dz z 1 dz z -1
- = Þ + =
dy y y 2 dy y y 2
⇒ (y – h)y ′′ + (y ′)2 +1=0 (3)
1
From Eq. (2) ⇒ ∫ d ( zy ) = ∫ − dy
x + yy ¢ y
h= (4)
1+ y ¢ Þ zy = - ln y + c
dy
[ x 2 + 4 x + 4 + y ( x + 2)] - y2 = 0 (x > 0)
dx e2
which is further simplified as follows: 3e
(x + 2)
dy
[( x + 2) + y ( x + 2)] - y 2 = 0
2
dx x
−2 0
Substituting x + 2 = t, we get
dx dt
=
dy dy Figure 25.6
Chapter 25 | Differential Equations 1143
That is, the solution curve intersects y = (x + 2) exactly at one point (A) −13, 14 (B) −13, −12
and not at two points. Therefore, option (A) is correct and option (C) −13, 12 (D) 12, −13
(B) is incorrect. q
d dx
dq ò0 1- cosq cos x
Checking for option (C), we have 7. The function f(θ) = satisfies the differential
3e
= e( x + 2 ) equation
( x + 2)2
df df
(A) + 2f (q )cot q = 0 (B) - 2f (q )cot q = 0
dq dq
which meets at two points for x < 0 and for x > 0, there is no
df df
intersection point (Fig. 25.6). (C) + 2f (q ) = 0 (D) - 2f (q ) = 0
Hence, option (C) is incorrect. dq dq
Checking for option (D), we have 8. If f(x), g(x) be twice differentiable function on [0, 2] satisfying
( x + 3)
2
( x + 2 ) +1+ 2( x + 2 )
2
2+
1
+( x + 2 )
f ′′(x) = g′′(x), f ′(1) = 4 and g′(1) = 6, f(2) = 3, g(2) = 9, then f(x)
3e
= e ( x +2) = e ( x +2) =e ( x +2) −g(x) at x = 4 equals
( x + 3)2
(A) 0 (B) −10 (C) 8 (D) 2
Therefore, there is no intersection point for x > 0.
9. If the general solutions of a differential equation are (y + c)2 = cx,
Hence, option (D) is correct. where c is an arbitrary constant, then the order and degree of
Hence, the correct answers are options (A) and (D). differential equation is
(A) 1, 2 (B) 2, 1 (C) 1, 3 (D) None of these
Practice Exercise 1
10. Solution of (x2 sin3 y– y2 cos x) dx + (x3 cos y sin2 y – 2y sin x)
1. The differential equation of all non-vertical lines in a plane is dy = 0 is
d2 y d2 x
(A) = 0 (B) =0 x 3 sin3 y
dx 2 dy 2 (A) =c (B) x3 sin3 y = y2 sin x + c
3
dy dx
(C) =0 (D) dy = 0 (C)
x 3 sin3 y
= y 2 sin x + c (D) None of these
dx 3
2. The differential equation of all non-horizontal lines in a plane
is xdy æ y ö
11. Solution of =ç 2 - 1÷ dx is
d2 y d2 x x +y22
èx +y
2
ø
(A) 2
=0 (B) =0
dx dy 2 y y
(A) x - tan-1 (B) tan-1 =c
dy dx x x
(C) =0 (D) dy = 0
dx y
(C) x tan-1 =c (D) None of these
2 x
æ dy ö dy
3. A solution of the differential equation ç ÷ - x + y = 0 is
è dx ø dx
12. Solution of
dy
+ 2 xy = y is
(A) y = 2 (B) y = 2x dx
(C) y = 2x − 4 (D) y = 2x2 − 4 (A) y = ce x - x
2 2
(B) y = ce x - x
4. The differential equation representing the family of curves
(D) y = ce - x
2
(C) y = ce x
y2 = 2c ( x + c ), where c is a positive parameter, is of
13. Solution of the differential equation
(A) Order 1 (B) Order 2
dy
(C) Degree 2 (D) Degree 1 = sin( x + y ) + cos( x + y ) is
dx
d3y d2 y
5. The solution of 3
- 8 2 = 0 satisfying y(0) = 1/8, y1(0) = 0 (x + y)
dx dx (A) log 1+ tan = y +c
and y2(0) = 1 is 2
1 æ e8 x 7ö 1 æ e8 x 7ö (x + y)
(A) y = ç - x + ÷÷ (B) y = ç + x + ÷÷ (B) log 2 + sec = x +c
8 çè 8 8ø 8 çè 8 8ø 2
d2 y dy (C) y 2 = 2 xy
dy
+1
(C) x 2 +x -y =0 (D) None of these (D) None of these
dx 2
dx dx
15. If y = sin (asin–1 x), then 22. The general solution of the differential equation
d2 y dy x(1+ y2) dx + y(1 + x2) dy = 0 is
(A) (1− x 2 ) 2 − x + a2 y = 0
dx dx (A) (1+ x2)(1 + y2) = 0 (B) (1+ x2)(1 + y2) = c
2 (C) (1 + y4) = c(1+ x2) (D) None of these
d y dy
(B) (1− x 2 ) 2
+x − a2 y = 0
dx dx dy 1- y 2
23. Solution of + = 0 is
d2 y dy dx 1- x 2
(C) (1− x 2 ) −x − a2 y = 0
dx 2 dx (A) sin−1x − sin−1y = c (B) sin−1y + sin−1x = c
(D) None of these (C) sin−1x = c sin−1 y (D) (sin−1x) (sin−1y) = c
(A) ln 5 (B) log5 e 27. The equation of the curve through the origin satisfying the
(C) 2 ln 5 (D) 2log5 e equation dy = (sec x + y tanx)dx is
(A) y sinx = x (B) y cosx = x
⎛ y⎞ (C) y tanx = x
f⎜ ⎟ (D) None of these
dy y ⎝ x⎠
19. The solution of the differential equation = + is dy y
dx x ⎛ y⎞ 28. The solution of x 2 - xy = 1+ cos is
f′⎜ ⎟ dx x
⎝ x⎠
y 1 y 1
æyö æyö (A) tan =c- 2 (B) tan = c +
(A) f ç ÷ = kx (B) xf ç ÷ = k 2x 2x x x
èxø èxø æ ö
y c y
(C) cos ç ÷ = 1+ (D) x = (c + x )tan
2 2
æyö æyö èxø x x
(C) f ç ÷ = ky (D) yf ç ÷ = k
èxø èxø
dy
29. If = e−2y and y = 0 when x = 5, then value of x where y = 3
æ dy ö dx
20. Solution of the differential equation ln ç ÷ = ax + by is is given by
è dx ø
e6 + 9
1 1 1 -by 1 ax (A) e5 (B)
(A) - e -by = e ax + c (B) e = e +c 2
b a b a (C) e6 + 1 (D) loge 6
1 -by 1 1 1 30. The equation of this curve passing through (1, 3) and having
(C) e = - e ax + c (D) - e -by = - e ax + c slope −[1+(y/x)] at (x, y) is
b a b a
(A) xy + 4x2 = 7 (B) 2xy + x2 = 7
21. The differential equation of the family of curves cy2 = 2x + c, (C) 3xy + 2x2 = 7 (D) x y + 4 x2 = 7
where c is an arbitrary constant is
dy
dy æ dy ö
2
d2 y 31. Given that = yex such that x = 0, y = e. The value of y (y > 0)
(A) y =1 (B) ç ÷ + y 2 = 0 dx
dx è dx ø dx when x = 1 will be
Chapter 25 | Differential Equations 1145
1 dy
(A) e (B) (C) ee (D) e2 41. The solution of the differential equation a+ x + xy = 0 is
e dx
32. The degree and order of the differential equation of all 2 / 3( 2 a - x ) x +a -2 / 3( a - x ) x +a
tangent lines to the parabola x2 = 4y is (A) y = Ae (B) y = Ae
2 / 3( 2 a + x ) x +a -2 / 3( 2 a - x ) x +a
(A) 2, 1 (B) 2, 2 (C) y = Ae (D) y = Ae
(C) 1, 3 (D) 1, 4 (where A is an arbitrary constant)
33. If f(x) = f’(x) and f(1) = 2, then f(3) = æyö æ y ö
42. Solution of the equation x sin ç ÷ dy = ç y sin - x ÷ dx is
(A) e2 (B) 2e2 è ø
x è x ø
(C) 3e 2 (D) 2e3
æyö
34. The equation of the curve passing through (1, 1) which (A) sin ç ÷ = log k x (B) y = x log k x
èxø
dy
satisfies the differential equation = e x − y + x 2e − y is
dx æyö æyö
(C) cos ç ÷ = log k x (D) tan ç ÷ = log k x
x 1 x 1 èxø èxø
(A) e y = e x + + (B) e y = e x + −
2 2 3 2 43. The general solution of y2dx + (x2 – xy + y2 ) dy = 0 is
x3
1 x3
1 æxö
(C) e y = e x + − (D) e y = e x − + (A) tan-1 ç ÷ + log y + c = 0
3 3 3 3 èyø
dy p æxö
35. Solution of 2 y sin x = 2 sin x cos x - y 2 cos x , x = , y = 1 is (B) 2 tan-1 ç ÷ + log x + c = 0
dx 2 èyø
given by
(A) y2 = sinx (B) y = sin2x (C) log( y + x 2 + y 2 ) + log y + c = 0
(C) y2 = cosx + 1 (D) None of these
æxö
dy (D) sinh-1 ç ÷ + log y + c = 0
36. The general solution of the equation = 1+ xy is èyø
dx
x2 x2 dy 1
-
44. The solution of the equation = is
(A) y = ce 2 (B) y = ce 2 dx x + y + 1
x2
− (A) x = ce y - y - 2 (B) y = x + ce y - 2
(C) y = ( x + c )e 2 (D) None of these
(C) x + ce y - y - 2 = 0 (D) None of these
dy
37. Solution of = e y + x + e y - x is 45. If integrating factor of x (1- x )dy + (2 x y - y - ax )dx = 0 is
2 2 3
dx
e∫Pdx, then P is equal to
(A) ex(x + 1) = y (B) ex(x + 1) + 1 = y
(C) ex(x − 1) + 1 = y 2 x 2 - ax 3
(D) None of these (A) (B) (2 x 2 - 1)
x (1- x 2 )
38. The differential equation of the family of parabolas with focus
2x2 -1 (2 x 2 - 1)
at the origin and the x-axis as axis is (C) (D)
ax 3 x (1- x 2 )
2 2
æ dy ö dy æ dy ö dy
(A) y ç ÷ + 4 x = 4y (B) - y ç ÷ = 2 x -y 46. The equation of the curve passing through the point
è ø
dx dx è ø
dx dx (1, p/4) and tangent at any point of which makes an angle
2 2
æ dy ö dy æ dy ö dy ⎛y y⎞
(C) y ç ÷ + y = 2 xy (D) y ç ÷ + 2 xy +y =0 tan−1 ⎜ − cos2 ⎟ is
è dx ø dx è dx ø dx ⎝x x⎠
é æ e öù ⎛ y⎞
39. The differential equation of the family of curves for which the (A) y = tan-1 êlog ç ÷ ú (B) y = xtan−1 ⎜ ⎟ + 1
ë è x øû ⎝ x⎠
length of the normal is equal to a constant k is given by
2 (C) y = xtan−1(1− log x ) (D) y − x = tan−1(1− log x )
dy æ dy ö
(B) ç y ÷ = k - y
2 2
(A) y 2
= k2 - y2
dx è dx ø 47. The equation of family of curves for which the length of the
2 2 normal is equal to the radius vector is
æ dy ö æ dy ö
(C) y ç ÷ = k + y (D) ç y ÷ = k + y
2 2 2 2
(A) y2 ± x2 = k (B) y ± x = k
è dx ø è dx ø
(C) y2 = kx (D) None of these
dy æ dy ö 48. A continuously differentiable function f ( x ) in (0 , p ) satisfying
40. The solution of the differential equation y - x = a ç y 2 + ÷ is
dx è dx ø y ′ = 1+ y 2 , y (0 ) = 0 = y (p ) is
(A) y = c(x + a)(1 + ay) (B) y = c(x + a)(1 – ay) (A) tan x (B) x(x – π)
(C) y = c(x – a)(1 + ay) (D) None of these (C) (x – π)(1 – e ) x (D) Not possible
1146 Mathematics Problem Book for JEE
d2 y dy
49. The rate of increase of bacteria in a certain culture is 57. The differential equation 2
+x + sin y + x 2 = 0 is of the
dx dx
proportional to the number present. If it doubles in 5 h, then following type
in 25 h its number would be (A) Linear (B) Homogeneous
(A) 8 times the original (B) 16 times the original
(C) Order two (D) Degree one
(C) 32 times the original (D) 64 times the original
3
æ d 2 y ö æ dy ö4
d2 y
ç dx 2 ÷÷ + çè dx ÷ø + y = x
58. The differential equation x ç 2 is of
50. The solution of = cos x - sin x is
dx 2 è ø
(A) y = - cos x + sin x + c1x + c2 (A) Degree 3 and order 2 (B) Degree 1 and order 1
(C) Degree 4 and order 3 (D) Degree 4 and order 4
(B) y = - cos x - sin x + c1x + c2
59. The order and degree of the differential equation
(C) y = cos x - sin x + c1x 2 + c2 x 3/ 2
⎡ ⎛ dy ⎞ 2 ⎤ d2 y
(D) y = cos x + sin x + c1x + c2 x
2
⎢x + ⎜ ⎟ ⎥ =a are , respectively
⎢⎣ ⎝ dx ⎠ ⎥⎦ dx 2
51. The solution of the differential equation (A) 2, 2 (B) 2, 3
dy
x4 + x 3 y + cosec( xy ) = 0 is equal to (C) 2, 1 (D) 2, 4
dx
60. The elimination of the arbitrary constants A, B and C from
(A) 2 cos( xy ) + x -2 = c (B) 2 cos( xy ) + y -2 = c y = A + Bx + Ce - x leads to the differential equation
(C) 2 sin( xy ) + x -2 = c (D) 2 sin( xy ) + y -2 = c (A) y ¢¢¢ - y ¢ = 0 (B) y ¢¢¢ - y ¢¢ + y ¢ = 0
x 2d 2 y (C) y ¢¢¢ + y ¢¢ = 0 (D) y ¢¢ + y ¢¢ - y ¢ = 0
52. The solution of the equation = ln x , when x = 1, y = 0
dy dx 2 61. The differential equation obtained on eliminating A and B
and = -1 is from the equation y = A cos wt + B sinwt is
dx
(A) y ¢¢ = -w y (B) y ¢¢ + y = 0
2
1 1
(A) (ln x )2 + ln x (B) (ln x )2 - ln x
(C) y ¢¢ + y ¢ = 0 (D) y ¢¢ - w y = 0
2
2 2
1 1
(D) - (ln x ) - ln x
2
(C) - (ln x )2 + ln x 62. If the substitution x = tan z is used, then find the transformed
2 2
d2 y dy
2 form of the equation (1+ x 2 )2 + 2 x (1+ x 2 ) + y = 0
d y dx 2 dx
53. The solution of the differential equation x 2 = 1 , given that
dy dx
y = 1, = 0 when x = 1, is d2 y d2 y dy
dx (A) + 2y = 0 (B) +2 - y =0
dz 2 dz 2
dz
(A) y = x log x + x + 2 (B) y = x log x - x + 2
d2 y dy d2 y
(C) y = x log x + x (D) y = x log x - x (C) 2
-2 + y = 0 (D) +y =0
dz dz dz 2
d2 y 1
54. The solution of the differential equation = - 2 is dy x log x 2 + x
dx 2
x 63. Solution of = is
dx sin y + y cos y
(A) y = log x + c1x + c2 (B) y = - log x + c1x + c2
(B) y sin y = x + c
2
(A) y sin y = x 2 log x + c
1
(C) y = - + c1x + c2 (D) None of these
(C) y sin y = x + log x + c (D) y sin y = x log x + c
2
x
d2 y
55. The solution of the differential equation cos2 x = 1 is æ 2 + sin x ö dy æp ö
dx 2 64. If ç ÷ = - cos x , y (0 ) = 1, then y ç ÷ =
è 1 + y ø dx è2ø
(A) y = logcos x + cx (B) y = logsec x + c1x + c2
1 1 1
(C) y = logsec x - c1x + c2 (D) None of these (A) 1 (B)
2
(C)
3
(D)
4
d2 y 65. The solution of the differential equation
56. The solution of = sec2 x + xe x is
dx 2 (3 xy + y 2 )dx + ( x 2 + xy )dy = 0 is
(A) y = log(sec x ) + ( x - 2)e x + c1x + c2 (A) x 2 (2 xy + y 2 ) = c 2 (B) x 2 (2 xy - y 2 ) = c 2
(B) y = log(sec x ) + ( x + 2)e + c1x + c2
x
(C) x 2 ( y 2 - 2 xy ) = c 2 (D) None of these
(C) y = log(sec x ) - ( x + 2)e + c1x + c2
x
dy y æ y ö
66. The solution of the equation = ç log + 1÷ is
(D) None of these dx x è x ø
Chapter 25 | Differential Equations 1147
1
æyö y (A) (B) 1 (C) 2 (D) 4
(A) log ç ÷ = cx (B) = log y + c 2
èxø x 4. The solution of differential equation
(C) y = log y + 1 (D) y = xy + c 2x3y dy + (1 − y2) (x2y2 + y2 − 1)dx = 0 is
(A) x2y2 = (cx + 1)(1 − y2) (B) x2y2 = (cx + 1)(1 + y2)
dy
67. Solution of x + y − x + xy cot x = 0 ( x ≠ 0 ) is (C) x2y2 = (cx − 1)(1 − y2) (D) None of these
dx
c
(A) xy = + 1− x cot x (B) xy = c sin x + x cot x Comprehension Type Questions
sin x
(C) xy sin x = c - cot x (D) y sin x = cx - x cot x Paragraph for Questions 5−7: If f: R → [0, ∞) be a function
satisfying the property f(x + y) − f(x − y) = f(x)[f(y) − f(−y)] for all real
68. The solution of ye - x / y dx - ( xe - x / y + y 3 )dy = 0 is x and y, f ′(0) = log a, f(0) = 1, then
y2 x2 5. f(x) is
(A) + e- x / y = k (B) + e- x / y = k
2 2 (A) ex (B) 2 lnx (C) 4x (D) ax
x 2
y2 6. f ′(x) is
(C) + ex/y = k (D) + ex/y = k
2 2 (A) ex (B) axlog a (C) 4 (D) 5x
69. The solution of the differential equation 7. The solution of differential equation
dy dy (loga f ( x ) + loga f ( y ))2
x + y = x 2 + 3 x + 2 is = is
dx dx [loga f ( x ) + 2][loga f ( y ) - 2]
x3 3 2 x4 y -2 æ y -2 ö
(A) xy = + x + 2x + c (B) xy = + x3 + x2 + c (A) - ln 1+2 ç ÷ = 2 log ( x + 2 ) + c
3 2 4 x +2 è x +2ø
x4 x3 x4 y +2
(C) xy = + + x2 + c (D) xy = + x 3 + x 2 + cx æ y +2ö
4 3 4 (B) - ln 1+ ç ÷ =c
x -2 è x -2 ø
70. If y is a function of x and y (1) = 0, then the solution of the æ x +2ö
dy y (C) (x + 2)(y − 2) = ln ç ÷+c
equation x − = x is è y -2 ø
dx x + 1 (D) None of these
x 1
(A) y = ( x + log x ) (B) y = ( x - 1+ log x ) Paragraph for Questions 8−10: A tangent drawn to the
x +1 x +1 curve y = f(x) at P(x, y) cuts the x-axis and the y-axis at A and B,
x +1 x respectively. If BP:AP = 3:1 and f(1) = 1, then
(C) y = ( x - 1+ log x ) (D) y = ( x - 1 + log x )
x x +1
8. The differential equation of the curve is
dy dy
(A) x - 3y = 0 (B) x + 3y = 0
Practice Exercise 2 dx dx
dy
(C) 3x +y =0 (D) None of these
Single/Multiple Correct Choice Type Questions dx
9. The curve passes through the point
1. The differential equation of the system of circles touching the
æ 1ö æ1 ö
(A) æç , 2 ö÷ (B) æç 2, ö÷
1 1
x-axis at origin is (C) ç 8 , ÷ (D) ç , 8 ÷
è 8 ø è 8 ø è 2 ø è2 ø
dy dy
(A) (x2 − y2) - 2 xy = 0 (B) (x2 − y2) + 2 xy = 0 10. Normal to the curve at (1, 1) is
dx dx
(A) x + 3y = 4 (B) 3x + y = 4
dy
(C) (x2 + y2) - 2 xy = 0 (D) (x2 + y2)
dy
+ 2 xy = 0 (C) x − 3y = 2 (D) −x + 3y = 2
dx dx
Paragraph for Questions 11−13: Consider a family of curves,
2. If the general solution for the differential equation where the ordinate is proportional to the cube of the abscissa and
dy y æxö x æxö
= + f ç ÷ is y = , then function f ç ÷ is let A be a fixed point, which has coordinates (a, b).
dx x èyø ln ( cx ) èyø 11. If tangents be drawn through A to the members of the family
x2 y2 y2 x2 of curves, then the locus of the points of contact is
(A) − 2 (B) − (C) (D) 2 (A) xy + bx − 3ay = 0 (B) xy − 4bx + 3ay = 0
y x2 x2 y
(C) 2xy + bx − 3ay = 0 (D) 2xy − 4bx + 3ay + 2 = 0
3. Let a function f(x) be such that f ″(x) = f ′(x) + ex and f(0) = 0 and
12. If normals be drawn through A to the members of the family
ì ( f ( 2 ) )2 ü
ï ï of curves, then the feet of these normals on the curves also
f ′(0) = 1, then ln í ý is
ïî 4 ïþ lie on the curve
1148 Mathematics Problem Book for JEE
(A) xy + bx − 3ay = 0 (B) xy − 4bx + 3ay = 0 17. The isogonal trajectories of a family of straight lines, y = cx
(C) x2 − 3y2 = ax − 3by (D) x2 + 3y2 = ax + 3by that cut the lines of the given family at an angle α, whose
tangent is k, have differential equation
13. If the tangent through A to a curve cuts the curve again at a
dy y dy y - kx
point B, then the locus of B is (A) = (B) =
(A) xy − 4bx + 3ay = 0 (B) 2xy + bx − 3ay = 0 dx x dx y + kx
(C) x2 − 3y2 = ax − 3by (D) a2x2 + b2y2 = 1 dy kx + y dy y
(C) = (D) =k
Paragraph for Questions 14−16: A curve passing through dx x - ky dx x
origin is such that slope of tangent at any point is reciprocal of 18. The isogonal trajectories of a family of parabolas, y2 = 4ax
sum of coordinate of point of tangents. that intersect the members of the family at an angle p/4,
have differential equation
14. Slope of tangent at ordinate ln 3 is
1 dy x - y dy 2 x + y
(A) 1 (B)
1
(C) (D) −2 (A) = (B) dx = 2 x - y
3 2 dx x + y
15. Area bounded by curve and the abscissa y = 0 and y = 1 is dy x - 2 y dy x 2 - y 2
(C) = (D) =
1 3 5 dx x + 2 y dx x 2 + y 2
(A) e − (B) e − (C) e − (D) e + 1
2 2 2 19. Let f(x, y) = 0 represents a family of circles touching the
[ sin a + cos a ]
axis of y at origin. The differential equation of the family of
16. If I = ò xe - y d (e y ) ; (where [.] denotes the greatest curves intersecting the above family orthogonally is
-1 dy y 2 - x 2
integer function), then I is (A) dy = 2 xy (B) =
dx x 2 - y 2 dx y 2 + x 2
1 1 1
(A) e - - (B) e - e - 2
e 3 dy y 2 - x 2
(C) = (D) Either (A) or (C)
1 1 1 dx 2 xy
(C) e - (D) e + +
e e 3 Paragraph for Questions 20−22: Consider a polynomial f(x),
Paragraph for Questions 17−19: Isogonal Trajectories: Suppose which satisfies the following conditions:
we have a one-parameter family of curves φ(x, y, c) = 0. Lines
(i) f ( x ) = {f ′( x )} ∀x
2
intersecting all the curves of the given family at a constant angle
are called isogonal trajectories. If this angle is a right angle, they 1
19
are orthogonal trajectories. Let the trajectories cut the curve of a (ii) ò f ( x )dx = 12
given family at an angle α, where tan α = k. 0
Answer Key
Practice Exercise 1
1. (A) 2. (B) 3. (C) 4. (A) 5. (A) 6. (B) 7. (A) 8. (B) 9. (A) 10. (C) 11. (D) 12. (A)
13. (D) 14. (A) 15. (A) 16. (B) 17. (D) 18. (C) 19. (A) 20. (A) 21. (C) 22. (B) 23. (B) 24. (C)
25. (C) 26. (A) 27. (B) 28. (A) 29. (B) 30. (B) 31. (C) 32. (A) 33. (B) 34. (C) 35. (A) 36. (D)
Chapter 25 | Differential Equations 1149
37. (D) 38. (B) 39. (B) 40. (B) 41. (A) 42. (C) 43. (A) 44. (A) 45. (D) 46. (C) 47. (A) 48. (D)
49. (C) 50. (A) 51. (A) 52. (D) 53. (B) 54. (A) 55. (B, C) 56. (A) 57. (C, D) 58. (A) 59. (A) 60. (C)
61. (A) 62. (D) 63. (A) 64. (C) 65. (A) 66. (A) 67. (A) 68. (A) 69. (A) 70. (D)
Practice Exercise 2
1. (B) 2. (B) 3. (D) 4. (C) 5. (D) 6. (B) 7. (A) 8. (B) 9. (B) 10. (D) 11. (C) 12. (D) 13. (A)
14. (C) 15. (B) 16. (B) 17. (C) 18. (B) 19. (A) 20. (B) 21. (D) 22. (C)
Solutions
Practice Exercise 1 Therefore,
log y2 = 8x or y2 = e8x
1. The general equation of all non-vertical lines in a plane is
y = mx + c Integrating both sides, we get
dy e8 x
⇒ =m [differentiating w.r.t. x]
dx y1 = + c2
8
d2 y Again putting x = 0, we have
⇒ =0 [differentiating w.r.t. x]
dx 2
c2 = −1/8
2. The general equation of all non-horizontal lines in xy-plane is
x = my + c So,
dx 1 8x 1 æ e8 x ö
⇒ =m [differentiating w.r.t. y] y1 = (e - 1) Þ y = çç - x ÷÷ + c3
dy 8 8è 8 ø
d2 x Putting x = 0. We have
⇒ =0 [differentiating w.r.t. y]
dy 2
1 1 7
3. The given equation can be written as c3 = - =
8 64 64
2
dy ⎛ dy ⎞
y=x −⎜ ⎟ 1 æ e8 x 7ö
dx ⎝ dx ⎠ Thus, y = çç - x + ÷÷ .
dy 8è 8 8ø
= p , then y = px − p .
2
If
dx dy d2 y d3y -x
= 4 e 4 x - 2e - x ⇒ 2 = 16e 4 x + 2e - x ⇒ 3 = 64 e - 2e
4x
On differentiating w.r.t. x, we get 6.
dx dx dx
dp dp dp dp
p= p+ − 2p ⇒ ( x − 2p) ⇒ =0 d3y dy
dx dx dx dx Putting these values in + A + By = 0 , we have
dx 3 dx
On integrating w.r.t. x, we get p = c
dy (64 + 4A + B) e4x + (−2 − 2A + 2B)e−x = 0
= c ; y = cx − c 2
dx Solving, we get A = −13 and B = −12.
If c = 2, then y = 2x – 4. 7. We have
4. We have q
d dx 1
f(θ) = ò = = cosec2q
y2 = 2c ( x + c ) (1) dq 0 1- cosq cos x 1 - cos 2
q
⇒ 2y y1 = 2c ⇒yy1 = c (2) df (q )
Therefore, = −2cosec2θ cotθ.
Eliminating c from Eqs. (1) and (2), we get dq
y 2 = 2 yy1( x + yy1 ) ⇒ y − 2 xy1 = 2 y y13 / 2 ⇒ ( y − 2 xy1)2 = 4 yy13 8. We have
f ′′(x) = g′′(x)
Clearly, it is a differential equation of order 1 and degree 3. On integrating, we get
5. We have f ′(x) = g′(x)+c
y3 Putting x = 1, we get
= 8 ⇒ln y2 = 8x + c1
y2 f ′(1) = g′(1) + c ⇒ c = −2
Putting x = 0, we have ⇒ f ′(x) = g′(x) − 2 ⇒ f(x) = g(x) − 2x + c1
⇒ f(2) = g(2) − 4 + c1 ⇒ c1 = −2
c1 = log y2(0) = log 1 = 0
1150 Mathematics Problem Book for JEE
Thus, we have
I.F. = e ò
( 2 x -1)dx
= ex -x
2
f(x) = g(x) − 2x − 2
Therefore, the solution of differential equation is
⇒ f(4) − g(4) = −10 2
-x
ye x =c
9. Hint: There will only one constant in the first-order differential
Þ y = ce x - x
2
equation.
Differentiating the given equation 13. Hint: Put x + y = v
(y + c)2 = cx (1) Putting x + y = v. Then
dy dv dy dv
dy dy dy æ dy ö 1+ = Þ = -1
Þ 2(y + c) × = c Þ 2y × = c - 2c × = c ç 1- 2 × ÷ dx dx dx dx
dx dx dx è dx ø
dv v v v
dy ⇒ = sin v + (cos v + 1) = 2 sin cos + 2 cos2
2y × dx 2 2 2
Þ c= dx
v⎛ v v⎞
1- 2 ×
dy = 2 cos ⎜ sin + cos ⎟
dx 2⎝ 2 2⎠
dv
Putting the value of c in Eq. (1) and simplifying we will get
a first-order and second-degree equation. Hence, (A) is the
Þ ò væ v vö ò
= dx + c
2 cos ç sin + cos ÷
correct answer. 2è 2 2ø
I.F. = e ò
dy - tan xdx
Þ = e ax × e by = cos x
dx
y cos x = ∫ sec x cos xdx = x + c
- by
Þ ò e dy = ò e dx + c
ax (c is a constant) ⇒ y cos x = x + c
At (0, 0) c = 0,
e -by e ax y cos x = x
= +c
-b a
dy 1 1 1 y
21. Differentiating the given differential equation 28. - y = 2 + 2 cos (1)
dx x x x x
cy2= 2x + c (1)
dy Put y = vx. Then
c ×2y × = 2 dy dv
dx =v+ x
1 dx dx dx
Þ c= ×
y dy Therefore, Eq. (1) becomes
dv 1 1
Putting this value of c in Eq. (1), we get v+x - v = 2 + 2 cos v
dx x x
1 dx 2 1 dx 3 dv
× × y = 2x + × Þ x = 1+ cos v
y dy y dy dx
1152 Mathematics Problem Book for JEE
dy y dy y
30. = -1- Þ + = -1 1
dx x dx x ⇒ ey = ex + x3 + c
3
Put z = y/x. Therefore,
dz The curve passes through (1, 1) implies
z+x + z = −1
dx 1
e=e+ +c
dz 3
⇒x = −(2 z + 1)
dx
1
⇒c = −
dz dx 3
⇒ + =0
2z + 1 x
Therefore, the equation of the curve is
Integrating, we get
dz dx x3 1
ey = ex + −
∫ 2z + 1 + ∫ x = c 3 3
1 35. On dividing by sin x,
⇒ log 2 z + 1 + log x = c ⇒ log 2 z + 1 + log( x 2 ) = c
2 dy
2y + y 2 cot x = 2 cos x
2 k ⎛ y⎞ k dx
⇒ 2 z + 1 = e c −log( x ) = 2 ⇒ 2 ⎜ ⎟ + 1 = 2
x ⎝ x ⎠ x Put y2 = v, we get
As the curve passes through (1, 3), so k = 7. Therefore, the
dv
equation of the curve is + v cot x = 2 cos x
dx
2 xy + x 2 = 7
I.F. = e∫cotxdx = elog sinx = sin x
dy Therefore, the solution is
31. = ex dx ⇒ ln y = ex + c
y v sin x = ∫sin x(2 cos x) dx + c
At x = 0, y = e. So, c = 0. ⇒ y2 sin x = sin2x + c
ln y = ex p
Therefore, at x = 1, y = ee. When x = , y = 1, then c = 0. Therefore,
2
1 y2 = sin x
32. Equation of any tangent to x2 = 4y is x = my + , where m is
m
an arbitrary constant. So, dy
36. - xy = 1 ⇒ P = −x, Q = 1
dy 1 dx
1= m Þm= x2
dx dy -
- ò xdx
dx I.F. = e =e 2
1
Therefore, putting this value of m in x = my + , we get - x2
-
x2
m Therefore, the solution is y × e 2 = òe 2 ×1× dx + c .
2
y dy æ dy ö dy
x= + Þç ÷ -x +y =0 dy
dy dx è dx ø dx 37. = e y × e x + e y × e - x = e y (e x + e - x )
dx dx
which is a differential equation of order 1 and degree 2. ⇒ e−ydy = (ex + e−x)dx ⇒ − e−y = ex − e−x + c
f ’( x )
33. = 1⇒ log f(x) = x + c 38. Equation of family of parabolas with focus at (0, 0) and the
f (x) x-axis as axis is y2 = 4a(x + a). (1)
Chapter 25 | Differential Equations 1153
2 é æ dy ö2 ù 2 dx x 2 - xy + y 2
æ dy ö + =0
y 1+ ç ÷ = k ⇒ y ê1+ ç ÷ ú = k
2
43.
è dx ø êë è dx ø úû dy y2
2
2 æ dy ö æ dy ö
2 2 dx æ x ö æ x ö
⇒ y + y ç ÷ = k ⇒ y2 ç ÷ = k2 - y2
2 2 ⇒ + ç ÷ - ç ÷ + 1= 0
è dx ø dy è y ø è y ø
è dx ø
Put v = x / y . Then
dy æ dy ö dy
= a ç y 2 + ÷ ⇒ y - ay = ( x + a)
2
40. y-x
dx è dx ø dx dx dv
x = vy ⇒ =v + y
dy dy
dy dx
⇒ =
y (1- ay ) x + a dv dv dy
v+y + v 2 - v + 1= 0 ⇒ + =0
On integrating both sides, we get dy v +1 y
2
I.F. = e ò
-x -1dy
dy = e- y
ò y ò x +a
= dx
Hence, the solution of the equation is
x +a-a a
log y = - ò dx = - ò x + a dx + ò dx x .e − y = ∫ ( y + 1)e − y dy + c ⇒ x = ce y - y - 2
x +a x +a
2
⇒ log y = - ( x + a)3 / 2 + 2a x + a + log A 45. x (1- x 2 )dy + (2 x 2 y - y - ax 3 )dx = 0
3
é æ 2 öù dy (2 x 2 − 1) ax 2
ê( x + a ç - 3 ( x + a ) + 2 a ÷ ú ⇒ + y=
-2 / 3( x + a )3/ 2 + 2 a x + a ë è øû
y = Ae = Ae dx x (1− x )2
(1− x 2 )
é æ -2 x - 2 a + 6 a ö ù 2x2 −1
ê x +a ç ÷ú
Therefore, P=
è øû [ -2 / 3 x + a ( x - 2 a )]
= Ae ë 3
= Ae x (1− x 2 )
x + a ( 2 a − x )]
⇒ y = Ae[2 / 3
46. We have
42. We have dy ⎛ ⎛y y⎞⎞
æyö æ y ö = tan ⎜ tan−1 ⎜ − cos2 ⎟ ⎟
x sin ç ÷ dy = ç y sin - x ÷ dx dx ⎝ ⎝x x⎠⎠
èxø è x ø y y
dy y sin( y / x ) − x = − cos2
= x x
dx x sin( y / 2) Put y = vx. Then
Put y = vx. Then dv dx
v+x = v − cos2 v ⇒ (sec2 v ) dv + =0
dv v sin v − 1 dx x
v+x =
dx sin v Integrating, we get
1154 Mathematics Problem Book for JEE
dx P log 2
∫ (sec v ) dv + ∫ =c = ´ 25 = 5 log 2 = log 32
2
log
x P0 5
æyö Hence, P = 32P0.
Þ tan v + log x = c Þ tan ç ÷ + log x = c
èxø
d2 y
The curve passes through (1, p / 4), so c = 1 Therefore, 50. = cos x - sin x
dx 2
y y On integrating both sides, we get
tan = 1− log x ⇒ = tan−1(1− log x )
x x dy
= sin x + cos x + c1
dx
2
æ dy ö
47. Length of the normal = y 1+ ç ÷ Integrating again, we get
è dx ø
y = - cos x + sin x + c1x + c2
2
æ dy ö dy
It is given that y 1+ ç ÷ = x + y .
2 2
51. x4 + x 3 y + cosec( xy ) = 0
è dx ø dx
⇒ x ( x dy + y dx ) + cosec ( xy ) dx = 0
2 2 3
æ dy ö æ dy ö
y2 + y2 ç ÷ = x2 + y2 ⇒ y2 ç ÷ = x2
è dx ø è dx ø ⇒ x d ( xy ) + cosec ( xy )dx = 0
3
⇒ ydy ± xdx = 0 ⇒ y 2 ± x 2 = k d ( xy ) dx
⇒ + =0
cosec ( xy ) x 3
dy dy On integrating both sides, we get
48. = 1+ y 2 ⇒ = dx
dx 1+ y 2 d ( xy ) dx
On integrating both sides ∫ cosec ( xy ) ∫ x 3
+ =0
dy -1
∫ 1+ y 2 = ∫ dx ⇒ tan y = x +c
∫ sin( xy ) d ( xy ) + ∫ x −3dx = 0
At x = 0, y = 0, then c = 0. æ x -2 ö -2
At x = π, y = 0, then tan–1 0 = p + c ⇒ c = –π - cos( xy ) + çç ÷÷ = c Þ 2 cos(xy) + x = c
Therefore, è -2 ø
tan–1 y = x ⇒ y = tan x = φ(x)
d 2 y ln x dy -(ln x + 1)
Therefore, the solution is y = tan x. 52. = Þ = +c
dx 2 x 2 dx x
But tan x is not a continuous function in (0, π).
Hence, φ(x) is not possible in (0, π). dy
At = -1, x = 1, y = 0, therefore c = 0. So,
dx
49. Let P0 be the initial population and let the population after
t years be P. Then ln x + 1 1
y = -ò dx = - (ln x )2 - ln x
dP dP x 2
= kP Þ = k dt
dt P d2 y d 2 y 1 dy
53. x = 1 Þ = Þ = log x + c1
On integrating, we have logP = kt + c. dx 2 dx 2 x dx
At t = 0, P = P0
⇒ y = x log x - x + c1x + c2 (on integrating twice)
Therefore, logP0 = 0 + c . So, dy
Given y = 1 and = 0 at x = 1. So, c1 = 0 and c2 = 2.
P dx
log P = kt + log P0 ⇒ log = kt
P0 Therefore, the required solution is y = x log x − x + 2.
When t = 5 h, P = 2P0. Therefore, d2 y 1
54.
2
=- 2
2P0 log2 dx x
log = 5k ⇒ k = Now on integrating both sides, we get
P0 5
dy 1
Therefore, = + c1 ⇒ y = log x + c1x + c2
dx x
P log 2
log = t
P0 5 d2 y d2 y
55. cos2 x = 1 ⇒ = sec2 x
When t = 25h, we have dx 2 dx 2
Chapter 25 | Differential Equations 1155
The order of highest derivative is 2 and degree is 1. Separating the variables and integrating
y ¢ = - Aw sinw t + Bw cos w t dy dv
61. So now putting y = vx and = v + x , we get
dx dx
⇒ y ¢¢ = - Aw 2 cos w t - Bw 2 sinw t = − w 2 ( A cos w t + B sinw t )
dv 3 x 2v + x 2v 2 dv -2v (v + 2)
v+x =- 2 ⇒x =
Therefore, y ¢¢ = -w y .
2 dx x + x 2v dx v +1
62. Given that x = tan z. Therefore, 1 v +1 é 1 1 ù
⇒ dx = - dv = - ê + ú dv
dy dy dz x 2v (v + 2) ë 2( v + 2 ) 2v ( v + 2 ) û
= ⋅
dx dz dx 2 é 1 1 1 ù
dy 1 ⇒ - dx = ê + - ú dv
= ⋅ x ë v + 2 2v 2(v + 2) û
dz 1+ x 2 On integrating, we get
So,
dy dy 1 1
(1+ x 2 ) = -2 loge x = log(v + 2) + log v + log c
dx dz 2 2
1156 Mathematics Problem Book for JEE
yæy ö 4 2 æ yö dy
⇒ v (v + 2 ) x 4 = c 2 ⇒ ç + 2 ÷ x = c , ç As v = ÷ 69. x + y = x2 + 3x + 2
xèx ø è xø dx
dy y 2
Hence, the required solution is ( y 2 + 2 xy ) x 2 = c 2 . + = x +3+
⇒
dx x x
dy y æ y ö 1 2
66. = ç log + 1÷ Here, P = , Q = x + 3 + , therefore
dx x è x ø x x
Put y = vx. Then 1
I.F. = e ò x
dy
dy dv =x
=v + x×
dx dx
ò d( xy ) = ò ( x + 3 x + 2)dx
2
Therefore,
dv 3 2
v+ x× = v (log v + 1) xy =
x
+
3x
+ 2x + c
dx 3 2
dv 70. Given equation is
⇒v+ x = v log v + v
dx
dv dy y
⇒x = v log v − =1
dx dx x ( x + 1)
dv dx The integrating factor is
⇒ = ⎛1 1 ⎞ ⎛ x +1⎞
x +1
dx
v log v x − ∫ x ( x +1) ∫
− ⎜ −
⎝ x x +1⎟⎠
dx log⎜
⎝ x ⎟⎠
I.F. = e =e =e =
On integrating both sides, x
Therefore, the solution is
dv dx
∫ =∫ ⇒ log log v = log x + log c ⎛ x + 1⎞ x +1
⎝ x ⎠⎟ ∫ x
v log v x y⎜ = dx + c
y e - x / y dx - ( xe - x / y + y 3 )dy = 0
2 2
68. æ æ xdx ö ö æ xdx ö
⇒ x2 + çç y - ç y + ÷ ÷÷ = ç y + ÷
( ydx - xdy ) è è dy øø è dy ø
Þ e - x / y ( ydx - xdy ) = y 3dy ⇒ e - x / y = ydy
y2 2 2
2 æ dx ö 2 æ dx ö 2 xydx
⇒ x2 + x ç ÷ = y + x ç ÷ +
2
⎛ x⎞ è ø
dy è ø
dy dy
⇒ e − x / y d ⎜ ⎟ = ydy
⎝ y⎠
2xydx
x2 = y2 +
On integrating both sides, we get dy
y2 y2 dy
k - e- x / y = ⇒ + e- x / y = k (x2 − y2) − 2 xy = 0
2 2 dx
Chapter 25 | Differential Equations 1157
1 7. (x + 2 = X, y − 2 = Y )
and log c + log x = dx ( x + 2 ) ( y - 2 )
v
dY ( X + Y )
2
⇒ −v2 = x dv =
dx dX XY
æ 1ö Put Y = tX. Then
⇒ f ç ÷ = -v 2
èv ø dt (1+ t )
2
t+X =
æxö y 2 dX t
⇒ fç ÷=- 2
èyø x
Þ dt -
dt
=2
dX
3. f ″(x) − f ′(x) = ex 2t + 1 X
e x f ’’( x ) − e x f ’( x ) y -2 æ 2( y - 2) ö
⇒ =1 Þ - ln 1+ ç ÷ = 2 log( x + 2) + c
e2 x x +2 è x +2 ø
⇒ f(x) = xex + c
M P (x , y )
Since, f(0) = 0, therefore c = 0.
Therefore,
ìï ( f (2) )2 üï
f(x) = xex Þ ln í ý=4 L A (x–y/dy/dx, 0)
ïî 4 ïþ
Figure 25.7
4. 2x3y dy + (1 − y2)(x2y2 + y2 − 1)dx = 0
Equation of the tangent to the curve is
2 y dy y2 1 1 dy
Þ + = Y−y= (X − x), so that the coordinates of A and B are,
(1- y ) dx 1- y 2 x x 3
2 2
dx
y2 æ ö
= u. Then y æ dy ö
Put
1- y 2 respectively, A ç x - ,0 ÷ and B ç 0, y - x ÷ . Also,
è dy / dx ø è dx ø
2 y dy du
= BP MP x
(1- y 2 )2 dx dx =3 Þ =3Þ =3
AP AL - y / ( dy / dx )
du u 1
Þ + =
dx x x 3 dy
⇒x + 3y = 0 (1)
1 dx
u⋅x = ò dx + c Þ x 2 y 2 = (cx - 1)(1- y 2 )
x2 9. From Eq. (1) of Solution 8, we have
5. f(x + y) − f(x − y) = f(x)[f(y) − f(−y)] dy
x3 + 3x2y = 0
f ( h) - f ( 0 ) f ( h) - 1 dx
f ′(0) = lim = lim = log a
h®0 h h®0 h ⇒ x3y = c = 1
f ( x + h) - f ( x - h) f ( x ) [ f ( h ) - f ( - h )] æ 1ö
and the curve passes through ç 2, ÷ .
f ′(x) = lim = lim
h®0 2h h®0 2h è 8ø
10. Putting (1, 1) in Eq. (1) of Solution 8, we have
f ( x ) é f (h) - 1 f ( -h) - 1ù f ( x )
= lim + = 2 log a
h®0 2 êë h -h úû 2 dy
=−3
dx
f ’( x )
= log a Þ log f ( x ) = log a x + c Hence, the equation of the normal to the curve at (1, 1) is
f (x) 1
f(0) = 1 ⇒ c = 0 ⇒ f(x) = ax y − 1 = (x − 1) ⇒ 3y − x = 2
3
1158 Mathematics Problem Book for JEE
dy 3 y y - b 3y v
11. y = lx3 ⇒ = ⇒ = ⇒ dv = dx
dx x x -a x 1+ v
Let point of contact on curve is P ≡ (h, k). ⇒ v − ln(v + 1) = x + c ⇒ y = ln (x + y + 1) + c
Slope of the curve at point P is 3k , which is equal to slope At x = 0, y = 0, therefore, c = 0. So,
k -b h
of line AP = . So, y = ln(x + y + 1)
h-a ⇒ x = ey − y − 1
k - b 3k
= Þ
dy
=
1
h-a h dx 2
ln3
⇒ 2kh − 3ak + bh = 0
1
Therefore, locus of P is 2xy − 3ay + bx = 0. 3
ò (e - y - 1)dy = e -
y
15. Area = 2
12. Let point on the curve is P(h, k). Then slope of normal 0
at P point is [ sina + cos a ]
-
h 16. ∫ xe − y d (e y )
3k −1
k -b 1 ≤ |sinα| + |cosα| ≤ 2
which is equal to slope of line AP =
h-a ⇒ [|sinα| + |cosα|]= 1
So, 1 1 1
h k −b Þ ò xe
-y
d (e y ) = ò xe
-y y
e dy = ò xdy
− = -1 -1 -1
3k h − a
1
⇒ h2 + 3k2 = ah + 3bk Þ ò (e - y - 1) dy = e - e -1 - 2
y
Therefore, 1
dy y 2 - x 2
a= ( since b ¹ 0 )
= 4
dx 2 xy Now,
1
æ dy ö æ dy ö 19
For orthogonal intersection, ç ÷ ç T
è dx ø è dx
÷ = -1. Therefore,
ø
ò f ( x )dx = 12
0
dyT 2 xy a b 19
= Þ + +c =
dx x 2 - y 2 3 2 12
dy 2 xy Therefore,
So, the differential equation is = .
dx x 2 − y 2 b 3
+ b2 = ⇒ b = 1
2 2
20. On differentiating a polynomial of nth degree, we get another
polynomial of (n − 1) degrees. So, since b > 0 and so c = 1
Hence, f ′(0) = b = 1.
f ( x ) = {f ¢( x )}2 Þ n = 2(n - 1) Þ n = 2
1
21. Let f(x) = ax2 + bx + c. Then f ′(0) = b > 0. 22. Using the value a = , b = 1 and c = 1, the function is
4
Also,
f(x) = {f ′(x)}2 x2
f (x) = + x +1
⇒ ax2 + bx + c = 4a2x2 + 4abx + b2 4
Thus, a = 4a2, b = 4ab and c = b2. Þ f ( x ) ¹ ± f (- x )
From which, we get Hence, f(x) is neither odd nor even function.
1160 Mathematics Problem Book for JEE
dy ⎛ cos x ⎞ dy
⇒ x2 −1 = 15( x + x 2 − 1)15 − 15( x − x 2 − 1)15
∫ y + 1 = −∫ ⎜⎝ 2 + sin x ⎟⎠ dx dx
c 1(2 x )
dy d2 y ⎛ 1(2 x ) ⎞
y + 1= (1) + x 2 − 1 2 = 15 × 15( x + x 2 − 1)14 ⎜ 1+ ⎟
2 + sin x 2 x − 1 dx
2 dx ⎝ 2 x 2 − 1⎠
⎛ 1− 1(2 x ) ⎞
Given that y(0) = 1. Therefore, −15 × 15( x − x 2 − 1)14 ⎜ ⎟
⎝ 2 x 2 − 1⎠
c
1+ 1 = ⇒c =4
2 x dy d2 y ( x2 −1+ x)
⇒ + x 2 − 1 2 = 225( x + x 2 − 1)14
Therefore, the equation of the curve is x 2 − 1 dx dx x2 −1
y + 1=
4 225( x − x 2 − 1)14 ( x 2 − 1 − x )
−
2 + sin x x2 −1
p
At x = , we get ⎡ x dy d2 y ⎤
2 ⇒ x2 − 1⎢ + x 2 − 1 2 ⎥ = 225( x + x 2 − 1)15
⎢⎣ x 2 − 1 dx dx ⎥⎦
y
y + 1=
2 +1 +225( x − x 2 − 1)15
4 1 dy d2 y
y = − 1= ⇒x + ( x 2 − 1) 2 = 225 ⎡( x + x 2 − 1)15 + ( x − x 2 − 1)15 ⎤
3 3 dx dx ⎣⎢ ⎦⎥
Hence, the correct answer is option (D).
Substituting ( x + x 2 − 1)15 + ( x − x 2 − 1)15 = y , we get
15 15 dy d2 y
2. If y = ⎡ x + x 2 − 1⎤ + ⎡ x − x 2 − 1⎤ , then ( x 2 − 1) 2 + x d2 y
⎢⎣ ⎦⎥ ⎢⎣ ⎦⎥ dx x dy
dx ( x 2 − 1) + = 225 y
is equal to dx 2 dx
(A) 225 y2 (B) 224 y2 Hence, the correct answer is option (D).
(C) 125 y (D) 225 y
⎛ 3x − 4 ⎞ 4
3. If f ⎜ = x + 2, x ≠ − , and ∫ f ( x )dx = A log 1− x + Bx + C ,
⎝ 3 x + 4 ⎟⎠
(ONLINE)
3
Solution: The given equation is then the ordered pair (A, B) is equal to (where C is a constant
15 15 of integration)
y = ⎡ x + x 2 − 1⎤ + ⎡ x − x 2 − 1⎤
⎣⎢ ⎦⎥ ⎢⎣ ⎦⎥ ⎛ 8 2⎞ ⎛ 8 2⎞
(A) ⎜ − , ⎟ (B) ⎜ , − ⎟
⎝ 3 3⎠ ⎝ 3 3⎠
Differentiating w.r.t. x, we get
Chapter 25 | Differential Equations 1161
−1
⎛ 8 2⎞ ⎛ 8 2⎞ ⎛ ⎞
(C) ⎜ , ⎟ (D) ⎜ − , − ⎟ ⎜⎝ 4 + 9 + x ⎟⎠
⎝ 3 3⎠ ⎝ 3 3⎠ dy 1 1
= = .
(ONLINE) dx 8 x 9+ x 4+ 9+ x 8 x 9+ x
Solution: It is given that 1 1
⇒ dy = . dx (1)
∫ f ( x )dx = A log(1− x ) + Bx + C 4+ 9+ x 8 x 9+ x
O Y
O Q jˆ
iˆ
Figure 26.2
X
12. Equality of vectors: Two vectors a and b are said to be
equal, if Figure 26.4
(a) | a | = | b | If a vector OP makes angles a , b and g with the positive direc-
(b) they have the same or parallel support and tions of X, Y and Z axes, respectively,
then cosa , cosb and cosg are
(c) the same sense. called the direction cosines of OP (Fig. 26.5).
Z
26.4 Rectangular Resolution of Vectors
(Orthogonal System of Vectors): P(x, y, z)
Resolution of a Vector in Two g
b
O
Dimensions a
Y
Any vector r can be expressed as a linear combination of two unit
r
vectors iˆ and jˆ at right angle, that is, r = xiˆ + yjˆ. The vector xiˆ and X
y jˆ are called the perpendicular component vectors of r . The sca-
Figure 26.5
lars x and y are called the components or resolved parts of r in the
directions of x-axis and y-axis, respectively, and the ordered pair
x y z
(x, y) is known as coordinates of point whose position vector is r cos a = , cos b = , cosg =
(Fig. 26.3). OP OP OP
Y x x
P(x, y) cos a = l = = ,
2
x +y +z | r |
2 2
r
ˆj y
y z z
cos b = m = = and cosg = n = =
2
x +y +z | r | 2 2 2
x +y +z |
2 r |
2
O X
iˆ
Clearly, l 2 + m2 + n2 = 1 (or cos2a + cos2b + cos2g = 1).
Figure 26.3 Here, a = ∠POX , b = ∠POY and g = ∠POZ and iˆ , jˆ, k̂ are the
unit vectors along OX , OY and OZ , respectively.
Also, the magnitude of r = x 2 + y 2 and let q be the inclination of
Unit vector in the direction of OP is
⎛ y⎞
r with the x-axis, then q = tan−1 ⎜ ⎟ . uuur
⎝ x⎠ OP x ˆ y ˆ z ˆ
= i+ j+ k = cos a iˆ + cos b ˆj + cosg kˆ
OP OP OP OP
26.5 Resolution of a Vector in Three
Dimensions Illustration 26.1 Find the direction cosines of the vector
3iˆ − 4 ˆj + 5kˆ.
In the orthogonal system of vectors, we choose these vectors
as three mutually perpendicular unit vectors denoted by iˆ , jˆ Solution:
and k̂ directed along the positive directions of X, Y and Z axes, r
r = 3iˆ − 4 ˆj + 5kˆ; | r | = 32 + ( −4 )2 + 52 = 5 2
respectively.
If the coordinates of P are (x, y, z), then the position vector of r
r 3 −4 5 3 −4
ˆ ˆ
can be written as r = xi + yj + zk. ˆ Hence, direction cosines are , , , that is, , ,
1 5 2 5 2 5 2 5 2 5 2
The vectors xiˆ , y jˆ and zk̂ are called the right-angled compo- .
nents of r . The scalars x, y, z are called the components or resolved 2
Chapter 26 | Vector Algebra 1165
⎛1 1⎞ (c) Polygon law of addition: For adding more than two vec-
Illustration 26.2 If a vector has direction cosines ⎜ , m, ⎟ , tors, we have a polygon law of addition which is just an
⎝2 2⎠
deduce the possible values of m. extension of the triangle law.
If in addition, it is stated that the vector makes an obtuse angle
OA + AB + BC + CD + DE + EF = OF
q with the y-axis, determine q.
A consequence of this is that, if the terminus of the last
Solution: The direction cosine (l, m, n) of any direction has the vector coincides with the initial point of the first
vector, the
property sum of the vectors is 0 (Fig. 26.8). To obtain a − b (differ-
l2 + m2 + n2 = 1 ence of two vectors), perform addition of a and ( −b ) . Also,
Therefore, a + o = a; a + ( − a ) = o;
1 1 1
+ m2 + = 1 ⇒ m = ± (k1 + k2 ) a = k1 a + k2 a ; k ((a + b ) = ka + kb )
4 4 2
C
Since the vector makes an obtuse angle q with the y-axis,
1
cos q = − =m
2 B E
and hence,
3p
q=
4 F
D
26.6 Properties of Vectors
O A
1. Addition of vectors
(a) Triangle law of addition: Given two vectors a and b, their Figure 26.8
sum or resultant written as ( a + b) is a vector obtained by (d) Properties of vector addition:
first bringing the initial point of b to the terminal point of
(i) Binary operation: The sum of two vectors is always a
a and then joining the initial point of a to the terminal vector.
point of b giving a consistent direction by completing the
(ii) Commutativity: For any two vectors a and b,
triangle OAB (Fig. 26.6).
a+b =b+a
B (iii) Associativity: For any three vectors a , b and c ,
b
a + (b + c ) = ( a + b ) + c
A
(iv) Identity: Zero vector is the identity for addition. For
any vector a , | a | = 3,| b | = 4.
a a +b
(v) Additive inverse: For every vector a its negative
G G G G
vector − a exists such that a + ( − a ) = ( − a ) + a = 0 , that
Triangle law of addition is, ( − a ) is the additive inverse of the vector a.
O
Illustration 26.3 If the vectors a and b represent two adjacent
Figure 26.6
sides of a regular hexagon, express the other sides as vectors in
(b) Parallelogram law of addition: The sum can
also be terms of a and b .
obtained by bringing the initial points of a and b together
and then completing the parallelogram OACB. Solution: See Fig. 26.9. ABCDEF is a regular hexagon.
Note that addition is commutative, that is, a + b =
D
b + a.
Also, a + ( b+ c ) = ( a + b) + c , that is, the
addition of
E C
vectors obeys the associative law. If a and b are collinear,
their sum is still obtained in the same manner although
we do not have a triangle or a parallelogram in this case
(Fig. 26.7).
F B
C
b a b
A A
a +b a
Figure 26.9
a
B
Let FA = a and AB = b . Then
O b
Parallelogram law of addition FB = FA + AB = a + b
Figure 26.7 FC = 2b ( FC is parallel to AB and lengthwise doubled)
1166 Mathematics Problem Book for JEE
Therefore,
AB + AC + AD + AE + AF = ED + AC + AD + AE + CD
BC = FC − FB = 2b − a − b = b − a (Since AB = ED and AF = CD )
= AC + CD + AE + ED + AD
CD = − a ; DE = − b ; EF = a − b
= AD + AD + AD = 3 AD
lllustration 26.4 Prove that the sum of three vectors determined
by the medians of a triangle directed from the vertices is zero. = 6 AO
This is the resultant required.
Solution: See Fig. 26.10. ABC is the triangle and AD is the median
through A. If AD be produced to a length DG = AD, then ACGB is a Illustration 26.6 ABCD is a parallelogram. A1 and B1 are the mid-
parallelogram. points of side BC and CD, respectively. If AA1 + AB1 = l AC , then find
the value of l .
A
Solution: See Fig. 26.12. Let PV
of A, B and D be 0, b and d,
respectively. Then PV of C = b + d .
F E D d B1 C
b+d
D
B C
A1
A0 bB
Figure 26.12
G
d
b
Figure 26.10 Also, PV of A1 = b + and PV of B1 = d + . So,
2 2
3
3
Hence, by the parallelogram law of addition of two vectors,
2
(
AA1 + AB1 = b + d = AC
2
)
AB + AC = AG = 2 AD
Hence, the value of l is 3/2.
Similarly,
Illustration 26.7 If ABCDEF is a regular hexagon, then find value
BA + BC = 2BE and CB + CA = 2CF
of AD + EB + FC ?
Adding, we have
Solution: See Fig. 26.13.
AB + AC + BA + BC + CB + CA = 2 AD + BE + CF ( ) E D
But the LHS is such that AB + BA = AB − AB = 0.
Similarly, the other two pairs also become zero. Hence, F C
AD + BE + CF = 0
Illustration 26.5 Five forces represented by AB , AC , AD , A B
AE and AF act at the vertex A of a regular hexagon
ABCDEF. Figure 26.13
Prove that their resultant is a force represented by 6 AO , where O We have
is the centre of the hexagon.
AD + EB + FC = ( AB + BC + CD ) + (ED + DC + CB ) + FC
Solution: See Fig. 26.11.
= AB + (BC + CB ) + (CD + DC ) + ED + FC
E D = AB + O + O + AB + 2 AB = 4AB
(ED = AB , FC = 2 AB )
2. Subtraction of vectors: If a and bG are two Gvectors, then their
F
O
C G G
subtraction a − b is defined as a − b = a + ( −b ), where −b is the
negative of b having same magnitude and direction opposite
to vector b. r
r
A B If a = a1iˆ + a2 ˆj + a3kˆ and b = b1iˆ + b2 ˆj + b3kˆ . Then
r r
Figure 26.11 a − b = (a1 − b1)iˆ + (a2 − b2 ) ˆj + (a3 − b3 )kˆ
Chapter 26 | Vector Algebra 1167
n m
C b
B A PV of D =
1+ m
let E divide AC in the ratio 1:l . Then
c
c
b a PV of E = ⇒l = m
1+ l
b+c
O PV of the midpoint of DE =
2(1+ m )
Figure 26.16 which lies on the median. Hence, the median bisects DE.
Illustration 26.16 The median AD of the triangle ABC is bisected
⇒ n(PV of C – PV of A) = m(PV of B – PV of C)
at E, BE meets AC in F. Then find AF : AC.
⇒ n(c − a ) = m(b − c ) Solution: Let the position vector of A with respect to B is a and
mb + na
mb + na that of C with respect to B is c . Then
⇒ c= or OC = 0+c c
m+n m+n Position vector of D wrt B = =
2 2
2. External division: Let A and B be two points with position
c
a+
vectors a and b, respectively, and let C be a point dividing AB Position vector of E = 2 =a+c (1)
externally in the ratio m:n (Fig. 26.17). Then the position vector 2 2 4
mb − na Let AF:FC = l : 1 and BE : EF = m : 1. Then
of C is given by OC = .
m−n lc +a
Position vector of F =
n 1+ l
m Now,
C
B A æ l c + aö .
mç +1 0
è 1+ l ÷ø
Position vector of E = (2)
a m +1
b From Eqs. (1) and (2), we get
c
a c m lm
+ = a+ c
2 4 (1+ l )(1+ m ) (1+ l )(1+ m )
O
1 m 1 lm
Figure 26.17 ⇒ = and =
2 (1+ l )(1+ m ) 4 (1+ l )(1+ m )
Note: 1
b+a ⇒l=
(a) If C is the midpoint of AB, then PV of C is . 2
2
Therefore,
mb + na 1
(b) We have c = . Hence, c is in the form of c = l a + m b , AF AF l 1
m+n = = 2
= =
n m AC AF + FC 1+ l 3 3
where, l = and m = . Thus, position vector of 2
m+n m+n
any point C on AB can always be taken as c = l a + m b
where l + m = 1. 26.11 Bisector of the Angle Between Two
(c) If the circumcentre is theorigin and vertices of a triangle
Vectors
have position vectors a , b , c , then the position vector of
orthocentre will be −(a + b + c ). Consider two non-zero, non-collinear vectors a and b. The bisec-
⎛ a b ⎞
tor of the angle between the two vectors a and b is k ⎜ + ⎟ ,
Illustration 26.15 ABC is a triangle. A line is drawn parallel to BC ⎝ | a | | b |⎠
where k ∈ R+.
to meet AB and AC at D and E, respectively. Prove that the median
through A bisects DE. Illustration 26.17 If the vector (iˆ − 3 ˆj + 5kˆ ) bisects the angle
between â and (2kˆ + 2 ˆj − iˆ ), where â is a unit vector, then find â.
Solution: Take the vertex A of the triangle ABC as the origin. Let b
Solution: According to the given conditions,
and c be the position vector (PV) of B and C. The mid-point of BC
b+c b+c 2kˆ + 2 ˆj − iˆ
has PV = . The equation of the median is r = t . l (iˆ − 3 ˆj + 5kˆ ) = aˆ +
2 2 3
Let D divide AB in the ratio 1: m. Then ⇒ 3â = 3l (iˆ − 3 ˆj + 5kˆ ) − (2kˆ + 2 ˆj − iˆ )
Chapter 26 | Vector Algebra 1171
⇒ 3â = iˆ(3l + 1) – ĵ (2 + 9l) + k̂(15l – 2) (C) 593 (D) 369 Ans. (C)
1
⇒ | a | = (3l + 1)2 + (2 + 9 l )2 + (15l - 2)2
3 8. If the position vector of a point A is a + 2b and a divides AB in
⇒ 9 = (3l + 1)2 + (2 + 9l)2 + (15l – 2)2 the ratio 2:3,
then the position vector of B is
2 (A) 2a − b (B) b − 2a
⇒ 315l2 – 18l = 0 ⇒ l = 0,
35 (C) a − 3b (D) b Ans. (C)
If l = 0, then ˆ
a = ˆ
i − 2 ˆ
j − 2 ˆ
k (which is not acceptable).
26.12 Product of Two Vectors
2 1
Therefore, for l = , aˆ = (41iˆ − 88 ˆj − 40kˆ ). Product of two vectors is processed by two methods. When the
35 105
product of two vector results is a scalar quantity, then it is called
scalar product. It is also known as dot product because we are put-
Your Turn 1 ting a dot (.) between two vectors.
When the product of two vector results is a vector quantity,
then this product is called vector product. It is also known as cross
1. If the vectors 4 iˆ + 11ˆj + mkˆ ,7iˆ + 2 ˆj + 6kˆ and iˆ + 5 ˆj + 4 kˆ are copla- product because we are putting a cross (×) between two vectors.
nar, then m is
(A) 38 (B) 0 26.13 Scalar or Dot Product of Two
(C) 10 (D) –10 Ans. (C)
Vectors
2. The value of l for which the four points 2iˆ + 3 ˆj − kˆ , iˆ + 2 ˆj + 3kˆ ,
3iˆ + 4 ˆj − 2kˆ , iˆ − l ˆj + 6kˆ are coplanar is See Fig. 26.18. If a and b are two non-zero vectors and q be the
angle between them, then their scalar product (or dot product)
(A) 8 (B) 0
(C) −2 is denoted by a ⋅ b and is defined as the scalar | a || b | cosq , where
(D) 6 Ans. (C)
| a | and | b | are moduli of a and b, respectively, and 0 ≤ q ≤ p .
r r r
3. If a = iˆ + ˆj + kˆ , b = 4 iˆ + 3 ˆj + 4 kˆ and c = iˆ + a ˆj + b kˆ are linearly
dependent vectors and | c | = 3 , then
(A) a = 1, b = −1 (B) a = 1, b = ±1 a
(C) a = −1, b = ±1 (D) a = ±1, b = 1 Ans. (D)
q
4. The position vectors of the vertices A, B, and C of a triangle are
b
iˆ − ˆj − 3kˆ, 2iˆ + ˆj − 2kˆ and −5iˆ + 2 ˆj − 6kˆ, respectively. The length acosq
of the bisector AD of the angle BAC where D is on the segment
Figure 26.18
BC is
Note:
3 1
(A) 10 (B) 1. a ⋅ b ≤ | a || b | .
4 4
11 2. If a ⋅ b > 0, then angle between a and b is acute.
(C) (D) None of these Ans. (A)
2 3. If a ⋅ b < 0 , then angle between a and b is obtuse.
5. The unit vector parallel to the resultant vector of 2iˆ + 4 ˆj − 5kˆ 4. The dot product of a zero and non-zero vector is a scalar zero.
and iˆ + 2 ˆj + 3kˆ is
26.13.1 Geometrical Interpretation of Scalar
1 iˆ + ˆj + kˆ
(A) (3iˆ + 6 ˆj − 2kˆ ) (B) Product
7 3
Let a and b be two vectors represented
by
OA and OB , respec-
iˆ + ˆj + 2kˆ 1 tively. Let q be the angle between OA and OB . Draw BL ⊥ OA and
(C) (D) ( − iˆ − ˆj + 8kˆ ) Ans. (A)
6 69 AM ⊥ OB .
B
M
6. If the sum of two vectors is a unit vector, then the magnitude of
their difference is
b
(A) 2 (B) 3
1 q
(C) (D) 1 Ans. (B) A
O a L
3
7. The length of the longer diagonal
of the parallelogram con- Figure 26.19
structed on 5a + 2 b and a – 3 b, given that | a | = 2 2 , | b | = 3
p From Δs, OBL and OAM, we have OL = OB cosq and
and angle between a and b is , is
4 OM = OA cosq . Here, OL and OM are known as projection of b
(A) 15 (B) 113 on a and a on b, respectively (Fig. 26.19). Now,
1172 Mathematics Problem Book for JEE
a ⋅ b = | a || b |cosq a ⋅b -1 æ a × b ö
⇒ cosq = ⇒ q = cos çç ÷÷
= | a |(OB cosq ) = | a |(OL ) | a || b | è | a || b | ø
r
= (Magnitude of a )(Projectionof b on a ) (1) r ˆ ˆ ˆ ˆ ˆ ˆ
If a = a1i + a2 j + a3k and b = b1i + b2 j + b3k, then angle
Again, between vector is
a ⋅ b = | a || b |cosq = | b |(| a | cosq ) ⎛ ⎞
a1b1 + a2b2 + a3b3
q = cos −1 ⎜ ⎟
= | b |(OA cosq ) = | b |(OM ) ⎜⎝ a2 + a2 + a2 b2 + b2 + b2 ⎟⎠
1 2 3 1 2 3
a⋅ b = (Magnitude of b) (Projection of a on b) (2)
Illustration 26.18 A unit vector in the plane of the vectors
Thus, geometrically interpreted, the scalar product of two
2iˆ + ˆj + kˆ, iˆ − ˆj + kˆ and orthogonal to 5iˆ + 2 ˆj + 6kˆ is
vectors is the product of modulus of either vector and the
projection of the other in its direction.
6iˆ − 5kˆ 3 ˆj − kˆ
(A) (B)
26.13.2 Properties of Scalar Product 61 10
2iˆ − 5 ˆj 2iˆ + ˆj − 2kˆ
1. Commutativity: The (C) (D)
scalar
product of two vector is commu-
29 3
tative, that is, a ⋅ b = b ⋅ a .
2. Distributivity of scalar product over vector addition: The Solution: Let a unit vector in the plane of 2i + ˆj + kˆ and iˆ − ˆj + kˆ be
ˆ
scalar product of vectors is distributive over vector addition, aˆ = a (2iˆ + ˆj + kˆ ) + b (iˆ − ˆj + kˆ ) = (2a + b ) iˆ + (a − b ) ˆj + (a + b ) kˆ
that is
As â is a unit vector, we have
(a) a ⋅(b + c ) = a ⋅ b + a ⋅ c (b) (b + c )⋅ a = b ⋅ a + c ⋅ a
(2a + b )2 + (a − b )2 +(a + b )2 = 1
3. Let a and b be two non-zero vectors a ⋅ b = 0 ⇔ a ⊥ b.
As iˆ , ˆj , kˆ are mutually perpendicular unit vectors along the
⇒ 6a 2 + 4a b + 3 b 2 = 1 (1)
coordinate axes, therefore
iˆ ⋅ ˆj = ˆj ⋅ iˆ = 0; ˆj ⋅ kˆ = kˆ ⋅ ˆj = 0; kˆ ⋅ iˆ = iˆ ⋅ kˆ = 0 As â is orthogonal to 5iˆ + 2 ˆj + 6kˆ, we get
5 (2a + b ) + 2 (a − b ) + 6(a + b ) = 0
4. For any vector a,
a ⋅ a = | a |2 ⇒ 18a + 9 b = 0 ⇒ b = −2a
As iˆ , ˆj , kˆ are unit vectors along the co-ordinate axes, therefore From Eq. (1), we get
1
iˆ ⋅ iˆ = | iˆ | = 1, ˆj ⋅ ˆj = | ˆj | = 1and kˆ ⋅ kˆ = | kˆ |2 = 1
2 2
6a 2 − 8a 2 + 12a 2 = 1 ⇒ a = ±
10
5. If m is a scalar and a , b be any two vectors, then 2
⇒ b =∓
( ma )⋅ b = m(a ⋅ b ) = a ⋅( mb ) 10
6. If m, n are scalars and a , b be two vectors, then ⎛ 3 ˆ 1 ˆ⎞
Thus, aˆ = ± ⎜ j− k⎟ .
⎝ 10 10 ⎠
ma ⋅ nb = mn(a ⋅ b ) = ( mna ) ⋅ b = a ⋅( mnb )
Hence, the correct answer is option (B).
7. For any vectors a and
b, we have r
(a) a ⋅ ( −b ) = −(a ⋅ b ) = ( − a )⋅ b (b) ( − a ) ⋅( −b ) = a ⋅ b Illustration 26.19 If three non-zero vectors are a = a1iˆ + a2 ˆj + a3kˆ ,
r r
8. For any two vectors a and b, we have b = b1iˆ + b2 ˆj + b3kˆ and c = c1iˆ + c2 ˆj + c3kˆ. If c is the unit vector per-
(a) | a + b |2 = | a |2 + | b |2 + 2a ⋅ b pendicular to the vectors a and b and the angle between a and b
2
(b) | a − b |2 = | a |2 + | b |2 − 2a ⋅ b a1 a2 a3
p
is , then find the value of b1 b2 b3 .
(c) (a + b ) ⋅ (a − b ) = | a |2 − | b |2 6
c1 c2 c3
(d) | a + b | = | a | + | b | ⇒ a || b
(e) | a + b |2 = | a |2 + | b |2 ⇒ a ⊥ b Solution: As c is the unit vector perpendicular to a and b , we have
(f) | a + b | = | a − b | ⇒ a ⊥ b | c | = 1, a. c = 0 = b . c
r r
9. If a = a1iˆ + a2 ˆj + a3kˆ and b = b1iˆ + b2 ˆj + b3kˆ, then, a1 a2 a3
2
a1 a2 a3 a1 a2 a3
a ⋅ b = a1b1 + a2b2 + a3b3 b1 b2 b3 = b1 b2 b3 b1 b2 b3
Thus, scalar product of two vectors is equal to the sum of the c1 c2 c3 c1 c2 c3 c1 c2 c3
products of their corresponding components. In particular,
a ⋅ a = | a |2 = a12 + a22 + a32 a12 + a22 + a32 a1b1 + a2b2 + a3b3 a1c1 + a2c2 + a3c3
10. If a , b be two vectors inclined at an angle q , then = a1b1 + a2b2 + a3b3 b12 + b22 + b32 b1c1 + b2c2 + b3c3
a ⋅ b =| a || b |cosq a1c1 + a2c2 + a3c3 b1c1 + b2c2 + b3c3 c12 + c22 + c32
Chapter 26 | Vector Algebra 1173
r (1)
| a |2 a . b a . c | a |2 a . b 0 a = 2 piˆ + ˆj = ( p +1) iˆ + jˆ
= a . b | b |2 b . c = a . b | b |2 0 = | a |2 | b |2 − (a .b )2 Now,
a . c b . c | c |2 0 0 1 iˆ = cosq iˆ + sinq jˆ
⎛ p⎞
2
⎛ 3⎞ jˆ = − sinq iˆ + cosq jˆ
= | a |2 | b |2 − ⎜ | a || b |cos ⎟ = | a |2 | b |2 ⎜ 1− ⎟
⎝ 6⎠ ⎝ 4⎠ Therefore, from Eq. (1)
1 2 2 1
= | a | | b | = ( Σa12 )( Σb12 ) 2 piˆ + ˆj = ( p + 1)(cosq iˆ + sinq ˆj ) + ( − sinq iˆ + cosq ˆj )
4 4
⇒ 2 piˆ + ˆj = {( p + 1)cosq − sinq } iˆ + {( p + 1)sinq + cosq } ˆj
26.13.3 Components of a Vector Along and
⇒ 2 p = ( p + 1)cosq − sinq (2)
Perpendicular to Another Vector
and
If a and b be two vectors represented by OA and OB . Let q be the
1 = ( p + 1)sinq + cosq (3)
angle between a and b. Draw BM ⊥ OA. In ΔOBM, we have
Squaring and adding,
OB = OM + MB ⇒ b = OM + MB
4 p2 + 1 = ( p + 1)2 + 1
Thus, OM and MB are components of b along a and perpendicular
to a, respectively (Fig. 26.20). 1
⇒ ( p + 1)2 = 4 p2 ⇒ p = 1, −
B 3
Solution: See Fig. 26.21. Without loss of generality, we can write Solution:
ˆ ˆ ˆ 24 3
y F = (24 3 ) i + 2 j + k = (iˆ + 2 ˆj + kˆ ) = 12 2( iˆ + 2 ˆj + kˆ )
Y | iˆ + 2 ˆj + kˆ | 6
X
j
Displacement r = Position vector of C – Position vector of A
Jˆ q = (iˆ + ˆj + kˆ ) − (2iˆ − 3 ˆj + 2kˆ ) = ( − iˆ + 4 ˆj − kˆ )
q iˆ x
O i Work done by the force is W = r ⋅ F
Illustration 26.22 Prove by vector method that Illustration 26.27 A particle acted on by constant forces
(a1b1 + a2b2 + a3b3)2 ≤ (a12+ a22 + a32) (b12 + b22 + b32) 4 i + ˆj − 3kˆ and 3iˆ + ˆj − kˆ is displaced from the point iˆ + 2 ˆj + 3kˆ to
ˆ
the point 5iˆ + 4 ˆj + kˆ. Find the total work done by the forces.
Solution: Let a = a1iˆ + a2 ĵ + a3k̂ and b = b1iˆ + b2 ˆj + b3kˆ. Then
Solution: Let F be the resultant of the forces and d , the
a . b = a1 b1 + a2b2 + a3b3 = | a| | b| cosq ≤ | a| | b|
displacement. Then
⇒ ( a . b )2 ≤ | a |2 | b |2
⇒ (a1b1 + a2b2 + a3b3 ≤ )2 ( a12 + a22 + a32 )( b12 + b22 + b32 ) F = (4 iˆ + ˆj − 3kˆ ) + (3iˆ + ˆj − kˆ ) = 7iˆ + 2 ˆj − 4 kˆ
Illustration 26.23 If | a | = 3, | b | = 1, | c | = 4 , and a + b + c = 0, find
( ) ( )
d = 5iˆ + 4 ˆj + kˆ − iˆ + 2 ˆj + 3kˆ = 4 iˆ + 2 ˆj − 2kˆ
Therefore,
the value of a ⋅ b + b ⋅ c + c ⋅ a .
Solution: We know, Total work done = F ⋅ d = (7iˆ + 2 ˆj − 4 kˆ )⋅(4 iˆ + 2 ˆj − 2kˆ ) = 40 units
(a + b + c )2 = (a + b + c ) ⋅ (a + b + c )
⇒ 0 = | a |2 + | b |2 + | c |2 + 2(a ⋅ b + b ⋅ c + c ⋅ a ) (Given a + b + c = 0)
Your Turn 2
⇒ 0 = (3)2 + (1)2 + ( 4 )2 + 2(a ⋅ b + b ⋅ c + c ⋅ a )
26 1. Let a , b and c be vectors with magnitudes 3, 4 and
5, respec-
⇒ a ⋅ b + b ⋅ c + c ⋅ a = – = –13 tively, and a + b + c = 0. Then find the values of a ⋅ b + b ⋅ c + c ⋅ a .
2
Ans. –25
Illustration 26.24 In a ΔABC, prove by vector method that r r
2. Let b = 3 ˆj + 4 kˆ, a = iˆ + ˆj and let b1 and b2 be component vec-
cos 2A + cos 2B + cos 2C ≥ –3/2 r 3 3
tors of b parallel and perpendicular to a. If b1 = iˆ + ˆj , then
Solution: As we know 2 2
(1) b2 = _____.
(OA + OB + OC )2 ≥ 0
[MP PET 1989]
and
3ˆ 3 ˆ 3 3
| OA |2 = | OB |2 = | OC |2 = R 2 (2) (A) i + j + 4 kˆ (B) − iˆ + ˆj + 4 kˆ
2 2 2 2
Now using Eq. (1), we get 3 3
(C) − iˆ + ˆj (D) None of these Ans. (B)
(
| OA |2 + | OB |2 + | OC |2 + 2 OA ⋅ OB + OB ⋅ OC + OC ⋅ OA ≥ 0 ) r
2 2
r r
3. (a ⋅ iˆ ) iˆ + (a ⋅ ˆj ) ˆj + (a ⋅ kˆ ) kˆ = _____.
⇒ 3R2 + 2R2 (cos 2A + cos2B + cos 2C) ≥ 0
⇒ cos 2A + cos 2B + cos 2C ≥ –3/2 (A) a (B) 2 a
(C) 3a (D) 0 Ans. (A)
Illustration 26.25 A particle is acted upon by constant forces
4 iˆ + ˆj − 3kˆ and 3iˆ + ˆj − kˆ which displace it from a point iˆ + 2 ˆj + 3kˆ 4. If | a | = 3,| b | = 4 , then a value of l for which a + l b is perpen-
to the point 5iˆ + 4 ˆj + kˆ. Then find the work done in standard units dicular to a − l b is
by the force. (A) 9/16 (B) 3/4
(C) 3/2 (D) 4/3 Ans. (B)
Solution:
r r
Total force F = (4 iˆ + ˆj − 3kˆ ) + (3iˆ + ˆj − kˆ ) = 7iˆ + 2 ˆj − 4 kˆ 5. The vectors a = 2l 2iˆ + 4 l ˆj + kˆ and b = 7iˆ− 2 ˆj +l kˆ make an
Displacement d = (5iˆ + 4 ˆj + kˆ ) − (iˆ + 2 ˆj + 3kˆ ) = 4 iˆ + 2 ˆj − 2kˆ obtuse angle, whereas the angle between b and k is acute and
less than p /6. Then domain of l is
Work done = F × d = (7iˆ + 2 ˆj − 4 kˆ ).(4 iˆ + 2 ˆj − 2kˆ ) = 28 + 4 + 8 = 40
1
Illustration 26.26 Find the work done by the force F = iˆ + ˆj + 2kˆ (A) 0 < l < (B) l > 159
2
acting on a particle, if the particle is displaced from the point 1
with position vector iˆ + 2 ˆj + 2kˆ to the point with position vector (C) − < l < 0 (D) Null set Ans. (D)
2
2iˆ + 3 ˆj + 3kˆ.
Solution: Here, F = iˆ + ˆj + 2kˆ and displacement, 26.14 Vector or Cross-Product of Two
d = (2iˆ + 3 ˆj + 3kˆ ) − (iˆ + 2 ˆj + 2kˆ ) = iˆ + ˆj + kˆ Vectors
Therefore,
Let a , b be two non-zero, non-parallel vectors. Then the vector
Work done = F ⋅ d = (iˆ + ˆj + 2kˆ )⋅(iˆ + ˆj + kˆ )
product a ×b , in that order, is defined as a vector whose magni-
= (1)(1) + (1)(1) + (2)(1) = 1+ 1+ 2 = 4 units tude is | a || b | sinq , where q is the angle between a and b whose
Chapter 26 | Vector Algebra 1175
direction is perpendicular to the plane of a and b in such a way It follows from the above property that a × a = 0 for every non-
G G
that a , b and this direction constitute a right-handed system. zero vector a, which in turn implies that iˆ × iˆ = ˆj × ˆj = kˆ × kˆ = 0.
In other words, a × b =| a || b | sinq ηˆ , where q is the angle 7. Vector product of orthonormal triad of unit vectors
i , j , k using the definition of the vector product, we obtain
between
a and b, η̂ is a unit vector perpendicular to the plane of a
G G
and b such that a , b , η̂ form a right-handed system. iˆ × ˆj = kˆ , ˆj × kˆ = iˆ , kˆ × iˆ = ˆj , ˆj × iˆ = − kˆ , kˆ × ˆj = − iˆ , iˆ × kˆ = − ˆj
G G
26.14.1 Geometrical Interpretation of the 8. Lagrange’s identity: If a , b are any two vectors, then
Vector Product | a × b |2 = | a |2 | b |2 − (a ⋅ b )2 or | a × b |2 + (a ⋅ b )2 = | a |2 | b |2
r
r product in terms of components: If a = a1i + a2 j + a3k
9. Vector ˆ ˆ ˆ
If
a , b be two non-zero, non-parallel vectors represented by OA and ˆ ˆ ˆ
and b = b1i + b2 j + b3k. Then,
OB , respectively, and let q be the angle between them. Complete
the parallelogram OACB. Draw BL ⊥ OA . r r
a × b = (a2b3 − a3b2 ) iˆ − (a1b3 − a3b1) ˆj + (a1b2 − a2b1) kˆ
In ΔOBL,
BL iˆ ˆj kˆ
sinq = ⇒ BL = OB sinq = | b |sinq (1)
OB = a1 a2 a3
Now, b1 b2 b3
a × b = | a || b |sinq η̂ = (OA)(BL ) η
ˆ 10. Angle between two vectors: If q is the angle between a and
= (Base × Height)ηˆ = (area of parallelogram OACB )η̂ |a ×b|
b, then sinq = .
= Vector area of the parallelogram OACB | a || b |
r r
ηˆ Expression for sinq : If a = a1iˆ+ a2 ˆj + a3kˆ, b = b1iˆ + b2 ˆj + b3kˆ
B C
and q be angle between a and b , then
a 7. Three points with position vectors a , b , c are collinear, if
b
( a × b ) + (b × c ) + ( c × a ) = 0
1
= b ×c −b ×a −d ×c +d ×a M = r × F , where r = BA
4
1 ⇒ | M | = | BA × F | = | F || BA |sinq ,
= a×b +b ×c +c ×d +d ×a
4
Hence, the correct answer is option (C). where q is the angle between BA and F
26.14.5 Moment of a Force | M | = | F |(BN ) = | F | a
a force F be applied at a point P. The
1. About a point: Let where a = BN is the arm of the couple and +ve or –ve sign is to be
moment of force F about a point O is defined (Fig. 26.28) as taken according as the forces indicate a counter-clockwise rota-
tion or clockwise rotation.
M = OP × F
Illustration 26.31 Find the moment about the point iˆ + 2 ˆj + 3kˆ
P F
of a force represented by iˆ + ˆj + kˆ acting through the point
q
−2iˆ + 3 ˆj + kˆ.
Solution: See Fig. 26.30. Let O be the point iˆ + 2 ˆj + 3kˆ and P the
point −2iˆ + 3 ˆj + kˆ. Then
O O F
Figure 26.28
r
(a) Moment of force about a point is vector quantity.
(b) Moment is independent of selection of pointP, in fact P
P
can be any point on the line of action of force F.
(c) If several forces are acting through the point P, then the Figure 26.30
vector sum of the moments of the separate forces about O
is equal to the moment
of their resultant force about O. OP = ( PV of P ) − ( PV of O )
(d) The
moment of F about a point O measures the amount of
F to turn the body about point O. If tendency of rotation ⇒ r = −3iˆ + ˆj − 2kˆ
is in the anticlockwise direction, the moment is positive,
otherwise it is negative. Let M be the vector moment of F acting at P about point O. Then
2. About a line: Let F be any given force, acting at a point P and
L be any directed line segment. The moment of force F about iˆ ˆj kˆ
line L is defined as M = r × F = −3 1 −2 = 3iˆ + ˆj − 4 kˆ
Ma = (OP × F )⋅ aˆ 1 1 1
where â is a unit vector in the direction of line and O is any Illustration 26.32 Forces 2iˆ + 7 ˆj , 2iˆ − 5 ˆj + 6kˆ , − iˆ + 2 ˆj − kˆ act at
point on the line.
(a) Moment about a point P whose position vector is 4 iˆ − 3 ˆj − 2kˆ. Find the vector
a line is a scalar quantity. moment of the resultant of three forces acting at P about the point
(b) Moment of F about the line L isthe projection along L, of
the vector moment of the force F about any point on the L. Q, whose position vector is 6iˆ + ˆj − 3kˆ.
r r r
26.14.6 Moment of a Couple Solution: Let F1 = 2iˆ + 7 ˆj , F2 = 2iˆ − 5 ˆj + 6kˆ , F3 = − iˆ + 2 ˆj − kˆ. Then
A system consisting of a pair of equal unlike parallel forces is called a the resultant force F is given by
r
couple. The vector sum of two forces of a couple is always zero vector. F = 3iˆ + 4 ˆj + 5kˆ
See Fig. 26.29. The moment of a couple is a vector perpen-
Let r = QP . Then
dicular to the plane of couple and its magnitude is the product
of the magnitude of either force with the perpendicular distance r = PV of P − PV of Q
between the lines of the forces.
= (4 iˆ − 3 ˆj − 2kˆ ) − (6iˆ + ˆj − 3kˆ )
F q A N
q = −2iˆ − 4 ˆj + kˆ
r
a Let M be the moment of the resultant force F about Q. Then
iˆ ˆj kˆ
B M = r × F = −2 −4 1 = −24 iˆ + 13 ˆj + 4 kˆ
−F
3 4 5
Figure 26.29
1178 Mathematics Problem Book for JEE
26.15.3 Tetrahedron
Figure 26.31
A tetrahedron is a three-dimensional figure formed by four triangle
See Fig. 26.31. The value of scalar triple product depends on the OABC is a tetrahedron with ΔABC as the base. OA, OB , OC , AB , BC
cyclic order of the vectors and is independent of the position of the and CA are known as edges of the tetrahedron. OA, BC ; OB , CA
dot and cross. These may be interchanged at pleasure. However, and OC , AB are known as the pairs of opposite edges. A tetrahe-
anticyclic permutation of the vectors changes the value of triple dron in which all edges are equal, is called a regular tetrahedron
product in sign but not in magnitude. (Fig. 26.32).
Chapter 26 | Vector Algebra 1179
A(a) = (u + v − w )⋅[(u × v ) − (u × w ) − 0 + (v × w )]
a
= [u u v ] + [v u v ] − [w u v ] − [u uw ]
− [v uw ] + [w uw ] + [u v w ] + [v v w ] − [w v w ]
b c = 0 + 0 − [u v w ] − 0 + [u v w ] + 0 + [u v w ] + 0 − 0 = [u v w ]
B(b) C(c)
= u .(v × w )
Figure 26.32
Illustration 26.34 Find the value of ‘a’ so that the volume of
26.15.4 Properties of a Tetrahedron parallelepiped formed by i + a j + k ; j + ak and ai + k becomes
minimum.
1. If two pairs of opposite edges of a tetrahedron are perpendic-
ular, then the opposite edges of the third pair are also perpen- Solution: Volume of the parallelepiped,
dicular to each other.
2. In a tetrahedron, the sum of the squares of two opposite edges V = [i + a j + k j + ak ai + k ]
is the same for each pair. = (i + a j + k ) × {( j + ak ) ´ (ai + k )}
3. Any two opposite edges in a regular tetrahedron are perpen-
dicular. = (i + a j + k ) × {i + a2 j - ak }
= 1+ a3 − a
26.15.5 Volume of a Tetrahedron
1. The volume of a tetrahedron is Now,
1 dV d 2V
(area of the base) (corresponding altitude) = 3a2 − 1 ⇒ 2 = 6a
3 da da
1 1
1
dV 1
= ⋅ | AB × AC || ED | = | AB × AC || ED |cos 0° for AB × AC || ED = 0 ⇒ 3a2 − 1 = 0 ⇒ a = ±
3 2 6 da 3
1
.
1
1
1
= ( AB ´ AC ) ED = [ AB AC EA + AD ] = [ AB AC AD] At a = ,
6 6 6 3
Because AB , AC , EA are coplanar, so [ AB AC EA] = 0 d 2V 6
= >0
da 2
3
2. If a , b , c are position vectors of vertices A, B and C with respect
1 1
to O, then volume of tetrahedron OABC = [a b c ]. Therefore, V is minimum at a = .
6 3
3. If a , b , c , d are position vectors of vertices A, B, C, D of a tetrahe-
1 Illustration 26.35 x, y, z are distinct scalars such that [xa + yb
dron ABCD, then its volume = [b − a c − a d − a ] .
6 + zc , xb + yc + za , xc + ya + zb ] = 0 , where a , b , c are non-coplanar
26.15.6 Reciprocal System of Vectors vectors. Then
(a) x + y + z = 0 (c) xy + yz + zx = 0
Let a , b , c be three non-coplanar vectors, and let
b ´ c
c ´ a
a ´ b (b) x 3 + y 3 + z 3 = 0 (d) x 2 + y 2 + z 2 = 0
¢ ¢
a = , b = , c = ¢
[abc ] [abc ] [abc ]
Solution: a , b , c are non-coplanar. Therefore,
Then a¢ , b¢ , c ¢ are said to form a reciprocal system of vectors for
[a b c ] ≠ 0
the vectors a , b , c .
Now,
If a , b , c and a¢ , b¢ , c ¢ form a reciprocal system of vectors, then
[ xa + yb + zc , xb + yc + za , xc + ya + zb ] = 0
1. a × a¢ = b × b¢ = c × c ¢ = 1
2. a × b¢ = a × c ¢ = 0 ; b × c ¢ = b × a¢ = 0 ; c × a¢ = c × b¢ = 0 ⇒ ( xa + yb + zc )⋅ {( xb + yc + za ) × ( xc + ya + zb )} = 0
1 ⇒ ( xa + yb + zc )⋅ {( x 2 − yz )(b × c ) +
3. [a¢ b¢ c ¢ ] =
[a b c ]
( z 2 − xy )(a × b ) + ( y 2 − zx )(c × a )} = 0
4. a , b , c are non-coplanar iff so are a¢ , b¢ , c ¢
⇒ x ( x 2 − yz )[abc ] + y ( y 2 − zx )[b c a ] + z ( z 2 − xy )[c a b ] = 0
Illustration 26.33 If u , v and w are three non-coplanar vectors,
then find the value of (u + v − w )⋅[(u − v ) × (v − w )] . ⇒ ( x 3 − xyz )[a b c ] + ( y 3 − xyz )[abc ] + ( z 3 − xyz )[abc ] = 0
Solution: As [abc ] ≠ 0, so
(u + v − w )⋅[u − v × (v − w )] x 3 + y 3 + z 3 − 3 xyz = 0
1180 Mathematics Problem Book for JEE
Solution: 1
3. Let a , b and c be non-zero vectors such that (a × b ) × c = | b |
3
a × (b × c ) = ( a ⋅ c ) b − ( a ⋅ b ) c | c | a and q is the acute angle between the vectors b and c.
b × ( c × a ) = (b ⋅ a ) c − (b ⋅ c ) a Then sinq equals
2 2 2
c × (a × b ) = (c ⋅ b ) a − (c ⋅ a ) b (A) (B)
3 3
Adding the three results, 2 1
(C) (D) Ans. (A)
a × (b × c ) + b × ( c × a ) + c × ( a × b ) 3 3
4. If a = i + j + k , b = i + j , c = i and (a ´ b ) ´ c = l a + m b , then l + m =
= ( a × c ) b - ( c × a ) b + (b × a ) c - ( a × b ) c + ( c × b ) a - (b × c ) a
=0 (A) 0 (B) 1
(C) 2 (D) 3 Ans. (A)
Illustration 26.37 Prove that a necessary and sufficient condi-
5. If a , b , c and p , q , r are reciprocal system of vectors, then
tion that a × (b × c ) = (a × b ) × c is (a × c ) × b = 0. a × p + b × q + c × r equals
Chapter 26 | Vector Algebra 1181
(A) [a b c ] (B) ( p + q + r ) Illustration 26.39 Let a , b , c be three non-zero
vectors such that
(C) 0 (D) a + b + c Ans. (C) any
two of them are non-collinear. If a + 2b is collinear
with c and
b + 3c is collinear with a, then prove that a + 2b + 6c = 0.
26.17 Scalar or Vector Product of Four Solution: It is given that a + 2b is collinear with c, so
Vectors a + 2b = l c (for some scalar l) (1)
26.17.1 Scalar Product Also b + 3c is collinear with a, so
b + 3c = m a (for some scalar m) (2)
(a × b )⋅(c × d ) is a scalar productof four vectors. It is the dot prod-
uct of the vectors
a ×b and c × d . It is a scalar triple product of the From Eqs. (1) and (2), we get
a, b c × d
vectors and as well as scalar triple product of the vectors
(1 + 2m ) b + (3 – ml) c = 0
a × b , c and d.
⇒ 1 + 2m = 0 and 3 – ml = 0 { b and c are non-collinear vectors}
a ⋅ c a ⋅d
(a × b )⋅(c × d ) =
b ⋅ c b ⋅d ⇒ m = – 1/2 and l = – 6
Substituting the values of l and m in Eqs. (1) and (2), we get
26.17.2 Vector Product
a + 2b + 6c = 0
(a × b ) × (c × d ) is a vector product of four vectors. It is the cross
product of the vectors a × b and c × d .
Illustration 26.40 Prove that
a × {b × (c × d )},{( a × b ) × c} × d are also different vector products of
[ a + b , b + c , c + a ] = 2[ a b b ]
four vectors a , b , c and d.
Solution:
Illustration 26.38 Let a , b , c be three mutually perpendicular
vectors of the same magnitude and the vector x satisfy the equa-
[a+b , b +c , c +a]
tion a × {( x − b ) × a } + b × {( x − c ) × b } + c × {( x − a ) × c }. Then find x.
= ( a + b ) ⋅ {( b + c ) × ( c + a )}
Solution: Here
= ( a + b )⋅( b × c + b × a + c × c + c × a )
(a × a )( x - b ) - {a × ( x - b )} × a + (b × b )( x - c ) - {b ×( x - c )} × = a⋅b × c + b ⋅c × a
b + (c × c )( x - a ) - {c× ( x - a )}c = 0
= [ a b c ]+[ a b c ] (As b × ( c ´ a ) = a × ( b ´ c ))
or GGG
G G G G G G G G G G G G G G G G G G = 2[ a b c ]
l 2 ( x − b + x − c + x − a ) = {a ⋅ ( x − b )}a + {b ⋅ ( x − c )}b + {c ⋅ ( x − a )}c
Illustration 26.41 Find l if l iˆ + ˆj + 2kˆ ; iˆ + l ˆj − kˆ and 2iˆ − ˆj + l kˆ
where | a | = | b | = | c | = l
are coplanar.
l 2 {3 x − (a + b + c )} = (a. x )a + (b . x )b + (c . x ) c Solution: The condition for coplanarity is
Let x = a a + b b + g c. Then l 1 2
1 l −1 = 0
a. x = a | a |2 = a l 2, b . x = b l 2 and c ⋅ x = g l 2
2 −1 l
⇒ l 2 {3 x −(a + b + c )} = l 2 x ⇒3 x − (a + b + c ) = x
Þ l ( l 2 - 1) - 1( l + 2) + 2( -1- 2l ) = 0
Hence,
Þ l 3 - 6l - 4 = 0
a+b +c
x=
2 By inspection it is seen that l = −2 is a root. Therefore,
26.18 Method to Prove Collinearity l 3 − 6 l − 4 = ( l + 2)( l 2 − 2l − 2)
and l 2 − 2l − 2 = 0 for l = 1± 3
1. Two vectors p and q are collinear if there exists k∈R such that
p = kq. The required value of l are
2. If p ´ q = o , then p , q are collinear.
l 1 = -2; l 2 = 1+ 3 ; l 3 = 1- 3
3. Three points A( a), B( b), C( c ) are collinear if there exists k ∈ R
such that AB = k (BC ), that is, b − a = k (c − b ). Illustration 26.42 If four points, A, B, C and D with position vec-
tors a, b , c and d are coplanar, prove that [ a b c ] = [ b a d ] +
4. If (b − a ) × (c − b ) = o, then A, B, C are collinear.
[ c a d ] + [ a b d ].
5. A( a), B( b), C( c ) are collinear
if there exists scalars l, m, n, (not all
zero) such that la + mb + nc = ox where l + m + n = 0. Solution: If the points A, B, C, D are coplanar, then the vectors
1182 Mathematics Problem Book for JEE
DA = OA − OD = − d + a ; If a , b are two known non-collinear vectors, then a , b , a × b
are three non-coplanar vectors.
DB = OB − OD = − d + b ;
Thus, any vector r = xa + yb + z(a × b ) where x , y , z are
unknown scalars.
DC = OC − OD = − d + c are coplanar.
Illustration 26.44 Let a = i − j , b = j − k , c = k − i and d is a unit
Hence,
vector such that a ⋅ d = 0 = [b c d ] . Then find d .
( − d + a ) ⋅ (( − d + b ) × ( − d + c )) = 0
Solution: Let d = a i + b j + g k.
Then
( − d + a ) ⋅( b × c − b × d − d × c ) = 0 (As d × d = 0 )
a ⋅ d = 0 ⇒ (i - j ) ×(a i + b j + g k ) = 0 ⇒ a − b = 0 ⇒ a = b
⇒ − [ d b c ]+[ a b c ]−[ a b d ]−[ a d c ] = 0
i j k
Therefore,
[b c d ] = 0 ⇒ ( b × c ) ⋅ d = 0 ⇒ 0 1 -1 ⋅ (a i + b j + g k ) = 0
[a b c ]=[d b c ]+[ a b d ]+[ a d c ]
-1 0 1
=[b c d ]+[ a c ]+[ c a d ]
⇒ (i + j + k ) ⋅ (a i + b j + g k ) = 0 ⇒ a + b + g = 0
d
and this is the desired result.
⇒ g = −(a + b ) = −2a ; ( b = a )
Illustration 26.43 Prove that if cosa ≠ 1, cosb ≠ 1 and cosg ≠ 1,
ˆ c = iˆ + jˆ + kˆ cosg | d | = 1⇒ a 2 + b 2 + g 2 = 1 ⇒ a 2 + a 2 + 4a 2 = 1
then the vectors a = iˆ cos a + ˆj + kˆ, b = iˆ + jˆ cosb + k,
1 2
can never be coplanar. ⇒a =± = b and g = ∓
6 6
Solution: Suppose that a , b , c are coplanar. So,
1
Therefore, d = ± ( i + j − 2k ) .
cos a 1 1 6
1 cos b 1 =0
Illustration 26.45 Let the unit vectors a and b be perpendicular
1 1 cosg
and the unit vector c be inclined at an angle q to both a and b. If
( R2 → R2 – R1 and R3 → R3 – R1 ) c = a a + b b + g (a × b ), then
(A) a = b = cosq ,g 2 = cos 2q
cos a 1 1
(B) a = b = cosq , g 2 = − cos 2q
⇒ 1−cos a cos b −1 0 =0
(C) a = cosq , b = sinq , g 2 = cos 2q
1−cos a 0 cosg −1
(D) None of these
or
Solution: We have,
cosa (cosb – 1)(cosg – 1) – (1 – cosa)(cosg – 1) – (1 – cosa) |a| = |b | = 1
(cosb – 1) = 0
a ⋅ b = 0; (as a ⊥ b )
Dividing throughout by (1 – cosa) (1 – cosb )(1 – cosg ), we get
c = a a + b b + g (a × b ) (1)
cos a 1 1
+ + =0
1− cos a 1− cos b 1− cosg Taking dot product by a, we get
1 1 1
⇒ –1 + + + =0 a × c = a | a |2 + b ( a × b ) + g [a a b ]
1− cos a 1− cos b 1− cosg
⇒ | a |⋅| c |cosq = a ⋅1+ 0 + 0
1 1 1
⇒ + + =1
1−cos a 1−cos b 1−cosg ⇒ 1⋅| c |⋅ cosq = a
a b g
⇒ cos ec2 + cos ec2 + cos ec2 = 2 , As | c | = 1. Therefore,
2 2 2 a = cosq
which is not possible as
Taking dot product of Eq. (1) by b, we get
a b g
cosec2 ≥ 1, cosec2 ≥ 1, cosec2 ≥ 1
2 2 2 b ⋅ c = b ⋅ a + b | b |2 + g [b a b ] ⇒ | b || c |cosq = 0 + b ⋅1+ 0
Hence, they cannot be coplanar. Therefore,
b = 1⋅1⋅ cosq = cosq
26.19 Vector Equation 2
| c | = 1 ⇒ a 2 + b 2 + g 2 = 1 ⇒ cos2 q + cos2 q + g 2 = 1
Generally, to solve a vector equation, we express the unknown vec-
Therefore,
tor as a linear combination of three known non-coplanar vectors
and then we determine the coefficients from the given conditions. g 2 = 1− 2 cos2 q = − cos 2q
Chapter 26 | Vector Algebra 1183
So, Therefore,
2
a = b = cosq , g = − cos 2q y = 3( x + 3)
Hence, the correct answer is option (B). This tangent cuts x-axis (that is, y = 0) at ( −3, 0 ). Therefore,
B ≡ ( −3, 0 )
Your Turn 5
OB = −3i + 0. j = −3i ;
1. a × [a × (a × b)] is equal to OA ⋅ AB = OA ⋅(OB − OA) = (i + 12 j ) ⋅( −3i − i − 12 j )
(A) (a × a )⋅(b × a ) (B) a ⋅(b × a ) − b ⋅ (a × b )
(C) [a ⋅(a × b )] a (D) (a ⋅ a )(b × a ) Ans. (D) = (i + 12 j ) ×( -4i - 12 j ) = −4 − 144 = −148
2. [b × c c × a a × b ] is equal to
2. Let u , v , w be such that | u | = 1, | v | = 2, | w | = 3. If the projec-
(A) a × (b × c ) (B) 2[a b c ]
tion v along u is equal to that of w along u and v , w are per-
(C) [a b c ]2 (D) [a b c ] Ans. (C) pendicular to each other, then find | u − v + w |.
3. If a = i + j + k , a ⋅ b = 1 and a ´ b = j - k , then b = Solution: Without loss of generality, we can assume
(A) i (B) i - j + k
v = 2i and w = 3j
(C) 2 j - k (D) 2i Ans. (A)
Let u = xi + y j + zk . Then
4. The point of intersection of ´ a = b ´ a and r ´ b = a ´ b where
r
a = i + j and b = 2i - k is | u |= 1 ⇒ x 2 + y 2 + z 2 = 1 (1)
(A) 3i + j - k (B) 3i - k
Projection of v along u = Projection of w along u
(C) 3i + 2 j + k (D) None of these Ans. (A)
5. Let the vectors a , b , c and d be such that (a × b ) × (c × d) = 0. ⇒ v ⋅ u = w ⋅ u ⇒ 2i ⋅( xi + y j + 2k ) = 3j ⋅( xi + y j + zk )
LetP1 and P2 be planes determined by pair of vectors a , b and
⇒ 2x = 3y ⇒ 3y − 2x = 0
c , d , respectively. Then the angle between P1 and P2 is
Now,
p
(A) 0° (B)
4 | u − v − w | = | xi + y j + zk − 2i + 3j |
p p
(C) (D) Ans. (A) = | ( x - 2) i$ + ( y + 3) j$ + zk$ | = ( x − 2)2 + ( y − 3)2 + z 2
3 2
y = x 2 + x + 10 (1) Solution:
a ⋅(a × b ) = a ⋅ c = 0
When x = 1. Then
Also,
y = 12 + 1+ 10 = 12
b ⋅ (a × b ) = b ⋅ c = 0
Therefore,
and b (b × c ) = a ⋅ b = 0
A ≡ (1, 12)
So, a , b , c are mutually perpendicular vectors, therefore
⇒ OA = i +12 j
| a × b | = | a | | b | = | c | and | b || c | = | a | ⇒ | b | = 1
From Eq. (1), we get ⇒ |a|=|c |
dy Hence, the correct answer is option (C).
= 2x +1
dx
4. Let A, B , C are three vectors respectively given by 2iˆ + kˆ , iˆ +
Equation of tangent at A is ĵ + kˆ and 4 iˆ − 3 ˆj + 7kˆ. Then vector R, which satisfies the relation
⎛ dy ⎞ R × B = C × B and R ⋅ A = 0 is
y − 12 = ⎜ ⎟ ( x − 1)
⎝ dx ⎠ (1, 12 ) (A) 2iˆ − 5 ˆj + 2kˆ (B) − iˆ + 4 ˆj + 2kˆ
⇒ y − 12 = (2 × 1+ 1)( x − 1) ⇒ y − 12 = 3 x − 3 (C) − iˆ − 8 ˆj + 2kˆ (D) None of these
1184 Mathematics Problem Book for JEE
Solution: We have n n n n
∑ ( PAi )2 = ∑ (OAi )2 + ∑ (OP )2 − ∑ 2OP ⋅ OAi
R × B = C × B and R ⋅ A = 0
i =1 i =1 i =1 i =1
Therefore,
n
= nR 2 + nr 2 − 2OP ⋅ ∑ OAi
A × ( R × B ) = A × (C × B ) ⇒ ( A ⋅ B ) R − ( A ⋅ R ) B = ( A ⋅ B ) C − ( A ⋅ C ) B
i =1
2
2
⇒ (2 + 1) R = 3C − (8 + 7) B ⇒ R = C − 5B = − iˆ − 8 ˆj + 2kˆ = n(R + r ) − 2OP ⋅ (0 )
Hence, the correct answer is option (C). Now
5. The points with position vectors 60iˆ + 3 ˆj , 40iˆ − 8 ˆj and aiˆ − 52 ˆj a p a p
R= cosec , r = cot
are collinear if 2 n 2 n
(A) a = − 40 (B) a = 40 O
(C) a = 20 (D) None of these
Solution: The points are collinear, therefore p /n
R
r
l (60iˆ + 3 ĵ ) + m (40iˆ− 8 ĵ ) + g (aiˆ − 52 ĵ ) = 0
with l + m + g = 0,
3l − 8m − 52g = 0 Ai a/2 Ai +1
60l + 40m + g a = 0
Figure 26.34
For non-zero set (l, m, g ),
⎛ 2p ⎞
60 40 a 2 2a2 ⎛ 2p 2p⎞ a2 ⎜ 1+ cos n ⎟
R + r = ⎜ cosec + cot ⎟ =
3 −8 −52 = 0 ⇒ a = −40 4 ⎝ n n ⎠ 4 ⎜ sin2 p ⎟
⎜⎝ ⎟
1 1 1 n ⎠
⎛ 2p ⎞
Hence, the correct answer is option (A).
n
na2 ⎜ 1+ cos n ⎟
⇒ ∑ ( PAi ) = 2
6. Let OA and OB are two vectors such that | OA + OB | = | OA + 2 OB |. 4 ⎜ sin2 p ⎟
i =1 ⎜⎝ ⎟
Then n ⎠
(A) ∠BOA = 90° (B) ∠BOA > 90°
(C) ∠BOA < 90° (D) 60° ≤ ∠BOA ≤ 90° 8. In a triangle PQR, S and T are points on QR and PR, respectively,
such that QS = 3SR and PT = 4RT. Let M be the point of inter-
Solution: Given
section of PS and QT. Then determine the ratio QM:MT using
| OA + OB | = | OA + 2 OB | vector methods.
On squaring, we get Solution: See Fig. 26.35.
Let QM:MT = l :1 and PM:MS = m :1 and QP = a and QR = b . Then
(OA)2 + (OB )2 + 2OA ⋅ OB = (OA)2 + 4(OB )2 + 4OA ⋅ OB
⇒ cosq < 0 ⇒ q > 90° ⇒ ∠BOA > 90°
l ⎛ 4b + a ⎞
QM = (1)
Hence, the correct answer is option (B). l + 1 ⎜⎝ 5 ⎟⎠
7. A circle is inscribed in an n-sided regular polygon A1A2… An
P(a)
having each side of a unit length. For any arbitrary point P on
⎛ 2p ⎞ r r
a2 ⎜ 1+ cos n ⎟
n 4 ⎛4b +a ⎞
the circle, prove that ∑ ( PAi ) = n ⎜
2
⎟.
m
T ⎜ 5 ⎟
4 2p M ⎝ ⎠
i =1 ⎜⎝ sin ⎟
n ⎠ l 1
1 1
Solution: See Figs. 26.33 and 26.34. Q R(b)
(0) 3 S 1
Let the centre of the incircle be the reference point. Then
Figure 26.35
PAi = OAi − OP
Also,
PAi ⋅ PAi = (OAi − OP ) (OAi − OP )
3
QS = b
( PAi )2 = (OAi )2 + (OP )2 − 2OP ⋅ OAi 4
So,
3
m 4 b + a
O QM = (2)
m +1
P From Eqs. (1) and (2), we get
Ai Ai +1 l ⎛ 4b + a ⎞ 3 m b + 4 a
=
Figure 26.33 l + 1 ⎜⎝ 5 ⎟⎠ 4( m + 1)
Chapter 26 | Vector Algebra 1185
On comparing, we get Solution: ABC and PQR are the given triangles. Let the
l perpendicular from A, B, C to the sides QR, PR and PQ intersect at
1
= (3)
O. Take O as the initial point. Let a , b , c , p , q , r be the position
m + 1 5( l + 1)
vector of A, B, C, P, Q and R, respectively. Since OA, OB and OC are
and perpendicular to QR, RP and PQ, so
4l 3m
= (4) a . (r - q ) = 0 , b . ( p - r ) = 0 and c . (q - p ) = 0
5( l + 1) 4( m + 1)
Let the perpendicular from P and Q on BC and CA, respectively,
16 15
So, m = and l = . Hence, QM:MT = 15:4 intersect at the point X whose position vector is taken as x. It
3 4
implies
9. Two systems of forces P, Q, R and P′, Q′, R′ are along the side BC,
CA, AB of a ΔABC. Prove that the resultant will be parallel if ( p - x ) . (c - b ) = 0 and (q - x ) . (a - c ) = 0
sin A sin B sin C ⇒ p . (c - b ) = x . (c - b ) and q . (a - c ) = x . (a - c )
P Q R =0 Adding, we have
P’ Q’ R’
x . (a - b ) = p . c - p . b + q . a - q c = c . ( p - q ) - p . b + q . a
Solution: See Fig. 26.36.
= -p . b + q .a
Unit vector along Q = cos(p − C ) iˆ + sin (p − C ) ˆj
= -b . r + a . r = r . ( a - b ) Þ ( r - x ) . ( a - b ) = 0
= − cos C iˆ + sinC ˆj
Therefore, XR is perpendicular to AB.
Unit vector along R = cos (p + B ) iˆ + sin (p + B ) ˆj Hence, perpendicular from R to AB passes through X.
= − cos B i − sin B j
If S and S′ be resultant in the two cases, then Previous Years’ Solved JEE Main/AIEEE
y Questions
A
1. The resultant of two forces P N and 3 N is a force of 7 N. If the
R
Q
direction of 3 N force were reversed, the resultant would be
19N. The value of P is
p −C (A) 5 N (B) 6 N
B P C x
(C) 3N (D) 4N [AIEEE 2007]
Solution: See Fig. 26.37. Originally,
Figure 26.36
72 = P 2 + 32 + 2 × 3 × P cosq (1)
S = P iˆ + Q ( − cos C iˆ + sin C ˆj ) + R ( − cos B iˆ − sin B ˆj ) (1)
Later,
Similarly,
S′ = (P′ − Q′ cosC − R′ cosB) iˆ + (Q′ sinC − R′ sinB) ĵ (2) ( 19 )2 = P 2 + ( −3)2 + 2 × ( −3) × P cosq (2)
If q and q ′ be the angles made by the resultant with x-axis, then
Adding we get,
Q sin C − R sin B
tanq = 68 = 2P 2 + 18 ⇒ P = 5
P − Q cos C − R cos B
and P
Q ’sin C − R ’sin B
tanq ′ =
P ’− Q ’cos C − R ’cos B
7
If the resultant are parallel, then q = q ′. Therefore, √19
Q sin C − R sin B Q ’sin C − R ’sin B q
tanq = tanq ′ ⇒ =
P − Q cos C − R cos B P ’− Q ’cos C − R ’cos B −3 O 3
On solving we get
(PQ′ − P′Q) sinC + (RP′ − R′P) sinB + (QR′ − Q′R) sinA = 0 Figure 26.37
x x − 2 −1 Also, c is normal to b, so
1 1 1 = 0 ⇒ 3 x + 2 − x + 2 = 0 ⇒ 2 x = −4 ⇒ x = −2 b ⋅ c = 0 ⇒ ( xiˆ + yjˆ + zkˆ ) ⋅ (iˆ − ˆj − kˆ ) = 0
1 −1 2
⇒ x−y−z=0 (1)
Hence, the correct answer is option (D).
It is given that a ⋅ b = 3, so
3. If û and v̂ are unit vectors and q is the acute angle between ( ˆj − kˆ ) ⋅ ( xiˆ + yjˆ + zkˆ ) = 3
them, then 2uˆ × 3vˆ is a unit vector for
(A) exactly two values of q ⇒ y−z=3 (2)
(B) more than two values of q
And, c = b × a
(C) no value of q iˆ − ˆj − kˆ = ( xiˆ + yjˆ + zkˆ ) × ( ˆj − kˆ )
(D) exactly one value of q [AIEEE 2007] ˆj kˆ
iˆ
Solution: = x y z
| 2uˆ × 3vˆ |= 1⇒ 6 | uˆ || vˆ || sinq |= 1⇒ 6(1)(1)sinq wˆ = 1 0 1 −1
For 6sinq wˆ to be unit,
= ( − y − z )iˆ + xjˆ + xkˆ
1 From this, we get, (y + z) = −1. (3)
| 6 sinq | = 1⇒| sinq | =
6 From Eqs. (2) and (3) we get x = −1, y = 1 and z = −2. So,
At only one value of q . Hence, there is exactly one value of q for
b = − iˆ + ˆj − 2kˆ
which 2uˆ × 3vˆ is a unit vector.
Hence, the correct answer is option (D). Hence, the correct answer is option (D).
7. If the vectors a = iˆ − ˆj + 2kˆ , b = 2iˆ + 4 ˆj + kˆ and c = l iˆ + ˆj + m kˆ
vector a =
4. The a iˆ + 2 ˆj + b kˆ lies in the plane of the vectors
are mutually orthogonal, then (l, m) =
b = iˆ + ˆj and c = ˆj + kˆ and bisects the angle between b and
c . Then which one of the following gives possible values of (A) (2, −3) (B) (−2, 3)
a and b ? (C) (3, −2) (D) (−3, 2) [AIEEE 2010]
(A) a = 2, b = 2 (B) a = 1, b = 2 Solution:
(C) a = 2, b = 1 (D) a = 1, b = 1
[AIEEE 2008] a × b = 0 , b × c = 0 , c × a = 0 Þ 2 l + 4 + m = 0 , l - 1+ 2 m = 0
Solution: Solving, we get l = −3, m = 2.
⎛ iˆ + 2 ˆj + kˆ ⎞ Hence, the correct answer is option (D).
a = l ( bˆ + cˆ ) ⇒ a ˆi + 2 ˆj + b kˆ = l ⎜ ⎟
⎝ 2 ⎠ 1 1
8. If a = (3iˆ + kˆ ) and b = (2iˆ + 3 ˆj − 6kˆ ), then the value of
⇒ l = 2a , l = 2 and l = 2 b 10 7
(2a − b ) ⋅ [(a × b ) × (a + 2b )] is
⇒ a = 1 and b = 1 (A) –3 (B) 5
(C) 3 (D) −5 [AIEEE 2011]
Hence, the correct answer is option (D).
Solution:
5. The non-zero
vectors a , b and c are related by a = 8b and G G G G G G G G G G G G G G
(2a − b ) ⋅ [(a × b ) × (a + 2b )] = (2a − b ) ⋅ [(a × b ) × a + 2(a × b ) × b ]
c = −7b . Then the angle between a and c is
G G G G G G G
(A) 0 (B) p /4 = (2a − b ) ⋅ [ − a × (a × b ) + 2( −b × (a × b ))]
(C) p /2 (D) p [AIEEE 2008] G G G G G G G G G G G G G G
= (2a − b ) ⋅ [{ −(a ⋅ b )a − (a ⋅ a )b } + 2{ −(b ⋅ b )a − (b ⋅ a )b }]
G G G G G G G G G
Solution: As a = 8b , we have c = −7b . Therefore, a and b are like = (2a − b ) ⋅ [ −(a ⋅ b )a + a2b − 2b2a − 2(b ⋅ a )b ]
vectors and b and c are unlike. This implies that a and c will be G G G G G G G G G G G G G G
= (2a − b ) ⋅ [ −(a ⋅ b )a + b − 2a − 2((a ⋅ b )b ] = (2a − b ) ⋅ [b − 2a ]
unlike. Hence, angle between a and c is equal to p .
G G G G G G
Hence, the correct answer is option (D). = −( 2 a − b ) ⋅ ( 2 a − b ) = − { 4 a 2 + b 2 − 2 a ⋅ b }
= − { 4 + 1− 0} = −5
6. Let a = ˆj − kˆ and c = iˆ − ˆj − kˆ. Then vector b satisfying a × b + c = 0
and a ⋅ b = 3 is Hence, the correct answer is option (D).
9. The vector a and bare not perpendicular
and c and d are two
(A) 2iˆ − ˆj + 2kˆ (B) iˆ − ˆj − 2kˆ
vectors satisfying, b × c = b × d and a ⋅ d = 0. Then the vector d
(C) iˆ + ˆj − 2kˆ (D) − iˆ + ˆj − 2kˆ [AIEEE 2010] is equal to
⎛ a⋅c ⎞ ⎛ b ⋅ c ⎞
Solution: Let b = xiˆ + yjˆ + zkˆ. It is given that (A) c + ⎜ ⎟ b (B) b + ⎜ ⎟ c
⎝ a ⋅b ⎠ ⎝ a ⋅b ⎠
a × b + c = 0 ⇒ c = −a × b = b × a
Chapter 26 | Vector Algebra 1187
⎛ a⋅c ⎞ ⎛ b ⋅ c ⎞
12. If the vectors AB = 3i + 4 k and AC = 5i − 2 j + 4 k are the sides
(C) c − ⎜ ⎟ b (D) b − ⎜ ⎟ c
⎝ a ⋅b ⎠ ⎝ a ⋅b ⎠ of a triangle ABC, then the length of the median through A is
[AIEEE 2011] (A) 72 (B) 33
Solution: (C) 45 (D) 18 [JEE MAIN 2013]
b ×c =b ×d Solution: From the following figure, we see that (Fig. 26.39),
AB + AC
⇒ a × (b × c ) = a × (b × d ) ⇒ (a ⋅ c )b − (a ⋅ b )c = (a ⋅ d )b − (a ⋅ b )d AM = ⇒ AM = 4i −
j + 4
k
2
⇒ (a ⋅ c )b − (a ⋅ b )c = −(a ⋅ b )d
⎛ a ⋅ c ⎞ Therefore,
Therefore, d = c − ⎜ ⎟ b . | AM |= 16 + 16 + 1 = 33
⎝ a ⋅b ⎠
Hence, the correct answer is option (C). C
10. Let â and b̂ be two unit vectors. If the vectors c = aˆ + 2bˆ and
d = 5aˆ − 4bˆ are perpendicular to each other, then the angle M
between â and b̂ is
p p
(A) (B)
6 2
p p A B
(C) (D) [AIEEE 2012]
3 4
Figure 26.39
Solution:
Hence, the correct answer is option (B).
c ⋅ d = 0 ⇒ 5 | a |2 + 6a ⋅ b − 8 | b |2 = 0
13. If [a × b b × c c × a ] = l [a b c ]2, then l is equal to
1
⇒ 6a ⋅ b = 3 ⇒ a ⋅ b = (A) 0 (B) 1
2 (C) 2 (D) 3
p
So, angle between a and b is . [JEE MAIN 2014 (OFFLINE)]
3
Hence, the correct answer is option (C). Solution:
[a ´ b b ´ c c ´ a ] = (a ´ b ) × [(b ´ c ) ´ (c ´ a )] = (a ´ b ) × [(b ´ c ) ´ c ´ a ]
11. Let ABCD be a parallelogram such that AB = q , AD = p and
∠BAD be an acute angle. If r is the vector that coincides with = ( a ´ b ) × [(b ´ c × a )c - (b ´ c × c )a ]
the altitude directed from the vertex B to the side AD, then r = ( a ´ b ) × [(b ´ c × a )c - 0]
is given by
3( p ⋅ q ) ⎛ p⋅q⎞ = (a ´ b × c )(a ´ b × c ) = (a ´ b × c ) = [ab c ]2
(A) r = 3q − p (B) r = − q + ⎜ ⎟ p
( p ⋅ q) ⎝ p⋅ p⎠ Therefore, l = 1.
Hence, the correct answer is option (B).
⎛ p⋅q⎞ 3( p ⋅ q )
(C) r = q − ⎜ ⎟ p (D) r = −3q + p
⎝ p⋅ p⎠ ( p ⋅ p)
14. If | a | = 2, | b | = 3 and | 2a − b | = 5 , then | 2a + b | equals
[AIEEE 2012]
(A) 17 (B) 7
Solution: See Fig. 26.38. AE is the vector component of q on p. So,
( p ⋅ q )
(C) 5 (D) 1
AE = p [JEE MAIN 2014 (ONLINE SET-1)]
( p ⋅ p)
Solution:
Therefore, from ΔABE;
| 2a − b |2 = 25 ⇒ 4 a2 + b2 − 4 a ⋅ b = 25
AB + BE = AE
⇒ 16 + 9 − 4 a ⋅ b = 25 ⇒ a ⋅ b = 0
( p ⋅ q )p ( p ⋅ q) Now,
⇒ q + r = ⇒ r = −q + p
( p ⋅ p) ( p ⋅ p)
| 2a + b |2 = (2a + b ) ⋅ (2a + b ) = 4 a2 + b2 + 4 a ⋅ b = 4( 4 ) + 9 + 4(0 ) = 25
D C
Therefore, | 2a + b |= 5.
p Hence, the correct answer is option (C).
E
r 15. Two ships A and B are sailing straight away from a fixed point
O along routes such that ∠AOB is always 120°. At a certain
A B instance, OA = 8 km, OB = 6 km and the ship A is sailing at
q the rate of 20 km/h while the ship B sailing at the rate of 30
Figure 26.38 km/h. Then the distance between A and B is changing at the
Hence, the correct answer is option (B). rate (in km/h):
1188 Mathematics Problem Book for JEE
(A) 55 (B) 51 21. Let a , b and c be three unit vectors such that
3
(C) 43 (D) 37 a × (b × c ) = (b + c ). If b is not parallel to c , then the angle
2
[JEE MAIN 2015 (ONLINE SET-1)]
between a and b is
Solution: 5p 3p
(A) (B)
| a | = 1, | b | = 1, | a + b | = 3 , c = a + 2b + 3(a × b ) 6 4
p 2p
Since, (C) (D)
2 3
| a + b |2 = 3 ⇒ (a + b ) ⋅ (a + b ) = 3 [JEE MAIN 2016 (OFFLINE)]
1 Solution: It is given that a , b and c are three unit vectors such
⇒ | a |2 + | b |2 + 2a ⋅ b = 3 ⇒ a ⋅ b = ⇒ 1+ 1+ 2 cosq = 3
2 that
1
⇒ cosq = 3
2 a × (b × c ) = (b + c )
2
⇒ q = 60° = Angle between a and b
Therefore,
⇒ a × b = | a || b | sinq ⋅ xˆ , x̂ = unit vector 3
(a ⋅ c )b − (a ⋅ b )c = (b + c )
Vector perpendicular to the plane containing a and b is 2
3 comparing both sides, we get
a×b = xˆ
2 3 3
Therefore, a⋅c = and a ⋅ b = −
2 2
⎛ 3 3 ⎞ ⎛ 3 3 ⎞
| c |2 = ⎜ a + 2b + xˆ ⎟ ⋅ ⎜ a + 2b + xˆ 3 3
⎝ 2 ⎠ ⎝ 2 ⎟⎠ Here, cos a = and cos b = − , where a is angle between
2 2
⎛ 9 × 3⎞ a and c ; b is the angle between a and b. Therefore,
= 1+ 2 × (2a ⋅ b ) + 4 + ⎜ ˆ = xˆ ⋅ b = 0)
⎝ 4 ⎟⎠
(Since, na
5p
b=
⎛ 1⎞ 27 55 55 6
= 1+ 4 ⎜ ⎟ + 4 + = ⇒ |c |= ⇒ 2 | c | = 55
⎝ 2⎠ 4 4 2 Hence, the correct answer is option (A).
Hence, the correct answer is option (A). 22. In a triangle ABC, right-angled at the vertex A, if the position
vectors of A, B and C are, respectively, 3iˆ + ˆj − kˆ , − iˆ + 3 ˆj + pkˆ
20. In a parallelogram ABCD, | AB | = a, | AD | = b and | AC | = c ,
and 5iˆ + qjˆ − 4 kˆ, then the point (p, q) lies on a line
then DB ⋅ AB has the value (A) making an obtuse angle with the positive direction of
1 2 2 2 1 2 2 2 x-axis.
(A) (a − b + c ) (B) (a + b − c ) (B) parallel to x-axis.
2 4
(C) parallel to y-axis.
1 1
(C) (b2 + c 2 − a2 ) (D) (a2 + b2 + c 2 ) (D) making an acute angle with the positive direction of
3 2 x-axis.
[JEE MAIN 2015 (ONLINE SET-2)] [JEE MAIN 2016 (ONLINE SET-1)]
Solution: See Fig. 26.41. Solution: See Figs. 26.42 and 26.43. From the triangle ABC as
D C shown here, we can write as
AB = −4 iˆ + 2 jˆ + ( p + 1)kˆ
AC = 2iˆ + (q − 1) ˆj − 3kˆ
A(3iˆ + jˆ − k)
ˆ
A B
Figure 26.41
| AB | = a, | AD | = b and | AC | = c ,
C B(−iˆ + 3jˆ + pk)
ˆ
AB + AD = AC Þ | AB |2 + | AD |2 + 2 AB × AD = | AC |2 (5iˆ + qjˆ − 4k)
ˆ
Þ a2 + b2 + 2 AB( AB + BD ) = c 2
Figure 26.42
Þ a2 + b2 + 2a2 + 2 AB × BD = c 2 Þ 3a2 + b2 - c 2 = 2 AB × DB
As AB is perpendicular to AC . Therefore,
1
Þ AB × DB = (3a2 + b2 - c 2 ) AB ⋅ AC = 0
2
There is no correct option for this question. ⇒ −8 + 2(q − 1) − 3( p + 1) = 0
1190 Mathematics Problem Book for JEE
⇒ −8 + 2q − 2 − 3 p − 3 = 0
Previous Years’ Solved JEE Advanced/
⇒ 3 p − 2q + 13 = 0
p q
IIT-JEE Questions
⇒ + =1
( −13/3) (13/2) 1. The number of distinct real values of l, for which the vectors
y − l 2iˆ + ˆj + kˆ , iˆ − l 2 ˆj + kˆ and iˆ + ˆj − λ 2k̂ are coplanar, is
(A) zero (B) one
13 (C) two (D) three [IIT-JEE 2007]
2 Solution: If the vectors are coplanar
x
−13 −l 2 1 1
3 1 −l 2
1 =0
Figure 26.43 1 1 −l 2
Therefore, the point (p, q) makes acute angle with positive direc-
tion of x-axis. ⇒ l 6 − 3l 2 − 2 = 0
Hence, the correct answer is option (D). ⇒ ( l 2 + 1)2 ( l 2 − 2) = 0
23. Let ABC be a triangle whose circumcentre is at P. If the position ⇒l =± 2
a +b + c
vectors of A, B, C and P are a , b , c and , respectively, Hence, the correct answer is option (C).
4
then the position vector of the orthocentre of this triangle is 2. Let the vectors PQ , QR , RS , ST , TU and UP represent the sides
of a regular hexagon.
⎛ a +b + c ⎞
(A) − ⎜ (B) a + b + c Statement-1: PQ × (RS + ST ) ≠ 0.
⎝ 2 ⎟⎠
because
(a + b + c ) Statement-2: PQ × RS = 0 and PQ × ST ≠ 0.
(C) (D) 0
2 (A) Statement-1 is True, Statement-2 is True; Statement-2 is a
[JEE MAIN 2016 (ONLINE SET-2)] correct explanation for Statement-1
Solution: See Figs. 26.44 and 26.45. Let ABC be a triangle whose (B) Statement-1 is True, Statement-2 is True; Statement-2 is
circumcentre is at point P. NOT a correct explanation for Statement-1
(C) Statement-1 is True, Statement-2 is False
A(a)
(D) Statement-1 is False, Statement-2 is True [IIT-JEE 2007]
Solution: See Fig. 26.46.
G ⋅P Statement-1: PQ × (RS + ST ) ≠ 0. Since PQ is not parallel to RT , we
get
B(b) C(c)
PQ × (RT )
Figure 26.44
That is, Statement-1 is true.
a+b+c
P= (circumcentre) T S
4
a+b+c
G= (centroid)
3 U R
O
P Q
2 1 Figure 26.46
Q P
G
Figure 26.45 PQ × (RT ) ≠ 0
The orthocentre (Q) is
Statement-2:
PQ × RS = 0 and PQ × ST ≠ 0. Since PQ is not parallel
2OP + 1OQ
to RS , Statement-2 is false.
= OG
3 Hence, the correct answer is option (C).
3. Let a , b , c be unit vectors such that a + b + c = 0. Which one of
a + b + c
Therefore,
the following is correct?
OQ =
2 (A) a × b = b × c = c × a = 0
Hence, the correct answer is option (C). (B) a × b = b × c = c × a ≠ 0
Chapter 26 | Vector Algebra 1191
So,
(C) a × b = b × c = a × c ≠ 0
1
(D) a × b , b × c c × a are mutually perpendicular Volume = [aˆ bˆ cˆ ] =
[IIT-JEE 2007] 2
Hence, the correct answer is option (A).
Solution: We have
5. Let two non-collinear unit vectors â and b̂ form an acute angle.
a+b+c = 0 (1)
A point P moves so that at any time t the position vector OP
Taking cross-product by a on both sides of Eq. (1), we get (where O is the origin) is given by aˆ cos t + bˆ sin t. When P is far-
a × (a + b + c ) = 0 thest from origin O, let M be the length of OP and û be the unit
vector along OP . Then
⇒a×a+a×b +a×c = 0
G G G G
0+a ×b−c ×a = 0 aˆ + bˆ
(A) uˆ = and M = (1+ aˆ ⋅ bˆ )1/2
| aˆ + bˆ |
⇒a×b = c ×a (2)
Taking cross-product by b on both sides of Eq. (1), we get aˆ − bˆ
(B) uˆ = and M = (1+ aˆ ⋅ bˆ )1/2
| aˆ − bˆ |
b × (a + b + c ) = 0
aˆ + bˆ
⇒b ×a+b ×b +b ×c = 0 (C) uˆ = and M = (1+ 2aˆ ⋅ bˆ )1/2
| aˆ + bˆ |
⇒ −a × b + 0 + b × c = 0 aˆ − bˆ
(D) uˆ = and M = (1+ 2aˆ ⋅ bˆ )1/2 [IIT-JEE 2008]
⇒a×b = b ×c (3) | aˆ − bˆ |
and (a ´ b ) || (c ´ d ) (B) Let y =
3x
. Then
p
Since a × c = 1/ 2 and b || d , we get
1 ⎡p ⎤
≤ y ≤ 3 ∀x ∈ ⎢ , p ⎥
2 ⎣6 ⎦
c ×d ≠1 Now,
f(y) = [2y] cos[y]
Therefore, we conclude that the vectors b and d are non-parallel. 1 3
Hence, the correct answer is option (C). Critical points are y = , y = 1, y = , y = 3 .
2 2
7. Let P(3, 2, 6) be a point in space and Q be a point on the line ⎧p p p ⎫
So, points of discontinuity ⎨ , , , p ⎬.
r = (iˆ − ˆj + 2kˆ ) + m ( −3iˆ + ˆj + 5kˆ ). Then the value of m for which ⎩6 3 2 ⎭
the vector PQ is parallel to the plane x − 4y + 3z = 1 is 1 1 0
1 1 (C) 1 2 0 = p ⇒ volume of parallelepiped = p
(A) (B) −
4 4 1 1 p
1 1
(C) (D) − [IIT-JEE 2009] (D) | a + b | = 3
8 8
Solution: Any point on the line can be taken as ⇒ 2 + 2 cos a = 3
⇒ 2 + 2 cos a = 3
Q ≡ {(1− 3 m ), ( m − 1), (5 m + 2)} p
⇒a =
PQ = { −3 m − 2, m − 3, 5 m − 4 } 3
Now, Hence, the correct matches are (A)ã(Q, S), (B)ã(P, R, S, T),
1( −3 m − 2) − 4( m − 3) + 3(5 m − 4 ) = 0 (C)ã(T), (D)ã(R).
⇒ −3 m − 2 − 4 m + 12 + 15 m − 12 = 0 9. Let P, Q, R and S be the points on the plane with position vec-
tors −2iˆ − ˆj , 4 iˆ , 3iˆ + 3 ˆj and −3iˆ + 2 ˆj respectively. The quadri-
8 m = 2 ⇒ m = 1/ 4 lateral PQRS must be a
Hence, the correct answer is option (A). (A) parallelogram, which is neither a rhombus nor a rectangle
(B) square
8. Match the statements/expressions in Column I with the values
(C) rectangle, but not a square
given in Column II.
(D) rhombus, but not a square [IIT-JEE 2010]
Column I Column II
Solution: See Fig. 26.47. Evaluating mid-point of PR and QS which
p
(A) Root(s) of the expression 2sin2q + sin22q (P) ⎡ iˆ ⎤
6 gives M = ⎢ + ˆj ⎥, same for both.
=2 2
⎣ ⎦
(B) Points of discontinuity of the function p
(Q) PQ = SR = 6iˆ + ˆj
é6x ù é 3x ù 4
f ( x ) = ê ú cos ê ú , where [y] denotes
ëp û ëp û PS = QR = − i + 3 ˆj
the largest integer less than or equal to y ⇒ PQ ⋅ PS ≠ 0
PQ || SR , PS || QR and | PQ | = | SR |, | PS | = | QR |
(C) Volume of the parallelopiped with p
its edges represented by the vectors (R) Hence, PQRS is a parallelogram but not rhombus or rectangle.
3
iˆ + ˆj , iˆ + 2 ˆj and iˆ + ˆj + p kˆ S(−3iˆ + 2jˆ ) R(3iˆ + 3jˆ )
(D) Angle between vectors a and b where p
(S)
a , b and c are unit vectors satisfying 2
M
a + b + 3c = 0
(T) p
P(−2iˆ − jˆ ) Q(4iˆ)
Figure 26.47
[IIT-JEE 2009] Hence, the correct answer is option (A).
Solution: iˆ − 2 ˆj
(A) 2sin2q + 4sin2q cos2q = 2 10. If a and b are vectors in space given by a = and
5
sin2q + 2sin2q (1 – sin2q ) = 1 2iˆ + ˆj + 3kˆ
1 b= , then the value of (2a + b ) ⋅ [(a × b ) × (a − 2b )] is
3 sin2 q − 2 sin4 q − 1 = 0 ⇒ sinq = ± , ±1 14
2 [IIT-JEE 2010]
p p Solution:
⇒q = ,
4 2 E = (2a + b ) ⋅ [2 | b |2 a − 2(a ⋅ b )b − (a ⋅ b )a + | a |2 b ]
Chapter 26 | Vector Algebra 1193
2−2 13. The vector(s) which is/are coplanar with vectors iˆ + ˆj + 2kˆ and
a ⋅b = =0
70 iˆ + 2 ˆj + kˆ, and perpendicular to the vector iˆ + ˆj + kˆ is/are
|a|=1
G (A) ĵ − kˆ (B) − iˆ + ˆj
|b|=1
(C) iˆ − ˆj (D) − ĵ + kˆ [IIT-JEE 2011]
ˆ ˆ
E = (2a + b ) ⋅ [2 | b |2 a + | a |2 b ] ˆ ˆ
Solution: Let a = i + j + 2k , b = iˆ + 2 ˆj + k and c = iˆ + ˆj + kˆ.
= 4 | a |2 | b |2 +2 | a |2 (a ⋅ b ) + 2 | b |2 (b ⋅ a ) + | a |2 | b |2
Any vector in the plane of iˆ + ˆj + 2kˆ and iˆ + 2 ˆj + kˆ is
= 5 | a |2 | b |2 = 5
r = l a + mb
Hence, the correct answer is (5).
= l (iˆ + ˆj + 2kˆ ) + m (iˆ + 2 ˆj + kˆ )
11. Two adjacent sides of a parallelogram ABCD are given
by AB = 2iˆ + 10 ˆj + 11kˆ and AD = − i + 2 ˆj + 2kˆ. The side AD = ( l + m )iˆ + ( l + 2 m ) ˆj + (2l + m )kˆ
is rotated by an acute angle a in the plane of the paral- Also, r ⋅c = 0
lelogram so that AD becomes AD'. If AD' makes a right ⇒ ( l + m ) ⋅ 1+ ( l + 2 m ) ⋅ 1+ ( 2 l + m ) ⋅ 1 = 0
angle with the side AB, then the cosine of the angle a is ⇒ 4l + 4 m = 0
given by ⇒l+m =0
8 17 Now,
(A) (B)
9 9
[r a b ] = 0
1 4 5 So, vectors ĵ − kˆ and − ĵ + kˆ satisfy this.
(C) (D) [IIT-JEE 2010]
9 9
Hence, the correct answers are options (A) and (D).
Solution: r
14. Let a = −iˆ − kˆ , b = −iˆ + jˆ and c = iˆ+ 2 ˆj + 3kˆ be three given
AD = AB × (AB × AD ) = 5(61iˆ − 10jˆ − 21k)
vectors. If r is a vector such that r × b = c × b and r ⋅ a = 0, then
the value of r ⋅ b is _____. [IIT-JEE 2011]
| AD ′ ⋅ AD | 17
Þcosa = = Solution:
| AD ′ || AD | 9
Hence, the correct answer is option (B). r ×b = c ×b
Taking cross with a
12. Let a = iˆ + ˆj + kˆ , b = iˆ − ˆj + kˆ and c = iˆ − ˆj − kˆ be three vectors.
A vector v in the plane of a and b, whose projection on c is a × (r × b ) = a × (c × b )
1 (a ⋅ b )r − (a ⋅ r )b = a × (c × b )
, is given by
3
Using a ⋅ r = 0 and a ⋅ b = 1, we get
(A) iˆ − 3 ˆj + 3kˆ (B) −3iˆ − 3 ˆj + kˆ
r = −3iˆ + 6 ˆj + 3kˆ
(C) 3iˆ − ˆj + 3kˆ (D) iˆ + 3 ˆj − 3kˆ
r ⋅b = 3 + 6 = 9
[IIT-JEE 2011]
Hence, the correct answer is (9).
Solution: 2 2
r r r 15. If a , b and c are unit vectors
2 satisfying | a − b | + | b − c | +
v =l a +m b | c − a | = 9 , then | 2a + 5b + 5c | is _____.
= l (iˆ + ˆj + kˆ ) + m (iˆ − ˆj + kˆ ) [IIT-JEE 2012]
Solution:
= (l + m ) iˆ + (l − m ) jˆ + (l + m ) k̂
| a − b |2 + | b − c |2 + | c − a |2 = 3(| a |2 + | b |2 + | c |2 )− | a + b + c |2
Projection of v on c,
r r
v ⋅c 1 [( l + m )iˆ + ( l − m ) ˆj + ( l + m )kˆ ] ⋅ (iˆ − ˆj − kˆ ) 1 ⇒ 3 × 3 − | a + b + c |2 = 9
r = ⇒ =
|c | 3 3 3 ⇒| a + b + c | = 0 ⇒ a + b + c = 0
⇒ l + m − l + m − l − m = 1⇒ m − l = 1⇒ l = m − 1 ⇒ b + c = −a
r
v = ( m − 1)(iˆ + ˆj + kˆ ) + m (iˆ − ˆj + kˆ ) = m (2iˆ + 2kˆ ) − iˆ − ˆj − kˆ ⇒| 2a + 5(b + c )| = | −3a | = 3 | a |= 3
r
v = (2 m − 1)iˆ − ˆj + (2 m − 1)kˆ Hence, the correct answer is (3).
At m = 2, 16. If a and b are vectors such that | a + b | = 29 and
r
v = 3iˆ − ˆj + 3kˆ a × (2iˆ + 3 ˆj + 4 kˆ ) = (2iˆ + 3 ˆj + 4 kˆ ) × b, then a possible value of
Hence, the correct answer is option (C). (a + b ) ⋅ ( −7iˆ + 2 ˆj + 3kˆ ) is
1194 Mathematics Problem Book for JEE
Now
(R) We have | a × b | = 40. Therefore,
a ⋅ b = (a ⋅ y )(b ⋅ z ){ y ⋅ z − y ⋅ x − z ⋅ z + z ⋅ x }
1
A = | (2a + 3b ) × (a − b )| = (a ⋅ y )(b ⋅ z ){1− 1 − 2 + 1}
2
1 = −(a ⋅ y )(b ⋅ z ) (3)
= ⋅5| a × b |
2 Therefore, from Eqs. (1), (2) and (3), we can conclude that the cor-
5 rect options are (A), (B) and (C).
= × 40 Hence, the correct answers are options (A), (B) and (C).
2
= 100
21. Let a , b and c be three non-coplanar unit vectorssuch that
(S) We have | a × b | = 30. Therefore, the angle
between every pair of them is p /3. If a × b + b ×c =
pa + qb + rc where p, q and r are scalars, then the values of
A = | (a + b ) × a |
p 2 + 2q 2 + r 2
=|b × a| is _____.
q2
= 30 [JEE ADVANCED 2014]
Hence, the correct answer is option (C). Solution: See Fig. 26.51. Given a × b + b × c = pa + qb + rc (1)
20. Let x , y and z be three vectors each of magnitude 2 and
the angle between each pair of them is p /3. If a is a non-zero
vector perpendicular to x and y × z and b is a non-zero vec- p p
3
tor perpendicular to y and z × x , then 3
a c
(A) b = (b ⋅ z )( z − x ) (B) a = (a ⋅ y )( y − z )
p
(C) a ⋅ b = −(a ⋅ y )(b ⋅ z ) (D) a = (a ⋅ y )( z − y ) 3 b
[JEE ADVANCED 2014]
Solution: See Fig. 26.50. Figure 26.51
A Taking dot product with a. Therefore,
æ pö æ pö
x y 0 + a × b ´ c = p(1×1× cos 0 ) + q ç 1×1× cos ÷ + r ç 1×1× cos ÷
B è 3ø è 3ø
O
q r
⇒ a ⋅b × c = p + + (2)
C 2 2
z Taking the dot product of (1) with b.
p r
Figure 26.50 0+0=
+q+ (3)
p 2 2
∠AOB = ∠BOC = ∠COA =
3 Taking the dot product of Eq. (1) with c
According to the question p q
c ⋅a × b + 0 = + + r (4)
a = l {( x ⋅ z ) y − ( x ⋅ y )z } 2 2
⎧⎛ p⎞ ⎛ p ⎞ ⎫ From Eqs. (2) and (4), we get
= l ⎨⎜ 2 cos ⎟ y − ⎜ 2 cos ⎟ z ⎬ = l( y − z )
⎩ ⎝ 3 ⎠ ⎝ 3 ⎭⎠
q r p q
p+ + = + +r
(Since, a × ( b × g ) = (a ⋅g ) b − (a ⋅ b )g ) 2 2 2 2
b = m {( y ⋅ x )z − ( y ⋅ z ) x } p r
= ⇒p=r
= m {z − x} 2 2
22. Let ΔPQR be a triangle. Let a = QR , b = RP , and c = PQ . If 23. Suppose that p , q and r are three non-coplanar vectors in
R3. Let the components of a vector s along p , q and r be 4, 3
| a | = 12, | b | = 4 3 and b ⋅ c = 24 , then which of the following
and 5, respectively. If the components of this vector s along
is (are) true?
( − p + q + r ), ( p − q + r ) and ( − p − q + r ) are x, y and z, respec-
tively, then the value of 2x + y + z is _____.
| c |2 | c |2
(A) − | a | = 12 (B) + | a | = 30
2 2 [JEE ADVANCED 2015]
(C) | a × b + c × a | = 48 3 (D) a ⋅ b = −72 Solution:
[JEE ADVANCED 2015]
s = 4 p + 3q + 5r
Solution: See Fig. 26.52.
Also,
P s = x( − p + q + r ) + y( p − q + r ) + z( − p − q + r )
⇒ s = ( − x + y − z )p + ( x − y − z )q + ( x + y + z )r
-x + y - z = 4 ü
ï
c ⏐b⏐= 4√3 Þ x-y-z =3 ý
x+y+z =5 ï
þ
−7 9
⇒z= , x = 4, y = ⇒ 2 x + y + z = 9
2 2
Q R
⏐a⏐= 12 Hence, the correct answer is (9).
where a is the angle between ω̂ and û × v . Therefore, (C) a ∈( −∞ , 1) (D) a ∈(1, ∞ )
(1)(1)cosa = 1
9. Let a , b , c be three non-zero vectors such that a + b + c = 0.
⇒a =0
Then l (b × a ) × b × c + c × a = 0, where l is equal to
which means that ω̂ and û × v are parallel vector or ω̂ is perpendic-
(A) 1 (B) 2
ular vector to û and v .
(C) −1 (D) −2
uˆ ⋅ ωˆ = 0
⎛ (iˆ + ˆj + 2kˆ ) ⎞ 10. If a × b = c , b × c = a where c ≠ 0, then
(u1iˆ + u2 ˆj + u3kˆ ) ⋅ ⎜ ⎟ =0
⎝ 6 ⎠ (A) | a | = | c |, | b | = 1 (B) | a | = | b |, | c | = 1
⇒ u1 + u2 + 2u3 = 0
(C) | b | = | c |, | a | = 1 (D) | a | = | b |, | c | = 1
If û lies in xy-plane then u3 = 0. Therefore,
u1 + u2 = 0 ⇒ u1 = −u2 ⇒ |u1| = |u2| 11. Let a , b , c be unit vectors such that a + b + c = x , a ⋅ x = 1,
Hence, option (C) is correct. 3
Hence, the correct answers are options (B) and (C). b ⋅ x = , | x | = 2. Then angle between c and x is
2
⎛ 1⎞ ⎛ 3⎞
(A) cos −1 ⎜ ⎟ (B) cos −1 ⎜ ⎟
Practice Exercise 1 ⎝ 4⎠ ⎝ 4⎠
⎛ 3⎞ ⎛ 5⎞
1. Any four non-zero vector will always be (C) cos −1 ⎜ ⎟ (D) cos −1 ⎜ ⎟
(A) Linearly dependent (B) Linearly independent ⎝ 8⎠ ⎝ 8⎠
(C) Either ‘A’ or ‘B’ (D) None of these 12. ABCD is a parallelogram A1 and B1 are the mid-points of side
2. If a and b are reciprocal vectors, then BC and CD, respectively. If AA1 + AB1 = l AC , then l is equal to
(A) a ⋅ b = 0 (B) a ⋅ b = −1 (A) 1/2 (B) 1
(C) a ⋅ b = 1 (D) None of these (C) 3/2 (D) 2
d × (a × d )
3. If a + b = p and a − b = q, then 13. If a = b + c , b × d = 0 , c ⋅ d = 0, then the vector is
always equal to | d |2
(A) | a |2 + | b |2 = | p |2 + | q |2
(A) a (B) d
(B) | a |2 − | b |2 = | p |2 − | q |2 (C) b (D) c
(C) 2(| a 2 | + | b |2 ) = | p |2 + | q |2 14. For any two vectors a and b, the expression
(a × iˆ ) ⋅ (b × iˆ ) + (a × ˆj ) ⋅ (b × ˆj ) + (a × kˆ ), (b × kˆ ) is always equal to
(D) 2(| a |2 − | b |2 ) = | p |2 − | q |2
(A) a ⋅ b (B) 2a ⋅ b
4. If three unit vectors
a , b , c satisfy a + b + c = 0, then angle (C) Zero (D) None of these
between a and b is
p 2p 15. Let a = a1iˆ + a2 ˆj + a3kˆ , b = b1iˆ + b2 ˆj + b3kˆ and c = c1iˆ + c2 ˆj + c3kˆ
(A) (B)
3 3 be three non-zero vectors such that | c | = 1, angle between
p
p 5p a and b is and c is perpendicular to a and b. Then
(C) (D) 4
6 6
a1 b1 c1
5. ˆ ˆ ˆ ˆ ˆ ˆ
Projection of i + 2 j + 3k on i − 2 j − 2k is equal to a2 b2 c2 = l( a12 + a22 + a32 )(b12 + b22 + b32 ) where l is equal to
(A) 3 (B) −3
a3 b3 c3
(C) 9 (D) −9
6. If a and b are two non-collinear unit vectors, then projection (A) 1/2 (B) 1/4
(C) 1 (D) 2
of a + b on a − b is equal to
(A) 2 (B) −2 16. Let a , b , c be three vectors such that a × b = c , b × c = a ,
(C) 1 (D) None of these
c × a = b . Then
7. ABCD is a parallelogram with AC = iˆ − 2 ˆj + kˆ and
(A) | a | = | b | = | c | (B) | a | ≠ | b | = | c |
BD = − iˆ + 2 ˆj − 5kˆ. Area of this parallelogram is equal to
(C) | a | = | b | ≠ | c | (D) | a | ≠ | b | ≠ | c |
(A) 5/2 sq. units (B) 2 5 sq. units
(C) 4 5 sq. units (D) 5 sq. units
17. Let a and b beunit vectors such that | a + b | = 3. Then the
ˆ
8. If a = xi + ( x − 1) j + k and b = ( x + 1)iˆ + ˆj + akˆ always make an
ˆ ˆ value of (2a + 5b ) ⋅ (3a + b + a × b ) is equal to
acute angle with each other for every value of x ∈R , then 11 13
(A) a ∈( −∞ , 2) (B) a ∈(2, ∞ ) (A) (B)
2 2
1198 Mathematics Problem Book for JEE
39 23 r r r
(C) (D) 27. The value of | a × iˆ |2 + | a × ˆj |2 + | a × kˆ |2 is
2 2
(A) a2 (B) 2a2
(C) 3a2
18. Let a , b , c be three unit vectors such that a ⋅ b = a ⋅ c = 0. If (D) None of these
the angle between b and c is p /4 then a = l(b × c ), where ‘l’ A⋅B × C B ⋅ A × C
28. If A, B , C are non-coplanar vectors, then + is
is equal to
equal to C × A⋅B C ⋅ A × B
(A) ±1 (B) ± 2
(A) 3 (B) 0
(C) ± 2 (D) None of these
(C) 1 (D) None of these
19. a = 2iˆ − ˆj + kˆ , b = iˆ + 2 ˆj − kˆ , c = iˆ + ˆj − 2kˆ. A vector coplanar 29. Consider ΔABC and ΔA1B1C1 in such a way that AB = A1B1
2 and M, N, M1, N1 be the mid-points of AB, BC, A1B1 and B1C1,
with b and c , whose projection on a is of magnitude is
3 respectively. Then
ˆ ˆ
(A) 2i + 3 j − 3k ˆ ˆ
(B) −2i − j + kˆ ˆ
(A) MM1 = NN1 (B) CC1 = MM1
(C) 2iˆ + 3 ˆj + 3kˆ (D) 2iˆ + ˆj + 5kˆ
(C) CC1 = NN1 (D) MM1 = BB1
20. Let â and b̂ be unit vector that are mutually perpendicular.
30. Let a = iˆ + ˆj + kˆ , b = x1iˆ + x 2 ˆj + x 3kˆ, where x1, x2, x3 ∈ {−3, −2,
Then for any arbitrary r
−1,
0, 1, 2}. Then number of possible vectors b such that a and
(A) r = (r ⋅ aˆ )aˆ + (r ⋅ bˆ )bˆ + (r ⋅ (aˆ × bˆ ))(aˆ × bˆ ) b are mutually perpendicular is
ˆ ˆ
(B) r = (r ⋅ aˆ )aˆ − (r ⋅ b )b − (r ⋅ (aˆ × bˆ ))(aˆ × bˆ ) (A) 25 (B) 28
(C) r = (r ⋅ aˆ )aˆ − (r ⋅ bˆ )bˆ + (r ⋅ (aˆ × bˆ ))(aˆ × bˆ ) (C) 22 (D) None of these
(D) None of these 31. Let a, b, c be distinct and non-negative. If the vectors
21. The line r = a × lb will not meet the plane r × n = q, provided aiˆ + ajˆ + ck,ˆ iˆ + kˆ and ciˆ + cjˆ + bkˆ lie in a plane, then c is
(A) AM of a and b (B) GM of a and b
(A) b × n = 0, a ⋅ n = q (B) b × n ≠ 0, a ⋅ n = q
(C) HM of a and b (D) equal to zero
(C) b × n = 0, a ⋅ n ≠ q (D) b × n ≠ 0, a ⋅ n = q
32. If | a | = 2, | b | = 5 and a ⋅ b = 0 , then a × (a × (a × (a × (a × (a ×
b))))) is equal to
22. If the projection of point P( p ) on the plane r ⋅ n = q is the point
S( s ), then (A) 320 a (B) 320 b
(C) −320b̂ (D) −320 â
(q − p ⋅ n ) (q − p ⋅ n )n
(A) s = 2 (B) s = p + 2 33. If b is the vector whose initial point divides the joining of
|n| |n|
5iˆ and 5 ˆj in the ratio k:1 and terminal point is origin. Also,
( p ⋅ n )n (q − p ⋅ n )n
(C) s = p − 2 (D) s = p − | b | ≤ 37 , then the interval in which k lies
|n| | n |2
(A) (−∞, −6] ∪ [−1/6, ∞) (B) (−∞, −6] ∪ [1/6, ∞)
23. Let a and b be unit vectors (C) (−∞, 6] ∪ [−1/6, ∞) (D) (∞, 6] ∪ [−1/6, ∞)
that are
perpendicular to each
other. Then [a + (a × b ), b + (a × b ), a × b ] will always be equal 34. If “a” is real constant and A, B, C are variable angles and,
to
(A) 1 (B) Zero a2 − 4 tan A + a tan B + a2 + 4 tan C = 6a , then the least
(C) −1 (D) None of these value of tan2 A + tan2 B + tan2 C is
24. A, B, C and D are any four points in the space. If (A) 10 (B) 11
(C) 12 (D) 13
| AB ´ CD + BC ´ AD + CA ´ BD | = l D ABC , where Δ ABC is the
area of triangle ABC, then l is equal to 35. If am iˆ + bm ĵ + cmk̂, m =1, 2, 3, are pairwise perpendicular unit
(A) 2 (B) 1/2
(C) 4 (D) 1/4 a1 b1 c1
25. If sec2A iˆ + ˆj + kˆ , i + sec2 Bjˆ + kˆ and iˆ + ˆj + sec2 Ckˆ are coplanar, vectors, then a2 b2 c2 is equal to
a3 b3 c3
then cot2A + cot2B + cot2C is
(A) equal to 1 (B) equal to 2 (A) 0 (B) 1 or –1
(C) equal to 0 (D) not defined (C) 3 or −3 (D) 4 or –4
26. If a , b , c are three non-coplanar vectors and p, q, r are vec- 36. If aˆ , bˆ , cˆ are three non-coplanar unit vectors, then
b ×c c ×a a×b [aˆ p q ]aˆ + [bˆ p q ]bˆ + [cˆ p q ]cˆ is equal to
tors defined by the relations p = , q = , r = ,
[abc ] [abc ] [abc ]
(A) (aˆ + bˆ + cˆ ) × ( p × q ) (B) aˆ + bˆ + cˆ + q + q
then the value of expression (a + b ) p + (b + c ) q + (c + a ) × r is
(C) p + q (D) p × q
equal to
(A) 0 (B) 1 37. The vector a( x ) = cos xiˆ + sin xjˆ and b ( x ) = xiˆ + sin xjˆ are collin-
(C) 2 (D) 3 ear for
Chapter 26 | Vector Algebra 1199
54. Prove by vector methods, that the altitudes of a triangle are (A) 3:2 (B) 2:3
concurrent.
(C) 3:4 (D) 4:3
55. If a and b are unit vectors, q is the angle between them, then 2
q 1 4. If x and y be unit vectors and z = such that z + z × x = y ,
show that sin = | a − b | .
7
2 2 then the angle q between x and z is
56. Show that the area of the triangle formed by joining the (A) 30° (B) 60°
extremities of an oblique side of a trapezium to the mid-point (C) 90° (D) None of these
of opposite side is half that of the trapezium.
5. If a non-zero vector a is parallel to the line of intersection of
57. The position vectors of the vertices A, B and C of a tetrahe-
the plane P1 determined by iˆ + ˆj and iˆ − 2 ˆj and plane P2 deter-
dron ABCD are iˆ + ˆj + kˆ , iˆ and 3iˆ, respectively. The altitude
mined by vector 2iˆ + ˆj and 3iˆ + 2kˆ, then angle between a and
from the vertex D to the opposite face ABC meets the median
vector iˆ − 2 ˆj + 2kˆ is
line through A of the triangle ABC at a point E. If the length
p p
2 2 (A) (B)
of the side AD is 4 and the volume of the tetrahedron is , 4 2
3 p
then find the position vector of the point E for all its possible (C) (D) None of these
3
positions (Fig. 26.53).
D 6. a and b are non-zero, non-collinear vectors such that | a| =
p
2, a ⋅ b = 1 and angle between a and b is . If r is any vector
3
satisfying r ⋅ a = 2, r ⋅ b = 8 , (r + 2a − 10b )(a × b ) = 4 3 and is
equal to r + 2a − 10b = l(a × b ), then l =
1
(A) (B) 2
C
2
A F 1
E(E2) E(E1) (C) (D) 4
iˆ + jˆ + kˆ 4
B
7. Let aˆ , bˆ , cˆ be unit vectors such that aˆ × bˆ = cˆ and aˆ ⋅ bˆ = 0.
Figure 26.53
Also, x is any vectors such that [ x bˆ cˆ ] = 3, [ x cˆ aˆ ] = 4 and
58. If the vectors b , c , d are non-coplanar,
then prove that the
[ x aˆ bˆ ] = 2. Then x is equal to
vector (a × b ) × (c × d ) + (a × c ) × (d × b ) + (a × d ) × (b × c ) is a
vector parallel to a. (A) 2aˆ + 3bˆ + cˆ (B) 3aˆ + 4bˆ + 2cˆ
59. Show that the solution of the
equation kx + x × a = b where (C) aˆ + 2bˆ + 3cˆ (D) None of these
k ≠ 0 is a scalar and a and b are any two vectors is given by
8. If A, B, C, D be four points in a plane with position vectors
1 ⎪⎧ a ⋅ b ⎪⎫
x= 2 2 ⎨ a + kb + a + b ⎬ . a , b , c , d , respectively, such that (a − d ) ⋅ (b − c ) = (b − d ) ⋅
(k + a ) ⎪⎩ k ⎪⎭
(c − a ) = 0, then
(A) D is the circumcentre of the triangle ABC
Practice Exercise 2 (B) D is the orthocentre of the triangle ABC
(C) A is the orthocentre of the triangle DBC
Single/Multiple Correct Choice Type Questions (D) A is the circumcentre of the triangle DBC
1. If the shortest distance between lines r = iˆ + 2 ˆj + 3kˆ + l1 9. [a × (b + c ), b × (c − 2a ), c × (a + 3b )] = ?
(2iˆ + 3 ˆj + 4 kˆ ) and r = 2iˆ + 4 ˆj + 5kˆ + l2 (3iˆ + 4 ˆj + 5kˆ ) is x, then
−1
cos cos 6x is equal to (A) [a, b, c]2 (B) 7[a, b, c]2
1 (C) −5[a × b , b × c , c × a ] (D) None of these
(A) (B) 0
2
10. If a , b and c are three non-coplanar unimodular vectors,
(C) 1 (D) p each inclined with other at an angle 30°, then volume of tet-
( ) ( )
2. If r = l a × b + m b × c + g (c × a ) and ⎡⎣a b c ⎤⎦ = , then l + m
1
8
rahedron whose edges are a , b and c is
+ g is 3 3 −5 3 3 +5
(A) (B)
(A) 8(r ⋅ a ) (B) 8(r ⋅ b ) 12 12
5 2 +3
(C) 8(r ⋅ c ) (D) 8r ⋅ (a + b + c ) (C) (D) None of these
12
3. In a triangle OAB, E is the mid-point of OB and D is a point on AB
11. Let a = aiˆ + bjˆ + ckˆ, b = biˆ + cjˆ + akˆ and g = ciˆ + ajˆ + bkˆ be
such that AD:DB = 2:1. If OD and AE intersect at P, then ratio of
OP three coplanar vectors with a ≠ b and v = iˆ + ˆj + kˆ. Then v is
is equal to perpendicular to
PD
Chapter 26 | Vector Algebra 1201
(A) a (B) b Comprehension Type Questions
(C) g (D) None of these Paragraph for Questions 16–18: The vertices of a triangle ABC
are A = (2, 0, 2), B = (–1, 1, 1) and C = (1, – 2, 4). The point D and E
12. Let a and b are two given perpendicular vectors, which are
divide the sides AB and CA in the ratio 1:2, respectively. Another
non-zero. A vector r satisfying the equation r × b = a , can be
point F is taken in space such that perpendicular drawn from F on
a × b a × b ΔABC meets the triangle at the point of intersection of the line seg-
(A) b − (B) 2b − 2
2 ment CD and BE, say P. If the distance of F from the plane of the
b b
ΔABC is 2 units, then
a×b a × b 16. The PV of P is
(C) a b − 2 (D) b b − 2
(A) iˆ + ˆj − 3kˆ (B) iˆ − ˆj + 3kˆ
b b
(C) 2iˆ − ˆj − 3kˆ (D) iˆ + ˆj + 3kˆ
13. Let unit vectors a and b are perpendicular and unit
vector c be inclined at an angle q to both a and b. If 17. The vector PF is
7 ˆ ˆ
c = a a + b b + g (a × b ) , then (A) 7 ˆj + 7kˆ ( j + k)
(B)
(A) a = b (B) 1 − 2a 2 = g 2 2
(C) ĵ + kˆ (D) None of these
1+ cos 2q
(C) a 2 = (D) a 2 − b 2 = g 2 18. The volume of tetrahedron ABCF is
2 3
(A) 7 cubic units (B) cubic units
14. If a and b are two vectors and angle between a and b is q , 5
then 7 7
(C) cubic units (D) cubic units
(A) | a × b |2 + (a ⋅ b )2 = | a |2 | b |2
3 5
Answer Key
Practice Exercise 1
1. (A) 2. (C) 3. (C) 4. (B) 5. (B) 6. (D)
7. (B) 8. (B) 9. (D) 10. (A) 11. (B) 12. (C)
13. (D) 14. (B) 15. (A) 16. (A) 17. (C) 18. (B)
19. (A) 20. (A) 21. (C) 22. (B) 23. (A) 24. (C)
25. (C) 26. (D) 27. (B) 28. (B) 29. (D) 30. (A)
31. (B) 32. (C) 33. (A) 34. (C) 35. (B) 36. (D)
37. (B) 38. (D) 39. (C) 40. (A) 41. (B) 42. (D)
43. (A) 44. (B) 45. (B) 46. (D) 47. (C) 48. (B)
49. (A) 50. (C) 51. (B) 52. (C) 53. (D) 57. E1 = 3iˆ − ˆj − kˆ,
E2 = − iˆ + 3 ˆj + 3kˆ
Practice Exercise 2
1. (C) 2. (D) 3. (A) 4. (B) 5. (B) 6. (B)
7. (B) 8. (B) 9. (B) 10. (A) 11. (A), (B), (C) 12. (A), (B), (C), (D)
13. (A), (B), (C) 14. (A), (B), (C), (D) 15. (B), (C) 16. (B) 17. (C) 18. (C)
19. 2 20. 3
1202 Mathematics Problem Book for JEE
Solutions
Practice Exercise 1 6. (a + b ) ⋅ (a − b ) =| a |2 − | b |2 = 0
1. Four or more than four non-zero vectors are always linearly Thus, projection of a + b on a − b is zero. Hence, (D) is the cor-
dependent.
Hence, (A) is correct answer. rect answer.
2. If a and b are reciprocal, then 7. Area vector of parallelogram is
a = lb , l ∈R + and | a || b | = 1 i j k
1
1
⇒| a | = | l || b | ( AC × BD ) = 1 −2 1
2 2
−1 2 −5
|a| 1
⇒| l | = = 2 1
|b | |b | = (8iˆ + 4 ˆj )
2
1
⇒| a | = 2 b = 4 iˆ + 2 ˆj
|b |
Therefore, area of the parallelogram is
1
⇒ a ⋅ b = 2 | b || b | cos 0 = 1 | 4 iˆ + 2 ˆj |= 2 5 sq. units
|b |
8. a ⋅ b = ( xiˆ + ( x − 1) ˆj + kˆ ) ⋅ (( x + 1)iˆ + ˆj + akˆ )
3. a + b = p
= x ( x + 1) + x − 1+ a
⇒ | a + b |2 = | p |2
= x2 + 2x + a −1
⇒ (a + b ) ⋅ (a + b ) = | p |2 We must have
⇒ | a |2 + | b |2 + 2 ⋅ a ⋅ b = | p |2 a ⋅ b > 0 ∀x ∈R
Also, ⇒ x 2 + 2 x + a − 1 > 0 ∀x ∈R
⇒ 4 – 4(a – 1) < 0
a −b = q
⇒a>2
⇒ | a − b |2 = | q |2
9. See Fig. 26.54. Clearly, a , b , and c represent the sides of a
⇒ (a − b ) ⋅ (a − b ) = | q |2 triangle.
A
⇒ | a |2 + | b |2 −2 ⋅ a ⋅ b = | q |2
Thus,
c b
2 ( a |2 + | b |2 ) = | p |2 + | q |2
4. a + b = − c B C
a
⇒| a + b |2 = | c |2 = 1
Figure 26.54
⇒| a |2 + | b |2 + 2a ⋅ b = 1
It is an area vector, such that
1
⇒ a ⋅b = − 1 1 1
2 a×b− c ×d = a×c
2 2 2
1
⇒ | a || b | cosq = − Thus,
2
1 a × b + b × c + c × a = −a × b
⇒ cosq = −
2 ⇒ 2(a × b ) + b × c + c × a = 0
2p
⇒q = − 10. a ´ b = c , b ´ c = a
3
a ⋅b Taking cross with b in first equation, we get
5. Projection of a on b is .
|b | b × (a × b ) = b × c = a
Thus, required projection is
Þ | b |2 a - (a × b )b = a
(iˆ + 2 ˆj + 3kˆ ) ⋅ (iˆ − 2 ˆj − 2kˆ )
Þ | b |2 = 1 and a × b = 0
1+ 4 + 4
1− 4 − 6 Also,
= = −3
|a ×b|=|c |
3
Chapter 26 | Vector Algebra 1203
Now,
= 2((c − a ) × (b − a ))
r ⋅ a = x1a ⋅ b + x 2a ⋅ c
= 2( AC × AB )
Þ ± 2 = x1(2 - 2 - 1) + x 2 (2 - 1- 2)
1
Þ| AB ´ CD + BC ´ AD + CA ´ BD | = 4 ( AC × AB ) = 4Δ ABC
⇒ x1 + x 2 = −2 or 2 2
If x1 + x 2 = −2, then 25. The vectors are coplanar, therefore
r = x1(iˆ + 2 ˆj − kˆ ) + x 2 (iˆ + ˆj − 2kˆ )
sec2 A 1 1
= iˆ( x1 + x 2 ) + ˆj (2 x1 + x 2 ) − kˆ ( x1 + 2 x 2 ) 1 sec B 2
1 =0
= −2iˆ + ˆj ( x − 2) − kˆ ( x − 2)
1 2 1 1 sec C2
= −2iˆ + ˆj ( x1 − 2) − kˆ ( −4 − x1)
So, cot2A + cot2 B + cot2 C + 1 = 0 which is not possible.
where x1 ∈R . Hence, (C) is the correct answer.
If x1 + x 2 = 2 , then
( a + b ) ⋅ (b × c ) a ⋅ (b × c )
r = x1(iˆ + 2 ˆj − kˆ ) + x 2 (iˆ + ˆj − 2kˆ ) 26. (a + b ) ⋅ p = = = 1
[abc ] [abc ]
= iˆ( x1 + x 2 ) + ˆj (2 x1 + x 2 ) − kˆ ( x1 + 2 x 2 ) Hence, the given scalar expression = 1 + 1 + 1 = 3.
= 2iˆ + ˆj ( x1 + 2) − kˆ (4 − x1) 27. | a × iˆ |2 + | a × ˆj |2 + | a × kˆ |2
20. Let r = x1aˆ + x 2bˆ + x 3 (aˆ × bˆ ). Then ⇒ |a|2 sin2 a + |a|2 sin2 b + |a|2 sin2 g
= 3|a2| − |a2|(cos2 a + cos2 b + cos2 g )
r ⋅ aˆ = x1 + x 2aˆ ⋅ bˆ + x 3aˆ ⋅ (aˆ × bˆ ) = x1
= 2|a2| = 2a2
Also,
r ⋅ bˆ = x1aˆ ⋅ bˆ + x 2 + x 3bˆ ⋅ (aˆ × bˆ ) = x 2 [ ABC ] [BAC ]
28. + = 1− 1 = 0
and [CAB ] [CAB ]
r ⋅ (aˆ × bˆ ) = x1aˆ ⋅ (aˆ × bˆ ) + x 2bˆ ⋅ (aˆ × bˆ ) + x 3 (aˆ × bˆ ) ⋅ (aˆ × bˆ ) = x 3 29. See Figs. 26.56 and 26.57.
C1(c1)
⇒ r = (r ⋅ aˆ )aˆ + (r ⋅ bˆ )bˆ + (r ⋅ (aˆ × bˆ ))(aˆ × bˆ ) C(c)
21. We must have b ⋅ n = 0 and a ⋅ n ≠ q .
N1
Hence, (C) is the correct answer. N
22. We have
A1(a1) B1(b1)
s − p = ln and s ⋅ n = q A(a) B(b) M1
M
⇒ ( ln + p ) ⋅ n = q
Figure 26.56 Figure 26.57
(q − p ⋅ n )n
⇒s = p+ 2
|n|
AB = A1B1 ⇒ b − a = b1 − a1
23. [a + (a × b ), b + (a × b ), a × b ] ⇒ b − b1 = a − a1 ⇒ B1B = A1A ⇒ AA1 = BB1
= (a + (a × b )) ⋅ ((b + (a × b )) × (a × b )
b + c b + c
b + c − b − c
⇒ NN1 = 1 1 − ⇒ NN1 = 1 1
(a + (a × b )) ⋅ (b × (a × b )) 2 2 2
(a + (a × b )) ⋅ (a − (a ⋅ b )b ) ⇒ 2NN1 = BB1 + CC1
= a ⋅ a + a ⋅ (a × b )
b − b + a1 − a
⇒ MM1 = 1 ⇒ 2MM1 = BB1 + AA1 = 2BB1 = 2 AA1
= 1 (as a ⋅ b = 0 , a ⋅ (a × b ) = 0 ) 2
⇒ MM1 = BB1 = AA1
24. Let PV of A, B, C, and D be a , b , c, and 0. Then
AB × CD = (b − a ) × − c , 30. a ⋅ b = 0 ⇒ x1 + x 2 + x 3 = 0
BC × AD = (c − b ) × − a Thus, we have to obtain the number of integral solution of
this equation.
and CA × BD = (a − c ) × −b
Coefficient of x° | (x−3 +x−2 +x−1 + x0 + x + x2)3
⇒ AB × CD + BC × AD + CA × BD
⎛ 1+ x + x 2 + x 3 + x 4 + x 5 ⎞
3
= c ×b +a×c +a×c +b ×a −a×b +c ×b = x° ⎜
⎟
= 2(c × b + b × a + a × c ) ⎝ x3 ⎠
= 2(c × (b − a ) − a × (b − a )) = x9 |(1 − x6)3 (1− x)−3 = 11C9 − 3.5C3 = 25
Chapter 26 | Vector Algebra 1205
a1 b1 c1
2
1 0 0 a1 b1 c1 Also,
2
35. a2 b2 c2 = 0 1 0 = 1⇒ a2 b2 c2 = ±1 | r2 |2 = (b − (a ⋅ b )a ) ⋅ (b − (a ⋅ b )a ) = | b |2 − 2(a ⋅ b )2 + (a ⋅b
b ) (a ⋅ a )
a3 b3 c3 0 0 1 a3 b3 c3 |r |
= | b |2 −(a ⋅ b )2 ⇒ 1 = 3
| r2 |
ˆ ] = projection of p × q in the direction of â.
36. [apq
p p p
Hence, the given vector is p × q . Thus, angles are , , .
2 3 6
37. Since a and b are collinear, for some l, we can write
G G 42. See Fig. 26.58. Let PV of A, B and D be 0, b , d. Then,
a=lb
⇒ cos xiˆ + sin xjˆ = l ( xiˆ + sin xjˆ ) AC = b + d
b + 4( b + d ) 4
⇒ cos x = x l and l = 1⇒ cos x = x ⇒E= =b+ d
5 5
1206 Mathematics Problem Book for JEE
( z − a)2
( z − b) 2
(z − c) 2
z 2
z 1 a 2
b 2
c 2 ⎛ n G⎞ ⎛ n G⎞ n G 2 n n
G G
⎜ ∑ a j⎟ ⋅ ⎜
⎜ ∑ a j ⎟
⎟
⎝ i =1 ⎠ ⎝ j =1 ⎠ i =1
= ∑ | ai | + 2 ∑ ∑ aj ⋅aj
i =1 j =1
1 1 1
n n
⇒ − a −b − c = 0 [since, X, Y, Z are non-coplanar]
⇒ 0 = n + 2∑ ∑ ai ⋅ a j
a2 b2 c2 i =1 j =1
n n
2 2 2
Hence, (a , a, 1), (b , b , 1) and (c , c , 1) are coplanar. n
⇒ ∑ ∑ ai .a j = −
2
44. Consider three non-coplanar vectors b, c and b × c . Any vec- i =1 j =1
tor a can be written as
50. We have
a = xb + yc + z (b × c ) (1)
4 a + 5b + 9c = 0
Taking dot product with b × c in Eq. (1), we get G G G
Therefore, vectors a , b and c are coplanar. So,
a ⋅ (b × c )
a ⋅ (b × c ) = z | b × c |2 ⇒ z = b × c and c × a are collinear
| b × c |2
⇒ (b × c ) × ( c × a ) = 0
Taking dot product with b in Eq. (1), we get
a ⋅ b = xb ⋅ b + yc ⋅ b + z ⋅ 0 = x
51. | c − a |2 = 8
Taking dot product with c in Eq. (1), we get
a⋅c = y ⇒ | c |2 + | a |2 −2c ⋅ a = 8 ⇒ | c |2 − 2 | c | + 1 = 0 ⇒ | c |= 1
Thus, i j k
a ⋅ (b × c )
a = (a ⋅ b )b + (a ⋅ c )c + (b × c ) a × b = 2 1 −2 = −2i + 2 j + k ⇒ | (a × b )| = 3
| b × c |2
1 1 0
45. For the point of intersection of the lines 1 3
| (a ´ b ) ´ c | = | (a ´ b )|| c | sin 30° Þ | (a ´ b ) ´ c | = 3 ´ 1´ =
a + t (b × c ) = b + s(c + a ) ⇒ ac + t (b × c )c = b ⋅ c + s(c × a )c 2 2
⇒ a⋅c = b ⋅c 52. Since r ⋅ a = 0 , r ⋅ b = 0 and r ⋅ c = 0; r must be perpendicular
to all the three vectors a , b and c . Hence a , b and c must be
46. Given
coplanar. So,
r ⋅ a = 10 ⇒ x + y + z = 10 , x , y , x ≥ 1 [a b c ] = 0
Chapter 26 | Vector Algebra 1207
53. Vector d is perpendicular to a , b and c which is possible only 56. See Fig. 26.61. Let ABCD be the trapezium and E be the mid-
point of BC. Let A be the initial point and let b be the PV of B
whenGaG, Gb and c are coplanar and then a ⋅ d = c ⋅ d = b ⋅ d = 0
and d that of D. Since DC is parallel
to AB, tb is a vector along
and [a b c ] = 0. Given
DC, so that the PV of C is d + tb . Therefore, the PV of E is
b × ( d × c ) = d × (b × c ) + a + d
b + d + tb d + (1+ t )b
⇒ [abd ] + [abc ] = (c ⋅ d )(ab ) − (d ⋅ b )(a ⋅ b ) + a ⋅ a + a ⋅ d =
2 2
⇒ [abd ] = [a ]2
1 d + (1+ t )b 1
So, Area of ΔAED = × d = (1+ t )| b × d |
(a ⋅ c )a = c ⇒| a |= 1 2 2 4
54. See Fig. 26.60. Let the altitudes AD and BE intersect at O.
(d )
Join CO and produce to meet AB in F. D C
Let OA = a ; OB = b ; OC = c
The vector a is perpendicular to E
BC = OC - OB = c - b
Therefore,
(b)
a (c - b ) = 0, Þ a × c = a × b (1) A
(0) B
A Figure 26.61
Area of the trapezium = Area (ΔACD) + Area (ΔABC)
1 1 1 1
= | b × ( d + tb )| + | ( d + tb ) × d | = | b × d | + t | b × d |
E 2 2 2 2
F 1
O = (1+ t )| b × d | = 2ΔAED
2
57. ABCD is the tetrahedron.
B AB = OB - OA = i - (iˆ + ˆj + kˆ ) = - jˆ - kˆ
D
C AC = OC - OA = 3i - ( iˆ + jˆ+ kˆ ) = 2iˆ - jˆ - kˆ
Area of ΔABC magnitude is equal to
Figure 26.60
1 | AB × AC |= 1 | (2iˆ − ˆj − kˆ ) × ( − ˆj − kˆ )|
Also the direction OB is perpendicular to AC 2 2
Therefore,
= | ˆj − kˆ | = 2
b ⋅ (c − a ) = 0 ⇒ b ⋅ c = a ⋅ b (2) Therefore,
From Eqs. (1) and (2) 1
volume of ABCD = (Area of the base) × Height
3
a⋅c = a⋅b = b⋅c
1 2 2
That is, = 2 × DE =
3 3
c ⋅ a − c ⋅ b = 0 ⇒ c ⋅ (a − b ) = 0 (3)
a − b = OA − OB = BA , Þ DE = 2
and by virtue of Eq. (3), c
is perpendicular
to BA; but c is a
vector
in the direction of OC . Hence, OC is perpendicular to AE = AD 2 − DE 2 = 16 − 4 = 2 3
AB, that is, CF is the third altitude of the triangle through C. Therefore, E falls outside of AF and lies on AF produced and
Hence, the three altitudes are concurrent at O. such that AF = FE.
2 2
55. a ⋅ a = a = | a |2 = 1; b ⋅ b = b = | b |2 = 1 Therefore, the position vector of E is
58. b , c , d are given to be non-coplanar and hence [b c d ] ≠ 0 Practice Exercise 2
Any other vector a can be expressed as a linear combination
of b, c , and d so that we have AB ⋅ b1 × b2 (iˆ + 2 ˆj + 2kˆ ) ⋅ ( − iˆ + 2 ˆj − kˆ ) 1
1. x = = =
r r r r b1 × b2 1+ 4 + 1 6
a = lb + mc + g d
a × d = lb × d + mc × d (since, d × d = 0 ) Now,
( a × d ) ⋅ c = l (b × d ) ⋅ c cos −1 cos 6x = cos–1 cos1 = 1
Therefore, a1 a2 a3
{
l = [a d c ] / [b d c ] } 2. [abc ] = a ⋅ (b × c ) = b ⋅ (c × a ) = c ⋅ (a × b ) = b1 b2 b3 = D
Similarly, c1 c2 c3
m = [a b d ] / [c b d ] Given
Therefore, 1
r = l (a ´ b ) + m (b ´ c ) + g ( c ´ a ),[a , b , c ] =
g = [a c b ] / [d c b ] 8
Now, Multiply the above relation with a, b, c, we have
(a × b ) × (c × d ) = [a b d ]c − [a b c ]d m
r × a = l × 0 + m [a × b × c ] + 0 =
8
(a × c ) × (d × b ) = [a c b ]d − [a c d ]b
⇒ m = 8(r . a ), g = 8(r . b ), l = 8(r . c )
(a × d ) × (b × c ) = [a d c ]b − [a d b ]c
Therefore,
G G G G
and [a b d ] = −[a d b ]; [a c b ] = −[a b c ] l + m + g = 8r . (a + b + c )
and [a d c ] = −[a c d ] 3. See Fig. 26.62. Let A(a ), B(b ). Then
Hence, by adding the three results and using above informa- 2b + a
PV of D =
tion, we get 3
( a × b ) × ( c × d ) + ( a × c ) × ( d × b ) + ( a × d ) × (b × c ) b
PV of E =
= 2{[adc ]b + [abd ]c + [acb ]d } 2
OP AP
⎪⎧ [adc ] [abd ] [acb ] ⎪⎫ Let = t, = l. Then
= 2[bdc ] ⎨ b + c + d ⎬ = 2[bdc ]a PD PE
⎩⎪ [bdc ] [bdc ] [bdc ] ⎭⎪
lb
This is certainly in the direction of a. t (2b + a ) 2 + a
PV of P = =
3(t + 1) l +1
59. kx + x × a = b ⇒ ka ⋅ x = a ⋅ b
a ⋅b B(b)
⇒ a⋅ x =
k 1
Also,
E
D
k a × x + a × ( x × a) = a × b 2
P
k (a ´ x ) + (a × a ) x - (a × x )a = a ´ b (using that a × x = k x − b )
O A(a)
Therefore,
Figure 26.62
a ⋅b
k 2 x − kb + a2 x − a = a×b Comparing coefficients of a and b, we get
k
2t
=
l
(1)
(substituting a . x = a . b/k )
3(t + 1) 2( l + 1)
Therefore, t 1
and = (2)
3(t + 1) l + 1
a ⋅b
x ( k 2 + a2 ) = a+kb +a×b 2 l
k ⇒ = ⇒l=4
l + 1 2( l + 1)
⎪⎫
1 ⎪⎧ a ⋅ b t 1 3
⇒x= 2 2 ⎨ a + k b + a × b⎬ ⇒ = ⇒ 3t + 3 = 5t ⇒ 2t = 3 ⇒ t =
(k + a ) ⎩⎪ k ⎭⎪ 3(t + 1) 5 2
Chapter 26 | Vector Algebra 1209
1 2 ⇒ a3 + b3 + c3 – 3abc = 0 = (a + b + c )[(a − b )2 + (b − c )2 +
⇒ z = =
1+ sin2 q 7 + (c − a)2 ] = 0
3 p ⇒ a + b + c = 0 as a ≠ b
⇒ sinq = ⇒ q = = 60° ˆ ˆ ˆ ˆ ˆ
2 3 ⇒ v ⋅ (ai + bj + ck ) = v ⋅ (bi + cj + akˆ ) = v ⋅ (ciˆ + ajˆ + bkˆ ) = 0
5. Normal vector to plane P1, −3k̂. 12. Since a , b and a × b are non-coplanar. So,
Normal vector to plane P2, 2iˆ − 4 ˆj − 3kˆ. So,
r = xa + yb + z (a × b )
a = l (2iˆ + ˆj ) Therefore,
Angle between a and vector iˆ − 2 ˆj + 2kˆ is given by r × b = a ⇒ xa × b + z {(a ⋅ b )b − (b ⋅ b )a } = a
l (2iˆ + ˆj ) ⋅ (iˆ − 2 ˆj + 2kˆ )
cosq = =0 ⇒ −(1+ z | b |2 )a + xa × b = 0 [sin ce a ⋅ b = 0]
l 5 9
p Therefore,
⇒q = 1
2 x = 0 and z = − 2
b
6. Let r = a a + b b + l(a × b ). Then
4a + b = 2 a × b
Also, Thus, r = yb − 2 , where y is the parameter.
a+b=8 b
Therefore,
a = − 2, b = 10 13. | a | = | b | = | c | = 1
Also, l = 2
Now, a⋅c b ⋅c
cos q = = = a ⋅ c = b ⋅ c
| a |⋅| c | | b |⋅| c |
r + 2a − 10b = 2(a × b ) = 2
a ⋅b = 0
7. aˆ × bˆ = cˆ and aˆ ⋅ bˆ = 0
a ⋅ c = a ⋅ a ⋅ a + b ⋅ a ⋅ b + g ⋅ a ⋅ (a × b ) = a = cos q
So, aˆ , bˆ , cˆ are mutually perpendicular vectors.
Similarly,
[ x bˆ cˆ ] = 3 ⇒ x ⋅ (bˆ × cˆ ) = 3 ⇒ x ⋅ aˆ = 3 b ⋅ c = cosq = b
[ x cˆ aˆ ] = 4 ⇒ x ⋅ bˆ = 4 So,
a. c = b. c = cosq
[ x aˆ bˆ ] = 2 ⇒ x ⋅ cˆ = 2
Þa = b
x = ( x ⋅ aˆ )aˆ + ( x ⋅ bˆ )bˆ + ( x ⋅ cˆ )cˆ = 3aˆ + 4bˆ + 2cˆ Now
8. DA ⊥ CB and DB ⊥ AC
1 = c . c = 2a 2 + g 2 | a ´ b |
Therefore, D is the orthocentre of the triangle ABC.
1 = 2a 2 + g 2 [| a |2 ⋅ | b |2 −(a ⋅ b )2 ] = 2a 2 + g 2
9. Let a ´ b = A, b ´ c = m, c ´ a = n. Then
14. a × b = | a | | b | sinq nˆ
1 0 -1
|a ×b|
[ A - n, m + 2A , n - 3m] = 2 1 0 [ A , m, n] = 7[a, b, c]2 ⇒ | a × b | = | a | | b | sinq ⇒ sinq = (1)
|a||b |
0 -3 1
a ⋅b
a⋅a a⋅b a⋅c a ⋅ b = | a | | b | cosq ⇒ cosq = (2)
2 1 1 ⎛ 3 3 5⎞ |a||b |
10. V = b⋅a b ⋅b b⋅c = ⎜ − ⎟
36 36 ⎝ 4 4⎠ From Eqs. (1) and (2),
c ⋅a c ⋅b c ⋅c
sin2 q + cos2 q = 1
⇒V=
1
12 ( 3 3 − 5) 2
( )
⇒ | a × b | + a ⋅ b = | a |2 | b |2
1210 Mathematics Problem Book for JEE
p Therefore,
If q = , then
4 7 ˆj + 7kˆ
1 PF = 2 = ˆj + kˆ
sinq = cosq = 49 + 49
2
Therefore, 1
18. Δ = Area of ΔABC = AB × AC
2
|a||b | |a||b |
| a × b |= and a ⋅ b =
2 2 1 7 2
= ( −3iˆ + ˆj − kˆ ) × ( − iˆ − 2 ˆj + 2kˆ ) = sq. units
2 2
Hence, Therefore,
| a × b | = a ⋅b 1
Volume of tetrahedron = × area of base × height
3
Thus,
r r 7
= cubic units
r r r r |a ||b |
a × b =| a | | b | sinq n =
ˆ nˆ 3
2
r r r r l −1ˆ m +2 ˆ
or a × b = (a ⋅ b )nˆ 19. See Fig. 26.63. PV of D = i + 4 ˆj + k
2 2
15. We have l −4 m −8
DR of AD = , 1,
2 2
(a × b ) × a = (a ⋅ a )b − (b ⋅ a )a =| a |2 b − 2a 1 1 1
But direction ratios of AD should be , , . So,
Therefore, 3 3 3
(a × b ) × a + 2a l −4 m −8
b= = 1=
| a |2 2 2
Now, l = 6, m = 10
2 2l − m = 2
(a × b ) × a = 4 iˆ − 5 ˆj + kˆ and a = 3
A(2iˆ + 3jˆ + 5k)
ˆ
Therefore,
(4 iˆ − 5 ˆj + kˆ ) + 2(iˆ + ˆj + kˆ )
b= = 2iˆ − ˆj + kˆ
3
16. Clearly, vector equations of CD and BE are
l
r = iˆ − 2 ˆj + 4 kˆ + (7 ˆj − 7kˆ ) (1) B D C
3 (−iˆ + 3jˆ + 2k)
ˆ (liˆ + 5jˆ + mk)
ˆ
m
r = −iˆ + ˆj + kˆ + (7iˆ − 7 ˆj + 7kˆ ) (2) Figure 26.63
3
20. The given expression reduces to
For point of intersection, equating r in Eqs. (1) and (2), we get
6 3
m = , l = ⇒ PV of P = iˆ − ˆj + 3kˆ ( − a × b + c × a − b × a + b × c ) ⋅ (a + 2b − c )
7 7
[a b c ]
17. We have AB × AC = 7 ˆj + 7kˆ
= [a b c ] + 2[a b c ]+ [a b c ] − [a b c ] = 3
Since PF is parallel to AB × AC and PF = 2 [a b c ]
Chapter 26 | Vector Algebra 1211
2. The area (in sq. units) of the parallelogram, whose diagonals Comparing the components, we get
are along the vectors 8i - 6 j and 3i + 4 j - 12k , is 0=k+p (1)
(A) 65 (B) 52 3=k–p (2)
(C) 26 (D) 20 4=r (3)
(ONLINE) Adding Eqs. (1) and (2), we get
Solution: The area of parallelogram when diagonals are given is
3
2k = 3 ⇒ k =
1 2
A = d1 × d2 (1)
2 From Eq. (1), we get
The given vectors are d1 = 8i - 6 j and d2 = 3i + 4 j - 12k . −3
p = −k =
Therefore, from Eq. (1), we get 2
3 −3
i j ⇒k = and p =
k 2 2
d1 ´ d2 = 8 -6 0 = i(72 - 0 ) - j ( -96 - 0 ) + k (32 + 18 ) Hence,
3 3
3 4 -12 b1 = i + j
2 2
= 72i + 96 j + 50 k and
-3 3
= (72)2 + (96 )2 + (50 )2 = 5184 + 9216 + 2500 = 16 , 900 = 130 b2 = i + j + 4 k
2 2
1212 Mathematics Problem Book for JEE
JEE Advanced 2017 Solution: The given geometrical situation is depicted in the
following figure:
1. Let O be the origin and
let
PQR
be
an
arbitrary triangle.
The
P
point S is such
that OP ⋅ PQ + OR ⋅ OS = OR ⋅ OP + OQ ⋅ OS = OQ ⋅
OR + OP ⋅ OS. Then the triangle PQR has S as its
(A) centroid. (B) circumcenter.
(C) incentre. (D) orthocenter.
OZ OY
Solution: The given geometrical situation for the triangle is
depicted in the following figure:
P
Q
R
OX
Now,
QR
OX =
QR
and
Q R
RP
OY =
RP
It is given that Therefore,
OP ⋅ OQ + OR ⋅ OS = OR ⋅ OP + OQ ⋅ OS = OQ ⋅ OR + OP ⋅ OS
QR RP QR × RP
(OX × OY ) = × =
QR RP PQ
Let us consider that PQ sin R
= = sin R = sin(p − ( P + Q ) = sin( P + Q )
OP ⋅ OQ + OR ⋅ OS = OR ⋅ OP + OQ ⋅ OS PQ
Note: The coordinates of centroid of a triangle having vertices 2. If (xr , yr , zr ), r = 1, 2, 3, 4, are vertices of a tetrahedron, then the
A(x1, y1, z1), B(x2, y2, z2) and C(x3, y3, z3) are coordinates of its centroid are
⎛ x1 + x 2 + x 3 y1 + y 2 + y 3 z1 + z2 + z3 ⎞ ⎛ x1 + x 2 + x 3 + x 4 y1 + y 2 + y 3 + y 4 z1 + z2 + z3 + z 4 ⎞
G⎜ , , ⎟⎠ ⎜⎝ , , ⎟⎠
⎝ 3 3 3 4 4 4
Illustration 27.3 If the x-coordinate of a point P on the join of 27.6.2 Area of a Triangle
Q(2, 2, 1) and R(5, 1, −2) is 4, then find its z-coordinate. Let A(x1, y1, z1), B(x2, y2, z2) and C(x3, y3, z3) be the vertices of a triangle.
Then
Solution: Let the point P be
y1 z1 1 x1 z1 1
⎛ 5k + 2 k + 2 −2k + 1⎞ 1
Δ x = y2 z2 1 , Δ y = x2 z2 1
1
⎜⎝ , , ⎟
k +1 k +1 k +1 ⎠ 2 2
y3 z3 1 x3 z3 1
Given that
and
5k + 2
=4 ⇒ k =2 x1 y1 1
k +1 1
Δz = x2 y2 1
Therefore, 2
−2(2) + 1 x3 y3 1
z-coordinate of P = = −1
2 +1 Area of ΔABC is given by the relation
cos g are called the direction cosines of the given line and are 4. Since −1 ≤ cos x ≤ 1 ∀x ∈R, hence values of l, m and n are such
generally denoted by l, m and n, respectively (Fig. 27.5). real numbers that are not less than −1 and not greater than 1.
Thus, l = cos a, m = cos b and n = cos g. Hence, direction cosines Î[ -1,1].
By the definition it follows that the direction cosines of the axis of 5. The direction cosines of a line parallel to any coordinate axis
x are respectively cos 0°, cos 90° and cos 90°, that is, (1, 0, 0). are equal to the direction cosines of the coordinate axis.
Similarly, direction cosines of the axes of y and z are, respectively,
6. The number of lines, which are equally inclined to the
(0, 1, 0) and (0, 0, 1).
coordinate axes, is 4.
z 7. If l, m and n are the dc’s of a line, then the maximum value of
B 1
lmn = .
3 3
A
8. The angles a, b and g are called the direction angles of the
γ line AB.
α 9. The direction cosines of the line BA are cos (p − a), cos (p − b)
O
β
and cos (p − g ), that is, −cos a, −cos b and −cos g .
10. Angles a, b and g are not coplanar.
Y 11. a + b + g is not equal to 360° as these angles do not lie in
same plane.
Figure 27.5 12. If P(x, y, z) be a point in space such that r = OP has dc’s l, m
and n, then x = l | r |, y = m | r |, z = n | r |.
27.7.1 Relation Between the Direction Cosines 13. Projections of a vector r on the coordinate axes are
l | r |, m | r |, n | r | .
Let OP be any line through the origin O, which has direction ) and r = (li + m
cosines l, m and n (Fig. 27.6) 14. r = | r | (l i + m j + nk ) .
j + nk
Let P ≡ (x, y, z) and OP = r. Then
OP2 = x2 + y2 + z2 = r2 (27.1) 27.8 Direction Ratios
From P, draw PA, PB and PC perpendicular on the coordinate axes, If a, b and c are three numbers proportional to the direction
so that cosines l, m and n of a straight line, then a, b and c are called its
OA = x, OB = y and OC = z direction ratios. They are also called the direction numbers or
Also, direction components.
∠POA = a, ∠POB = b and ∠POC = g Hence, by definition we have
From the triangle AOP,
l m n
x = = = k (say) ⇒ l = ak
l = cos a = ⇒ x = lr a b c
r
Similarly, y = mr and z = nr. m = bk
and
Hence, from Eq. (27.1) n = ck
Z ⇒ k2(a2 + b2 + c2) = l2 + m2 + n2 = 1
or
P(x, y, z) cos2 a + cos2 b + cos2 g = 1
C
z or
γ sin2 a + sin2 b + sin2 g = 2
O α
X 1 1
β x A
y ⇒k =± =±
B
a +b +c
2 2 2
∑ a2
Y Therefore,
Figure 27.6 a
l=±
r2(l2 + m2 + n2) = x2 + y2 + z2 = r2 ⇒ l2 + m2 + n2 =1 ∑ a2
Note: Similarly,
1. If the coordinates of any point P be (x, y, z) and l, m and n be the
direction cosines of the line OP, O being the origin, then lr, mr, b
m=±
nr will give us the coordinates of a point on the line OP which is
at a distance r from (0, 0, 0).
∑ a2
2. If OP = r and the coordinates of point P be (x, y, z), then dc’s of c
and n=±
the line OP are x/r, y/r and z/r.
a b c
å a2
3. Direction cosines of r = ai + bj + ck are , , . where the same sign, either positive or negative, is to be chosen
|r | |r | |r | throughout.
Chapter 27 | Three-Dimensional Geometry 1217
Note: or
1. Direction cosines of a line are unique but direction ratios (dr’s) of r = 6i − 9 j + 18k
a line is in no way unique but can be infinite, i.e. a2 + b2 + c 2 ≠ 1 .
So, the components of r along OX, OY and OZ are 6i , − 9 j and
2. Let r = ai + b j + c k be a vector. Then, its dr’s are a, b and c.
18k respectively.
If a vector r has dr’s a, b and c, then
|r |
( ai + b j + ck )
Illustration 27.8 Find the angle between the vectors with direc-
r=
a2 + b2 + c 2 tion ratios 4, −3 and 5 and 3, 4 and 5.
Solution: Let a be a vector parallel to the vector having direction
For example, if r = 2i − 3j + 10k , then its direction ratios are 2, ratios 4, −3 and 5
−3 and 10 or 4, −6 and 20 or any positive multiple of the com-
a = 4i − 3j + 5k
ponents or direction cosines of r .
and b be a vector parallel to the vector having direction ratios 3,
4 and 5
27.8.1 Direction Cosine and Direction Ratio of a
b = 3i + 4 j + 5k
Line joining Two Given Points
Let q be the angle between the given vectors.
See Fig. 27.7. The direction ratios of the line PQ joining P(x1, y1, z1) Then,
and Q(x2, y2, z2) are a.b 12 − 12 + 25 1 p
x2 − x1 = a (say) cosq = = = ⇒q =
| a || b | 16 + 9 + 25 9 + 16 + 25 2 3
y2 − y1 = b (say)
Thus, the angle between the vectors with direction ratios 4, −3, 5
and z2 − z1 = c (say)
and 3, 4, 5 is 60°.
Then direction cosines are
( x 2 - x1)
l= Illustration 27.9 Find the direction ratios and direction cosines
å ( x2 - x1)2 of the line joining the points A(6, −7, −1) and B(2, −3, 1).
Solution: Direction ratios of AB are
( y 2 - y1)
m= (4, − 4, − 2) = (2, − 2, − 1)
and å ( x2 - x1)2 a2 + b2 + c2 = 9
( z2 - z1) ⎛ 2 2 1⎞
n= Direction cosines are ⎜ ± , ∓ , ∓ ⎟ .
⎝ 3 3 3⎠
å ( x2 - x1)2
Illustration 27.10 A line makes the same angle q with each of
the x- and z-axis. If the angle b, which it makes with y-axis, is such
Z Q (x2, y2, z2)
that sin2 b = 3 sin2 q , then cos2 q equals.
Solution: We know that
P (x1, y1, z1)
cos2 a + cos2 b + cos2 g = 1
Since line makes angle q with x- and z-axis and angle b with y-axis.
O X
cos2 q + cos2 b + cos2 q = 1
⇒ −(2 cos2 q − 1) = cos2 b (1)
Y Given that
sin2 b = 3 sin2 q (2)
Figure 27.7
From Eqs. (1) and (2),
Illustration 27.7 A vector r has length 21 and direction ratios 1 = 3 sin2 q − 2 cos2 q + 1
2, −3 and 6. Find the vector r .
⇒ 0 = 3(1− cos2 q ) − 2 cos2 q
Solution: The direction cosines of r are
2 −3 6 ⇒ 5 cos2 q = 3
± ,± ,±
2 + ( −3) + 6
2 2 2
2 + ( −3) + 6
2 2 2
2 + ( −3)2 + 62
2 ⇒ cos2 q = 3 / 5
Since r makes an acute angle with x-axis, therefore cos a > 0, i.e. Illustration 27.11 Find the direction cosines of the line that
l > 0. makes equal angles with the three axes in a space.
2 3 6
So, direction cosines of r are , − , . Solution: Since,
7 7 7
Therefore, l 2 + m2 + n2 = 1
⇒ cos a + cos b + cos g = 1
2 2 2
⎛2 3 6 ⎞
r = 21 ⎜ i − j + k ⎟ [using r = | r | (li + m j + nk ) ] Now,
⎝7 7 7 ⎠ a = b =g
1218 Mathematics Problem Book for JEE
⇒ 3 cos2 a = 1 and
AP = ( x − a )2 + ( y − b )2 + ( z − c )2
⇒ cos a = ±1/ 3
That is, Therefore, the perpendicular distance of point P
l = m = n = ±1/ 3 PN = AP 2 − AN 2
1 1 1
Hence, required dc’s are ± ,± ,± . P (x, y, z)
3 3 3
Your Turn 1
1. If the vertices of a triangle are A(1, −1, 2), B(2, 0, −1) and C(0, 2, 1),
then find the area of the triangle. Ans. 2 6
2. If the points (5, 2, 4), (6, −1, 2) and (8, −7, k) are collinear, then
A (a, b, c) N
find k. Ans. −2
3. A line that makes angle 60° with y-axis and z-axis, then the Figure 27.9
angle, which it makes with x-axis is Illustration 27.12 Find out the perpendicular distance of point
(A) 45° (B) 60° (C) 75° (D) 30° P(0, −1, 3) from a straight line passing through A(1, −3, 2) and
Ans. (A) having direction ratios 1, 2 and 2.
4. A line passes through the points (6, −7, −1) and (2, −3, 1). The
Solution: Direction cosines of the line are
direction cosines of the line are directed so that the angle made 1 2 2
, , 1 2 2
by it with the positive direction of x-axis is acute are 2 , that is,
, ,
2 −2 −1 −2 2 1 1 +2 +2
2 2 2
1 +2 +2
2 2 2
1 +2 +2
2 2
3 3 3
(A) , , (B) , ,
Therefore,
3 3 3 3 3 3
2 −2 1 2 2 1 PN = l(x − a) + m(y − b) + n(z − c)
(C) , , (D) , , Ans. (A) 1 2 2 5
3 3 3 3 3 3 = (0 − 1) + (−1 + 3) + (3 − 2) =
3 3 3 3
⎛ 1 1 1⎞
5. If the direction cosines of a line are ⎜ , , ⎟ , then
⎝ c c c⎠ AP = (0 − 1)2 + ( −1+ 3)2 + (3 − 2)2 = 6
(A) c > 0 (B) c = ± 3 Therefore, the perpendicular distance
(C) 0 < c < 1 (D) c > 2 Ans. (B) 25 29
G PN = AP 2 − PN 2 = 6 − =
6. If r is a vector of magnitude 21 and has dr’s 2, −3 and 6. Then 9 3
G
r is equal to Illustration 27.13 If A, B, C and D are the points (3, 4, 5), (4, 6, 3),
(A) 6i − 9f + 18k (B) 6i + 9f + 18k (−1, 2, 4) and (1, 0, 5), then the projection of CD on AB is
(C) 6i − 9f − 18k (D) 6i + 9f − 18k Ans. (A) 3 −4 3
(A) (B) (C) (D) None of these
4 3 5
27.9 Projection of a Line
Solution: Let l, m and n be the direction cosines of AB. Then
See Fig. 27.8. Projection of the line joining two points P(x1, y1, z1) and
4 −3 1
Q(x2, y2, z2) on another line whose direction cosines are l, m and n is l= =
( 4 − 3)2 + (6 − 4 )2 + (3 − 5)2 3
AB = l(x2 − x1) + m(y2 − y1) + n(z2 − z1)
6−4 2
P (x1, y1, z1) Q (x2, y2, z2) and m= =
3 3
Similarly,
−2
n=
3
Therefore,
A B
⎛ 1⎞ ⎛ 2⎞ ⎛ 2⎞
Figure 27.8 Projection of CD on AB = [1− ( −1)] ⎜ ⎟ + [0 − 2] ⎜ ⎟ + [5 − 4 ] ⎜ − ⎟
⎝ 3⎠ ⎝ 3⎠ ⎝ 3⎠
27.9.1 Perpendicular Distance of a Point 2 4 ⎛ 2⎞ 4
= − +⎜− ⎟ = −
from a Line 3 3 ⎝ 3⎠ 3
Let AB be a straight line passing through the point A(a, b, c) and Hence, the correct answer is option (B).
having direction cosines l, m and n (Fig. 27.9).
AN = projection of the line AP on the straight line Illustration 27.14 The projections of a line on coordinate axes
AN = l(x − a) + m(y − b) + n(z − c) are 2, 3 and 6. Then find the length of the line.
Chapter 27 | Three-Dimensional Geometry 1219
Solution: Let AB be the line and its direction cosines be cos a, cos b Note:
and cos g . Then the projections of the line AB on the coordinate x − x1 y − y1 z − z1
1. The parametric equations of the line = = are
axes are AB cos a, AB cos b and AB cos g. a b c
Therefore, x = x1 + al , y = y1 + bl , z = z1 + c l , where l is the parameter.
AB cos a = 2, x − x1 y − y1 z − z1
2. The coordinates of any point on the line = =
AB cos b = 3 a b c
and AB cos g = 6 are ( x1 + al , y1 + bl , z1 + c l ), where l ∈ R.
3. Since the direction cosines of a line are also direction ratios,
So,
therefore equation of a line passing through (x1, y1, z1) and
AB 2 (cos2 a + cos2 b + cos2 g ) = 22 + 32 + 62 = 49 having direction cosines l, m and n is
⇒ AB2(1) = 49 ⇒ AB = 7 x − x1 y − y1 z − z1
= =
l m n
27.10 Equation of a Straight Line in 4. Since x-, y- and z-axis pass through the origin and have direction
cosines 1, 0, 0; 0, 1 , 0; and 0, 0, 1, respectively. Therefore, the
Space equations are
Every equation of the first degree represents a plane. Two x −0 y −0 z −0
x-axis: = = or y = 0 and z = 0,
equations of the first degree are satisfied by the coordinates of 1 0 0
every point on the line of intersection of the planes represented by x −0 y −0 z −0
y-axis: = = or x = 0 and z = 0
them. Therefore, the two equations together represent that line. 0 1 0
and
Therefore, ax + by + cz + d = 0 and a′x + b′y + c ′z + d ′ = 0 together
x −0 y −0 z −0
represent a straight line. z-axis: = = or x = 0 and y = 0.
0 0 1
5. In the symmetrical form of the equation of a line, the coeffi-
27.10.1 Vector Equation of a Line Passing
cients of x, y and z are unity.
Through a Given Point and Parallel to
a Given Vector 27.10.3 Vector Equation of a Line Passing
Let A be a fixed point having position vector a and the line is
Through Two Given Points
parallel to the vector b . P is an arbitrary point having position
Let A and B be two fixed points having position vectors a and b .
vector r on the line (Fig. 27.10). P is a variable point on the line (Fig. 27.11).
From ΔOAP, From ΔOPA again,
OP = OA + AP ⇒ r = a + lb OP = OA + AP
b P (r )
B(b )
P(r )
A(a) A(a)
O O
Solution: Let A, B and C be the points with position vectors If the direction ratios of two lines a1, b1, c1, and a2, b2, c2 are giv-
2i − j + k , −i + 4 j + k and i + 2 j + 2k , respectively. en, then the angle between the two lines is given by
a1a2 + b1b2 + c1c2
We have to find the equation of a line passing through the point A
cos q =
and parallel to BC . a12 + b12 + c12 ⋅ a22 + b22 + c22
Now,
C
BC = position vector of C − position vector of B
Similarly,
27.11.1 Cartesian Form
a1 b1 c1
See Fig. 27.12. Let q be the angle between two straight lines AB = =
a2 b2 c2
and AC, whose direction cosines are l1, m1 and n1 and l2, m2 and n2,
Note:
respectively, and is given by ⎛ 1⎞
1. The angle between any two diagonals of a cube is cos −1 ⎜ ⎟ .
cos q = l1l2 + m1m2 + n1n2 ⎝ 3⎠
Chapter 27 | Three-Dimensional Geometry 1221
and 2m − a − l m − l + a m − l
= =
( 4i - k ) + m (2i + 3k ) = (2 m + 4 )i + 0 j + (3 m - 1)k 2 1 2
Then,
If the lines intersect, then they have a common point. So, for some l = 3a
values of l and m, we must have and
(3l + 1) i + (1− l )j − k = (2 m + 4 )i + 0 j + (3 m − 1)k m=a
⇒ 3l + 1 = 2m + 4, 1 − l = 0 and −1 = 3m − 1 Therefore, points of intersection are P(3a, 2a, 3a) and Q(a, a, a).
Solving the last two of these three equations, we get l = 1 and Hence, the correct answer is option (B).
m = 0. These values of l and m satisfy the first equation. So, the giv-
en lines intersect. Putting l = 1 in first line, we get 27.13 Shortest Distance Between Two
r = (i + j − k ) + (3i − j ) = 4i + 0 j − k Non-intersecting Lines
which is the position vector of the point of intersection. Thus, the
Two lines are called non-intersecting lines if they do not lie in the
coordinates of the point of intersection are (4, 0, −1).
same plane. The straight line that is perpendicular to each of the
non-intersecting lines is called the line of the shortest distance.
x −1 y +1 z −1
Illustration 27.19 If the lines = = and And the length of the shortest distance line intercepted between
x −3 y −k z 2 3 4 the two lines is called the length of the shortest distance.
= = intersect, then find k.
1 2 1
Solution: We have 27.13.1 Vector Form
x −1 y +1 z −1
= = = r1 If r = a1 + lb1 and r = a2 + lb2 are two skew lines, the shortest
Let 2 3 4 distance between them is the perpendicular distance (Fig. 27.13).
x = 2r1 + 1, y = 3r1 − 1, z = 4 r1 + 1 It is obtained as
( a − a ) ⋅ (b × b )
That is, the point is (2r1 + 1, 3r1 − 1, 4 r1 + 1) . d = PQ = projection of AB on PQ = AB . e = ± 2 1 1 2
and | b1 × b2 |
x −3 y −k z
= = = r2
1 2 1 b1 A(a1)
That is, the point is (r2 + 3, 2r2 + k , r2 ) . P
If the lines are intersecting, then they have a common point. So,
d B(a2)
2r1 + 1 = r2 + 3, 3r1 − 1 = 2r2 + k , 4 r1 + 1 = r2
On solving
Q
r1 = −3 / 2, r2 = −5 b2
Hence,
k = 9/2 Figure 27.13
Illustration 27.20 A line with direction cosines proportional to More appropriately (Fig. 27.14),
2, 1 and 2 meets each of the lines x = y + a = z and x + a = 2 y = 2 z . ( a − a ) ⋅ (b × b ) [b b (a − a )]
d = 2 1 1 2 = 1 2 2 1
The coordinates of each of the points of intersection are given by | b1 × b2 | | b1 × b2 |
(A) (2a, 3a, 3a) (2a, a, a)
(B) (3a, 2a, 3a) (a, a, a) Clearly, two lines intersect if [b1b2 (a2 − a1)] = 0 .
(C) (3a, 2a, 3a) (a, a, 2a)
(D) (3a, 3a, 3a) (a, a, a) If the lines are parallel, r = a1 + lb and r = a2 + lb , the formula to
calculate shortest distance becomes
Solution: Given lines are
| (a2 − a1) × b |
x y+a z d=
= = = l (say) |b |
1 1 1
Therefore point is P(l, l − a, l) and
A
x+a y z
= =
That is, 1 1/ 2 1/ 2 d
x+a y z
= = = m (say)
2 1 1
→
Therefore, the point is Q(2m − a, m, m). Since direction ratios of the b
given lines are 2, 1 and 2, and direction ratios of
PQ = (2m − a − l, m − l + a, m − l) B
According to the question, Figure 27.14
Chapter 27 | Three-Dimensional Geometry 1223
27.13.2 Cartesian Form Illustration 27.22 Find the shortest distance between the lines
Let the equation of two non-intersecting lines be x −3 y −8 z −3 x +3 y +7 z −6
= = and = = .
x − x1 y − y1 z − z1 3 −1 1 −3 2 4
= = = r1 (say) (27.4)
l1 m1 n1 Also find the equation of line of the shortest distance.
and Solution: Given lines are
x − x2 y − y2 z − z2 x −3 y −8 z −3
= = = r2 (say) (27.5) = = = r1 (say) (1)
l2 m2 n2 3 −1 1
and
Any point on line (27.4) is P(x1 + l1r1, y1 + m1r1, z1 + n1r1) and on line
x +3 y +7 z −6
(27.5) is Q(x2 + l2r2, y2 + m2r2, z2 + n2r2). = = = r2 (say) (2)
−3 2 4
Let PQ be the line of the shortest distance. Its direction ratios
will be [(l1r1 + x1− x2− l2r2), (m1r1 + y1− y2− m2r2), (n1r1 + z1− z2− n2r2)]. Any point on line (1) is P(3r1 + 3, 8 − r1, r1 + 3) and on line (2) is
This line is perpendicular to both the given lines. By using Q(−3 − 3r2, 2r2 − 7, 4r2 + 6).
condition of perpendicularity, we obtain two equations in r1 and r2. If PQ is the line of the shortest distance, then direction ratios of
So by solving these, values of r1 and r2 can be found. And sub- PQ = (3r1 + 3) − (−3 − 3r2), (8 − r1) − (2r2 − 7), (r1+ 3) − (4r2 + 6).
sequently points P and Q can be found. The distance PQ is the That is,
shortest distance. 3r1 + 3r2 + 6, −r1 − 2r2 + 15, r1 − 4r2 − 3
The shortest distance can be found by As PQ is perpendicular to lines (1) and (2), therefore
x1 − x 2 y1 − y 2 z1 − z2 3(3r1 + 3r2 + 6) − 1(−r1 − 2r2 + 15) + 1(r1 − 4r2 + 3) = 0
PQ = l1 m1 n1 ⇒ 11r1 + 7r2 = 0 (3)
l2 m2 n2 and
−3(3r1 + 3r2 + 6) + 2(−r1 − 2r2 + 15) + 4(r1 − 4r2 + 3) = 0
Note: If any straight line is given in general form, then it can be ⇒7r1 + 11r2 = 0 (4)
transformed into symmetrical form and we can further proceed. On solving Eqs. (3) and (4), we get r1 = r2= 0.
So, points P(3, 8, 3) and Q(−3, −7, 6). Therefore, length of the
Illustration 27.21 Find the shortest distance between the lines
shortest distance
r = ( 4i − j ) + l (i + 2 j − 3k ) and r = (i − j + 2k ) + m (2i + 4 j − 5k ).
PQ = {( −3 − 3)2 + ( −7 − 8 )2 + (6 − 3)2 } = 3 30
Solution:We know that the shortest distance between the lines
Direction ratios of shortest distance line are 2, 5 and −1.
r = a1 + lb1 and r = a2 + mb2 is given by
Therefore equation of the shortest distance line is
(a2 − a1) ⋅ (b1 × b2 ) x −3 y −8 z −3
d= = =
| b1 × b2 | 2 5 −1
Comparing the given equations with the equations r = a1 + l b1
and r = a2 + m b2 , respectively, we have
27.14 Point and Line
27.14.1 Foot of Perpendicular from a Given
a1 = 4i − j , a2 = i − j + 2k , b1 = i + 2 j − 3k Point to the Given Line
and
27.14.1.1 Cartesian Form
b2 = 2i + 4 j − 5k
See Fig. 27.15. Point A(a, b, g ) and equation line
Now,
x − x1 y − y1 z − z1
a2 − a1 = −3i + 0 j + 2k l
=
m
=
n
=r
and A(α , β , γ )
i j k
b1 × b2 = 1 2 −3 = 2i − j + 0k
2 4 −5
Therefore,
(a2 − a1) ⋅ (b1 × b2 ) = ( −3i + 0 j + 2k ) ⋅ (2i − j + 0k ) = −6 + 0 + 0 = −6
and | b1 × b2 | = 4 + 1+ 0 = 5
P
Therefore,
x − x1 y − y1 z − z1
( a − a ) ⋅ (b × b ) −6 6 = = =r
d = 2 1 2 1 = = l m n
| b2 × b1 | 5 5
Figure 27.15
1224 Mathematics Problem Book for JEE
If P be the foot of perpendicular, then P is (lr + x1, mr + y1, nr + z1). 11i − 2 j − 8k + l(10i − 4 j − 11k )
Find the direction ratios of AP and apply the condition of
perpendicularity of AP and the given line. This will give the value = (11+ 10 l )i + ( −2 − 4 l )j + ( −8 − 11l )k
of r and hence the point P that is the foot of perpendicular. Then,
27.14.1.2 Vector Form PL = position vector of L − position vector of P
See Fig. 27.16. Let L be the = [(11+ 10 l )i + ( −2 − 4 l )j + ( −8 − 11l )k ] − [2i − j + 5k ]
foot ofG a perpendicular drawn from
P(a ) on the line r = a + l b . Since r denotes the position vector
= (9 + 10 l )i + ( −1− 4 l )j + ( −13 − 11l )k
on the line r = a + l b . So, let the position vector of L
of any point
be a + l b . Since PL is perpendicular to the given line and the given line is
Then, parallel to b = 10i − 4 j − 11k .
æ (a - a )b ö Therefore,
PL = a - a + l b = (a - a ) - çç 2 ÷÷ b
è |b | ø PL ⊥ b
^ ^ ^
P(2i − j + 5k )
P(α )
r = a + λb L
^ ^ ^ ^ ^ ^
A B (11i − 2j − 8k ) − λ (10i − 6j − 11k )
L = a + λb
Figure 27.18
Figure 27.16
The length PL is the magnitude of PL , and the required length of ⇒ PL ⋅ b = 0
⇒ [(9 + 10 l )i + ( −1− 4 l )j + ( −13 − 11l )k ] ⋅ (10i − 4 j − 11k ) = 0
a perpendicular.
⇒ 10(9 + 10l) − 4(−1 − 4l) − 11(−13 − 11l) = 0
27.14.2 Reflection or Image of a Point in a
⇒ 90 + 100l + 4 + 16l + 143 + 121l = 0
Straight Line
⇒ 237l = −237 ⇒ l = −1
See Fig. 27.17. If the perpendicular PL from point P on the given Putting the value of l, we obtain the position vector of L as i + 2 j + 3k
line be produced to Q such that PL = QL, then Q is known as the Now,
image or reflection of P in the given line. Also, L is the foot of the
perpendicular
or the projection of P onthe line. PL = (i + 2 j + 3k ) − (2i − j + 5k ) = −i + 3j − 2k
Let Q( b ) is the image of P in r = a + l b
Hence, the length of the perpendicular from P on the given line is
P
| PL | = 1+ 9 + 4 = 14
A B Illustration 27.24 Find the image of the point (1, 6, 3) in the line
L
x y −1 z − 2
= = .
1 2 3
Q (image)
Figure 27.17 Solution: Let P(1, 6, 3) be the given point, and let L be the foot
Then of the perpendicular from P to the given line (Fig. 27.19). The
coordinates of a general point on the given line are given by
⎛ 2(a − a ).b ⎞
b = 2a − ⎜ ⎟ b .a
⎝ | b |2 ⎠ x − 0 y −1 z − 2
= = =l
1 2 3
Illustration 27.23 Find the foot of the perpendicular drawn from That is,
the point 2i − j + 5k to the line r = (11i − 2 j − 8k ) + l (10i − 4 j − 11k ). x = l,
Also find the length of the perpendicular. y = 2l + 1
and
Solution: See Fig. 27.18. Let L be the foot of the perpendicular drawn
z = 3l + 2
from P (2i − j + 5k ) on the line r = 11i − 2 j − 8k + l(10i − 4 j − 11k ).
Let the position vector of L be Let the coordinates of L be (l, 2l + 1, 3l + 2).
Chapter 27 | Three-Dimensional Geometry 1225
3. Equation of plane parallel to XOY-plane (or perpendicular to Solution: We know that the vector equation of a plane passing
z-axis) at a distance ‘c’ from it is z = c.
through a point having position vector a and normal to n is
(r − a ) ⋅ n = 0
27.16.6 Equation of Plane Perpendicular to
Coordinate Plane or Parallel to or r ⋅n= a⋅n (1)
Coordinates Axis
Since the given plane passes through the point (3, −3, 1) and is
1. Equation of plane perpendicular to YOZ-plane or parallel to normal to the line joining the points A(3, 4, −1) and B(2, −1, 5).
x-axis is By + Cz + D = 0.
Therefore,
2. Equation of plane perpendicular to ZOX-plane or parallel to
y-axis is Ax + Cz + D = 0. a = 3i − 3j + k
3. Equation of plane perpendicular to XOY-plane or parallel to and
z-axis is Ax + By + D = 0.
n = AB = P.V. of B − P.V. of A
27.16.7 Equation of Plane Passing Through a = (2i − j + 5k ) − (3i + 4 j − k ) = −i − 5j + 6k
Point and Having Given Direction Ratio
Substituting a = 3i − 3j + k and n = −i − 5j + 6k in Eq. (1), we
The equation to the plane passing through P(x1, y1, z1) and having
obtain
direction ratios (a, b, c) for its normal is
a(x − x1) + b(y − y1) + c(z − z1) = 0
r ⋅( −i − 5j + 6k ) = (3i − 3j + k ) ⋅ ( −i − 5j + 6k )
or
27.16.8 Equation of Plane Passing Through
r ⋅( −i − 5j + 6k ) = −3 + 15 + 6 or r ⋅( −i − 5j + 6k ) = 18
Three Non-Collinear Points
The equation of the plane passing through three non-collinear This is the vector equation of the required plane. The Cartesian
points (x1, y1, z1), (x2, y2, z2) and (x3, y3 , z3) is equation is
( xi + y j + zk ). ( −i − 5j + 6k ) = 18
( x − x1) ( y − y1) ( z − z1)
⇒ −x − 5y + 6z = 18
( x 2 − x1) ( y 2 − y1) ( z2 − z1) = 0 or
( x 3 − x1) ( y 3 − y1) ( z3 − z1) x + 5y − 6z + 18 = 0
Illustration 27.25 Reduce the equation r ⋅(3i − 4 j + 12k ) = 5 to
normal form and hence find the length of perpendicular from the Illustration 27.27 Write the equation of the plane whose inter-
origin to the plane. cepts on the coordinate axes are −4, 2 and 3.
Solution: The given equation is Solution: We know that the equation of a plane whose intercepts
on the coordinate axes are a, b and c, respectively, is
r .(3i − 4 j + 12k ) = 5 or r ⋅ n = 5
x y z
where n = 3i − 4 j + 12k . + + =1
a b c
Since
Here, a = −4, b = 2 and c = 3.
| n | = 32 + ( −4 )2 + 122 = 13 ≠ 1 So, the equation of the required plane is
Therefore, the given equation is not in the normal form. To reduce x y z
+ + =1
it to normal form, we divide both sides by | n | , that is, −4 2 3
n 5 or
r. = −3x + 6y + 4z = 12
|n| |n|
⎛ 3 4 12 ⎞ 5
or r . ⎜ i − j + k ⎟ = 27.17 Point and Plane
⎝ 13 13 13 ⎠ 13
This is the normal form of the equation of the given plane. The 27.17.1 Position of Two Points w.r.t the Plane
5
length of the perpendicular from the origin is . Two points P(x1, y1, z1) and Q(x2, y2, z2) lie on the same or opposite
13 sides of a plane ax + by + cz + d = 0 according to ax1 + by1 + cz1 + d
Illustration 27.26 Find the equation in the Cartesian form of and ax2 + by2 + cz2 + d are of same or opposite signs. The plane
the plane passing through the point (3, −3, 1) and normal to the divides the line joining the points P and Q externally or internally
line joining the points (3, 4, −1) and (2, −1, 5). according to P and Q lying on same or opposite sides of the plane.
Chapter 27 | Three-Dimensional Geometry 1227